You are on page 1of 573

Rahul Sardana with www.puucho.

com

www.puucho.com
Rahul Sardana with www.puucho.com

ADVANCED
JEE PHYSICS
for both Main & Advanced levels of JEE
OPTICS & MODERN PHYSICS

www.puucho.com
Rahul Sardana with www.puucho.com

ADVANCED
JEE PHYSICS
for both Main & Advanced levels of JEE
OPTICS & MODERN PHYSICS

RAHUL SARDANA
M.Sc (Hons). Physics

--7

. Ef·,1-- --=1~11;
__I l

ELSEVIER

A division ofRe¢ Elsevier India Pvt. Ltd.

www.puucho.com
/
v-· Rahul Sardana with www.puucho.com

ADVANCED JEE PHYSICS, Optics & Modem Physics


Rahul Sardana

ELSEVIER
A Division of
Reed Elsevier India Private Limited.

Copyright © 2014 Effective Testbooks India (ETI)

This book contains information obtained from authentic and highly regarded sources. Reprinted material is quoted
with permission, and sources are indicated. Reasonable efforts have been made to publish reliable data and
infonnation, but the author and the publisher cannot assume responSibility for the validity of all materials or for the
consequences of their use.

Neither this book nor any part may be reproduced or transmitted in any form or by any means, electronic or
mechanical, including photocopying, microfilming and recording, or by any information storage or retrieval system,
without prior remission in writing ~om the publishers.

The consent of ETI does not extend to copying for general distribution, for promotion, for creating new works, or
for resale. Specific permission must be obtained in writing from ET/ for such copying.

The export rights of this book are vested solely with the publisher.

ISBN: 978-93-81269-97-8

Published by Elsevier, a division of Reed Elsevier India Private Limited, under special arrangement with ETI.

Registered Office: 305, Rohit House, 3 Tolstoy Marg, New Delhi-110 001.
Corporate Office: 14th Floor, Building No. JOB, DLF Cyber City, Phase-II, Gurgaon-122002, Haryana, India ..

Printed and bound at Rajkamal Electric Press, Kundli, Haryana.

www.puucho.com
Rahul Sardana with www.puucho.com

text overview, goals & focus


In the past few years, the UT-JEE has evolved itself as an examination designed to check out true scientific
skills. The examination pattern wants us to see those little details which, others fail to see. Those details which tell
us how much in depth we should know to describe as much as possible. Keeping the present day scenario in mind,
this book is written for students, to allow them not only to learn the tools that "Optics & Modem Physics"
provides but also to see why they work so nicely in explaining the beauty of ideas behind the subject. The central
goal of this text is to help the students develop a thorough understanding of the principles of "Optics & Modem
Physics". This book stresses on building a rock solid technical knowledge based on firm foundation of the
fundamental principles than on a large collection of formulae. The primary philosophy of this book is to act as a
guide who creates a careful, detailed groundwork for strong conceptual understanding and development of
problem solving skills like mature and experienced physicists.

features of book

theory with illustrations


"Optics & Modem Physics" are important topics, and in this book I have tried to make these topics lively,
clear and precise to the greatest levels. I have generally seen students not stressing on the theoretical details. They
always feel that doing more numerical problems will solve their purpose. But let me tell you here, that numerical
problems are just the special cases of the theoretical concepts. The entire Physics is based on a simple program
"IF~ THEN~ ELSE". Try to follow this and see how you get to your ultimate goal i.e., IIT~JEE. So, keeping this in
mind, the entire theory part of all the chapters has been kept elaborative, simple to understand with supportive
Illustrations at all the places. DO NOT TRY TO ATTEMPT ILLUSTRATIONS WITHOUT GOING THROUGH THE
THEORY.

conceptual notes, remarks, words of advice, misconception removal


Throughout the text, the Conceptual Notes and Remarks are highlighted which focus on the principal ideas
and concepts that a student must take care of. Places where students commonly develop a misconception have
been supported by Misconception Removal, highlighted in grey and supported by Words of Advice. Throughout
my teaching career of 18 years I have always found my students getting benefitted from these Conceptual Notes,
Remarks, Words of Advice, Misconception Removals. All these are actually used to provide warnings to the.
students about common errors and ways to avoid them.

problem solving techniques


These techniques, highlighted in grey, always ensure that the students become capable of solving a variety
of problems in an easy way. Wherever necessary, the text is supplemented with them for having a thorough
understanding to the application processes.

in chapter exercises (ice) : topic wise


After you study the theory and apply it to the Illustrations, its time you practice something on your own and
that too topic wise. For this purpose I have created In Chapter Exercises (ICE) (except for Mathematical Physics).
Each ICE has the name of the topic(s) clearly mentioned on it. Please note that ICE are based on simple, single .
concept classification technique. They are fully solved, so that if you come across only problem, then you just refer
to the solutions.

www.puucho.com
Rahul Sardana with www.puucho.com

C::::: vi

solved problems
After you have gone through the entire Theory (with Illustrations) and all the supplements (ICE, Conceptual
Notes, Remarks, Words of Advice, Misconception Removal, Problem Solving Techniques), its high time to do
problems that are a true mix of concepts studied. This section has problems that involve multiple concept usage so
that your brain is exposed to the ultimate throttle required to extract the best from you at !IT-JEE.

practice exercise sets {fully solved)


Now comes the time when you are very much ready to do the practice as per the !IT-JEE pattern. This
section contains all the variety of questions that have been asked in the !IT-JEE. In this section you will come across
the following variety of questions.

single correct choice type (SCCT)


Each question, in this section, has four choices (A), (B), (C) and (D), out of which ONLY ONE is correct.

multi11le correct choice t}'pe (MCCT)


Each question, in this section, has four choices (A), (BJ, (C) and (D), out of which ONE OR MORE is/ are
correct.

reasoning based questions/ assertion-reason type (ART)


This section contains Reasoning type questions, also called Assertion-Reason type question, each having four
choices (A), (B), (C) and (D) out of which ONLY ONE is correct. E;ach question contains STATEMENT 1-(Assertion)
and STATEMENT 2 (Reason). You have to mark your answer as
Bubble (A) If both statements are TRUE and STATEMENT 2 is the correct explanation of STATEMENT 1.
Bubble (B) If both statements are TRUE but STATEMENT 2 is not the correct explanation of STATEMENT 1.
Bubble (C) If STATEMENT 1 ls TRUE andSTATEMENT2 is FALSE.
Bubble (DJ If STATEMENT 1 is FALSE butSTATEMENT2is TRUE.

linked comprehension tyl)e (LCT) / paragraph type


This section contains Linked Comprehension Type Questions or Paragraph based Questions. Each set
consists of a Paragraph followed by questions. Each question has four choices (A), (BJ, (C) and (D), out of which
only one is correct. (F_or the sake of competitiveness there may be a few questions that may have more than one
correct options).

matrix match type (MMT) / column matching


Each question in this section contains statements given in two columns, which have to be matched. The
statements in COLUMN-I are labelled A, B, C and D, while the statements in COLUMN-II are labelled p, q, r,
s (and t). Any given statement in COLUMN-I can have correct matching with ONE OR MORE statement(s) in
COLUMN-II. The appropriate bubbles corresponding to the answers to these questions have to be darkened as
illustrated in the following examples:
If the correct matches are A -> p, s and t; B -> q and r; C -> p and q; and D -> s and t; then the correct
darkening of bubbles will look like the following :
P q r s t
A@@©©CD
B@@©©CD
c@@©©CD
D@@©©CD

www.puucho.com
Rahul Sardana with www.puucho.com

vii t=:J

integer answer type questions (IATQ) / numerical type questions


In this section the answer to each of the question is a four digit integer, ranging from O to 9999. The
appropriate bubbles below the respective question numbers in the ORS have to be darkened. For example, if the
correct answer to question number X (say) is 6092, then the correct darkening of bubbles will look like the
following:
X.@e@@
CD CD G) CD
®@®•
®@®®
@@©@
®@®®
~~~~
®®®®
@@e@

answers & solutions


Each· chapter contains answers followed by solutions to the problems. The solutions· are exhaustive with
complete methods and reasons which will help you a lot to understand a particular concept. Short cuts are also
included (wherever necessary) for-enhancing you problem solving skills.
This book, I hope, will nourish you with the concepts involved such that you get a great rank at !IT-JEE. To
conclude, I apologise in advance for the errors (if any) that may have inadvertently crept in the text. I would be
grateful to the readers who bring errors of any kind to my attention. I truly welcome all comments, critiques and
suggestions at sardanaeducation@gmail.com. PRAYING TO GOD FOR YOUR SUCCESS AT !IT-JEE, GOD BLESS
YOU!

The Author
RAHUL SARDANA
M.Sc.(Hons.) Physics

www.puucho.com
Rahul Sardana with www.puucho.com

CHAPTER 1 = Matrix Match Type Questions (Column Matching Type), _ _ _..,,,40

RayOptics :i Integer Answer Type Questions .43

(a) Reflection at Plane & Curved Surfaces _ _ _ _ _ _ _ _ _ _ 1.1 Answers to In Chapter Exercises (ICE) & Practice Exercise Set .45
Solutions to In Chapter Exercises (ICE) .48
(b) Refraction at Plane Surfaces 1.27
Solutions to Practice Exercise Sets .52
(c) Refraction at Curved Surface 1.55
Solved Practice Problem 1.96
Practice Exercise Sets CHAPTER4
:> Single Correct Choice Type Questions _ _ _ _ _ _ _ _ J, 108 Atomic & Nuclear Physics
:> Multiple Correct Choice Type Questions 1. 135 AtomicPhysics ..... _ _ _ _ _ _ _ _ _ _ _ _ _ _ _ _ _ _ _ 4_7

:> Reasoning Based Questions (Assertion Reason Type) 1. 141 Nuclear Physics.... 4.21
:> Linked Comprehension Type Questions (Paragraph Type) .......... 1. 144 Solved Practice Problem 4.48
:> Matrix Match Type Questions (Column Matching Type) .............. 1.154 Practice Exercise Sets
:> Integer AnswerTypeQuestiohs ___________ 1,160 :i Single Correct Choice Type Question~--------4.58
Answers to In Chapter Exercises (ICE) & Practice Exercise Sets ____ 1,163 :i Multiple Correct Choice Type Question 4J3
Solutions to In Chapter Exercises (ICE) _ _ _ _ _ _ _ _ _ _ _ 1, 169 :i Reasoning Based Questions (Ass'ertion Reason Type) _ _ _ _ 4.78
Solutions to Practice Exercise Set 1. 193 :, Linked Comprehension Type Questions (Paragraph Type) ............. 4.81
:, Matrix Match Type Questions (Column Matching Type) ................ .4.89

CHAPTER2 :, Integer Answer Type Questions _ _ _ _ _ _ _ _ _ _ _4.92

Answers to In Chapter Exercises (ICE) & Practice Exercise Set,_ ____4.94


Wave Optics
Solutions to In Chapter Exercises (ICE) _ _ _ _ _ _ _ _ _ _ _4.99
WaveOptiCS-------------------~-1
Solutions to Practice Exercise Set 4.106
Solved Practice Problems.... .41

Practice Exercise Sets


:i Single Correct Choice Type Question,,_ _ _ _ _ _ _ _ _..,_.52

= Multiple Correct Choice Type Question .66


:i Reasoning Based Questions (Assertion Reason Type) .69
:i Linked Comprehension Type Questions (Paragraph Type) ............. 2.71
:i Matrix Match Type Questions (Column Matching Type) ................. 2.77
:i Integer Answer Type Questions .80
Answers to In Chapter Exercises (ICE) & Practice Exercise Set.~---~,.83
Solutions to In Chapter Exercises (ICE) .86
Solutions to Practice Exercise Sets .92

CHAPTER3
Dual Nature of Radiation & Matter
Dual Nature ofRadiqtion &Matter _ _ _ _ _ _ _ _ _ _ _ _ _..,,. I

Solved Practice Problem .1 S


Practice Exercise Sets
:i Single Cofrect Choice Type Question•s.._ _ _ _ _ _ _ _ __,_2 7

o Multiple Correct Choice Type Question .33


:, Reasoning Based Questions (Assertion Reason Type) .36
:, Linked Comprehension Type Questions (Paragraph Type) .............3.38-·

l www.puucho.com
Rahul Sardana with www.puucho.com

\
I

Ray Optics

- --··- - -- - -- - ---- - - ------ - - --- '


Contents

(a) REFLECTION AT PLANE & CURVED SURFACES ..................................................... 1.1


(b) REFRACTION AT PLANE SURFACES ......................................................................1.27
(c) REFRACTION AT CURVED SURFACES ...................................................................1.55
Solved Practice Problems ..........••....•...•.•....•...•••••.••••...••....••...••...•••..•••....••...•....•.•..••....•••..1.96
Practice ·Exercise Sets
:> Single Correct Choice Type.Questions .•.....••..............•........................................1.108
:, Multiple Correct Choice Type Ques1ions .............................................................1.135
:> Reasoning Based Questi?ns (Assertion Reason Type)-...................................... 1.141
:, Linked Comprehension Type Questions (Paragraph Type) ................................. 1.144
::> Matrix Match Type Questions (Column Matching Type) .........................••....•...... 1.154
::> Integer Answer Type Questions ..................................................................•••.....1.160
Answers to In Chapter Exercises (ICE) & Practice Exe!cise Sets .................._................. 1.163
Solutions to In Chapter Exercises (lCE) ...........................................................................1.169
Solutions to Practice Exercise Sets ................................. ;...............................................1.193

www.puucho.com
Rahul Sardana with www.puucho.com

NATURE OF LIGHT: An Introduction E


-=c
ight is a form of energy that makes object visible to

L
B
our eyes or light is the form of energy that produces in In 1905, Albert Einstein revived the. old corpuscular theory
us the sensation of sight. In Seventeenth century using Plank's Quantum· Hypothesis and through his
Newton and Descartes believed that light consisted of a photoelectric effect experiment showed that light consists of
stream of particles, called corpuscles. Huygens proposed discrete energy packets, called photons. The energy of each
wave theory of light and proposed that light is a disturbance photon is
in a medium called Ether. This theory could explain the
phenomena of interference, diffraction, etc. Thomas Young, E=hf=hc
A,
through . his double slit experiment, measured the
wavelength of light. So, in view of these developments, light must be regarded to
Maxwell suggested the electromagnetic theory of light. have a dual nahlre i.e., it exhibits the characteristics of a ·
According to this theory, light consists of electric and particle in some situations and that of a wave in other
magnetic fields,.in mutually perpendicular directions, and sihlations. So the question'' Is light a particle or a wave?" is
both are perpendicular to the direction of propagation. purely inappropriate to be -asked. At present, it is believed
Heinrich Hertz produced in the laboratory the that light has dual nature, i.e., it has both the characters,
electromagnetic waves of short wavelengths. He showed wave-like and particle-like.
that these electromagnetic waves possessed all the properties
of light waves. OPTICS : An Introduction
E Optics is the study of the properties of light, its propagation
through different media and its effects. In most of the
sihlations, the light encounters objects of size much larger
than its wavelength. We can assume that light travels in
straight lines called rays, disregarding its wave nature. This
allows us to formulate the rules of optics in the language of
Direction of geometry, as rays of light do not disturb each other on
propagation intersection. Such study is called geometrical (or ray) optics.
It includes the working of mirrors, lenses, prisins, etc.
ii When light passes through very narrow slits, or when it
passes around very small objects, we have to consider the
Light travels in vacuum with a velocity given by
wave nature of llght. This study is called wave (or physical)
1 . . 8' -1
optics.
c= ~=3x10 ms
Vµoto ·
where µo and Eo are the permeability and permittivity of free DOMAINS OF OPTICS
space (vacuum). The study of light can be categorized into three broad
The magnitudes of electric and magnetic fields are related to domains.
the velocity of light by the relation · ·· a) Geometrical Optics (Ray Optics)
b) Physical Optics (Wave Optics)

1.1 ,._....,;

www.puucho.com
Rahul Sardana with www.puucho.com .,

Advanced JEE Physics Optics & Modem Physics


c) Quantum Optics 5. The Laws of Refraction (discussed later). The Laws of
Please note that these domains are not strictly disjoint as the Refraction govern the bending of light when the light
transitions between them are continuous and not sharp. goes from one medium to the other (rarer to denser or
However for convenience we consider them as distinct. denser to rarer) medium.
These domains are distinguished as follows.
BASIC TERMS & DEFINITIONS
A. GEOMETRICAL OPTICS (RAY OPTICS)
This branch involves the study of propagation of light based SOURCE
on the assumption that light travels in fixed straight line as it A body which emits light is called source. ·Tue source can be
passes through a uniform medium and its direction is ·a point one or an extended one. A source is of two types.
changed when met by a surface. of a different medium or if
a) Self luminous : The source which possess light of its
the optical properties of the medium are non uniform either
own.
in time or in space. The ray approximation is valid for the
EXAMPLE : sun, electric arc, candle etc.
wavelength A. very small compared to the size of the obstacle
b) Non-luminous : It is a source of light which does not
(d) or the size of the opening through which the ray passes.
possess light of its own but acts as source of light by
This approximation 1,. « d proves to be very good for the reflecting the light received by it.
study of mirrors, lenses, prisms and associated optical EXAMPLE : moon, objects around us, book etc.
instruments such as microscope, telescope, cameras etc.
REMARK(S)
B. PHYSICAL OPTICS (WAVE OPTICS) Sources are also classified as isotropic and non-isotropic.
This branch involves the study of propagation of light in the Isotropic sources give out light uniformly in all directions
form of a wave and it deals with the phenomenon of whereas non-isotropic sources do not give out light uniformly in
1-~ll_dir~~!i?n~. _
interference, diffraction, polarization etc. This nature of light
has to be taken when the light passes through very narrow
slits or when it goes past very small objects. So this branch RAY
works effectively when 1,. » d . The straight line path aiong which the light travels between
two points in-a homogeneous medium or in a pair of media
C. QUANTUM OPTICS is called a Ray: It is represented by an arrow head on a
straight line, the arrow head represents the direction of
This branch involves the study of propagation of light as a
propagation of light. A ray of light will always follow a path
stream of particles called as Photons. This concept of light
along which the time taken is the minimum.
behaving as particles called photons is of utmost importance
while studying the origin of spectra, photoelectric effect,
concept of radiation pressure, Compton effect etc.

-- - - - . - . ., '--
Ray

FUNDAMENTAL LAWS OF GEOMETRICAL OPTICS


To a first approximation, we can consider the propagation of
light disregarding its wave nature and assuming that light
I REMARK(S)
_A ~ingle_!~~-~ann_?~ be_~~ol~.t~~ !r<:_rTI _a_~oun:E:. of light.

propagates in straight lines called rays. This allows us to MEDIUM


formulate the laws of optics in the language of geometry.
Thus, the bfanch of optics where the wave nature of light is Substance through which light propagates or tends to
neglected is called geometrical (or ray) optics. propagate is called a medium. It is of following three types.
Geometrical optics is based ~m five fundamental laws. a) Transparent: It is a medium through which light can be
propagated easily.
1. Law of Rectilinear Propagation of Light. It states that
EXAMPLE : glass, water etc.
light propagates in straight lines in homogenous media.
2. Law of Independence of Light Rays. It states that rays b) Translucent : It is a medium through which light is
do not disturb each other upon intersection. propagated partially.
EXAMPLE : oil paper, ground glass etc.
3. The Law of Reversibility of Light. According to this
law, if a ray of light, after suffering a number of c) Opaque : It is a medium through which light cannot be
reflections and refractions, has its path reversed at any propagated.
instant, then the ray retraces its path back to the source. EXAMPLE : wood, iron etc.

4. The Laws of Reflection. The Laws of Reflection govern


the bouncing back of the incident ray after striking a BEAM
surface to the medium from which it was coming. A bundle or bunch of rays is called a beam. It is of following
three types.

= 1.2

www.puucho.com
Rahul Sardana with www.puucho.com

Ray Optics : Reflection at Plane & Curved Surfaces

a) Parallel beam : It is a beam in which all the rays .-----· - ----- . . -... - .
CONCEPTUAL NOTE(S)
·1
constituting the beam move parallel to each other and Virtual object cannot be seen by human eye, because for an
diameter of beam remains same. A very narrow beam is o. bject or an- image io be seen by ihe eyes, the rays received by
called a Pencil of Light. t~~~x~s_m~st_~~di~e~ging. ___ -·--------- -- - "

b)
0:
Convergent beam : In this case diameter of beam
IMAGES
An optical image is a point where reflected or refracted rays
of light either intersect or appear to intersect. Thus, the
decreases in the direction of ray. image of an infinite object is actually an assembly of the
image points corresponding to various parts or the points of
~e object. The images formed can again be real or virtual.

A. REAL IMAGES
If the rays after reflection or refraction actually converge (or
meet) at a point then the image is said to be real and it can be
obtained on a screen.
c) Divergent beam: It is a beam.in which all the rays meet
at a point when produced backward and the diameter
of beam goes on increasing as the rays proceed forward.

Real Image

B. VIRTUAL IMAGES
However, if the rays do not actually converge but appear to
OBJECTS diverge from a point (or appear to meet at a point), then the
The object for a mirror can be real or virtual. image so formed is said to be virtual image. A virtual image
cannot be obtained on a screen.
A. REAL OBJECTS
If the rays from a point on an object actually diverge from it
and fall on the mirror, the object is said to be real. In simple
language the incident rays are diverging and the point of
divergence is the position of the real object. The following
diagrams support the arguments given.
Virtual Image

CONCEPTUAL NOTE(S)
0 0
a) The real images can be obtained on a suitably placed
screen, but virtual images cannot be obtained on-a screen.
b) Human eye cannot distinguish between the real image and
the virtual image because in both the cases the rays are
d!ve!gi~Q-_. _~ _ __________ _
B. VIRTUAL OBJECTS
If the rays incident on the mirror appear to converge to a REFLECTION OF LIGHT
point, then this point is said to be virtual point object for the When light strikes the. surface on an object, some part of the
mirror. In simple language the incident rays are converging light or the complete light is sent back into the same
and the point of convergence is the position of the virtual medium. This phenomenon is called as reflection. The
object. The following diagrams support the arguments given: surface, which reflects light, is called mirror. A mirror could

1
I'·......'::-.o
rl·--~::::»o
/
be plane or curved.

CONCEPTUAL NOTE(S)
In reflection, the frequency, speed and wavelength remain

l unchanged, but a phase change may occur depending on the


__nature_. of reflec~ing.surface.
1.3 =
www.puucho.com
Rahul Sardana with www.puucho.com

--- - - - - - -----
Advanced JEE Physics

The reflection from a denser medium causes an addition phase


-- Optics & Modern Physics

To understand this theorer;n, let ·us· consider two points A

I 2
1.
change of n or a path change of - (by Stoke's Law) while
reflection from rarer.medium does not c~Use.an; phase c,hange:_..

Diffused (irregular) reflection takes place from a rough


-- and B in the same medium. Since, we know that between
these two points light travels in a straight line, so the time
taken by the light to go from A to B must logically be the.
minimum.

surface where as Specular (regular) reflection takes place A LIGHT PATH B


from an extraordinarily smooth surface. However, the Law's
of Reflection are applicable for both kinds of surfaces, LAWS OF REFLECTION USING FERMAT'S THEOREM
Consider a plane mirror on which light is incident as shown.
LAWS OF REFLECTION
A
a) The incident-ray, the reflected-ray and the normal to the B
reflecting surface at the point of incidellce, all lie in the
same plane.
i
a
'i
'
''
r,' i
b
b) The angle of reflection is equal to the angle of incidence
'''
(i = r) . ' -0-"
The angle of incidence i is the angle made by the 1-4- X -----+t+d - X+I
incident ray with the normal. ----d----.a
The angle of reflection r is the angle made by the
reflected ray with the normal. - Let the incident light start from A , hit the mirror at O and
get reflected to point B. Let the points A and B be at

Y~. 0
Plane Surface
.o
Concave Surtace Convex Surface
perpendicular distances a and b from the mirror and let A
and B have a separation d between them as shown in
figure. The time taken by the light to go from A to O to B
is given by
f = f A-+O f- fO-+B
AO OB
I SPECIAL CASES i ~ t=-+-
i a} If i = P·, then r =O . It means a ray incident no.rmally on a J
c C

i b~oundary,
afte~ reflecti:~~~:~~ces its p ~ , - - - - , , .. i t =!:.( .Ja
C
2
+x' +~b' +(d-x)2 )

i I / L) -{' \ I
Now,.according to Fermat's,Principle, t is MINIMUM, so

-~~~~~~,,) !
~=0
I j ('---~---" dx

_!._(.Ja 2 +x 2 ) +_!,_Ub 2 + (d-x)') = 0


Plane Mirror Concave Mirror Convex Mirror .dx dx _
(b) (c)
I b) (a) ,, 1( 2x ) 1( 2(d-x)(-1)) 0
I The angle made by the incident ray with the plane reflecting i ' 2 ..Ja'+x' +2 ~b'+(d-x)' =
! Surface-Js called- glancing· angle. ;Thus, the, glancing angle l
! =90°-i. I X (d-x)
'
, c) For grazing. i_ncidence, the incident ray grazes the refle~ting j .Ja +x
2 2
~b'+(d-x)'
'i
I surface, so i-), i and hence r ---t g. as shown in ·the-figure .. I From the figure, we observe that
i
.I
! ''
' I
x
~--=s1nz
..Ja'+x2
· · an"d (d-~
)b'+(d-x) 2 ,
sm· r

~ - I ::::> sini = sinr


··::::> i =r. {The Law'of Reflection}
FERMAT'S PRINCIPLE OF LEAST TIME
According to this theorem, light will always follow the path I PROBLEM SOLVING TRICK(S)
taken between any two points by a ray of light is the path I a) Basic Problems in Optics : Most of the problems asked in
along which the time taken is the minimum. ·This principle is I optics. expect us fo find tl)e position and na!ure of the final
sometimes taken as the definition of a ray of light. I imag·e formed by-certain optical systems for a given abject.
L_ The ..aptical_system_may b~ jul;;t a mirror, m~a lernr or a__

= 1.4

www.puucho.com
Rahul Sardana with www.puucho.com

Ray Optics: Reflection at Plane & Curved'Surfaces


combinatiOn of several reflecting and ~r9fractinQSurface5.
b) Basic Strategy for Solving the Problems : To handle
these kinds of problems, first of all, we identify the
sequence in which the reflection and refraction are taking
place. The several events of reflection or refraction can be
named as Event 1, Event 2 and so on following the
· sequence in which they occur.
Now; the ihlage of Event t would be object for Event 2,
image of Event 2 will be object of Event 3 and so on. This
way one can proceed to find the flnal Image. I
ANGLE OF DEVIATION (6) - -- - - - - - - - - - - - - - - -
CONCEPTUAL NDTE(S)
· Deviation (6) is defined as - the angle between the initial The same ls fol!nd to'hold·good for three plane mirrors arranged
direction of the incident ray and the final direction of the mutually perpetldicular to each other thus fohning the comer of
reflected ray or the. emergent ray. a cube such that the light incident on this arrangement suffers
one reflection from each of the mirrors so as to emerge out anti-
parallel to the incident light. This arrangement of three mutually
perpen_dicular plane mirrors fonning the comer of a cube is
called the CORNER REFLECTOR. .

Illustration 1
Two plane mirrors are inclined to each other at an
Deviation produced in Reflection is 6 = 180° -(i + r) angle 0 . A ray of light is reflected first at one mirror and
Since r=i then at the other. Find the total deviation suffered by the
=> 6=180°-2i ray.
The variation of deviation (6) with the angle of incidence Solution
(i) js shown in figure. a be the angle of in~idence for mirror M 1
p be the angle of incidence for mirror M,
8_= Jt 61 be the deviation due to mirror M 1 and
62 be thE! deviation due to mirror M 2
D

: PROBLEM SOLVING TRICK($) • I


I ,a):he deviation is maximu~ for normal incidence i.e., when I
I· 1=0 then, o=o= =180 . .
b) The deviation is minimum for grazing incidelice i.e., when I
i-+i,then 6=6rr,;0 =0°. 1

[ c) While dealing with the case of multiple reflections suffered !


by a ray, the net deviation suffered by the incidentray is the : Fi-om figure, we observe
algebraiC; sy.m of deviation due to.each single reflection. So,
61 =1t-2a, 62 =1t-2P
6total = L 6!ndMdual ,_
"""""" ' Also ray is rotated in same sense i.e., anticlockwise, so
DO NOT FORGET TO TAKE INTO ACCOUNT THE SENSE OF I
Snet =~i'=Total deviation =31 +l\
ROTATION WHILE SUMMING UP THE DEVIATIONS DUE TO 1'

SINGLE REFLECTION.
=> 6=2it-2(a+P)
Now in !J.OBC , LOBC + LBCO + LCOB = 180°
TWO IDENTICAL PERPENDICULAR PLANE MIRRORS
=> (90'-a)+(90°-p)+0=1so 0
If two plane mirrors are inclined to each other at 90° , the
emergent ray is always antiparallel to the incident ray if it => a+P=0
suffers one reflection from each (as shown in figure) => 6=,2it-20=360°~20
whatever be the angle of incidence.

1.5 =
www.puucho.com
Rahul Sardana with www.puucho.com

Advanced JEE Physics Optics & Modern Physics

Alternative Method: Illustration 2


5 = LBEC + LCEA + LAED Find the co-ordinates of the location of the image
Now, LBEC = LAED (vertically opposite angle) formed for an object kept at origin as shown in figure.
=> LBEC=180°-2(o:+p) y·

=> LBEC = 180° -28 {·: 8=o:+P)


Also, LCEA=2o:+2P => LCEA=2(0::1-P)=28
=> 5=(180°-28°)+28+(180°-28°)
=> 5.=360°-28

REFLECTION FROM A PLANE SURFACE OR PLANE


MIRROR - Solution
When a real object is placed in front of a plane mirror, the The first thing we obsenre is that the object is virtual,
image is always erect, virtual and of same size as the object. because the ray of light is converging on plane mirror. Also,
It is at same distance behind the mirror as the object is in the co-ordinates of object are (0, 0, 0) and the image co-
front of it. ordinates are the reflection of object coordinates in the
mirror as shown in figure.

0
"4------ d -----t+f-- d ----+t d-d- 14-- 8 cm -----+1
(a) Point Object (b) Extended Object
The image lies on norma~ of mirror at I . From MOP, we
have
LATERAL INVERSION
The image formed by a plane mirror suffers lateral- sin(30°) = PO
8
inversion. That is, in the image the left is tu.med to the right
_ and vice-versa with respect to object. Howev.er, the plane => PO=4cm
mirror .does not turn up and down, as shown in figure. => OI = 2(PO) = 8 cm
So co-ordinates of I are
X = -8cos(60°) = -4 cm,
y=8sin(60°)=4.J3 cm and
Z=O
Objeet Image Objeet Image So, the co-ordinates of image are (-4, 4../3, 0)
Actually, the plane mirror reverses forward and back in
three-dimensions (and not left into right). If we keep a right- ; PROBLEM SOLVING TRICK(S)
handed coordinate system in front of a plane mirror, only the j For _finding, the :loC:Btion of an image of a point object placed in
z-axis is reversed. So, a plane mirror changes right-handed i front of. a plane· mirror, we must see the perpendicular distance··of
co-ordinate system (or screw) to left-handed. l the object f_rom the mirror. ·
y y
I -,,.!

¾
0 ' ' I
•-' -- - .---•
J----•x x'+----to·
0 ', -
Coi-rect Incorrect
z z' OM=Ml OM=Ml
Right Handed Left Handed
System System
= 1.6

www.puucho.com
Rahul Sardana with www.puucho.com

Ray Optics: Reflection at Plane & Curved Surfaces


FIELD OF VIEW OF AN OBJECT'
=
MINIMUM SIZE OF A PLANE MIRROR TO SEE A
Suppose a point object O is placed in front of a mirror, then COMPLETE IMAGE
a question arises in mmd whether this mirror will form the CASE-I:
image of this object or not. The answer is yes, it will form. A To find the minimum size· bf mirror to see a full image we
mirror, irrespective of its size, forms the ini.ages of all objects use the fact that light rays from extreme parts of object
lying in front of it. But every object.has its own field of view should reach eye after reflection from mirror. Let us consider
for a given mirror. following two situations
Field of vi_ew is the region where diverging rays· from object a) The minimum size of mirror to see one~s full height is
or image are present. If our eyes are present in field of view
then only we can see the object or an image as the case may H where H is the height of man. To see full image
2
be. Field of view of image is decided by rays which get mirror is positioned in such a way so that rays from
reflected or refracted from the extremeties or the extreme head and foot reach eye after reflection from mirror, as
ends of the mirror or a lens and depends on the location of shown in the figure.
the object in front of mirror or lens.

--r-
(x+y)
y
_J __
f
y
Field of view of image
i E
Man.

b) A ray starting from head (A) after reflecting from


upper end of the mirror (F) reaches the eye at C .
Similarly the ray starting from the foot (E) after
reflecting from the lower end (G) also reaches the eye
at C . In similar triangles ABF and BFC
Field of view of object
AB=BC=x (say)
Similarly in triangles CDG and DGE , we have
CONCEPTUAL NOTE(S)
It _has been observed that a convex mirror gives a wider field of
CD=DE=y (say)
view than a plane mirror. Therefore, the convex mirrors are us~d Now, we observe that height of the man is 2( x + y) and that
as rear view mirrors in vehicles. Though they make the
estimation of distances more difficult but still they are preferred the length of mirror is (x + y) , i.e., the length of the mirror is
because for a large movement of the object vehicle there Is only
a small movement of the image. half the height of the man. Please note that the mirror can be
placed anywhere between the centre line BF (of AC) and
DG (of CE);

CONCEPTUAL NOTE(S)
a) In· order to see full image of the man, the mirror is
Field Field positioned such that the lower edge of mirror Is at height
of of half the eye level from the ground.
view view Minimum size is independent of the·distance between man
of
0 of a
b)
convex plane and ·mirror.
, mirror mirror

CASE-ff:
The minimum length of the mirror required to see the full
image of a wall behind the man who is standing at the
middle of the mirror and the wall is ; , where H is the
·height of wall. The ray diagram for this situation is shown in
figure.

1.7 =
www.puucho.com
Rahul Sardana with www.puucho.com

Advanced JEE Physics Optics & Modern Physics

A · . 360°
f
2x
Ca'.culate · · --
8

t
(x+y)
i
(X;cY) 360' is an INTEGER
8 •., 8
.
. 360' is a· FRACTION

f 1
2y
:t IMsan EVEN! ltis an ODD• n is an Integral'
C Wall Man Mirror Integer lnteg"ef i-~~ Part of fraction
I+-- d d-+t

In mangles HBI and IBC let HI= IC= x . Now, in mangles


HBI and ABF , we have Object lies
symmetrically Object lies
AF FB
' on th~ angle . UnsYmmetrically
HI BI bisector
\ AF 2d
~
-;-=a
~ AF=2x · n='_(· -
360'
- - 1')
_Similarly if, CK= Kf = y·, then DG = 2y. Now, we o]:,serve : . a
that height of the wall is 3(x+ y) while that of the mirror is ,----~~- . ....-:-.-:-----__,.---- -
' • CONCEPTUAL NOTE(S! .
(x+y). a) If an object is placed between two•parallel mirrors (8 = 0°), .
·the number of im?Q_;s formed Wm,be infinite,
~b) All th8·. images. lie on a,_clrcle with radius equal to the
NUMBEROF IMAGES IN INCLINED MIRRORS
distanc~, "-~etween ~e object_,;~ and th0 point· of
Let 0 be the angle between two plane mirrors and' n be the ?1
intersection the mirrors c·. • '• w

number of images formed: The riu(nber ·of images formed may b8 different from the
numberfOf Images see_n (which depends on .the· position ·of,.
360 if 360 is odd the observer). · · · ·
8 ' · 8 ,,'' '

Then n= (360 ) . 360.


M,
[ -1 , if 1s even
8 8
Further when
360
is odd, then
J
1
:
------
J.
1~ 0=9no,.~,.,
l" ,.,.'I-
a
.
·0 ~------6~ "..~., M; "
.l12 I,. _______ ,! __ '.'° ___ -, liz
if.object lies symmemcally on the
~=[(3:360
0
1
_ ), angle bisec:to! of tw~ mirrorS

if object lies tinsymmemcally


a'
Further if
3
:o is a fraction, then the number of images
ROTATION OF A.PLANE MIRROR
When a mirror is rotated by an angle 0 (say anticlockwise),
form~d will be integral part of the fr~ctlon e.g. il :o
3
is 4.8,
keeping the incident ray fixed, then· the reflected ray rotates .
by 28 along the same sense, i.e., anticlockwise.
then, n = 4 . Following diagram shows _the process to y(or N)
calculate n . N (ory)-
'.
N' .. t R'.
Net deviation produced by two plane mirrors.inclined at an ' R '' ''
'''
\ •· 'I

angle 0 is \' :...._,.~


' ,, .
6=360°-28
. '' \ i-0
,a•
Clearly 6 is independent of the angle of incidence of the ray '
oflight. · · · ·
j,•j i-0~
,,
'
Initially
On Rotation of Mirror

= 1.8

www.puucho.com
Rahul Sardana with www.puucho.com

Ray Optics: Reflection at Plane & Curved Surfaces

Let I be the incident ray, N the normal and R the reflected d$ + 2 da =O


ray, then on rotation, I remains as it is, N and R shift to dt dt
N' and R'.
d$ =~2 da
From the two figures we can observe that the reflected ray dt dt
earlier made an angle i with y-axis while after rotating the
So, the angular speed of the reflected ray is double the
mirror it makes the angle (i-20). So, we conclude that the angular speed of the mirror.
reflected ray has been rotated by an angle 20. Since, y = xtan~

CONCEPTUAL NOTE(S) dy =xsec'$d$


If a plane mirror rotates with angular velocity ro , then the dt dt
reflected ray rotates with angular velocity 2ro (excluding rotation
of ~-~~r ~ith n?r~al ~s_t~e ~is). Since l~;l=2oo

2
Illustration 3 => l'Jil=(xsec $)(2oo)
A plane mirror hinged at O is free to rotate in a
vertical plane. The point O is at a distance x from a long So, the speed of the spot is 1:1 = 2xoosec' $
screen placed in front of the mirror as shown in figure. A
laser beam of light incident vertically downward is
VELOCITY OF IMAGE IN A PLANE MIRROR
reflected by the mirror at O so that a bright spot is formed
at the screen. At the instant shown, the angle of incidence To understand and interpret the moving images of moving
is 8 and the mirror is rotating clockwise with constant objects in front of plane mirror, we must understand the
angular velocity o, , Find the speed of the spot at this following cases.
instant. CASE!:
Object moving along the normal to the plane mirror which is
Normal (N) at rest. All velocities measured w.r.t. grOund frame.
,/ Screen
,, y
8 ,'

0 · -------------
X

.- Mirror ··-::.._.;
V
0___.,_ V
______ ,-------....._I Lx
00

Solution (v.= 0)
Let P be the bright spot, shown on the screen. Let the Velocity of object with respect to mirror is
distance of point P from 0 1 be y at this instant shown in Varn= vi
figure. Then according to the problem we need to calculate
Velocity of image with respect to mirror is
dy
dt vhn =-vi
Velocity of object with respect to image is
V01 =V0 -V1 =(2v)i
CASEII:
Object moving parallel to the plane of mirror (at rest)

From the figure


0+0+$=90° ... (1)
!-------:- -_____ J
I
Lx
8+$+a=90° ... (2) l
=> a=0 (v. = 0)
=> $+20=90° Velocity of object w.r.t. mirror is
=> $+2a=90° Dom =vj

1.9 =
www.puucho.com
Rahul Sardana with www.puucho.com

Advanced JEE Physics Optics & Modem Physics

Velocity of image w .r. t. mirror is Solution


Vrm = vj
Velocity of object w.r.t. image is
iio1 =0
CASE III:
Object moving neither along the normal nor alohg the
parallel to the plane mirror (at rest).
The component of velocity of image perpendicular to mirror

o~------
Vo,
1

------~ 1
Vr,,,
Lx is
½=2Vm - \1 0

°" (v,L =2<-2)-(6)=-10 ms-1


For component of velocity of image parallel to the mirror
(v. = 0) (V,)11 =8 ms-1
Clearly; we observe this case to be a combination of Case .I
and Case II. So, here -
(v 01 ), =2v0 x and (v01 ), =0 ·

, PROBLEM SOLVING TRICK(S)


; While solving problems that involve·the calculation of image of an Velocity of image(½)= (v,): +(V,),:
I object w.r,t. any observer, then .
' Step 1 : Firstly, calculate the velocity of image ·w.r.t. mirror 1
J
=> V, =-./100+64 =-./164 ms- and 0=tan-'(¾)
: keeping in rl'llnd·that ·
'
( Vlm ) along l'Ofrror ={ijOm)along mirror
Illustration 5
=> (v,.)11 =(vem)u A point object is moving with a speed of 10 ms-I in
; Since, both the object and the image approach the mirror with front of a mirror moving With a speed Of 3 ms-I as shown
i equal and ORposite speed, so we have
in figure. Find the velocity of image of the object with
(Vimtormaltomlrror = -(Vam)no,ma]tomirror

t·-·
respect to mirror, object and gro~nd.

=> VI-Vm=-(Va-Vm)
=> V1 =2Vm-Vo
; Step 2 : Then the velocity of image w.r.t. mirror is
V,m =.(~1m\ +(V1m)1. Solution
, However, velocity' of image w.r.t. any· other observer, say A is

,-<:ic/--
t
3ms-'
: then given by
5./3 ms-' .
f
I
VIA=V;-VA .
- ~ 5 ~.-, -----
I
Illustration 4
10 ms-
1
't
Find the velocity of image of a,moving particle shown Velocity ofobject, v0 =(-s-./31-sJ) ms-1
in figure.
, Velocity of mirror, VM =3i ms-I
/ For component of velocity perpendiCular to mirror, we have
10 ms-'.,{
/_:\.53° (vlM), =-(VoM)_,_ =-(Vo-VM)
/
,, => (v™)_,_ =-(-s-./31-31)=(5-./3+3)1 ms-1
For component of velocity parallel to mirror, we have
(iit~)1 -(v0M)1 = v0 -iiM = -5}-o =-5}

= c:::=================================
1.10

www.puucho.com
Rahul Sardana with www.puucho.com

Ray Optics: Reflection at Plane & Curved Smfaces

Since, (vlM)=(v,Mt +(v1M) 11


2. A ray of light travels from paint A to a point B after-being
reflected from a plane mirror as shown in figure. From
=> (v,M)=(5,J3+3)i-5f where should it strike the mirror?
B
t
''
=> (s,/3+3)1-sJ=v,-(31) ''
'''
20cm
A
=> (s,/3+3)i+31-s]=v, t ''
5cm
=> v, =[(s,/3 +6)1-sJ] ms-' ' '
1+- 20 cm ---+1

3. A plane mirror is inclined at an angle 8 =60° with


=> v,0 =(5,/3+6)1-sJ-(-s,131-sJ) horizontal surface. A particle is projected from point P on
the ground (see figure) at t =0 with a velocity v at an
=> v10 =(5,/3 +6+5,J3)i +(5-5)] angle ct with horizontal. The image of the particle is
observed from the frame of the particle projected. Assuming
=> v10 = (10,ffi + 6)1 ms-1 the particle does not collide the mirror. Find the time when
image will come momentarily at rest with respect to particle.

Illustration 6
A plane mirror in y-z plane moves with a velocity
-3i as shown in figure. An object O starts moving with a
velocity 4/ + J- 4k . Find the velocity of the image. V

0
e a
p GROUND

4. Two plane mirrors are inclined to each other such that a ray
of light incident on the first mirror and parallel to the second
0• is reflected from the second mirror parallel to the first mirror.
(a) Find the angle between the two mirrors.
(b) Also calculate the total deviation produced in the
incident ray due to the two reflections.
Mirror (M)
Solution 5. Two plane mirrors M1 and M2 _ are inclined at angle 0 as
Since the mirror is placed in y-z plane, so the y and z shown in figure. A ray of light 1, which is parallel to M1
components of the velocity of the image remain the same as strikes M2 and after two reflections, the ray 2 becomes
that of the object. However, perpendicular to the mirror, the parallel to M2 • Find the angle e.
velocity of approach of object towards the mirror is always
equal and opposite to the velocity of approach of the image
M, 2
towards the mirror, so, we have
(voM); =-(v,M),
=> (v 0 ),-(vM), =-(v,), +(vM),
=> (v 1 ), =2(vM),-(vo).
=> (v,), =2(-31l-41 =-lOi 6. Calculate the deviation suffered by an incident ray in the
situation shown in figure after it suffers three successive
So, v, =-10i + ]-4k reflections.
M,
~ ICE I c;;l 1' ' j •.. 50°
BASED ON REFLECTION AT PLANE SURFACES. · · ~~.
(Solutions on page 1.169) :

1. A ray of light travelling in the direction i(i + .Jsj) is incident 30°


M,
on a plane mirror. After refleciion, it travels along the
direction iU -.Jsj). Find the angle of incidence.
7. Two plane mirrors are placed parallel to each other and
40 cm apart. An object is placed 1O cm from one mirror.

1.11 =
www.puucho.com
Rahul Sardana with www.puucho.com

Advanced JEE Physics Optics & Modern Physics

Find th8 distance frolTl the object tO the iesp0ctive image for 13. A ray of liQht is incident on 8.11 arrangement of two plafle--:
each of the five images that are closest to the object. mirrors inclined at an angle 0 with each other. It suffers two '
reflections one from each mirror and finally moves in a ,
1 a. Find the number of images formed of an object O enclosed , direction making angle u with the incident ray ( a is acute). ,
by three mirrors AB , BC , AC having equal lengths ln · Find the angle a and show that it is independent of angle 1
situation shown in figure. of Incidence.
A

6
14. A ray of light is incident at an angle of 30" with the
horizontal. At what angle with horizontal must a plane mirror !
0 be placed in its path so that it becomes' vertically upwards !
after reflection? '
..
s·· · ··c 15. Two plane mirrors are inclined to each other at an angle of
70" . A ray is incident on one mirror at an angle 8 . The
, 9. A point l\>Ource of light S , placed at a distance L in front of ,
· ray reflected from this mirror falls on the second mirror from
the centre of a mirror of width d, hangs vertically on a wall. where it is reflected parallel to the first mirror. Find the
A man Walks in front of the mirror along a line parallel to the value 9.
mirror at a distance 2L from it as shown. Find the greatest
distance over which he can see the image of the light ·
source in the mirror.
1
16. A ray of light is incident on a plane mirror along a vector j
T+ J- k . The normal at the point of incidence is along

l }---------1
I
i + j . Find a unit vector along the reflected ray.

REFLECTION FROM CURVED SURFACES

i.-L_..
A small curved reflecting surface can be considered to be a
part of a sphere. Hence, such surfaces are called spherical
i+----2L----+I
mirrors. Depending upon the surface silvered, these are of
' 10. Find the smallest size of a looking glass which a man with a : two type&--eoncave and convex, as shown in figure. Some
face 24 cm x 16 cm should purchase that will enable him to important terms are described below.
see his·whole face completely, lf the a) Pole or Vertex: Centre P of the surface of the mirror.
(a) man is one eyed.
(b) man is two eyed. b) Centre of Curvature : Centre C of the sphere.
Given that the separation between his eyes is 8 cm . c) Radius of Curvature : Radius R of the sphere.
11. In what direction should A beam of light is to be sent from
d) Principal Axis : Line PC , joining the pole and the
point A (shown in figure) contained in a mirror box for it to centre.
fall onto point B after being reflected once from each of the e) Linear Aperture : Distance XY between the extremities
four walls. If the points A and B are in one plane of the mirror surface.
perpendicular to the walls of the box (i.e., in the plane of the ,
drawing) then in what direction should the beam be sent , Note that Sil).ce lenses are also made of spherical surfaces, the
fromBtoA? above terms also apply to lenses, except that the pole is
replaced by a new term called as Optical Centre.
•A Silvered
surface
•B
ap _f_ri_!l_g!J)_aJ
axis
112. The object and the mirror move with velocity shown in ,
/I~
Silvered R
A
figure. Calculate the velocity of the image.
surface y , ,
', / y
1 ' ,
5ms Concave'-- .. ____ ... _.,.,. Convex
mirror mirror
------- 30° -- Important Terms and Definitions
a) Centre of curvature : It is the centre of the sphere of
which the mirror/lens is a part.

/
10 ms-1

----od3-°~-------
0bject
Mirror
Lx b) Radius of curvature : It is the radius of the sphere of

c)
which the mirror/lens is a part.
Pole : It is the geometrical centre of the spherical
reflecting surface of which the mirror/lens is a part.

= ==================================
1.12

www.puucho.com
Rahul Sardana with www.puucho.com

· Ray Optics: Reflection at Plane & Curved Surfaces

d) Principal axis (for a spherical mirror) : It is the straight SIGN CONVENTIONS FOR MIRRORS
line joining the centre of curvature to the pole. While solving problems, we must follow a set of sign
e) Focus : When a narrow beam of rays of light, parallel to conventions given for convenience. According to this sign
the principal axis and close to it, is incident on the convention
surface of a mirror (lens), the reflected (refracted)
a) Origin is placed at the pole ( P) .,
beam either converges to a point or appears to diverge
from a point on the principal axis. Th.is point is called b) All distances are to be measured from the pole ( P) .
the focus (F) . c) Distances measured in the direction of incident rays
f) Focal length (for a mirror) : It is the distance between are taken as positive.
pole and the principal focus (F) . d) Distances measured in a direction opposite to that of
the incident rays are taken as negative.
g) Real image : If reflected (or refracted) rays converge to
a point (i.e. intersect there), then the point is a real e) Distances above the principal axis are taken as positive.
image. f) Distances below the principal axis are taken as negative.
h) Virtual image: If reflected (or refracted) rays appear to g) This sign convention is used to find the position and
diverge from a point, then the point is a virtual image. nature (virtual or real, erect or inverted) of the image
i) Real object : If the incident rays diverge from a point, formed by the mirror (or lens).
· then the point is a real object. h) Object distance is denoted by u , image distance by v ,
j) Virtual object : If incident rays converge and appear to focal length by f and radius of curvature by R.
intersect at a point behind the mirror (or lens), then the i) Note that generally we keep the object to the left of the
point is a virtual object. mirror (or lens), so that the ray of light starting from
object must go from left to the right i.e., towards
PARAXIAL RAYS positive direction of x-axis. Now· since the distances
Paraxial rays are the rays which are either parallel to the have to be measured from the pole consequently,
principal axis or make small angles with it i.e., these rays are u must always be negative,
nearly parallel to the principal axis, Our treatment for the v is positive (for a virtual image) and negative (for a
spherical mirrors has been restricted to these rays and due to real image).
this we shall be considering the curved mirrors that have / is positive (for a convex mirror) and negative (for a
smaller aperture. However, for the sake of convenience,
concave mirror).
comfort and clarity, we shall be drawing the diagrams of
larger size. For both the mirrors and lenses.
Magnification for a real image is negative i.e., mreal = 0
FOCUS AND FOCAL LENGTH Magnification for a virtual image is positive i.e.,
When a narrow beam of light, parallel to the principal axis mvmua1 =EB
and close to it, is incident on the surface of a mirror (lens),
the reflected (refracted) beam is found to converge to or CONCEPTUAL NOTE(S)
appears to diverge from a point on the principal axis. Titis The convention that all distances measured along the ray of light
point is the focus also called Principal Focus in case of are ,positive and all distances measured opposite to the ray of
mirror(s). The plane passing through the focus and light are negative matches exactly with the Cartesian
perpendicular to the prigcipal axis is called focal plane. coordinate system, where we can simply place the origin at the
pole P and say that all distances to the left of the pole are
negative, all distances to the right of the pole are positive, all
distances above the pole are positive and all distances below
the pole are negative.
+
C

Concave mirror Convex mirror


-
+-----.~---+
+
'
Incident Ray:
'
''P
''
Focal length (f) is the distance of focus (F) from the pole ''
''
(p) of the mirror or the optical centre for a lens. t'

j) For solving problems in which any of u, v, f(or R) is


to be found, we must make ·sure that no convention
should be applied on the quantity to be found. The
================================== 1.13 =
www.puucho.com
Rahul Sardana with www.puucho.com

Advanced JEE Physics Optics & Modern Physics

tmknown quantity will automatically take up its sign


from which we shall make obvious conclusion.
k) The diagrams show the application of sign convention '._-- ...
--- .......
(
',
to curved mirrors.
.--,
·---·------------
c
---
F
p -------p
', ..... _
----r---- -c-·
1(±)1 ------,
:'...t:'
• _)
\~_~:
b) A ray of light passing through the focus (in case of
C F concave mirror) or appearing to pass through the focus

' ' ,-i-· (in case convex mirror) is reflected parallel to the
ce-~.J
--r---
'' ''
''8''
,-------
' f=.B.:'
principle axis.

'
:+--t ~'
·...~) I
'\ 2 ,'
!+--- R .,._
...... --- ',::-------~ ... -..,.... __
--- '..----
P.is Pole, F-isfocus and C is·Centre of Curvature
:(£)1 ------
·--------
c -- F----- F C
:... t:
1 I r
l~_i_j
e'
• _)
p• F C
c) A ray of light passing through the centre of curvature
frincipal Axis -,---_-_-_-_-,----;::>+\·-_--~...--i,--i---],-+-- falls .normally on the mirror and is therefore reflected
r------, t§:}_~] back along the same path i.e., retraces its path.
'f - R '' ' ' '8'
,_ -_ !
'1 -2 ,' :+- f ~ - .
R ----.:
----- ---- ........ __ _
RULES FOR OBTAINING IMAGE F C

These rules are based on the laws of reflection, i.e: the angle
of incidence equals the angle of reflection, i = r and are used
to find the location, nature (real or virtual, inverted or erect) d) Incident and reflected rays at the pole of a mirror are
and size of the image formed by a spherical mirror. Take any synunetrical about the principal axis. (Because for the
two rays coming from any given point on the object. Find pole principle axis acts as normal and by Laws of
out at which point these rays actually meet (or appear to Reflection i = r ). So by observing the size of erect image
meet) after reflection from the mirror. This point is the real in a mirror we can decide the nature of the mirror i.e.,
(or virtual) image. In this way, taking one point after another whether it is convex, concave or a plane mirror.
on the object, the entire image can be constructed. M M
a) A ray of light coming parallel to principal axis, after
reflection passes through the focus (in case of co:ri.cave
mirror) or appears to come from the focus (in case of
convex mirror).

M" M'

IMAGE FORMATION BY CONCAVE MIRROR

I OBJECT
POSITION_
DIAGRAM POSITION OF IMAGE NATURE OF IMAGE
I
~

At infinity
/ ,p
At the principal focus ( F) Real, inverted and
C or in the focal plane extremely diminished
F~

= 1.14

www.puucho.com
Rahul Sardana with www.puucho.com

Ray Optics: Reflection at.Plane & Curved Surfaces

Real, inverted and


Beyond C Between F and C
dirn)nished

Real, inverted and of same


At C At C
size as the object

Between F Real, inverted and


Beyond C
and C magnifi~d

At F or in the Real, inverted and highly


At infinity
focal plane , magnified

Between F Virtual, erect and magnified


Behind the mirror
and P
I

IMAGE FORMATION BY CONVEX MIRROR

I OBJECT
POSITION
DIAGRAM POSITION OF IMAGE
NATURE AND SIZE OF
IMAGE

For all --- Images formed between the AIWays forms a Virtual, Erect
positions of Pole and the focus (F) . and Di_minished Image
object 0 F C
p~

RELATION BETWEEN FOCAL LENGTH (!) AND RADIUS


tani= AN
OF CURVATURE (R) NC
A ray parallel to the principal axis passes through the focus For paraxial rays and_mirrors of small aperture, we have
(as in concave mirror) or appears to pass through the focus
(as in convex mirror). The normal to the mirror(s) •at the ~ieei=·AN ... (1)
point of reflection i.e., A must pass through the centre of NC
curvature. In triangle CAN , we have In triangle FAN , we have

================================::::i 1.15 =
www.puucho.com
Rahul Sardana with www.puucho.com

Advanced JEE Physics Optics & Modem Physics

tan(2i) = AN P=a+i
NF and y=r+P

CONCAVE MIRROR
--
A,- - --A CONVEX MIRROR

-- f ......... -- ..... ...i1


--!~ f''.::--....
~
·,,1
' ... , .................. _
2·' - ....I
--- ,c
0
,1
..... - 1C
'
''
,F
''
'
2i
pp p( I
F,
''
'
''
'
'
'''
''' _ _ _ V ----+I
R
-r-
'''
' t,t-- f----...:
''
'
R
'''
''
'
,
o+----R
i+------u--------><

Again for paraxial rays and mirror of small aperture, we Since by Laws of Reflection, we have
have i=r
tan(2i)" 2i = ~~ ... (2) => a+y=2P
Applying paraxial ray approximation, we get
From (1) and (2), we get
AP AP AP
tana=a=-, tanP.,P=- and tany=y=-
2 (AN)=AN . PO PC PI
NC NF
=> tana+tany=2tanp
2 1
=> - - - - ... (3)
NC NF => AP+ AP = 2 (AP)
PO PI PC
Since, aperture is small, so N coincides with P, so we have
Using sign conventions, we have·
NC e; PC and NF" PF
PO=-u, PI=-v and PC=-R
For convex mirror, we have
1 1 2
PC=+R and PF=+f => --+--=--
(-u) (-v) (-R)
=> t=¾ R
Since we know that / = - , so we get
2
For concave mirror, we have
1 1 2 1
PC=-R and PF=-f -+-=-=- (Mirror Formula}
u v R f
f=~ For Convex Mirror
2
Similarly we can drive a formula for a convex mirror.
So, for a curved mirror of small aperture, focal length is half
the radius of curvature. Since from geometry we know that in a triangle, external
angle equals sum of internal opposite angles, so for triangle
CAO and triangle CAI, we have
MIRROR FORMULA
i = a.+y
For Concave Mirror
Consider a point object O placed on the principal axis of a and P=r+y
concave mirror. A ray of light, incident on the point A at an
angle of incidence i on the mirror. makes an angle r with
-
the normal as sho\\ n in the figure. From the Laws of
Reflection we know that i = r . Further to find the location of
0
' ' 13" -- --... _ ___c
r--·-lr'-<_
the image let us take another ray along the principal axis so p --
~
1,
that it hits the mirror normally at the point P to reverse its ''
path and meet the other ray at I . This point of intersection '
of the two rays happens to be the place where the image is '''
formed. o+--v-'
Since from geometry we know that in a triangle, external i+-----u-------- R----•
angle equals sum of internal opposite angles, so for triangle Since i = r , so we get
CAO and triangle CAI , we have -a+P=2y

= 1.16 c:::=================================
www.puucho.com
Rahul Sardana with www.puucho.com

Ray Optics: Reflection at Plane & Curved Smfaces

Applying paraxial ray approximation, we get For spherical mirrors positive value of m means v and u
-tana+tanP=2tany are having opposite signs i.e. when u is negative v is

2(AP)
positive and vice versa.
=,, _AP+ AP = So for a real object if the image formed is virtual, erect and
PO PI PC
three times the size of the real object then, we have m = +3.
Using sign conventions, we have
Similarly for a real object if the image formed is real,
PO=-u, PI=+v and PC=+R inverted and one third the size of the real object then
1 1 2 1
=,, ---+-=- 1n=--.
(-u) v _R 3

1 1 2 1
=> -+-=-=- [Mirror Formula) Illustration 7
" V R I
An object is placed at a distance of 15 cm from a
Interestingly, the mirror formula is the same irrespective of
the mirror used. concave mirror of focal length 10 cm . Describe the size,
nature and position of the image formed.
NEWTON'S FORMULA Solution
If instead of measuring the object distance and the image The rough figure indicating the pole of the mirror,
distance from the pole, the distances are measured from_ the focus, and the given distances is shown. The sign convention
focus, then we get a modified mirror formula. This modified is also given.
mirror formula is called the Newton's Formula. Let Since, u =-15 cm (negative since it lies to the left of O )
~ 1 be the distance of object from focus and f = -10 cm (negative since it lies to the left of O )
x2 be the distance of image from focus, then Since we have, from mirror formula that
u:::: f 1 1 1
+X1 -+-=-
v u f
and v=f+x2
1111 1 11 -15+ 10 5
According to the mirror formula, we have =,, -=---=-----=--+
v f u (-10) (-15) 10 15 150 150
1 1 1
-+-=-
v u f 150
=,, V=--=-30cm
5
1 1 1
=,, ---+---=-
(/ +x1 )· (f +x,) f
Object +
=,,

=,,
(2/ +x1 +x,)f=(f +x1 )(f +x,)
2/ 2 +(x1 +x,)f = / 2 +(x1 + x,)f +x1x 2
_____!____________ _
C F
p
0 +
=,, X1X2 = f2 '
This i; known as Newton's formula. ''
: Sign convention
'
i.-10 cm-.i'
LINEAR MAGNIFICATION OR LATERAL 1-4-------15 cm------+1'
MAGNIFICATION OR TRANSVERSE MAGNIFICATION
To have an idea of the relative size of the image and the
object, we define linear magnification also called as lateral The negative sign for v shows that the image lies to the left
magnification as of O.
size of the image h Now, the magnification is given by
m=-----~- 2 V -30
size of the object h1 m=--=--=-2
u -15
For both concave and convex mirrors, it can be shown that
The negative sign for m indicates that the image is inverted,
V
m=-- and hence real and is double the size of the object.
u
Thus, we find that the image is real, inverted, twice the size
.
Smcewe knowth 1 1 1 so we get
at-+-=- of the object, and is formed 30 cm in front of the mirror. The
V u I ray diagram is shown in figure.
v f f-v
m=--=--=--
u f-u I

==================================== = 1.17

www.puucho.com
Rahul Sardana with www.puucho.com

Advanced JEE Physics Optics & Modern Physics

--------------•---
c
p

F O C
'
(
~'-----------------¥;/:
Image 14-1 O cm---:
...-15 cm--------ti-t
i + - - - - - - 3 0 c m - - - - - -'
. . V 60
Ma gnif1cation, m=--=--=-2
. u 30
Illustration 8 · Hence, the image· formed will be virtual, inverted and
A beam of light converges towards a point O , behind enlarged, and at a distance of 60 cm behind the mirror.
a convex mirror of focal length 20 cm . Find the nature and
position of image if the point O is
------ - ·-- -
CONCEPTUAL NOTE(S)
__ _______
,, -----·---
(a) 10 cm behind the mirror Note that for the real objects, a convex mirror always gives
virtual and diminished image, but for virtual objects it gives real
(b) 30 cm behind the mirror image if u < f and virtual image if u > f •
Solution
(a) Here, in this case the object is virtual. So, for this we LONGITUDINAL MAGNIFICATION OR AXIAL
have MAGNIFICATION
u=+lOcm, /=+20cm When an object is placed with its length along the principal
axis, then instead of defining the linear magnification we
..
U smgrrurror 1 1 1
farmula, -+-=-,we get define the axial magnification. Mathematically we define
V U f axial magnification, for small objects as
1 1 1 1 1 1-2 -1 Size of image along principal axis
f 20 +10 20 20 m...i
V U Size of object along principal axis
=> v=-20 cm
a 1cation, m =-- =- - - =
M gnif
. . V

u
(-20)
10
2
1 1 1 .
Further since we know that -+-=- 1.e.,
--- V U f
Talcing the derivative of this equation with respect to u , we
get

:Jv- )+ :u(u-')= :Jr')


1

• _ dv _2
=> -v 2 - - u =0
du

=> m-=::=-:: =-(f~uJ =-(ffvJ


Hence, the image formed will be real, erect and
enlarged, and at a distance of 20 cm in front of the
AREAL MAGNIFICATION
mirror.
Areal magnification is simply defined as the ratio of area of
(b) Again, in this case too the object is virtual. So, we have
image to the area of object, so
u =+30 cm, f =+20 cm
.. 1 1 1 we get
farmula-+-=-, m• .Area ofimage A=!C=(_l_)' =(f-v)'
U smgnurror
V U f
= Area ofobject Ao u' f - u f
1 1 1 1 1 3-2 1
RELATION BETWEEN OBJECT AND IMAGE VELOCITY
vfu2030 60 60 FOR CURVED MIRRORS
=> v=60 cm According to the mirror formula, we have

= 1.18 ==================================

www.puucho.com
Rahul Sardana with www.puucho.com

Ray Optics: Reflection at Plane & Curued Surfaces

1 1 1 When the object lies between F and P , then the image


-+-=-
v u f becomes virtual i.e., u and· f are negative while v is
=> v-1 + u-1 =1-1 = constant positive. So from mirror formula we get,
1 1 1 .1 1 1
Differentiating with respect to time, we get -+--=--
v (-u) (-!)
=> ---=-
u V f
-v-2 dv -~-2 du =O
dt dt
~ -u-'(~;)-v-'(!:)=o
~ ~:=-(::)~; ... (1)

Here dudt is the rate at which the object distance u is (-~:)=(::)(-~:)


Now, when u is further decreased, v also decreases to keep
changing i.e., it is the object speed if the mirror is stationary.
1
- constant. So, -du.th
- is e rate at w hichb. ·
o ~ect 1s
Similarly, dv is the rate at which v (distance between f di
dt
image and mirror) is changing i.e., it is image sPeed if the approaching towards mirror and (- ~:) is rate at which the
mirror is stationary. So if at a known values of v and u, the
object speed is given, we can find the image speed from the image is approaching towards the mirror.
above formula.
------::;1' /
---+
/
, ''
C ,;,-" I

p +-
""'oa+, -----"'~,..,.,,,--+ P
''
''' I

1'
' ''t+-- V -----+I
'>+----u----+< Further in this case we observe that the image is always
enlarged i.e., v > u . Therefore, image speed is more than the
Let us take the example for a concave mirror. object speed. Thus, the above entire discussion can simply be
Suppose the object is moved from infinity towards focus, concluded as follows.
then since u is decreasing therefore,
CONCLUSION
-(du) = rate of decrease of u
dt .
{object speed) When an object is moved from -oo to F , the image (real) moves
from F to -oo and then when the object is further moved from F
to P image (now virtual) moves from +«.> to P .
=> ( !~) = rate of increase of v {image speed}
~---------------
CONCEPTUAL NOTE(S)
Further, when the object lies between oo and C, then v < u , ·When the object Is either at centre of curvature C ot at pole P ,
the two speeds are equal. When the object is at pole, then due
{from equation (1)) to the small aperture of the mirror, it car) be assumed, as if the
image is being formed by a plane mirror.
Hence, when the object is moved towards the mirror, its
----------·
image (which is real) will recede from the mirror with speed FINDING COORDINATES OF IMAGE OF A POINT
less than the speed of object.
If the coordinates of a point object (-x 0 , -y0 ) with respect
When the object is at C, image is also at C
to the coordinate axes shown in figure are known to us and
~ V=U
the coordinates of image be (x;, Yi) then for finding the
x-coordinate, we use the mirror formula, acc-0rding to which
1 1 1
-+-=-
Hence, when the object is at C speed of image is equal to the v u f
speed of object.
1 1 1
When the object lies between C and F then v > u -+-=-
X; Xo f
so, the image speed is more than the object speed.

1.19 =
www.puucho.com
Rahul Sardana with www.puucho.com

Advanced JEE Physics Optics & Modern Physics

=> X
i
=___Eg_
Xo-f-
So, image coordinates are ( - ~O, -1) cm 7 I

y
I PROBLEM SOLVING TRICK(S)
i\ a) Place the object to the left of the mirror (or lens), so that
(-Xo, -yo)• II sign convention matches with the familiar sign convention in
_ ___.,\,________.x ,
bj
the coordinate geometry.
Both for concave as well as convex mirrors, use ,he same

JP mirror formula i.e .

..!.+..!.=! and m=-~


V U f U
: c) Substitute the numerical values of the given quantities with
For finding the y-coordinate, we apply the concept of I proper sign (+ve or -ve) as per sign convention.
magnification (m) , according to which, we have I
' d) Though the SI unit of distance' is metre, it may be more
convenient in some problems to take the given distances, in
cm rather than in m. But then your answer too will be in cril.
e) Do not ,-give any sign to .the quantity to be determined. In
your answer, the unknown quantity will be obtained with its
=> proper sign.
In addition to the above hints, if you remember the following
facts, it will ·help you.
a) Since the object is always placed to the left of the mirrc;>r-so,
I EXAMPLE 'I
u is always negative.
I A point object is placed at (-40, 1) cm in front of a concave
b) For a concave mirror, f is negatiVe.
! mirror of focallength 5 cm having Its pole at the origin
1
c) For a convex mirror, f is positive.
(0, 0). Assuming the principal axts to be along x-axis, flrid d) A real image is formed in front of the mirror, so for a real
the pOsitlon of th.e image form~d. image v is neg·ative.
SOLUTION e) A virtu·a1 image is formed behind the mirror, so for a virtual :
The si!uation discussed in the problem is shown in figure. image v Is positive. 1
y-axis f) A real Image is always inverted, so for a real image h· IS
negative.
(\ ' g) ·A virtual image is always erect, so for a virtual image h is
I positive. ·

7cm \p i h) For the real image of a real object and the virtual image 6f. a
virtual object, m is negative.
--'------l-------__.~---x-axis
(0,0)
,/ Ii ")I
For the virtual-image of a real object and the real image of a
virtual object, .m is positive.
l
1,
'
'
, ''
..,__4ocm-1>-1 GRAPH BETWEEN ! VERSUS !
V U

Let us first take the case of a concave mirror. Here, two cases
1 1 1 are possible.
-+-----
v (-"-40) (~5) Case .1 : 'When the Image formed is Real.
40 When the image is real, i.e., object lies between F and
=> v = ----,. cm
7 infinity. In such a situation u, v and f are negative.
. h1 V
S1nce, m·=-=-- Hence, the mirror formula i.e., ! + ,! =.!. becomes
h0 U V u I
h, (-~o} 1
----=--
v u
1 1
I
h0 "'.~ -40
1 1 1
~=-! => -+-=-
h 07 v u I
But h0 =7 cm 1 1 1
=> -=--+-
=> h, =.-1 cm _____ _\ v u I
--- - --···- --·-- ---

= 1.20 r::::=================================
www.puucho.com
Rahul Sardana with www.puucho.com

Ray OpHcs: ReflecHon at Plane & Cnrved Surfaces


Comparing with y = mx + c, the desired graph. )Viii be a 1 1 1
straight line with slope -1 and intercep,t on y-axis is equal to v+ C---u) =t
1 1 1 1
~ -=,-+-
f v u I
Do not confuse here, the slope m with magnification. Comparing with y = mx + c , the desired graph is a straight line_
1/v
of-slope m = ,1 and intercept on y-axis equal·to i. The graph i$
thus shown in figure.
1/1~.
1/v

I~
4 ---""~"---1/u
1/f
[/
/ 1/f
Case 2 : When the Image formed is Virtual. ,/
When the image is virtual, i.e., object lies between l' and P . .. ~45°
~....cc=---1-----------1/u
Under such situation .u and / are negative whil~ v is
positive. The'mirror formula thus becomes
PROBLEM SOLVING TRICK(S)
-v1 ---
1 1
u f a) As focal-length of a spherical mirror f =~- dep·ends only on
Comparing it with y = mx + c the desired graph is a straight the radius of mirror and is independent of wavelength of
. . .· .. . 1 light and refractive index of medium so the 'focal length of a
line with slope m ': 1 andintercept on y-axis is equal to - . spherica! mir_ror in air or water' and for rec;J or blue light is
1 same. This is also why the image _formed by mirrors do not
1/v show Chromatic aberration.

b) In case of'spherical mirror if R---+ oo (i.e., it becomes plane),

...J._ _...,<..,4,;5,__'_ _ _ 1/u


f= ~ = oo , the mirror formula
/1/f
//,.,, 111' 11
-+-=- reduces to-+-= O i.e., v =-U
V U f V U
-1/f ,.
i.e., image is· at same distance behind the mirror as the·
object Is infront of it. This in turn verifies .the correctness. of
mirror formula.
The graph is thus shown in figure. The two graphs can be c) Every part of a mirror forms complete image. If some
.drawn in one single graph as in figure. portion of a mirror is obstructed (say covered with, black
paper), then complete image will be formed _but intensity will
be reduced.
1/v
d) In case of spherical mirrors ii object distance .(x,) and
image distance (x2 ) are measured from focus instead of
pole, u=(l+x,} and v=(i+x,) ~o the mirror formula
1
..:!. + .:!. = ! reduces to - -- + -1- = ! which on
V u I (l+x,) {f+x,) I
simplificatiori gives x 1x 2 = f2 • This result is called 'Newton's
formula;.
e) If an object is moved .at constant.speed towards a concave
mirror from Infinity to focus, the image will ·move (slower in
the beginning and faster later on) away from·the mirror. This
CONCEPTUAL NOTE(S) is because, during the time the. object.move~ from oo to C
the image will move from F to. C· and when the object
Please note that .!U and ..! are actually the magnitudes of
V U moves from C to F the image wUI move from C to co • At
C the speed of object and image will be equal.
and .!V (i.e., without sign)
f) concave mirror behaves as convex lens (both convergent)
For a convex mirror, the image· forrh8d,,is always virtual, i.e., u while convex mirror behaves as concave lens (both
is always negative while v and f- are always positive. Hence, divergent). This is shown in figure.
the mirror for.mul~~-~co!'JlBS,, _ _ _ _ _ _ _ _ _ __
1-------- ~-------------------
1.21 =
www.puucho.com
Rahul Sardana with www.puucho.com

Advanced JEE Physics Optics & Modern Physics

(21, 21). This all'is shown'ln figure.


V

·F
'' U=V
I'
''
'
Concave mirror Convex lens 2f ___
-+-'' ,.._--------
- ,--1---------
(a) Convergent behaviour '
IL-L-...L-----+U
I 21

Illustration 9

A thin rod of length l_ is placed along the principal


3
axis of a collcave mirror of focal length f such that 'its
image, which is real and elongated, just touches the rod.
·convex mirror Concave lens What is the -magnification ?
(b).Divergenf behaviour Solution
g) As convex mirror gives erect, Virtual and diminished image, According to the problem, the image is real and
field of view is increased. This is Why it is used as rear-view enlarged, the object must have been placed between C and
mirror'in vehicles. Concave mirrors give enlarged erect and
virtual image (if object is between F and P ) so are used by
F . Since one end of the image ju~t touches one end of the
dentists for examining teeth. Further due to their converging object so, this end must lie on C .' Let AB be the object and
property concave mirrors are also used .as reflectors in A'B' be its image, such that A and A' both lie at C, as
automobiles head lights aniJ' search lights and by ENT shown in figure.
surgeons in ophthalmoscope"
h) For .real extended objects, ifihe image formed by a sinQle
mirror Is· erect it is always virtual and in this situation if the
size of the image is
A B
smaller.than object
i
equal to object larger than object
TL--===~--tfc.c~=::ii ·------~--------------- p

• the mirror Is convex. • the mlrror·ls plane • the mirror Is concave s· i+-1/3 _,


''
'
,.__ _ _ _ _ 21 - - - - - I . t i

F O VP I Now, as the length of the object AB is l_, so the distance of


3
, end B of the object from the pole P is
m<+1 m=+1 m>+1
i) For real extended objects, if the image formed by a single
mirror is inverted, it is always real (i.El., m is ~ve) and ,the u,=-(PA-f)=~(2/-f) =-(¾)t
mirror.is concave. In this situation If the size of image is
The distance of the image of end B , v, , is calculated by
i
smaller than object
i
equal 'to object
i
larger than object
using the mirror formula,
• object is between • object is at C • object is between 1 1 1
-+-=-
ooand C • and image is at C C and F v u f
• 8.nd l,mage between • and Image between
F-and-C Candoo · 1 1 1
-+--=-
v, -~f f
3

J
5
v, =-2.f
Therefore, the size of the image A'B' is
-1 <m<D m=-1 m>-1
f j) .In case of spherical _mirrors if we plot a graph between u A'B' =Iv, 1-1 v, I=~f -2f ='I_ f
I• and v the graph will be a hyperbola as foi' u = f , v = oo . 2 2
~ _ _and for u = oo, v = f. A line u =:__v will c~t.!his _h.Y~~~~~ at

:::::::J 1.22

www.puucho.com
Rahul Sardana with www.puucho.com

Ray Optics : Reflection at Plane & Curved Surfaces

.. . A'B'
Now, magnification, m = - AB = -(
2
=-
3 (f)!)
2
Illustration 11
Illustration 10 Find the location, size and the nature of the image of
A gun of mass m1 fires a bullet of mass m1 with a an object of height 2 mm kept between two mirrors (as
shown in figure) after two successive reflections,
horizontal speed -v0 • The gun is fitted with a concave considering the first reflection at the concave mirror and
mirror of focal length f facing towards the receding then at the convex mirror.
bullet. Find the speed of separations of the bullet and the M,(f, =15 cm) M,(f, = 20 cm)
image at the instant just after the bullet is fired from the
gun.
Solution
Let v1 be the speed of gun (or mirror) just after the P, 2mm :(
ii
firing of bullet. By Law of Conservation of Linear
Momentum, we have
m2Vo = m1V1 !
I .\\

=> ... (1)


'
14--20cm-+-1' '
1
' '
•'-----50cm-----'
'
Now, du is the rate at which .distance between mirror and
dt Solution
bullet is increasing, so
As asked in the problem, let us. first consider the
du
-=V1 +Vo ... (2) reflection at mirror M 1 . Before executing the mirror
dt
formula, we must keep two things in mind.
1. The incident light must go from the object to the mirror
Since, we know that.1dvl =(__j_)'ldul
dt
m,
dtf-u ... (3)
and we preferably take it parallel to the principal axis.
2. All distances have to be measured from the pole of the
respective mirror for which reflection is being
considered.
m, 3. All distances measured along the incident ray are
Vo,..__ . . . . ---v,
Bullet positive and all distances measured opposite to the
incident ray are negative.

Since, at the instant just after the bullet is fired from the gun,
the bullet is actually very close to the pole of the mirror, so
u ---t O and hence we get at that instant
{
:: =(f~uJ =m'=l ?I
I
,'~
. l'
I,_

I
I

So, from (2) and (3), we get


: : :~ :
I
'
I
I I
'
I
I
. '
I
I
dv du i+-20cm-+iI 1+-20cm....,.
dt = dt = V1 + Vo ... (4) I I , I

' ' '


'4-----50cm---------~
10cm
where dv is the rate at which distance between image (of Figure is just representative and not to scale
dt
bullet) and mirror is increasing. Now, for reflection at concave mirror M 1 , the incident ray
Therefore, speed of separation of bullet and image will be, from the object goes to left of object and object distance is
v, =2(v1 +v0 ) measured towards right of pole ,Pi , so
Substituting value of v1 from equation (1) we get u=-20cm
Similarly, / 1 = -15 cm
Now, according to mirror formula, we have
1.23 =
www.puucho.com
Rahul Sardana with www.puucho.com

Advanced JEE Physics· Optics & Modern Physics

1 1 1 Solution
-+-=-
v u t The ray diagram for the situation is drawn in figure
(but not to scale).
1 1 1
=> ;+ (-20) = (-15)
=> v=-60 cm
Negative sign with v means that it is formed to right of pole
P1 at a distance of 60 cm from P1 , ( 10 cm· behind M, ).
V . (-60)
=> m, =--;;=- -20 =-3
So, image ( 11 ) formed is real, inverted and three times size
of object i.e., 6 mm.
This image (I1 ) formed now acts as object for the convex
mirror. Further, this image formed is 10 cm to the left ~f P,
M,
and the incident ray from the original object goes to the right 20 cm --+1
1+-
for reflection at M 2 to take place, so .
1+- 30 cm ---+1
u=+l0 cm 50 cm - - - - - - ; M
Similarly, / = +20 cm i+------60cm--------<M

Applying the mirror formula,.!.+.!:.=.!., we get


V u I For reflection at concave mirror M 1 , .we have
1 1 1 u=-20 cm
-+-=-
v 10 20 '!, =-15 cm
=> v=-20 cm
(-20) Since, .!:_ + .!:_ = .!.
=>
V
m2 =--=---=2 V u I
u 10
1 1 1
So, image (I) formed is virtual, erect and two times the size => -+--=--
v, (-20) (-15)
of object (here 11 ). Hence the size of I is 12 mm . So, finally
=> v1 =-60 cm
I is formed at 20 cm in front of convex mirror M2 , with
size 12 mm ; virtual and erect. So, magnification m., = _.'.J. = - -GO = -3 (Inverted)
u -20
Illustration 12 => A'P' =m, (AP)=3x2=6 mm
Find the co-ordinates of image of point object P For reflection at convex mirror M2 , we have
formed after two successive reflections in figure, u=+l0cm
considering the first reflection at concave mirror and then
at convex mirror. f, =+20 cm
y f.:,=20 cm
Since.!:.+.!:.=.!.
V u I
f,=15cm P ' 1 1 1
~ -+-=-
v, 10 20
a
~O-------E-\--'--_.X -+ 2mm =>
.
Agam, ma
v 2 =-20 cm

gnif' . v, (-20)
1cation, m2 = - - = - - - = 2
u 10
=> C'P'=m,(CP')=2x8=16rnrn
M,
M, So, the co-ordinate of image of point object P as measured
t+-20 cm_.,. from the origin O is (30 cm, -14 mm)
1+-----SOcm-------iM

= 1.24

www.puucho.com
Rahul Sardana with www.puucho.com

Ray Optics: Reflection at Plane & Cttrved Sttrfaces


to the image ofthe swinging bait
(b) At what point does the ball appear to coincide with Its
BASED ON REFLECTION AT CURVED SURFACES image.
(Solutions on page 1.173) (c)_ What will be the lateral magnification of the .image of
1. An object -of height 2.5 cm is placed at a 1.5 f from a T
the. ball· at time t = , where T is time period of
concave mirror, where f is the magnitude of the fcical 2
length of the mirror. The height of the object is oscillation?
perpendicular to the principal axis. Find the height of the
image. Is the image erect or inverted?
C
2. A mir/'or (in a laughing gallery) .forms an erect image fbur x~axis+----------H
times enlarged, of a boy standing. 2.5 m away. Is the. mirror X=O
concave or convex? What is its·radiu~ of curvature?

3. A concave mirror forms the real Image .of a point source


8. An Image I is, formed· of a point object 0 by a mirror
lying on the optical axis at a distance of 50 cm from the
whose principal axis is AB as shown-in figure.
mirror. The focal length of the mirror is 25 cm . The mirror is
cut' in two halves and these halves are drawn apart at a 0
distance of 1 cm in a direction perpendicular to the optical

axis. How will the images formed by the halves of the miriOr A B
be arranged?
• I

(! __+____________ _ (a) State whether it is a convex mirror or a concave


mirror.
(b) ,Draw a ray diagram to locate.the mirror.and its focus.
' \ 1 ~m __________ _ Write down the steps of construction of the ray
diagram. Consider the possible two cases:
(i) When distance of I from AB is more than the
distance of O from AB and
(ii) When ·distance ·of o· from AB is more than the
4. Find the distance of Object from a concave mirror of focal
length 1O cm so that image size Is four times the size ·o_t the distance of I from AB
object.
9. Convex and concave mirrors have the same radii of
curvature R. The distance between the mirrors is 2R. At
5. A concave mirror has a radius of curvature of 24 cm . How
what point on the common optical axis of the mirrors should
far is an object from the mirror when the image formed is
a point source of light A be placed for the rays to converge
(a) virtual and 3 times the size of the object.
at the point A after being reflected first on the convex and
(b) real and 3 times the size of the object and
then on.the concave mirror?
(c) real and ¾the size of the object?
1D. An object ABED is placed In front of a concave mirror
6. A thin flat glass plate is placed in front of a convex mirror. At beyond centre'of curvature C as shown in figure. State the
what distance b' from the plate should a point source of shape of the image.
light S be placed so that its image produced by the rays
reflected from the front surface of the plate coincides with
-1-B_,_E_____---J\P
the,image formed by the rays reflected fi'om the mirror? the
focal length of the mirror is f = 20.cm and-the distance from
the plate to. the ·mirror a= 5 cm . How can the coinciderice
of the images be established by direct observation?
A D C F I
,,. An_ object is 30 cm from a spherical mirror, along the
central axis. The absolute value of lateral magnification of
an inverted image is i. Find the focal length of the mirror?
-a-----b----+t

12. A thin-rod of length ½is placed along the principal axis of a


7. A ball swings back and forth- In front of a concave mirror.
The motion of the ball is described approximately by the concave mirror of focal length f such that its image just
,__ _,,toLJchesJhl;! rod. Calc_ulate magnifica_te;io,..n~._______...,
equation x =fcos(rot), where f is the focal length of the
mirror and x is measured along the axis of mirror. The
origin is taken at the centre of curvature of the mirror.
(a). Derive an exRression for the di_stance from the mirror
1.25 =
www.puucho.com
Rahul Sardana with www.puucho.com

REFRACTION OF LIGHT AT PLANE SURFACES is simply called as the Absolute Refractive Index of
medium 2, expressed as µ 2 or simply µ .

T
he phenomenon of the bending of light rays as they
travel from one medium to the other is called
. Refraction. The surface separating two media is called REFRACTIVE INDEX (RI)
an Interface. In other words, the phenomenon of bending of
The refractive index of a medium is not determined by its
light rays at the boundary between two media is called
density. It is,govemed by the velocity of light in the medium.
refraction.
The lesser the value of the velocity of light, the more is the
Incident Ray
N refractive index of the medium, and the denser. is the
A
medium. A medium having greater refractive index is called
: Normal
i' denser medium whereas the other medium is Called rarer
Medium 1 (µ,)
Interface , Q1 1'
medium.
, ~
1r
:
-t> 1,Medium 2 (µJ
-!, -j ABSOLUTE REFRACTIVE INDEX
~

, ~
-!, I -t, -t,
The absolute refractive index of a medium is defined as the
, ~
Refracted Ray
B
ratio of the speed of light in vacuum to the speed of light in
the medium,
speed of light in vacuum C >1
LAWS OF REFRACTION
µ speed of light in medium V
a) The incident ray, the refracted ray and normal at the
point of incidence to the surface separating the Mo Absolute refractive index is more than one because the speed
media all lie in the same plane. of light is maximum in vacuum/ air.
b) Snell's Law
RELATIVE REFRACTIVE INDEX
For two media, the ratio of sine of angle of incidence
i to the sine of the angle of refraction r is constant The relative refractive index of medium 2 with respect to
(for a beam of particular wavelength). For a given set of medium 1 is denoted by 1 µ 2 and is given by
media this constant is called the refractive index of the
medium 2 with respect to medium 1 (represented as
1µ2) i.e.,
1 _ µ, _(
µ, -;,--(
:J:J-_ v,
v,
sini µ
-.- =constant= -2 = 1µ 2 (SNELL'S LAW)
smr µ1
The relative refractive index of medium 1 with respect to
OR µ 1 sini=µ 2 sinr
medium 2 is denoted by 2 µ 1 and is given by
where µ 1 and µ 2 are Absolute Refractive Indices of
Medium 1 and 2 respectively and 1 µ 2 is the refractive
index of medium 2 with respect to medium 1. If
2 µ, ( v,
µ,=-=-(-=-
µ, v,
:J:J
medium 1 happens to be the vacu~m, then the constant

===================================== = 1.27

www.puucho.com
Rahul Sardana with www.puucho.com

Advanced JEE Physics Optics & Modern Physics

CONCEPJ"UAL NOTE(S) 1 => sini < sinr


a) The velocity of light in air Is nof.much different from that in => i'< r
vacuum. Hence, while defining the refractive index of a
me_dium we often take velOcity of light in air rather, than that
REFRACTION : IMPORTANT POINTS
in vacuum
Medium Refractive Index'"' ' a) Whenever light goes froni. one medium to another, the
frequency of light (/) remains unchanged. Since
Water .±=1.33
3 C Speed of light in vacuum
µ=
Glass , !=1.50 V Speed of light in medium
2
b) Relative refractive index can be less than one. If we
calculate the refractive index afwater with respect to glass,
then
where A-., and "modi= being wavelengths of light in
8 '
9µ =·µw =-=-<1
(34) air and medium respectively.

• µ, (¾) g
sini µ2
C

v2 v1 "-1

c) Refractive index is different for different wavelengths for a


sinr - µ - c - v - "-
1 2 2
---~~r_:ifmedia,because µ 1A1 =µi. 2 - _ _ _ _ _ _ _ _ __ V1

(MODIFIED FORM OF SNELL'S LAW)


BENDING OF A LIGHT RAY From above we conclude that
a) When light passes from rarer to ,denser medium, it µ1"-'1 = µ2"-2
bends towards the normal i.e., a·light ray passing from o µ;\, =constant
air to ·water bends towards the normal as shown in the
figure. Also, we conclude that
µ1V1 =µ2V2
''
i'' => µv = q:mstant
f', Air
2 1
µ, b) µ1 x µ 2 =1
Water
'r 1
'
I'
2
µ1=-
'µ,
c) When light propagates through a series of parallel
According to Snell's Law layers of different medium as shown in the figure, then
µ 1 sini = µ 2 sinr the Snell's Law may be written as
µ 1 sin0 1 = µ 2 sin0 2 = µ 3 sin03 = µ 4 sin04 = constant
si~i=~>l ,.
sinr µ 1 · In.general, µsine= constant
=> sini >sinr
=> i > r
b) When light passes from denser to rarer medium it
bends away from the normal as i.e., a light ray ·passing
from water to air bends away froffi the normal shown in
the figure.

d) If light is incident normal to a boundary (i.e. i = 0 ),


Air
then, it passes undeviated from the boundary as shown
µ, Water in the figure.

µ,'

According to Snell's Law


µ 1 sini=µ 2 sinr

sini =~<l µ2* µ1


sinr µ 1 Condition for no refraction
= 1.28 ==================================
www.puucho.com
Rahul Sardana with www.puucho.com

Ray Optics: Refraction at Plane Surfaces

e) ·If the refractive indices of the two media are equal as C


V
shown in figure, then also the light ray is not refracted
and the boundary between the two media is not visible. n(1+;J
0

This is why a transparent solid is invisible in a liquid of y


same refractive index. .
Smce, dx sowe h ave
v=-,
dt
-----T-------~
dx C
'' '
l'
dt n,(1+;.) '
1, V

G) :®
Jl:?=µ1 =µ
fdt=~J(1+..::..)ax
0 2a
C O
'
x-no.t:---::-,-_..-:c--,x~-~a
0 •x

Condition for no refraction


t = 5n0 a
fJ Note that for sound waves,
4c
speed in air, v 1 = 330 ms-1
speed in water, vw =1500 ms-1 Illustration 15
Therefore, the refractive index of water with respect,to For the arrangement shown in the figure, a light ray is
air, for sound waves is incident at an angle of 60° on the layer of water. Find the
angle betweeh this ray and the normal to the glass.
, = !!,_ = 330 = 0.22
µw Vw 1500 6001 ''
Air(µ,,= 1)
Thus, we find that for the refraction of sound waves,
water is rarer than air.
Water (µ, = 4/3)

Illustration 13
A ray of light falls on a glass plate of refractive index
n = -./3 . What is the angle of incidence of the ray if the
angle behveen the reflected and refracted rays is 90° ? Solution
Solution According to Snell's Law, we have
According to Snell's Law µ 0 sin(60°) =µ 1 sinr1 = µ2 sinr2
sini
n=--
sinr µ 0 sin(60°)
Since i+r=90° µ,
~ r=90-i
-./3 = sini tani
sin(90-i)
i = tan-1 (-./3) = 60°
Illustration 16
Illustration 14 A ray of light goes .from air to medium of refractive
A ray of light passes through a medium whose index µ. If· i be the angle of incidence, r be the angle of

refractive index varies with distance as n=n + ;a) .If


0( 1
refraction and 6 be the angle of devi~tion, then prove that

ray enters the medium parallel to x-axis, what will be the tan(¾)=(~:~}an(i;r )-
time taken for ray to travel between x = 0 and x =a?
Solution
Solution The angle of deviation 6 is given by
Since, we know that µ = 5.. O=i-r ... (1)
V
According to Snell's Law,
C
=> V=- sini
µ µ=-.-
smr
So, if v be the speed at a distance x from y-axis, then

==================================== 1.29 =
www.puucho.com
Rahul Sardana with www.puucho.com

Advanced JEE Physics Optics & Modern Physics

9=i
tan9= tani
AIR
dy
-=tani
. . .. (1)
MEDIUM(µ) dx
y

Applying componendo and dividendo, we get


µ-1 sini-sinr ---7 ~ ----P(x, y)
µ+1 sini+sinr

µ-1
2cos c+r).
2
sin c-r)
2 a'
'-'-.....:C-"'--,,o+----~---'--•x
=>
µ+1
z z
. (i+r) cos (i-r)
2sm
Applying Snell's Law at O and P, we get
µ0 sini0 =µpsinip
µ-1 tan(%)
=>
µ+1 =>. (l)(sm90°) =( µ'x)sini
tanC;r)
1--
r
(2a) = (µ-1)
tan

Illustration 17
µ+l tan (i+r)
2
sini=[ ~
1
~J
0

A long rectangular slab of transparent medium of


thickness d is placed on a table with length parallel to the tan i = --,==('=l=-='~")'==7 ... (2)
x -axis and width parallel to the y-axis. A ray of light is
µ;-(1-~)'
travelling along y-axis at origin. The refractive index µ of

the medium varies as µ = µ(\), where µ 0 and r(> 1) are


From equations (1) and (2), we get
1- -
r
constants. The refractive index of air is 1.
dy = --,==("'l=-=~=)-,,.dx
y µ;-(1-~)'

A Integrating, we get

i
d
d

fdy=f
X 1-~
r dx

l ~--o=<------~-x o o µ~-(1-~)'
(a) Determine the x-coordinate of the point A, where the => x=r[l- µ;-(~+1µ;-1)']
ray intersects the upper surface of the slab-air
boundary.
(b) Write down the refractive index of the medium at A . (b) Atpoint A, l-~= µ;-(~+1µ;-1)'
(c) Indicate the subsequent path of the ray in air.
Solution
·.-µ=~}
(a) Refractive index is a function of x, i.e., the plane { 1--
separating the two media is parallel to y-z plane or r

normal to this plane at any point is parallel to x-axis. (c) After A, medium is again air. Hence, from Snell's Law,
Further refractive index increases as x is increases. So, angle of incidence will again become 90' or it will
the ray of light will bend towards normal and the path move parallel to y-axis as shown.
is shown in figure. Let at the point P(x, y) the angle of
incidence be i . Then

= 1.30 ==================================
www.puucho.com
Rahul Sardana with www.puucho.com

Ray Optics: Refraction at Plane Surfaces


y
h=(o.2{½)( ~)=0.086 m
A Hence, height from the mirror is
d = h + R = 0.1 + 0.086 = 0.186 m
(b) Using the principle of reversibility of light, we get
'--~o+---------'----+X i=2r=60°

Illustration 18
A cylindrical glass rod of radius 0.1 m and refractive
index ./3 lies on a horizontal plane mirror. A horizontal
ray of light moving perpendicular to the axis of the rod is
T
incident on it.
(a) At what height from the mirror should the ray be Now, QU =coti=cot(60°)= ~
incident so that it leaves the rod at a height of 0.1 m TU v3
above the plane mirror? _TU _0.1
=> Qu - ./3 - ./3
(b) At what distance a second similar rod, parallel to the
first, be placed on the mirror, such thcit the emergent
So, the desired distance is
ray from the second rod is in line with the incident ray
on the first rod?
OC = 2(0.1) + 2( ~) = 0.315 m

Illustration 19
• An opaque sphere of
radius R lies on a
horizontal plane. On the R
perpendicular through the
Solution point of contact there is a
Let us first draw the ray diagram for the situation. point sonrce of light a
distance R above the
sphere. R
(a) Find the area of the

I R
t------
shadow on the plane.
(b) A transparent liquid of refractive index ./3 is filled
above the plane such that the sphere is just covered
with the liquid. Show that new area of the shadow.
Solution
(a) Since, PO= OQ (a) The situation is shown in the fignre
g.:::\I'-.::.
=> LOPQ = LOQP = r (say) ,,--
....

Also, i=r+r=2r
In Af'OS, we have
h = OPsini = 0.lsini
=> h = 0.1sin2r
=> h=0.2sinrcos,r ... (1)
Applying Snell's Law at P, we get
r,; __ sini 2sinrcosr
3
'V;) = 2 cosr
sinr sinr
Since, we observe that
=> r=30°
Substituting in equation (1), we get sin0=0Q= R =1
OS 2R 2

1.31 =
www.puucho.com
Rahul Sardana with www.puucho.com

Advanced JEE Physics Optics & Modern Physics

=> 0=30° A'= rcR'2 = 2rcR2


Further, radius of shadow is given by
r =MP= MStan30° CONCEPT OF OPTICAL PATH LENGTH (OPL) AND
REDUCED THICKNESS
=> ~~(3Ri(l)=v'3R If a distance L separates two buildings, then the measured
distance has nothing to do with the medium between the
So, area of the shadow is buildings. If this separation is filled with water, then too the
2 distance behveen the buildings is L . However the time
A= 1t( v'3R) = 3itR 2
taken by the light to travel between the buildings is different
(b) The situation is again shown in the figure. for different media between the buildings. This time
difference is due to the interaction of the light with the
molecules of the .medium which impede (slow down) the
light's velocity and this cause the light to take more time to
travel the same physical distance for different media.
Due to this a,new concept of distance needs to be introduced
that accounts for the delay in the travelling time of the light
in water (or a denser medium) in comparison to air (or a
rarer medium). This new distance is called the Optica! Path
LIQUID Length (OPL) or Optical Path and takes .into account the
slower velocity of light within a denser medium and it is
simply the product of the distance with the refractive index
AB=AD+BD i.e.1
AB=AE+BD [by geometry} OPL=µL
Further AB = R' + R tan i Thus, light passing through a denser medium seems to travel
a longer distance than the light propagating in free
AC 2R
=> cosr= AB R'+Rtani space/vacuum, during the same time intervals for both the
media.
=> R'=2Rsecr-Rtani . . . (1) Let me illustrate this thing to you. For that let me take_ two
Also, we observe that media, one rarer of length L1 , refractive index µ1 and other
R' =BC +CM =2Rtanr+Rtani ... (2) denser of length L2 and refractive index µ 2 , as shown.
From equations (1) and (2), we get
2
2secr-tani = 2tanr + tani
RARER(µ,) DENSER(µ')
=> secr-tanr=tani
V1={f; V2= ~2
Using the concepts of trigonometry, we get

---L,---.. ---L,---..
1-sinr
,tani Time taken by light to travel a distance L, in rarer medium
cosr
. h speed v = -C 1s
wit •
1
µ,

=> tani t,=h=(L')=µ,L, ... (1)


V1 :, C

The time taken by light to travel a distance L2 in denser


=>
medium with a speed v2 =_..:... is
=> 2i =!:__!.. ... (3) µ,
2 2
Also, from Snell's Law, we get ... (2)
sini=v'3sinr ... (4)
Solving the above equations, we get Now if both times are equal, as said above, then
R' =.fi.R tl = t2
So the new area of shadow is ~ µ1L1 = µ2L2 ... (3)

= 1.32

www.puucho.com
Rahul Sardana with www.puucho.com

Ray Optics: Refraction at Plane Surfaces

Since, Optical Path Length (OPL) is the distance travelled d 10--


by light in vacuum/air/rarer medium during the same time and t, = v: = (3x10') 6x10-15 s
it travels a distance L2 in medium. So, from (3), we get
1.8
µlLl =µairLair = µmediumLmedium =µ2L2 So, tmin = 2 X 10- 15
s
Since, µair =1 , so, we get
(b) The total number of wavelengths in a film of refractive
OPL = Lrur = µmediumLinedium index µ , thickness d is
{for same time in air and medium} n = Optical Path Length
Since light always travels slower in denser medium, s6 the Wavelength of Light
OPL (the distance in air corresponding to same time in both) µd
is always longer than the actual thickness L of the medium. ~ n=-
'/,.
So, total number of wavelengths, is
CONCEPTUAL NOTE(S)
Also, note that µ 1L1 is OPL in air/rarer medium and µ 2 L2 is OPL n = µ,d, + µ,d, + µ,d,
'/,. 1,, '/,.
in denser medium. However for standard purposes OPL is the
distance travelled by light in vacuum/air to travel a distance L in 1
_ a medium during the same time in either air or medium. ______ _
~ n=i:(µ 1d1 +µ 2 d2 +µ 3 d3 )

10--
So, from equation (3), we conclude that for a pair of media, ~ n= xlO_, ((1.2)(1)+ (l)(l.5)+(1.8)(1))
600
Optical Path Length ) = (Optical Path Length) n = 1000 (4.5) = 4500 = 75
( in Air/Rarer Medium in Denser Medium ~
600 600
=> µ1 L1 =µ2L2
Illustration 21
~ L2- -µ,- L1
µ, A light ray enters the atmosphere of a planet and
descends vertically 20 km to the surface. The index of
Since µ 1 < µ 2 , so we get
refraction where the light enters the atmosphere is 1 and it
L2 < L1 increases linearly to the surface where it has a value 1.005.
How long does it take the ray to traverse this path.
Due to this reason, L2 is also called the Reduced Thickness.
Solution
So, in general, we get
Since variation is linear, so we have
R~duced) = ( µra=
( Thickness
)L,=, = OPL in air x-0 2xl0 4 -0
µdenser µdenser --=
µ-1 1.005-1
0.005x
Illustration 20 ~
µ= 1 + 2xl0 4
A light beam of wavelength 600 nm in air passes
Now, by definition, we have
firstly through film 1 of thickness 1 µm and refractive
C
index n1 =1.2 , then through an air film 2 of thickness µ(x) = v(x)
1.5 µm and finally through film 3 of thickness 1 µm and
refractive index n3 = 1.8 . ~ v(x) = µ[x)
(a) Which film does the light cross in the least time and
what is that least time? where µ(x)=µ=l+ O.OOS: and c=3xl0 8 rns-1
(b) Calculate the total number of wavelengths (at any 2x10
instant) across all three films together. X=O, µ= 1
Solution
d, 10-'
4x10-15 s x,µ
i
(a) Since, 11
v, (3xl08)
1.2
d, 1.5x10-'
Similarly, f2 2x10-15 s X, = 2 X 10' m, µ= 1.005
C 3xl08

==================================== 1.33 =
www.puucho.com
Rahul Sardana with www.puucho.com

Advanced JEE Physics Optics & Modern Physics

C 3xl0 8 3xl08 X
~ini and d-x sini
=> v(x) = µ(x) 0.005
1 + - -4x Jb 2 +(d-x)'
2xl0
1 .. 1 .
dx 3xl08 => -smz=-smr
C V
=>
di 1+2.5x10-7 x
sini C
1 => -=-=µ [The Law of Refraction}
=> di = --, [(1+2.5x10-'x)dx] sinr v
3xl0
t 1 2><104 VECTOR FORM OF SNELL'S LAW
fdl=--, f (1+2.5x10-'x)dx
0
3x 10 0 µ 1 sini =µ 2 sinr
4 Using our knowledge of cross product of vectors, we have
=> 1 =-1-[ 2 xl0' + (2xl0 )' (2.5x10-')]
ixfi=(l)(l)sini, 0 outwards
3xl08 2
5
=> 1=6.68x10- s
i
LAWS OF REFRACTION USING FERMAT'S PRINCIPLE
Medium 1 (µ 1)
Consider a refracting surface / interface separating Medium (µ,)
medium 1 from medium 2. Let the incident light start from
A , in medium 1, hit the surface at O and get refracted to a
point B, in medium 2. Let the points A and B be at A
-n
'
perpendicular distances a and b_from the interface. Further,
let A and B be at a separation d as shown in figure. The and -ilxf=rxfi=(l)(l)sinr, 0 outwards
time taken by the light to go from A to O to B is So, from above we conclude that
t=tA...,.o+to-)oa
µ 1 (1 X fi) = µ 2 (f X fl)

=>

Ja' +x' Jb' +(d-x)'


=> I---+~----
C V

Now, according to Fermat's Theorem, t is MINIMUM, so

..'!!_=0 In Medium 1 In Medium2


dx
REFRACTION THROUGH A COMPOSITE SLAB
' Consider the refraction of light ray through a series of media
''
.',o (d-x) Medium 1 (vacuum) as·shown in figure. The ray AB is incident on interface X1Y1
Interface X 1 Medium2 at an angle i . The ray is deviated in medium 2 along BC
'r ' towards the normal. Then it falls on interface X2Y2 and is
'' :' b
r' ! again deviated towards normal along CD. If the last
medium is again Medium 1, the ray emerges parallel to the
B incident ray. Let r1 and r2 be angles of refraction in Medium
I+---- d ---+I
2 and Medium 3 respectively. Then from Snell's Law,
sini µ 1
J:..i( Ja' +x' )+J:..i( Jb' +(d-x)') - . - = -2= µ2 ... (1)
=> smr1 µ 1
C di V di
1 2x 1 ( 2(d-x)(-1)) 0 sinr µ 2
=> - . -1= -3= µ3 ... (2)
2c Ja' +x' + 2v Jb +(d-x)' 2 = smr2 µ2
sinr2 µ 1 3
1 X 1 (d-x) - .-. =-= µ1 ... (3)
=> Slill µ3
cJa'+x' vJb'+(d-x)'
µ 1 = refractive index of medium 1
From the figure, we observe that
µ2 = refractive index of medium 2
µ3 = refractive index of medium 3
·= ====================================
1.34

www.puucho.com
Rahul Sardana with www.puucho.com

Ray Optics: Refraction at Plane Sn,faces

A Solution

1 Given, wµ 8 =2. and aµ,=~


X, f Y, 8 2
t.
' X,
3
t Y,
~
~
X, I Y,

Multiplying (1), (2) and (3), we get


1µ2x2µ3x3µ1 =1

=> tµ2x2µ3=1µ3 LATERAL SHIFT ON PASSING THROUGH A GLASS


SLAB
In general if a ray passes through a number of conposite
parallel plate glass slabs, then Consider a ray AO incident on the slab at an angle of
incidence i through the glass slab EFGH of thickness t .
1 µ2 X 2µ3 X 3µ4 X 4µ5 = 1µ5
After refraction the ray emerges parallel to the incident ray.
Let PQ be perpendicular dropped from P on incident ray
Illustration 22 produced as OQ .
A light beam passes from a parallel plate glass slab of The lateral displacement caused by plate,
refractive index µ 1 placed·in a medium of refractive index
x = PQ = OPsin(i-r)
µ 1 • Show that the emerging beam is parallel to the
incident beam. OM . (·1-r)
x=--sm /._-OP= OMl
Solution
COST l cosr

Applying Snell's Law at A, we get ~


t . (.i-r )
x=--sm
COST
µ 1 sini =µ 2 sinr1
~
t (smzcosr-cosismr
x=-- . . .. )
~
1:!.= sinr1 ... (1) COST
µ2 sini
~ x = t(sini- cositanr)
A

Air(µ= 1)

µ, µ,

Applying Snell's Law at B, we get


µ 2 sinr2 = µ 1 sine ,e
Air(µ= 1)
''
=> 1:!. = sinr2 ... (2)
'
N' B
µ2 sine '
. sini
From equation (1) and (2), we get Smce µ=-.-
smr
i=e
. sini
i.e., the emergent ray is parallel to incident ray. => smr=--
µ
sini
Illustration 23 ::::::> tanr
Refractive index of glass with respect to water is 9/8 .
Refractive index of glass with respect to air is 3/2 . Find ~

the refractive index of water with respect to air.


1.35 =
www.puucho.com
Rahul Sardana with www.puucho.com

Advanced JEE Physics Optics & Modern Physics

If i is very small then r is also very smalt hence, b) At near normal incidence (small" angle of incidence~ i )'
sin i ---+ i , and cos i ---+ 1, apparent depth (d') is given by

Then expression for lateral displacement takes the form. Observer -;;;11.
Medium 'I
f
:=n(1-¾) RARER
I

RARER DENSER '~·


APPARENT DEPTH
DENSER
An object O placed in a medium of 'refractive index µ is
observed from air at a small angle a to the normal to the
Observer 4
Medium
interface (in figure, angle a is shown exaggerated for
clarity) i.e., for near normal incidence. ' d - iV
d' =- - and v' =-·--

~
µrerawu µrolaUvo

'' .,' , RI of medium of incidence obje_qt


'' ' where µ ra· =
B "'
'A Air ro live RI of medium of refraction observer
'' ,, ' Medium(µ)
''
d fo·•"~,'
d' P:
'
'''
d =distance of abject from the' interface = real depth.~

l 6X

io '
',p !'
d' = distance of image from the interface
depth. - · - ,..
= apparent
v :;= v13locity of object perper:idicular to interface, relative to
··,

surface.
H the object O is at a real depth d from the interface, its v: = ve1acity at iffiage Perp~ndicu1ar
ta interface relative ta
apparent depth d' can be calculated. From t,.s ABO and surta'c8.' -- · \\, · ..
ABO',
tancx d Illustration 24
tanp d' A fish rising vertically to the surface of water in a lake
Since angles a. and j3 are small, so sin a. ~ tan a. and at a uniform speed of 3 ms-1 • It observes that a bird
sinp "tanp. diving vertically towards the water at a uniform speed of
Therefore, from Snell's Law, we get, 9 ms-1 • If the refractive index of water is _!, find the
3
µ=-=--,S--=-
smt sina. tana. d
sinr sinP tanp d'
actual speed of dive of the bird.
Solution
d
=:, Apparent depth, d' = - Let x be the depth of the fish F below the surface of
µ
water, and y be the height of the bird B above the surface
The apparent shift in normal direction (or the normal shift) at an instant.
in the position of the object is
To the fish, the bird will appear to be farther away, at
dX= d-d' =d(1-¾) an apparent height y' given by
y
In case the object is seen through n number of slabs with
different refractive indices, the total apparent shift is simply ( µ""'.'m•H•m.)
µfishmedium
the sum of individual shifts, so
Ax=Ax1 +Ax2 +&3 + .... +filn =:, ' y
y = (;) =µy
=,, ax=ai(1- :J~a,(1-:J+a,(i-:J+ . +a"(i- :J The total apparent distance of the bird from the fish is
s=x+y'
. CONCEPTUAL NOTE(S)
=:> s=x+µy
a) If the medium in which the object is placed is rarer (µ,,< 1')
and it is seen from the denser medium, the apparent shift Differ~ntiating w.r.t. time t, we get
calculated will be negative. If means· that the object C

apparently shifts away from the observer. ds dx dy


If the shift comes out to be ,positive, the ima9e of the·
dt ~ dt + µ dt
object shifts towards the observer.

= 1.36

www.puucho.com
Rahul Sardana with www.puucho.com

Ray Optics: Refraction at Plane Surfaces


•B' SHIFT OF POINT OF CONVERGENCE OR DIVERGENCE

!f
y
•B
rs
Air
If a glass slab of thickness t., refractive index µ is placed in
the path of a convergent (or divergent) beam of light, the
point of convergence (or divergence) gets shifted by

X
Water
~={1-¾)
i •F
µ

Substituting the values, we get

9=3+(±) dy
3 dt
1+-t-+1 i+-t ....
Therefore, the actual speed of dive of the bird is given by
(A) Convergent beam (B) Divergent beam
dy =(9-3) (~) = 4.5 ms-'
dt 4 ,.
Illustration 26
A point object O is placed in front of a concave
Illustration 25 mirror of focal length 10 cm . A glass slab of refractive
A vessel is filled with a non-homogeneous liquid
whose refractive index varies with the depth y from the index µ = ¾and thickness 6-cm is inserted between-object
free ·surface of liquid as µ·= ( 1 +;} Calculate the
and. mirror, Find the position of final image when the
distance x shown in figure is
apparent depth as·seen by an observer from above, if H is (a) 5 cm
the height to which the liquid is filled in the vessel. (b) 20cm
Solution 6cm
Let us consider a thin layer of liquid of thickness dy at
a distance y below the free surface of liquid. The apparent

depth of this layer having real depth dy is dH' = dy .



0

Free Surface
µ

-x-
1-4---32 cm-+i

r
H
y Solution
The normal shift produced by a glass slab is given by

l ~=(1-¾}=(1-¾}(6)=2 cm

i.e., for the mirror the object is placed at a distance


=> dH;= dy (32- ~) = 30 cm from it. Applying mirror formula i.e.
µ
1 1 1
-+-=-, we get
=> dH'= dy V U f
.' ' ·( ' H
y)
, 1+- 1 1 1
---=--
v 30 10
ToW _apparent depth is obtained. by integrO:ting this
expreSsion within appiopriate ~imifs. So, => v=-15 cm
H (a) When x =5 cm
Hlog,(H +y) The light falls on the slab after being reflected from the
mirror as shown. But the slab will again shift it by a
distance tu= 2 cm . Hence, 'the final real image is
H'=Hlog,2 formed at a distance (15 + 2) = 17 cm from the mirror.

================================== = 1.37

www.puucho.com
Rahul Sardana with www.puucho.com

Advanced JEE Physics Optics & Modern Physics

4 sin(45°)
Further,
3 sinr
=:> r=32°

Now EF
GE
=tanr =tan(32°)
(b) When x = cm 20 h-lO =0.62
h
This time too the final image is at a distance 17 cm from
the mirror but it i~ virtual as shown. Solving this, we get
h=26.65 cm
MULTISLABS
If a number of slabs (or immiscible liquids) of depth d1 , d,,
d3 , •••• and refractive index µ 1 , µ 2 , µ 3 , •••• are placed one
over the other, the real depth is
d=d,+d,+d,+ ....
Illustration 27 The apparent depth is given as
A cubical vessel with non-transparent walls is so d' =E.!..+E1_+ d3 + ...
located that the eye of an observer does not see its bottom µ1 µ2 µ3
but sees all of the wall CD. To what height should water Therefore, for the combination, the effective µ is
be poured into the vessel for the observer to see an object
F arranged at a distance of b =10 cm from comer D ? The µ=I d,+~+~+... L~
face of the vessel is a = 40 cm and refractive index of water
.
IS-,
4
d' (i:)+(i:)+(i:)+... r(iJ
3 If there are only two slabs, of equal thickness, d, =d, =d,
d+d

d, --.-
f,,-===~==='~ __ t_
!;}}jit;tir.1~!f~
d2
--+·
Solution µJ d3
1 . __ _ _ _ _ _.J. __ t_
Since, the vessel is cubical, LGDE=45° and
GE=ED=h (say)then EF=ED-FD
Illustration 28
Eye~A
The bottom of a tub has a black spot. A glass slab of
thickness 4.5 cm is placed over it and then water is filled
to the height of 8 cm above the glass slab. Looking from
top, what shall be the apparent depth of the spot below the
water surface? Also find the effective refractive index of
the combination of glass slab and water layer. (Refractive
index of glass is ~ and of water is ± ).
2 3
Solution
But tan(45')=1=- GE
ED
The apparent depth is given as
d d 4.5 8
cc,, ED=GE=h d,=µ:+µ:=rn)+(¾)=3+6=9cm
cc,, EF=ED-FD=h-10

= :::::=================================
1.38

www.puucho.com
Rahul Sardana with www.puucho.com

Ray Optics: Refraction at Plane Surfaces

I 6. A point source of light is arranged at a height h above the


surface of water. Where will the image of this source in the

!'"
I. 4 flat mirror-like bottom of a vessel be if the depth of the
µ2=3
vessel full of water is d ? Refractive index of water is

,. 3
µ1=2 •· 4.5cm n =~ . co·nsider only two steps.

\
Spot 7. A plate with plane parallel faces having refractive index 1.8
rests on a plane mirror. A light ray is incident on the upper
The effective refractive index is given as
face of' the plate at 60° . How far from the entry point will
Real Depth the ray ·emerge after reflection by the mirror of the plate is
µeffective
Apparent Depth 6 cm thick?

=> µclfuctire = !!,_ = d, + d, = 4.5 + 8 = 12.5 = 1 _39 60~1''' '''


d, d, 9 9 ,M N:'
IC:Eilli
BASED ON GENERAL REFRACTION
(Solutions on page 1•.176)
1. An object lies 100 cm inside water. It is viewed from air MirrOr
nearly normally. Find the apparent depth of the object.
8. A pole 4 m high is driven into the bottom of a lake and
2. A concave mirror is placed Inside water with its shining happens to be 1 m above the water. Determine the length
surface upwards and principal axis vertical as shown. Rays of the shadow of the pole at the bottom of the lake if the
are jncident parallel to the principal axis of concave mirror. sunrays make an angle of 45° with the Water surface. The
Find the position of final image.
~.
!! Air
refractive index of water is

water
4/3
30cm
f 9. A ray of light falls onto a plane-parallel glass plate 1 cm
thick at an angle of 60° . The refractive index of the glass Is
./3 ,
~
~--
R=40cm
! Some of the light is reflected and the rest, being
refracted,, passes into the glass Is reflected from the bottom
of the plate, refracted a ~econd time and emerges back into
the air parallel to the first reflected ray. Determine the
djstance £ between the rays .

3. A small object Is placed on the principal axis of a concave


.
10. A ray of light ·is refracted through a sphere whose ·material
spherical mirror of radius 20 cm at a distance of 30 cm . has a refractive index µ in such a way that it passes
By how .much will the position and size of the image alter,
through the extremities of two radil which make an angle .~
when a parallel-sided slab of glass of thickness 6 cm and
with each other. prove that if a;. is the deviation Of the ray
refractive index 1.5 is Introduced between the centre of
curvature and the object? The parallel sides are caused by its passage through the sphere,
perpendicular to the principal axis.
cos(~;")= µcos(%}

4. The velocity of light in air is 3 x 108 ms-1 • If yellow light of


wavelength 6000 A is passed-from air to glass of refractive 11. A vertical beam of light of cross-sectional radius r is
index 1.5, determine the velocity, the wavelength and the incident symmetrically on the .curved surface of a glass
colour .of light in glass.
hemisphere (µ =%) of radius 2r placed with its base on a
5. A 2 cm thick layer of water covers a 3 cm thick glass slab. horizontal table. Find the radius of the luminous spot formed
A coin Is placed at the bottom of the slab and is being on the table.
observed from the air side along the normal to the surface.
Find the apparent PC?Sition· of the coin from the surface.
12. A material having an index of refraction µ is surrounded by
Air
vacuum and i_s in the Shape of a quarter circle of radius R .
f
2cm
A light ray parallel to the base of the material is incident
h, Water from the left at a distance of 'L above the base .and

f
3cm h, Glass
emerges out of the material at an angle 8 • Determine an
expression for a .

t Coin

================================== 1.39 =
www.puucho.com
Rahul Sardana with www.puucho.com

Advanced JEE Physics Optics & Modern Physics

CRITICAL ANGLE
According to Snell's Law, we have
sini d
-.-= µ,
smr
sin C = µrarer
j 13. A circular disc of diameter d lies horizontally in~ide a sin 90 µdensl'r

metallic hemispherical bowl of radius a. The disC. iS just


· C =µrarer 1 --
I visible to an eye looking over the .edge. The bowl is now
filled with a liquid of refractive ·index µ. Now, the whole of
sm --=-
µderner µdenser

the c;lisc is ju$t visible to the eye· in the same positioli. Show
that , ·
d=2a(µ'- 1
µ2 +1
)
where µdenser is ~he refractive index of the denser medium

V-
Eye ............ _ _ _ _ _ _ _ _ _ __
w.r.t. the rarer medium. The lesser the value of µdenser, the

--, ---- .......... _ greater is the critical angle C .


For a given pair of media, since µ depends on the
.............. _ wavelength of light the critical angle ·also depends on the
........... wavelength. The greater the wavelength, the greater will be
the critical angle.

Critical angle
TOTAL INTERNAL REFLECTION (TIR) Media Pili .
C =sin -1(-1-..· ·,)
When a ray of light goes from a denser to a rarer medium, it µdense'(
bends away from the normal. If the angle of incidence in the
µd· _µ._4(3_4
_____ _
denser medium is increased the angle of refraction in the Water-Air 49°
rarer medium also increases. At a particular angle of µ, 1 3
incidence in the denser medium (called as the Critical angle
C ), the angle of refraction in the rarer medium is 90° (i.e., I
I
Glass-Air
µ,
µd=-=-=-
µ,
3/2 3.
1- · 2
the refracted ray grazes the interface). This angle of
refraction in the denser medium for which the refracted ray Glass-Water
grazes the interface is called the critical angle for the pair
63°
of interface.
'
Please note, that for small angles of incidence, both reflection EXAMPLES OF TOTAL INTERNAL REFLECTION
and refraction occur, however we shall be neglecting the
a) Mirage : Mirage is an optical illusion ol;,served in
reflection at the interface as most of the light is refracted.
deserts and roads on a hot day. When the ai_;- near the
However, when i > C, no part of light is refracted· and the
entire light is reflected back to the denser medium itself. This
ground is hotter
(and hence rarer)
<:l1,,-_ ______ o
phenomenon is called total internal reflection (TIR) and was
than the air above, E '........ Denser
first noted by Kepler in 1604. j > 8~
there occurs a
continuous decrease Rarer
of refractive index
of air towards the '
'Earth
ground. A ray of
light from a point
0 of a tree is,
therefore, refracted more and more away from the
Denser µ2
normal. Ultimately it gets totally reflected to reach the
µz:> µ1
0 eye E. To tf1e observer it appeai;-s to come from I,
which is the image of O . This image gives the
Images formed by TIR are much brighter than those formed impression of reflection from a pond of water.
by the mirrors (or lenses). Some loss of intensity always
b) Looming : Similarly, in extremely cold regions (near
takes place, when light is reflected from a mirror (or
refracted through a lens). polar regions), the refractive index decreases ·with
height. Due to TIR (shown in figure), the image of a hut
appears hanging in the air. This is called looming.

= 1.40 ==================================
www.puucho.com
Rahul Sardana with www.puucho.com

Ray Optics: Refraction at Plane Surfaces

rm
' '
~-------~.._,...... \:,.,-/
Rarer ..........................
! goo
~--·r---
' 1-:c:c:__4r
''
d ''
Earth-

c) The µ of diamond is 2.5, for which C is only 24°.


Diamonds are cut such that _i > C ; so TIR takes place
L-~-------l
Fish in Glass Tank
µ

again and again inside it The light coming out from Similarly, if a source of light is kept in ·a pond, its light will
few meticulously cut surfaces makes it sparkle.
come out only through a circular region. For any incident
d) Air bubbles in water shine due to TIR angle i greater than C, the light will be totally reflected
e) The working of an optical fibre is due to multiple TIR back into the water, making corresponding region on the
inside it. surface of water appear dark.
f) Porro prisms used in periscopes or binoculars bend the
ray due to TIR. Some examples are shown in figure.
Illustration 29
Light is incident making an angle 0 with the axis of a
45"

/
transparent cylindrical fiber of refractive index. n = ~ as
i =45°
/ shown in figure. Determine the maximum value of 8 so
90' that the light entering the cylinder does not c~me out of. the
45' curved surface. ·

(a) Bending of rays by 180' (b) Bending of rays by 90'

t)-----~ - -(-)-
Solution
The ray of light is incident at A and it just gets
A' --+--<f--<-r 90' reflected totally at B . Therefore incident angle at B is equal
B'fr_.,__+-_
to the critical angle give~ as C = sin ~l ( ; )

(c) Erecting of image

FIELD OF VISION OF A FISH


A fish inside a pond does not see the outside world through
the entire surface of water. The light from outside can reach
the fish only through a circular patch, which forms a cone of
half angle equal to the critical angle.
If r is the radius of the circular patch, d is the depth of the Snell's Law ofrefraction at A gives
fish and µ is the refractive index of water, then
sin0
--=n
r=dtanC=dsinC =d sinC sinr
cosC .Jl-sin2 C . sin0
::::::,. smr=-- ... (1)
. . C =-
1 n
Smce, sm
µ Since r+C=90°
d ~ sinr=sin(90°-C)=cosC
~ r= ~µ'-1 For a ray not to come through the curved surface,
r,;90-C

==================================· 1.41 =
www.puucho.com
Rahul Sardana with www.puucho.com

Advanced]EE Physics Optics & Modern Physics

=> sinr5.Jl-sin C
2
5'11-: 2 ... (2)
=>
=:,
µ2>1+1
µ > Jz
( ·: maximum value of 0 can be 90°}

Eliminating sin r from (1) and (2), we get So, the minimum value of refractive index is
sin0:<;)l-~ µml" =Fz
n n
=:, sine :5. .Jn' -1 Illustration 31
=:, sin2 0,; 1.25-1 Light is incident at an angle a. on one planar end of a
=:, sin2 0:5.0.25 ' transparent cylindrical rod of refractive index n .
Determine the least value of n so that the light entering
=:, . 0 5-
sm 1
the rod does not emerge from the curved surface of the rod
2
irrespective of the value of a ..
=:, 0:<;30°
=:, 0max = 3QO

Illustration 30
Solution
A rectangular block of glass is placed on a printed
page lying on a horizontal surface. Find the minimum .
Smce, . C =-
sin 1
value of the refractive index of glass for which the letters n
on the page are not visible from any of the vertical faces of In triangle ABN, r' + r + 90° = 180°
the block.
Solution
:s
Light will not emerge out from the vertical face BC,
when
De-------, C
Glass
-~()
=:, r'=90-r
=:, (r')mm = 90° -(r )=
sin(i)max sin90° (i = 90')
and n
sin(rl- sin(r)mu max

Then, sin(r)max =.!.=sinC


n
(r)=,=C
i > Critical Angle (C)
=> sini > sinC =:, (r')m1n = (90° -C)

. . 1 Now, if minimum value of r' i.e., 90° -Sc is greater than ec,
=> smz>- {·: sinC=;}
µ then obviously all values of r' will be greater than 8c i.e.,
Applying Snell's Law at O, we get total internal reflection will take place at face AB in all
conditions. Therefore, the necessary condition is
1sin8=µsinr
(r')- ?:C
=:, sin0=µsin(90°-i)=µcosi
=:, (90°-C)?:C
. sin9
cosz=-- =:, sin(90°-C) ?:sinC
µ
=:, cotC ~ sinC
sini=.Jl-cos 2 i =~1- sin'O =:, cosC ~ 1
µ'
Therefore, the condition for no light to emerge from vertical
=:, .Jn'-1 ?:l
face BC becomes, =:, "2 ~2
1
=:, n?:Jz
>-
µ Theref9re, minimum value of n is J2
2 2
=> µ >l+sin 8

= ==================================
1.42

www.puucho.com
Rahul Sardana with www.puucho.com

Ray Optics: Refraction at Plane Su,faces

Illustration 32 Solution
A point source of light is placed at a distance h below Let the critical angles at 1 and 2 be C1 and C2
the surface of a large and deep lake. Show that the fraction respectively. Then
f of light that escapes directly from water surface is ~~-----~~
µ, =-,/2
;
i I
independent of h and is given by, f '
! T
2
p
µ2=2 i I
Solution
Due to TIR, light will be reflected back into the water
µ,=../3 2

for i > C . So only that portion of incident light will escape


which passes through the cone of angle 8 = 2C . C_· -1(b_)- · -,(j_)-45°
1 -Sll1
µ,
-Slfl In
..,2
-

C
A 1 1
and C2 =sin- (~:)=sin- ( ~)=60°
h
f ForTIR, i>C2
s Therefore, minim.urn angle of incidence, for total internal
reflection to take place on both slabs must be 60° .
+ irnin =60°

So, the fraction of light escaping is given by


f = Area of Surface ACB BASED ON TOTAL INTERNAL REFLECTION {TIR)
Total Area of Sphere (Solutions on page 1.179)
1. Light refracts from medium 1 into a thin layer of medium 2,
~ f= 21tR2 (1-cosC) 1-cosC crosses that layer and then is incident at the critical angle
4nR2 2 on the interface between media-2 and 3 as shown in figure.
Now, as f depends .on C, which depends only on µ, hence
f is independent of h .
Since, we know that
. C =-
SIIl
1
µ

cosC= ~µ'-l
" ' µ'
=JI- µ'1 C

(a) Find. the angle e .


1-J1- l (b) If 8 is decreased, wiffthe l!ght be refracted to medium
µ'
!=~~~ 3?
2
2. A ray of liQht enters into a glass slab from air as shown in
figure. If refractive of glass slab varies with t, the thickness
Illustration 33 ' of the Slab measured from the top as µ =A - Bt where' A
AB and CD are two slabs. The medium between the
and B are constants. Find the maximum depth travelled by
slabs has refractive index 2. Find the minimum angle of ray in the stab. Assume thickness of slab to be sufficiently
incidence of Q , so that the ray is totally reflected by both large.
the slabs.
A a Air

Slab
p
C

================================== 1.43 =
www.puucho.com
Rahul Sardana with www.puucho.com

Advanced JEE Physics Optics & Modern Physics

3. A container contains water suj)to a height of 20 cm ,and·. AIR


there is a point Source of light at the benti'e of the .bott6m-of
the cohtainer. A rubber ring of. radius ~ floats centr'ally On
the water. The ceiling of the.l'oo·m is 2.m· above the Water
surface. ',, "
(a) Find the radius of the shadow of the ring 'formed, on
theceilingif a=15cm. "·· ' "· = 1.3
C
(b) Find the maximum iJalue of 8. for which the shadOw of n2·=·1.4
the ring iS fonrted on the cejflng.
n3 = "1.32
Refr,active index,of w9-;er =~--·
n..,=1.45

4. ABCD· is,the plane of glasS'cube of ref~ctive in~ex µ. A"


horizOnfal beam of light enters th_e·face AB at the graiing
incidence.
(a) Show that the angle e which any ray emerging from'
BC would make with noml_al to BC is given b·y
sine= cot a, -~here a 'is the.critical 8ngle. ·

))---------' ----- )----


1 o. A point source of light s is placed at the bottom of a vessel
o,._______, C
containing a. liquid- of refracti~e i~dex ~ . A persOn is
(b) What Is the greatest valuidhat the·refractlon index of
" 3
glass may have if any oFthe .light is io emerge from viewirig the source from, above 'the surface. There i_s an·
.BC? .. ,
opaque diSc Of' radius 1 cm fl08ting on the surfaC:e. The,
center of the disc lieS verticially. above the.source S. The'
5. Plot the deviation (S) versus the angle of incidence (I) liquid from. the vessel is gradually _Pralned out through a tap.
graph for a ray travelling frorii c1enser tO rarer medium. What Is the maximum height of the liquid for which th~
source Cann_of at all seen from· above.
6. In figUre,. light refracts into material 2, crosses that material
and is. then incident at the critical angle on the interface 11. A rectangular glass block is placed on top of a sheet ,of_
between materials 2 and 3. ,i · paper on .which there is a small cross. When the paper is
, MEDIUM 1 Soaked in alcohol and a' sodium. lamp is placed opposite to
one vertical-of the block the cross can be·seen through th_e
01=1.6 opposite vertical face Up to a. ·priint where the angle qf
emergence of the light is 30° . If the refractive index of the .
glass fs 1.5, find the refractive index of alcohol. Why can'.t
the black cross be seen through the-face when the paper Is
MEDIUM :i dry. ' ,,
n3 = ·1.:r
12. Rays·oflighlfall on the plane surlace of a half cylinder at an
angle 45°, in -the plane perpe_ndicular to the. axis (sf2!eM·
(a) What is angle e? figure). Refractive index of glass. is ./2..
Dis~uSs ·, fh~"
(b), , If 0' is increased, is there refraction of light into condition-that the rays do flat suffer tot8.! intemal'reflection.
·, .. " • -+·
material 3~

7. An isotropic point source is placed at a depth h· below tt}e ·


water surface. An opaque disc 9apable of floating on water
surface,.1s ·placed on th~ surface of water so that the bµlb is
not visible from the slirface. Find the minimum radius of the
disc fol° the bulb not to be viSible. Take ;refractive index of
water=µ.,

8. In figure, light begins from medium of- refractive ind8.x , 13. Find the maximum value of -k·, so that the beam of_ light·
n1 = 1.3', undergoes three refractions as it heads downward
inci_c;fenl normally .at the face A of a U shaped glaSs tube
and a reflection and then a refraction· to reach the air. The ~-~em=erges th!Q.~Qh B as shown- iri the figure. The refractive·
initial angle 01 = 30° . Find the_ value:,s of the angles

= 1.

www.puucho.com
Rahul Sardana with www.puucho.com

Ray Optics: Refraction at Plane Snrfaces

between incident ray EF and emergent ray GH (produced


index of glass is µ = ¾.
backw~ds) is called angle of de':'iation D .
A

E H
B<-µ_ _ _ _ _ _ ___,C
8
A
In triangle OFG ,
PRISM D =(i-r1 ) + (e-r,)
Prism is a traJl'iparent medium bounded by any number of ~ D=(i+e)-(r, +r,) ' ... (1)
surfaces in such a way that the surface on which light is
Also in quadrilateral AFNG ,
incident and the surface from which light emerges are plane
and non-parallel. A+ 90° + 0+90' = 360"
Refracting angle of prism, or simply the angle of prism is ~ A+0=180' ... (2)
the angle between the faces on which light is incident and And in triangle FGN ,
from which light emerges. In all the prisms shown in figure r1 +r2 +8=180° ... (3)
above, angle A is the angle of prism. Comparing equations (2) and (3), we get
Angle of deviation (D) is the angle between the incident A=r1 +r2 ... (4)
ray and the emergent ray. Sometimes the angle of deviation From (1 ), we get
is also denoted by 6 . D=i+e-A
~ i+e=A+D ... (5)
If µ is the refractive index of material of prism, then from
Snell's I.aw
sini sine
µ=--=-- ... (6)
sinrt sinr2
For a prism with small refracting angle, we have
D=(µ-l)A

CONCEPTUAL NOTE(S)
a) Angle of deviation (D) means the angle between
emergent and incident rays i.e., the angle through which
incident ray turns in passing through a prism. It ls
represented by D and is shown in figure.
A

Please note that, for a glass-slab, the angle of prism is zero,


and the incident ray emerges parallel to itself, i.e., there is no
deviation. If µ of t!J.e prism material is same as that of its
surroundings, no refraction takes place and light passes
through undeviated.

REFRACTION THROUGH A PRISM A


Consider a monochromatic ray EF to be incident on the face
AB of prism ABC of refracting angle A at angle of
incidence i . The ray is refracted along FG , r1 being angle of
refraction. The ray FG is incident on the face AC at angle of
incidence r2 and is refracted in air along GH . Thus GH is
the emergent ray and e is the angle of emergence. The angle - - - _____ B c_______ ,

================================== 1.45 =
www.puucho.com
Rahul Sardana with www.puucho.com

Advanced JEE Physics Optics & Modern Physics

b) If the faces of a prism on which light is incident and· from Thus, a ray of light will not emerge out of a prism (whatever
which it emerges becomes parallel (as in figure), angle of
prism will be zero and as incident ray will e·merge parallel be the angle of incidence) if A> 2C , that is, if µ > cosec(;).
to itself, deviation will also be zero i.e., the prism will act as
a slab. '
c) If µ of the material of the prism becomes equal to that of CONDITION FOR GRAZING EMERGENCE
surroundings, no refraction at its faces will take place and
light will pass through it undeviated. So, deviation is zero.
A ray can enter a prism in such a way that the angle of
i.e., D=O
emergence, e = 90° , as shown in the figure.
---------- --- ------ ----
A
CONDITION OF NO EMERGENCE
The light entering the prism at surface AB , will not be able
to come out from the surface AC, if TIR takes place at this
surface. For any angle of incidence, this condition will be
satisfied, provided we have at surface AC,
(r,)mm > C
Since, r1 + r2 = A
We can determine the angle of incidence i for such grazing
~ '2 =A-r1 emergence. We should have
~ (r,)""' = A-(r1 ) - ... (1) '2 =C
A Since, for a prism, r1 + r2 = A
=> r1 =A-r2 =A-C
' ',, p r, Using Snell's Law,
i r,
lsini=µsinr1 =µsin(A-C)
~ sini = µ(sinAcosC-cosAsinC)

B ~ sini = µ[(sinA),/1-sin 2 C -(cosA)(sinC)]

But (r1 )max is possible when i=imax =90° i.e., incident ray
grazes the interface AB .
Now, applying Snell's Law at AB, => sini=sinAJµ 2 -l ~cosA
lxsini=µsinr1
=> i=sin-1 (sinAJµ 2 -1-cosA)
~ sin(90°) = µsinr 1 The light will emerge out of the. prism only if the angle of
~ r1 =sm . -1(1)
·µ
incidence i is greater than the above value.

MAXIMUM DEVIATION
~ '1 =C ... (2)
From equations (1) and (2), we get The angle of deviation D is maximum when the angle is
r1 +r2 >2C maximum, i.e., i = 90°.
... (3)
Dmu = (i + e)-A = (90°+ e)-A
Since, r1 + r2 = _A
Therefore, the condition becomes Under such conditions of grazing incidence, r1 = C
A>2C A

~ sm(1)>sinC

~ . (ZA) >µ1
sm

~ µ > cosec( 1J B.'------------' C


And at the. second surface,

= 1.46 ================================:::i
www.puucho.com
Rahul Sardana with www.puucho.com

Ray OpHcs: RefracHon at Plane Surfaces

µsinr2 = lsine emergent ray for the same incident ray, indicating the
values of au the angles.
=> sine= µsinr2
A
Since r1 +r2 =A .,
~ sine=µsin(A-r,)=µsin(A-C)
,,,,,/
~ e = sin-' (µsin(A-C))
--+----i-----,;cr.'"
/
Illustration 34
An isosceles glass prism has one of its faces coated 90' .45'
. with silver. A ray of light is incident normally on the other C B
face (which is equal to the silvered face). The ray of light is
reflected twice on the same sized faces and then emerges Solution
through the base of the prism perpendicularly. Find angles For total internal reflection to take place at surface AB,
of prism. we have
Solution i>C
As the ray is incident normally at the face AB , so => sini>sinC
'1 =0 .
Smce, . C =-
sm 1
µ

~ sin45'>(t)

E ~ µ>Ji
~ µmin =Ji
When the prism is immersed in water, the boundary AB now
separates glass from water.
B 0 C

Since, we know that r1 + r2 = A , so we get


r2 =A=180'-20 ... (1)
Now, LDfE=180°-90°-2r2
~ LDfE=180°-90°-360°+40 {·: r2 =180-20)
~ LDFE=40-270' ... (2)
Since, r, = 90° - LDFE ... (3)
~ -- c =sm
· -•(. µ,=, · -1 (Ji)
· - -) =Sm --
r,=360'-40 µ,_ 1.33
Again LBFG = 90° - 0 = 90° - r,
~ C=70.12°
=> T3 =8 ... (4) Since i = 45° and also, we.observe that i <C
From equations (3) and (4), we get Hence, TIR will not take place.
50=360° From Snell's Law, we get
~ 0=72' and 180°-20=36' µ 1 sini = µ 2 sinr
So, the angles of .prism are 72°, 72° and 36°. Ji sin(45') = 1.33sinr
Illustration 35 Ji( 1 )
A ray of light incident normally on one of the faces of sinr T2
1.33
0.752
a right angle isosceles prism is found to be totally reflected
as shown. What is the minimum value of the refractive r = sin-• (0.752) = 48.75'
index of the material of the prism? When prism is
immersed in water (µ =1.33) trace the path of the

=================================== = 1.47

www.puucho.com
Rahul Sardana with www.puucho.com

Advanced JEE Physics Optics & Modern Physics

Illustration 36 r2 >C
A ray of light is falling on face AB of a tetrahedral of =,. sinr2 > sinC
refractive index µ at angle of incidence i . The ray after
getting internally reflected on face BC emerges from AD =,. sin(45°) > 1_
µ
perpendicularly to the incident. beam. Find the range of µ
and i. 1 1
=,. ->- {--- sinC =;}
B .fi. µ
=,. µ > .fi.
sini sini
Further, we have µ = . 2sini
S1IlT1 sin(30°)
Since, µ > .fi.
=,. 2 sin i > .fi.,
D
Solution . . 1
=O> Sin I > .fj_
Since, r1 + r2 = LB = 75° ... (1)
=> i>45°
B

MINIMUM DEVIATION
The angle of deviation D varies with the angle of incidence
i , as shown in the figure.

D
__ J ''
''
i=e
From figure, we observe that e = i , because 1 and 2 are
perpendicular
It is found that D is minimum when i = e . Thus,
Dm1n =(i+e)-A=2i-A
Using Snell's Law,
-----1 - -------
·, e lsini = µsinr1
1 o/ I 2
c§): and µsinr2 = lsine = sini
''
(Ray 1) 1- (Ray 2) => µsinr1 =µsinr2
In quadrilateral QCDR, we have =,. r1 = r, = r (say)
(90° -r,)+(90°+r,)+60° + 135° = 360° Since, rt' + r2 =A
r, = 360°-60°-135°-(90°-r,)-90° ... (2) A
::::> r=-
T3 =r2 -15° 2
sini sine
Further,µ=-.-=-.- smi
.. . (A+Dm;•)
Slil
2
smr1 smr3 µ=--=
=>. r3 = r1 {because i = e} ... (3) sinr1 .
Sin (A)
-
2
Solving equations (1), (2) and (3), we get
Note that if .the prism is equilateral or isosceles, then the ray
r2 = 45° and r1 = 30°
inside the prism is parallel to its base.
Now, for TIR (total internal reflection) to take place at the
face BC; we have

= 1.48

www.puucho.com
Rahul Sardana with www.puucho.com

Ray Optics: Refraction at Plane Surfaces


A A

s~-------~c
r1 = r3 = r
Illustration 37 i1=i2=i
r2~c
The angle of minimum deviation for a glass prism
with refractive index ./3 equals the refracting angle of the Since, r1 + r2 = r2 + r3 =60°
prism. What is the angle of the prism? =>. r1 = r3 = r (say)
Solution Similarly by symmetry, we have i1 =i, =i (say)
Since we know that
Also, r2 ::::; C
s. m
(A+6m)
-- =>. r=60°-C
µ= 2
Given, OTotaI = 120°
sin(1) =>. 6,+6Q+6, =120°
Since Bminimum =()min =AI so we get =>. (i-r)+(180-2C)+(i-r)=120°
sinA 2i-2(60°-C)+180°-2C=120'
./3 =>.

sin(1) =>. 2i = 60°


=> i=30°
=>. ./3 = 2cos( 1) Now, according to Snell's Law, we have
sini sini
µ = -1- = -
=>. cas(1)= ~ sinr1 sinr
But r1 +r2 =60°
=>. A=30° =>. r+C=60°
2
=>. r=60'-C
=>. A=60°
sin(30°)
=>. µ
sin(60°-C)
Illustration 38
The path of a ray of light passing through an 0.5
=>. µ
equilateral glass prism ABC is shown in the figure. ./3 cosc--1.sinC
A 2 2

But sin C = -1.


µ

BL.__ _ _-',L_ _ __:,.C

The ray of light is incident on face BC at an angle just


greater than the critical angle for total internal reflection to =>. ./3Jµ'-1 =2
take place. The total angle of deviation after the refraction
at face AC is 120° . Calculate the refractive index of the =>. 3(µ'-1)=4
glass. =>. 3µ 2 =7
Solution
The ray diagram is drawn for the sake of convenience.
================================== 1.49 =
www.puucho.com
Rahul Sardana with www.puucho.com

Advanced JEE Physics Optics & Modern Physics

B
=> µ=~ µ=A+-
,_ 2

=> µ=1.52
' where A and B are constants. The smaller the value of A,
the larger is the value of µ . Thus, µ is maximum for violet
WHITE LIGHT
colour and minimum for red. The deviation of a ray depends
on µ it is larger for higher µ . Hence, violet suffers ~e
White light consists of infinite number of continuous maximum deviation and red the minimum.
wavelengths (colours) ranging from 4000 A to 7800 A. For
convenience it is divided into seven colours.
If light from sodium lamp falls on a prism then it disperses
Violet, Indigo, Blue, Green, Yellow, Orange, Red called as (breaks) into two lines called D1 ( 5890 A) and D, ( 5896 A)
'VIBGYOR' pattern. lines. Thus we observe that a prism causes deviation as
The Violet having least wavelength (maximum frequency) well as dispersion.
and Red having maximum wavelength (minimum If Dv, DR and '!)y are the deviations caused by prism for
frequency). violet, red and mean yellow rays, then for prism with small
refracting angle (A) , we have
VARIATION OF REFRACTIVE INDEX WITH COLOUR Angular Dispersion D=Dv -D, =(µv -µ,)A
(CAUCHY'S FORMULA)
The refractive index (µ) of a medium varies with
DISPERSIVE POWER OF A PRISM
wavelength (:\,) according to Cauchy's formula
The ratio of angular dispersion to the mean deviation is
B C called dispersive power, so Dispersive Power is
µ=A+,_,+,,,+ ...
0)
Angular Dispersion D Dv -D,
where A , B and C are constants. Mean Deviation Dy Dy
From above we observe that refractive index decreases with where Dy is the deviation of mean light i.e., yellow light,
increase of wavelength. It is maximum for violet and
minimum for red colour and due to this variation of. the whose wavelength is considered as mean of all the
refractive index with the wavelength or the colour, a wavelengths present. Further for a prism of small refracting
composite beam of light entering a prism splits into angle A , we have
constituent colours. D=(µ-l)A
So, we have
DISPERSION
Dv=(µv-1)A, D,=(µ,-1)A and Dr=(µr-1)A
It has been observed that when a beam of composite light
So the dispersive power ro becomes
(consisting of several wavelengths) passes though a prism, it
splits into its constituent colours. This phenomenon is called o, (µv-µ,)A (µv-µ,) ~
dispersion. The band of colours thus obtained on a screen is (µy-1)A µy-1 µ-1
called the spectrum.
where dµ:=µv-µ, and µ=µy
If white light is used, seven colours are obtained as shown in
the figure. The sequence of colours is VIBGYOR, from The dispersive power co has no units and no dimensions. Its
bottom to top. value depends on the material of the prism.

COMBINATION OF TWO PRISMS


Red From a single prism, it is not
Orange possible to get deviation without
Yellow dispersion, or to get dispersion P'
Green without deviation. However, two
Blue
small angled prisms may be
Indigo
combined to produce Dispersion p
Violet
without Deviation or Deviation
without Dispersion. The prism
The dispersion of light takes place because the refractive placement for both is shown here. The placement remains
index µ of the medium depends on the wavelength of light the same. It is just that we are to decide the relation between
their refractive indices such that required condition may be
as given by Cauchy's formula, according to which
achieved.

= c==================================
1.50

www.puucho.com
Rahul Sardana with www.puucho.com

Ray Optics: Refraction at Plane Surfaces

A. DISPERSION WITHOUT DEVIATION This give~


Two prisms can be combined in such a way that the A'=(µv-µ')A
deviation of the mean ray produced by one is equal and µ~-µ~
opposite to that produced by the other. Such a combination
Also from (1) we get
is called a direct vision pris'm.
A (µv -µ,)A=(µ~ -µ~)A'

=> (µv -µ,)(µ-l)A=(µ~ -µ~)(µ' -l)A'


'White µ-1.. µ-1 .
1""""'~~~R7-c1~:....
ligh::,I => coD =ro'D'
V is the condition for Deviation without Dispersion.
Crown The net mean deviation is
~-----~A,'
D-D' =(µ-l)A-(µ' -i)A'
So, in this arrangement of prisms, the mean deviation ·(o)
caused by one prism is cancelled by the mean deviation (D')
caused by the other prism i.e. Illustration 39
D-D'=0
The refractive indices of the crown glass for blue and
red light are 1.51 and 1.49 respectively and those of the
=> (µ-l)A-(µ' ~ l)A' =0 flint glass are 1.77 and.1.73 respectively. An isosceles prism
of angle 6° is made of crown glass. A beam of white light
or A''= (.!:.=!.)A.
µ'-1 is incident at a small angle on this prism. The other flint
glass isosceles prism is combined with the crown glass
The net dispersion produced is =(Dv -D,)-(D~ -D~) prism such that there is no deviation of the incident prism.
(a) Determine the angle of the flint glass prism.
Net Dispersion =(µv -µ,)A-(µ~ -µ~)A'
(b) Calculate the net dispersion of the combined system.
. Net_ ·)=(µv-µ•J(µ-l)A-(µ~-µ~)(µ'-l)A' Solution
( D1spers10n µ -1 µ' - 1 (a) When angle of prism is small and angle. of incidence is
o Net Dispersion = roD- ro'D' also small, the deviation is given by o= (µ -1 )A
where co and ro' are dispersive powers of prisms P and · Net deviation by the two prisms-is.zero, when deviation
P'. due to one cancels the deviation due to the other. So,
01 -0,=0
B. DEVIATION WITHOUT DISPERSION ... (1)
(ACHROMATIC PRISM)
Here, µ 1 and µ 2 are the refractive indices for crown
It is possible to combine two prisms of different .materials in
such a way that each cancels the dispersion due to the other, . and flint glasses respectively, where
Thus, the net dispersion is zero but a deviation is produced.
So, in this arrangement of prisms, the dispersion (Dv -DR)
caused by one. prism is cancelled by dispersion ( D~ - o;)
produced by the other prism.
/
Crown

1.51 + 1.49 l 5 d 1.77 + 1.73 l 75


µ, 2 · an µ, 2 ·
Angle of prism !or crown glass is A, = 6°
Substituting this values in equation (1), we get
(1.5-1)(6°)-(1.75-l)A, =0
This gives A2 =4°
i.e., (Dv-D,)-(D~-D~)=0 Hence, angle of flint glass prism is 4°
or (µv-µ,)A-(µ~-µ~)A'=0 ... (1)

==================================== 1.51 =
www.puucho.com
Rahul Sardana with www.puucho.com

Advanced JEE Physics OpHcs & Modern Physics

Flint COLOURS OF OBJECTS AND COLOUR TRIANGLE


The colours of objects are due to·a number of phenomena.
The colours of opaque bodies are due to Selective
Reflection. For example grass appears green because when
white light is incident on grass, it absorbs all colours except
green which is reflected. Black appears black because it
6,+62=0
absorbs all colours falling an it an reflects nothing. Similarly
white appears white because it reflects all colours falling on
(b) Net dispersion due to the two prisms is given by it and absorbs nothing. ·
Net Dispersion=(µ~ -µ,,)A,-(µ,, -µ,,)A, The colours of transparent bodies are due to Selective
Transmission. For example a glass appears blue, because it
=> Net Dispersion
absorbs all colours except blue, which it transmits.
= (1.51-1.49)(6°)-(1.77 -1.73)( 4°) =-0.04° The colours of sky, rising and setting sun are due to
~ Net dispersion =--0.04° scattering while the colours of soap bubble and kerosene oil
film are due to interference.
Illustration 40
COLOUR TRIANGLE
A ray of light is incident on a prism ABC of
refractive index -./3 as shown in figure. If, Red (R), Green (G) and Blue (B) are primary colours. If
B D P denotes Peacock Blue also called Cyan, M denotes
Magenta, Y denotes Yellow and W denotes White, then
60' from colour triangle we observe that
R

E
(a) Find the angle of incidence for which the deviation of
light ray by the prism ABC is minimum.
(b) By what angle the second prism must be rotated, so
G B
that the final ray ·suffer net minimum deviation.
Solution R+G+B=W
(a) At minimum deviation, we have r1 = r2 = 30° R+G=Y
According to Snell's Law, we have G+B=P
sini R+B=M
µ=-.-1 B+Y=W
smrt
R+P=W
~
.Js sini1
sin(30°) G+M=W

~
. . =2
Stnl1
.rs RAYLEIGH LAW

~ i1 =60° According to Lord Rayleigh, intensity (I) of scattered light


is inversely proportional to the fourth power of the
(b) In the position shown, net deviation suffered by the ray wavelength 1,. • So,
of light should be minimum. Therefore, the second
prism should be rotated by 60° (anticlockwise). l
Ioc-
1,. 4
B. Dl'CT=---..--,E
60° 60° It can also be concluded that the amplitude (a) of the
60° scattered light is inversely proportional to the square of the
wavelength.
1
60' So, aoc ..2 {·.-Joca 2 }
60' 6_0' 1
A C

= ==================================
1.52

www.puucho.com
Rahul Sardana with www.puucho.com

Ray Optics: Refraction at Plane Surfaces

COLOUR OF THE SKY


When light from the sun travels through earth's atmosphere,
it gets scattered by the large number of molecules of various
gases. It is found that the amount of scattering by molecules,
called Rayleigh scattering, is inversely proportional to the
fourth power of the wavelength. Thus light of shorter
r 4.
b) ii. the system is placed in wale. r what will be the net
deviation? Refractive index of water

.
=i
'
3

A ray of light incident.on the face of a prism is refracted and


escapes through- an adjacent face. What ·is the maximum
permissible angle of refraction of th~ prism, If it is made of
glass with a- refractive index of µ = 1.5 .
wavelength is scattered much more than the light of longer
wavelength. Since blue colour has relatively shorter
wavelength, it predominates the sky and hence sky appears 5. In an isosceles prism of angle 45° , it is found that when the
bluish. angle of incidence is same as the prism angle and the
emergent ray grazes the emergent surface.
(a) Find the refractive Index of the material of the prism.
COLOUR OF CLOUDS (b) For what angle of incidence the angle of deviation will
be minimum? -
Large particles like water droplets and dust do not have this I
selective scattering power. They scatter all wavelengths
almost equally. Hence clouds appear to the white. I 6. A prism of fli_nt glass (µ =¾) with an angle of refraction
9

30° i_S placed inside water ( µw = i) .


(a) At what angle should a ray of light fall on the face of
BASED ON PRISM the prism so that inside the prism the ray is
(Solutions on page 1.183) perpendicular to the bisector of, the angle of the prism.
1. The path of a ray undergoing refraction in an equilateral (b) Through what angle will the ray tum after passing
prism is shown in figure. The ray suffers refraction at the through both face5: of the prism?
faqe AB ,and the refracted ray is incident on the face AC
at an angle slightly greater than the critical angle and 7, Light rays from ~ source are incident on a glass prism of_
hence, totally reflected. After refraction- at the face BC the index of refraction µ and angle of _priSm q.. At near normal
emergent ray makes an angle of 30° with normal at BC at" incidence, calculate the angle of deviation of the emerging
rays.
the point of emergence. Find the
A 8. One face of a prism with a refractive angle of 30° is coated
with silver. A ray incident on another face at an angle of
45° is refracted and reflected from the silver coated face
and retraces its path. What is the refractive index of the
prism?

9. A ray of light is incident at an angle of 60° at one face of a


prism having refracting angle 30° . The ray emerging out of
the prism makes ah angle of 30° with the incident ray. Find
the angle of emergence and calculate the refractive index of
(a) corresponding angle of incidence i. the material of the prism.
(b) refractive index of the prism.
10. A ray of light is incident upon one face of a prism (angle of
2. A beam of light enters .a glass prism at an angle a and
emerges into the air at an angle J3 . Having passed through
prism < i) in a direction perpendicular to the other face.

th_e prismj the beam is deflected from the original direction Prove ttiat the ray will fail to emerge from the other face if
by an angle 6 . Find the refracting angle of the prism and cotA < cotC-1 , where C is critical angle for the material
the refractive index of the materiatof the prism. of prism.
1
3. In a prism of refractive index µ = 1.5 and refracting angle 11. The index of refraction for violet light in silica flint glass is I
60°, the condition for minimum deviation is fulfilled. If face 1.66 and that for red light is 1.62. Find the angular
AC is polished dispersion of visible light passing through a prism of apex
angle 60° , if the angle of incidence is 50° ·:
A
12. A light ray is passing through a prism with refracting angle
60° A= 90° .and· refractive index µ = 1.3 . Find the minimum
and maximum-angle of deviation.
µ= 1.5
13. A ray of li9ht. is incident at an angle of 60° on the face of a
prism h~ving refracting angle 30° . The ray emerging out of
s~------~c the prism makes an angle 30° with the incident ray. Find
the angle of emergence of the ray.
L_(?) Find the net deviation,~-~-----------~ ,_
================================== = 1.53

www.puucho.com
' Rahul Sardana with www.puucho.com

Advanced JEE Physics Optics & Modern Physics

14. The .refracting angle of a glass prism is 30° ., A ray is strikil"lg at ,the first face again and emerging from it. in a
incident onto .one of the faces- perpendicular to it. Find the direction making an angle of 4¢< with the reversed direction_
angle 8 between- the incident~ray and the ray that leaves of the inCident beam. The refracted beam is found to have
the prism. The refractive indeX Of .glass is n =1.5. · undergone a deviation of 1° from- the original direction.
Ca,lcula,te.the refractive index of the glass and the angle ·of
the prism.
15. The refractive index of the mB.terial of a prism is,1.6 fora
certain monochromatic ray . .What should be the max;finum
angle of incidence of this ray on the priSm so that no· total
internal reflection occurs when the ray leaves ·the prism?
The refracting angle of the prism is 45° .

16. A ray of white light falls onto the side surface of an


isosceles prism at such an angle that the red ray leaves the
prism normally-to the second face. Find the deflection of the
red and violet rays from the initial direction if the refraction
a11gl8 of the prism is 45° . The refractive indices of the
prism material for red and violet rays are 1.37 and 1.42,
resp~ctively.

17. A parallel beam of light falls normally on the first face of a


prism 'of small refracting ang18 .. At the second face it is
_Rartly_Jrahsmitte_Q_anQ..p_artly..reflected,Jhe_mflected beam~

1.54

www.puucho.com
\
Rahul Sardana with www.puucho.com

SINGLE REFRACTING SURFACE b) The distances measured against the incident ray are
spherical surface which separates two media of taken as negative.

A different refrac~ve index is called a single refracting


. surface. The copvexity or concavity of the surface is
decided by looking at it from rarer medium as shown in
c)

d)
The. distances measured along the incident ray are taken
as positive.
All transverse measurements done above the principal
figure. ax.is are taken as positive while. the ones done below the
N principal axis are taken as negative.
',,, ,,,.x_____--: x~N~----
µ, µ, I µ, µ, I
Rarer Rarer
, ASSUMPTIONS
Denser! / Denser'/
/ 'While obtaining some relations, in ray optics, we make some
p C C, p '
' assumptions given below. All those formulae will hold good
'' 'l' only if these conditions are satisfied.
' YL--------'
l V a) The object/ source is considered to be point
'
,._R-+< ',._R-+i
' object/ squrce placed on principal axis.
Convex Refrac!ing Surface Concave Refracting Surface b) TI1e aperture of the surface/lens is small.
c) Rays of light make smaller angles with the principal
SOME TERMS C6NNECTED WITH SINGLE REFRACTING axis i.e., are paraxial in nature.
SURFACE
REFRACTION OF LIGHT AT CURVED SURFACES
1. Pole (p) : It is a point which bulges out most (in case of
convex surface) or is depressed most (in case of concave For the curved surfaces the same law of refraction are
surface) as seen from the rarer medium. applicable. When a light-ray enters a denser medium, it
bends· towards the normal. The figures show six situations.
2. Centre of Curvature (C): It is the centre of the sphere
The shaded region is denser.
of which the surface forms a part. ,
'' ,'
3. Radius of Curvature (R): It is the radius of the sphere ''
, Real' '
of which the surface forms a part.
-~------ - i
-- ______ _..._
'' ------- Real
4. Aperture (XY) : The diameter of the refracting surface C µ, l 0 ~11 C µ, I
Denser Denser
is called the aperture of the surface.
(A) (B)
5. Principal axis : The line joining the· pole and centre of
curvature and extended on either side of the surface is In Figs. (A) and (B), the object O is kept relatively far from
called the principal axis. the refracting surface, and the image formed is real.

SIGN CONVENTIONS
\ Virtual
Following sign conventions must be used while dealing with -·---------
Cµ2
'
!!J•1 fl:?
ray diagrams. Denser 1 : Denser
a) All the distances will be measured from the pole of the (C) (D)
surface.

================================== 1.55 =
www.puucho.com
Rahul Sardana with www.puucho.com

Advanced JEE Physics Optics & Modern Physics

In Figs. (C) and (D), the object is nearer the refracting AM AM


a.::::tana.=--=--
surface, and the image is virtual. MO PO'
AM AM
paatanP=-=- and
- _____.,.............."'-,
Virtual
MI PI
0 µ, I C
AM AM
enser yaatany=--=--
(E) MC PC
(F)
Therefore (5) becomes
In Figs. (E) and (F), the refraction always directs the ray
away from the central axis, and hence virtual images are => µ,(~~)+µ,(":)=(µ,-µ,)(~)
formed.
Note the major difference from the images formed due to => l1_ + ~ = µ2 - µ1
reflection from a spherical mirror. Here, real images are PO PI PC
formed on the other side of the refracting surface, and virtual Since PO=-u, PI=+v, PC=+R so we get
images are formed on the same side as the object.
12+µ2 =µ2-µl
-u V R
REFRACTION AT CONVEX SURFACE
If the object is in air, then µ 1 = 1 and µ 2 =µ,so we get
CASE 1 : When the object lies in the rarer medium and the
image formed is real. 1 µ' µ-1
-+-=--
Consider a spherical surface of radius R separating the two -u V R
media 1 and 2 (µ, > µ 1 ) . A point object O is placed on the CASE 2 : When the object lies in the rarer medium and the
principal axis to the left of the pole P · at a considerable
image formed is virtual.
distance from it. The incident ray from O falls on point A Consider a spherical surface of radius R separating the two
and is refracted according to media 1 and 2 (µ 2 - µ 1 ) . A point object O is placed on the
. ... (1) principal axis to the left of the pole P . The incident ray from
0 falls on point A and is refracted according to
Since the rays are paraxial, so the angle a is small and hence
the angles i and r will also be small. Thus, applying such ... (1)
paraxial approximation, then sin i ~ i and sin r ;;; r , so we Since the rays ~e paraxial, so the angle a is small and hence
get from (1) the angles i and r will also be small. Thus, applying such
... (2) =
paraxial approximation, then sini i and sinr r, so from =
(1), we have
... (2)
µ,

µ, I
0 I I '1
(Real) I Denser

'''
'' y
,.__R_.,
' ''
,.__ U - - - , - - - V ---+I
!+- u ~ '' R ----+1
Using the geometrical property that an exterior angle of a l+--V----+i
triangle is equal to the sum of the two internal opposite Using the geometrical property that an exterior angle of a
angles, we get from triangles AOC and AIC, triangle is equal to the sum of the two internal opposite
i=a+y ... (3) angles, we get from triangles AOC and AIC,
and r=r+p ... (4) i"=a+y ... (3)
Substituting the value of i and r from Equations (3) and (4) and r=P+r ... (4)
in Equation (2), we get Substituting the value of i and r from Equations (3) and (4)
µ 1 (a+r)=µ 2 (r-P) in Equation (2), we get
=> µ 1a+µ 2 P=(µ 2 -µ 1 )r ... (5) µ, (a+y) = µ 2 (P+r)
Now, since the aperture of the refracting surface is small, so => µ 1a-µ 2P=(µ·2 -µ 1 )r ... (5)
M and P are very close to each other and hence, We have Now, since the aperture of the refracting surface is small, so
M and P are very close to each other and hence we have

= 1.56 i:::::===================================
www.puucho.com
Rahul Sardana with www.puucho.com

Ray Optics: Refraction at Curved S111faces

AM AM AM AM
a:=:tana=--ee-- yeetany=--ee--
MO PO' MC PC
AM AM
peetanP=-ee- and µ,( ,:)+ µ,( ,:)=(µ, -µ,)( ~ )
MI PI
AM AM
yaatany=--ee-- .l:L+l:L= µ2-µt
MC PC PO PI PC

µ,( :)-µ,( ,:)=(µ, -µ,)( ~~) Since PO =-n, PI =+v, PC =-R so we get
12+12 = µ1 -µ2
_&_-~= µ2-µ1 -u V R
PO PI PC Simply replace subscript 2 by 1 and 1 by 2 in the formula
Since PO=-u, PI=-v, PC=+R so we get derived in Case 1 or Case 2.

11+ µ2 = µ2 -µ1 CASE 4 : When the object lies in the denser medium and
-u V R the image formed is virtual.
CASE 3 : When the object lies in the denser medium and Consider a spherical surface ·of radius R separating the two
the image formed is real. media 1 and 2 (µ 2 > µ,) . A point object O is placed on the
Consider a spherical surface of radius R separating the two principal axis to the left of the pole P . The incident ray from
media 1 and 2 (µ, > µ,) . A point object O is placed on the 0 falls on point A and is refracted according to
principal axis to the left of the pole P . The incident ray from ... (1)
0 falls on point A and is refracted according to Since the rays are paraxial, so the angle a is small and hence
... (1) the angles i and r will also be small. Thus, applying such
paraxial approximation, then sini::: i and sinr =r, so from
Since the rays are paraxial, so the angle a is small and hence
(1), we have
the angles i and r will also be small. Thus, applying such
paraxial approximation, then sin i =i and sin r =r , so from ... (2)
(1), we have
... (2)
µ,
Denser

C p

- R __,___ V---+!
Using the geometrical property that an exterior angle of a
14----u----.a
triangle is equal to the sum of the two internal opposite
Using the geometrical property that an exterior angle of a angles, we get from triangles AOC and AIC
triangle is equal to the sum of the two internal opposite
a.=i+y ... (3)
angles, we get from triangles AOC and AIC
and P=r+y ... (4)
y=a.+i ... (3)
Substituting the value of i and r from Equations (3) and (4)
and r=P+r ... (4)
in Equation (2), we get
Substituting the value of i and r from Equations (3) and (4)
µ, ( u-r) = µ1 (P-r)
in Equation (2), we get
µ, ( r- a)=µ, (P +r) ~ µ,a.-µ,P=(µ,-µ,)y ... (5)

~ µ,a.+ µ,p = (µ, -µ,)y ... (5) Now, since the aperture of the refracting surface is small, so
M and P are. very close to each other and hence we have
Now, since the aperture of the refracting surface is small, so
AM AM
M and P are very close to each other and hence we have a:=tana=--aa--
MO PO'
AM AM
a:::tana.=--aa-- AM AM
MO PO' paatanP=-ee- and
MI PI
AM AM
peetanP=-aa- and AM AM
MI PI 1 "' tany= MC" PC

==================================== 1.57 =
www.puucho.com
Rahul Sardana with www.puucho.com

Advanced JEE Physics Optics & Modern Physics

~ ~
µ,( :)-µ,( ~)=(µ, -µ,)( ~ ) µ,( :)-µ,( ~)=(µ, -µ,)( ~ )

...!:L_..!::1= µ2-µ1 ~
-1:1_ _ ..e..t = µ1 - µ2
~
PO PI PC PO PI PC
Since PO =-u, PI =-v, PC =-R sowe get Since PO=-u, PI=-v, PC=-R .soweg~t
11 +11 = µ1 - µ2 h+ µ2 = µ2 -µ~
-u V R -u V R

REFRACTION AT CONCAVE SURFACE CASE 2: When the object lies in the denser medium.
For a concave. refracting surface the image formed is always Consider a spherical surface of radius R separating the two
virtual irrespective of the placement of the object. media 1 and 2 (µ 2 > µ 1 ). A point object O is placed on the
CASE 1: When the object lies in the rarer medium. principal axis to the· left of the pole P . The incident ray from
Consider a spherical surface of radius R separating the two 0 falls on point A and is refracted according to
media 1 and 2 .(µ 2 > µ 1 ) • A point object O is placed on the µ 2 sini = µ 1 sinr ... (})
principal axis to the left of the pole P .' The incident ray from Since the rays are paraxial, so the angle a. is small and hence
0 falls on point A and is refracted according to the angles i and r will also be small. Thus, applying such
: .. (1) paraxial approximation, then sini =i and sinr.:= r, so from,
Since the rays are paraxial, so the angle a is small and hence (1), we have
the angles i and r will also be small. Thus, applying such ... (2)
paraxial approximation, then sin(:: i and sinr::: r, so from -------A
(1), we have
.... (2) µ,
Rarer

µ, µ,
Rarer Denser 0 C

/ f;~;
·R--
0 l C
'--------'B
Using the geometrical property that an exterior angle of a
triangle is equal to the sum of the two' internal opposite
angles, we get from triangles AOC and .AIC
Using the geometrical property that an exterior angle of a
triangle. is equal to the sum of the two internal opposite i=a+y ... (3)
angles, we get from triangles AOC and AIC and r=P+y ... (4)
y=a+i ... (3) Substituting the value of i and r from Equations (3) and (4)
and r=P+r ... (4) in Equation (2), we get
Substituting the value of i and r from Equations (3) and (4) µ, (a +y) = µ, (P +y)
in Equation (2), we get ~ µ,a-µ,p = (µ, -µ,)y ... (5)
µ,(y-a)=µ,(y-p)
Now, since the aperture of the refracting surface is small, so
~ µ,a-µ,P=(µ, -µ,)y ... (5) M and P are very close to each other and hence ~e have ·
Now, since the aperture of the refracting surface is smalt so AM AM
a=:tana=--ee--
M and P a_re very close to each other and hence we have MO PO'
AM AM AM AM
a=:tana=--ee-- peetanP=-ee-·- and
MO PO' MI PI
AM AM · AM AM
peetanP=-ee- and yeetany=·MC ee PC
MI PI
AM AM
yeetany=--ee--
MC PC
~ µ,( :)-µ,( ~)=(µ, -µ,)( ~ )

= 1.58 ==================================
www.puucho.com
Rahul Sardana with www.puucho.com

Ray Optics: Refraction at Curved Surfaces

=> J:L_l:.!.= µ1-µ2 => ~=11 ... (1)


PO PI PC µ,
Since PO=-u, PI=-v, PC=+R so we get Now, in MBP and M'B'P, we have
12+1:!.=µl-µ2 AB=utani=ui ... (2)
-u V R and A'B'=vtanr~vr ... (3)
The magnification is defined as
CONCEPTUAL NOTE(S)
height of image A'B'
a) For both ·convex and concave spherical surfaces, the m
refraction formulae are same,, only proper signs of u , v height of object AB
and R are to be used. Using Equations (1), (2) & (3), we get
b) For refraction from rarer to denser medium, the refraction
formula is
.&.+ µ2 = µ2-µ,
m= ~:· =:; =(:)(;,)
-u V R
=> m=~l::1
ci) For refraction from denser to rarer medium, we Inter- u µ,
change µ 1 and µ 2 and obtain the refraction formula,
H m is positive, the image is erect and virtual.
J:g__+&=~-µ2 H m is negative, the image is inverted and real.
-u .v R
d) If the rarer medium is air (µ 1 = 1) and the ,denser medium ,------------------
CONCEPTUAL NOTE(S)
- -·-- - - -
has refractive index µ (i.e., µ 2 =µ ), then
The refraction formula 11+ µ2 = ~ -µ1
is equally applicable to
(i) for refraction from air to medium, we have R -u V

_!___+!'.= µ-1 plane refradting surfaces i.e., surfaces for which R -i- oo . Let us
-u V R . Real Depth
denve µ -'-'--'-"'--'""'-"--~ using this.
(ii) for refraction from medium:to air, we have Apparent Depth

.2._+l'.= µ-1 Applying .Hi.+ µ 2


µ1 =µ 2 -
-v u R -uR V
with proper sign conventions and values, we get
e) The factor µ2 - µ1 is called power of the spherical
R _µ_+.!.= 1-µ =0
refracting surface. It gives a measure of the degree to -(-d) V oo
which the refracting surface can converge or diyerge the
rays of light passing through It. For air-medium interface, d
:::) V=--
the power is µ I

P= µ-1 I
R
R->oo 2 I
TRANSVERSE MAGNIFICATION
Instead of a point object O let us now, keep an extended
i
d
f+ve !

object AB at point O such that its image A'B' will be


formed at point I. The distance x(=-u) and y(=v) are
!~~
0
related by the above formula. i.e., image of object O is formed at a distance ~ on same side.
~------~ µ
B
I So d ::. dactuar
µ
A"! , app

A
Real Depth
s-1 µ = Apparent Depth
,-u V

A ray from point B of the object is incident at point P and Illustration 41


is refracted, in accordance with Snell's Law, such that In figure, a fish watcher watches a fish through a
µ 1 sini=µ 2 sinr 3 cm thick glass wall of a fish tank. The watcher is in level
{applying paraxial approximation)

1.59 =
www.puucho.com
Rahul Sardana with www.puucho.com

Advanced JEE Physics Optics & Modern Physics

For face CD , we have


with the fish; .the index of refraction of the glass is _ll_ and
. . . 5 ,8
that of the wateris !. _2_ - ~ =0
3 AI, -11.16
(a) To the fish, how far away does the watcher appear to
be? · o AI, =-(11.16)(¾)=--6.975 cm
(b) To the watcher, how far aw'ay does the fish appear to
be? o FI, =8+6:975- •
o Ff; = 14.975 cm
:..-a cm 6.8cm
''
''
Illustration 42
'
·----------
Observer
There are two objects 0 1 and 0 2 at an identical
distance of 20 ·cm on the two sides of the pole of a
spherical_ ·concave refracti.ng J,oundary of radius 60,cm.
The indices ·of• refraction of the llledia on two sides of the
Solution I ,, ' I ,.

(a) OA=3cm boundai; are 1 and (¾} respecfo'.ely. Find the location of
So, AI; =(n, )(OA) the object
(a) o, 'wheri see;,_ from o, .
o AI, =(¾)c3l=4.8 cm
(b) O, when see!' from· 0 1 •
C E Solution

:f.,
The formula for-refraction from a curved boun.dary ls
___ J..: __ _&_.!:':!_= µ,-µ,
-4
B, 'f. F
.' v u' R
(a) From the ray diagram drawri, we get,
D G - ---+- +ve ' . ' µ =1, µ =-
4 ·
u1 =-20cm, R=--60cm, 1 2
For refraction at EG(R--> oo), using 3

n2 _!::!_= n2 -11i µ! =1 ... ~· µ,= (4/3)


-.::=::-_-,:-.:---~-
c~ --r o,_ P---~-----------
V u R
4· 8. ~.

3
BI,
5
-(4.8+3)
0

BI, =-(7.8>(¼)(¾)=-6.5 cm
1+-- -
20cm:
60 cm _____.._'

So, FI, =6.5+6.8=13.3 cm


(b) For face Ef, we have
v=-24 cm
8· 4 ,. i' '. '
Thus, the-object 0 1 , will appear at a distance of 24 cm
...2._ - _3_ = 0
BI, --6.8 from P towards C .
(b) Keeping the object 0 2 on the left of the· pole P as
o BI, =-(6.8>(¾)(¾)=-8.16 c_m shown, here, we get

C E 4
u1 =-20cm, R=+60cm, µ 1 ,=-, µ2 =1
. 3

O•--------A . ''·',B.
--- ----- "7
~F • µ, -(4/3)
··, i . _,:_'_________ ~
, -- -- -.. ____ _
--------
O, I P O, C
D G +ve....,_
------
20cm 1 60 cm ---+1

= 1.60 i:::::::================================:::=:i
www.puucho.com
Rahul Sardana with www.puucho.com

Ray Optics: Refraction at Curved Surfaces

1 (¾) 1-(¾) Solution


(a) Applying the formula for the refraction at the curved
; - (-20) =----;;a boundary i.e., '
=> v=-16.36 cm µ2 _l:!_ = µ2 -µ1
Thus, the object 0 2 will appear at a distance of V u R
16.36 cm from P towards 0 2 • For, refraction at the first surface, the pole is P1 and we
observe that
Illustration 43
An object of height 1 cm is kept at a distance of
40 cm fr~m a concave spherical surface having radius of
curvature R = 20 cm , separating air and glass having 1 (¾) 1-(¾)
refractive index µ = 1.33 . Find the location, height and the ; - (-oo) = (+R)
nature of the image formed. V=-3R
µ,=1.33 Thus, the first image 11 is formed at a distance of 3R to
1 emf the left of pole P1 •
C Titis image acts as an object for the refraction at the
' I' secon.d surface, with pole P2 • For this refraction, we
,..........
20cmi
have
i+-40 cm-+-:' 4-
U=-(3R+2R)=-SR, R=-R, µ1 =1, µ 2 =-
Solution 3
According to Cartesian sign convention, ±
l, __1_=_3_
±_1
u=-40cm, R=-20cm
V -SR -R
Applying the formulaµ,_&_=µ,-µ, ,we get
V u R v=-~R
2
1.33 _ _1_= 1.33-1
(-40) (-20) SR
V Thus, the final image 12 is at a distance from P2
2
=> v=-32cm towards left.
µ,v lx(-32) (b) The ray diagram is shown in figure.
Magnification, m _h2 0.60
h, µ,u 1.33x(-40) ------7
So, height of image, h, = mh., = 0.6 x 1 = 0.6 cm _ _!µ~~;'.'.4/'_:3 _ __,~-=--~?""
-
_________ ...
I
The positive sign of magnification indicates that the image is --------- I
virtual and erect.
I
---....r~... --- ------«:{_. ...... ------•------
!; -
----- ...........12___ .......... P1 . P2

Illustration 44 --- ............... . .


A parallel beam of light travelling in water ( µ = ¾) is --------------------
o+---3R---a>t+--2R_.,
refracted by a spherical air bubble of radius R situated in ---5R/2---M
water.

Illustration 45
R
A solid glass with radius R and an index of refraction
1.5 is silvered over one lielllisphere. A small object is
P, P,
located on the axis of the sphere at a distance 2R to the
µ=1
left of the vertex of the unsilvered hemisphere. Find the
µ=4/3 µ=413 position of final image after all refr~ctions and reflections
(a) Find the position of the image due to refraction at the have taken place.
first surface and the position of the final image. Solution
(b) Draw the ray diagram showing the position of the two The ray of light first gets refracted then reflected and
images. then again refracted. For first refraction and then reflection

================================== 1.61 =
www.puucho.com
Rahul Sardana with www.puucho.com

Advanced JEE Physics Optics & Modern Physics

the ray of light travels from left to right while for the last B C
refraction it travels from right to left. Hence, the sign
conventio:h. will change accordingly.

A'-'------~D

Solution
Applying Snell's Law at face AB, we get
First Refraction :
(1)sin45° = (../2)sinr
Using µ2 _!:!_= µ2-µl with appropri9:te sign
V u R . 1
=> smr=-
conventions, we get 2
1.5 1 1.5-1 => r=30°
-;;;-- (-2R) = ~ i.e., ray becomes parallel to AD inside the block.
=> V1 ~oo
Now applying, µ, - µ, = µ, - µ, on face CD, we get
Second Reflection : V u R
1 1 1 2 1.514 ../2 1.514-../2
Using -+-=-=-
v u f R
with appropriate sign
OE oo 0.4
conventions, we get Solving this equation, we get
1 1 2 OE"6 m
-+-=--
Vi 00 R
R Illustration 47
=> =--
Vz
2 A glass sphere of radius R =10 cm having refractive
Third Reflection :
index µ8 =! is kept inside water. A point object O is
2
Again using µ2 _ 1:2_ = µ 2 - µ 1 with reversed sign
V u R placed at 20 cm from A as shown in figure. Find the
convention, we get position and nature of the image when seen from other
1 1.5 1-1.5 side of the sphere. Also draw the ray diagram. Given
----=--
v, -1.5R -R refractive index of water is µw =_! .
- 3
=> v,=-2R

0 A B
0 M- 20 cm---.i

Solution
R/2 A ray of light starting from O gets refracted twice. The
1+-1,SR~ ray of light is travelling in a direction from left to right.
Hence, the distances measured in this direction are taken
i.e., final image is formed at the vertex of the silvered
face i.e., at the pole of silvered/ curved surface.
positive. Applying

11+&= µ2 -µ1 , twice with appropriate signs at the


-u V R
Illustration 46
two refracting surfaces, we get
Figure shows an irregular block of material of
p M I
refractive index ../2 . A ray of light strikes the face AB as 2 2
shoWII. in tlie figure. After refraction it is incident on a
spherical surface CD of radius of curvature 0.4 m and 0 B
enters a medium of refractive index 1.514 to meet PQ at
-----+- +ve
E . Find the distance OE upto two places of decimal.

= 1.62 i::::=================================
www.puucho.com
Rahul Sardana with www.puucho.com

Ray Optics : Ref.-actio11 at Curved Sm/aces

µ 1 (µ-1)
(¼) + (¾) (¾-¼) ;,-- (-2R) =-R-
-(-20) AI1 10
µ 2µ-3
~ AI1 =-30 cm -=--
v, 2R
Now, the first image 111 acts as an object for the second
v,·= 2µR ... (1)
surface, so~ we have 2µ-3
BI1 = u =-(30+20)=-50 cm The first image acts as object for refraction at second surface
S2 • The origin of our Cartesian coordinate system is now at
Again applying .l:L+ µ, = µ, -µ, , we get
-u V R vertex/pole of surface 52 • Object distance for second
refraction is
(¾) + (¼) = ¼-¾ u, =-(3R-v,)
-(-50) .BI, -10
~ BI, =-100 cm ~ u2 = -(3R ( 2: 3 ))
i.e., the final image 12 is virtual and is formed at a distance
100 cm (towards left) from B . The ray diagram is as shown. u, =-(4µ-9)R
2µ-3
N For refraction at curved surface 52 , we have
1 µ (1-µ)
-----::::::-- ... (2)
r, r,
"4-20 cm--+1
o A C 8 v, -(4µ-9)R
2µ-3 (%)
On solving the above expression for v 2 , we get
1+--- 30 cm -----+1
1+-------100 c m - - - - - - + 1 (4µ-9)
v, ... (3)
Following points should be kept in mind while drawing the (10µ-9)(µ-2)
ray diagram. The image will be real if v 2 is positive, i.e.,
(i) At P the ray travels from tarer to a denser medium.
Hence, it will bend towards normal PC . At M, it (4µ-9) >0 ... (4)
travels from a denser to a rarer medium, hence, moves (10µ-9)(µ-2)
away from the normal MC. The equation (4) is satisfied when
(ii) The ray PM when extended backwards meets the Origin for refraction
at second surface
principal axis at I1 and the ray MN when extended Origin for first
refraction A
meets the principal axis at I2 •
0 B
Illustration 48
''
A ·glass rod· has ends as shown in figure. The : C I D
refractive index of glass is µ. The object O is at.a distance S' S'2
1

2R from the surface of larger radius of curvature. The ,._ 2R _ _ R.,_3R/2___..


R/2
distance between apexes of ends is 3R.
Case I:
O air (4µ-9)>0
R R/2
9
-J;?'4~~~ r';l~ µ>-
4
..-2R - ; - - - 3R ____..,
and (10µ-9)(µ-2)>0
(a) Find the distance of image formed of the point object
from right hand vertex. ~ 0.9<µ<2
(b) What is the condition to be satisfied if the image is to there is no common solution for this condition and
be real? hence this is rejected.
Solution Case II:
For refraction at curved surface 51 , 4µ-9<0

1.63 =
www.puucho.com
Rahul Sardana with www.puucho.com

Advanced JEE Physics Optics & Modern Physics

9 Solution
=> µ<-
4 Since parallel rays after passing thr9ugh a lens must
(10µ-9)(µ-2)<0 converge (or appear to converge) atthe point. So this point is
the place where focus is located and the final image is also
=> µ>2 OR µ<0.9 formed at the focus.
Hence the common resul_t is i < µ < 2.25 For refractio:ri at first surface, we get ·
1:1__1:1_ = µ2 -µ1 ... (1)
Illustration 49 V1 -00 +R
A quarter cylinder of radius R and refractive index
1.5 is placed on a table. A point object P is kept at a
distance of mR from it. ·Find the value of- 111 for which a
ray from P will emerge parallel to the table as shown in I
1.-.-.-.-
., ...... .-

figure.
~ +ve -->---!-"<'"- -- ,-,
:'
' ' '
!+- V2 ---+:
,..'s - - - V , - - - - + !

For refraction at 2nd surface, we get


1+ mR +t+- R----+1 · µ3 _ µ2·,=:µ3 -µ2 · ... (2)
Solution Vi V1- +R
Adding equations (1) and (2), we get
Applying µ, ~11= µ,-µ,, firstly at the plane surface
u.V R . µ3 =µ3-µl
and then at the curved surface. v, R
For the plane surface, we get
-µ,R
=> Vz=---
1.5 1 _ 1.5-1 _ 0 {·: R-->oo) µ3 -µ1
Al, - (-mR) ----;;;--
Hence, focallength of the given lens system is
=> AI,= -(l.5mR)
-·1=...l!L
For the curved surface, since the final image is formed at µ3 -µ1
infinity, so we get ·
1 1.5 1-1.5
Illustration 51
00 -(l.5mR+R) -R
A parallel beam of light travelling in water having
1.5 0.5
=> refractive index·¾ is refracted by a spherical air bubble of
(l.5m+l)R. R
=> 3=1.5m+l radius 2 nun situated in water. Assuming the light rays to
be paraxial.
3
=> m,,,.2 (i) F;_;,d the position of the image due to refraction at the
2 first surface and the position of the final image;
4 (ii). Draw a r_ay diagram showing.the positions.of.both the
=> m=-
3. images.
Solution
Illustration 50 (i) To get the desired result(s), we shall be applying
In the figure, light is incident on the thin lens as
shown. Th~ radius of curvature_ for both the surface is R .
µ 2 - µ1 =µ2 - µ 1 , one by one on two spherical surfaces.
V u R .
Determine the focal length of this system.
F~r first re-~actio~ at AP1B, ~~ have

µ, 4 1-±
.!a - 1. =-'-1.
V1 oo +2
1 1
==> -=--
v, 6

= 1.64 i;:;;:;=====================::::::=============
www.puucho.com
Rahul Sardana with www.puucho.com

Ray Optics: Refraction at Curved Surfaces


- ====- ~~

=> V1 =-6 mm sphere of refractive index 1.5 and of radius r is silvered to


make the inner side reflecting. An object is placed at the
A axis of the sphere at a distance 3r from the centre of the
sphere. The light from the object is refracted at the
P, P, unsilvered part, then reflected from the silvered part and
again refracted at the unsilvered part. Locate the final image
formed.

So, the first image will be formed at 6 mm towards left


~\
of P, --•------------
0
-----•------l
For second refraction at AP2B, the distance of first /
image I, from P, will be 6mm+4mm=10mm ,.__ _ _ 3r _ _ _...., -~
(towards left). So, we get
± ±_1
_Q_ __l_ _3_
3. Consider the figure shown. A hemispherical cavity of radius
R is carved out from a sphere (µ =1.5) of radius 2R such
v, -10 -2 that principal axis of cavity coincides with that of sphere.
One side of sphere is silvered as shown. Find the value of
4 1 1 4 x for which the image of an object at O is formed at 0
=> ---------
3v, 6 10 15 itself.

=> Vi=-5 mm
(ii) The ray diagram is shown in figure·
Q,/ ----+- +ve

..... --
.--------------··· ...~x::·..,·~.: .'
~

I, I, C

2mm 2mm
i.- 6 mm-->...--<---.! 4. A glass sphere has a radius of 5 cm and a refractive index
t<f---5mm--+1 of 1.6. A paperweight is constructed by slicing through the
sphere on a plane that is 2 cm from the centre of the
sphere and perpendicular to a radius of the sphere that
CONCEPTUAL NOTE(S) passes through the center of the circle formed by the
a) At P and Q both normal will pass through C. intersection of the plane and the sphere. The paperweight is
b) At P ray of light is travelling from a denser medium placed on a table and viewed from directly above by an
(water) to rarer medium (air) therefore, ray of light will bend observer who is 8 cm from the tabletop, as shown in figure.
away from the normal and on extending meet at 11 • When viewed through the paperweight, how far away does
the tabletop appear to the observer?
Similarly at 0. ray of light bends towards the normal.
Observern
c) Both the images 11 and 12 are virtual.

~---- _____ • ________ICE_V_I _ . -~ -- I


BASED ON REFRACTION AT CURVED SURFACES
Bern

(Solutions on page 1.187} 3cm


1. A spherical convex surface separates object and image
5cm
space of refractive index 1 and .±3 . If radius of curvature of
the surface is 1O cm, find its power.
5. One end of a long glass rod having refractive index µ = 1.5
1+-40cm~
is formed into the shape of a convex surface of radius
6 cm. An object is located in air along the axis of the rod, at
a distance of 1O cm from the end of the rod.
(a) How far apart are the object and the image formed by
F the glass rod?
(b) For what range of distances from the end of the rod
must the object be located in order to produce a virtual
fl, '-µ,-'------- image?
2. A hemispherical portion of the surface of a solid glass

================================::: 1.65
www.puucho.com
Rahul Sardana with www.puucho.com

Advanced JEE Physics Optics & Modern Physics

f ,fi. An o~ject is at a distance of d =·2.5 cm from the surface of a


and cancavo-convex lenses. Sometimes, converging lens is
j a glass sphere with a radius R = 1O cm . Find the,position of represented symbolically by a double headed arrow as
the image produced by the sphere. The refractive index of shown.
the glass is µ =1.5.

00 ~ ~ I
7. A glass hemk:fphere of radius 10 cm and refractive. index
1
1 µ =1.5 is Silvered over its curved ,surface. There is an air
l bubbl0 Jn the glass 5 ems fr9m the _plane surface along the
axis. Find the position of the Images of this bubble seen _by.
observer lo(lking along thB- axis ·into the flat surface of the Equi- Bi- Plano- Concavo-convex Symbolic
hemisphere. convex convex convex (Convex meniscus) representation

8. A hollow sphere of glass of refractive index µ has a small


mark on its interior surface which is observed from a point CONCAVE OR DIVERGING LENSES
outside the sphere on 'the sid9 opposite the center. The
inner cavity is concentric with external surface and the Diverging lenses convert a parallel beam of rays into a
thick11ess of the glass is everywhere equal to the. radius of divergent beam. Diverging lenses are concave, i.e. such that
the inner surface. prove that the ·inark will appear nearer the thickness at. their edges is larger than the thickness at the
than it really is, by a distance (µ -l)R , where R is the middle. A concave l_ens is thinner at ·the centre. They include
(3µ-1) concave-concave~ piano-concave and convexo-concave
radius,of the inner surface. lenses. Sometimes, a diverging lens is represented
~-----------------' symbolically by a line with inverted arrows at its two ends.
THIN SPHERICAL LENSES
A lens is a piece of transparent material with two refraCting
surfaces, at least one of them being curved. It may hav~ one
surface plane.
A spherical lens has spherical surfaces as bounds. If the
thickness of the lens is small (compared to the radius of
curvature of spherical surfaces, the object distance, the image
distance, etc.), it is said to be thin. Equi- Bi- Plano- Convexo-concave Symbolic
There are two types of lenses : concave concave concave ~ (Concave meniscus) representation

a) convex or converging lenses,


b) concave or diverging lenses. OPTICAL CENTRE OF LENS
The central portion of a lens (both convex and concave)
NAMING CONVENTION FOR LENSES behaves as a flat slab. Optical centre O is a' point through
While naming a lens, the surface· with larger radius- of which any ray passes ~deviated.
curvature is named first. The lens has a nature of the surface
that has the smaller radius of curvature. Face 1 Face2 Face 1 ~-~ Face 2
---.. / ---..: /
EXAMPLE : A lens with one surface plane and the other surface
convex will be named as Plano-Convex irrespective
of its placement and this lens will have converging
. nature (the saine as the nature of the surface c, c, C, C,
having smaller radius of curvature).
Principal _Principal
Similarly a Convexo-Concave lens will have a diverging axis axis
nature and ConcaVo-Convex lens will have a converging
nature. Corwexlens Concave Lens
To summarise, we can say that the first name of a bifocal lens
is derived from the name of the surface with bigger radius of PRINCIPAL AXIS OF A LENS
curvature and the last name of.the lens is derived froirt the
nature of the lens. The line joining the centres of curvature C1 C1 is called the
principal axis of the lens.
CONVEX OR CONVERGING LENSES
PRINCIPAL FOCUS
Converging lenses convert a parallel beam of incident rays
into a convergent beam. Converging lenses are convex, i.e. A lens has two focal points. The first focal point F, is a
such that the thickness at the middle is larger than . the point object on the principal axis for which the image is at
thickness of edges. A cohvex lens is thicker in the centre than infinity.
at its edges. They include convexo-convex, piano-convex,

= 1.66 ~=================================
www.puucho.com
Rahul Sardana with www.puucho.com

Ray Optics: Refraction at Curved Surfaces

, '
'''
/
/
F, F, F, F, F,
----:.~,. F,

2. A ray passing through the optical centre goes through


the lens undeviated.
The second focal point f 2 is a point image on the principal
axis for which the object is at infinity.

F, F, F, F,

F,
--
-- ........ 3. A ray passing through the focus (in case of a convex
lens) or appearing to pass through the focus (in case of
a concave lens) is rendered parallel to the principal axis
after refraction through the lens.
Focal length (/) is the distance between O and the second
focus F2 •

Aperture is the effective diameter of the light transmitting ',


',,
area of the lens. The intensity of the image formed by the
lens, F, F, F, F,
2
I oc (Aperture )
Converging and diverging action of a lens is due to the fact Any two of the above three rays can be used to obtain the
that a lens may be thought of a combination of small prisms, location of the image.
as shown in figure. A parallel beam of light, when incident
on a convex lens, converges to a point called focus F .

,, ,

Eiiiii:::--- .

',· Convex Lens

Converging action Diverging action

A concave lens diverges a parallel beam of light. It appears A


to be diverging from a point F, called focus.
For thin lenses, we neecl not consider refraction of light at the
two surfaces separately. Instead, we say that the light-ray is B
bent (towards the principal axis in case of convex lens, and
away from the principal axis in case of concave lens) when it
passes through a thin lens.
Concave Lens
RULES FOR OBTAINING IMAGES IN LENSES
THIN LENS FORMULA FOR A CONVEX LENS
1. A ray parallel to the principal axis, after refraction
Assumptions used in the derivation of lens formula
through the lens, converges to the focus (in case of a
convex lens) or appears to diverge from the focus (in a) The lens used is thin.
case of a concave lens). b) The aperture of the lens is small.
c) The incident and refracted rays make small angles with
the principal axis.

==================================== = 1.67

www.puucho.com
Rahul Sardana with www.puucho.com

Advanced JEE Physics. Optics & Modern Physics

d) The object is a small object placed on the principal axis.


CASE 1 : When a real image is formed
Consider an object AB placed perpendicular to the principal
axis of a thin convex lens between its F' and C' . A real, _
inverted and magnified image A'B' is formed beyond C on
the other side of the lens, as shown in figure.

Since OM= i\.B


A'B'
-v--
B'F
=:>· --=- ... (2)
AB OF.
From (1) and (2), we get .
OB' B'F OB'+OF
Since, M'B'O alld AABO are similar,
OB OF OF
A'B' .OB'
... (1) Using new Cartesian sign converttion,
AB OB
Object d~tarice BO= -u
Also M'B'F artd llMOf are sll?lllar,·
Image distance OB'= -v
A'B'.- FB'
--=-· Focallength OF=+/
MO OF
-v -v+ f
Since MO = AB, -u=-f-
A'B' FB'
--=- ... (2) -vf =uv-uf
AB OF
From (1) and (2), we get =:> uv=uf-vf
OB' .FB' OB'-OF Dividing both sides by, uvf , we again get
-=-
OB Of· OF
Using new-Cartesian sign convention, we get
.!.=.!._I.
f V U
Object distance, OB= -u
Thin lens fomtuia for a Concave Leris
Image distance, OB'= +v
Let O be the optical centre and F be the principal focus of
Focal length, OF=+ f concave lens of focal length / . AB is an object placed
.!'_=V-f perpendicular io its principal axis. A virtual, erect and
-u f diminished image A'B' is formed due to refraction through
the lens. ·
=:- vf=-uv+uf
=:> UV =Uf-vf
Dividing both sides by uvf , we get
1 1 1
-=---
! V U

Ca~e 2 : When a virtual image is formed


When an object AB is placed between the optical centre 0
and the focus F of a conVex lens, the image A'B' formed by Since, M'B'O and MBO are similar
the convex lens is virtual, erect and magnified. as shown in A1B1 OB'
figure. So, AB= OB ... (1)
· ·since, triangles A'B'O and ABO are s~ar, so Wf: have - Also, M'B'F arid ~OF are similar
A'B' OB' A'B' FB'
-=- ... (1) So,--=-
AB OB OM ·OF
Also, triangles A'B'F and MOF are similar, so we have Since OM= AB, therefore
1
A'B' B'F •

OM OF

= :::::=================================
1.68

www.puucho.com
Rahul Sardana with www.puucho.com

Ray.Optics: Refraction at Curved Surfaces

A'B' FB'
--=- ... (2) =>
-v = -f +v
AB OF -u -f
From (1) and (2), we get => vf=uf-uv
OB' FB' OF-OB'
=> uv=uf-vf
OB OF OF
Dividing both sides by uvf , we again get
Using new Cartesian sign convention, we get
OB=-u, OB'=-v, OF=-/ 1 1 1
I V u

IMAGE FORMATION BY CONVEX LENS

OBJECT NATURE AND SIZE OF


DIAGRAM POSITION OF IMAGE
POSITION IMAGE

F 2F At the principal Focus Real, inverted and


At infinity
2F (F) or in the focal plane extremely diminished

F 2F Real, inverted and


Beyond 2F Between F and 2F diminished

Real, inverted and of


At 2F At 2F
2F same size as the object

Between F Real, .inverted and


Beyond 2F
and 2F 2F magnified

Real, inverted and highly


At F At infinity
2F magnified

1.69 =
www.puucho.com
Rahul Sardana with www.puucho.com

Advanced JEE Physics Optics & Modern Physics

OBJECT NATURE AND SIZE OF


DIAGRAM POSITION OF IMAGE
POSITION IMAGE

Between F
2F On the same side as the Virtual, erect and
and optical
object magnified
centre 2F F

IMAGE FORMATION BY A CONCAVE LENS

I OBJECT
POSITION
DIAGRAM POSITION OF IMAGE
NATURE AND SIZE OF
IMAGE

For all hnages formed between Always forms a Virtual,


2F
positions of the optical centre of the Erect and Diminished
2F
object lens and the focus (F) Image

Illustration 52
A point object O is placed at a distance of 0.3 m ''
from a convex lens of focal length 0.2 m . It then cut into
'
o~-~-~-~-~---~-~-.ri:~t'_-_-_________ : ____ ~!3__ _
two halves each of which is displaced by 0.0005 m as ' '
shown in figure. Find the position of the image. If more ''
than one image is· formed, find their number and the
distance between them.
,_,__ V = 0.6 m -------+I

Since the triangles OL,,L2 and 01112 are similar. So, we have
0 j_ 2 X 0,0005 ffi
I,I, =OB= u+v
L,L, OA u
I,I, = 0.3 + 0.6 0.9 =3
L,L, 0.3 0.3
1+- 0.3 m --+1
=,, I,I, = 3(L,L,) = 3(2x 0.0005) = 0.003 m
Solution
Each part will work as a separate lens and will form its
Illustration 53
own image. For any part, we have u = ..:..Q.3 rn, / = +0.2 m .
An object is placed 45 cm from a converging lens of
1 1 1 focal length 30 cm . A mirror of radius 40 cm is to be
Therefore, from lens formula, - - - = - , we have
V U f placed on the·other si«:fe of lens so that the object coincides
1 1 1 with its image. Find the position of the mirror if it is
V 0.3' 0.2 · (a) convex?
=,, v=0.6m (b) concave?
Solution
So, each part forms a real image. of the point object O at
0.6 m from the lens, as shown in figure. The object and image will coincide only if the light ray
retraces its path and it will happen only when the ray strikes
the mirror normally. In -other words the centre of the
curvature of the mirror and the rays incident on the mirror
are collinear.

= ==================================
1.70

www.puucho.com
Rahul Sardana with www.puucho.com

Ray Optics : RefractioH at C11rued S11rfaces


(a) The rays after refraction from lens must be directed => V1 =120 cm
towards the centre of curvature of mirror at C . If x is
In the second case, shift due to the glass slab is given by
the separation, then for the lens
u=--45cm, v=x+40, /=30cm fil=(1-f;c}=(1-/8 }=4 cm

r_ _ _ i.e., u will now become ( 40-4) = 36 cm, so now we have


1 1
-+-=-
1
0, I
t----c =>
v, 36 30
v2 =180 cm
Therefore, the screen will have to be shifted 60 cm away
1+- 45 cm-+t---- x - - - 4 0 cm+i from the lens.
1 1 1
Using lens formula - - - =-
V U f Illustration 55
1 1 1 Find the distance of an object from a convex lens of
----=-
x+40 --45 30 focal length 10 cm if the image formed is two times the
45 3 size of object. Focal length of the lens is 10 cm .
X= ( 0) -40=50cm
45-30 Solution
(b) In case of concave mirror, the refracted rays from lens A convex lens forms both type of images, real as well as
virtual. Since, nature of the image is not mentioned here, so
meet at C, the centre of curvature (C) of the mirror.
we will have to consider both the cases.
CASE I : When image is real
In this case v is positive and u is negative with
lvl =2lul, so if
0, l u=-x then v=2x and f =10 cm
. . . 111
Sb
u stitutmgm ---=-,we get
V U f
1+-45cm-+1+-----x----~
1 1 1
1 1 1 -+-=-
Using lens formula ---=- where u=-45cm, 2x X 10
V U f 3 1
v=x-40, /=30cm,weget 2x 10
1 1 1 => x=l5cm
x,-40 --45 30 x = 15 cm, means object lies between F and 2F.
x- 40 = 45x30 CASE II : When image is virtual
45-30 In this case v and u both are negative. So if
x=90+40=130 cm u=-y then v=-2y and f=lOcm
. . . 1 1 1
Su bstitutmg m, - - - = - , we get
Illustration 54 V U f
A lens with a focal length f = 30 cm placed at a 1 1 1
distance of a= 40 cm from the object produces a sharp --+-=-
-2y y 10
image of an object on the screen. A plane parallel plate
with thickness of d =9 cm is placed between the lens and 1 1
the object perpendicular to the optical axis of the lens. 2y 10
Through what distance should the screen be shifted for the y=5cm
image of the object to remain distinct? The refractive index
of the glass of the plate is µ = 1.8 . y = 5 cm, means object lies between F and P .

Solution
LINEAR MAGNIFICATION (m)
In the first case,
The linear magnification (also called lateral or transverse
1 1 1 magnification) m produced by a lens is defined as the ratio
-+-=-
v, 40 30 of the height of image to the height of the object. So,
1.71 =
www.puucho.com
Rahul Sardana with www.puucho.com

Advanced JEE Physics - Optics & Modern Physics

Height of Image I
m {·: f = cons_tant}
Height of Object 0
Since triangles ABC and A'B'C are similar, so
( dv)=(~)du
dt u' dt
.. ,(1)
A'B' CA'
AB CA Further, substituting proper values in lens formula, we get
Using Conventions, we get 1 1 ·1
-+-=- {': u=--OA m__ ,/=0.3 rn}
A'B'=-l V 0.4 0.3
AB=O => V=1.2 rn
CA=-U Substituting the values in equation (1 ), we get
...CA'=+v Magnitude of rate of change of position of image is
I V
- dv = 0.09 rns-1
=> --=-
0 -u dt
- - . u ----,--v -
I V V
=> ni=-=- Lateral magnification, m =-
0 u u
Please note that for both the lens and mirror we have Rate of change of lateral magnification is given by
mreal = NEGATIVE i.e. mroal < 0 dv du
u---v- (--0.4)(0_09)-(1.2)(0.01)
dm dt dt
= POSITIVE ;> 0
mvirtual i.e. mv1rtua1
dt u' (0.4)2 -
dm '
CONCEPTUAL NOTE($)
- =--0.3 per second
dt
a) Linear/Transverse/Lateral Mcmn_lfication proauced by a
lens'is'
=> Magnitude of rate·of change of lateral magnification is
I
·m----------
0
V f
u f+u
f-v·
f
dm .
=
dt 0.3 per seco~d
where I. is size ·of image perpendicular to Principal Axis
~nd O is size of object perpendlcular to principal Axis. Illustration 57
b) Axial Magnification :· Axial magnification iS the ratio of the
size of image along the principal ·axis to the size of the
Two thin convex·lenses of focal lengths f, and f, are
object along the ·pril1cipal axis. SO separated by a horizontal distance d (where d < f, , d < f, )
m ·= Size of Image along Principal Axis dv and their priitcipal axes are separated .by a vertical distance
axiai Size of <;)bject along Principal Axis du b as shown in the figure. Taking the centre of the first lens
dv v2 2 ( 0) as the origin of co-ordinate system and considering a
m?XJa1 = du =u2=m . parallel beam of light coming from theieft, find the X and
c) Areal Magnification : Areal fl'lagnificatiDn is the ratio of - y-coordinates of the focal point of'this lens system.
the area of_ imaJ~e to the area·of object. ' y
m . _ Area of Image_ .A 1 L,
areat Area of Object A0 I
L,
2 I
A1 V 2
mareai =A='if"=m
0 tb X
C
0 I
Illustration 56
-
11/
An ·Object is approaching a thin convex lens of focal .
'
length 0.3 m with a speed of 0.01 ms-1 • -Find the d
magnitudes of the rates of change of position and lateral
Solution
magnification of image when the object is at a distail~e of
0.4 m from the lens. Fo:r; the refraction through the firs~ lens, . We have
u-+oo,so
Solution
0 v, = I,
1 -1 1 "h
. . the 1ens formµ 1a - •.
Differentiating - - = - wit respect Since, d < f, , the first image (formed by L,) lies to the right
V u I
to time, we get of second lens L2 , so

= ====================================
1.72

www.puucho.com
Rahul Sardana with www.puucho.com

Ray Optics: Refraction at Curved Surfaces

u, =+(!, -d) This is called Newton's formula.


where x1 ~ F1 O = . distance of object froni. first principal focus
1 1
Applying Lens Formula - - - =-1 , we get 1\
V U f
1 1 1 x2 = F2 I = distance of image from second principal focus F2
v,-(11 -d)=f,
LENS MAKER'S FORMULA FOR THIN LENS
V f2U1 -d)
Consider a thin Jens having its optical centre at C and let 0
2 f,+f,-d be the point object situated on its· principal axis as shown in
~ x=v,+d f,f,+d(f,-d) figure. Light starting from O strikes the first surface of t\te
f, + f,-d lens at A 1 and heads towards I1 , however, refraction takes
place at the second surface, thereby giving a final real image
Magnification for second Jens is given by
at I.
m v, !,
u, !, + f,-d µ,
The image due to the second Jens is formed below its
principal axis and is of the size mb . So, the y coordinate of
the focal point system is given by
0
-- I,
y=b-mb
i.-R,+--<-t
,~ y=b f,b ,..___ u-~--1--i--+---
f,+f,-d
' (!, -d)b
~ y=
f,+f,-d Consider refraction at the first surface only. Let a 1 , P, and
So, the coordinates of the focal point of this system are y1 be the angles which the incident ray (OA1 ), refracted ray

(x, y )-_rl1,1,+a(f1 -a) ~


(11--d)b]
~-~~~, ~- ( A1I 1 ) and normal ( A1C1 ) make with the principal axis.
f, + f, -d f,+f,-d According the general law of refraction, applied at A,, we
get
NEWTON'S FORMULA µ 1 sin i1 =µ 2 sin r1
If the distapce of object and image are not measured from Since the angles are smal), so sini1 ~ i1 ·and sinr1 =r1
optical centre ( C) , but from ·first and second principal foci
~ µ1i1 = µ 2 r1 ••• (1)
respectively, and if x 1 is the distance of the object from the
first focus x2 is the· distance of the image from the second ·In A.4.iC1O, we have it =rt +at
focus and_ if f is the focal length of the lens, then we have In M 1C1I,, we have y1 = r1 + P
u=-(f +x,),_ v= f +x, => rt=Yt-P1
Substituting for i1 and r, in equation (1 ), we get
µ1 (r1 +a 1 )=µ 2 (r1 -P,)
0 ~ µ,p, +µ,a,=(µ, -µ,)y,
Since the angles are small, so they can be replaced by thej.r
tangents.

,.___U--<M+--V----+I
=> µ 2 tanj3 1 + µ1 tana 1 =(µ 2 -µ~)tanyt

According to the lens formula, we have


1 1 1 ·
=> (A,M,) + µ1 (A,M,)
µ2 M1J~
( )(A,M
M10 = µ2 -.µ1 .M1C1.
1)

---=-
v u f For· a thin lens, Mt lies close to C . Therefore, all ,the
distances measured from M1 can be replaced by those
1 1 1
measured from C . Hence we have
f +x, -(f +x,) f
... (2)
~ x,x, =('

==================================== = 1.73

www.puucho.com
Rahul Sardana with www.puucho.com

Advanced JEE Physics Optics & Modern Physics.

Now consider r~fraction at the second surface. The ray


A 1A, which is the refracted ray for first surface becomes the l.=('µ,-1)(1-_1-) ... (6)
I R, R,
incident ray for second surface. Applying general law of
refraction at A, (light going from denser to rarer medium), If the first medium is air, then 1 µ 2 =µ,so we have
we get
l. =(µ-1)(1-_1-)
µ 2 sin i2 =µ 1 sin r2 I R, R,
::::::,. µ2i2 = µ1r2 ... (3) This formula is called Lens Maker's Formula .
In M,C2I1 we have i, = r2 + P1
In M,C,I, we have r, =r, + P, I PROBLEM SOLVING TRICK(S)
I a) A concave lens forms virtual, erect and diminished image.
Substituting for i, and r, in equation (2), we get
µ,(r, +P1)=11t(Y, +P,)
I b) A convex lens may form real and virtual images. The real
image Is inverted, it may be diminishe9 or magnified while
virtual Image (armed by convex lens is erect and enlarged.
=> µ1P,-µ,P1 =(µ,-µ1)r, c) Different Media 0'1 either side of Lens
1f a lens of refractive index µ 2 has different media an either
Since angles ·are small so replacing the angles by their
1 I
tangents, we get side, the medium of object space has refractive index ,µ1 I

µ1tanp, -µ 2 tanp1=(µ, - µ1)tany2 and that of image space has refractive index µ 3 , then focal
length f of lens is

::::::,. µ1 (A,M,)
Mi -
(A,M,)
M2I1 =
( J(AM2C2
µ2
M,) ~2 - µ1
2
µ3 =µ2-~+µ3-~
f R, R,
Since the lens is thin, M 2 lies close to C , so we get As a special case if we put µ 3 =µ 1 , we get the Lens
Maker's Formula.
b. - l'L = µ, - l't ... (4)
CI CI1 CC,
Adding equations (2) and (4), we get Illustration 58
12_ + l::L +l:!_ _-12. = µ2 - µ1 + µ2 - µ1 A piano-convex lens has a thickness of 4 cm. When
CI1 CO CI CI1 CC1 CC, placed on a horizontal table, with the curved surface in
contact with it, the apparent depth of the bottom most ·
point of the lens is found to be 3 cm . If the lens is inverted
µ,(c~ +;I)=(µ,-µ,i(c~, +c~J such that the plane face is in contact with the table; the
apparent depth of the centre of the plane face is found to
25
be cm . Find the focal length of the lens. Assume
8

;I
c~ + =(~:- 1)( c~1 +c~J
thickness to be negligible while finding its focal length.
Solution
When placed on a horizontal table with curved
c~ +;I =(l,, -l)( c~, +c~J surface in contact with it~
In this case refraction of the rays starting from O takes place
Applying sign convention, we have from a plane surface as shown in Figure I.
CO=-u, CI=+v, CC1 =+R1 , CC2 =-R2
v
:u +~=(l,, -1)(~ +-~J
f
1 1
;---;;-=(l,, -1) (1R, - R,1)
Since, focal length of a convex lens is defined as the distance
of that point from the centre of lens where a beam coming
... (5)
4cm
! 'C7 •
0
Figure I
parallel to principal axis comes to focus after refraction So, we can use
through the lens, so when
Real Depth
U-)-oo we have p=f Apparent Depth
µ
Substituting in equation (5), we get

= 1.74 i:::::=================================
www.puucho.com
Rahul Sardana with www.puucho.com

Ray Optics: Refraction at Curved Surfaces

4 Solution
=> µ=-
3 (a) Image of object will coincide with it, if the ray of light
When the plane surface is in contact with the horizontal after refraction from the concave surface falls normally
table. on concave mirror so formed by silvering the convex
In this case refraction takes place from a spherical surface as surface i.e., image after refraction from concave surface
shown in Figure IL should be formed at centre of curvature of concave
mirror or at a distance of 20 cm on same side of the
combination. Let x be the distance of pin from the

4lm0
given optical system.
Applying, &,_& = µ, R- µ, , we get for
V II

v=-20 cm, U=-x, R=-60 cm


0 1.5 1 1.5-1
---=
Figure II -20 -x -60
Hence, applying 1 3 1 8
~-µ 1 =µ 2 -µ 1 ,we get X 40 120 120
V u R 120
=> X=-=15 an
4 4 8
1
1 3 -3 (b) When the concave part is filled with water, then before
~ - --4 = -R striking with the concave surface, the ray is first
8 refracted from a plane surface. So, let x be the distance
1 1 8 1 of pin, then the plane surface will form its image at a
=> -=---=-
3R 3 25 75 4
distance of liapp = µh i.e., x from it.
=> R=25 cm 3
Now, to find the focal length we will use the lens maker Now, using 1:2. _ ~ = µ 2 - µ 1 with proper signs, we
formula: V u R
get
..!:.=(µ-1)(2.._2..) 4 4
f R, R, 1.5--
15. 3 3

7=(¾- 1)(~- -~5)=;5


-20 -(¼x) =-=w

/=75 cm 1 -3 1 -26
-=-+-=-
x 40 360 360
Illustration 59 360
X= =-13.84 an
The convex surface of a thin concavo-convex lens of
-26
glass of the refractive index 1.5 has a radius of curvature
20 cm . The concave surface has a radius of curvature LENS IMMERSED IN A LIQUID
60 cm . The convex side is silvered and placed on a If a lens (made of glass) of refractive index µ 8 is immersed
horizontal surface. in a liquid of refractive index µ 1 , then its focal length in
''
'' liquid, f, is given by
'
'

:s:±2:,
..!:.=('µ,-1 )(2-_2..)
f, R, R,
If 10
is the focal length of lens of air, then

(a) Where should a pin be placed on the optic axis such ]__=( oµ,-1 )(2-_2..)
that its image is formed at the same place? lo R, R,
(b) If the concave part is filled with water of refractive
index 4/3 , find the distance through which the pin
should be moved, so that the image of the pin again
!,=[~:=: ]lo
coincides with the pin.
Now three cases arise which are discussed here.
1.75 =
www.puucho.com
Rahul Sardana with www.puucho.com

Advanced JEE Physics Optics & Modern Physics

a) If µ, > µ1 , then f, and /, are of same sign and f, > f, . 1 1 1


---=- ,-u --1+---V-
That is the nature of lens remains unchanged, but its V u f
focal length increases and hence power of lens 1· 1 1 zw
~ ----=- F,
decreases. In other words the convergent lens becomes D-u -u f -~-+--+-t-------t ~
less convergent and divergent lens becomes less 0 0
divergent. 1 1 1 (/)
~ --+-=-
b) If µ g =µ 1 , then f, ~ co and the lens behaves as a simple D-u u f
.
I+----- D----~
glass plate. ~ Df=(D-u)u
c) If µ, < µ, , then f, and /, have opposite signs and the ~ u 2 -Du+Df=0
nature of lens changes i.e. a convergent lens becomes 2
divergent and vice versa. D±~D -4JD
~ u
2
For u to be mathematically real,
Illustration 60
A lens has a power of +5 dioptre in air. Calculate _its D 2 -4JD?:0
~ D?:4/
power if it is completely immersed in water? Given µg =·~.
2
4
and µw= . PROBLEM SOLVING TRICK(S)
3 ISo, if the,object and the screen are placed at a distance less than
Solution j 4f, then a virtual image will be formed. Hence, for a real image

Let /, and fw be the focal lengths of the lens in air


I to be formed D _2>: 41 ~

and water respec_tively, then


Casel:For D=4/
P =_l and P10 =µ·
afa fw D
U=V=-
Since lens has power +5 D in air, so 2
i.e. the lens is placed exactly between the object and the
1 1
f, =p=s=0.2 m=20 cm screen.
CaseII:For D>4/
Using Lens Maker's formula, we get
We get two different position of lens (L1 and L2 ) for
_l =(µ,
f,
-1 )(_!_ __!_)
RI R,
... (1) which the image of object.on the screen is distinct and·clear.

Similarly,
.
_!_=(.t2-1)(_!_
!. µ.
__!_)
R, R,
14-U,---1--------V,----------<>I

~ pw = ;: =(µ,-µ.)(;! -;J ... (2)

Dividing-_equation (2) by equation (1), we get,


P. (µ,-µ.) 1
P, (µ,-1) 3

p =p'=+~D
w 3 3

DISPLACEMENT METHOD
Consider an object and a screen fixed at a distance D apart. L, First Position of Lens
Let a lens of focal length f be placed between the object and
L, Second Position_ol the Same Lens (shownin grey)
the screen. I
From figure we observe that ,i NOTE:. _
u+v=D ~ v=D-u / Do not Develop· a'rriisconceptl0:rfthat there are two lenses,
!· intact the same lens is displace~Jhrough x from position L1
Also from Lens formula I to L,.

= ====================================
1.76

www.puucho.com
Rahul Sardana with www.puucho.com

Ray Optics : Refraction at Curved Surfaces


The object distances for these two positions are given by i.e. size of the object (O) is the geometric mean of the sizes
D-JD'-4JD of the image for two position oflens L, and L2 •
... (1)
2 .' Also,
D+JD'-4JD D+x D-x
u, m,-m =-----
... (2) 2 D-x D+x
'2
Since u+v=D, so 4Dx ,
m1-m2=-D,
-x
D+JD'-4JD
v, ... (3) X
2

D-JD -4JD 2 (D:;x')


v, ... (4)
2 X
m, -m, =-
We observe that· !
u, =V2 = u(say) ... (5) •• X
=> f=-- .. ,(15)
v1 =u, =v(say) ... (6) m1-m2

Let the Jens be displaced through x, then we observ~ fro\11 Further if m, = m , then m, = ..!_
figure that , m

x=v1 -u,=JD'-4JD .... (7) f =...!!!!_ ... (16)


m 2 -1
2
=> x =D'-4JD Finally, we observe that

"" I D I .. '.(S) m1 =(D+x)' ... (17)


m2 D-x
Using (7) in (1), (2), (3) and (4), we get
'D-x PROBLEM SOLVING TRICK(S)
U1 =V2 =-2-=U ... (9) Dear Students, you must J(eep in mind that actually
"Displacement Method" is not in the syllabus, but the Examiner
D+x generally· askS the problems not in its name ·but' by its concept_
V1=U2=:=-2-=V ... (10)
e.g. an examiner's mind may fabricate a problem not having the
name Displ~cement method but then the problem must be having
If m1 is the magnification for the first position of lens i.e. ~1 ,
a clue which may state D > 4f or the lens is displaced to get'two 1
then real images on Screen and stuff like that. So, you are advised not I
to overlook the.topic as this is very important (not by name) but'\
m, _b,_5.._!:,_ D+x ... (ll) by the concept involved.
0 u1 u D-x
If m2 is the magnification for the second position,of the Lens
Illustration 61
i.e. L2 , then
A thin converging lens of focal length f is moved
I v u u D-x
m, = 2- = -
2 1
=- =-=-- ... (12) behveen a candle and a screen. The distance behveen the
0 u2 v1 v D+x candle and th~ screen is J?(> 4/). Show that for two
From (11) and (12), we observe that different positions of the lens, two different images can be
,m,m,=1 , ... (13) obtained on the screen. H the ratio of dimensions of the
SO, if magnification for position L1 , is m , then maghification image is p, fi~d the value of (P +½)- _
. . L2 1s
for position . _;_
1 .
m Solution
Also from (13), we get Let x be the separation between two positions of the
lens for which a real image is formed on ·the sc;reen.
b_ I, =1
00 Then, v+u=D ... (1)
=> 02 =1112 and v-u=x ... (2)
. D-x D+x
=> 0=.,JY;i; ... (14) Solvmg we get u = - - and v = - -
2 .2

1.77 =
www.puucho.com
Rahul Sardana with www.puucho.com

Advanced JEE Physics Optics & Modern Physics

I D+x => 5u(u-4)-5(u-4)=0


Now, m1 =...1..=--
0 D-x => (Su-5)(u-4)=0
I D-x => u=1m and u=4m
and m, -..1.=--
- 0 D+x
Both the values are real, so this m~ans that there exist
two positions of lens that form images of object on the
=> .!i_=(D+xJ =~ screen.
I, D-x
V
D+x=..ffi (b) m=-
=> u
D-x
(5-4) (5-1)
m, = -(--) = -0.25 and m, = _(_)_ = -4
=> 1-4 ~
x=(j:~)D
Hence, both the images are real and inverted, the first
has magnification -0.25 and the second -4 .

Now f=D'-x' =
D' l'
._ffi - 1
- ~-
v)' D
Also, we observe that
, 4D 4D m,m, =(-0.25)(-4)=1

Illustration 63
For two positions of a converging lens between an
object and a screen which are 96 cm apart, two real images

~+Jl=1 (D7-2)' -2 are formed. The ratio of the lengths of the two images is 4.
Calculate the focal length of the lens.
Solution

Illustration 62 Since, m1 =4
An object is 5 m to the left of a flat screen. A
m,
converging lens for which the focal length is / = 0.8 m is
placed between object and screen. (~~:)' =4
(a) Show that two lens positions exist that form images D+x = .
on the screen and determine how far are these 2
D-x
positions from the object?
(b) How do the two images differ from each other? Substituting D = 96 cm, we get
Solution 96+x =2
96-x
(a) Using the lens formula .!. _.!_=.!.,we get => x=32 cm
V u I
D2-x2
f=O.Bm Since, f = ~
B
=> f = 21.33 cm

A POWER OF A LENS
The power of a lens P is actually the measure of its ability to
deviate the incident rays towards axis. The greater the
1+-- u ---- 5 - u
curvature of the two surfaces (i.e., the shorter the focal
1 1 1 length / ), the greater is the lens action. The shorter the
----=-
5-u -u 0.8
focal length of a lens the more it converges or diverges the
1 1 5 light, as shown in figure.
=> --+-=-
5-u u 4
5-u+u 5
(5-u)u 4
=> 20 = 25u-Su'
=> 5u2 -25u + 20·= 0
=> 5u 2 -20u-5u+20=0 I, ,,
= 1.78 c::===================================
www.puucho.com
Rahul Sardana with www.puucho.com

Ray Optics : Refraction at Curved Surfaces

The power ofa lens placed in air is actually the reciprocal of Solution
the focal length of the lens in metre and is given by Here P1 =6D, P2 =-2D
P= 1 100 Power of the combination is given by
f(in metre) f(in cm)
P=P1 +P, =6-2=4 D
SI wlit of power is dioptre (D) .
Power of a lens placed in a medium is defined as Since/=½

P.,=L 1
m /med =:> /=-=0.25m=25cm
4
where µ is the refractive index of the medium and /med is
the focal length of the lens in that medium. Illustration 65
As a convention, the power of a converging lens (or convex
An object 4 cm high is placed at a distance of 10 cm
lens) (with focal length positive) is taken to be positive. The
from a convex lens of focal length 20 cm . Find the
power of a diverging lens (or concave \ens) (with focal
position, nature and size of the image. Also find the power
length negative) is taken to be negative.
of the lens.
AlsO we must note that for a mirror, power is defined as
Solution
p _ __::i__ 100
Here, u = -10 cm (the object assumed to be kept to the
/(inmetre) /(incm) left of optical centre)
Thus a convex lens and concave mirror have converging f = +20 cm (positive for a convex lens)
nature and hence they have poSitive power, whereas the
concave lens and convex mirror have diverging nature and h,_ = +4 cm (object kept above the principal axis)
hence have negative power. Using the lens formula, we get
1 1 1
---=-
Power v u f
Nature Focal 1
of Lens Length
plens = f, Converging =:,
1 1 1
Ray Diagram
/ Diverging V -10 20
Mirror (f)
pmlrrcr =-r1 =:, v=-20 cm
That is, the· image is 20 cm from the lens, on the same side

~
as the object. Hence, the image is virtual. The linear
Concave -ve +ve Converging magnification,
mirror
h V
m=i=-
h, "

~
So, size of the image is
Convex
+ve +ve Converging
lens 2
h, =h 1 (!:.)=4x - 0 =8 cm
u -10
The positive sign indicates that the image is erect (and

Convex
mirror
+ve -ve

'
Diverging
=;t virtual).
Since, the power of the lens is given by
p 1
/(in m)
1 =+5 D
+0.2

Concave
lens

Illustration 64
-ve -ve Diverging

=f- Illustration 66
A converging lens forms a five folds magnified image
of an object. The screen is moved towards the object by a
distance d = 0.5 m and the lens is shifted so that the image
has the same size as the object. Find the lens power and the
initial distance behveen the object and the screen.
A convergent lens of power 6 D is combined with a Solution
diverging lens of -2 D . Find the power and focal length of In the first case image is five times magnified. Hence
the combination. lvl=5lul
1.79

www.puucho.com
Rahul Sardana with www.puucho.com

Advanced JEE Physics Optics & Modern Physics

In the second case image and object are of equal size. Hence Here too, P11 P21 .••.. are to be substituted with proper signs
lvl=lul attached.
The magnification of the combination is

M=m 1 xm 2 x ... =t'rm


j,.]

In many optical instruments, the combination of lenses in


1+-X-+++-----Sx-------.l contact are uSed so as to improve the performance of ·the
CASE-I instrument.
''
'
d=O.sm:'

1+---Y----<---Y-----++
CASE-II
From the two figures, we get Consider two lenses of focal lengths I, and I, kept in
6x=2y+d contact. Let a point object O be placed at a distance u from
=> 6x-2y=0.5 ... (1) the combination. The first image (say I1 ) after refraction
Using the lens formula for both the cases, we get for from the fjrst lens is formed at a distance v1 (whatever may
CASE-I, be the sign of v,) from the combination. This image I, acts
1 1 1 as an object for the second lens and let v be the distance of
---=-
5x -x I the final image from the combination. Applying the lens
1 1 1
6 1 formula---=-, we get
-=-
5x I
... (2) V u I
CASE-II, For the first lens, _!_ _ _! = _!_ ... (1)
1 1 1
V1 U /1
---=-
1 1
y -y I and for the second lens, - - - =-1 ... (2)
2 1
V Vt /2
=> -=- ' ... (3) Adding equations, (1) and (2), we get
y I
1 1 1 1 1
Solving these three equations, we get ---=-+-=- (say)
x = 0.1875 m and I= 0.15625 m vuf,l,I
Therefore, initial distance between the object and the screen
where, I is the equivalent focal length of the combination.
is
6x=l.125 m 1 1 1
-=-+-
1 1 F fi, I,
Power of the lens, P = - = - - - D = 6.4 D
I 0.15625
TWO THIN LENSES SEPARATED BY A DISTANCE
LENSES IN CONTACT If two thin lenses of focal lengths Iv I, are placed at a
If two or more lenses of focal lengths Iv I,, ... are placed in distance x apart, then equivalent focal length · of
contact, then their equivalent focal length / is given by combination is
1 1 1 X
-=-+----
.!=_!_+_!_+ ... =
I I, I, .
i;.!.
,., J;,
F I, I, Iii,
or Power for the combination is
Where / 1 , / 2 , •••• are to be substituted with proper signs
P=P1 +P2-XP1P2
attached.
The net magnification of the combination is still remains
The power of combination
m=m1xm2
P=P,+P,+ ... =:tP;
i~t

= ==================================
1.80

www.puucho.com
Rahul Sardana with www.puucho.com

Ray Optics: Refraction at Curved Surfaces

Illustration 67 (d) When d = 2/: The incident parallel beam emerges out
Consider a co-axial system of two thin convex lenses as a parallel beam but inverted. The combination
of focal length / each separated by a distance d . Draw
ray diagrams for image formation corresponding to an
object at infinity placed on the principal axis in the
following cases: (a) d < f (b) d =f (c) f < d < 2/ (d)
-r-
behaves as a plane glass slab, which inverts the beam.

d = 2/ and (e) d > 2/. Indicate the nature of the


combination (concave, convex or plane) in each case.
Solution
The formula
(e) When d > 2 /: The incident parallel beam emerges out
1 1 1 d
-=-+---- as a convergent beam. The combination behaves as a
F /1 /, f,f,
is valid only for small values of d compared to / 1 and /,.
Therefore, we cannot use this formula in the given cases.
However, we can draw the ray diagram to decide the nature.
-r-
convergent or convex lens.

of the combination.
(a) When d < f : The ray diagram is shown in figure. The
out-coming rays are convergent. Obviously, the
combination is a convex lens with F < f.

Illustration 68
Two equi-convex lenses of focal lengths 30 cm and
··-····--... 70 cm , made of material of refractive index =1.5 , are held
----------· in contact coaxially by a rubber band round their edges. A
liquid of refractive index 1.3 is introduced in the space
between the lenses filling it completely. Find the position
---F--....., of the image of a luminous point object placed on the axis
of the combination lens at a distance of 90 cm from it.
(b) When d = f : The incident parallel beam converges to
Solution
a point and then passes without any more deviation.
The combination behaves like a convex lens of F = f . According to Lens Maker's Formula, we have
J_= (1.5-l)(J_ __
30 R1
l_)
-R1
=> R1 =30cm
Similarly, radius of curvature of the second lens is 70 cm .
Since
1 1 1 1
-=-+-+- ... (1)
----F---- F /1 f, f,
Here, / 1 =30 cm,/, =70 cm
(c) When / < d < 2/: The incident parallel beam emerges
out as a divergent beam. the combination behaves as a
divergent or concave lens.

Now / 3 is calculated again using the Lens Maker's Formula,


so we get
,.___F----+1 1
.!:..=(1.3-1)(---J_)
/, -30 70

1.81 =
www.puucho.com
Rahul Sardana with www.puucho.com

Advanced JEE Physics Optics & Modern Physics

=> I, =-70 cm P=-2_=3_-~


=> F =30 cm {from equation (1)) f /, Im
According to Lens formula, applied on the combination of where f, is focal jength of lens and Im is focal length of
lenses, we have spherical'rnirror formed due to silvering of surface.
1 1 1 To have · a fundamental understanding of this we Can
---=-
v u F understand the silvering of lenses using the following'
1 1 1 arguments.
; - (-90) = 30 A ray incident on a lens with its backside· silvered will be
refracted through the lens twice and will be reflected from
v=45 cm
the mirror once, as shown.
a) Light from object O passes through lens ·10 form image
LENSES WITH ONE SILVERED SURFACE
I,.
When one face of a lens is silvered as shown in figure it acts
like a lens-mirror combination. b) The image ! 1 acts as an object (virtual) for the curved
mirror to form image 12 •
c) The image I2 acts as an object (virtual) for the lens· to
form the firtal image I •

=
Lenses with one face silvered act like lens-mirror combination
It is obvious from ilie ray diagram as shown in figur~ that
the incident ray of light is refracted through the lens twice
(i.e., once when light is incident on the lens and second time
when reflected by the mirror) and reflected from the m!rror
once.

+ +

The silvered lens acts like a mirror with equivalent focal


length F , given by '
The combination acts like a mirror whose effective power is 111121
given by ------+-----
F /, Im f, f, Im
P,,e1 =2Pi+Pm
where I, is focal length of lens and Im is focal length of
whei-e Pi is the power of the lens and Pm is the power of the spherical mirror formed due to silvering of surface.
mirror.
Since for a mirror we have SIGN CONVENTION
p =-~ While using the above formula, we make use of tti_e following sign
m Im conventions.
a) f is positive for converging (convex) lens and- concave
and for a lens, we have mirror.
b) f Is negative for diverging (concave) lens and convex
P, =2_=(µ-1)(~-~) mirror.
f, R, R2
So, the combination acts like a mirror having net focal length
given by For example, for a plano-convex lens, from Lens Maker's
Formula we get
1
Fnet=--
pnel 2_ = (µ-1)(2--2-) = µ-1
f, Rao R

= ==================================
1.82

www.puucho.com
Rahul Sardana with www.puucho.com

Ray Optics: Refraction at Curved Surfaces

R 0.03 m and when viewed normally through the curved


=> /,=-
µ-1 surface it appears to be 0.036 m . If the actual thickness is
a) when plane surface is silvered, fm-+ co 0.045 m , find the
(a) refractive index of the material of the lens.
Since we know that
(b) radius of curvature of lens.
1 2 1 2(µ-1)
--=--- (c) focal length if its plane surface is silvered.
F f, "' R
(d) focal length when the curved surface is silvered.
R Solution
=> F
2(µ-1)
Real Depth
(a) Since, µ
. b) when convex surface is silvered, then in general we Apparent Depth
know the relation between radius of curvature and the
focal length is given by
R
=> µ= d-.
dapp
= 0.045 =1.5
0.03
fm=z
Since we know that
1 2 2 2(µ-1) 2 2µ
--=-+- +-=-
F f, R R R R
=> F=-~

Illustration 69

(b) Using1 .!::,__b_=µ,-µ, weget


-
0.045 m

The plane surface of a plano•convex lens of focal V U R I

length 60 cm is silver plated. A point object is placed at a 1 1.5 1-1.5


distance 20 cm from the convex face of lens. Find the (-R)
(-0.036) (-0.045)
position and nature of the final image formed.
=> R=0.09 m=9 cm
Solution
Since, P = 2Pi + Pm
1 2 1
=> -I'= t,- Im
where, Ji = +60 cm and fm -+ co
1 2 1 1
=> =---=
F60oo30
(c) If the plane surface is silvered, then
=> F=-30 cm
1 2 1
The problem is reduced to a simple case where a point object -=-+-
F f, f.,
is placed in front of a concave (converging) mirror of focal
length 30 cm . But f," --> oo
R1 =+9 cm
Using mirror. formula i.e., 1 2
=>
1 1 1 F I,
-+-=-
v u f
where, 2.=(µ-l)(J__l_)
where u=-20cm and /=-30cm I, R, "'
1 1 1 1 2(µ-1)
=> -+-=- 1+-- 20 cm -----+f =>
v -20 -30 F R1
=> v=60 cm
1 2(1.5-1)
The image is virtual and erect =>
F +9
=> F=9 cm
Illustration 70 The nature is given by applying negative sign to the
The greatest thickness of a planoconvex lens when final result. So, this will behave as a concave mirror.
viewed normally through the plane surface appears to be

1.83 =
www.puucho.com
Rahul Sardana with www.puucho.com

Advanced JEE Physics Optics & Modem Physics

(d) When curved surface is silvered shows spherical aberration for a concave mirror for an object
then R, ----> oo , R, = -9 cm at infinity. The rays parallel to the principal axis are incident
on the spherical surface of the co_ncave mirror. The rays close
1 2 1 to the principal axis (Paraxial Rays) are focused at the
-=---
F f, fm geometrical focus F of the mirror. The rays farthest from the
principal axis (Marginal Rays) are focused at a point F'
=> _! = 2(µ-1)(~-_!_)-~ somewhat closer to the mirror. The intermediate rays focus
F ,oo-9-9
at different points between F and F'. Also, the rays
=> .!=2(µ-1)+3. reflected from a small portion away from the pole meet at a
F 9 9 point off the axis. Thus, a three- dimensional blurred image
_!=2µ is formed.
=> The intersection of this image with the plane of figure is
F 9
1 1.5x2 called the Caustic Curve.
~ -=-- If a screen is placed perpendicular to the principal axis, a
F +9
disc image is formed on the screen. As the screen is moved
=> /=3cm parallel to itself, the disc becomes smallest at one position.
This disc is closest to the ideal image and its periphery is
DEFECTS OF IMAGES : ABERRATIONS called the Circle of Least Confusion. The magnitude of
The simple theory of image formation developed for mirrors spherical aberration may be measured from the distance FF'
and lenses suffers from various approximations. As a result, between the point where the paraxial rays converge and the
the actual images formed 'contain several defects. These point where. the marginal rays converge.
defects can be broadly divided in two categories. The parallel rays may be brought to focus at one point if a
a) Monochromatic Aberration parabolic mirror is used. Also, if a point source is placed at
The defects, which arise when light of a single colour, is the focus of a parabolic mirror, the reflected rays will be very
used, are called monochromatic aberrations. nearly parallel. The reflectors used in automobile headlights
are made parabolic and the bulb is placed at the focus. The
b) Chromatic Aberration
light beam is then nearly parallel and goes up to large
The index of refraction of a transparent medium differs distance.
for different wavelengths of the light used. The defects
arising from such a variation of the refractive index are
termed as chromatic aberrations.

MONOCHROMATIC ABERRATIONS
F
A. SPHERICAL ABERRATION
Throughout the discussion of lenses and mirrors with
spherical surfaces, it has been assumed that the aperhlre of
the lens or the mirror is small and the light rays of interest A lens too produces a blurred disc type image of a point
make small angles with the principal axis. Only then, it is object (due to finite aperture of lens). Figure shows the
possible to have a point image of a point.object. sihlation for a convex and a concave lens for the rays coming
parallei'to the principal axis. We see from the figure that the
marginal rays deviate a bit strongly and hence, they meet at
a point different from that given by geometrical optics
formulae. Also, in the situation shown, the spherical
aberration is opposite for convex and concave lens. The point
FM, where the marginal rays meet, is to the left of the focus
for convex lens and is to the right of the focus for the
concave lens.

M-+---+. M--+-----\--1
P-+--1-...__·, P------A-;.,r
The rays reflect or refract from points at different distances
from the principal axis. In general, they meet each other at P-+--~rl-/ P----,---L
different points. Thus, the image of a point object is a blurred M-+----¥ M --+------f---¾.
surface. Such a defect is called Spherical Aberration. Figure

= 1.84

www.puucho.com
Rahul Sardana with www.puucho.com

Ray Optics: Refraction at Curoed Surfaces

The magnitude of spherical aberration for a lens depends on further above P1 • The image seen on the screen. thus have a
the radii of curvahrre and the object distance. For minimum comet-like appearance.
spherical aberration the ratio of radii of curvahrre of lens is
Image of P ~,
R1 2µ 2 -µ-4
R1 µ(2µ+1)
However, it cannot be reduced to zero for a single lens which
forms a real image of a real object.
A simple method to reduce spherical aberration is to use a R
stop before and in front of the lens. A stop is an opaque sheet Coma can be reduced by properly designing the radii of
with a small circular opening in it. It only allows a narrow curvature of lens surfaces. It can also be reduced by using
pencil of rays to go through the lens hence reducing the appropriate stops placed at appropriate distance from the
aberration. However, this method reduces the intensity of lens.
the image as most of the light is cut off.
Otherwise, the spherical aberration is less if the total C. ASTIGMATISM
deviation of the rays is distributed over the two surfaces of
Spherical aberration and coma refer to the spreading of the
the lens. Example for this is a planoconvex lens forming the
image of a point object in a plane perpendicular to the
image of a distant object. If the plane surface faces the
principal axis. The image is also spread along the principal
incident rays, the spherical aberration is much larger than
axis. Consider a point object placed at a point off the axis of a
that in the case when the curved surface faces the incident
converging lens. A screen is placed perpendicular to the axis
rays. In the former case, the total deviation occurs at a single
and is moved along the axis. At a certain distance, an
surface whereas it is distributed at both the surfaces in the
approximate line image is focused. If the screen is moved
latter case.
further away, the shape of the image changes but it remains
on the screen for quite a distance moved by the screen. The
spreading of image along the principal axis is known as
Astigmatism (you must not confuse this with a defect of
vision having the same name).

D. CURVATURE
So far we have considered the image formed by a lens on a
plane. However, it must be kept in mind that the best image
The spherical aberration can also be reduced by using a may not be formed along a plane. For a point object placed
combination of convex and concave lenses. A suitable off the axis, the image is spread both along and
combination can reduce the spherical aberration by perpendicular to the principal axis. The best image is, in
compensation of positive and negative aberrations. general, obtained not on a plane but on a curved surface.
If two thin lenses are separated by a distance d, then This defect is known as curvature. It is intrinsically related to
condition for minimum spherical aberration is astigmatism. The astigmatism or the curvature may be
d=f,-f, reduced by using proper stops placed at proper locations
along the axis.
B. COMA
E. DISTORTION
It has been observed that if a point cbject is placed on the
It is the defect arising when extended objects are imaged.
principal axis of a lens and the image is received on a screen
Different portions of the object are, in general, at different
perpendicular to the principal axis, the image has a shape of
distances from the axis. The relation between the object
a disc because of spherical aberration. The basic reason is
distance and the image distance is not linear and hence, the
that the rays passing through different regions of the lens
magnification is not the same for all portions of the extended
meet the principal axis at different points. If the point object
object. Hence a line object is not imaged into a line but into a
is placed away from the principal' axis and the image is
curve and shown.
received on a screen perpendicular to the axis, the shape of
the image is like a comet. This defect is called Coma. the lens
fails to converge all the rays passing at different distances
from the axis at a single point. The paraxial rays form an
image of P at P' . The rays passing through the shaded zone
forms a circular image on the screen above P' . The rays
through outer zones of the lens form bigger circles placed (b) (c)
Object (a) and its distorted Images (b) & (c)

1.85 =
www.puucho.com
Rahul Sardana with www.puucho.com

Advanced JEE Physics Optics & Modern Physics

CHROMATIC ABERRATION B. LATERAL CHROMATIC ABERRATION


The inability of a lens to form the white image of a white This is the spread of images perpendicular to principal axis
object is called chromatic aberration. In this case the lens and is given by
forms coloured images of a white object.
The chromatic aberration arises .due to the fact that the focal
I,-Iv = v,O _ VvO =(v,-vv
u u
)0 =(rov' )0
u f u
length of a lens depends upo~ the refractive index of
material of the lens. The lens has different refractive indices
for· different colours or wavelengths in accordance with ACHROMATISM & ACHROMATIC DOUBLET
Cauchy's formula given by The lens system free from chromatic aberration is called
B achromatic combination. This is obtained by using two
µ=A+,
). lenses of different materials and different focal lengths and
process is called, to Achromatise which satisfies the relation
Accordingly, the refractive index is maximum for violet
(). =4000A) and minimum for red (). =7800A) . Since ro,+ro'=O
f, f,
7=(µ-1i(~ -~) f, =- ro,
f, ro,
1
=> foc--
µ-1 where ro1 and ro 2 are dispersive powers of materials of

Hence focal length of a lens is maximum for red and


lenses for focal length f, and f, respectively.
minimum for violet a) As ro1 and ro, are always positive, therefore f,/ f,
=> freo > J,,,;,,~. must be negative. This means the combinatj.on mu~t
Figllre_ represents the chromatic aberration caused by a lens
have one lens convergent and other divergent.
in the image of an object AB of size O . b) For the achromatic combination (also called Achromatic
Doublet) to be convergent, the powCr of convex lens
FR and Fv are second principal foci for red and violet
must be greater or the focal length of convex lens must
colours respectively. The images of object AB are of be smaller than that of concave lens. As dispersive
different sizes and of different colours between AvBv and power for crown glass is less than that for flint glass,
ARBR . The chromatic aberration is of two types. therefore the convex lens must be made of crown glass
>+---V,---+< while concave lens must be made of flint glass.
14--.Vv~ Condition for minimum chromatic aberration obtained
by two thin lenses of same medium separated by a
distance d is
d~f.,+f,
2

DEFECTS OF EYE
A, A normal eye has nearer point at D ( 25 cm) called distance
Chromatic Aberration of distinct vision and far point at c.o.

A. SHORT-SIGHTEDNESS OR MYOPIA
A. AXIAL CHROMATIC ABERRATION
A short-sighted eye can see only nearer objects. It is due to
This is the spread of images along the principal axis and is elongation of eye-ball. It is corrected by using a concave lens,
given by whose focal length is ~qua! to the far point of defective eye.
rov'
VR-Vv =1 ~\_
where ro is dispersive power, v is distance of image from
lens for mean (yellow) colour and f is mean focal length of
lens.
Corrective
If object is at infinity, then axial chromatic aberration, DEFECTIVE EYE Lens
f,-fv=rof Image is not Created on
the Retina DEFECT CORRECTED

1.86 c:::========================================:::i

www.puucho.com
Rahul Sardana with www.puucho.com

Ray Optics: Refractio11 at Curved Surfaces


B. LONG-SIGHTEDNESS OR HYPERMETROPIA When a small object is placed between optical centre and
A long sighted eye can see only farther objects. It is due to focus of a convex lens, its virtual, erect and magnified image
contraction of eye-ball. It is corrected by using convex lens. is formed on the same side of the lens. The lens is held close
This lens brings the nearer point of defective eye at a to eye and the distance of the object is adjusted, till the image
distance which equals to the distance of distance of distinct is formed at the least distance of distinct vision from the eye.
vision D( = 25 cm). For a normal eye, the least distance of distinct vision is
25cm.
~\_
MAGNIFYING POWER

-~Tu Case I : When image is formed at D


It is defined as the ratio of the angle subtended by the image
I l
at the eye and the angle subtended by the object seen
Corrective directly, when both lie at the least distance of distinct vision.
DEFECTIVE EYE Lens
Image Js Created It is also called angular magnification produced by the
beyond the Retina DEFECT CORRECTED simple microscope. It is denoted b)' M.
By definition, magnifying power of the simple microscope is
C. PRESBYOPIA given by
A presbyopic eye can see objects only within a definite M=Q_
range. This defect is corrected by using bifocal lenses. a
In particle, angles a and p are small. Therefore, angles a
D. ASTIGMATISM and p can be replaced by their tangents i.e.
It arises due to distortion in spherical shape in cornea. This
·M = tanp =CA'=~
defect is corrected by using cylindrical lenses. tana CA u
If f is focal length of the lens acting as simple microscope,
OPTICAL INSTRUMENTS
then
An optical instrument is a device which is constructed by a
suitable combination of mirrors, prisms and lenses. The 1 1 1
---=-
principle of working of an optical instrument in based on the v u f
laws of reflection and refraction of light
=> ~=1-~
The common types of optical instrument are u f
a) Projection instruments: These are used to project on
the screen a real, inverted and magnified image of an => M=l-~
opaque or transparent object so as to be viewed by a f
large audience. The object is, however, so fitted that its B"
image is seen in erect form.
An eye, a photographic camera, a projection lantern, an
episcope, an epidiascope, an over~head projector, a film ... ____________ _
Q
projector, etc., are examples of projection instruments.
b) Microscopes: These are used to see very small objects in
magnified form which otherwise cannot be seen A' F" F
distinctly when placed close to the naked eye.
EXAMPLE 4
-u-
A simple microscope and a compound microscope.
c) Telescope: These are used to see astronomical and
distant objects in magnified form which, otherwise 1+--f-
cannot be seen clearly with the naked eye. ~ - - - D ----+<
EXAMPLE
Since, image is formed at distance of distinct vision, so
An astronomical telescope, a Galilean telescope, a
according to new Cartesian sign convention.
terrestrial telescope, a reflecting telescope, etc.
v=-D and f=+f
SIMPLE MICROSCOPE (MAGNIFYING GLASS) D
=> M=l+/
A convex lens of short focal length can be used to see
magnified image of a small object and is called a magnifying
glass or a simple microscope.
1.87 ~

www.puucho.com
Rahul Sardana with www.puucho.com

Advanced JEE Physics Optics & Modern Physics

" CONCEPTUAL NOT!=(S) .· b) In science laboratories, a magnifying glass is used to see


a) From above it follows that lesser'is the focal length of the slides and to read the vernier scales attached to the
convex. lens used as simple microscope, greater is the· instruments.
value of the magnifying power obtained. c) The use of magnifying glass enables us to place the
b) Further, the positive value of magnifying power of a simple object close to eye, making it appear bright and yet
microscope tells that .image formed is erect and hence
virtuat.
clearly visible. In position AB, object lies close to the
eye. In absence of lens, the object will not be clearly ·
visible.
Case II: When image is formed at infinity d) It is also used by astrologers to read the fate lines of the
It is defined as the ratio of the angle formed by the image hand.
(situated at infinity) at the eye to the angle formed by object e) Used by Biology students to see slides.
at the eye, when situated at least distance of distinct vision. It
f) Used by detective department to match finger prints.
is denoted by M .
Illustration 71
A man with normal near point (25 cm) reads a book
with small print using a magnifying glass (a thin convex
lens) of focal length 5 cm. Find the
(a) closest and farthest distance at which he can read the
book when viewing through the magnifying glass.
(b) maximum and minimum maguifying power possible
I+-- u = f-----+-i
using the above simple microscope.
i.----D---- Solution
Draw a line· A'B' = AB and perpendicular to principal axis at (a) For a normal eye, far and near points are co and
a distance CA'= D (least distance of distinct vision). Joint 25 cm, respectively. So, we have
B' C . Then LB' CA' = a is the angle formed by object at the Vmax -> --<0 and Vmm =-25 cm
eye, when situated at distance D . , 1 1 1
The angle formed by the image situated at infinity at the eye Using lens formula,---=-
v u f
is same as the angle formed by the object AB at the eye.
Thus, LBCA = P is the angle formed by the image at the eye, U=-/-
By definition, (f)-1
M=f_ So, u will be minimum, when v is minimum i.e.,
ex
Vmm =-25 cril
In practice, angles a. and 13 are small. Therefore, angles a.
and p can be replaced by their tangents i.e. (u)m1n
5
~25 =--4.17 cm
6
M= tanp =CA'=~ {-: A'B'=AB) -(15)- 1
tancx CF u And u will be maximum, when v is maximum i.e.,
Applying new cartesian sign conventiOns vmax-), co
CF;,,-/ and CA'=-D
5
(u)m~=-(s) =-5cm
M=D - -1
f 00

(b) Since magnifying power for a lens is


CONCEPTUAL NOTE($) V
It folloWs that magnifying power of the simple microscope is one m=-
u
less, when the image is formed at.inffnity. However, the vi0w(.ng
Of the image is more comfortable. Magnifying power will be minimum, when u is
maximum i.e., um',.,.,; = -5 cm
USES D -25
=> (mlm1n =-=-=5
a) Jewellers and watch makers make use of convex lens of f , -5
short focal length to obtain a magnified view of the fine m will be maximum, when u is minimum i.e.,
jewellery work and the small components of the 25
watches. Umin =- = --4.17 cm
6
= ==================================
1.88

www.puucho.com
Rahul Sardana with www.puucho.com

Ray Optics: Refraction at Curved Surfaces

-25
~ (mlm~ =~=6
6 Since the angles u and p are small, they can be replaced by
their tangents·i.e.
COMPOUND MICROSCOPE tanp A'B"
M=--=--
A compound microscope is used to see extremely small tanu AB
objects.
A"B' A'Q AB )
It consists of two lenses. A lens of short aperture and short ·:tanP=-- and t a n u = - = - (asA"Q=AB)
\ C'A" C'A" C'A"
focal length facing the object is called object lens and
another lens of short focal length but large aperture is called Multiplying and dividing by A'B', we have
eye lens. The two lenses are placed coaxially at the two ends A"B" A'B' A'B' A"B"
M=--X--=--X--
of a tube. To focus over an object, the distance of the object AB A'B' AB A'B'
lens from the object is adjusted with the help of rack and
pinion arrangement.
When a small object is placed just outside the focus of the v 0 A'B' M gnif" .
where m0 =-=--= a 1cation produced by Object
object lens, its real, inverted and magnified image is u0 AB
produced on the other side of the lens between F and 2F . lens.
The image produced by object lens acts as object for the eye
lens. The distance of object from the object lens is so adjusted and me= :e =~::~=Magnification produced by Eye lens.
that the final image is formed at the least distance of distinct '
vision from the eye. . Further, for the eye lens,
- Let AB be an object placed just outside the focus F0 of the 1 1 1
---=-
object lens. Its virtual image A'B' is formed on the other side Ve Ue le
of the lens. The image A'B' lies between focus F~ and optical
~
centre C' of the eye lens and it acts as object for the eye lens.
Using the rack and pinion arrangement, the distance
between object lens and the object AB is adjusted, till it ~ m =l-~
virtual and magnified image A"B" is formed on the same
' I,
side at the least distance of distinct vision. Applying new Cartesian sign conventions we get
v, =-D and I, =+I,
MAGNIFYING POWER D
~ m =1+-
Case I : When image i~ formed at D ' I,
It is defined as the ratio of the angle subtended by the final So, finally we get
image at the eye to the angle subtended by the object seen
directly, when both are placed at the least distance of distinct M=m 0 xm, = Vo(1+
Uo
D)
fe
vision.
EYE LENS In practice, the focal length of the object lens is very short
OBJECT LENS and the object AB is placed just outside the focus of object
8 a lens. Hence
u, = lo (in magnitude)
So that the final image is formed at least distance of distinct
vision from the eye, the image A'B' must lie between optical
'
!''
centre and focus of the eye lens. Since, the focal length of the
,,..,,,.,,.9, eye lens is also small, the distance of the image A'B' from
I
1
.,,,.~
__ ,,. - - - - -
_,.!-- the object lens is nearly equal to the length ( L) of the
B"t.,..-- i+--- u.,--.i
i+----v, _ __,., microscope tube i.e. v0 = L
l+----1,---.i According to new Cartesian sign conventions we have
f+-----D----><
Uo ';::!,-lo and Vo ';::!,L
Let LA" C'B" = p be the angle subtended by the final image
at the eye. Cut A"Q equal to AB and join QC'. Then, Substituting for u0 and V0 we get
LA"C'Q = u, the angle subtended by the object at the eye,
when situated at the least distance of distinct vision. ·By
M=-.!:..(1+
lo
D)
I,
definition, magnifying power of the compound microscope,

================================== 1.89 =
www.puucho.com
Rahul Sardana with www.puucho.com

Advanced JEE Physics Optics & Modern Physics

CONCEPTUAL NOTE(S) ASTRONOMICAL TELESCOPE (REFRACTING TYPE)


a) From the above expression, it follows that a compound An astronomical telescope is used to see heavenly objects. It
microscope will have large magnifying power, if both the
produces a virtual and inverted image. As such bodies are
object lens and the eye .lens are ot·small. focal length. In
practice, focal length of object lens is smaller than that of round, the inverted image does_not"affect the observation.
eye lens i.e. t <~ . Further the· negative value of An astronomical refracting telescope consists of two- lens
magnifying power of compound microscope tells that. final systems, Th~J~ns system facing the object is called objective.
image fanned is Inverted. It has large aperture and is of large focal length (/0 ) • The
b) In practice, to · eliminate chromatic aberration, a.
other lens system is called eye-piece. It has small aperture
combina,ion of two lenses in coritact is used. It is called
objective. and is of short focal length (I, ) . The objective and the
c) In place Of an ~ye lens,_ a combination of two lenses at eye-piece are mounted coaxially ill two- metallic tubes. The
certain distahce apart satisfying certain co·nditions (to tube holding the eye-piece can be made to slide into the tube
minimize chromatic-and spherical aberrations) is used. It.is
called eye piece. holding the objective with the help of rack and pinion
arrangement.·
The objective forms the real and inverted image of the
Case II : When image is formed at infinity
distant object in its focal plane. The position of the eye-piece
The magnifying power of a compound microscope, is adjusted, till the final' image is formed at least distance of
M=m0 xme distinct vision. In case, pdsition of the eye-piece is adjusted
If u0 is distance of object from the object lens and v0 , then such that final image iS formed at infinity, the telescope ·is
said to be in normal adjustment. The_ image formation is
distance of image from it, then as obtained eailier, discussed in the following n:vo cases. ·
V
m, =--'1.
u, Case I :. _When final 'image is formed at infinity (Normal
Further, as in the case of a simple microscope we have adjustment)
D When a parallel beam of light rays from the distant object
m=- falls on the objective, its real and -inverted image A'B' is
' f,
formed on the other side of the objective. If the position of
where f, is focal length of the eyeJens. eye-piece is adjusted, so that the image A'B' lies at its focus,
Therefore, magnifying power of a compound microscope is then the final highly magnified image will be fanned at
given by infinity. Under such a situation i.e. in normal adjustment tne
v, D magnifying power of a telescope is defined as the ratio of the
M=-x- angle subtended by the image at the eye as seen through the
Uo fe
telescope to the angle subtended by the object seen directly,
when both the object and the image lie at infinity. It is also
Illustration 72 called angular magnification of the telescope and is denoted
A compound microscope has a magnifying power 30. by M. '
The focal length of its eye-piece is 5 cm . Assuming the
final image ·to be at the least distance of distinct .vision OB
CTIVE
J~E- EVE PIECE

(25 cm), find the magnification produced by the objective.


Solution
For a compound microscope, we have
,"- C "- $.\~ A'*~ -
:,;:~:..::
y

.,: .. (1) ::>•'


f(\Ql,\

Since the final image is formed at least distance of distinct ! + - - - - !,.------- f,+l

vision, the magnification of eye-piece is As the object is at a very large distance, the angle subtended
25 by it at the eye is practically the same as that subtended by it
m =(1+ D)=l+ =6 at the objective.
' f, 5
Thus, LA'CB' = a. may be considered as the angle subtended
From equatio;.,, (1), we get
by object at the eye. Let LA'C'B' =p .
-30=m0 x6
Then
30
;:::> m0 =--=-5
6 M=Ia.
Negative sign implies that image formed by the objective is
inverted.
Since the angles a. and p are small, a. " tan a. and f= tan p.
Therefore,.

= 1.90 ::;:;::===================================
www.puucho.com
Rahul Sardana with www.puucho.com

Ray Optics: Refraction at Curved Surfaces

M= tan~= CA'
tana. C'A'
Using new Cartesian sign conventions we get For eye lens, we have
CA'=+fo 1 1 1
---=-
{·: distance of A'B' from object lens is along the incident Ve Ue le
light} 1 1 1
C'A'=-fe =CO> -=---
u, v, f,
{·: distance of A'B' from eye lens is against incident light}

=CO> M=fo
=CO> _!_=_.!.( 1-f,
Ue fe Ve
J
f,
-·--·--------------------· --- - - - - , =CO> M=- 1•(1-f,
le ·Ve
J
CONCEPTUAL NOTE(S)
a) It follows that the magnifying power of a telescope in Applying new Cartesian sign conventions we get
normal adjustment will be large, if objective is of large focal f,=+f,, v,=-D, f,=+f,
length and the eye·piece -is of short focal length.
b) Further, when telescope is in normal adjustment, the
distance between the two lenses is equal to sum of their =:> M=-f'(1+f')
f, D
focal lengths ( + t t ).
Therefore, a refracting telescope will have large magnifying
c) Further, the negative value of the magnifying power of the.
_telescope.tells that final image formed is inverted and real.
power, if the object lens is of large focal length and eye lens
·is of short focal length. Further, the negative value of
magnifying power of the telescope tells that the final image
Case II : When final image is formed at least distance of formed is inverted and real. Out of the two adjustments
.distinct vision discussed, this adjustment gives a higher magnification,
When a parallel beam of light rays from the distant object
falls on the objective, its real and inverted image A'B' is since the fact_or ( 1 + -{; ) is gre~ter than one.
formed on the other side of the objective. The position of
Also, a telescope does not increase the size of object, but it
eye-piece is adjusted so that the final image NB' is formed
forms an image nearer so that the angle of vision is increased
at least distance o( distinct vision. Under such a situation the
and hence it appears to us as if the bigger image of object is
magnifying power of a telescope is defined as the ratio of the
formed.
angle subtended at the eye by the image formed at the least
distance of distinct vision to angle subtended at the eye by
the object lying at infinity, when seen directly. Illustration 73
g OBJECTIVE The objective of a telescope is a convex lens of focal
EYE PIECE
length 100 cm . Its eye-piece is also a convex lens of focal
::; l.:------LJ
>- length 5 cm . Determine the magnifying power of the
oa: (.)
L1J
lL al
telescope for normal adjustment.
ct C
0 Solution
L~;-;;Ei;; =-- For normal adjustment, the magnifying power of a
B'' , t4-- fa -->-t telescope is given by
I+-- D ------,--tot
M=f'
Again, as the object is at a very large distance, the angle a f,
subtended by it at the objective is practically the same as that
subtended by it at the eye. Therefore, if LA'C'B' = P, then
Here, Ip =100 cm, le =5 cm
=CO> M= 100 =20
M=I 5
a
Again, as angle a and p are small, they can be replaced by TERRESTRIAL TELESCOPE
their tangents,
A terrestrial telescope is used to observe obj~cts on earth.
=:, M = tanp = CA' An astronomical telescope is used. to view heavenly objects
tana C'A' since the inversion .of their images does not produce any
Since, CA'= lo and C'A' = ue complication..While viewing earthly objects we would prefer
to have their images erect and hence, astronomical telescope
is not suitable in such cases. By usinl? an additional convex

================================== = 1.91

www.puucho.com
Rahul Sardana with www.puucho.com

Advanced JEE Physics Optics & Modern Physics

lens O (of focal length f ) in between 0 1 and 0 2 of an


astronomical telescope, we can have the final erect image.
The lens O is called erecting lens, while the improved
a
version of the telescope is calle.d Terrestrial Telescope.
Rays from the distant object get refracted through the
o, a

objective 0 1 , giving a real inverted image A1B1 • The


erecting lens O is so adjusted that its distance from Ai.B1 is J+--0---+I

equal to twice its (erecting lens) focal length. An image A,B,


Parallel beam of incident rays from infinity are focussed by
having same size as that of A 1B11 inverted w.r.t. A1 B1 and the objective 0 1 • An inverted image A 1B1 (shown in grey)
hence erect w.r.t. the object is obtained at a distance 2/ on would have been formed after refraction through 0 1 • Before
other side of O . A2 B2 acts_ as an object for lens at 0 2 and the rays meet at A1 , a concave lens ( at 0 2 ) intercepts them.
finally an erect and magnified image is obtained after The beam diverges and the final erect image A,B2 is
refraction through 0 2 • If the distance 0 2 B2 is equal to focal
obtained. The distance 0 2B1 is so adjusted that final image is
length f, of the eye lens 0 2 , final image is formed at infinity
formed at the distance of distinct vision. If O2 B1 is equal to
and the telescope is said to be in normal adjustment as in
Figure 1. the focal length f, of eye lens at 0 2 final image is formed at
infinity and the telescope is said to be set in normal
adjustment. In such a case the length of the tube is equal to
the difference between the focal lengths of two lenses. The
a field of view of this telescope is small because of the use of
concave lens.
When set in normal adjustment, its magi:iifying power M iS
given by
A1B1
FIGURE 1
M =I= tan~= B10 2 = B,01
If the distance O2 B2 is less than fe then corresponding to a a. tancx A,B, B,O,
certain value of this distance, a virtual and magnified image B101
is obtained at the distance of distinct vision as shown in M F Focal length of objective
Figure 2. =>
f Focal length of eyeiens

RESOLVING POWER
The resolving power of an optical instrument is defined as
a the reciprocal of smallest angular separation between two
neighbouring objects whose images are just distinctly
formed by the instrument. The smallest angular separation is
called the Limit of Resolution.
1+2f . . . 2f+I <f
~ A. LIMIT OF RESOLUTION FOR MICROSCOPE
1+---D---+1

FIGURE2
Limit of Resolution is given by

Since the sizes of AzB2 and A1 B1 are same, ~troduction of S=-"/,,-


2µsin8
the erecting lens O has nor produced any change in its
where µsin0 is called numerical aperture and µ is
magnifying power but, has helped in getting the final image
erect only. It may also be noted that the use of erecting lens refractive index of medium in which it is placed. For high
0 results in an increase (equal to four times the focal length resolution of microscope a beam of electrons is used which
of erecting lens) in the length of the tube of telescope. has wavelength of the order of 1 A .

GALILEO'S TELESCOPE B. LIMIT OF RESOLUTION FOR TELESCOPE


Instead of using a combination of two, lens 0 1 and 0 2 for If a is aperture or diameter of telescope and A. the
wavelength, then resolving limit is
getting an erect image, Galileo used only one concave lenses
to get the final erect image. d8oc~
a

= 1.92 [:::===================================
www.puucho.com
Rahul Sardana with www.puucho.com

Ray Optics : Refraction at Curved Surfaces


1
For spherical aperture d0 = ·22 '- · E. INVERSE SQUARE LAW OBEYED
. a BY ILLUMlNANCE
When radiant energy falls normally· on a surface, the
,,
Resolving power oc !!..
illuminance E of the surface is inversely proportioflal to the
Resolving power of telescope or microscope has no concern square of distance of surface point from source i.e.
with focal lengths of lenses. 1
Eoc-;, ... (1)
PHOTOMETRY
F. LAMBERT'S COSINE LAW OBEYED
A. RADIANT FLUX BY ILLUMINANCE
It is the radiant energy emitted by a body per second in all When radiant energy falls obiiquely o~~.a, surface, the
directions including all wavelengths. Its unit is watt. illuminance E of surface is directly proportional to .the
cosine of angle made by normal to the surface with the
direction of incident radiation. So,
B. LUMINOUS FLUX
E oc cose ... (2)
The radiant energy emitted by a body per second in the
Combining (1) and (2), we get
visible region (i.e., between wavelengths from 4000A to
7800A) is called luininous flux. Its unit is lumen. E=Icos8 . .. (3)
r'
C. LUMINOUS INTENSITY OF A LIGHT SOURCE :rotal Luminous Energy falling on a surface is given by
Q=EAt
The luminous intensity of a light source in any direction may
be defined as the luminous flux per unit solid angle in that where E = illurninance of the surface
direction. A=areaand
Its unit is lumen/ steradian or candela. t = exposure time.
Luminous Intensity The total luminous energy required to be incident on a given
type of camera film is constant.
M (in. lumen/steradian or candela)
I= ,ill
For a box type camera

Time of exposure oc ( ~ )'


If the light source is isotropic, then luminous flux is uniform
in all directions, so that total luminous flux is given by
F =4111 .(since total solid angle for all directions is 41t) where d= diameter
If we plot the ratio luminous f =focal length of camera lens.
flux/radiant flux against
wavelength of radiation, we get a 685
graph as shown in figure. G. THE LUMINANCE (L) OR BRIGHTNESS
The graph indicates that the ratio Luminance of a surface is the luminous flux reflected by unit
luminous flux/radiant flux is area of the surface normally.
maximum for sssoA i.e. for => Luminance= Illuminance x Reflection Coefficient.
yellow colour. 1his indicates that "-,--~~"-~-1,.
our eye is most sensitive for 4000A 5550A 7800A H. PRINCIPLE OF PHOTOMETRY
5550A i.e. yellow colour. The maxirnti.m value of ratio If two sources of light of illuminating power 11 and 12 are
luminous flux/radiant flux is 685 lumen/watt. Thus when placed at distances r1 and r2 from the screen, then the
the r~tio luminolls flux/radiant flux is 685 lumen/watt, the
screen will be equally illuminated due to two sources when
luminous efficiency is said to be 100%.
The tungsten filament bulb converts 2-3% electrical energy [1 12
into visible light energy, while fluorescent tube converts
?"=?"
1 2

8-9% electrical energy into visible light energy.


ICE Vil
D. ILLUMlNANCE OF A SURFACE (E)
BASED ON LENS FORMULA I
It may be defined as the luminous flux falling per unit area
,. {Solutions on page 1.189) I
on the surface. Its unit is Jumenm-2 or lux. The distance between two point sources of light is 24 cm .
Find out where would you place a conv~rging lens of focal i
E=M length 9 cm, so that the images of both the sources are !
t,S
formed attt:Le same ppint. ~ -·-··- --~- ·: ______ j

1.93 =
www.puucho.com
Rahul Sardana with www.puucho.com

Advanced JEE Physics Optics & Modern Physics

n object !s ·moved along the, ·prfncipal axis of a convex lens paralieltoihe major optical-aXiSOfthe system to leave
the lens, remaining parallel to th8 optical BJCiS, after being
ns. An image. three times the !:;ize of the .object is ob.taine_d
hen the object is at a.distance ·of 16 cm from the lens ·and reflected from the mirror? Find the' image of the object
produced by the given opfical system.
'a' distance of 8 cm from the lens. Find the focal leng~h
of the'lens. · ·
13. A _convex lens of focal length \ is placed infront of a

3. The radius of curvature of. the. convex surface of a planq- luminous point object. The separation between the object
convex lens is 10 cm and its focal length-is 30.cm-. What and the lens Is 3\ . A glass slab of thickness t is placed
should be the refractive index of its material? between object and the lens. A real image of the o!)ject !S
formed at the shortest possible distance from_ the object.
(a) Find the refractive Index of the slab.
4. One face of an equi-convex lens {µ = 1.5) of focal length
eo· cin is silvered. Does it behave like a concave mirror or
(b) If a concave lens of very large focal length t
is
placed in contact with the convex lens, find the shifting
ca~vex mirror? Also determine the equivalent focal l~nQth
.of the image,
of the,mlrror.
14. An Opti98.I system consists of two convergent lenses with,
5. A bicorivex lens made of glass with a refractive index of
µ =1.6 .has a focal length of f =1O cm in air. Calculate the focal lengths \ = 20 cm and t
=1O cm . The distance
focal length of this Jens if It is placed into a transparent between -the ,lenses is d =30 cm .. An object is placed at a
medium' · distance of u1 = 30 cm from !he first. lens. At what distance
(a) with a.refractive index of µt = 1.5 from thEi second !_ens will the image be obtained?
(b) · with a refractive index ofµ, ~1.7
15. If r be the ,radius of curvature of each fBce of thin
converging lens whose one face is silvered and µ. is the
6. A biconvex thin tens is prepared from glass of refractive refractive index of _lens material, prove that the l_ens is
index %. The two bounding surface5; have equal radii of. equivalent to a concave mirror of focal length _r_.
, , 4µ-2
25 cm each. One of th? surfaces is silvered .from outside to
make it reflecting. Where should an object be placed•before 16. Three convergent thin lenses of focal lengths 4a , a and
this lens.so th~t the image coincides with the object.
4a respectively are placed 'in order_ along the axis so that
the distance between· consecutive lenses is 4a . Prov~ that
7. A converging lens of focal length 5 cm is placed in contact this combination simply inverts every small object _on the
with· a diverging lens of focal .leng,h 10 cm. Find the 8.Xis without change of magnitude orposition.
combined focal length of the system_.
/
17. The distance between an object and a divergent lens is m
8. A~ biconvex lens of refractive index 1.5 has a focal length of times.,greater than the focal length ,of the lens. How many
~ = 1o cm . One of the lens surfaces having a radius ·of times will'the image be smaller than the object?
curvature of R=10 cm Is coated with silver. Determine the
position of the image if ~the object is at a distance- of 18. An image I is formed of point object O by a lens w_hose
u == 15 cm from the lens. optic axis·is .AB as shown In figure.
0•
9. A convex lens is held 45 cm above the bottqrn of an empty
tank. The image of a point at the-bottom of a tank is formed
36-cm above the lens. Now a liquid is poured into the-tank A B
to a depth of 40 cm . It is found that the distance of the
• I
image of the same point on the bottom of the tank is 48 cm .(a) State whether it is a convex ler:iS or concave?
above the lens. Find the" refractive index of the liquid. (b) Draw a ray'diagram to locate 1he le_ns and Its focus.

10. A concave spherical mirror with a radius of curvature of 19. Two thin lenses having focal lengths \ =7 cm and
0:2 m is filled with water. Calculate'the focal length of this
t = 6 cm are placed at a distance d,= 3 cm apart. whaUS
system? Given that refractive index of water is
' ' 3
.± . the distance of the focus of the ~ystem from. the second
lens? Assume th·e system to ,be a centred one.

11. A convex lens of focal length ~ is placed lnfront of, a 20. Two glasses with refractive in\'.flc~s of µ 1 =1.5 and µ 2 ,=1.7
luminous point P so that the distance of the .point P from are used to make two identical,double convex lenses.
len~ is greate_r than focal length and the image formed is at (a) Find the ratio betwee"n their focal lengths.
the shortest possible-distance. If now a concave lens of very (b) How will each of these lenses act on a ray parallel 1o
large. focal length t. be placed in contact with first, find the Its. optical axis if the lenses are submerged into a
transparent liquid with a refraclive·index of 1.6? "
shift In.the position of the image .

.12. At what •distance from a biconvex lens of focal length


21. A parallel beam of light is incident on a system consisting of
f.= 1 m shou[d a concave spheri~I mirror with a radius of
three thitUenses WithyCOmJTIOQ- opJLcal axis._ The focal
curvature· of R = 1 m be placed Jar a beam incident on the
' I 1,94

www.puucho.com
Rahul Sardana with www.puucho.com

Ray Optics : Refractio11 at Curved Surfaces

lengths of the lenses are equal to i =+1O cm , t =-20 cm surface the Image of the object pin is formed at the position
of the ob_ject. pin. placed at a distance of x 2 from the
and t = +9 cm , respectively. The distance between the
first and the second lenses is 15 cm and between the silvered I8ns. Find, in terms of x 1 and x 2 , the
second and the third 5 cm . Find the position of the point at (a) focal length of lens
which the beam converges when· it leaves the system of (b) radius of curvature of the curved surface and
(c) index of refraction of the medium of lens.
lenses.
--------~
22. Consider a piano-concave lens with one of the radii of
curvature r made up of a transparent material whose
refractive index varies with intensity .(I) of incident light as
3
µ =.µ 0 + al , where a> 0 and O< µ 0 < . Calculate the
2
intensity' when the focal length Is equal .to two times the
radius of. curvature r .

23. Paraxiat rays are incident on surfaces of a thin equiconvex


glass lens of refractive index µ and having radius of
curvaturE',I R . If the final image is formed after n internal
reflectlons, calculate distance of this image from pole of the
lens.

24. When the plane surface of a piano-convex lens is silvered. it


is found that the image of the object pin is formed at the
p9sition of the object pin placed at a dis!ance of x1 from the
silvered lens. When the same l~n?,j?,_silvered on th8 CU'-'Y§lsl

================================== = 1.95

www.puucho.com
Rahul Sardana with www.puucho.com

Problem 1 Further, _parallel tci ~e mfrror,.i.e., along y-axis, we have


A plane mirror is mo~ng with a uniform speed of (v,), =(vo), =10]
5 ms-1 along negatiVe x-direction and an ·observer P is
moving with a velocity of 10 ms-• along +x direction. Since v, = (v, ), +(v, ),
Calculate the velocity of image of an object O, moving So, absolute velocity of the image is
y
with a velocity of 10,J2, ms-• as shown in the figure, as v, = -20/ + 10]
observed by the observer. Also firid its magnitude and
direction. · Now i\p.=V1 -iip
=> i\, =-20/ + 10]-101
10./2 ms- 1 -J+-----'~L.:"f--.-x
=> v,, = -30/ + 10] 30ms 1

04__ __ _ => Jv"l=-.'900+100=10Mms-•


If p is the angle made by VIP with -x axis, then

1
10
5ms tanP=
30
Solution
Let v0 be the velocity of the object O, v, be the => P = tan-• (½): with -x axis
velocity of the observer P, vM be the velocity of the mirror
and ·v, be the velocity of image (Assume all these velocities
Problem 2
w.r.t. ground), then
Consider the situation shown in figure. The elevator
- 10,J2, !
Vo= ,J2, ('l + J') is going up with an acceleration of 2 ins_, and the· focal
! Ly length of the mirror is 12 cm . All the surfaces are smooth
v0 =10(/ + ]) I0 X
and the pulley is light. The mass pulley system is released
from rest (w.r.t. the elevator) at t = 0 ..when the distance of .
VP =lOi B frQm the mirror is 42 cm . Find the distance between th~
5ms 1 iroage of the block B and the mirror at t = 0.2 s . Take
VM =-5i
g=10 ms-•.
(v,M), = -(VoM);, Where !he axis perpendicular to the,
mirror is .the x~axis.
=> (v,), -(vM), =-(vo). +(vM).
=> (v,). =2(vM), -(v0 ),
=> (v,), =2(-s1l-107
=> (v, ), =-.207

= ==========::::;:=======================
1.96

www.puucho.com
Rahul Sardana with www.puucho.com

Ray Optics

Solution Problem 3
Let us assume thatthe acceleration ofblocks A and B A fixed cylindrical tank of height H = 4 m and radius
to be a w.r.t. lift and a, be the acceleration of lift. R = 3 m is filled up with a liquid. An observer observes
G;msider block A , as seen from the reference frame attached through a telescope fitted at the top of the wall of the tank
to the lift (a non-inertial frame), we get and inclined at 0 = 45° with the vertical. When the tank is
completely filled with liquid, he notices an insect, which is
N=mg+maL ... (1)
at the center of the bottom of the tank. At t = 0 , he opens a
T=ma ... (2) cork of radius r = 3 cm at the bottom of tank. The insect
N· moves in such a way that it is visible for a certain time.
Determine

T
m ---+a

Free Body Diagram of A


Now, consider block B, as seen from the reference frame
H=4m
attache~ to the lift! we have
mg+ma~ -T=ma ... (3)
Cork
T
Insect
(a) the refractive index of the liquid
(b) the velocity of insect as a function of time.
Solution
(a) At t=0
. 3
smr=-
Free Body Diagram Of B 5
Slilt 5
On adding equations (2) and (3), we get
µ = sinr = 3../2
a= g+a, = 10+2 =6 ms-'
2 2
'' ~Eye
So, distance fallen by block (B) is x =½at' :.-,
'-
~

2
=> x=2-x6x(0.2)
2
4m
=> x=0.12 m=12 cm
Now, consider reflection at convex mirror, we have
u=-(42-12) =-40 cm 3m
J=+12cin (b) Let at time t , insect be at a distance x from centre of
· . 1 1 1 the tank. Since,
Smee-+-=-
v u .f X
---1. = tan r =-3
h 4
1 1 1
=> -+--=- 3
v (-40) 12 => x1 =-h
4
(40)(12) 480
=> V So, x=(H-h)+x, -3
40+12 52
=> v=9.23 cm 3
=> x=4-h+~h-3
4
Therefore, the distance between the image of block (B) and
mirror is 9.23 cm h
=> X=l--
4

1.97 =
www.puucho.com
Rahul Sardana with www.puucho.com

Advanced JEE Physics Optics & Modern Physics

Solution·
... (1)
(a) For refraction at first half )ens, using lens formula,
From Equation of Continuity, we have 1 1 1
---=-,we get
V U f
-A,(~~)=A,"2gh 1 1 1
V -20 15
=> (- dh) = A,.fii dt => v=60 cm
Jii A,
. . V 60 -3
M agnif1cation, m=-=-=
-J.:!1:.=" 3 xrn,- ,l2x9.Bf dt
h ( ')' '
u -20
H Jii ,c{3) o The image formed by first half lens is shown in Figure-1

~5' i
B

A
F _________ A,_ ___ _
AB=2 mm
A1B1 =6 mm
A0 1 = 20 cm
4mm 0 1F= 15 cm
:
:45°
H-h
H-h C,
B,
1
2mm O1A1=60cm

'
Figure-1
r,
'' h Now, the point B1 is 6 mm below the principal axis of
the lenses. Plane mirror is 4 mm below it. Hence,
* 4 mm length of A1 B1 (i.e.,A1C1 ) acts as real object for
mirror. Mirror forms its virtual image A2 C2 . So, 2 mm
Substituting H = 4 m, we get length of A,B1 (i.e., C1 B1 ) acts as virtual object for
2
h = (2-2.21x10--!t) mirror. Real image C,B, is formed of this part. Image
formed by plane mirror is shown in Figure-2.
- dh =4.42x10--<(2-2.21x10-4 t)
dt B,
2mm
So, speed of insect is C,

V= ~: =¼(-~~) ''
:' 4mm
=> V=1.1x10--<(2-2.21x10-4t) ms-1 ''
'
•A,
Problem 4 Figure·2
A convex lens of focal length 15 cm is split into two For the second half of the lens, using lens formula
halves and the two halves are placed at a separation of 1 1 1
---=-,we get
120 cm . Between these two halves of the convex lens, a V U f
plane mirror is placed horizontally and at a distance of 1 1 1
4 mm below the principal axis of the lens halves. An ---=-
v -60 15
object of length 2 mm is placed at a distance of 20 cm
from one half lens as shown in figure. => v=+20
f=15cm f=15cm
V 20 1
m=-=-=--

!. ~ ~
u -60 3

2mm
½
So, length of final image A,B, = A2 B2 = 2 mm .

20 cm ,_.....,...__ ___._4 ~m However, point B2 is 2 mm below the optic :axis of

1+---- 120 cm -------H second half lens. Hence, its image B3 is formed I mm
3
(a) Find the position and size of the final image. above the principal axis.
(b) Trace the path of rays forming the image. Similarly, point A, .is 8 mm below the principal axis.

Hence, its image is ~ mm above it.


3

· 1.98 =======================:::::;:========::i
www.puucho.com
Rahul Sardana with www.puucho.com

Ray Optics

Therefore, image is at a distance of 20 cm behind the 1 1 2


-+--=-
v, -12.5 -20
second half lens and at a distance of I3 mm above the
::::::> V1 =-50 cm
principal axis.
V (-50)
The size of image is 2 mm and is inverted as compared
to the given object. Image formed by second half lens is
m,=-;;-= (-12.5) -4

shown in Figure-3. So, the image formed by the mirror is at a distance of


50 cm from the mirror to the left of it. It is inverted and
four times larger.
Step 2 : Image formed by lens
O'
---+---------- --------•-· The image formed by mirror acts as an object for the
O"
B, lens and the image formed by the mirror is at a distance
of 25 cm to the left of lens. Using the lens formula,
1 1 1
- - - = - we get
A,
V U f .
Figure-3
1 1 1
(b) The ray diagram for the final image is shown in v, 25 -16.7
Figure-4.
V2 =-50.3 cm
V -50.3
and m1 =-=--=-2.012
u 25
Net magnification is given by
m=m1 xm2 ~s
Hence, the final image is at a distance 25.3 cm to the
,,
,,,,
,,
right of the mirror, virtual, upright enlarged and
,, approximately 8 times. Positions of the two images are
,, ,,,, shown in figure.
' ,' B"
A,,V
'
•''
''

Problem 5
Figure-4

A'
\( B

A
'
''
''

l\
A"
An object lies midway between the lens and a mirror.
The mirror's radius of curvature is 20 cm and the lens has
a focal length of -16.7 cm. Considering that the rays that B'
leave the object travel first towards the mirror, locate the !+-- 25 cm --+1<---M<<---t- 25.3 cm -...i
12.5 cm 12.5 cm
final image formed by this system. Is this image real or
virtual. Is it upright or inverted? What is the overall
magnification? Problem 6
A thin piano-convex Jens of focal length f is split

\/I I
into two halves. One of the halves is shifted along the
optical axis as shown in figure. The separation between
object and image planes is 1.8 m. The magnification of the

li H--- 25 cm -----...i
image, formed by one of the half lens is 2. Find the focal
length of the lens and separation between the two halves.
Draw the ray diagram for image formation.

Solution
Step 1 : Image formed by mirror I--''-'-'_ _ ___,
0
.. f l 1 1 1 2 \ '.
U sing mirror ormu a-+-=-=-, we get \J
V u f R 1.Sm

================================== 1.99 =
www.puucho.com
Rahul Sardana with www.puucho.com

Advanced JEE Physics Optics & Modern Physics


y
·solution
+
For both the halves, position of object and image is ''
''
--
same, however the only difference is of magnification.
Magnlfication for one of the halves is given as 2(> 1). This
can be for the first one, because for this, [vi> [u[ . Therefore,

magnification, [ml= l~I > 1. So, for the first half, we have 0'
''
'''

=> [v[=2[u[ - - - - 21 - - - + <


Let u=-x,then v=+2x (a) Find the co-ordinates of the final image formed by the
system of lenses taking O as the origin of the
and [u[+lvl=1.8m
co-ordinate axes.
=> 3x=1.8 m
(b) Draw the ray diagram.
=> x=0.6m
Solution
Hence, u=-0.6m and v=+1.2m
(a) For concave lens (L,)
.11111 1
Usmg - = - - - = - - - - = - 1 1 -1
! V U 1.2 -0.6 0.4
v -fcosa. f
=> f = 0.4 m
For the second half, we have => ( fcosa.) ... (1)
v=- l+cosa.
1 1 1
-=--+---- The magnification is given by
! 1.2-d -(0.6+d)
V 1
1 1 1 m
-=--+ u 1 + cos a.
0.4 1.2-d (0.6+d) L,
Solving this, we get d = 0.6 m L,

Magnlfication for the second half will be


''
'' ,
V 0.6 1 r'f------ f ----+t, ... Ct,
m, =-;; = -(1.2) = -2 I a ,,. \
,'\ ...---- /,
fsina\

----1-
12 ,..,...--_ni;d- ;
).,, \V-- '
and magnlfication for the first half is m1 = !'. = ( · ) = -2
u - 0.6 ,,- ~

The ray diagram is as follows:


1+--d----..
So, height of I, from the principal axis of L, is
A -------+---------
//, I ...... -- .; B, __ '
f sin a
h=(fsina.)m ~ - ... (2)
,/'.1 ; ... ,......... ! l+cosa.
f=0.4m ./ .. .....-...-f"'o.4m i Hence the x-co-ordinate of image 12 is given by
~ I ' .,.... ... ----4M IB
A ,/ I .. ":'I "... .. -- ,....,...- ---..
,.,,. j 2 x=2f-lvlcosa.-hsina.
,/ .,, ------- .... ------\ 2 ,,:,,.,,--
,~~-::::-----
I ,/ II '(A,, A,)
=> x=2f- fcos'a. /sin 2 a.,

B
-----------+-----------.· !
l+cosa l+cosa.

\~ l => x=2f- f
l+cosa.,
t+--:" 0.6 m ---,..i+-- 0.6 m--+++- 0.6 m
=> x=f(2cosa.+1)
cosa+l
Problem 7
Two thin lenses of same focal length f are arranged
0
1------1' '' .,..,. ,
a.1/
,
with their principal axes inclined at an angle a as shown 2', I .,. IVI
in figure. The separation between the optical centers of the h'',~//
-'.,,
lenses is 2/ . A point object lies on the principal axis of the
convex lens at a large distance to the left of convex lens. Similarly, y co-ordinate of image I,

=1.100 ===========================::::.:;:======
www.puucho.com
Rahul Sardana with www.puucho.com

Ray Optics
y = _:(lvlsincx-hcoscx) 1 1 . 1
v, - (15-5) = 20
On substituting the_values of lvl and h from (1) and
(2),weget y=O. 20
=> v2 = - cm /;;:::J 6.67 cm
So, the coordinates of the final irhage·are 3
V 20/3 2
t(2coscx+l), o] and m2=-=--=-
[ cosa.+1 u 10 3
About Final Image
(b) Ray diagram is shown in figure
Net magnification is given by

m=m1m2 =-31
i.e., height of the image is 3 ~ ½ = 1 cm

Since, the net magnification is-negative, so the final image is


inverted.
Further y -coordinate of a point of the image will be,

CONCEPTUAL NOTE(S)
Y1 =myo-m2.6. ... (1)
The Y-co-ordinate of 12 is zero is very obvious because a ray of with respect to the principal axis of L1
light starting from 11 and passing ihrgugh O' will suffer no So, y -coordinate of image of A is
deviation. Hence, 12 must be forined an this line itself [e.,
Y=O. y,. =(-½)<oi-(¾)(½)=-½ cm and
·-------------
y -coordinate of image of B is
·Problem 8
Two thin lenses f, = 10 cm and f, = 20 cm are y,. =(-½)<3)-(¾)(½)=-¼ cm
separated by a distance d = 5 cm . Their optical centres are

0
Thus final image is as shown in figure.
displaced a distance 6. = 0.5 cm . A linear object of size
3 cm placed at 30 cm from the optical centre of left lens.

0
Find the naturt; position and size of final image.

Lx
y
-1--- -- ------------ :: ::i,:::!f
B
1/3 cm
·
f
-t--- 0-- --------------
.B
A
L, L,
B' l 413cm
!
1
6.67 cm
L, L, Problem 9
Solution A prism of refractive index n1 and another prism of
Step 1 : Refraction from the first lens L, refractive index n2 ·are stuck together with a gap as shown

Using the lens formula, .!._.!.=.!_,we get in the figure. The angles of the prism are as shown, n1 and
V U f n, depend on 1,, , the wavelength of light according to the
1 1 1
relations given by n1 -- l "20 +10·8'J...2x 10' and
v, -30 10
=> v1 =15 cm 145 + 1.80/..,2x 10' where 1,, is in nm.
n2=.
V 15 1
and m, = - - - = - - D
u -30 2
Step 2 : Refraction from th'e second lens L2

Again using the lens formula, .!. _ .!. =· .!_, we get


V U f

c================================= 1.101 =
www.puucho.com
Rahul Sardana with www.puucho.com

Advanced JEE Physics Optics & Modern Physics

(a) Calculate the wavelength 1.. 0 for which rays incident 4


opening at one end of a tank filled with water µ =- . On
at any angle on the interface BC pass through 3
without bending at that interface. the opposite side of the lens, a mirror is placed inside the
tank on the tank wall _perpendicular to the lens axis, as
(b) For light of wavelength i..,, find the angle of
shown in figure. The separation between the lens and the
incidence i on the face AC such that the deviation mirror is 0.8 m. A small object is placed outside the tank in
produced by the combination of prisms is minimum. front of. Find the position (relative to the lens) of the image
Solution of the object formed by the system.

Since, -~ = 1.20 + 10·8,.,x 10'


ff and
_
145 +
nz-.
1.80x10 4
').}
~ 0.9 m - - ~ 0.8 m ---+-1

where, A is in nm.
(a) The incident ray will not deviate at BC only if n1 = n,
•-----------
=> 1 _2 + 10.s:10• 1 _45 + 1.so:10• (i..=i..,)
Ao A.o

=> 9x;O' =0.2S Solution


i..,
1 ·
ApplyingLensMaker'sFormula, 1
-=(µ-1) (1
---1) ,
,_ = 3x10 f R, R,
=> 0
0.5
we get
=> 1..0 =600 run
(b) The given system happens to be a part of an equilateral __!_ =(~
0.3 2
-1)(!R --.!..)
-R
{·: R, =Rand R2 =-R}
prism of prism angle 60° as shown in figure.
=> R=0.3
Now applying, !!:1__ µ 1 = µ 2 -µ 1 at air glass surface, we get
V u R

=> v1 =2.7 m
So, the first image 11 will be formed at 2.7 m from the lens.
At minimum deviation, we have
This image 11 will act as the virtual object for glass water
60'
f1=f2=-=30°=r (say} surface.
2
Since according to Snell's Law, we have Therefore, applying !!:1__!:!_ = µ 2 -µ 1 at glass water surface,
V u R
sini
n1 =-.- we get
smr
=> sini=n'1 sin(30')
4
3
3
2
(¼)-(¾)
. v, 2.7 -0.3
Smee, ni =1.2 + 10.Sxl0'
2
, where A. 0 = 600 run
i.., => v, =1.2 m
So, the second image 12 is formed at 1.2 m from the lens1or
=> . ·-{l.2 + 10.SxlO'}(l)-
Slilt- - -1.5
- -_ -
3
(600)1 2 2 4 0.4 m from the plane mirror.
This iITI4ge 12 will act as a virtual object for mirror.
=> , , -1(3)
t=sm - Therefore, third real image 13 will be formed at a distance of
4
0.4 m in front of the mirror after reflection from it. Now this
image acts as a real object for water-glass interface. Hence
Problem 10
A thin equiconvex lens of glass of refractive index applying, & - 1:2 = µ, R- µ, , we get
V U
µ = ~ and of focal length 0.3 m in air is sealed into an
2

= ==================================
1.102

www.puucho.com
Rahul Sardana with www.puucho.com

Ray Optics

Applying Snell's Law at face AB, we get


3
2
4
3 (¾)-(¼) 1 sini
-(0.8-0.4) 0.3 n=--
sinr1
=> v, =-0.54 m => sini1 =nsinr1
So, the fourth image is formed to the right of the lens at a => i1 =sin-1 (nsinr1 )
distance of 0.54 m from it.
Substituting value of r1 , we get
Now finally applying the same formula for glass-air surface,
we get i1 =sin-1 {nsin(45°-C)}

1-(I)
I2
l_--- = - -2-=-0.9 m
=> i1 = sin-1 [ n(sin45°cosC -cos45°sinC)]
v, -0.54 -0.3
=>
Hence, the position of final image is 0.9 m relative to the
lens (rightwards) i.e., the image is formed 0.1 m behind the
mirror. => 11 ~ n,J]
. =sm. -1[ .J2n [ y1--;,:--;
Problem 11
A right angle prism ( 45° -90° -45°) of refractive
index n has a plane of refractive index n1 (n 1 <n) Therefore, required angles of incidence ( 4) at face AB
cemented to its diagonal face. The assembly is in air. The for which the ray strikes at AC at critical angle is given
ray is incident on AB . by

A i1 = sin-
1
[ }i.(Jn'-n; -n, )]
(ii) The ray will pass undeviated through face AC when
___fl_ n1 =nor r2 =0°
i.e., ray falls normally on face AC
Since it is given that rl-i < n , so the option 11i = n is
8 C ruled out, hence
(i) Calculate the angle of incidence at AB for which the '2 =00
ray strikes the diagonal face at the critical angle.
=> r1 =A-r2 =45°-0°=45°
(ii) Assuming n = 1.352 , calculate the angle of incidence
at AB for which the refracted ray passes through the Now applying Snell's Law at face AB, we get
. diagonal face undeviated. sini1
n=--
Solution sinr1
(i) Critical angle C at face AC will be given by 1.352 sini,
=>
sin(45')
.
C =Sm -1(11,)
-
n
=> · sini1 =(1.352)(1)
=> sinC =:i
n => sin i1 = 0.956
=> i, = sin-1 (0.956) "73°
Therefore, required angle of incidence is 4 = 73°.

Problem 12
A ray of light travelling in air is incident at grazing
angle (incident angle = 90°) on a long rectangular slab of a
B C transparent medium of thickness t = 1.0 m . The point of
Now, it is given that r2 =C incidence is the origin A(O, 0). The medium has a
=> r1 =A-r, =(45'-C) variable index of refraction n(y) ~ven by

c==================================== = 1.103

www.puucho.com
Rahul Sardana with www.puucho.com

Advanced JEE Physics Optics & Modern Physics

n(y) = [kt/I'+ 1]1/' where k = 1.0(meterr''' dy = y'1' or y_,1'dy = dx


dx
The refractive index of air is 1.0
y
=,, fy_, 14dy =fdx
y '
or 4y114 =x ... (3)
0 0
Air P(x,, y,)
, The required equation ·of trajectory is 4y 114 =x

t=1.0 m ---K~- .
,~ B(x, y),
=,,

,,..<' 1 Medium (c) At the point of intersection on the upper surface,


_,,.,,.," I a
-'--7==----"-=-----,-,-------+X y=lm
A(0,0) Air
4
=o> X = (256)1/ = 4 ffi
(a) Obtain a relation between the slope of the trajectory
of the ray at a point B(x, y) in the medium and the So the co-ordinates are ( 4 m, 1 m)
incident angle at the point. (d) As nA siniA =np sinip and as nA = np =1
(b) Obtain an equation for the traje~tory y(x) of the ray Thei-efore, ip =iA =90° i.e., the ray will emerge parallel
in the medium. to the boundary at P i.e., at grazing emergence.
(c) Determine the co-ordinates (x1 , y 1 ) of the point P,
where the ray intersects three upper surface, of the Problem 13
slab-air boundary.
. (d) Indicate the path of the ray subsequently.
A thin .biconvex lens of refractive index ~ is placed
2
Solution on a horizontal plane mirror as shown in the figure. The
(a) i+8=90°, 8=90°-i, space between the lens and the mirror is then filled with
Slope of tangent = tan8 = tan(90° -i) = coti water of refractive index _! . It is found that when a point
3
(b) tan8 = ! object is placed 15 cm above the lens on its principal axis,
the object coincides with its own image. On repeating with
dy . another liquid, the object and the image again coincide at a
-=cotz ...(1) distance 25 cm from the lens. Calculate the refractive index
dx
of the liquid.
Applying Snell's Law at A and B
nAsiniA =n 8sini8
nA=1 because y=O
1,,/liiu~,,,,mr
Solution
Let R be the radius of curvature of both the surfaces of
the equi-convex lens, then in the first case, the situation is
shown in figure.
µ2=~
r\~-~Le,;s,~';?ol.!\,,
! - ", -. -:.2··"-
- --
Water I
/)/if f I I l l ) J f l l / 7 ) 7 ) )

Let f, be the focal length of equi-convex lens of refractive


sin iA =1 because iA =90° Grazing incidence index µ 1 and· / 2 be the focal length of piano-concave lens
na =~Ky312 + 1 =~y3/2 + 1, (made of water) of refractive index µ 2 • The focal length of
the combined lens system is given by
because K = 1.0 (ni t 12

~=..!:..+..!:..=(µ, -1)(~-J:...)+(µ, -1i(J:...-~)


(1 )(1) = J(y' 1' + l) sini F !, I, R -R -R <X)

. . . 1
=,, Sllll=~-- =,, ½=(¾-1)(¾)+(¼-1)(-½)=½- 3~ = 3~
Jy3f2 + 1
=,, F=3R
=,, coti = ~y312 or y314 ... (2) 2
Equating equations (1) and (2), we get

= ====================================
1.104

www.puucho.com
Rahul Sardana with www.puucho.com
'.
RayOpHcs
Now, image coincides with the object when ray of light
retrace$ its path i.e., it is,fall_s normally on the _plane mirror. A
This is possible only'when object li~s at centre of curvature
p
of theJens system.
• B Q
(I) -. -----------·S
0

f
F=U=15cm 1+-- 30 cm ---+t+ 20 cm -+1
"
I Solution

... ~ Rays coming from object AB first refract from the lens
and then reflect from the mirror.
=> f=15 cm [ ·: Distance of object is 15 cm) For refraction from the lens, we have

3 u=-20 cm, [=+15 cm


=> R =15 cm
2
A
=> R=lO cm
Optic Axis
In the second case, let µ be the refractive index of the liquid B, of Lens
filled between lens and mirror and, let f' be the focal length
_ of new lens system. Then,
-r- ----r---- 0.6cm r B
-------------
Optic Ax.is
of Mirror
3 cm
:, =(µ, -1)(!- -~)+(µ-1)(_~ - !) i A,
..,.._ 30 cm ------ao cm_.,.. 20 cm -11-1
1 =(-' 1)'(2)_(µ-1)= 1_µ-1={2-µ)
F'2 R RRR R 1. 1 1
Applying lens formula, ---=-,we get
V U f
F' =~ = ___!Q_ {·: R=lO cm}
2-µ 2-µ 1 1 1
:;;- (-20) = 15
Now, the image coincides with the object when it is placed at
25 cm distance. => v=+60 cm
=> F'=25 and linear magnification is given by
V +60
___!Q_ = 25 m1 =-=--=-3
'=> 2-µ u (-20)
50-25µ=10 So, the first image formed by the lens will be 60 cm from it
=>
(or 30 cm from the mirror) towards left and 3 times
=> 25µ=40 magnified but inverted. Length of first image A 1B1 would be
40 A 1B1 = 1.2 x 3 = 3.6 cm (inver(ed).
=> µ=-=1.6
25
For reflection from mirror, we have
=> µ=1.6
Image formed by lens (A1B1 ) will behave like a virtual
object for the mirror at a distance of 30 cm from it as
Problem 14
shown. Therefore u = +30 cm , f = -30 cm .
A convex lens of focal length 15 cm and a concave
mirror of focal length 30 cm are kept with their optic axis 1 1 1
Applying mirror formula, -+-=-,we get
PQ and RS parallel but separated in vertical direction by V U f
0.6 cm as shown. The distance between the lens and mirror 1 1 1
ls 30 cm. An upright object AB 'of height 1.2 cm is placed -+-=--
v 30 30
on tlie optic axis .PQ of the lens at a distance of 20 cm
=> V=-15 cm
fr~m the lens. If A' B' is the image after refraction from the
and linear magnification is given by
lens and the reflection from the mUTor, find the distance of
A'B' from the pole of the mirror and obtain· its V (-15) 1
m =--=---=+-
magnification. Also l~cate positions of A' and B' with , u +30 2
respect to the optic axis RS .

c:=================================== 1.1os:-;,
www.puucho.com
Rahul Sardana with www.puucho.com

Advanced JEE Physics Optics & Modern Physics

So, the final image A'B' will be located at a distance of Problem 15


15 cm from the mirror (towards right) and since Monochromatic light is incident on a plane interface
AB between two media of refractive indices tt 1 and n2
magnification is +.!., length of final image would be
2 (n2 > n1 ) at an angle of incidence O as shown in the figure.
The angle 0 1s infinitesimally greater than the critical
A'B'=3.6x½=1.8 cm angle for the two media so that total internal reflection
=> A'B' = 1.8 cm takes place. Now if a transparent slab DEFG of uniform
thickness and of refractive index n3 is introduced on the
Since the p"oint B1 is 0.6 cm above the optic axis of mirror,
interface (as shown in the figure), show that for any value
therefore, its image B' would be (0.6)(½) =0.3 cm above of n3 all light will ultimately be reflected back again into
medium II. Consider separately the cases
optic axis. (a) n3 < n1 and
Similarly, point A 1 is 3 cm below the optic axis, therefore,
(b) n, > n1
its image A' will be 3 x ½=1.5 cm below the optic axis as Medium I
(n,)
shown. D ------------ E
: Medium Ill :

I
G: (nJ lF
A--~~-~--~---B
e,' Medium II
Optic Axis of Lens
''' (n,)
1.5 cm Optic Axis of Mirror ''
'
A' ''
A'B'= 1.8 m
'
Solution
15cmj
At interface AB, 0 is infinitesimally greater (slightly
Net magnification of the image is given by
greater) than the critical angle for interface, so
m=m1 xm =(-3i(+½)=-%
2
0>sin-
1
(::)

~ A'B'=(m)(AB)=(-%)<1.2)=-1.il cm (a) When n3 < n1

C.ONCEPTUAL NOTE(S)
If the co·ordinates of the object (X 0 , Y0 ) are generally known to
us with reference to the pole of an optical instrument (whether it,
1 1
is a lens 'or a mirror), the corresponding co·ordinateS of image sin- ( :: ) < sin- ( ~)
(X,.~) are found as follows. ·
Hence, critical angle for Medium III and Medium II will
X1 is obtained using .!+.!. := ! (for a mirror) be less than the critical angle for Medium II and
V U f
Medium I. So, if TIR is taking place between Medium I
1 1 1
OR ;;-u = f (for a lens) and Medium II, then TIR will definitely take place
between Medium 1 and Medium III.
Here, v is actually X1 and· u is X0 le., the above formula can
(b) When n3 > n1 , then Mo further cases may arise.
be written as .!_ ± ...!.. =! · Case I: n1 < n3 < n2
X1 X0 f
In this case there will be no TIR between Medium I and
Similarly, Yi Is obtained from m ='fj Medium III but TIR will take place between Medium III
and Medium II. This is because :
Here, I is Yi and O -is Y0 i.e., the above formula can -be
Ray of light first enters from Medium II to Medium III
written as ·m =~ or Yi = mY0 • i.e., from-denser to rarer. So,
Y,
i>8

= 1.106 i=:===================================
www.puucho.com
Rahul Sardana with www.puucho.com

Ray Optics
Medium I Medium I

Medium Ill ,.' q Medium Ill I'


p' '

Medium II e,' i>9 i <8

Applying Snell's Law at P , we get Since, sin 8 slightly greater than n,


n,
n2 sine= n3 sini
So, sin i will be slightly greater than "'- x n, = n,
~ sini=(::}ine n3 "2 ~

However " 1 is sin C for Medium I and Medium ill


Since, sin8 is slightly greater than !!,_, so n,
n, interface, so
sini is.slightly greater th~ " 2
x5..= 1Z-i sini > sinC for Mediwn I and Medium Ill interface.
n3 1½ n3 ~ i >(C),,111
However, 1?-i is nothing but sinC for Medium I, Therefore, TIR will again take place between Medium I
n, and Medium III and the ray will be reflected back.
Medium III interface, so
sini is slightly greater than sinC for Medium I,
Medium III interface. CONCEPTUAL NOTE(S)
~ i>(C)1>m The Cases I and II for n3 > "1 can be explained by single
equation only. But two cases are deliberately formed for b~tter
Hence, TIR will now take place on Medium I and
understanding of refraction, Snell's Law and total internal
Medium III interface and the ray will be reflected back reflection (TIR).
to Medium IIL
Case II: f1i < n2 < n3
1bis time while moving from Medium II to Medium III,
ray of light will bend towards normal. Again applying
Snell's Law at P , we get
n2 sin0·= n3 sini

~ sini = " 2 sin9


n,

================================== 1.101=
www.puucho.com
Rahul Sardana with www.puucho.com

r• ,• ,

1bis section contains Single Correct Choice Type Questions. Each question has four choices (A), (B), (C) and (D), out of which
ONLY ONE is correct.
. . -
1. A transparent hemisphere has 'a radius of curvature
8 cm and an index of refraction of 1.6. A small object
0 is placed on the axis halfway between the plane 0 2mm
surface and the Spherical surface i.e. 4 cm from each.
1

The distance between the two images when- viewed


along t,he axis from the two sides of the hemisphere is
approximately 1+- 20 cm --+1.
(A) the distance between the.images is 2 mm
./ (B) the distance between the images is 4 mm
µ= 1.6
(C) the distance between the two images formed by
--+-- · suchalensis-6min ·
(D) only one image will be formed by the lens

5. Ah object is placed at 26 cm from a convex mirror of


(A) 7.5cm (B) 8.5 cm · -focal length 20 cm. The distance of the image from the
(C) 2.5cm (D) 13.5 cm pole of the mirror is
(A) infinity (B) 10 cm
2. A square wire of side 3.0 cm is placed 25 cm in front of (C) 15 cm (D) 40 cm
a concave mirror of focal length 10 cm with its centre on
the axis of the mirror and its plane normal to the axis. 6. A point object is placed at a distance of 25 cm from a
The area enclosed by the image of the wire is convex lens of focal length 20 cm . When a glass slab of
(A) 7.5 cm' (B) 6.0 cm2 thickness t and refractive index 1.5 is inserted between
(C) 4.0 cm' (D) 3.0 cm' the lens and the object, the image is formed at infinity.
The thickness t of the slab is,
3. An object is placed at a distance 2/ from the pole of a (A) 5 cm (B) 10 cm
convex mirror of focal length f . The linear (C) 15 cm (D) 20 cm
magnification is
1 2 7. Light is incident normally on face AB of a prism as
(A) 3 (B)
3 showri in figure. A liquid of refractive index µ is
3 placed on face AC of the prism. The prism is made of
(C) 4 (D) 1
glass of refractive index i. The limits of µ for which
2
4. A convex lens of focal length 10 cm is painted black at total internal reflection takes place at the face AC is
the middle portion as shown in figure. An object is
placed at a distance of 20 cm from the lens. Then

= 1.108 c:===================================
www.puucho.com
Rahul Sardana with www.puucho.com

Ray Optics

I Liquid I (C) concave, 15 cm (D) convex, 15 cm


14. A boy of height 1.5 m with his eye level at 1.4 m stands
before a plane mirror of length 0.75 m fixed on the well.
~ c The height of the lower edge of the mirror above the
90° floor is 0.8 m. Then
(A) the boy will see his full image.
B
(B) the boy cannot see his hair.
(A) µ<-.
-J3 (C) · the boy cannot see his feet.
2 (D) the boy cannot see both his hair and feet.

(C) µ<-4-
3-/3
15. A horizontal ray of light
passes through a prism of M

8.
µ =1.5 whose apex angle
An object is placed in front of a convex mirror at a
distance of 50 cm. A plane mirror is introduced is 4 ° and then strikes a
covering the lower half of the convex mirror. If the vertical mirror M as
distance between the object and the plane mirror is shown. For the ray, after
30 cm , there is no parallax between the images formed reflection to become
by the two mirrors. The radius of curvature of the horizontal, the mirror
convex mirror is must be rotated through
(A) 60cm (B) 50 cm an angle of
(C) 30 cm (D) 25 cm (A) 1° (B) 2°
(C) 30 . (D)
9. A concave lens forms.the image of an object such that
the distance between the obje_ct and image is 10 cm 16. A man of height 1.6 m wishes to see his full image in a
plane mirror placed at a distance of 2 m. The minimum
and the magnification produced is ¼. The focal length length of the mirror should be
(A) 0.4 m (B) 0.8 m
of the lens will be (C) 1.6 m (D) 2.4 m
(A) 10 cm (B) 8.6 cm
(C) 6.2 cm (D) 4.4 cm 17. A plane mirror reflects a beam of light to form a real
image. The incident beam is
10. For a concave mirror, the magnification of a real image (A) parallel (B) convergent
was found to be twice as great when the object was (C) divergent (D) any one of the above
15 cm from the mirror as it was when the object was
20 cm from the mirror. The focal length of the mirror is 18. A plane mirror is approaching you at 10 cms-1 • You
(A) 5,0 cm (B) 7.5 q:n can see your image in it. The image will approach ·you
(C) 10 cm (D) 12.5 cm with a speed
(A) 5 cms-1 (B) 10 cms-1
11. The image formed by a convex mirror of focal length
(C) 15 cms-1 (D) 20 cms-1
20 cm is half the size of the object. The distance of the
object from the mirror is
(A) 10 cm (B) 20 cm 19. An object is placed at A(DA> f) , where, f is the focal
(C) 30 cm (D) 40 cm length of the lens. The image is formed at B . A
perpendicular is erected at O and C is chosen such
12. A concave mirror of focal length_ f in vacuum is placed that LBCA = 90°. Then the value of f (in tenns of a,
in a medium of refractive index 2. Its focal length in the bandc)is
medium is C
(A) L (B) f
2
(C) 2/ (D) 4/
: 0 :
13. A spherical mirror forms an ere~t image three times the ' '
size of the object. If the distance between the object and :' :'
the image is 80 cm, the nature and the focal length of :' :'
the mirror are ': . - a -a.:+-- b ........:'
(A) concave, 30 cm (B) convex, 30 cm

i=:=================================== 1.109=
www.puucho.com
Rahul Sardana with www.puucho.com

Advanced JEE Physics OpHcs & Modem Physics

26. A concave mirror forms the image of an object on a


(a+ b)3 (a+b)c
(A) -c-,- (B) screen. If the lower half of the mirror is covered with an
(a+c)
opaque card, the effect would be
c' a' (A) to make the image less bright.
(C) (D) - - -
a+b a+b+c (B) to make the lower half of the image disappear.
(C) to make the upper half of the image disappear.
20. An observer moves towards a plane mirror with' a (D) to make the image blurred.
speed of 2 ms-1 . The speed of the image with respect to
the observer is 27. Two plane mirrors are inclined at 70°. A ray incident
(A) 1 ms01 (B) 2ms-1 On one mirror at angle 0, after reflection falls on the
(C) 4ms-1 (D) 8 ms-1 second mirror and is reflected from there parallel to the
first mirror. e is
21. A concave mirror of focal length / produces a real
(A) 45° (B) 50°
image n times the size of the object. The distance. of the (C) 55° (D) 60°
objet from the mirror is
(A) (n-1)/ (B) Cn+1)f 28. A man stands in a room with his eyes at the centre of
(C) ( n:1 )t (D) ( n:1 )t the room. The height of the ceiling is H . The length of
the shortest plane mirror, fixed on the wall in front of
the man, so that the man can see the full image of the
22. Two plane mirrors are arranged at right angles to each wall behind him is
other as shown in figure. A ray of light is incident on H
(A) 2H (B)
the horizontal mirror at an angle e. The value of 0 for 3 2
which the ray emerges parallel to the incoming ray after
reflection from the vertical mirror is (C) H (D) H
3 4

29. An object is ·placed between two parallel mirrors. The


number of images formed is
(A) 2 (B) 4
(A) 30' (B) 45° (C) 8 (D) infinite
(C) 60' (D) all of the above
30. A bulb is placed between two plane mirrors inclined at

(;r
23. A convex mirror of focal length / produces an image an angle of 60° . The number of images formed is
(A)5. (B)6
of the size of the object. The distance of the object (C) 4 (D) 3
from the mirror is 31. Two plane mirrors are placed perpendicular to each
(A) nf (B) l_ other. A ray strikes one mirror and after reflection falls
n on the second mirror. The ray after ,reflection froni. the
(C) (n+l)f (D) (n-1)/ second mirror: will be
(A) perpendicular to the original ray.
24. A real image formed by a concave mirror is 4.5 times (B) parallel to the original ray.
the size of the object. If the mirror is 20 cm from the (C) at 45° to the original ray.
object, its focal length is (D) can be at any angle to the original ray.
90 (B) 120 cm
· (A) -cm
11 11 32. A real image is formed by a convex lens, then it is
180 brought in contact with a concave lens such that again a
(C) 150 cm (D) -cm
11 11 real image is formed. This image will
(A) remain in its original position
25. An object is placed 10 cm in front of a convex mirror of (B) shift towards the lens system
focal length 20 cm. The distance of the image from the (C) shift away from the lens system
mirror is (D) shift to infinity
(A) 10 cm (B) 20 cm 33. Plane mirrors A and B are kept at an angle 0 with
3 3
respect-to each other. Light falls on A, is reflected, then
40
. (C) 10 cm (D) -cm falls on B and is reflected. The emergent ray is opposite
3
to the incident direction. Then the angle 8 is equal to

= ====================================
1.110

www.puucho.com
Rahul Sardana with www.puucho.com

Ray Optics
(A) 30° (B) -450
(C) 60° (D) 90° (D) .-./3 '
34, A point source of light B · is 4i. An object is moving towards a concave mirror of focal
placed at a distance L in front of f J B length 24 cm. When it is at a distance of 60 cm from the
1:

l
the centre of a mirror of width d mirror its speed is 9 ems-' . The_ speed of its image at
hung vertically on a wall. A man t that instant; is
walks in front of the mirror along
(A) 4 ems·' towards the mirror
a line parallel to the mirror at a
distance 2L from it as shown. (Bj 9 ems·' towards the mirror
I+- L --.i
The gr~atest distance over which M--- 2L -------+l (C) 4 ems·' away from the mirror
he .can see the image of the light
(D) 9 cms·1 away from the mirror
source in the mirror is
(A) d (B) d • 42. A ray of light passes through an equilateral prism such
2
(C) 2d

35. A thin rod of length f


(D) 3d

lies along the axis of a concave


incidence and each is equal to (
prism. The angle of deviation is
¾ r.
that the angle .of emergence is equal to the angle of

of the angle of

. nui;or of focal length f . One end of its image touches (A) 45° (B) 39°
(C) 20° (D) 30°
an end of the rod. The length of the image is
(A)° f (B) j_ 43. An object is placed in front of a concave mirror of focal
2 length f as shown in figure. The correct shape of the
(C) 2f (D) j_ image is represented by
4

36. How many images will be formed if two mirrors are a C


fitted on adjacent walls and one mirror oi:,. ceiling?
(A) 5 (B) 7
(C) 11 (D) 2

37. The wavefront that represents the light waves travelling I+- X > 2f ----+(

in vacuum along the y-axis is


(A) x + y + z = constant (B)_ x = constant
(C) y = constant (D) z =constant
(A) (B)
38. A boy stands straight in front of a mirror at a distance
of 30 cm from it. He sees his erect image whose height is
.!:.5 of his-real height. The mirror he is using is
.
(A) plane (B) convex
(C) concave (D) piano-concave
(C) (D)
39. The image of an object placed in front of a concave
mirror of focal length 12 cm is formed at a point which
is 10 cm more distant from the mirror that the object.
The magnification.of the image is
(A) 1.5 (B) . 2 44. The index of refraction of diamond is 2.0. Velocity of
(C) 2.5 (D) 3 light in diamond in cms·1 is approximately
(A) 6xl0 10 • (B) 3xl010
40. The minimum value of the refractive index for a 10
(C) 2 X 10 (D) 1.5 X 1010
90° - 45° - 45° - prism which is used to deviate a beam
through 90° by total internal reflection is
45. A plane mirror is placed at origin parallel of y-axis,
(A) ~ (B) ·..fi. facing the positive x-axis. An. object starts from
-./3 '
================================== = 1.111

www.puucho.com
Rahul Sardana with www.puucho.com

Advanced JEE Physics Optics & Modem Physics

50. Total internal reflection of a ray of light is possible


(2,0,0) m with a velocity of (21+2])ms-1 • The
when the ray goes from
relative velocity of image with respect to object is along (A) denser to rarer medium and the angle of incidence
(A) positive x-axis (B) positive y-axis is greater than the critical,angle.
(C) negative x-axis (D) negative y-axis (B) denser to rarer medium and the angle of incidence
is less than the critical angle.
46. A ray of light passes from vacuum into a medium of (C) rarer to denser medium and the angle of incidence
refractive index n . If the angle of incidence is twice the is greater than the critical angle.
angle of refraction, then the angle of incidence is (D) rarer to denser medium and the angle of incidence
is less than the critical angle.
(A) cos-'(%) (B) sin-'(%)
51. The critical angle of light going from medium A into
(C) 2cos-'(%) (D) 2sin-'(%) medium B is 0. The speed of light in medium A is v .
The speed of light in medium B is
47. A point of source of light is placed at the bottom of a V
(A) (B) vsin0
vessel containing a liquid of refractive index ~. A sine
3 V
(C) (D) vtan0
person is viewing the source from above the surface. lane
There is an opaque disc of radius 1 cm floating on the
surface. The centre of the disc lies vertically above the 3 .
\ source. The 'liquid from the vessel is gradually drained 52. Glass has refractive index 2 and water has refractive
out through a tap. The maximum height of the liquid
for which the source cannot be seen at all from above is index ±.
3
If the speed of light in glass is 2.00 x 10 8
ms-1 ,
3 4
(A) -cm (B) -cm the speed of light in water in ms-1 is
2 3
2 3 (A) 1.50xl08 (B) 1.78xl08
(C) -cm (D) - cm (C) 2.25xl08 (D) 2.67xl08
3 4

48. A beam of light consisting of red, green and blue 53. Two sides of an isosceles right prism are coated with a
f=Olours is incident on a right-angled prism as shown. reflecting coating. A ray of light falls on the hypotenuse
The refractive index of the material of the prism for the at an arbitrary angle i . The value of i for which the ray
above red, green and blue wavelengths are 1.39, 1.44 leaving the prism is parallel to the incident ray is
and 1.47 respectively. The prism will
. ,,,/
I/

1.
µ=-
../3
(A) separate part of the red colour from the green and
blue colours.
(B) separate part of the blue colour from the red and (A) 30°
green colours. (B) 60°
(C) separate all the three colours from one another. (C) 450
(D) not separate even partially any colour from the
(D) any arbitrary angle from 0 < i < 2:
other two colours. 2·

49. A thin prism P1 with angle 4° and made from glass of 54. A diver in a lake wants to signal his distress to a person
refractive index is 1.54 is combined with another thin sitting on the edge of the lake flashing his water proof
prism P2 made from glass of refractive index 1.72 to torch. He should direct the beam
produce dispersion without deviation. The angle of the (A) vertically upwards.
prism P2 is (B) horizontally.
(C) at an angle to the vertical which is slightly less
(A) 5.33° (B) 4° than the critical angle.
(C) 30 (D) 2.6° (D) at an angle to the vertical which is slightly more
than the critical angle.

= ====================================
1.112

www.puucho.com
Rahul Sardana with www.puucho.com

Ray Optics

55. Critical angle of light passing from a glass to water is (Refractive Index of Glass = 1.5, Refractive Index of
minimum for Water =4/3)
(A) red colour (B) green colour
(C) yellow colour (D) violet colour (A) sin-
1
( 1)
3
(B) . -1( - 1 )
sm
../3
56. Mirage is observed in a desert due to the phenomenon
of (C) sin-1( f)
4
(D) . -1( -9../3)
sm -
16
(A) interference (B) total internal reflection
(C) scattering (D) double refraction 62. A stone lies at the bottom of a stream. A boy wants to
hit it with a stick. Taking aim the boy holds the stick in
57. The distances of an object and its virtual image from the the air at an angle of 45° . At what distance from the
focus of a convex lens of focal length f are 1 cm each, stone will the stick hit the bottom, if the depth is 32 cm
then f is (given 'µw = 4/3 )
(A) (2+"2) cm (B) (,Jz +1) cm (A) 8 cm (B) 12cm
(C) 16cm (D) 12v'2 cm
(C) 2v'2 cm (D) 4 cm
63. When the surface of the lake is calm, a fish submerged
58. Total internal reflection can occur when light tends to in water will see the entire out-side world within
pass from inverted cone whose apex is situated at the eye of the
(A) a denser to a rarer medium. fish and the cone subtends an angle of
(B) a rarer to a denser medium. (A) 10° (B) 60°
(C) one medium to another of different refractive (C) 98° (D) 30°
index irrespective of which medium has greater
refractive index. 64. A ray of light strikes a glass slab of thickness t . It
(D) one medium to another of equal refractive index. emerges on the opposite face, parallel to the incident
ray but laterally displaced. The lateral displacement is
59. A diverging beam of light from a point source S l!.x.
having divergence angle a falls symmetrically on a (A) l!.x = 0 (B) l!.x = tsin(i-r )cosr
glass slab as shown. The angles Of incidence of the two
tsini l!.x tsin(i-r)
extreme rays are equal. If the thickness of the glass slab (C) l!.x=-- (D)
is t and the refractive index n , then the divergence
cosr cosr
angle of the emergent beam is
65. In cold countries the phenomenon of looming (i.e. ship
s appears in the sky) takes place because
(A) refractive index of air decreases with height.
" (B) refractive index of air increases with height.
(C) refractive index does not change with height.
(D) refractive index becomes infinity at the surface.

n
"ft 66. If D is the deviation of a normally falling light beam on
:t a thin prism of angle A and Ois the dispersive power
of the same prism then
(A) ZERO (B) a (A) D is independent of A .
(C) . -1(1)
sm - (D) 2 sm 1(1)
· - ·- (B) D is independent of refractive index.
n n (C) 8 is independent of refractive index.
(D) 8 is independent of A .
60. A ray incident at an angle of incidence 60° enters a
glass sphere of refractive index µ = ../3 . This ray is 67. For an equilateral prism, it is observed that when a ray
strikes grazingly at one face it emerges grazingly at the
reflected and refracted at the farther surface of the
other. Its refractive index will be
sphere. The angle between reflected and refracted rays
at this surface is
(A) 40° (B) 60°
(A) f (B) l
(C) 70° (D) 90° (C) 2

61. A water film is formed on a glass block. A light ray is 68. A rectangular block of glass (refractive index 3/2 ) is
incident on water film from air at an angle 60°. What is kept in water (refractive index 4/3 ). The critical angle
the angle of incidence on glass block? for total internal reflection is

r=:=================================== 1.113=
www.puucho.com
Rahul Sardana with www.puucho.com

Advanced JEE Physics Optics & Modern Physics

emerges as QR. The condition of minimum deviation is


(A) sin-1 ( %) for a ray of light passing from glass to satisfied in the prisms
water.

(B) sin-1 ( %) for a ray of light passing from water to B

glass. \ __

C ._.
1
(C) sin- ( ¾) for a ray of light passing from water to 0: R
glass. (A) A and C (B) Band C
(D) sm . -1(8)- for a ray of light passing from glass to
9
(C) AandB (D) in ali prisms A, Band C

air. 77. A beam of white light is


incident on a hollow prism of
69. The refractive index of a given piece of transparent glass. Then
quartz is greatest for (A) the light emerging from
(A) red light (B) violet light prism gives no spectrum.
(C) green light (D) yeliow light (B) the light emerging from
prism gives spectrum but the bending of ali
70. A well cut diamond appears bright because colours is away from base.
(A) it emits light (C) the light emerging from prism gives spectrum, all
(B) it is radioactive the colours bend towards base, the violet most and
(C) of total internal reflection red theleast.
(D) of dispersion (D) the light emerging from prism gives spectrum, all
the colours bend towards base, the violet the least
71. The maximum refracting angle of ·a prism of refractive and :red the most.
index 2is
(A) 30' (B) 45' 78. If the critical angle for the medium of a prism is C and
(C) 60' (D) 90' the angle of prism is A , then there will be no emergent
ray when
72. When light passes from one medium to another, the (A) A<2C (B) A=2C
physical quantity that remains unchanged is (C) A>2C (D) AS2C
(A) velocity (B) wavelength
(C) frequency (D) None of these 79. The angle of a prism is 60° . What is the angle of
incidence for minimum deviation? The refractive index
73. A monochromatic beam of light passes from a denser to of the material of the prism is -/'i. .
a rarer medium. As a result its
(A) 45° (B) 60°
(A) velocity increases (B) velocity decreases
(C) frequency decreases (D) frequency increases (C) 30° (D) sin-'(¾)

74. When a ray of light enters a glass slab from air


(A) its wavelength decreases. 80. A ray of light is incident at angle i on one surface of a
(B) its wavelength increases. prism of smali angle A and emerges normally from the
(C) its frequency increases. opposite surface. H the refractive index of the material
(D) neither its wavelength nor its frequency changes. of the prism is µ, the angle of incidence i is nearly
equal to
75. A number of images of a candle flame are seen in a A
(A) A (B)
thick mirror µ 2µ
(A) the first image is the brightest. µA
(B) the second image is the brightest. (C) µA (D)
2
(C) the last image is the brightest.
(D) all images are equally bright.
81. If 'µ i represents the refractive index when a ray of light
76. Three glass prisms A, B and C of same refractive index goes from medium i to medium j, then the product
are placed in contact with each other as shown in figure 2
µ 1 x 3µ 2 x 4µ 3 is equal to
with no air gap between the prisms. Monochromatic ray 3 3
of light OP passes through the .prism assembly and (A) µ1 (B) µ2

= ====================================
1.114

www.puucho.com
Rahul Sardana with www.puucho.com

Ray Optics
1 89. The path of a refracted ray of light in a prism is parallel
(C) 'µ, (D) 'µ, to the base of the prism only when the
(A) light is of a particular wavelength.
(B) ray is incident normally at one face.
82. An air bubble inside a glass slab ( µ = ¾) appears to be (C) ray undergoes minimum deviation.
(D) prism is made of a particular type of glass.
6 cm deep when viewed from one side and 4 cm deep
when viewed from the opposite side. The thickness of
90. A convex lens forms a real image three times larger
the slab is
than the object on a screen. The object and screen are
(A) 10 cm (B) 6.67 cm
moved until the image becomes twice the size of the
(C) 15 cm (D) None of the above
object. If the shift of the object is 6 cm then the screen
has to be shifted by
83. The refracting angle of a prism is A and the refractive

index of the material of the prism is cot( 1). The angle


(A)' 9 cm
(C) 36cm
(B) 18 cm
(D) 72cm

of minimum deviation is 91. A vessel of depth d· is half filled with a liquid of


(A) 180°-3A (B) 180°+2A refractive index µ 1 and the other half is filled with a
(C) 90°-A (D) 180°-2A
liquid of refractive index µ 2 • The apparent depth of the
84. The angle of a prism is 30°. The rays incident at 60° at vessel, when looked at normally, is
one refracting face suffer a deviation of 30°. The angle
of emergence is
(A) d(µ, +µ,) (B) a(_!_+_!_J
µ1 µ2
(A) 0°
(C) 60°
(B) 30°
(D) 90°
d
(C) 2(µ,+µ,) (D) ~(~ + :J
85. A ray falls on a prism A
92. Two point sources S1 and S2 are 24 cm apart. Where
ABC(AB=BC) and travels as
sho'Wn in the figure. The minimum
should a convex lens of focal length 9 cm be placed in
refractive index of the prism between them so that the images of both sources are
material should be formed at the same place?
(A) 6 cm from S1 (B) 10 cm from S1
4
(A) (B) J'i. B C (C) 12 cm from S1 (D) 15 cm from S1
3
(C) 3 (D) ../3
2 93. Light travels ·through a glass plate of thickness t and
having refractive index n . If c is the velocity of light in
86. Critical angle is minimum when a light ray passes from vacuum, the time taken by light to travel this thickness
(A) air to glass (B) glass to air of glass is
(C) glass to water (D) water to glass (A) t
nc (B) n 2c
87. A point source of light is placed 4 m below the surface (C) nt n 2t
(D)
of a liquid of refractive index ~. The minimum C C
3
diameter of a disc, which should be placed over the 94. A fish looking up through the water sees the outside
source, on the surface of the liquid to cut off all light world contained in a circular horizon. If the refractive
coming out of water, is
(A) oo (B) 6m index of water is ±
3
and the fish is 12 cm below the
(C) 4m (D) 3m surface of water, the radius of the circle in cm is
(A) 36../5 (B) 36ft
88. A man standing in a swimming pool looks at a stone
lying at the bottom. The depth of the swimming pool is 36
(C) ,ff (D) 4../5
h . At what distance from the surface of water is the
image of the stone formed? Line of vision is normal.
Refractive index of water is n . 95. A diver inside water sees the setting sun at
n (A) 41 ° to the horizon (B) 49° to the horizon
(A) h (B) (C) 0° to the horizon· (D) 45° to the horizon
n h
(C) h (D) Im

================================== = 1.115

www.puucho.com
Rahul Sardana with www.puucho.com

Advanced JEE Physics Optics & Modem Physics .

incidence at which minimum deviation will be obtained


96. If the refractive index of water is i3 and that of glass is is

~ , then the critical angle of incidence for light tending (A) (-/2)
• -1 -
Sill
3
(B) .30°
3
to go from glass to water is (C) 60° (D) 45°
(A) sin- ¾)
1
( (B) sin-
1
( ¾) 103. Light is incident at an angle ex on one planar end of a

(C) . -1(4)
sm -
5
(D) . -1( 32)
Sill -
transparent cylindrical rod of refractive index n . The
least value of n for which the light entering the rod will
not emerge from the curved surface of rod, irrespective
97. Two media A and. B of refractive indices µ 1 =1.5 and of value of a is

µ 2 = 2 are separated by x-z plane. A ray of light (A)


1
-/2 (B) -/2
travels from A to B . The incident ray and the reflected
1
ray are represented by unit vectors U1 = ai + b] and (C) ./3 (D) ./3
U2 =ci+d}. Then
a 3 a 4 104. For a prism the refractive index ( µ) is related to
(A) -=- (B)
C 4 C 3 wavelength (;\.) as µ = A +
).
~ . The dispersive power is
b 3 b 4
(C) -=- (D)
d 4 d 3 large if
(A) A is large (B) B is large
98. The speed of light in medium A is 2.0xl0 8 ms-1 and (C) µ is large (D) A and µ are large
that in medium B is 2.4 x 10 8 ms-1 . The critical angle of
105. An infinitely long rod lies along the axis of a concave
incidence for light tending to go from medium A to
mirror of focal length f . The near end of the rod is at a
medium Bis
distance u > f from the mirror. The length of the image
(A) . -1( 125 )
Sill - (B) sin-
1
( ¾) of the rod is

(D) . -1(3)
Sill -
4
(A) _!![_
u+f
(B) __f_
u+f
(C)
__f_ (D)
_!![_
99. The speed of light in glass of refr~ctive index 1.5 is
2 x 108 ms-1 • In a certain liquid the speed of light is
u-f u-f
2.5 x 108 ms-1 • The refractive index of the liquid is 106. Two transpar_ent slabs have the same thickness as
(A) 0.64 (B) 0.80 shown in figure. One is made of material X of refractive
(C) 1.20 (D) 1.44 index 1.5. The other is made of two materials Y and Z
having thicknesses io the ratio 1 : 2. The refractive index
100. A ray of light travelling inside a rectangular glass block of Z is 1.6. If a monochoromatic parallel beam passing
of refractive index -/2
is incident on the glass-air through the slabs has the same number of wavelengths
inside both, the refractive index of Y is
surface at an angle of incidence of 45° . The refractive
1+--t-i
index of air is 1. The ray will
(A) emerge into air without any deviation.
(B) be reflected back into glass.
(C) be absorbed.
(D) emerge into air with an angle of refraction equal to
90°.

101. A fish in water sees an object which is 24 cm above the


surface of water. The height of the object above the
surface of water that will appear to the fish is
---~LB .........c .......
(A) 24 cm (B) 32 cm 1.6
(C) 18 cm (D) 48 cm
(A) 1.1 (B) 1.2
102. The angle of minimum deviation equals the angle of (C) 1.3 (D) 1.4
prism A of an equilateral glass prism. The angle of

= ====================================
1.116

www.puucho.com
Rahul Sardana with www.puucho.com

., ·Say Optics
107. A curved mirror of focal length f (in vacuum) is (C) 30 , (D) 25
placed in a medium of refractive index 2. Its new focal
length in the medium is f' . 114. A prism having an apex angle 4° and refractive index
· 1.5 is located in frorit of a vertical plane mirror as
(A) f' < f (B) f' > f
shown in figure. The total angle through which the ray
(C) f'= f (D) f'e;,f is deviated after reflection from the mirror is given by

108. If t 0 is the absolute permittivity of free space, µ 0 is

absolute permeability of free space, e is the permittivity


of medium, µ is permeability of medium and n is the
refractive index of medium then,

(A) (B) n=J µoto


µe
(A) 176° (B) 4'
(C) n=Jµ~, (C) 178° (D) 2'

115. A slab of glass of thickness 3 cm and refractive index ~


109. The critical angle of glass (µ, = f) is 0 1 and that of
is placed with its face perpendicular to the principal
2

axis of the concave mirror. If the radius of mirror is 10


water (µw = ¼) is 02 • The critical angle for water-glass cm, the distance at which an object must be placed from
interface is the mirror so that the image coincides-with.the object is
(A) less than 01 (B) less than 82 (A) 9 cm (B) 10 cm

;
(C) 11 cm (D) 12cm
(C) between 0, and 0, . (D) greater than 02
116. A tank contains a transparent p
110. Two plane ~ors M1 and .M 2 are inclined to eaCh liquid of refractive index n the
bottom of which is made of a

ill
other at 70°. A ray incident on the mirror M1 at an
angle 0 falls on M, and is then reflected parallel to M 1 mirror as shown. An object O lies
at a height d above the mirror. A d
for
person P vertically above the l .

(A) 0=45° (B) 0=50' object sees O and its image in the
(C) 0 =55° (D) 0=60° mirror and finds the apparent separation to be
2d
111. An object is placed at 20 cm from a convex mirror of (A) 2nd (B)
n-1
focal length 20 cm. The distance of the image from the
(C) 2d d
pole of the mirror is (D) -(l+n)
(A) infinite (B) 10 cm n n
(C) 15cm (D) 40cm
117. A ray of light enters an anisotropic medium from
112. The s~ (diameter D ) subtends an angle 0 radian at the vacuum at grazing incidence. If 0 is the angle made by
the reflected ray inside the medium with the interface
pole of a concave . mirror of focal length f . The
and n ( 0) is the refractive index ofthe medium then,
diameter of the image of the sun formed by the mirror
is (A) n(0)sin0=1 (B) n(0)cos0=1
(A) f0 (B) 2f0 (C) n_(0) =l (D) n(0) =l
2f0 sm8 cos8
(C) (D) D0
D
118. · A person runs with a speed u towards a bicycle. moving
away from him with speed v. The person approaches
113. An object is placed in front of a convex mirror. ai a
his image in the mirror fixed at the rear of bicycle with a
distance of 50 cm. A plane mirror is introduced
speed of
covering the lower hall of the convex mirror. If the
(A) u -v (B) u-2v
distance.. between the Object and the plane ·mirror is
(C) 2u-v (D) 2(u - v)
30 cm , there is no parallax between the images formed
by the two mirrors. The radius of curvature of the
~ 119. Light travels through a glass plate of thickness t
convex mirror (in cm) is
· having refractive index µ. If c is the velocity of light in
{A) 60 (B) 50

====================================1.111=
www.puucho.com
Rahul Sardana with www.puucho.com

Advanced JEE Physics Optics & Modem Physics

vacuum, the time taken by the light to travel this 125. The light on reflection from a plane mirror can give a
thickness of glass is real image when
(A) the convergent rays are incident on the mirror.
(A) I (B) µ'c
µc (B) the divergent rays are incident on the mirror.
(C) an object is placed very close to the mirror.
µI µ't
(C) (D) (D) an object is placed very far away from the mirror.
C C

126. Parallel beam of light is incident on the system of two


120. A real image I is formed by a converging lens L on its
convex lenses of focal length / 1 = 20 cm and
optic axis. On introduction of a rectangular glass slab of
thickness d and refractive index µ between the image Ji= 10·cm. The distance between the two lenses, so
and lens the image displaces it by that rays after refraction from both the lenses pass
(A) d(µ-1) away from L undeviated is
(B) d(µ-1) towards L

(C) a(1-;) away from L,

(D) d(1-;) towards L


f, f,
(A) 30 cm (B) 40 cm
121. When-a ray is refracted from one medium to another,
(C) 60 cm (D) 90 cm
the wavelength changes from 6000 A to 4000 A. The
critical angle for the interface will be 127. The plane faces of two identical piano convex lenses,
each with focal length f are pressed against each other
(A) cos- (¾)
1
(B) sin-'(1) using an optical glue to form a usual convex lens. The
distance from the optical centre at which an object must
(C) . -1(2)-3
sm (D) cos-' ( 1 ) be placed to obtain the image same as the size of object
is

122. A boy stands straight in front of a mirror at a distance (A) f_ (B) f_


4 2
of 30 cm away from it. He sees his erect image whose
height is one fifth of the original height. The mirror (C) f (D) 2/
used by him is
(A) plane (B) convex 128. A parallel beam of light incident on a concave lens of
(C) concave (D) piano concave focal length 10 cm emerges as a parallel beam from a
convex lens placed coaxially, the separation between
123. The image of an object, formed by a piano-convex lens the lenses being 10 cm. The focal length of the convex
at a distance of 8 m behind the lens, is real and is one- lens incmis
third the size of the object. The wavelength of light (A) 10 (B) 20
(C) 15 (D) 30
inside the lens is ~ times the wavelength in free space.
3
The radius of the cured surface of the lens is 129. A ray oflight is incident on a glass sphere of refractive
(A) 1 m (B) 2m index %. The angle of incidence for which a ray t.hat
(C) 3 m (D) 6m
enters the sphere· does not come out of the sphere is
124. A diverging beam of light from
a · point source S having
s (A) tan-1 (¾) (B) sin-1 (¾)
divergence angle a, falls (C) 45° (D) 90°
symmetrically on a glass slab as
shown. The angles of incidence 130. A thin prism P1 of angle 4° and made from glass of
of two extreme rays are equa( If t
the thickness of the glass slab is 1 n

t and the refractive index n , then the divergence angle


refractive index 1.54, is combined with another thin
prism P2 made from a glass with refractive index 1.72,
of the emergent beam is to produce dispersion without deviation. The angle of
(A) ZERO (B) a P2 is
(A) 5.33° (B) 4°
(C) sin-'(;;) (D) 2sin-'(;) (C) 3° (D) 2.6°

= ====================================
1.118

www.puucho.com
Rahul Sardana with www.puucho.com

Ray Optics
131. A transparent sphere of. radius R made of material of incidence i . It is observed that no ray emerges from the
refractive index ~ is kept in air. The distance from the other face. For this the minimum value of i should be
2 (A) µsinA-cosA
centre of the sphere must a point object be placed so as (BJ sin-'(sinA-µcosA)
to form a real image at the same distance from the
sphere is (C) sin-1 [~µ 2 -lsinA-cosA]
(A) R (B) 2R
(D) ~µ 2 -1 sin A- cos A
(C) 3R (D) 4R

132. An air bubble in water is to be placed in a way such that 138. As the position of an object (u) reflected from a
a real image is obtained at the same distance from concave mirror is varied, the position of the image ( v)
also varies. By allowing the u to change from O to +co,
bubble. Taking µw,.., =¼we have the distance of object the graph between v versus u will be
from the air bubble as
V
(A) R
(B) 2R
(C) 3R
(D) An air bubble is incapable to form a real image.
(A) f-+---•u (B)
1
133. An object is placed at a distance of 12 cm from a convex
lens on its principal axis and a virtual image of certain
V V
size is formed. On moving the object 8 cm away from
the lens, a real image of the same size as that of virtual
image is formed. The focal length of the lens in cm is
(C) (D) k:--t----+u
(A) 15 (B) 16
(C) 17 (D) 18

134. An air bubble inside a glass slab appears to be 6 cm


deep when viewed from one side and 139. A parallel beam of light emerges from the opposite
surface of the sphere when a point source of light lies at
4 cm deep when viewed from the other side. Assuming
the surface of the sphere. The refractive index of the
µg) =~ , the thickness of slab is .. sphere is
~· 2
(A) 3 5
20 (B)
(A) 10cm (B) -cm 2 3
3
(C) 15cm (D) 20cm (C) 2 (D) 5
2
135. On two sides of an oily paper screen, two bulbs A and B
are placed at a distance of 20 cm and 140. Two spherical mirrors M 1 and
30 cm, so that equal intensity is obtained on both sides M2 , one convex and other
of screen. If PA and P, be the powers of the bulbs A concave having same radius of
curvature R are arranged
and B respectively then PA is coaxially at a distance 2R
P,
(consider their pole separation to
(A) 0.44 (B) 2.25 be 2R). A bead of radius a is
(C) 1.5 (D) 0.67 placed at the pole of the convex M1 M,
mirror as shown. The ratio of the
136. An achromatic combination pair of a telescope objective
sizes of the first three images of the bead is
will be
(A) lenses of f = -50 cm and power +2 D (A) 1 : 2: 3 (B) 1 : :1:_: :1:_
2 3
(B) lenses of powers 3 D and -5 D respectively
1 1 1
(C) lenses of f = +20 cm and power -4.5 D (C) -·-:- (D) 3: 11: 41
3 ·11 41
(D) lenses of f = +40 cm and power +2 D
141. Light is incident at an angle a. on one planar end of a
137. A ray of light enters the face of a glass prism of transparent cylindrical rod of refractive index n . The
refracting angle A, refractive index µ at an angle of least value of n for which the light entering the rod will

================================== = 1.119

www.puucho.com
Rahul Sardana with www.puucho.com

Advqnced JEE Physics Optics & Modern Physics

not·en,.1;rie.-fromthe curved surfaCe of.rod, irrespective 146. Rays-of light from a luminous object are brought to
of :value of a. is .. focus at a point A. The rays are intercepted, before
meeting at A by a convex lens- of focal length 20 cm
(A) . }i .(B) .Ji. placed at 24 cm from A ,and are forced to meet at B.
Then AB e!J_uals
, (in cm)
1
(C) .J3 ' (D) .J3 (A) 12 - (B) 24
. (C) 6 (D) 48

142. A ray of light is incident on one face of prism with 147. A point object is placed at a distance of 0.3 m from a
refracting angle A (< 90°) . Tl_te incident ray is normal to convex lens of focal length 0.2 m cut into two equal
the other face of the prism, If e is the critical angle for . halves, each of wliich is displaced by .0.0005 m, as
prism-air interface, then the ray_ will emerge from this shown 'in figure. If. e, and e, be their optical centres
face only if ' then;
(A) cote< cotA + 1 · (B) cote> cotA+ 1
(C). cotA < cote +.1 (D) cot A> cote+ 1

143. The image of point P when viewed from top of the


glass Slabs is 0

µ=1.5 1·.scm (A) an ,itnag~ is f~rm~d at ~- distance ·of 0.6 m from e1


1.5cm or, C2 along principal ·a~is.
1

(B) two images are formed; one at a distance of 0.6 m ·


µ= 1.5
I 1.5cm
and other at a distance of 1.2 m from e, or e,
2 cm along principal axis.
P• (C) an image is formed at a distance of0.12 m from e,
(A) 2cmabove P (B) 0.5 cm above P ·or c; al~ng principal axis. . '.
(C) 0.5 cm below P (D) 1 cm above P (D) two images are formed at a distance. of.0.6 m from
C1 or C2 along principal axis at a separation of
144. An isosceles prism has refracting angle A . Its one face 0.003m.
is·si!vered (other than the base). A ray of light falling
normally on the face not silvered emerges through the 148. All of the following statements are correct except that
base of the prism normal to it. (A) the magnification produced by a convex mirror is
(A) A=45' . (B) A=90' always less than one . .
(C) A=36' (D) A=72' ·(B) a virtual, erect, same sized. image can be obtained
by using the plane mirror.
145. Two identical thin isosceles prisms of refracting angle (C) a virtual, erect, magnified image can be formed by
· A and refractive index µ'are placed with their bases using thE:! _concave mirror.
touching each other and this systeni can collec.ti_vely act (D) a' real, inverted, same sized image can be formed
as a crude converging lens. A parallel beam of light is by using .a convex mirro,r.
incident on this system as shown. The focal length of
this so. called ~onverging lens "is 149. A ray of ligltt undergoes deviation of 30° when
incident on an equilateral prism of refractive index .Ji. .
The angle made by the ray inside the prism with the
f
2h~--+--'+---
base of the prism is
(A) 0° (B) 15°
1 _·
+----'ri!I (C) 30° · (DY 45°

(A) (B) f= ! 150. A convex lens of focal length f forms an image of ·a


heavenly body. The area · of the image formed is
. h proportional to
(C) f (µ-1)A
(D) f=_E_ (A) f' , (B) f'
µ-1
'(C) !' (D) f'

= ==================================
1.120

www.puucho.com
Rahul Sardana with www.puucho.com

Ray Optics

151. A piano convex lens has a thickness of 4 cm. When (A)' n1 =n, =n, (B) n1 =n, ,en,
placed on a horizontal table with curved surface -in
(C) 1+~,"=n, +n, (D) l+·n;=n;+ni
contact with it, the apparent depth of the bottom most
point of the lens is found to be 3 cm. If the lens .is
inverted such that the plane face is in contact with tl~e 156. Four lenses are made from same type of glass. The
· · radius of curvature of each face is given . Out of .these,
table, the apparent depth of the centre of plane face is
25 · the 1ens having the greatest po~itiv:e. power is
found to be cm. The focal length of the lens is (A) 10 cm convex and 15 cm convex.
8 (B) 20 cm convex and 30 cm concave.
(A) 50cm (B) 75 cm (C) 15 cm convex and plane.
(C) 100cm (D) 150cm (D) 5 cm convex and 10 cm concave.
152. If an object is placed between two .parallel mirrors, an 157. The sides of an isosceles right angled prism are silvered.
infinite number of images are formed. If the mirrors are A ray of light falls on the hypotenuse of the prism at an
at a distance 2b and an object is placed at the middle of angle $0 as shown. The ray leaving the prism will
the two mirrors, the distance of the nth image from the
object is
1
(A) nb (B) -nb
2
1
(C) 2nb (D) -nb
4

153. A ray of light is incident on the plane mirror at rest. The


mirror starts turning at a uniform acceleration of (A) graze the face AC.
(B) emerge normally to the face AC.
21t rads' 2 • The reflected ray, at the end of -1 s must (C) be parallel to the incident ray.
4
(D) make an angle of 30° :with incident ray.
have turned through
(A) 90° (B) 45°
158. A right angled prism .(45°-90°-45°), of refractive index
(C) 22.5° (D) 11.25°
n has a plate of refractive index n1 ( n1 < n) cemented to
its diagonal face. The assembly is in air. A ray is
154. In the situation shown in figure, water (µw = ¾) is incident on AB as shown. If the ray strikes the diagonal
face AC at critical angle then
filled in a beaker upto a height of 10 cm . A plane
mirror is fixed at a height of 5 cm from the surface of (A) sini=(~) A
water. Distance of image from the mirror after reflection
from it of an. object' 0 at the bottom of the beaker is ' (B) .. (n)
smz= ~
n,

(C) sini =
F¥I
-fli
~n
2
-11i
B

(D) sini
J'i.
- 159. An astronomical telescope has an angula:i;
0 magnification of magnitude 5 for distant objects. The
(A) 7.5cm (B) 10cm separation between ~he. objective and the eye piece is
(C) 12.5 cm (D) 15 cm 36 cm and the final image is formed at infinity. The
focal length f, of the objective and the focal length f,
155. Three right angled prisms of refractive indices n,, n,
of the·eye piece are
and n3· are fixed together 'Using an optical glue as (A) f, = 45 crri and f, = -9 cm
shown in figlJ!e. If a ray passes ,through the prisms (B) f, =50 cm and f, =10 cm
without sufferfug any deviation, then
(C) f, = 7.2 cm and f, =5 cm
(D) f,=30cm and f,=6cm

c:=================================== 1.121 =
www.puucho.com
Rahul Sardana with www.puucho.com

Advanced JEE Physics Optics & Modern Physics

160. The slab of a material of refractive index 2 shown in y


figure has a curved surface APB of radius of curvature
10 cm and a plane surface CD. On the left of APB is air
and on the right of CD is water with refractive indices
as given in figure. An object O is placed at a distance of
--
--
---+-l~o+-----<
.. J..
,_~tJ.,,__.. x
15 cm from pole P as shown. The distance of the final
image of O from P, as viewed from the left is --
-- d----->l

(A) x= f,f, , y=!J.


f, + f,
(B) X
f, (!, +d) , y =6.-2 -
f,+f,-d f,+f,
(C) X
f,f, +d(f, -d) y t>.(f, -d)
~~~~

(A) 20 cm (B) 30 cm f,+f,-d f,+f,-d


(C) 40 cm (D) 50cm f,f, +d(f, -d)
(D) x y=O
f,+f,-d
161. A thin rod of length .!:_ f is placed along the optic axis
3 165. Spherical aberration in a thin lens can be reduced by
of a concave mirror of focal length f such that its (A) using a monochromatic light.
image which is real and elongated just touches the rod. (B) using a doublet combination.
The magnification is (C) using a circular annular mask over the lens.
{A) 4 5 (D) increasing the size of the lens.
(B)
3 3
3 166. A circular beam of light of diameter d = 2 cm falls on a
(C) (D) None of above plane surface of a glass slab. The angle of incidence is
2
60° and refractiVe index of glass is µ =~ . The diameter
162. A 2 cm diameter coin lies flat at the bottom of a bowl 2
of the refracted beam is
in which the water (µ. =¼), is 20 cm deep. If the coin (A) 2.52 cm (B) 3 cm
(C) 3.26 cm (D) 4cm
is viewed directly from above, the apparent diameter of
the coin is
167. Two thin lenses, wheri in contact, produce a
(A) 1.67 cm (B) 1.5cm combination of power +10 D . When they are
(C) 2.67cm (D) 2cm
0.25 m apart, the power reduces to +6 D . The focal
163. A ray of light undergoes deviation of 30° when lengths of the lenses (in m) are
(A) 0.125 and 0.5 (B) 0.125 and 0.125
incident on an equilateral prism of refractive index ,Ji .
(C) 0.5 and 0.75 (D) 0.125 and 0.75
The_ angle made by the ray inside the prism with the
base of the prism is
168. A real image of a distinct object is formed by a piano
(A) 30° (B) 45° convex lens on its principal axis. Spherical aberration
(C) 60° (D) 0° {A) is absent.
(B) is smaller if the curved surface of the lens faces the
164. Two thin convex lenses of focal lengths / 1 and /, are object.
separated by a horizontal distance d (where d < f, and (C) is smaller if the plane surface of the lens faces the
d < f, ) and their centres are displaced by a vertical object.
separation 8. as shown. Taking the origin of coordinates
(D) is the same whichever side of the lens faces the
object.
0, at the centre of the first lens, the x and y
coordinates of the focal point of this lens system, for a 169. A concave mirror is placed on a horizontal table, wi"Ut
parallel beam of rays coming from the left, are given by its axis directed vertically upwards. Let O be the pole
of the mirror and C its centre of curvature. A point
object is placed at C . It has a real image, also located at

1.122 i::::===================:::;================:::::::========
www.puucho.com
Rahul Sardana with www.puucho.com

Ray Optics

C . If the mirror is now filled with water, the image will


be
(A) real and will remain at C .
A C 8
(B) real and located at a point between C and oo.
60'
- ---------------
(C) virtual and located at a point between C and O .
(D) real and located at a point between C and O .

170. A spherical surface of radius of curvature R separates 3


air (refractive index 1.0 ) from glass (refractive index · (A) µ = -/2 (B) µ=-
2
1.5). The centre of curvature is in the glass. A point
(C) µ=s/3 (D) µ=2
object P placed in air is found to have a real image Q in
the glass. The line PQ cuts the surface at the point 0
175. A rectangular glass slab
and PO - OQ . The distance PO is equal to
ABCD of refractive -- ---------- -----
(A) SR (B) 3R
(C) 2R (D) 1.5R
index n1 , is immersed
in water of refractive 1--- -
:,~~A.... n, r::~::::
171. A concave lens of glass, refractive index 1.5, has both index n2 (n1 > n2 ). A -,:,:,:::g -- _,-, - _- __-c __-_-- __
surfaces of same radius of curvature R. On immersion
ray of light is incident ~-------------- ·----------- --------
in a medium of refractive index 1.75, it will behave as a at the surface AB of the slab as shown. The maximum
(A) convergent lens of focal length 3.5R . value of the angle of incidence amax, such that the ray
(B) convergent lens of focal length 3.0R . comes out only from the other surface CD is given by
(C)
(D)
divergent lens of focal length 3.5R .
divergent lens of focal length 3.0R .
1
(A) sin- {::cos( sin-
1
( :: J)}
172. A quarter cylinder of radius R and refractive index 1.5
is placed on a table. A point object P is kept at a
(B)
1
sin- {~cos(sin-
1
(:J)}
distance of mR from it. The value of m for which a ray
from P will emerge parallel to the table is (C)
1
sin- ( :: J

P,~
(D) sin-
1
(::J
I
;+- mR ----+-+- R --+i
176. A ray of light passes through
four transparent media with
(A) 1 2
(B) refractive in_dices µ 1 , µ2, µ3 and
3 3 8
µ4 as shown in figure. The c
(C) 1 (D) 4 surface of all media are A µ 1 112 ~ µ4
3 parallel. If the emergent ray ~~~~-~==
CD is parallel to the incident ray AB, we must have
173. A hollow double concave lens is made of very thin (A) µ 1 = µ 2 (B) µ 2 = µ 3
transparent material. It can be filled with air or either of
(C) µ, = µ, (D) µ, = µ,
two liquids L1 or L2 _ having refractive indices n1 and
n2 respectively ( n2 > rLi > 1). The lens will diverge a 177. A given ray of light suffers minimum deviation in an
parallel beam of light if it is filled with equilateral prism P. Additional prisms Q and R of
(A) air and placed in air. identical shape and of same material as P are now
(B) air and immersed in L1 • added as shown in figure. The ray will now suffer
(C) L1 and immersed in L, .
(D) L, and immersed in L1 •

174. A ray of light falls on a transparent sphere of refractive


index µ, having centre at C as shown in figure. The
ray emerges from the sphere parallel to line AB, then (A) greater deviation.
(B) no deviation.
(C) same deviation as before.

c::===================================== 1.123 =
www.puucho.com
Rahul Sardana with www.puucho.com

Advanced JEE Physics Optics & Modern Physics

(D) total internal reflection. 182. Critical anglefor a prism is 36°. The maximum angle of
prism for which the emergent ray is possible is
178. Two identical equiconvex lenses of focal length f , (A) 18° (B) 36°·

made of glass (µ, =%) ·are kept in contact. The space (C) 72° (D) 144°

183. A screen is placed 90·cm from an object. The image of


between the two lenses is filled with water (µw =½). the object on the screen is formed by a convex lens at
two different positions separated. from each other by
The focal length of the combination is 20 cm . The lens has a focal length of
3 (A) 32.1 cm (B) 15.8 cm
(A) j_ (B) /
2 4 (C) 21.4 cm (D) 10.7 cm
(C) f (D) 4/
3 184. A prism, having refractive index .Ji. and refracting
angle 30°, has one of-the refracting surfaces polished. A
179. An observer can see through a beam of light incident on the other r<efracting surface
pin hole, the top· of.. a thin rod of will retrace its path if the angle of incidence is
height h , placed as shown in . (A) 0° (B) 30°
figure. The beaker's height is 3h (C) 45~ (D) 60°
and its radius is h . When the
beaker is filled with a liquid upto 185. -Two plane mirrors are inclined at angle 8 as shown in
a height 2h, he·can see the lower figure. If a ray parallel to OB strikes the other mirror at
end of the rod.. Then the P and finally emerges parallel to , OA after two
refractive index of liquid must be ~2h-----+1
refle'ctio~, then 8 equals
(A) ~ (B) #
(C) #. (D) 3
2

180. Which one of the following spherical lenses does not


exhibit dispersion ?
(A) 30° (B) 45°

(A) R·
' [i R, (B)
_R_ ~ - ""
(C) 60° (D) 90°

186. An object is placed 20 cm in front of a block of glass 10


cm thick having its farther side ,silvered. Th,e image is
formed 23.2 cm behind the silvered face. The refractive
index 9£ glass is

~
(A) 1.41 (B). 1.46
(C) R R (D) H "" (C) 1.51 (D) 1.61
-~
187. Two parallel rays are travelling in a medium of
181. Two plane mirrors A and B are alligned parallel-to each· . .m d·ex µ 1 =-4
refractive . However, one of h
t e rays
other, as,shown in figure. A ligll.t ray is incident atan 3 .
angle of 30° at a point just inside one end of A: The passes through a parallel glass slab of thickness t and
plane of incidence coincides with the plane .of figure. refractive index µ 2 =~. The path diff~rence between
The maximum number of- times the ray undergoes 2
reflections (including the first one) before it emerge_s out · the two rays due to the glass slab is
is t t
(A) 8 (B) '6°
3!
(C) ~- (D) 2

188. A ray of light entering from air to glass (refractive index


1.5) is partly reflected and partly refracted. If the
(A) 28 (B) 30. incident and the reflected rays are at -right angles to
(C) 32 (D) 34 each other, the angle of refraction is
= 1.124 i::::=======================::;:::=============
www.puucho.com
Rahul Sardana with www.puucho.com

Ray Optics

(A) sin-'( Ji) (B) S!Il' -1(./2.)


-
3
195. One side of a glass slab of refractive index 1.5 is silvered
as shm--vn. A ray of light is incident on the other side at
angle of incidence i = 45° . The deviation of the ray of
(C) 'sin,'( 1) (D) sm . -1( -./31 ) light from its initial path when it comes out of the slab
is
189. A beam of light is converging towards a point on a ''
screen. A plane parallel sided plate of glass of thickness ''
t and refractive index µ is introduced in the path of 450,''
the beam. The convergence point is shifted by

(A) { 1-;) away (B) { 1+;) away µ = 1,5

(C) { 1-;) nearer (D) '{ 1+;) nearer

(A) 45' (B) 90°


190. The distance of an' object from the first focus -of an (C) 120° (D) 180°
equiconvex lens is 10 cm 'and the distance of its real
image from the second focus is 40 cm . The foca.l length 196. A piano-convex lens. has a thickness of 4 cm. When
of the lens is placed on a horizontal table with the curved surface in
(A) 10cm (B) 20 cm contact with it, the apparent depth of the bottom-most
(C) 25cm (D) .40 cm point of the lens is found to be 3 cm. If the lens is
inverted, such that the plane face _is in contact with the
191. A beam of monochromatic Hght is incident .on one face table, the apparent depth of the centre of the plane face
of an equilateral prism, the angle of incidence being 25
55°. If the angle of emergence is 46° then the angle of is found to be cm. The focal length of the lens is
8
minimum deviation is
(A) 50 cm (B) 75 cm
(A) 41° (B) < 41° (C) 100cm (D) 150cm
(C) > 41° (D) ;, 41°
197. If J 8 and JR are the focal lengths of a convex lens for
192. When a ray of light is refracted by a prism such that ihe
angle of deviation is minimum, then blue and red lights respectively and F, and FR are the
(A) the angle of emergence is equal to the angle of respective values for a concave lens, then
incidence. (A) J, > JR and F, >FR ,.(B) f, <JR and F, >FR
(B) the angle of emergence is greater than the angle of (C) J, > JR and F, <FR (D) J, <JR and F, <FR
incidence.
(C) the angle of emergence is smaller than the angle of 198. A cubic container is filled with a liquid whose refractive·
incidence.
index increases linearly from top to bottom. The path of
(D) the sum of the angle of incidence and the angle of a ray of light inside the liquid is best represented by
emergence is equal to 90° .

193. The image of a square hole in a screen illuminated by


light is obtained on another screen with the help of a
converging lens. The distance of the hole from the lens
is 40 cm . If the area of the image is nine times that of
the hole, the focal length of the lens is
(A) 30 cm (B) · 50 cm
(C) 60 cm (D) 75 cm

194. A short linear object of length b lies along the axis of a


concave mirror of focal length J at a distance u from
the pole of the mirror. The size of the image is
approximately equal to

(A) bH (B) b(u;JJ 199. The focal length of a convex lens is J and the distance
of an object from the principal focus is x . The ratio of
the size of the real image to the size of the object is
(C) b~u~J (D)
b(u~ J )'
i::::::=================================· = 1.125

www.puucho.com
Rahul Sardana with www.puucho.com

Advanced JEE Physics Optics & Modem Physics

l X (C) 2/ (D) 4/
(A) (B)
X I
205. In PROBLEM 204, if the lens is cut along PQ and RS
(D) _!_
f+x
(C) simultaneously, the focal length of each part will be
f f+x
(A) l_ (B) I
200. Focal length of a convex mirror is 10 cm 2-
(A) image of an object placed at 20 cm is also at (C) 2/ (D) 4/
20cm 206. The layered lens as shown is made
(B) image of an object placed at 10 cm is at infinity
(C) both (A) and (B) are correct of two types of trarJSparent
(D) both (A) and (B) are incorrect
materials-one indicated by
horizontal lines and the other by
201. An object is placed at a distance x1 from the principal vertical lines. The number of
images formed of an object will be
focus of a lens and its real image is formed at a distance
(A) 1 (B) 2
x, from the principal focus, The focal- length of the lens (C) 3 (D) 6
is
(A) (B) x, x, 207. The distance between an object and its real image
2 formed by a convex lens cannot be
.X1 +X2 (A) greater than 2/ (B) less than 2/
(C) (D) ~x, x, ·
2 (C) greater than 4/ (D) less than 4/
202. The plane faces of two identical planoconvex lenses, 208. Two thin symmetrical lenses of different nature and of
each having focal length of 40 cm , are pressed against different material have equal radii of curvature
each·other to form a usual convex lens. The distance in
R = 15 cm are placed close together and immersed in
cm from this lens, at which an object must be placed to
obtain a real 'image with magnification unity is
(A) 10 (B) 20
water (µ. =¼). The focal len~ of the system in water
(C) 40 (D) 80 is 30 cm . The difference between refractive indices of
the two lenses is
3 . 1 1
203. ·A plane refracting surface of refractive index - .JS (A) (B)
2 2 , 3
placed at a distance of 10 cm from a thin convex lens of 1 3
focal length 30 cm . The parallel rays incident on lens (C) (D)
4 4
will converge at a distance of
209. A needle of length 5 cm, placed 45 cm from a lens forms
an image on a screen placed 90 cm on the other side of
the lens. The type of lens and its focal length are
(A) convex, 30 cm (B) concave, 30 cm

i+- 1Ocm --+1


~J (C) convex, 60 cm

(A) real, 20 cm
(D) concave, 60 cm

210. In PROBLEM 209, the nature and size of the image are
(B) real, 10 cm
(A) 30 cm from the lens.. (B) 25 cm from the lens.
(C) virtual, 20 cm (D) virtual, 10 cm
(C) 20 cm from the lens. (D) 40 cm from the lens.

204. The figure shows an equiconvex)ens of focal length / . 211. An object is placed 50 cm in front of a convex surface of
radius 20 cm. If the surface separates air from glass of
If the lens is cut along PQ, the focal length of each half
refractive index 1.5, the distance of the image from. the
will be
lens and its nature are
p
(A) 30 cm, real (B) 30 cm, virtual
(C) 300 cm, real (D) 300 cm, virtual
R ---+,>+---S ·
212. One of the refracting surfaces of a prism of angle 30° is
.silvered. A ray of light incident at an angle of 60°
Q retraces its path. The refractive index of the material of
prism is '
(A) l_ (B) I
2

= ====================================
1.126

www.puucho.com
Rahul Sardana with www.puucho.com

Ray Optics
3 (B) 25 cm
(A) -Jz (B)
(C) 20cm
2
(C) fj (D) 2 (D) cannot be estimated with given data

213. A slab of glass of refractive index 1.5 and thickness 3 crri 218. A ray of light ravelling in the direction ½(1 + /3]} is
is placed with the faces perpendicular to the principal
axis of a concave mirror. If the radius of curvature of incident on a plane mirror. After reflection, it travels
the mirror is 10 cm, the distance at which an object must
be placed from the mirror so that the image coincides
along the direction .!(1 - /3J) . The angle of incidence is
2
with the object is (A) 30° (B) 45'
(A) 9cm (B) .10 cm (C) 60° (D) 75'
(C) 11cm (D) 12cm
219. A bi-convex lens is formed with two thin piano convex
214. Figure represents a convergent lens placed inside a cell lenses as shown in the figure. Refractive index n of the
filled with a liquid. The lens has a focal length +20 err\ first lens is 1.5 and that of the second lens is 1.2. Both
when in air and its material has refractive index 1.50 . If the curved surfaces are of the same radius of curvature
the liquid has a refractive index 1.60, the focal length of R = 14 cm. For this bi-convex lens, for the object
the lens in the new system is distance of 40 cm , the image distance will be

Lens
t R = 14 cm
(A) -280 cm (B) 40 cm
(A) -80 cm (B) +80 cm (C) 21.5 cm (D) 13.3 cm
(C) -160 cm (D) -24 cm
220. An equiconvex lens, having radius of curvature 33 cm,
is placed on a horizontal plane mirror and a pin held 20
215. A point object O is placed on the principal axis of a cm above the lens coincides with its image. Now the
convex lens of focal length 20 cm at a distance of 40 cm space between the lens and the mirror is filled with a
to the left of it. The diameter of the lens is 10 cm. If the liquid. In order to coincide with the image the pin has
eye is placed 60 cm to the right of the lens at a distance to be raised by 5 cm. The refractive index of the liquid is
h below the principal axis, then the maximum value of (A) 1.33 (B) 1.53
h to see the image will be (C) 2.33 (D) 2.66
(A) 0 cm (B) 5 cm
(C) 2.5 cm (D) 10 cm 221. A real image is formed by a convex lens. If we put a
concave lens in contact with it, the combination again
216. For two positions of a lens, the images are obtained on a forms a real image. The new image .
fixed screen. If the size of object is 2 cm and the size of (A) is closer to the lens system.
diminished image is 0.5 cm, the size of the other image (B) is farther from the lens system.
will be (C) is at the original position.
(A) 1 cm (B) 4cm (D) may be anywhere depending on the focal length of
(C) 8 cm (D) 16cm the concave lens.

217. The. medium on both sides of lens is air. The distances 222. A concave mirror has a focal length 20 cm . The
of object O , image I from first and second foci F1 and distance between the two positions of the object for
F, are shown in figure. The focal length of lens is which the image size is double of the object size is
(A). 60 cm (B) 40 cm
(C) 30 cm (D) 20 cm

- -
F, F,
0 223. A convex lens of focal length 40 cm is in contact with a
16 cm concave lens of focal length 25 cm. The power of the
L 2 5cm combination in dioptre is
(A) 16cm (A) -1.5 (B) -6.5

c======================================== 1.127
www.puucho.com
Rahul Sardana with www.puucho.com

Advanced JEE Physics Optics & Modern Physics

(C) +6.5 (D) +6.67 3/ I


(D) f, 3:
4'2
224. Two lenses of powers1 +12 D and -2 D are in contact.
The focal length of the combination is 229. When a ray C>f light goes from air to a glass slab, then
(A) 10 cm (B) 12.5 cm (A) its wavelength increases
(C) 16'6 cm (D) 8.33 cm (B). its wavelength decreases ,
(C) . its frequency increases
225. A point object O is placed (D) neither.its wavelength nor its frequency changes
at a distance of 20' cm
from a convex lens of focal 230. In the displacement method, a convex lens is placed in
length 10 cm as shown in 0 between ,an object .µ1d a screen. If the magnification in
figure. The distance x the two positions are m1 and m2 ( m1 > m, ) , and the
from the lens where a distance between the two positions of the lens is x , the
concave mirr(?r o,f focal focal length of the Jens is
. length 60 cm has to, be placed so th~t final image X x
coincides with the obje.ct is (A) -- - (B)
m1 +m2 m1-m2
(A) 10 CI_ll
X X
(B) 20 cm (C) (D)
, (C) 40cm (m, +m,)' (m, -m,)'
(D) final image can never coincide with the object
under the conditions provided. 231. A screen is placed 90 cm from an object. The image of
the object on the screen is formed by a convex lens at
226. A lens forms a sharp image on a screen. On inserting a two different locations separated by 20 cm. The focal
parallel sided glass slab between the lens and the length of the Jens is ·
screen, it is _found necessary to move the screen a (A) 10.7cm (B) 21.4cm
distance d away from the lens in order for the image to (C) 15.8cm (D) 32.1 cm
be sharp again. If the refractive index of the material of
the slab is n , the thickness of the slab is 232. A light ray travelling in glass medium is incident on
d glass-air interference at an angle of i_J;i.cidence 8 . The
(A) nd (B)
n reflected (R) and transmitted (T) int~nsities, both as
(C) (n-1)d · nd function of 0, are plotted. The.correct sketch is
(D)
n n-1
,Intensity
227. A plano convex glass
100o/o -----------, -·
Jens (µ, =¾) of
(A)
T
,,
,'
radius of curvature
R =10 cm is placed at 0 ___R.. --~_,.,..'
a distance of y from a o~----9LO'-+ 0
concave lens .of focal ,..._ X - - - Y ___,..
length 20 cm . The , Intensity
distance x of a point object O from the piano convex
lens so that the position· of final image is independent of 100%
T
y'is
(B)
(A) 20cm (B) 30cm
(C) 40cm (D) 60cm

228. A thin lens has focal length /, and its aperture has
diameter D . Ii forms an image of intensity i'. If the Intensity

central part of the_ aperture, of diameter ~ , is blocked


100%
by an opaque paper, the focal. length of the lens and the T
intensity of image will become (C)

(A) fr,½ (B) __ f:!_.-


o~--.L-~9~0--0

1.128 c============================================
www.puucho.com
Rahul Sardana with www.puucho.com

Ray Optics
. Intensity (B) a virtual, erect, same-sized image can be obtained
using a plane mirror.
100%
T ,,- ----- (C) a virtual, erect, magnified image can be formed
using a concave mirror.
(D) (D) a real, inverted, same-sized image can be formed
using a convex mirror.
R --
O!=--:.:-::-c_-_ _...:::::90=,=---+. e
239. When an object is at distances x and y from a IellS, 'a

233. A lens is placed between the source of light and a wall. real image and a virtual image is formed respectively
It forms images of area A1 and Ai on the wall for its having· same magnification. The focal length of the lens
is given by
two different positions. The area of the source of light is (A) x-y (B) x+y
(A) ~A,A, (B) A,; A, (C) ..{xy (D)
x+y
2
(C) (D) 2.+2.)-1
( A, A, 240. If the central portion of a convex lens is
wrapped in black paper as shown in the figure, ~
(A) no image will be formed by the remaining
234. The plane face of a plano-convex lens is silvered. If µ
portion of the lens. ·
be the refractive index and r the radius of curvature of
(B) full image will be formed, but it will be less
the curved surface, then the system behaves like a bright.
concave mirror of radius
(C) the central portion of the image will be missing.
r r (D) there will be two images, each producecl by one of
(A) (B)
µ µ-1 the exposed portions of the lens.
(C) rµ (D) r(µ-1)
241. A plane mirror made of glass slab (µ, =1.5) is 2.5 cm
235. A ray of light falls on the surface of a spherical ·paper thick and silvered at the back. A point object is placed
weight making an angle a with the normal and is 5 cm in front of the unsilvered face of the mirror. The
refracted in the medium at an angle p. The angle of position of the final image is
deviation of the emergent ray from the direction of the 16
incident ray is (A) cm from unsilvered face
3
(A) (a-P) (B) 2(a-p) 25
(B) cm from unsilvered face
(C) (a-P) (D) p-a 3
2 (C) 12 cm from unsilvered face
(D) 14 cm from unsilvered face
236. The magnification of an object placed in front of a
convex lens of focal length 20 cm is +2 . To obtain a 242. The distance between an object and the screen is 100
magnification of -2, the object has. to be moved by a cm. A lens proQ.uces an image on the screen when
distance equal to placed at either of two position~ 40 cm apart. The power
(A) 40 cm (B) 30 cm of the lens is approximately
(C) 20cm (D) 10cm (A) 4.25 D (B) 4.50 D
(C) 4,75 D (D) 5.0 D
. 237. A concave mirror is placed on a horizontal table with its
axis directed vertically upward. Let ( 0) be the pole of 243. A real image of an object is formed ·by a convex lens at
the mirror and C its centre of curvature. A point object the bottom of an empty beaker. The beaker is now filled
is placed at C . It has a real image also located at C . If with a liquid of refractive index 1.4 to a depth of 7 cm.
the mirror is now·filled with water, the.image will be In order to get the image again at the bottom, the beaker
should be moved
(A) real and will remain at C .
0
(B) real and located at a point between C and oo .
(C) virtual ru,d located at a point between C and 0.
(D) real and located at a point between C and 0.

238. All of the following statements,are correct except


(A) the magnification produced by a convex mirror is
always less than one.

==================================1.120=
www.puucho.com
Rahul Sardana with www.puucho.com

Advanced JEE Physics Optics & Modem Physics

(A) downward by 2 cm (A) 10 (B) 20


(B) upward by 2 cm (C) 30 (D) 40
(C) downward by 3 cm
(D) upward by 3 cm 250. An object is placed at a distance of 12 cm from a convex
lens on its principal axis and a virtual image of certain
244. The convex surface of a thin concavo-convex lens size is formed. If the object is moved 8 cm away from
(refractive index 1.5) has a radius of curvature 20 cm. the lens, a real image of the same size as that of the
The concave surface has a radius of curvature 60 cm. virtual image is formed. The focal length of the lens in
The convex side is silvered and placed on a horizontal cmis
surface. At what distance from the lens should a pin be (A) 15 (B) 16
placed on the optic axis such that its image is formed at (C) 18 (D) 19
the same place?
(A) 15 cm (B) 7.5 cm 251. A thin converging lens of refractive index 1.5 has a
(C) 22.5 cm (D) 30 cm power of +0.5 D . When this lens is immersed in a
liquid, it acts as a diverging lens of focal length 100 cm.
245. In PROBLEM 244, if the concave part is filled with The refractive index of the liquid is
water (refractive index ±
3,
), the distance from the lens at (A) 4 (B) 3
3 2
which the pin should be placed to form the image at the
same place is (C) 5 ()
D 2
3
90 45
(A) cm (B) -cm
13 13 252. The distance between an object and its real image
. 135 180 formed by a lens is D . If the magnification is m , the
(C) -cm (D) -cm
13 13 focal length of the lens is
(A) (m-l)D mD
(B)
246. An equiconvex lens of glass (µ, =1.5) of focal length m m+l
10 cm, silvered on one side behaves like a (m-l)D mD
(C) (D)
(A). convex mirror of focal length 5 cm m' (m+l)'
(B) convex mirror of focal length 20 cm
(C) concave mirror of focal length 2.5 cm 253. A piano convex lens of focal length 16 cm, is to be made
(D) concave mirror of focal length 10 cm of glass of refractive index 1.5. The radius of curvature
of the curved surface should be
247. A piano-convex lens of focal length 30 cm has its plane (A) 8 cm (B) 12 cm
surface silvered. An object is placed 40 cm from the lens (C) 16 cm (D) 24 cm
on the convex side. The distance of the image from the
lens is 254. A real image of a point object O was formed by an
(A) 18 cm (B) 24cm equi-convex lens of focal length f and the
(C) 30 cm (D) 40 cm magnification was found to be unity. Now the lens is
248. Refraction takes place at a concave spherical boundary cut into two symmetrical pieces as shown by the dotted
' 3
separating glass air medium. If µg = , then for the line and the right part is removed. The position of the
2 image formed by the remaining part is at
image to be real, the object distance
(A) is independent of the radius of curvature of the
refracting surface
(B) should be greater than the radius of curvature of 0
the refracting surface
(C) should be greater than two times the radius of
curvature of the refr~cting surface f
(D) should be greater than three times the radius of
curvature of the refracting surface (A) f (B) 2/

(C) i_ (D) Infinity


249. A parallel beam of light incident on a concave lens of 2
focal length 10 cm emerges as a parallel bearu from a
convex lens placed coaxially~ the distance between the 255. A convex lens, made of a material of refractive index 1.5
lenses being 10 cm. The focal length of the convex lens and having a focal length of 10 cm is immersed in a
incm is. liquid of refractive index 3.0. The lens will behave as a

= ====================================
1.130

www.puucho.com
Rahul Sardana with www.puucho.com

Ray Optics
' lens of focal length 10 cm.
(A) converging (C) 12cm (D) 6cm
(B) diverging lens of focal length 10 cm.
10 260. The maximum and minimum distances between a
(C) converging lens of focal length cm.
3 convex lens and an object, for the magnification of a real
(D) diverging lens of focal length 30 cm. image to be greater than one are
(A) 2/ and f (B) f and zero
256. A point source S is placed at a height h from the (C) oo and 2/ (D) 4/ and 2/
bottom of a vessel of height H (< h). The vessel is
. polished at the base. If the water is gradually filled in 261. A point object is placed on the optic axis of a convex
the vessel at a constant rat'e a m 3s-1 , the distance d of lens of focal length f at a distance of 2f to the left of
image of the source from the bottom of the vessel varies it. The diameter of the lens is d . An observer has his
with time. t as eye at a distance of 3 f to the right of the lens and a
S• distance h below the optic axis. The maximum value of

i r
• h
h to see the image is
(A) d
4
(B)
d
3
H
d
(C) 2 (D) d

262. A convex lens is immersed in a liquid of refractive

t=, l=,
index greater than that of glass. It will behave as a
(A) convergent lens (B) divergent lens
(C) plane glass (D) homogeneous liquid
(A) 0)
263. If the top half of a convex lens is covered with black
paper,
(A) the bottom half of the image will disappear.
(B) the top half of the image will disappear.

1=, l=,
(C) the magnification will be reduced to half.
(D) the intensity will be reduced to half.

(q (D) 264. In displacement method, the lengths of images in the


two positions of the lens between the object and the
screen are 9 cm and 4 cm respectively. The length of the
object must be ·
(A) 6.25 cm (B) 1.5 cm
257. A convex lens of glass has power P in air. If it is
(C) 6 cm (D) 36 cm
immersed in water its power will be
(A) more than P 265. A convex lens of focal length 15 cm is placed on a plane
(B) less than P mirror. An object is placed 20 cm from the lens. The
(C) p image is formed
(D) more than P for some colours and less than P for (A) 12 cm in front of the mirror
others (B) 60 cm behind the mirror
(C) 60 cm in front of the mirror
258. A biconvex lens, made of a material of refractive index (D) 30 cm in front of the mirror
1.5, has radius of curvature of each side equal to 0.5 m.
The power of the lens is 266. A convex lens of focal length 40 cm is
(A) 0.5 D (B) 1.0 D held coaxially 12 cm above a concave 0
(C) 1.5 D (D) · 2.0 D mirror of focal length 18 cm. An object xcm
held x cm above the lens gives rise to an
259. A convex lens forms a real image 4 cm long on a screen. image coincident with it. The x is equal
When the lens is shifted to a new position without to
disturbing the object or the screen, again real image is 12cm
(A) 12cm (B) 15 cm
formed on the screen which is 16 cm long. The length of (C) 18cm (D) 30cm
the object is
(A) 8 cm (B) 10 cm

====================================· = 1.131

www.puucho.com
Rahul Sardana with www.puucho.com

Advanced JEE Physics Optics & Modern Physics

267. Two thin lenses of powers 2 D and 3 D are placed in concave mirror of focal length f : To obtain a real
contact. An object is placed at a distance of 30 cm from image of same magnification, the object has' to moved
the combination. The distance in cm of -the image from 1;>y a distance
the combination is
(A) 30 (ll) 40 (A) f_ (B) 2/
2 3
(C) 50 . (D) 60
(C) f (D) 3/
268. Two convex lenses of focal lengths f, and f, are 2
mounted coaxially separated by a distance. If the power
274. An astronomical telescope has an angular magnification
of the combination is zero, the distance be~~en the
of magnitude 5 for distant objects. The separation
lenses is
between the objective and the eyepiece is 36 cm and the
(A) lf,.-f,I (ll) h + !, final image is formed at infinity. The focal length f, of
(C)
f,f, (D) I,!, the objective and f, of the eyepiece are
It, - !,I h+f, (A) f 0 =45cm and f>,-9cm
(ll) f 0 =50cm and /,=10cm
269. Chromatic aberration in a lens is caused by
(A) reflection (ll) interference (C) f,=7.2cm and /,=5cm
(C) diffraction (D) dispersion (D) / 0 =30cm and /,=6cm

270. A plane mirror is Rlaced at the bottom of a tank 275. A piano convex lens .of radius of curvature R fits
containing a liquid ofrefractive index µ . A small object exactly into a plano concave lens such that their plane
P lie~ at a height h above the mirror. An observer O , surfaces are parallel to each other. If the lenses are made
vertically above P , outside the liquid, observe P and of differen~ materials of refractive indices µ1 and µ 2 ,
its image in the mirror. The apparent distance between then focal length of the combination is given by
these two will be
o, (A) R (ll) R
2-(µ, +µ,) 2(µ,-µ,)
''' ZR R
'' (C) (D)
'
'I
Ph ' 276. A compound microscope has an objective of focal
:~ length 2.0 cm and an eye piece of focal length 6.25 cm
separated by 15 cm. If the final image is formed at the
least distance of distinct vision (25 cm), the distance of
2h the object from the objective is
(B) .µ-1 (A) 1.5 cm (B) 2.5 cm
(C) 3.0 cm (D) 4.0 cm
(D) 2h
µ
277. In PROBLEM 276, the magnifying power of the
microscope is
271. A person can see clearly between 1 m and (A) 10 (B) 15
2 m .. His corrective lenses should be (C) 20 (D) 30
(A) bifocals with pow~r -0.5 .D and additional +3.5 D
(B) bifocals with power ~1.0 D and additional 278. A point object is placed at a distance of 20 cm from a
+3.0D
(C) concave with power LO D
glass slab (µ, = ¾) half immersed in water (µ. = ¾) as
(D) convex with power 0.5 D shown in figure. The distance between two images
when se~n-from the other side of the slab is
272. Spherical aberration in a thin lens can be reduced by 9cm
l+-+i
(A) using a monochromatic light. --:-
(B) using a doublet combination.
(C) using a circular. annular mask over the lens. 0
(D) increasing the size of the lens.
-
273. A virtual image of an object is formed with a
magnification of 2, when the object is placed infront ofa 1+20 cm+i

= 1.1s2c:::====================================
www.puucho.com
Rahul Sardana with www.puucho.com

Ray Optics

(A) 1 cm (B) 2cm possible magnification, we choose the lenses of focal


(C) 4cm (D). 6cm lengths
(A) 100 cm, 0.3 cm (B) 10 cm, 0.3 cm
279. A compound microscope has .a magnification of 30. The (C) 10 cm, 4 cm (D). 100 cm, 4 cm
focal length of the eye-piece is 5 cm. If the final image is
formed at the least distance of distinct vision ( 25 cm), 287. The angular magnification of a telescope which contains
the magnification produced by the objective is an objective of focal length / 1 and eyepiece of focal
(A) 5 (B) 7.5 length /, is
(C) 10 (D) 15
(AJ I, (BJ /1 + /,
280. The least distance of distinct VisiOn is 25 cm. The focal !1 I,
length of a convex lens is 5 cm . It can act as a simple (C) /1 (DJ
f,f,
microscope of magnifying power I, !1 + I,
(AJ 4 (B) 5
(C) 6 (D) None of these 288. An achromatic combination is to be made using a
convex and a concave lens. Tfle two lenses should have
281. An astronomical telescope has an eye piece of focal (AJ their power equal.
length 5 cm. If the angular magnification of normal (B) their refractive indices equal.
adjustment is 10, the distance between the objective and (C) their dispersive powers equal.
the eye.piece is (DJ the product of their powers and dispersive powers
(A) 45cm (B) 50 cm equal. ·
(C) 55cm (D) 110 cm
289. For a thin equiconvex lens, the optics axis coincides
282. The focal lengths of the objective and the eyepiece of an with the x-axis.and the optical centre coincides with the
astronomical telescope are 100 cm and 20 cm origin. The co-ordinates of a point object and its image
respectively. Its magnifying power in normal are (--40, 1) cm and (50,-2) cm respectively. Lens is
adjustment is
located at
(A) 5 (B) 2
(C) 25 (D) 4 (A) x=0 (B) X=-lOcm
(C) x=+20 cm (D) x~-30cm
283. Two convex lenses of focal lengths 0.3 m and 5 cm are
used to make a telescope. The distance kept between 290. The near point of a person is 50 cm and the far point is
them is equal to 1.5 m. The spectacles required for reading purpose and
(A) 0.35 m (B) 5.3 cm for seeing distant objects are respectively
(C) 5.3m (D) 0.15m
(A) +2 D, -(¾) D (B) +(¾) D, -2 D
284. To have larger magnification by a telescope
(A) the objective should be of large focal length and (C) -2 D, +(¾) D (D) -(¾) D, +2 D
the eyepiece should be of small focal length
(B) both the objective and the eyepiece should be of
291. Astigmatism for a human eye can be removed by using
large focal lengths
(A) concave lens (B) convex lens
(C) , both the objective and the eyepiece should be of
(C) cylindrical lens (D) prismatic lens
small focal lengths
(D) the objective should be of small focal length and
292. A hollow convex lens of glass behaves like a
the eyepiece should be of large focal length
(A) plane mirror (B) concave lens
(C) convex lens (D) glass plate
285. The angle of incidence for an equilateral prism is 60° .
The refractive index of prism so that the ray inside the
293. The far point of a myopic eye is 250 cm. The correcting
prism is parallel to the base of the prism is lens should be a
9 ..
(A) (B). ..fi. (A) diverging lens of focal length 250 cm.
8 (B) converging lens of focal length 250 cm.
(C) 4 (D) ,/3 (C) diverging lens of focal length.125 cm.
3 (D) converging lens of focal length 125 cm.

286. Four convergent lenses have focal lengths 100 cm, 294. A person cannot see clearly beyond 50 cm. The power
10 cm, 4 cm and 0.3 cm. For a telescope with maxirnuffi Of the lens required to correct his vision is
(A) -0.5 D (B) +0.5 D

c::==========================:.::::::======== 1.133=

www.puucho.com
Rahul Sardana with www.puucho.com

Advanced JEE Physics Optics & Modern Physics.

(C) -2 D (D) +2 D 296. A person cannot see clearly objects at a distance less
than 100 cm,. The power of the spectacles required to
295. A ray travelling in negative x-direction is directed see clearly objects at 25 cm is
towards positive y-direction after being reflected from a (A) +1 D (B) +3 D
surface at point P . The reflecting surface is represented (C) +4 D (D) +2 D
by the equation x2 + y2 = a2 • Then co-ordinates of point
P are 297. An object is kept at a distance.of 16 cm from a thin lens
and the image formed is real. If the object is kept at a
distance of 6 cm from the same lens the hnage formed
is virtual. If the size of the images formed are equal, the
focal length of the lens will be ·
(A) 8 cm (B) 5 cm
(C) 11 cm (D) ../% cm

298. A person can see clearly objects lying between 25 cm


and 2 m from his eye. His vision can be corrected by
(A) (a, 0) (B) (0.6a, O.Ba)

c;, Jz)
using spectacles of power
(A) +0.25 D (B) +0.5D
(C) ( 0.Ba, 0.6a) (D) (C) -0.25 D (D) -0.5D

= 1.134 ==================================
www.puucho.com
Rahul Sardana with www.puucho.com

This section contains Multiple Correct Choice Type Questions. Each question has four choices (A), (B), (C) and (D), out of which
ONE OR MORE is/ are.correct.

1. The x-y plane is the boundary between two 6,


(C) -=µ (D)
transparent media. Medium 1 with z 2:: 0 has a 61
refractive index ..fi. and medium 2 with z < 0 has a
4. A ray of light from a denser medium strikes a rar~r
refractive index ·../3. A ray of light in medium 1 given medium at angle of incidence i . The reflected and the
by the vector A= 6../31 + s../3J-10k is incident on the refracted rays make an angle of 90° with each other.
plane of separation. The refracted ray makes angle r The angles of reflection and refraction are r and r'
with +z axis and incident ray makes an angle i with respectively. The critical angle is
-z axis. Then, (A) sin-1 ( tanr) (B) sin-1 ( tani)
(A) i = 120° (B) i =60° 1
(C) sin- (tanr') (D) tan-1 (sini)
(C) r=45° (D) r =135°
5. A single converging lens is used as a simple
2. A ray of light travels from a medium of refractive index microscope. In the position of maximwn magnification.
µ to air. Its angle of incidence in the medium is i, Select the correct statement(s).
measured from the normal to the boundary, and its (A) the object is placed at the focus of the lens.
angle of deviation is a. The curve that best represents (B) the object is placed between the lens and its focus.
the plot of deviation 6 (along y-axis) with angle of (C) the image is formed at infinity.
incidence i (along x-axis) is (D) the object and the image subtend the same angle at
il
the eye. ·

6, 6. A light of wavelength 6000 A in air enters a medium of


(A) refractive index 1.5 . Inside the medium, its frequency
o,
is v and its wavelength is 'A. .
0'-""':.._4-_.,,__..
0 (A) v=5x10 14 Hz (B) v=7.5x1014 Hz
(C) 1. = 4000 A. (D) ,_ = 9000 A.

ilkut.
il il
7. If a converging beam of light is incident on a concave
mirror, the reflected light
' ' ' ''
(C) '' (D) (A) may form a real image
6
il 1 ~ il , ' n
I J!.
'' -2 (B) must form a real image
,, 2 .
0 i 0 i (C) may form a virtual image
0 0 (D) may be a parallel beam

3. In PROBLEM 2, 8. Two points P and Q lie on either side of an axis XY


as shown. It is desired to produce an image of P at Q
(A) 9=sin-'(¾)

==================================1.135 =
www.puucho.com
Rahul Sardana with www.puucho.com

Advanced JEE Physics Optics & Modern Physics

using a spherical mirror, with XY as the optic axis. The C


(A) b (B)
mirror must be C b
p
• ab
(C) a (D)
C
X+----------•Y

14. A lens ofrocal length / is placed in between an object



Q and screen fixed at a distance D . The lens forms two
(A) converging real images of object on the screen for two of its
(B) di verging different positions, a distance x apart. The two real
(C) positioned to the left of P images have magnifications m1 and m2 respectively
(D) positioned to the right of Q (m 1 >m,).
X
9. An object and a screen are fixed at a distance d apart. (A) !=-- (B) m1m2 =1
m1 -mi
When a lens of focal length / is moved between the
D2-x2
object and the screen, sharp images of the object are (C) ! =4D
-- (D) D?.4/
formed on the screen for two positions of the lens. The
magnifications produced at these two positions are M 1
15. A planet is observed by , an astronomical refracting
and M2 •
telescope having an objective of focal length 16 m and
(A) d>2f (B) d>4f
an eye piece of focal length 2 cm .
d
(C) MM =1
1 2 (D) IM,1-IM,1=7 (A) The distance between objective and eye piece is
16.02 m.
(B) The angular magnification of the planet is -800.
10. Resolving power of an electron n,,.icroscope is R, and (C) The image of the planet is inverted.
that of optical microscope is R0 . (D) The objective is larger than the eye piece.
(A) R, > R, (B) R, < R0
16. A parallel beam of white light falls on a combination of
(C) R, =R, (D) Data Insufficient a concave and a convex lens, both of same material.
Their focal lengths are 15 cm and 30 cm respectively for
11. In PROBLEM 10, the correct argument for the correct the mean wavelength in white light. On the other side
selected option is that of the lens system, one sees
(A) electrons have greater wavelength than visible (A) a coloured pattern with violet at the outer edge.
light. (B) a coloured pattern with red at the outer edge.
(B) electrons have lesser wavelength than visible light. (C) white light again.
(C) resolving power is inversely proportional to the (D) that it is unable for the lens to converge the rays at
wavelength of the wave used for detecting an a point.
object by the microscope.
(D) resolving power is inversely proportional to the 17. Consider a ray of light going from A to B. Let the ray
square of the wavelength of the wave used for traverse, in going from A to B, distances
detecting an object by the microscope. S 11 s2 ,s 3 , ................... sm in media of indices
n1 , n2 , n3 , ••...•..••.•. nm respectively.
12. The distance between two point objects P and Q is
1 m
32 cm . A convex lens of focal length 15 cm is placed (A) Total time offiight t=-Ln,s,
C i=l
between them so that the images of both the objects are
formed at the same place. The distance of P from the
lens could be
(B) Total time offiight t = _! f s,
C i=l
(A) 20cm (B) 18 cm
(C) 16 cm (D) 12cm (C) Optical path length is (O.P.L.) = f n,s,
i=l
B
13. The graph shows the
variation of magnification
m
(D) For inhomogeneous media the O.P.L. = n(s)ds f
A
m produced by a convex and the ray travels along 'Stationary Pathways'.
lens with the image distance
v . The focal length of the
lens is
L,.__ _.L._ _ _ _ -'--+ V
,.__ a,---c-----
= 1.136c:::=================================
www.puucho.com
Rahul Sardana with www.puucho.com

Ray Optics

18. A point object is placed ai 30 cm from a convex glass on the lens returns as a parallel beam from the

lens (µ, =¾) of focal length 20 cm . For the final


arrangement. Select the correct statement(s).
(A) The beam diameters of the incident and reflected
image of object to be formed at infinity, which of the beains must be the· same.
following is/ are correct? ' CBl a= 211,Hl,I
(A) A concave lens of focal length 60 cm is placed in <Cl a=IJ,Hl,I
contact with the convex lens
(D) If the entire arrangement is immersed in water, the
(B) A convex lens of focal length 60 cm is placed at a conditions will remain unaltered.
distance of 30 cm from the convex lens.
(C) The entire convex lens system is immersed in a 24. An astronomical telescope and a Galilean telescope use
liquid of refractive index i3 identical objective lenses. They have the same
magnification, when both are in normal adjustment.
(D) The ~ntire convex lens system is immersed in a The eyepiece of the astronomical telescope has a focal
length I . Select the correct statement(s).
liquid of refractive index !!_
8 (AJ The tube lengths of the two telescopes differ by I .
(BJ The tube lengths of the two telescopes differ by
19. A converging lens is used to for_m an image on a screen. 21.
When the upper half of the lens is covered by an
(C) The Galilean telescope has shorter tube length.
opaque screen
(D) The Galilean telescope has longer tube length.
(A) half.of the image will disappear
(B) complete image will be formed
25. Two plane mirrors M 1 and M 2 are placed parallel to
(C) intensity of the image will increase
(D) intensity of the image will decrease each other 20 cm apart. A luminous point object 10 1 is
placed between them at 5 cm from Mt as shown.
20. For a mirror, the linear rnagnificatioh ·is +2 . The ,._...20 cm---1>1
conclusion(s) that can be drawn from this information
is/are
(A) The mirror is concave
(B) The mirror can be convex or concave but it cannot .............. !··--·---···
be plane i 0 j
(C) The object lies between pole and focus I
(D) The object lies·beyond focus M,! M,

21. A ray of light has speed v0 frequency lo and (A) The distances (in cm) of three nearest images from
mirror Mt are 5, 35 and 45 respectively.
wavelength A.0 in vacuum. When this ray of light-enters
in a medium of refractive index µ , corresponding
(BJ The distances (in cm) of three nearest images from
mirror M, are 5, 35 and 45 respectively.
values are v , I and "- . Then
(C) The distances (in cm) of three nearest images from
(A) I= lo (B) mirror M 1 are 15, 25 and 55 respectively.
' µ (D)_ The distances (in cm) of three nearest images from
(C) V = Vo (D) I= lo mirror M 2 are 15, 25 and 55 respectively.
µ
26. In the case of hypermetropia
22. For which of the pairs of u and I for curved mirror(s), (A) the image of a near object is formed behind' the
the,image formed is smaller in size. retina.
(A) u=-45cm, l=-l0cm (BJ the image of a distant object is formed in front of
(B) u=-10cm, l=20cm the retina.
(C) a concave lens should be used for correction.
(C) u=-60cm, l=30cm
(D) a convex lens should be used for correction.
(D) u=-20cm, l=-:-30cm
27. Which of the following produce a virtual image longer
23. A di~erging lens of focal leng\h I, is placed in front of in size than the object?
and coaxially with a concav~ mirror of focal length / 2 • (A) Concave lens (BJ Convex lens
Their separation is d . A parallel beam of light incident
(CJ Concave mirror (D) Convex mirror

==================================== 1.137 =
www.puucho.com
Rahul Sardana with www.puucho.com

Advanced JEE Physics Optics & Modem Physics

28. A concave mirror has focal length 15 cm. Where should


an object be placed in front of the mirror so that the
image formed is three times the size of the object?
(A) 7.5 cm
(C) 17.5 cm
(B) 10 cm
(D) 20 cm
--C µ = 1.5

(A) appear to meet after extending the refracted rays


backwards.
29. A concave mirror of focal length f forms an image 2 (B) actually meet at some point.
times the size of object. The object distance from the (C) meet (or appear to meet) at a distance of 60 cm
mirror is from the spherical surface.
(D) meet (or appear to meet) at a distance of 30 cm
(A) j_ (B) 4/
4 3 from the spherical surface.

(C) 3/ j_
2
(D) 2 35. The focal length of a lens in air and refractive index are
f and µ respectively. The focal length changes to / 1
30. A point object P moves towards a convex mirror with when the lens is immersed i:r:t a liquid of refractive
a constant speed V, along its optic axis. The speed of index !:and it becomes / 2 when the lens is immersed
.the image 2 '
(A) is always less than V. in a liquid of refractive index 2µ . Then
(B) may be less than, equal to or greater than V, 2(µ-1) (B) 2{µ-1)
depending on the pcisition of P . w h f h f
(C) increases as P comes closer to the mirror.
µ-1
(D) decrease as P comes closer to the mirror. · (C) (D) !,=-
!
31. A bird flies down vertically towards a water surface. To
a fish inside the water, vertically below the bird, the 36. Two thin lenses, when in contact, produce a
bird will appear to combination of power +10 dioptre. When they are
(A) be closer than its actual distance. 0.25 m apart, the power is reduced to +6 dioptre. The
(B) be farther away than its actual distance. respective powers of the lenses in dioptre, are
(C) move slower than its actual speed. (A) 1 and 9 (B) 2 and 8
(D) move faster than its actual speed. (C) 4 and 6 (D) 5 each
32. There are three optical media 1, 2 and 3 with their 37. A solid, transparent sphere has a small, opaque dot at
refractive indices µ 1 > µ 2 > µ 3 • S~lect the correct its centre. When observed from outside, the apparent
statement(s) position of the dot will be
{A) When a ray of light travels from 3 to 1 no TIR will (A) independent of the refractive index of the sphere.
take place. (B) closer to the eye than its actual position.
(B) Critical angle between 1 and 2 is less than the (C) farther away from the eye than its actual position.
critical angle between 1 and 3. (D) the same as its actual position.
(C) Critical angle between 1 and 2 is more than the
critical angle between 1 and 3. 38. For a concave mirror
(D) Chances of TIR are more when ray of light travels (A) virtual image is always larger in size
from 1 to 3 as compare to the case when it travel (B) real image is always smaller in size
from 1 to 2. (C) real image is always larger in size
(D) real image may be smaller or larger in size
33. An equilateral prism has a refractive index ..Ji. . Select
the correct alternative(s). 39. During refraction, ray of light passes undeviated, then
(A) Minimum deviation from this prism can be 30° (A) medium on both sides is same
(B) Minimum deviation from this prism can be 45° (B) angle of incidence is 90°
(C) At angle of incidence 45°, deviation is minimum (C) angle of incidence is 0°
_(D) At angle of incidence 60°, deviation is minimum (D) medium on other side is rarer

34. Parallel rays of light are falling on a convex spherical 40. A ray of light travelling in a transparent medium falls
surface of radius of curvature R = 20 cm and refractive on a surface separating the medlUm from air at an angle
of incidence 45° . The ray undergoes total internal
index µ = 1.5 as shown. After refraction from the
reflection. If n is the refractive index of the medium
spherical surface, the parallel rays

= ====================================
1.138

www.puucho.com
Rahul Sardana with www.puucho.com

Ray Optics
with respect to air, select the possible value(s) of n (D) If the_ entire arrangement is.immersed in water, the
from the following , · conditions will remain unaltered.
(A) 13 (B) 1.4
(C) 1.5 (D) 1.6 45. Check the wrong statement(s)
(A) A concave mirror can give a virtual image. ·
(B) A concave mirror can give a diminished virtual
41. A convex lens made of glass ( µ, = ¾) has focal length image.
f in air, The image of an object placed in front of it is (C) A convex mirror can give a real image.
(D) A convex mirror can give a diminished virtual
real, inverted and magnified. Now the whole image.
arrang~ment is immersed in water (µ"' =¾) without
46. ,When lights of different colours move through water,
changing the distance between object and lens, then they must have different ,
(A) the,new focal length becomes 4/ (A) wavelengths (B) frequencies
(C) velocities (D) amplitudes
(B) the new focal length becomes J_
. 4
47. A thin concavo-convex lens has two surfaces of radii of
(C) the new image formed will be virtual and
magnified. curvature R and 2R . The material of the lens has a
(D) the new image formed will be real and diminished. refractive index µ . When kept in air, the focal length of
the lens
42. . A thin, symmetric double-convex lens of power P is (A) will depend on the direction from which light is
cut into ,three parts A , B and C as shown. The power incident on it.
of (B) will be the same, irrespective of the direction from
which light is incident on it
2
A
(C) will be equal to R .
µ-1
(D) will be equal to ~.
µ-1
B C
48. A convex; mirror is used to form .an image of a real
object. The image ,
(A) Ais·P (B) A is 2P (A) always lies between the pole and the focus.
. p . p (B) is diminished in size.
(C) B lS - (D) B IS - (C) is erect.
_2 ' 4
(D) is.real.
43. A watch glass having uniform thickness and having
average radius of curvature of its two surfaces much 49. Which of the following form(s) the virtual and erect
larger than its thickness is placed in the path of a beam image for all positions of object ?
(A) concave _mirror (B) convex lens
of parallel light. The beam will
(A) be completely unaffected. (C) convex mirror (D) concave lens
(B) converge slighUy.
50. A ray of light is incident on a prism of refracting angle
(C) · diverge slighUy.
A . C is the critical angle for the material of ihe prism
(D) converge or diverge slightly depending on'
whether the beam ·is incident from the concave or with respect to the surrounding material (say
the convex side. air/vacuum). .
(A) An emergent ray will be there for all values of C .
44. A converging lens of focal length / 1 is placed in front (B) An emergent ray will be there only for A < 2C .
(C) A-ray incident at an angle i can pass through the
of and coaxially with a convex mirror of focal length f, .
Their separation is d . Aparallel beam of light incident. prism if sini> sin(A-C) for C<A<2C.
· sinC
on · the lens returns as a parallel beam from the
(D) None of above is correct.
. arrangement. Select the correct'statement(s) ..
(A) The beam diameters of tne incident and reflected
beams must be the same. · .,.
51. A thin plane-convex lens of focal length f is split into
two equal halves. One of the halves is shifted along the
(Bl d =t, -211,I
optical axis as shown. The separation between object
(q d :c1, -lt,1 and image planes is 1.8 m and the magnification of
1.139=

www.puucho.com
Rahul Sardana with www.puucho.com

Advanced JEE Physics Optics & Modern Physics

image formed by one of the half lens is 2: The


separation between two halves is d . (D) f =½[1- ~1- :, ]
53. An object is placed at a distance 2/ from the pole of a
curved mirror of focal length f
(A) The linear magnification is 1 for both. types of
curved mirror.
(B) The linear magnification is 1 for a concave mirror.
,+----1.8 m-----+<
(C) The linear magnification is !3 for a convex mirror.
(A) /=0.4m (B) f =0.6 m
(C) d=D.6m (D) d=0.4m (D) Data Insufficient.

54. If a convergent beam of light passes through a


52. A point source of light is placed at a distance h below
diverging lens, the result
the surface of a large and deep lake. If f is the fraction of
(A) may be a convergent beam.
light energy that escapes directly from water surface (B) may be a divergent beam.
and µ is refractive index of water then, (C) may be a parallel beam.
(A) f varies as a function of h (D) must be a parallel be~.
(B) f is independent of value of h
1
(C) f 2~µ 2 -1

= ====================================
1.140

www.puucho.com
Rahul Sardana with www.puucho.com

This sectiqn contains Reasoning type questions, each having four choices (A), (B), (C) and (D) out of which ONLY ONE is
correct. Each question contains STATEMENT 1 and STATEMENT 2. You have to mark your answer as
Bubble (A) If both statements are TRUE and STATEMENT 2 is the correct explanation of STATEMENT 1.
Bubble (B) If both statements are TRUE but STATEMENT 2 is not the correct explanation of STATEMENT 1.
Bubble (C) If STATEMENTl is TRUE and STATEMENT 2 is FALSE.
Bubble (D)·lf STATEMENTl is FALSE but STATEMENT 2 is TRUE.

1. Statement-I: 5. Statement-I:
A parallel beam of light traveling in air can be displaced The images formed due to total internal reflections are
laterally by a parallel transparent slab by distance more much brighter than those formed by mirrors or lenses.
than the thickness of the plate.
Statement-2:
Statement-2: There is no loss of intensity in total internal reflection.
The lateral displacement of light traveling in air
increases with rise j.n value of refractive index of slab. 6. Statement-I:
A bird in air is diving vertically with speed v0 over a
2. Statement-I: tank filled with water and having flat silvered bottom
Even in absolutely clear water, a diver cannot see very serving as plane mirror, it observes velocity of its image
clearly. in silvered bottom of tank as 2v0 upward relative to
Statement-2: itself.
Velocity of light is reduced in water. Statement-2:
Bird and its image in bottom mirror are always
3. Statement-I: equidistant from bottom mirror.
Spherical aberration of a lens can be reduced by
blocking the central portion or peripheral portion of the 7. Statement-1:
lens. We cannot produce a real image by plane or convex
Statement-2: mirrors under any circumstances.
Spherical aberration arises ol1 account of inability of the Statement-2:
lens to- focus central and peripheral rays at the same The focal length of a convex mirror is always taken as
point. positive.

4. Statement-I: . 8. Statement-I:
For total internal reflection, angle of incident in denser If a light ray is incident on aily one of the Mo mirrors
medium must be greater than critical angle for the pair inclined at 90° with each other, then finally the
1 r
of media in contact. emergent ray is antiparallel with incident ray.
Statement-2: Statement-2:
1 Finally, the reflected and initially incident rays are in
µ = sinC , where the symbols have their standard
same phase when successively reflected from two
meaning. perpendicularly inclined mirrors.
r::::=====================================i.141 =
www.puucho.com
Rahul Sardana with www.puucho.com

Advanced JEE Physics Optics & Modern Physics

9. Statement-1: 15. Statement-1:


The formula connecting u , v and f for a spherical For observing traffic at our back, we prefer to use a
mirror is valid only for mirrors whose sizes are very convex mirror.
small compared to their radii of curvature. Statement-2:
Statement-2: A convex mirror has a much larger field of view than a
Laws of reflection are strictly valid for plane surfaces, plane mirror or a concave mirror.
but not.for large spherical surfaces.
16. Statement-1:
10. Statement-I: When a ray of light enters glass from air, its frequency
The focal length of the mirror is f and distance of the decreases.
object from the focus is u , the magnification of the Statement-2:
The velocity of light in glass is less than that in air.
mirror is L.
u
17. Statement-1:
Statement-2: A ray incident along normal to the mirror retraces its
size of image path.
Magnification=
size of object
Statement-2:
In reflection, angle of incidence is always equal to angle
11. Statement-1: of reflection.
Although the surfaces of the lens used in goggles are
curved, it does not have any power. 18. Statement-1:
Statement-2: A concave mirror of focal length in air is used in a
In case of goggles, the lenses are concavo-convex and medium of refractive index 2. Then the focal length of
both the surface of lens have equal radii of curvature. mirror in medium becomes double.
Statement-2:
12. Statement-1: The radius of curvature of a mirror is double of the
A convex lens behaves as a concave lens when placed in focal length.
a medium of refractive index greater than the refractive
index of its .material. 19. Statement-1:
Statement-2: Light from an object falls on a concave mirror forming a
Light in that case will travel through the convex Jens real image of the object. If both the object and mirror are
from denser to rarer medium. It will bend away from immersed in water, there is no change in position of the
normalr i.e., the convex lens would diverge the rays and image.
behave as collcave. Statement-2:
The formation of image by reflection does not depend
13. Statement-1: on surrounding medium, so there is no change in
The minimum distance between an object and its real position of image.
image formed by a convex lens is 2 f .
20. Statement-1:
Statement-2:
A convex lens can be convergent in one medium and
The distance between an object and its real image is
divergent in other medium.
minimum when its magnification is one.
Statement-2:
14. Statement-1: In denser medium, convex lens is convergent and in
rarer medium, convex lens is divergent.
The lens formula .!. =.!. _.!, indicates that focal length
f V U

of a lens depends on distances of object and image from 21. Statement-1:


the lens. For a prism of refracting angle 60° and refractive index
../2. minimum deviation is 30° .
Statement-2:
The formula does indicate but when u is changed v Statement-2:
also changes, so that /. of a particle lens remains
At minimum deviation, r1 = r2 = A =30°
constant. 2

= 1.142 ====================================
www.puucho.com
Rahul Sardana with www.puucho.com

Ray Optics
22. Statement-1: Statement-2:
There exist two angles of incidence for the same Dispersion of light is. the phenomenon of splitting of a
magnitude of deviation (except minimum deviation) by beam of white light into its constituent colours .
a prism kept in air. •
27. Statement-1:
Statement-2:
Convex mirror always form a virtual i:rhage.
For a prism kept in air, a ray is incident on first surface
and emerges out of second surface (of prism) along the Statement-2:
previous emergent ray, then this ray emerges out of Focal length of a mirror is half of the radius of
first surface along the previous incident ray. This curvature.
principle is called the Principle of Reversibility of Light.
28. Statement-1:
23. Statement-1: A fish inside a pond will see a person standing outside
A plane convex lens is silvered from plane surface. It taller than he is actually.
can act as a. diverging mirror.
Statement-2:
Statement-2: Light rays from person converges into eyes of fish on
Focal length of concave mirror is independent of entering water from air.
mediuni.
29. Statement-1:
24. Statement-1: Optical fibre has thin glass core coated by glass of small
Maximum distance of image formed by convex mirror refractive index and is used to send light signals.
from pole of mirror equals 'f' for all the objects
Statement-2:
(real/virtual). All the rays of light entering the fibre are totally
Statement-2: reflected even at very small angles of incidence.
Convex mirrors forms virtual images for objects placed
in front_ of mirror. 30. Statement-1:
The mirror used in search light are parabolic and not
25. Statement-1: concave spherical.
We cannot produce a real image by plane or convex Statement-2:
mirror under any circumstances. In a concave spherical mirror the image formed is
Statement-2: always virtual.
Reflection Law is valid for plane mirror as well ,as
convex mirror.

26. Statement-1:
There is no dispersion of light refracted through a -
rectangular glass slab. ·

i:::::===================================1.143=
www.puucho.com
. .
Rahul Sardana with www.puucho.com

This section contains Linked Comprehension Type Questions or Paragraph based Questions. Each set consists of a Paragraph
followed by questions. Each question has four choices (A), (13), (C) and (D), out of which only one is correct. (For the sake of
competitiveness there may be a few questions that may h~ve more than one c·orrect options)

Comprehension 1 20,,'3-1
(A) 20( ,/3 -1) (B)
A ray of light is incident at 45° on the face AB of an ,/3
equilateral prism ABC which has the face AC silvered. 20
(C) ,/3 (D) 10( ,,'3 - 1)
Based on the information provided answer the following
questions.
A
. 4. The linear magnification produced by convex lens and
\ concave lens individually is
~
\
\
(A) ( ,,'3 + 1) and 1
\
(13) ,,'3 and 1
B C (C) ( ,/3 + 1) and ( ,/3 - 1)
1. The refractive index µ of the material of th~ prism so (D) ,/3 and (2,/3 -3)
that when the ray falls on face BC (after reflecting from
AC) it makes an angle 60° with it is Comprehension 3
A telescope is an optical instrument used to increase the
(A) ,,'3 (B) .J2 visual angle of distant objects such as stars, planets etc. Ah
(C) 2 (D) 1.5
astronomical telescope consists of two converging lenses.
The one facing the object is called objective and the lens close
2. The total deviation, when the ray of light finally
to the eye is called an eyepiece. It can be adjusted by
emerges from BC is displacing relative to the objective. The angular
(A) 120° (B) 180° magnification is defined as the ratio of focal length of
(C) 150° (D) 90° objective and eyepiece. One can see the image with
unstrained eye if it forms at infinity. An astronomical
Comprehension 2 telescope has an objective of focal length 50 cm and a
A convex lens of focal length 20 cm and a concave lens magnification of 20. Based on above information, answer the
. of focal length 10 cm are placed 20 .cm apart. In between following questions .
them an object is placed at distance x from the convex lens.
Based on the information provided answer the following 5. Focal length of the eyepiece is
questions. (A) 2.5 cm (B) 5 cm
(C) 7.5 cm (D) None of these
3. The value of x (in cm) so that images formed by both
the lenses coincide is 6. To view remote object by an unstrained normal eye,
separation between two lenses will be

= ====================================
1.144

www.puucho.com
Rahul Sardana with www.puucho.com

Ray Optics

(A) 55 cm (B) 57.5 cm (D) As frequency of light increases then refractive


(C) 60 cm (D) 52.5 cm index of glass increases

7. If object is at a distance 600 m from the telescope to see Comprehension 5


the image with unstrained eye separation between two The figure shows a convex lens of focal length 15 cm .
lenses should be (in cm) A point object is placed on the principle axis of lens at a
(A) 46.65 (B) 47.65. distance 20 cm from it as shown. On the other side of the
(C) 49.96 (D) 49.65 lens two observer eyes 0 1 and 0 2 are situated at a distance
100 cm from the lens at some distance above and below the
Comprehension 4
principle axis.
Speed of light in a medium of refractive index n is
f=15cm
given by ~ where c is speed of light in vacuum refractive
n
index of a medium depends on wavelength (;\.). As 0
wavelength increases refractive index decreases. It is also 20cm
given
A.red > Aorange > /!.yellow
Based on above information, answer the following questions. 1+---- 100 cm ------t-1

Now half position of lens below principle axis is painted


8. In glass black Based on above information, answer the following
(A) orange light travels faster than yellow light
questions.
(B) yellow light travels faster than orange light
(C) yellow light travels faster than red light 13. In initial setup (before painting the lens) which of the
(D) orange light travels faster than red light following statement is correct.
(A) Observer 0 1 will see a real image at 60 cm from
9. The quantity that remains unchanged if light enters
from water to glass is the lens but observer 0 2 will not be able to see it
(A) Wavelength and colour (B) Observer 0 2 will see a real image at 60 cm from
(B) Refractive index and frequency the lens but observer 0 1 will not be able to see it
(C) Frequency and velocity
(C) Both the observers will see a real image at 60 cm
(D) Colour and frequency
from lens irrespective the positions of 0 1 and 0 2
10. The phenomenon that happens because of variation of (D) Both the observers may or may not be able to see
wavelength is the image at 60 cm from lens depending on the
(A) Aberration positions of 0 1 and 0 2
(B) Dispersion
(C) Total internal reflection 14. After painting the lens, which of the following observer
(D) Bending of light will not be able to see the image of object, if before this
activity both were seeing the image
11. Which of the following statement is true? (A) 0 1 (B) 0 2
(A) Time taken (t) by yellow light to travel distance (C) Both 0 1 and 0 2 (D) Neither 0 1 nor 0 2
X0 in refractive index n can be t::;;; nxo
c, 15. After painting the lens, for which observer the intensity
(B) Time taken (t) by yellow light to travel distance of image will be reduced to half?
(A) For 0 1
·
x0 .mrefactivem nx,-
r " " d ex n can b e t ~
c (B) For 0 2
(C) Since wavelength of yellow light increases in (C) Both for 0 1 and 0 2
refractive index n its frequency must decreases. (D) Neither for 0 1 nor for 0 2
(D) None of the above
/
12. Which of the following statement is false? Comprehension 6
(A) Light is a electromagnetic wave The figure shows a ray incident at an angle i =2: . If the
(B) Speed of light of each colour is same in vacuum 3
(C) Time to cover distance x0 in a medium is same for
plot drawn shows the variation of Ir - ii versus 1:1 = k ,
each colour µ,

c========================================:::i 1.145

www.puucho.com
Rahul Sardana with www.puucho.com

Advanced JEE Physics Optics & Modern Physics

where r is the angle of refraction, then based on above (C) J, < J. (D) None of these
information, answer the following questions.
Ir-ii 20. If angle of incidence is 60°, then focal length of this
marginal ray is
92 ------------------ R
(A) R (B) 2
µ,
µ, e, (C) 2R (D) 0

21. The total deviation suffered by the ray falling on mirror


at an angle of incidence equal to 60° is
(A) 60° (B) 90°
(C) 30° (D) Cannot be determined
16. The value of k1 is
2 Comprehension 8
(A) (B) 1
../3 The XY plane is the boundary between two
1 transparent media. Medium-I with z ';?_ 0 has a refractive
(C) (D) ../3
../3 2 index ..ff. and medium-II with z:, 0 has a refractive index
../3. A ray of light in medium-I given by
17. The value of 0 1 is A = 6$; + B../3] - lOk is incident on the plane of separation.
1t 1t Based on the above facts, answer the.following questions.
(A) (B)
3 2
1t 22. The vector representing the incident ray has a
(C) (D) ZERO
6 magnitude of
(A) 5 units (B) 10 units
18. The value of k2 is (C) 15 units (D) 20 units
(A) 1 (B) 2
23. The angle of incidence is
1
(C) -2 (D) None of these (A) 30° (B) 45°
(C) 60° (D) 90°
Comprehension 7 24. The angle of refraction is
Spherical aberration in spherical mirrors is a defect (A) 30° (B) 45°
which is due to dependence of focal length f on angle of (C) 60' (D) 90'
incidence 0 as shown in figure is given by
R 25. The refracted ray is represented by the vector given by
J=R--sec0
2 (A) _6../3i -8../3 j + 10../3 k
where R is radius of Curvature of mirror and 0 is the angle
(B) 8../3]
of incidence. The rays which are closed to principal axis are
called paraxial rays and the rays far away from principal axis (C) -10../3k
are called marginal rays. As a result of above dependence (D) 6$; + s../3]- 10../3/;
different rays are brought to focus at different points and the
image of a point object is not a point. 26. The vector representing the refracted ray has a
B ,- magnitude of
_,,,,-1! (A) ,/6 units (B) 10 units
~P.'.cri'.'..'.nc~ipt'.'.a~l.'.'.ax~is:'.,ac,''...'_'_~----+:\.P'..co~le (P) (C) 10,/6 units (D) 20,/6 units
C
27. The unit vector along refracted ray is
3 , 4 , 1 •
Based on above information, answer the following questions. (A) 5..fi. i+ 5..fi_ I - ..fi. k
(B) 3/ +4]-5k
19. If J, and J. represent the focal length of paraxial and
3 , 4 , 1 •
marginal rays respectively, then correct relationship is (C) 5..fi., - 5..fi_ I+ ..fi. k
(A) J, =J. (B) J, > J. (D) 2i -3k
= 1.146r::::=================================
www.puucho.com
Rahul Sardana with www.puucho.com

Ray Optics
Comprehension 9 Comprehension 10
The lens governing the behaviour of the rays namely Consider an equiconvex lens of radius R , made of a
rectilinear propagation, laws of reflection and refraction can material of refractive index µ. Its focal length is / 1 when
be summarised in one fundamental law known as Fermat's any one face is silvered. Now consider another plano-convex
Principle. According to this principle a ray of light travels lens of radius R, made of same material having focal length,
from one point to another $UCh that the time taken is at a
/ 2 when no face is silvered, / 3 when plane face is silvered
stationary value (maximum or minimum). If c is the
.velocity of light in a vacuum, the velocity in a medium of and / 4 when curved surface is silvered. Based on above
information, answer the following questions.
refractive index n is .£., hence time taken to travel a
n
31. f, equals
distance I is nl . If the light passes through a number of R 2R
C (A) (B) ---
(2µ-1) (2µ-1)
media, the total time taken is ( ~) L nl or ~ fndl if refractive R 2R
(C) (D)
index varies continuously. Now, Lnl is the total optical 2(2µ-1) 2(2µ + 1)
path, so that Fermat's Principle states then the path of a ray
is such that the optical path in at a stationary value. This 32. f, equals
principle is obviously in agreement with the fact that the ray R
(A) R (B)
are straight lines in a homogeneous isotropic medium. It is 2µ-1
found that it also agrees with the classical laws of reflection R R
and refraction. Based on above information, answer the (C) (D)
µ-1 µ+1
following questions.

28. If refractive index of a slab varies as m = 1 + x 2 where


33. f, equals
x is measured from one end, then optical path length R R
(A) (B)
of a slab of thickness 1 m is 2µ 2(µ-1)
4 3 2R
(A) - m (B) - m (C) (D) 2Rµ
3 4 µ
(C) lm (D) None of these
34. f. equals ·
29. The optical path length followed by ray from point A R 2R
to B, given that laws of reflection are obeyed as shown (A) (B)
µ µ
in figure is
R
A B (C) 2Rµ (D)

Comprehension 11
SITUATION-I
Two identical piano-convex lenses L1 and L2 having
(A) Maximum (B) Minimum radii of curvature R = 20 cm and refractive indices µ, =1.4
(C) Constant (D) None of these and µ 2 =1.5 are placed as shown in the figure .

30. The optical path length followed by ray from point A


......
......
to B, given that laws of reflection are obeyed as shown ......
in figure is ......
A B
......
---+- ---------

......
......
......
SITUATION-II
p Now, the second piano-convex lens is shifted vertically
(A) Maximum (B) Minimum downward by a small distance of 4.5 mm and the extended
(C) Constant (D) None of these parts of L1 and L2 are blackened as shown in figure.

c:=================================== 1.147=
www.puucho.com
Rahul Sardana with www.puucho.com

Advanced JEE Physics Optics & Modern Physics


---+
---+. 38. The nature of final image of the object when x = 2R is
---+
---+
(A) Erect and.magnified
Principal axis~
~----,----,-- - ~ ----,--_--- of lens Li
(B) Inverted and magnified
---+ -(q· Erect.and same size
---+ (D)· Inverted and same size
---+ L,
---+ 39. Jt, is observed that for x = R , . a ray starting from 0
strikes the spherical surface .at grazing incidence. Th_e
Based on above information, answer the following questions.
angle with the normal at which the ray emerges from
35. In SITUATION-I, the position of the image of the the plane S1.!I'face is
parallel beam· of light relative to the common principal (A) 90° (B) 0°
axis is (C) . 30° (D) 60°
(A) 1~0 cm (B) 100 cm
3 Comprehension 13
200 The refractive indices of'the crown glass for blue and
(C) 200 cm (D) - c m red lights are 1.51 and 1.49 respectively and those of the flint
3 9
glass are 1.77- and ·1.73 respectively. An isosceles prism of
36. In SITUATION-II, the new position of the image of the angle 6~ is made of crown glass .. A beam of white light is
parallel beam is incident at a small angle on this prism. The other flint glass
2 0 isosceles prism is combined with the crOwn·glass prism such
(A) ~ cm in front of the lens 2 mm below the that there is no deviation of the incident light. Based on the
above facts, answer the following questions.
principal axis of L, .

(B) l~O cm behind the lens 2 mm below the principal 40. The refractive.index·of crown glass f9r yellow,colour is .
(A) ,1.51 (B) 1.49
axis of_ L,. (C) 1.50 (D) 1.59
2 0
(C) ~ qn behind the lens 2.5 mm below the 41. The refractive index of flint glass for yellow colour is
principal axis of L,. (A) 1.70 (B) 1.72
(C) 1:73 (D) 1.75
2 0
(b) ~ cm in front of the lens 2.5 mm below the
42. The refracting angle of flint glase;prisni is
principal axis of L, . (A) +2° (B) +4° .
(C) -20 (D) -40
Comprehension 12
A small object O is placed in air at the principal axis at 43. The net dispersion produced by the combined system is
a distance x from the pole of the curved .surf~ce of a (A) 0,02° (B) , -0.02°
transparent hemisphere having refractive index 2 ,and (C) +0.04° (D) -0.04°
radius R as shown. Based on above information, answer th~
following questions. . Comprehension 14
An equilateral prism AllC is placed in air with its base
side BC lying horizontally along x-axis as shown in figure.
A ray of light represented by equation ,J3z + x =10 is
n=2 incident a:t a point P ·on- the face AB of prism. Based on
0 above information, answer the following questions.
z
• A

,_ X --+--R---+<

37. The value of x, for which the final image of the object
at O will be virtual is " • BJLJ-6_0_'---'---'(2__ ,,,.,. X ·
(A) 2R (B) 3R " / 0,0,0)
'/
(C) ! (D) l.5R y"/
3

= ====================================
1.148

www.puucho.com
Rahul Sardana with www.puucho.com

. Ray.Optics

44. The value of µ, for which the ray.grazes the face AC is the eyepiece as. the first image? Assume that the first
(A) -~ . (B) 4 · . image distanc~ is _d; from the eyepiece. '
·2 ".' 3· (A) d, < f. ' . (B) d, =J,°
·2 .ff, (C) J,<d,<2/, (D). d,>2/,
(C) .fj (D) 2
49. Two compound microscopes A ·!ID~. B were compared.
Both had objectives and eyepieces with the same
45. The direction of the finally refracted ray for µ = ~ is
2 magnification but 4, gave an overall magnification that
(A) parallel to x-axis (B) parallel to z-axis, was greater than that of B . Which of the following is a
(C) parallel to y-axis · (D) parallel to face AB possible explanation? ,. . .
(A) The distance between object'and eyepiece in A is
46. The equation of ray emerging out of prism, if the greater thaii. the cOITesponding distance in B.
bottom BC is silvered is (B) The dis.lance between object and eyepiece in A is
(A) z + .fSx =10 (B) .f3z+x=10 le~s than the corresponding distance in B .
(q, The eyepiece anq. objective positions were reversed
\C) z+.fSx ;;20 (D) X + Z =10.fS in A. '
(D) The eyepiece and objective positions were reversed
Comprehension 15 in B.
The schematic diagram of a compound microscope is
shown in the adjacent figure. Its main components are two Comprehension 16
convex lenses: one acts as the main· magnifying lens and iS
A ray of light travelling in air is incident at grazing
refer,ed to ~s the objective, and another lens called the angle on a long rectangular slab of a transparent medium of
eyepiece. The two lenses act independently of each other thickness t = 1.0 m . The point of incidence O is the origin
when bending light rays. ·
(0, 0). The.medium has a variable index of refraci;ion µ(y)
' 'Eyepiece
1
.
given by µ(y)=[Ky'1'+1]1/2 where K=1.0(m)"1'. The
Objective
refractive index of air is 1.0.
V
0

/ 0= focal point of objective


J, = focal point of eyepiece
Llght from the object (0) first passes through the objective
and enlarged, inverted first image is formed. The eyepiece
then magnifies thisimage. Usually the magnification of the Based on the above facts, answer the following questions.
eyepiece is fixed (either x 10 or x 15) and three rotating
objective lenses are used : x 10, x· 40 and x 60. Angular 50. The relation between the slope of the trajectory of the
magnification is defined as the angle subtended by the final ray at the point B( x, y) in the medium and the. angle
image at the eye to the angle subtended by the object placed of incidence (i) at that point is given by
at least distance of distinct vision (,,, 25 cm) when viewed (A) tan0.= sini (B) tane = 2sini
by the naked eye. Based on above information, answer th'e (C) tane = coti.. (D). , tane = 2coti
following questions.

47. The type of image that would have to be produced by


51. The equation for the trajectory y(x) of the ray in.the
the objective is medium is
(A) Either virtua} or real " x' x'
(A) y=- (B) y=16 ·
(B) Virtual 16
(C) Real x' ·x•
(D) It depends on the focal length of the lens. (C) y= 16 (D) y= 256

48. Where would the first image have to be produced by


52. The co-ordinates ( x, y) •of the point where the ray
the objective relative to the eyepiece such that a second,
enlarged image would be generated on the same side 6£ ' intersects the upper surface of the· slab-air boundary are

i============::::::;=======================1.149 =
www.puucho.com
Rahul Sardana with www.puucho.com

Advanced JEE Physics Optics & Modern Physics

(A) (1,l)m (B) (2, 1) m 57. The position at which the image is formed is
{A) 30 cm, right oflens (B) 40 cm, left of lens
(C) (3, 1) m (D) (4, 1) m (C) 60 cm, right of lens (D) 70 cm, left of lens

53. The ray finally emerges 58. The total number of images generated by the
{A) parallel to the incident ray arrangement is/ are
(B) perpendicular to the incident ray (A) 1 (B) 2
(C) at an angle of 30° to the incident ray (C) 4 (D) 6
(D) ·at an angle of 45° to the incident ray
59. The spacing between the images so formed is
Comprehension 17 (A) 0.1 cm (B) 03 cm
The convex surface of a thin concavo-convex lens of (C) 0.5 cm (D) 1 cm
glass of refractive index 1.5 has a radius of curvature of
20 cm . The concave surface has a radius of curvature of Comprehension 19
60 cm . The convex side is silvered and placed .on a A thin biconvex lens of refractive index ~ is placed on
horizontal surface as shown in the figure. 2
a horizontal plane mirror as shown in figure.
''
''
~ r=60cm

The space between the lens and the mirror is then filled with
20cm
---- ---
Based on above information, answer the following questions.
water of refractive index ! . It is found that when a point
3
object is placed 15 cm above the lens on its principal axis, the
54. The focal length of the combination has the magnitude object coincides with its own image. On repeating with
{A) 1.5 cm (B) 15 cm another liquid, the object and the image again coincide at a
(C) 7.5 cm (D) 8.6 cm distance 25 cm from the lens. Based on the above facts,
answer the following questions.
55. The combination behaves like
(A) a convex mirror (B) a concave mirror 60. The radius of curvature of both the surfaces of the
(C) a convex lens (D) a concave lens convex lens is R , then the focal length of the convex
lens is
56. A small object is placed on the principal axis of the R
combination, at a distance of 30 cm in front of the (A) R (B)
2
mirror. The magnification of the image is
R R
1 3· (C) (D) 8
(A) (B) 4 4
3
1 61. When .the space between the lens and mirror is filled
(C) 5 (D)
4 with water, the focal length of water concave lens is
(A) R (B) 2R
Comprehension 18 (C) 3R (D) 4R
A point object O is placed at a distance of 0.3 m from
a convex lens (focal length 0.2 m) cut into two halves each of 62. The radius of curvature R of each surface of convex
which is displaced by 0.0005 m as shown in the figure. lens is
f =20m (A) 2cm (B) 5 cm

0
_________ fi _ _ t____ _
2 X 0.0005 ITT
(C) 10cm

20
(A) - cm
(D) 15cm

63. The focal length of the liquid concave lens is


(B)
40
- cm
3 3
---------v
' . ·i. ___ t____
' _ 50
(C) -cm
70
(D) - cm
l 3 3
l-4--30cm _.,.
64. The refractive index of the liquid is
Based on above information, answer the following qu'7stions.
(A) 1.1 (B) 1.2
(C) 1.4 (D) 1.6
= ====================================
1.150

www.puucho.com
Rahul Sardana with www.puucho.com

Ray,Optics
Comprehension 20 68. The x -coordinate of the focal point of this lens system
The diagram shows an equilateral prism. Th.e medium is
on one side of the prism has refractive index µ 1 • ·The d(/1 ~d)+ f,f, f,f,
refractive index of the prism is µ = Ji · The diagram shows
(A)
f,+f,-d
d(/1 -d)
(B)
f1+f,-d
2d(/1+d)- Ji!,
variation of magnitude of angle of deviation with respect to (C) (D)
!1+f,-d !1+f,-d
µ 1 • Consider the light ray to be incident normally on the
first face.
69. The y -coordinate of the focal point of this lens system
is
C
.Q
(A)
(/1 + d)d 2(/1 + d)
;;; (B)
·~ P, U1 + I, -d) U1 + !, -d)

6
----i-- - -,-----------

"0 '' 2(/1 -d)d (/1 -d)d


P, '
____ 1____

U1 + I, +d) (D)
;ii
O> '' (CJ U1 + !, -d)
''
C
-,:
fl,
0 k, k, Comprehension 22
Based on above information, answer the following questions; The figure shows a surface XY separating two
transparent media, Medium-1 and Medium-2. The lines ab
65. Value of k, is and cd represent wavefronts of a light wave travelling in
6 4 Medium-1 and incident on XY . The lines ef and gh
(A) (B)
-J3 ../3 represent wavefronts of the light wave in Medium-2 after
refraction.
(C)
../3 (D)
8
b d
2 ../3
a~/ Medium-1
66. Value of k1 is
X f h y
(A)

(C)
4
3
8
(B)

(D)
5
3
10
e/4/ Medi~m-2 _

3 3 Based <;>n the above facts, answer the following qu.estions.

67. Value of p1 -P, is 70. Light travels as a


(A) 20' (A) parallel beam in each medium
(B) 30'
(B) convergent beam in each medium
(C) 60° (D) 90'
(C) divergent beam in each medium
(D) divergent beam in one medium and convergent
Comprehension 21
beam in the other medium
Two thin convex lenses of focal lengths / 1 and f, _are
separated by a horizontal distance d ( d < / 1 and d < f,) and 71. The phases of the light wave at c , d , e and f are t,
their centers are displaced l?y a vertical separation as shown $,, $, and $1 respectively. It is given that $, * $j
in the.figure. Take the o_rigin of coordinates O at the center (A) $, cannot be equal to $,
of first lens. For a parallel beam of light coming from the left,
(B) $, can be equal to $,
as shown in figure.
y (C) ($,-t1 ) is equal to($,-$,)

-- ---- ... __ J
(D) (t,-$.) isnotequalto

72. Speed of light is :


($ 1 -$,)

--
~._.-c+--1-------+-1-----'./J.-• X
0

----d---•
Based on above·inforrnation, answerlhe following.questions.
(A) the same in medium-1 and medium-2
(B) larger in medium-1 than in medium-2
(C) larger in medium-2 than in medium-1
(D) different at b and d

t:::=================================== 1.151 il::!::l

www.puucho.com
Rahul Sardana with www.puucho.com

Advanced JEE Physics Optics & Modern Physics

Comprehension 23 77. The distance from itself at which the fish sees the image
A thin piano-convex lens of focal length f is split into· . of eye by viewing in the mirror is
two halves. One of the halves is shifted along the optical
axis. The separation between the object and image planes is
WaG+~ ~ HG+ID
1.8 m. The magnification of the image formed by one of the
half lenses is 2. ·- ·

78. The distance from itself at which the eye sees the, image
of the fish by directly observing the fish is

(A) 2H(1+ 21J (B) 2H(½+¾)


i+----1.8 m - - - . , (C) H(1+ 21J (D) H(½+¾)
Based on above information, answer the following questions.
79. The distance from itself at which the eye sees the image
73. The focal length of the lens used is, of the fish by viewing in the mirror is
(A) 0.4 m
(C) 1 m
(B) 0.6 m
(D) 2m (A) 2(H +~)
2µ (B) H +~2µ ··
74. The separation between the two halves of the thin
plano-conveX lens is
(C) H(½+;) (D) H(l+ :J
(A) 0.2m (B) ,0.4m·
(C) 0.6m (D) 0.8m Comprehension 25
Most materials have the refractive index, n > 1 . So,
75. The magnification for the second half lens is when a light ray from air enters a naturally occurring
(A) 0.5 (B) -0.5
(C) 0.4 (D) -2 materiat then by Snell's law, sin 81 = n2 , it is understood
sin0 2 n1
that the refracted ray bends towards the normal. But it never
Comprehension 24
emerges on the same side of the normal as the incident ray.
Consider a beaker filled with water (of refractive index
According to electromagnetism, the refractive index of the
µ ) to a height H . A fish F is at a height H from the
2
transparent base of the beaker which lies on a surface that
medium is given by the relation, n =(~)=±~erµ, , where _c

happens to be a mirror. An observer whose eye E iS at a is the speed of electromagnetic waves in vacuum, v its
height 2H from the base of beaker is also there. Based on speed in the medium, e, and·µ, are the relative per~ttivity
and permeability of the medium respectively.
above information, answer the following questions.
In normal materials, both e, and µ, .~e p9_s!tive, imply~g

l
,;(/ E
positive n for the medium. When both e, and µ, are
negative, one most choose the _negative root of n . $uch
negative refractive index materials can now be artificially
2H prepared _and. are called· meta-materials. They exhibit

i
H
significantly different optical behaviour, without violating
any physical laws. Since n is negative, it results in a change
t in the direction of propagation of the refracted light.
H/2 However, similar to normal materials, the frequency of light
remains unchanged upon refraction even in metci-m";terials.

76. The distance from itself at which the fish will see the 80. Choose the correct statement.
image of the eye by direct observation is (A) The speed oflight in the meta-material is v =clnl
(A) H(µ_+½) (B) ~{1+~) (B) The speed of light in the meta-material is v =l:I

(C) H(~+ 1) (D) ~ (µ+½) (C) The speed of light in the meta-materials is v =c
(D) The wavelength of the light in the meta-material
(\,) is given by '-m=A_,lnl,where '-.,

= ==================================
1.152

www.puucho.com
Rahul Sardana with www.puucho.com

Each question in this section contains statements given in .two columns, which have to be matched. The statements llJ.
COLUMN-I are labelled A, B, C and D, while the statements in COLUMN-II are labelled p, q, r, s (and t). Any given statement
in COLUMN-I can have correct matching with ONE OR MORE statement(s) in COLUMN-II. The appropriate bubbles
corresponding to the answers to these questions have to be darkened as illustrated in the following examples:
If the correct matches are A--> p, s and t; B --> q and r; C--> p and q; and D --> s and t; then the correct darkening of bubbles will
look like the following :
P Q r s t
A@@(D@@)
B@@@®CD
c@@©®CD
D@@(D@@

1. For a real object, match the magnification situations in COLUMN-I, with their respective matches in COLUMN-II.
r· COI.UMN-I CJOLUMNcII
(A) m<O (p) Plane mirror
(B) m>O (q) Convex- mirror
(C) lml<l (r) Concave mirror
(s) Convex.lens
(D) 1ml 2cl
(t) Concave lens

2. Four particles are moving with different velocities in front of stationary plane mirror that lies in the y-z plane. At t = O,
velocity of A is v, =i, velocity of B i; v, =-i +3],' velocity of C is Ve =5i +6], velocity of D is v0 =3i -] . The
J
acceleration of particle A is iiA= 2i + and acceleration of particle C is iic = 2i + J, whereas.the particle B and D move
with uniform velocity. Assume no Collision to take place till t = 2 s, all quailtities·to be in SI units, the relative velocity of
image of object A with respect to object A to be denoted by vA"A • If velocity of images relative. to corresponding objects
are given in COLUMN-I and their values at t = 2 s are given in COLUMN-II , then match the quantities in COLUMN-I
with the corresponding values in COLUMN-II.
A
D • •B
• C

y

Lx
1.154c::::===================================
www.puucho.com
Rahul Sardana with www.puucho.com

Ray Optics

COLUMN-I COLUMN-II
(A) VA'A (p) 21
(B) VB'B (q) -61
(C) VC'C (r) -121+4]
(D) VD'D (s) -101
(t) Perpendicular to the plane of mirror

3. The COLUMN-I shows some probable directions of velocity of images formed due to system shown in COLUMN-II.
Match the quantities of COLUMN-I with the respective possibilities shown in COLUMN-II.
COLUMN-I COLUMN-II

-
y
0


Principle axis
(p) ----+~---
(A)
Real point object

(B)

+· (q) -
0
Principle axis

~ Real point object

.........0

-+·
Principle axis
(r)
(C)
Real point object

(D)

-+· (s) o/
1-
Real point object

0
.,__ Principle axis
(t) ----++----
\ Virtual point object

4. A bird in air is diving vertically over a tank with a speed of 6 cms-1 . The base of the tank is silvered. A fish in the tank is
rising upward along the same line with a speed of 8 cms-1 • Taking µwater =±, match the quantities in COLUMN-I with
3
their respective values in COLUMN-II.

t==================================== = 1.155

www.puucho.com
Rahul Sardana with www.puucho.com

Advanced JEE Physics Optics & Modern Physics

COLUMN-I COLUMN-II
1
(A) Speed of the image of fish, in cms- as seen by the (p) 16
bird directly (q) 0
(B) Speed of the image of fish, in cms-1 formed after (r) 12
reflection from the mirror as seen by the bird (s) 8
(C) Speed of image of bird, in cms-1 relative to the fish
looking upwards
(D) Speed of image of bird, in cms-1 relative to the fish
looking downwards in the mirror

5. Match the descriptions in COLUMN-I with corresponding plot(s) in COLUMN-II.


i COLUMN-I COLUMN-II
(A) In convex mirror, when object is real and image is
virtual · l~I
(B) In convex mirror, when object is virtual and image is
real. 1+1
(p)
(C) In concave mirror, when object is real and image is
virtual.
(D) In concave mirror, when object is real or virtual and
image is real.

l~I
(q)
l+I
IJI
...c.-··_--····-1-···- - -1

l~I

(r)
- + - - , L - - - -1
l+I __....-
IJI

1.156 c:=====================================
www.puucho.com
Rahul Sardana with www.puucho.com

Ray Optics

-1~1- .
(s)

_,,/_IJI
Iii . . . .-
6. If (µ 1 , i..10 v1 ) and (µ,, i..,, v,) are the refractive indices, wavelengths and speeds of two light waves
respectively, then match the entries of COLUMN-I with the entries ofCOLUMN-U.
COLUMN-I COLUMN-II
(A) µ1 > µ2 (p) V1 < V2
(B) µ1 < µ2· (q) Vl > Vz
(C) µ1 -;t; µ2 (r) A-1 = A.2
(D) µ1 =µ2 (s) A.1 < A.2

7. Match the descriptions in COLUMN-I to corresponding details in COLUMN-IL


I COLUMN-I COLUMN-II
(A) In refraction from a rarer to a denser medium. (p) Speed of wave does not change.
(B) In refraction. (q) Wavelength must be decreased.
(C) In reflection from a denser medium. (r) Frequency does not change.
(D) In reflection. (s)
,.
The reflected ray suffers an additional path change of

· 8. Light rays are incident on devices which may cause either reflection or refraction or both. The nature of the incident light
and the devices are described in COLUMN-I. Some possible results of this on the rays are. given in COLUMN-II.
[ COLUMN-I COLUMN-II
(A) A ray of white light passes from an optically denser (p) Divergent beam
medium to an optically rarer medium. (q) Total internal reflection
(B) A parallel beam of monochromatic light passes (r) Lateral shift
symmetrically through a glass lens. (s) Dispersion
(C) A ray of white light is incident at an angle on a thick
glass sheet.
(D) A ray of white light ls incident on one face of an
equivalent glass prism.

9. For a real object, match the descriptions in COLUMN-I to the corresponding details in COLUMN-II.
\ COLUMNsI . COLUMN-II
(A) Convex mirror (p) Virtual image
(B) Concave mirror (q) Real image
(C) Convex lens (r) Enlarged image
(D) Concave lens (s) D~shed image

==================================== = 1.157

www.puucho.com
Rahul Sardana with www.puucho.com

Advanced JEE Physics Optics & Modem Physics

10. An optical component and an object S placed along its optic axis are given in COLUMN-I. The distance between the
object and the component can be varied. The properties of images are given in COLUMN-II. Match all the properties of
images from COLUMN-II with the appropriate components given in COLUMN-I.
I COLUMN-I COLUMN-II I
(p) Real image
~ (q) Virtual image
(A)
s (r) Magnified image
(s) Image at infinity

b
7
s
(B)

(C)
s


I
(D)
s
\

11. Match the details of COLUMN-I with the respective name and nature described in COLUMN-II.
COLUMN-I COLUMN-II
(p) Converging
/ j
(q) Concavo-convex
(A) (r) Convexo-cOncave
R ft .
(s) Diverging
\,,

(B) R µ R

(C) 2R
R

www.puucho.com
Rahul Sardana with www.puucho.com

Ray Optics

(D) R µ 2R

12 Match the following


I COLUMN-I COLUMN-II
(A) Concave mirror, virtual object (p) Real image
(B) Convex mirror, virtual object (q) Virtual image
(C) Convex lens, real object (r) Magnified image
(D) Concave lens, real object (s) Diminished image

13. For a concave mirror of focal length 20 cm, match the object distances in COLUMN-I to the corresponding details of
images formed in COLUMN-II.
I COLUMN-I COLUMNcII
(A) 10cm (p) Magnified, inverted and real
(B) 30cm (q) Equal size, inverted and real
(C) 40cm (r) Smaller, inverted and real
(D) 50cm (si. Magnified, erect and virtual

14. A point object is placed in front of a plane mirror as :shown and moving with velocity 3 ms-1 towards mirror. Mirror is
moving with speed 2 ms-1 towards object, then

2ms'--1
ms-'
....._.
3 /
l COLUMN-I COLUMN-II I
(A) Speed of image w.r.t. ground (p) 10 ms- 1

(B) Speed of image w.r.t. mirror (q) sms-1


(C) Speed of image w.r.t. object 14ms·1
(r)
(D) Speed of mirror w.r.t. object
(s) 7 ms-1

15. A right angled prism of refractive index µ, is placed in a rectangular block of refractive
index µ 2 , which is s~ounded by a medium of refractive index µ 3 , as shown in the 8 ,-,.+-rl-----'li;:E==---+.- g
figure. A ray of light e enters the rectangular block at normal incidence. Depending
upon the relationships between µ 1 , µ 2 and µ 3 , it takes one of the four possible paths h
ef, eg, eh, or ei. Match the paths in COLUMN-I with conditions of refractive indices in
COLUMN-II and select the correct answer usin the codes ·ven below.
COLUMN-I COLUMN-II
(A) e--> f (p) µ,>,ff.µ,
(B) e-->g (q) µ2>µ1 and µ2>µ3
(C) e-->h (r) µl =µ2
(D) e-). i
(s) µ, < µ1 < ,ff.µ, and µ, > µ 3

1.159=

www.puucho.com
Rahul Sardana with www.puucho.com

In this section the answer to each of the question is a four digit integer, ranging from 0 to 9999. The appropriate bubbles below
the respective question numbers in the ORS have to be darkened. For example, if the correct answer to question numbe_r X (say)
is 6092, then the correct darkening of bubbles will look like the following :
X.@e@@
(DG)G)G)
®®®·
®®®®
@@)@@
®@®®
~~~~
@@®®
®@e®
1. Two plane mirrors A and B are aligned parallel to distance, in cm, from the concave mirror should a flat
each other, as shown in the figure. A light ray is mirror be placed for the rays to converge again at the
incident at an angle 30° at a point just inside one end of point S having been reflected from the concave mirror
A . The plane of incidence coincides with the plane of and then from the flat one? Will the position of the
the figure. Find the maximum number of times the ray point where the rays meet change if they are first
undergoes reflections (including the first one) before it reflected from the flat mirror? The radius of the concave
emerges out. mirror is 80 cm .
,...__ _ 2-$ m---..
B 5. A concave mirror forms on a screen a real image of
f
0.2m
thrice the linear dimensions of the object. Object and
screen are moved until the image is twice the size of the
object. If the shift of the object is 6 cm, find the shift of
i the screen and the focal length of the mirror (both in
A cm).
2. Where should an object be placed, in cm, in front of a 6. A fish is rising up vertically inside a pond with velocity
concave mirror of focal length 30 cm so the i.Irtage size
4 cms-1 and notices ~ bird, which is diving vertically
is 5 times the object size?
downward and its velodty appears to be 16 cms-1 (to
3. A rod of length 20 cm is placed along the optical axis the fish). What is the actual velocity of the diving bird,
of a concave mirror of focal length 30 cm . One end of in cms-1 , if refractive index of water is· 4/3 .
the rod is at the centre of curvature and the other end
lies between F and C . Fin~ the magnitude of the linear 7. A portion of a straight glass rod of diameter 4 cm and
magnification of the rod refractive index 1.5 is bent into an arc of radius R cm
and a parallel beam of light is incident on it as shown in
4. A point source of light S is placed on the major optical figure. Find the smallest R, in cm, which permits all the
axis of concave mirror at a distance of 60 cm . At what light to pass around the arc.

= ====================================
1.160

www.puucho.com
Rahul Sardana with www.puucho.com

Ray Optics

13. A thin· converging lens of focal length f := 1.5 m is


placed along y-axis such that its optical centre coincides
with the origin. A small light source S is placed at
R
(-2; 0.1) m . 1; plane mirror.inclined' at an angle 0,
(where tan 0 = 0.3 ) is placed as shown in figure, such
8. A man of height 2 m is standing on level road where that y co-ordinate of final image is 0.3 m . Find the
because of temperature variation the refractive index of distance d , in metre. Also find the x co-ordinate of
air is varying as µ =-~1 + ay , where y is height. from final image, in metre.
y
road. If a =2xl0.. m-1 • Then find the maximum
distance, in km, till which he can see on the road. •'' '''
''
s '' '
9. A ray of light falls on a glass sphere of refractive index • : ,f
'0
./3 such that the directions of the incident ray and
emergent ray. when produced meet the surface at the --0-,+.....;--~------~f", ----x
same point on the surface. Draw the ray diagram and ,·,
find the value of angle of incidence, in degree. ''
''
----- d _ _ _ _.,''
10. A spherical ball of transparent material has index of
refraction µ : A narrow beam of light AB is aimed as
shown. What must the index· of refraction be in. order 14. An object is placed 12 cm to the left of a diverging lens
that the light is focussed at the point C on the opposite of focal length --<i cm . A converging lens with a focal
end of the diameter from where the light entered? length of 12 cm is placed at a distance d to the right of
Given that x <<i R . the diverging lens. Find the distance d , in cm , that
corresponds to a final image at infinity.
A
fX 15. Determine the position of the image, in cm, produced
I----+--....---""-IC by an optical system consisting of a concave mirror with
a focal length of 10 cm and a convergent lens with a
R focal length of 20 cm . The distance from the mirror to
the lens is 30 cm and from the lens .to the object 40 is
cm. Consider only hvo steps.
11. The pe:rpendh;ular faces of a right isosceles prism are
coated with silver. The rays incident at an arbitrary 16. The figure shows an arrangement of an equii:onvex lens
angle on the hypotenuse face emerge from the prism of focal length 20 cm and a concave mirror of radius of
after suffering a deviation of@) degree. Find x . curvature 80 cm . A point object O is placed on the
s, ~0:L principal axis at a distance 40 cm from the lens Such
12. A , transparent solid
sphere of radius 2 cm
· and density p floats in
·-----------r that the final image is also formed at the position of the
object. Find the distance d, in cm . Also draw the ray
· 10tcm
diagram.·
a transparent liquid of
density 2p kept in a
----- ,
------
beaker. The bottom of
the beaker is spherical
in shape with radius of
curvature 8 cm and is
silvered to make it
concave mirror as
I I, I,

1+-----100 cm----+<
1-4------ 40 cm ---+1+- 30 cm-+1
shown in the figure. When an object is placed at a
distance of 10 cm directly above the centre of the 1+----aocm----+1
sphere its final 1ima.ge coincides with it. F~d h (as
•t-.. /\ 17. A converging beam of rays is incident on a diverging
shown in figure)fthe height of the liquid surface in the lens. After passing through the lens the rays intersect at
beaker, in cm, from the apex of the bottom. Consider a point 15 cm from the lens. If the lens is removed, the
the paraxial rays only. The refractive index of the point where the rays meet, move 5 cm closer to the
sphere is ~ and that of the liquid is ± .
2 3

==================================== = 1.161

www.puucho.com
Rahul Sardana with www.puucho.com

Advanced JEE Physics Optics & Modern Physics

mounting that holds the lens. Find the focal length of 22. A lens with a focal length of 16 cm produces a sharp
the lens, in cm. image of an object in two positions which are 60 cm
apart. Find the distance, in cm, from the object to the
18. A lens with a focal length of f =30 cm produces on a screen.
screen a sharp image of an object that is at a distance of _
a = 40 cm from the lens. A plane-parallel glass plate 23. An intense beam parallel to the principal axis is incident
having µ = 1.8 and a thickness of d = 9 cm is placed on a convex lens. Multiple extra images F1 , F2 , . . . are
between the lens and the object perpendicular to the formed due to feeble internal reflections, called flare
optical axis of the lens. Through what distance, in cm, spots as shown in the figure. The rardii of curvature of
should the screen be shifted for the image of the object the lens are 30 cm and 60 cm and the refractive index
to remain distinct? is 1.5. Find the position of the first flare spot, in cm.

19. The height of a candle flame is 5 cm. A lens produces


an image of this flame 15 cm high on a screen. Without
touching the lens, the candle is moved over a distance Principal
of £"' 1.5 cm away from the lens and a sharp image of axis
the flame 10 cm high is obtained again after shifting
the screen. Calculate the focal length of the lens, in cm.

20. The focal length of a convex lens in air is 10 cm . Find


24. A parallel beam of rays is incident on a convergent lens
its focal length, in cm, in water. Given that µ 8 = ~ and with a focal length of 40 cm . Where should a divergent
2
lens with a focal length of 15 cm be placed for the
beam of rays to remain parallel after passing through
the two lenses. Give your answer in cm.

21. A converging beam of rays passes through a round 25. One side of radius of curvature R, =120 cm of a
aperture in a screen as shown in figure. The apex of the
convex lens of material of refractive index µ =1.5 and
beam A is at a distance of 15 cm from the screen.
How will the distance from the focus of the rays to the focal length f, =40 cm is silvered. It is placed on a
screen change, in cm~ if a convergent lens is inserted in horizontal surface with silvered surface in contact with
the aperture with a focal length of 30 cm ? Plot the path it. Another convex lens of focal length / 2 = 20 cm is
of the rays after the lens is~itted. fixed coaxial d = 10 cm above the first lens. A luminous
point object O on the axis gives rise to an image
coincident with it. Find its height, in cm, above the
upper lens.
,'''
------------,'' _i''______ _ A
--L--. 26. A source of light is located from a convergent lens of
''','' '' '' focal length f = 30 cm at a distance double the focal
length of the convergent lens. At what distance from the
r 1 5 c m -' lens should a flat mirror be. placed so that the rays
reflected from the mirror are parallel after passing
through the lens for the second time? Give your answer
incm.

= ====================================
1.162

www.puucho.com
Rahul Sardana with www.puucho.com

...

• ICE I ?: :;,," ,, BASED;ONREFLECTJON AT PLANE SURFACES j


J .1. 30° 9. 3d

2. 4cm 10. (a) 12 cmx8 cm

ucoso.(tann- tan8) (b) 12cmx4cm


3.
g
12. -5(1+-/a)l +5]
4. (a) 60°
13. a=20
(b) 240°CCW
14. 30°
s. 60°
15. 50°
6. 100° (CW)
16. 1 ("i+j+k
-- • ')
7. 20cm, 60cm, 80cm, 100cm and 140cm
-/3
8. 12
--------------·-·-- --- -- --------- -- -·· -~- -------------------'

• ICEII 0
BASED ON REFLECTIONA(CURVED SURFACES>\

1. 5 cm inverted 8. (a) Concave .

2~ Concave, 6.67 m 1
9. ( ../3 + )R from the convex mirror
2
4. 7.5 cm, 12.5 cm
11. 10cm
5. (a) 8cm
(b) 16cm

(c)_ 48cm

6. 15 cm
2+cosrot)
7. (a) ( 1+cosrot 1

(b) X =0

(c) m->ro

====================================1.163 =
www.puucho.com
Rahul Sardana with www.puucho.com

Optics & Modern P_hysics

BASED ON GENERAi! REF.RAC'FION

1. 75 cm 7. 6.6cm
2. 7.5-cm 8. 2.88m

.6.V=0.55 cm
1
3. I l"fl2 z1.1 9. -cm
m, .f3
4. 2 x 10• ms-• , 4000 A yellow _ 11. 0.7r

5. 3.5cm 12. sin-•(~, (~µ'R' - L' -,!R' ~ L' ))

6. !h+d
3

BASED ON ir0FAI! INiliERN.A REF.l!EC'TION (iTIR)


1. (a)_ 54.34° 8. (a) 40.54°
(b) Yes (b) 26.6°

i(A- -:) 9. 67.3°


4
10. - _cm
3. (a) 2,81 m 3
(b) 0:23,m
11. -J2
4. (b) -J2
12. ·OPi~
6. (a) 26.8°
(b) Yes
13.
2
h
7.
~µ'-f


1. (a) 30°' 7. sin-'(µsina).-a 0 •
BASED ON li!RISM

(b)
l 8.

9.
-J2
0'' .f3
2. A=<:t+~-6
11. 6v-6R =4.59,
.
µ=sinp 1+'( . _st". )+cot(a+P-6))'- 12. 22Q 56°
, smf}sm e1+.fi-6, . I

3. (a) 157.2° 13. 0'


14. 19°
(b) 128.4°
15. 10.1 °

16. 6"8<!.= 30.6° .•· 6vlalet_=33.4°,


5. (a) -./5
17. 2,.10
(b) 58.8°
6: (a) 34.2'
(b) 8.4°

= 1.164c::::=================================

www.puucho.com
Rahul Sardana with www.puucho.com

Ray Optics

BASED ON REFRACTION AT .CURVED SURFAC!=S I


1. 2.5 D 5. (a) 80 cm
2. Final image is formed at pole of the mirror (b) U<12cm
3. x z 0.75R 6. Final image is formed at 65 cm from first face on the same
.' 4.
side of the object.
7.42 cm
7. 3.33 cm, lnfi~ity

BASED. ON LENS FORMU!cfilj


1. 6 cm from either of the object 14. 7.5cm

. 2. 12 cm 17. (m + 1) times smaller

4 18. (a) Convex


'3.
3
19. 2.4cm
4. Concave mirror of focal length 15 cm 20. (a) 1.4
. 5. (a) 90cm 21. Rays will become parallel to the optic axis.
(b) 102cm
22. 1=3-2µ0
. 6. 12.5 cm in front of the silvered lens 2a
7. 10cm R
23.
8. 2.14 cm 2(µn+µ-1)

9. 1.37 24. (a) x,


. 10. 7.5cm
(b) ~
11.
4i' X1 -X 2
t
(c)
_x_,_
12. 2m, 1 m X1 -X 2

13. (a)
t
t-i
(~ No Shift

• ANSWERS SINGLE CORRECT CHOICE TYPE QUESTIONS.I

1. C 2. C 3. A 4. D 5. B
6. C 7. C 8. D 9. D 10. C
11. B 12. B 13. A 14. C 15. B
16. B 17. B 18. D 19. C 20. C
21. C 22. D 23. D 24. D 25. B
26. A 27. B 28. C 29. D 30. A
--- - - ------- ------ - . - --- -
, 31. B 32. C 33. D 34. D 35. B
L__ - - - ---- - - - -
36. B 37. C 38. B 39. A 40. B
----------------
41. C 42. D 43. B 44. D 45. C
------ --- - -- -- -
46. C 47. B 48. A 49. C 50. A
51. A 52. C 53. D 54. C 55. D ..
I

56. B 57. B 58. A 59. B 60. D

c:=================================== 1.165=
www.puucho.com
Rahul Sardana with www.puucho.com

Advanced JEE Physics Optics & Modern Physics

61. A 62. B 63. C 64. D 65. A ·I


66. D 67. C 68. A 69. B 70. C
71. C 72. C 73. A 74. A 75. B
76. C n. A 78. C 79. A, 80. C
81. C 82. C 83. D 84. A 85. B
J
86. B 87. B 88. A 89. C 90. C
91. D 92. A 93. C 94. C 95. A
96. C 97. B 98. B 99. C 100. D
101. C 102. C 103, B 104. B 105. C
106. C 107. C 108. B ·109. D 110. B
111. B 112. A 113. D 114. C 115. C,
116. C 117. B 118. D 1.19. C 120. C
121. C 122. B 123. C 124. B 125. A
126. A 127. C 128. B 129. D 130. C
131. C 132. D 133. B 134. C 135. A
136. C 137. C 138. D 139. C 140. C
141. B 142. A 143. D 144. C 145. C
146. A 147. D 148. D 149. A 150. C
151. B 152,. C 153. C 154. C 155. D
156. ·A 157. C 158. D 159. D 160. B

I 161. c· 162. D 163. D 164. C 165. B


166. C 167. A 168. B 169. D 170. A
171.
-116.
A 172. D 173. D 174. C 175. A
J
D 1n. C 178. B 179. B 180. C
181. B 182. C 183. C 184. C 185. C
186. C 187. A 188. B 189. A 190. B
191. B 192: A 193. A 194. D 195. B
196. B 197. D ' 198. D 199. A 200. D
201. D 202. C ,203. D 204. C 205. C I
206. B 207. D 208. B 209. A 210. B
211. C 212. C 213. C 214. C 215. C
216. C 217. C 218. A 219. B 220. A
221. B 222. D 223. A 224. A 225. B
I
226. D 227. A 228. D 229. B 230. B
231. B 232. C 233, B 234. B 235. B ·1
236. C 237. D 238. D 239. D 240. B
241. B 242. C 243. A ·244: A 245. D
246. C 247. B 248. D. 249. B 250. B
251. C 252. r,D 253. A 254. D 255. B
256. C 257. B 258. D 259. A 260. A
261. A· 262. B 263. D 264. C 265. A
266. B 267. D 268. B 269. D 270. D
271. A 272. C· 273, C 274. A 275. D
276. B 2n. C 278. B 279. A 280. C
2111. C 282. A 283; A 284. A 285. D

=1.166

www.puucho.com
Rahul Sardana with www.puucho.com

Ray Optics
286. A 287. C 288. D 289. B 290. A
291. C 292. D 293. A 294. C 295. D
296. B 297. C · 298. D

lfNSWERS l~h:a ·iiiM~iJ:1i1PiilE[c0RREc;r,cHR1cE~Ti~E~au]~~10Nsl


1. BC 2. A 3. AD 4. AB 5. BD
6. AC 7. ACD 8. AC 9. BCD 10. A
11.
16.
BC
BD
12.
· 17.
AD
ACD
13.
18.
BC
AD
14.
19.
ABCD
BD
15.
20.
ABCD
AC
:=J
21. BCD 22. ABC 23. AB 24. BC 25. AD
26. AD 27. BC 28. BD 29. CD 30. AC
31. BD 32. ACD 33. AC 34. BC 35. B
36. B 37. AD 38. AD 39. AC 40. CD
41. AC .42. AC 43. C 44. AB 45. BC
46. ABC 47. BC 48. ABC 49. CD 50. BC
51. AC 52. BD 53. BC 54. ABC a

(ANSWERS I ljlEASQ.l;llNG!BASElil!QUESTl0NS$
1. D 2. B 3. A 4. B 5. A
6. C 7. D 8. B 9. C 10. D
11. A 12. A 13. D 14. D c·15. A
16. B 17. A 18. D 19. A 20. C
21. B 22. A 23. B 24. D 25. D
26. B 27. D 28. C 29. C 30. C

1. B 2. C 3. B 4. D 5. A
6. D 7. C 8. A 9. D 10. B
11. D 12. C 13. D 14. A 15. D
16. D 17. C 18. A 19. C 20. D
21. A 22. D 23. C 24. B 25. D
26. C 27. A 28. A 29. B 30. A
31. C 32. C 33. B 34. D 35. D
36. C 37. C 38. D 39. C 40. C
41. D 42. D 43. D 44. C 45. B
46. B 47. C 48. A 49. A 50. C
51. D 52. D 53. A 54. C 55. B
56. A 57. C 58. B 59. B 60. A
61. C 62. C 63. C 64. D 65. B
-1
66. C 67. B 68. A 69. D 70. A

1.167=

www.puucho.com
Rahul Sardana with www.puucho.com

Advanced JEE Physics Optics & Modern Physics

71. C 72. B 73. A 74. C


-------- "]
- --- _ 75._D - - - - - ·
76. A 77. B 78. C 79. D 80. B
81. C
' - - - - - ' - - - - - - - - - - - - - - - - - - ---··---·-··

I-
I
l. ~: ~~-~- r, s, I)
C-> (q: r, s, t)
--- 2.
C _, (s, I)
: : ~:: :\ - - --- -- 3 ' ·
C-> (q, r, s)
~== ~~~
C-> (p)
q, r) -- ---·
C-> (s)
~ ~==-~~)-· 5
· ~:% q) -

D-> (p, r, s) . D-> (t) D-> (p) D-> (p, q) •


' - - - - - , . - - - - - ~ _ . , D ->_(g,J) - - - - - ·-·------
6. A-> (p, s) 7. A-> (cj, r) 8. A-> (p, q, s) 9. A-> (p, s) 10. A-> (p, q, r, s)
B-> (q) B-> (r) B-> (p, q) B-> (p, q, r, s) B-> (q)
C -,· (p, q, s) C-> (p, r, s) C-> (r) C-> (p, q, r, s) C-> (p, q, r, s)
r---',-D +(r). _______ D~(p, r) _________ ., D-> (p, q, s~)_ _ _ _ _D_-> (p, s~)_ _ _ _ _ _D_->~(p~,~q,~r~,s)
1 11. A->(P) 12. A->(p,s). 13. A->(S) 14c A->(S) 15. A->(q) -·]
! B-> (p) B-> (p, q, r, s) B ->.(p) B-> (q) B-> (r)
! C-> (r, s) C-> (p, q, r, s) C-> (q) C-> (p) C-> (s)
~i'--~D~->~(g,_p) D -'>_(g,_s) D-> (r) . .P ->_(g)_ D ±(R).-'-

•' '"~IN'TEGER ANSW~R' TY~E,QUESTION$'·


0

I 1. 30.' . . -. 24, 36
2. 3. 3 4. 90 _____ , _____ _J

5. 36 6. 9 7. 12 8. 2
I9,-------------------
60 10. 2 11. 180
- 5- - -
- - 15.
12. 15
13. 5, 4 14. 8 16. 30
111. 30 · 18. 60 19_._9_ _ _ _ _ _ _ _ _2_0._:..~? _______7__,·
21. 5 22. 100 23. 12 24. 25
! 25. 10 26. 45
--------

1.168 ======================================
www.puucho.com
Rahul Sardana with www.puucho.com

1. The angle between the incident ray and the reflected ray is
180-20, so, we have =>
i+.,/3]
-2- ' i-.JJf
-2-
:::::) X=4cm
:' So point of incidence of light from A should be at 4 cm from D
' on mirror.
''' 3. The image will be momentarily at rest when. the particle moves
'' parallel to the mirror. Let at the time t the particle has a velocity
e'e
v parallel to the mirror.
\180~-28
''
' ''.
V

( i+/31)
2
(i--.13])
2
cos(18O°-20) = •
1i+;13j11i-;311
(1-3) ucosa
=> -cos(20) = - 4-
1
vsin0=usina-gt ... (1)
and vcos0 =ucosa
=> -cos(20) = _ _! ucosa
2 :::::,. V=-- •.• (2)
COS0
1 From (1) and (2)
=> cos(20) =
2
:::::) 20=60° ( ucosa)sin0 = usina-gt
case .
:::::) 0=30° = ucosa(tana-tanB)
=> 1
g
2. Drawing the ray diagram and using the Law of Reflection, we get
B
4.

20cm

A
.semi i
'

i=r ... (1)


:::::,. sini =sinr From the figure, we observe 'that
So we can say that ~DO and .6.OBC are similar 30=180°

====================================1.169 =
www.puucho.com
Rahul Sardana with www.puucho.com

Advanced JEE Physics Optics & Modern Physics

=> 0=60° ,,
-..--------"
3" ,,
I \ I \
So, S, =180°-2(30°)=120° (CCW) I ),,. - - - ... ../ \
I ,, \ /' \
ands, =180°-2(30°)=120° (CCW) I 2",f \ I \,4" \
I 1 \ I \ \
I I \';). \ \
So, total deviation 6 =61 + 62 ~---+---7 ----~---~
I \ \ 1" I \ I / \
=> S = 240° (CCW) I \ \ ' ' 1
5" I \
I }.----.._ • \_.--1'-_--...f \
Alternatively from the figure, we observe that I "\ 1',, ',0., ,,"1' I', \
S = 180° + 8 = 240° (CCW) or 120° (CW) I
I 2.' \\ ...... _-..!_..- \.
I~ , I
I \2' \ \
6Q0 1\
1 ,•
I I \J V \ \
-----1---)
5. Various angles made are as shown in figure. In triangle ABC,
/___ Tr----
\
\
t
~
Bi\
I \,: \
I \ J
,-r•.-,,-::r,,c
r

I,
:, I
we observe that , r,5 1 r3 1
0+0+0=180° \ 3 ,, I
1
\. I' 5, /
, ,
\ I' I
\ 'f-. ... _._.--{ )-. ... _. __ -( I
=> 0=60° ' ,, J
1
4 ' .} _______
_______ 4 ' ' \/ ,, 1
,, 1

So, the number of images formed by the combination is given by


N=6-1=-5

Combination of Mirrors Images Formed


AB & BC 1,2,3,4,5
AC & BC 1', 2', 3', 4', 5'
AB & AC 1", 2", 3", 4", 5"
6. From the figure, we observe that
. S, =180°-2(50°)=100° (CCW) These images along with the obJect must lie on a circle as
shown in figure with an angular separation of
s, =180°-2(20°)=140° (CW)
360° = 360° = 720
s, =180°-2(10°)=160° (CW) N 5
Similarly, the other two combination of mirrors also form 5
images each but we find from symmetry that 5 and 5' , 1 and
5" , 1 and 1" coincide. So the total number of images formed by
three mirrors AB , BC and AC is
N'=(5)(3)-3=12

• M, 9. The ray diagram is shown in figure. We observe that


Hl=AB=d
So, total deviation S = 100°(CCW) + 140°(CW)+ 160°(CW) DS=CD=_cl_
=> S = 100° (CW) or 260° (CCW) 2
G
7. The image is formed as far behind the mirror as the object is in
front of it. Also in:iage formed by mirror 1 i.e., 11 , acts as object
for mirror 2, so 1; is formed 50 cm behind the mirror 2 as
H
shown.

I", I', I, 0 I, I' I"


_________ .., __________ I ''
20 60 10 10 30 30 20
' 60
' B :E
'
F
2 J
Taking all distances to be in cm and plotting them as shown (but Also, AH = 2AD
not to scale), we get
011 = 20 cm 012 = 60 cm => GH=2CD=2(%)=d

01; = BO cm 01; = BO cm Similarly, IJ =d


=> GJ=GH+HI+IJ=d+d+d=3d
O1;'=100cm O1;=140cm
So, the respective distances are 20 cm , 60 cm , 80 cm , 10. (a) For a one eyed man, the required size will be half the each
100 cm and 140 cm dimension of the face i.e., 12 cm x 8 cm
(b) For a two eyed man, the
8. Let us first consider the mirrors AB and BC , for which we have Smallest length of the mirror = Half the length of face
360° = 6 => (Smallest l~ngth) = _!_ x 24 = 12 cm
60° ' of-the Mirror 2

= 1.11oc=:===================================
www.puucho.com
Rahul Sardana with www.puucho.com

Ray Optics
The smallest breadth of the mirror is calculated by using the fact
=> v,=-5(1+.JsH+s]
that the rays from extreme part of face should reach one of the
eyes after reflection from the mirror. The common overlapping
portion is then the required breadth of the mirror. The ray 13. Let AB be the incident ray and angle of incidence at the mirror
diagram is shown in figure. M1 be i, then
p ----------------- ..."""P'
M ,...,,.......... -- :'
-- ''
A

''
''
' '-., I
'''
----------------- '.:JQ'
From figure, we get
MM'= .!.pa __!_E,E
2 2 '
Smallest Breadth)= _!_( 16 _ 8) = 4 cm
(of the Mirror 2 LCBO = go -i 0

So, the shortest size of mirror is 12 cm by 4 cm. => LBCO = 180° -0 -(goo -il
=> LBCO = go -0+i
0

11. Let us first find the image of point 8 in mirror bd (shown in Using the Laws of Reflection, we get
figure). Let us then construct image B1 in mirror cd. Also, 8 3 is LDCB=20-2i
=> LCDB=180°-20
the image of B2 in mirror ac and B4 is the image of 8 3 in
~ cr.=20
mirror ab . Let us connect points A and 8 4 • Point C is the The angle between incident and emergent ray is 28 and it is
point of intersection of ab with line AB 4 • Let us now draw line independent of the angle of incidence i .
B 3 C from 8 3 and connect point D at which this line intersects
14. Suppose that a plane mirror is kept horizontal as shown in
ac with 8 2 , E with 8 1 and F with 8 . figure. The reflected ray will make an angle of 30° with
horizontal, or an angle of 60° with the vertical.
B4,,
:' ', ............ '''
: ............. Incident ray : Reflected ray
! . . . . . . . . ...
lL-"-----------~~',C
I
I
I
I
q_... . As b'

.......
,.,.,.
..,.,,
,,... -----
'
,,/'
---·B
,.,.1 1

F
I
J
~i/
, __ - 0 ··- -- -

To make the reflected ray to go vertically upwards, the mirror is


: _,,.... C - --E',-d--l required to be rotated about O counterclockwise by 60°. To
I _,," I .., 1
~ B2
1
83 l,-'_,, - - - - - - - - - - - - - - - - - - - - - - - - - - - -- ... achieve this, therefore, the plane mirror is required to rotate
It can be stated that the line ACDEFB is the sought path of the about O by half the angle, i.e., by 30° , as shown in figure.
beam. Further, we observe that since, B3 CB4 is an isosceles Reflected ray
triangle, CD is the reflection of beam AC . Incident ray
Similarly, we can show that DE is the reflection of CD and so
on. This solution of the problem is not unique, as the beam will
not necessarily always be sent initially to mirror ab .

12. Along x-direction i.e., perpendicular to the mirror, we have


Relative Velocity of)= -(Relative Velocity of) 30'
( Image w.r.t. mirror Object w.r.t. mirror
=> V1-Vm=-(vo-vm) 15. Ray AB is incident on mirror OP at an angle e. The reflected
ray BC is incident on second mirror OQ. Finally, the reflected
=> v, -(-5cos30°)=-(10cos60°-(-5cos30°))
ray CD is parallel to OP. Since CD and OP are parallel, and
=> v, =-5(1+./3) ms-' CO cuts them,
In the direction parallel to the surface of mirror, i.e., along LQCD = LCOP = 70°
y-direction we have
=> LDCN = gQ 0 - LQCD = go 0 - 70° = 20'
V1 =Vo
=> LNCB=LDCN=20°
::::) v, = 10sin(60°) = 5 ms-1
So, velocity of the image
v,=(v,),i+(v,),]

====================================1.171 =
www.puucho.com
Rahul Sardana with www.puucho.com

Advanced JEE Physics Optics & Modern Physics

Q 16. Reflection of a ray of light is just like an elastic collision of a ball


with· a horizontal ground. The component of the incident ray
along the inside normal gets reversed while the component
perpendicular to it remain!=i unchanged. So, the component of
A incident ray vector A= i +} -k parallel to normal, i.e., i + J
gets reversed while perpendicular to it, i.e., -k remains
unchanged. So, the reflected ray is written as,
R=-1-]-k
>l.!::::3.:::==::::';;:== p A unit vector along the reflected ray will be,
B
• Fi -1-]-k
Further, LOCB = 90° - LNCB = 90° - 20° = 70° ' =Fi =_Ja__,a_
Now, in .6.COB , we ~ave
LCBO = 180°-(LCOB+LOCB) = 180'-(70'+ 70°) = 40° => f=- ~(i+j+i<)
=> 0 = LNBC = 90' - LCBO = 90° - 40'= 50°

= ====================================
1.172

www.puucho.com
Rahul Sardana with www.puucho.com

. f
1. S1nce, m=-- ~ ---=-
f-u V 50 -25
Now, f=-f and U=-1.5f,so :::::,. V=-50cm
-f -f
m=--- m=-- So, m=-~=-1
-f+1.5f 0.5 f u
:::::i- m=-2
4. Given, f = -1 O cm . Since a concave mirror can fomi real as well
Since m=~=-2 as virtual image and since the nature of image is not given in the
h,
question. So we will consider two possible cases.
:::::,. h; =-2h0 =-5 cm Case 1 (when Image is real) :
The image is 5 cm long. The minus sign shows that it is So, m = --4
inverted. . f
Smce m=--
f-u
2. Given : u = -25 m , m = +4 (since the image is erect).
Now, the magnification is given by => .-4=_-=!Q_
-10-u
V
m=-- => u=-12.5cm
u
Please note that here,
=> v=-mu=-(+4)x(-2.5)=10 m
lul > !ti and we know that in case of a concave mirror,
Using the mirror formula, we get
image is real when object lies beyond F .
1 1 1 Case 2 (When image is virtual) :
-+--=-
10 -2.5 f So, m=+4
f1 = 0.1-0.4 = -0.3 Since m=--
1
1-u
1
I= ---=-_1_()_ m => 4=_-=!Q_
0.3 3 -10-u
Since f is negative so, the mirror is concave. =:> U=-7.5cm
The radius of curvature of the mirror is given by
Please, note that here, lul <!ti, as we know that image is
R=2f=2 - · ( 310) =- 20 m=-6.67m
3
virtual when the object lies between F and P .

5. Since f=·~=-12 cm
3. According to the mirror formula, we have .!.+.!.=!
V U f
Let the object be placed at a distance u from the pole.
M,
Since, we know that magnification m is given by
f
m=--

"'~"':~~4==::~-~=='j,: 1::: f-u


So, now we apply this formula to these situations one by one.
a) Here, we have m = +3
optic-;;;;~1;:,,~-,\ -------------~r-- !oscm => 3=~
-12-u
~ ! => --4=-12-u
M, => 12+U=4
i-.-------- 50 cm -------Joi => u=-8cm

====================================1.173 =
www.puucho.com
Rahul Sardana with www.puucho.com

Advanced JEE Physics Optics & Modern Physics

b) Here, m = -3 The following steps of construction for drawing the ray


diagrams are used.
=> -3 = ~ (i) From I or O drop a perpendicular on principal axis,
-12-u
=> 4=-12-u => U:=c-16 cm such that CJ= CD or OC =CD.
1 (ii) Draw a line joining D and O or D and I so that it.
c) Here, m=-
3 meets the principal axis at P . The point P will be
the pole of the mirror as a ray reflected from the pole
is always symmetrical about principal axis.
(iii) From O draw a line parallel to principal axis towards
=> 36=-12-u => LI=-4Bcm
the mirror so that it meets the mirror at M . Join M to
I , so that it intersects the principal axis at F . F is
6. Distance of image formed by the plane mirror is (b-a) i.e., the focus of the mirror as any ray parallel to principal
(b - 5) cm and distance of object from mirror is (b+a) i.e., axis after reflection from the mirror intersects the
principal axis at the focus.
(b + 5) cm . Using mirror formula, ..!. + ..!. =! we get
V U f
9. Let the point A be at a distance x from the convex mirror as
1 1 shown in mirror, then assuming the origin to be placed at the
(b-5)-(b+S) = 20 pole of convex mirror, we get
Solving this equation, we get
b=15cm 1

,r'- :.-:;,
The coincidence of the images can be established by observing 2

~
• Incident Incident ., ,
the changes in the relative position of the images when the eye
Ray Ray ,
is moved away from the optical axis of the mirror.
When the images are at various distance from the eye the A '

l
images will be displaced with respect to each other.
When the images are at the same distance, they will coincide [ : t "4----X---+-: ,
irrespective of the placement of the eye. : :
, , ----2R---~I
I

7. (a) At any instant t, .we have ''


i+-'-1,_-x_.......v,-t,,t'
'
u = -(2f + x) = -(2f + f cos rot) ''
1-4---
'
2R ------+1' .'
.
Usmg,t hem1rror
. I -+-=-,we
formua, 1 1 1 get For convex mirror, we have
V U f
1 1 2
-1
v 2f+fcoscot
::::, V=-(2+coscot)t
1+coscot
For concave mirror, we have
i.e., distance of image from mirror at time any instant t is
1 1 2
2 + cos oot)t R
( 1+coscot -(2R- x) -( 2R+ ~ )
2x+R
(b) Ball coincides with its image at centre of curvature, i.e., at
Solving this equation, we get
X=O
(c) At t =
T
2
, we have x=(1\F3)R and X=-( v'3/ 1)R
Ignoring the negative value, as we have already used a negative
rot= re
sign with x, so the object should be placed at a distance
=> x = fcos(x) = -f
So, u = -f i.e., ball is at focus. So, its image is formed at x = ( Js +
1)R from the convex mirror.
2
infinity, so m -), co

10. Object is placed beyond C. Hence, the image will be real and it
8. (a) Since the image is on the opposite .side of the principal
axis, the mirror is concave. Because convex mirror·always will lie between C and F. Further u , v and f all are negative,
forms a virtual and erect image. hence the mirror formula becomes
(b) The ray diagrams for two different cases are shown in 1 1 1
figure. -v-u=-1
D, 0
,-r 'M 1 1 1 u-f D' A'

-- --
-=---=-- -c---,--r---F~---mp
M,'
0 ... -- '
1 V f U Uf
'
'' f B'
A p,' B A B V=--f • E'
C'
'' :P 1--
'' ,-' I '' u ...,....__ VMJ -----+I
' ''t---- --- ' Now since, uAs > uEo 14----Vm----+l
Case 1
D
Case2 ''
= 1.174

www.puucho.com
Rahul Sardana with www.puucho.com

Ray Optics

/·: m=-t) => V=-


51
2
Therefore, shape of the image will be as shown in figure. Length of Image
So, magnification, m
Also note that v AB < uAB and .VEo < u~0 , lmABI < 1 and fmE 0 ] < 1 Length of Object
_51_(-21) 3'
11. Since the image is inverted, so the hlirror is concave. => m VA -Ve ~2;__ _
Now, u =-30 cm UA - Uc _ 5f ~(-21) 2
1 V 3
m=-2=-u Case JI : When the other end lies beyond C

=> v=~=-15 cm For A, we have


2
. 1 1· 1
Smee-+-=-
U=-(2t+¾)=-~f
v u f f =-f

=>
=>
1
(-15) + (-30)
f=-10 cm
1
=, 1
Since .:!.+.:!.=!,soweget
V U f

-+--=-
1 1 1
v (-i) -f
12. Since image touches the rod, the rod ·must be placed with one
end at centre of curvature. However, two cases arise here.
Case I : When the other end lies between C and F 71
=> V=--
For A, we have 4

U=-(2t-¾)=-~ So, magnification, m = vA -vc


UA -Uc

f=-f -~-(-21)
4 3
Since, .!.+.!=!,so we get => m
V U f F p _ 71 -(-21) =-4
A' 3
1 1 1
-+----
v (-:')- (-0

.:::=================================== 1.175=

www.puucho.com
Rahul Sardana with www.puucho.com

1. d"=-d-= 100 =75 cm · Magnification, m2 = ...:.'!...;,,. 4).55_


nm,attve 4/3 .. - u .· J
1 So, Liv;,, 0.55 cm

2. The incident rays will pass undeviclted through_ the water surface
m2 i::l1.1
m1 .
and strike the mirror parallel -to its principal axis. Therefore for
the mirror, object is at oo . Its image A (in figure) will be formed
at focus which is ·20 cm from tl]e mirror. Now for·the interlace 4• The refractive index of glass,
V C
between water and air, d =10 cm . µ·=~=..;_
vglass v·

:::) v=E.= 3x10a =2x10a nis-1


µ 1~ .
Air Since "the frequency of light rerriains the same when it passes
from one medium to another, so we have
10 crh water
4/3 C=fAo and V=fA
'' 30cm .
µ=v~T
c. A. 0
' S ince,

-...-:~
''
R =40 cm
l A= i.., = 6000 =4000A
µ 1.5
The colour remains yellow, as the colour depends on the
d' d 10 ·7 5 frequency and not on the wavelength.
=(~:)=(4;3f . cm
5. l)sing equation, the total apparent shift is

3. Case I : When No Slab Is Inserted AX =h,(1- ~,)+h,(1- µ:)


According-to mirror formula, we have

4J3)+s(1- 3J2)=1.5cm
1 1 1 => - Ax=2(1..:
-+-=-
v u I
1 1 1 Thus, h=h,+h2 -filC=2+3-1.5=3.5 cm
-+-=-
V1 -30 -10
6. When we consider only two st8ps, then the ray of light starting
~ V1 =-15cm
from object O first gets refracted and then reflected. Distance of
.. . V 1 image 11 formed after refraction from the plane surface is given
Magm11cat1on, m1=--=--,-=-0.5
u 2
by O•
Case II : When Slab Is Inserted f
X=nh+d=.±h+d
Shitt=(1-¾}=(1- 5 )6=2 cm ,1 3

Again, applying the mirror formula, we get


1 1 1
Therefore, distance of image 12
formed by plane mirror will be t
~
v, -(30-2)
v 2 =-15.55cm
-10 .±h+d
3
I
= 1.176

www.puucho.com
Rahul Sardana with www.puucho.com

Ray Optics

7
_ _ = sin(60°) ·
18
s1nr => AB=(2)(1)(~)= 1 cm
=> r =28.76" So, ·the distance between rays 1 and 2 is given by
BE=ABsin(30°)= ~ cm
P N.

I'
' 10. Total deviation suffered by the ray is given by

' ()Total =Sp + C\:i


''' => a=0-r)+0-r)
r'
' => .
1-r=-
2
" ... (1)
'-- --'
0 Mirror Further, in ~OPQ , we have
Since, MP= POtanr r+r+P=180°
=> MP= (6)tan(28.76°)
=> r=90°-.!l. ..• (2)
=> MP=3.3cm 2
So, MN =2MP =6.6 cm /

8. From Snell's Law, we have


i=
sin(45°)
3 sinr

=> sinr=¾sin(45°)

=> r=32°

A
From equation (1),.we get

i=r+i=90°+(a;P) .•. (3)


According to Snell's Law, we have
sini
µ=-.-
smr.

sin[90°+(";~)] cos(P;")
=> µ
sin( 90"-%) cos(%)

o+----£--->t
F
=> cos( p; a)= µcos~
Since, EF =ECtanr
=> EF=(3)tan32°=1.88 m . . r 1
Length of shadow at the bottom of the lake is
11. sm11= 2r =2.
£=DF=DE+EF=2.88 m ::::) i1 =30°

· sin(60°)
9. From Snell's Law, we have ,l3 =-·-.-
smr
''
=:i- r=30° ''
1 '
E ''
''
I 2 ' ' \ ~,.
60°160° ' ,
30° D
C
'''
A•

r
1 cm
:r
I
''
''
Applying Snell's _Law at P , we get
3 sini1
' 2= sini2
l r'r

C
=> i2 =19.5°
Now, Applying Sine Law (Lami's Theorem), on ACPR , we get
2r CR
Since; AB= 2(AD) =2(DCtanr) {·.- LPCR = 60°}
sin(180° -60° -19.5°) sin(19.5")
=> CR=0.7r

==================================== = 1.177

www.puucho.com
Rahul Sardana with www.puucho.com

Advanced JEE Physics Optics & Modern Physics

Let LAOM=0
Then by symmetry, LAIB= e
12. Since, sin i =~

=> i = sin-
1
(~)
V
Eye
According to Snell's Law applied at A , we have
sini
µ= sinr
1
B
__,. . _,(sini)
r1 =sin - . -'(- L)
=sm
µ µR Now from geometry of figure, we have
LA0!=2LIBA
=> LIBA = i:(90° - 0)

Lr= LIBA = 90'- 9 ... (1)


2
Also Li=goo-0+0=9O0+0 •.• (2)
Deviation suffered by the ray is 2 2
S= i-r =sin- (ij)-sin- (µLR)
1
1 1 Now, Snell's Law gives
. (90°+0)
sini sin - 2 -
This is also the angle r2 , so we have
µ= sinr
s. , n - - (90°-0)
. _,(L)
r2 =sin . '( µRL)
R ~sm-
2

From the knowledge of inverse trigonometry, we have


cos-+sin-
e2 2
0}' =µ'
sin-'(C)-sin-'(D) = sin-,(cJ1-D' -0J1-C') ( 0 . 0
cos--sm-
2 2

r2 =sin-,[!:_ Ct3 __!:__


Af-µ2Fr µRf-f\2 Ci'] 1+sin0
1-sin0=µ
2

'-1
Now, again applying Snell's Law at B , we get sin0=~
µ +1
sin0
µ=-.- Since, sin a=~
smr2
2a
8 = sin-1 (µsinr2 ) 1
=> d =2a(µ:- )
µ +1
0=sin-'(µLJ1- L' _!:_J1- L')
R µ 2 R2 R R2

1 2 2 2 2 2
=> 0 = sin- ( ~2 ~µ R -L - ~ 2 ../R -L )

13. In the figure, let AB be the disc and O be the centre of the
bowl.

= ============================-=======
1.178

www.puucho.com
Rahul Sardana with www.puucho.com

1. (a) Crltical angle between 2 and 3, is given by


1+--R--+1
sinC=~ l+X+I
1.8
Now, applying Snell's Law, we get

1.6sinB = 1.8sinC = (1.8)(-1..2) =1.3


1.8

(b)
8 =Sm
13
. -'(ffi ) .o::: 54 •34°
If 9 is decreased, the angle of incidence at the interface i•
i
20cm

'''
l
between 2 and· 3 gets decreased or i < C, so the light will µ=½
refract into medium 3.
''
2. The path of ray is curved as shown in figure. As it travels
successively into denser layers, it bends away from normal and So, the radius of shadow is R = ( 15 + B~O) cm
TIR takes place at depth where angle of incidence approaches
845
.:: ~ R= cm=2.81m
2 3
(b) For shadow to be formed, angle of incidence must be less
''
6001'
than critical angle.
Using Snell's Law, we get
,p

___ _!____ I\,.,


±[ a=.
3 ~a!ax + 202
J
= 1sin(90°)

a => 16a!ax = 9a!ax + 9 (202 )


Slab => 7a~~ = 9(20')

Applying Snell's Law at interfaces P and Q , we get => a= = ( ~)<20 cm) = 0.23 m

1sin(60°) = µ 8 sin(i) 4. (a) At interlace AB , applying Snell's Law. we get

=> ~ = (A-Bt,,,..) 1sin(i) = µsin a

t
max
=.!_(A-
B
,/3)
2
.
stna=-
1
µ
... (1)

At interface BC , applying Snell's Law again, we get


3. (a) Using Snell"s Law, we get µsin(90- a)= 1(sin0)
µsini = sinr :::::, sin0 = µcos a ..• (2)
From equation (1) and (2), we get sine= cola
¾(,'15;~20' )=1(,/x':200') (b) For emergence from BC , we must have
Solving, we get 90-as:C
800 At grazing incidence we have
x=-cm
3

:::::=================================== 1.179=
www.puucho.com
Rahul Sardana with www.puucho.com

Advanced JEE Physics Optics & Modern Physics

a=C 6
~ 90-u~C A B
:::::, 2c;;::so 0 ',a n-2C
:::::, c~45° ''
'
=> sin-
1
(¾)~45°
=> _!>_1_
µ - J2. 0 C n/2
D '------~-__, C Deviation versus angle of Incidence graph when TI_R is
=> µ~,/2.
taking place
So, the greatest value of refractive index is
µ-=h CONCEPTUAL NOTE(S)
When ray is travelling from rarer to denser medium then-deviation
5. Case I: is given by
When angle of incidence (i) is less than critical angle C
15 = i - sin-~ (1µ 2 sini)
i.e., i<C_
6

1Lsfrf1(1~) ---------
2
Rarer medium (µ,)
Denser medium (µJ

0 l!.
2

Since, 8 =Deviation =r - i ---(1)


From Snell's Law, we get
6_ (a) Critical angle between 2 and 3
µ 1 sini = µ 2 sinr
sinC=g=~
:::::, r.= sin-
1
( ~:i) 1_4 7

Substituting in equation (1 ), we get


1_6
0 - .,(sini.)
u=Sln - , - -I-·
µ, 1.4
This is a non·linear function and graph is given below
6

;-,~-------------, Applying, Snell's Law, at P , we get


1_6sin0 =1.4sin(9O° -C) =1.4cosc
! . j
C 36
1_4J1-
sine 49 O.45
Deviation versus angle of incidence graph when TIA is not =>
1-6
taking place. ·
~ Si::::26.8°
Case II:
When the angle of incidence i is greater than the critical (b) As 8 is increased, i1 will increase or i2 will decrease or
angle C,i.e., i>C i2 < C and hence the light will refract in medium 3.
In this case TIR will take place as shown, so deviation is
6=n-2i ___ (2)

'
''
/
/ 0 Rarer medium
Denser mediu'm

This is a linear function and so the graph is given below 7. As shown in figure, the light from the source will not emerge out~
of water if i=C.

1:1so================================::::i
www.puucho.com
Rahul Sardana with www.puucho.com

·Ray Optics

!'!=tanC=_:i_
h . 4

i
h
,,
'
,i}c
h=±R=± cm
3 3
I+- R-+<
'
'

! s
i h
''
''
''
''
'
c,'
5 3
:c
Therefore, minimum radius R corresponds to the situation when
i=C l ''
s
C
4

11. At the maximum, the ray can enter the glass at the grazing
angle, so (Dmax = 90° .
According to Snajl's Law
µ 9 sinr, =µsini
C 2
sinr, =~sin(90°) = µ
2
1.5 3
~µ -1 sinC= 1µ 2
(r,J.~ = sin·' ( ; )
In llSAB,
Since r, + r2 =90° , so
~= tanc
2
R =htanC (r,)., =i-sin·'( ; )
=>
=> R=-h- Again, using Snell's Law, we get µ
9
= sin~ 3 oo)
~µ' -1 smr2

8. (a) Applying Snell's Law, we get


=>
1.3 sin 81 =(1) sin 05
13
=> sin 0, = (1.3)sin(30°) = ; = 0.65
=> ·i
=> 85 r::::40.54°
=> GJa,;s
(b) Applying Snell's Law, we get -- 3_0~-
1.3sin01 =1.45sin9 4
=> 1- 4µ2 =..!.
9 9
1.a(.!.)
2 4µ2 =~
=> sin0 4 = - --=0.49 => i,
1.45 9 9 Alcohol
Layer .I ''
::::> 84 ~ 26.6° µ = -./2
=>
Now, when the paper is dry then µ = 1
C=sin- (.!.) 1-) = 47.3°
~
1 1
9. Critical angle, = sin- (- 1
µ 1.36 r1 =sin- (~}·:42°
For TIR to take place at B , we have
Since r1 + r2 = 90°, so we get
P>C 8
r2 = 48°
=> p > 47.3°
For this to happen, we have
a <90°-47.3°
) Critical angle at glass air interface, is given by

C = sin·' (--) ~ 42°


1
=> a<42.7° 1.5
Applying Snell's Law at A , we get So, we observe that as r2 > C , so it can't be seen.
0 < sin-1 (µsin42.7°)
12. According to Snell's Law, we have
=> 0 < sin·' [1.36sin(42.7°)]
-./2 = sin(45°)
=> e < 67.3° sinr
So, the maximum val~e of 0 for TIR to take place at B is ~ r=30°
67.3°. The critical angle C is given by
1
C=sin- (¾)=45°

:::==================================== = 1.181

www.puucho.com
Rahul Sardana with www.puucho.com

Advanced JEE Physics Optics & Modern Physics

13. Incident angle i is least fqr ray AP and this angle should be
greater than the critical angle C
i.e., i>C
=> sini > sinC
R 1
=> - - > -
R+r µ
R 2
=> -->-
R+r 3
Applying, Sine Law (i.e., Snell's Law) in t.OPM , we get 3R > 2R+2r
OP OM => R>2r
sinC sin(90° + r) r 1
=> -<-
OP R R 2
=> {R = radius)
(~)=cosr A
2

=> OP=~
As we move away from O , angle PMO will increase.

Therefore, OP "f> ~ . Same is the case on left side of O .

= ..:::::===================================
1.182

www.puucho.com
Rahul Sardana with www.puucho.com

1". The ray diagram for the situation discussed is shown in figure.
A => µ=~

2. r,+r2 =A
Since i+e=A+c5
=> 6=a+p-A
Further applying Snell's Law at incident surface and emergent
surface, we get
µ= sina. and sinr2 =!
sinr1 sinf3 µ
sin a= sinf3
sinr1 sinr2
(a) From the figure, we observe that sina sinp
r1+r2 =r2 +r3 = 60° sin(A-r,) sinr2
~ r1 =ra sin(A-r,) sina
Applying Snell's Law at the faces AB and BC , we get sinr2 sinf3
sIn1 sin(30°) sinAcosr.; cosAsinr2 sina·
µ=-=---
sinr1 sinr3 sinr2 sinr2 sinp
Since r1 = r3 . sina
smAcotr2 =--+cosA
~ i=30° sinp
(b) Since. r1 + r2 =A= 60° . sin~ +cotA
sInf3smA
~ r1 =60°-r2 :=60-C (·: r, = C}
5
sini sin(30°) Sinceµ= !np =sinpcosecr2
Further, µ smr2
sinr1 sin(60° -C)
0.5 · µ =sin13:J1 + cot2 r~
µ sin(60°)cosC-cos(60°)sinC

Since sin C = !
.
µ=sInp 1+ ( sma
. - .
s1nf3smA
+cotA )'
µ
Since A=a+f3-c5
1
cosc=J1-
µ' .
µ=smp 1+
( .
sma
. (.
smpsm a+P,-<5
) +cot(a+p-6) )'
1
µ(..J3)J1 - 2 _ _! =0.5
2 µ 2
3. At minimum deviation, we have

( ~)~µ'-1 =1 r=t=30°
2

c==================================== 1.183=

www.puucho.com
Rahul Sardana with www.puucho.com

Advanced JEE Physics Optics & Modem Physics

A sini sin(45°)
µ sinr, sin(45°-C)
sin(45°) ·
µ sin(45°)cosC-cos(45°)sinC

Since sine=..! -
µ

=> cosC=~1- ; 2

1
(a) Applying Snail's Law at AB , we get => µ~1- -1=1 (sin45° = cos45°)
µ'
15=~ => µ' -1 =4
· sin(30°)
=> i =48.6° => µ=-15
• Since, 6roiai =Op + 60 + 6R (b} At minimum deviation, we,have

=> BT~, =0-r)+(1B0°-2r)+(i-r) r, =f2 =i=22,5°


=> ()Total =180°+2i-4r-
. r. sini
Since, v5 - .. ( )
=> °"roiat =157 .2o sin 22.5°
(b) Again applying Snell's ,Law for water-glass interface, we => i = 58:8°
get
6. (a) Applying Snell's law ai D , we·get
isini' = ¾sin(30°)

..~ ~, i' =34.2~


(1)sini =(¾)sin30° ..
=> 6ro1a1=180°+2i'-4r => i =34:2°

=> OTotal =128.4° A

4. Since, the condition for no emergence is


A>2C

=> A>2sin-
1
(¾)
1
A> 2s1~-1 ( --) >83.62°
1.5 B~-~-----~c ·
Therefor~, Ama:,,. ="83.62°, for escaping of the ray, thr~ugh the
(b) Total deviation suffered bythe.ray·;s_
adjacent face. 6=60 +6E =260
5. T~e situation is shown in figure => 6 = 20-30°) = 8.4°
p
7. At near normal incidence, i 1:::: r1 = 0°
Since r1 + r2 =A
~ r2 =Cl
From Snell's Law apRlied at the face from where the refracted
ray emerges, we get
_ sine
µ=-.-'
s1nr2
~ e = sin-1 (µsina)

Now, deviation O= i+e-A =sin-1 (µsina)-a


~ O=sin-1 (µsina)-a {·: i = 0°}
(a) Since, e = 90°

Also, r2 =C=sin-
1
(¾) 8. For the ray to retrace its path, it must be incident normally to the
face AC .:So, we have · "
Now, i=A=45° and r, =A-r2 =45°-C r2 =Oo
• Applying Snell's Law at AB , we get

= ====================================
L184

www.puucho.com
Rahul Sardana with www.puucho.com

Ray Optics
A sinA
sinr1 sinC
=> sinr1 = sinAsinC ... (3)
The ·ray does not ·emerge from the other face AC, when
r2 >C
Since, r1 + r2 =A
=> A-r, >C
=> r1 <A-C
=> sinr, < sin(A-C)
Since r1 +r2 =A => sinAsinC < sinAcosC-sinCcosA
=> _r1 =A=30° => 1<cotC-cotA
From Snell's Law, we have => cotA<cotC-1

µ= ~ini =../2
smr1 11. For Violet Light: According to Snell's Law, applied at the plane
of incidence, we get
9. From the. statement of the problem, we gather the information sini
µ=-.-
that Slnr1
i=60°, A:::30°, 6=30°, 0
1. = sin(so )
Since, B=i+e-A => 66
sinr1
=> e=6-i+A=30°-60°+30°=0°
A => r1 =27.5°

Since, r1 + r2 =A
=:> r2 =A-r1 = 32.5°
Applying Snell's Law at the plane of emergence, we get
sine
µ=-.-
s1nr2 .

=> 1 _66 = sine


sin(32.5°)

i.e., the emergent ray is _perpendicula~ to the face through which => e=63.1°
it emerges. Since, 6v =Ci+e)-A
Further, r2 = O and r, + r2 = A
=:> Sv=53.1°
=> r1 =A=30° {as e = O}
For Red Light : According to Snell's Law, applied at the plane of
From Snell's Law applied at face AC , we get incidence, we get
µ= ~ini =>/3 1.6 = sin(50°)
smr1 2
sinr1

10. Fron:i geometry, we observe that the angle of incidence at the :::::) f1 =28.2°
face AB is A . Applying Snell's Law at face AB , we get
Since, r1 + r2 =A
sinA
µ=-.- ... (1)
smr1 =:> r2 = A-r1 =31.8°
A Applying Snell's Law at the plane of emergence, we get

1 _62 = sine
sin(31.8°)
:::::) 8=58.6°
Since, SR = (i + e)-A =48.6°
So, angular dispersion is given by

B c __ _ _ _ __,C
Sv -6R =4.5°
If C is the critical angle of th_e prism, then 12. For minimum deviation, we have
1 ... (2) . (A+o.)
S in--
µ=sine
µ= 2
From (1) and (2), we get
sin(i)

==================================1.1ss=
www.puucho.com
Rahul Sardana with www.puucho.com

Advanced JEE Physics Optics & Modern Physics

=> 1.3sin(45°)=sin(45°+B.) :::::, r, =A=45°


Solving, this we get Now, according to Snell's Law, we have
6m =22°
137=~
For maximum deviation, we have the emergent ray to be grazing · sin(45°)
on the surface of emergence. So, => i = 75.6°
e=90°, r2 =C and r, =90°-r2 =90°-C => 6red=i+e-A=30.6°
=> 6max =i+e-A For violet ray, we have
. C 1 1.42 = sin(75.6°)
We can find i by using µ
sini
= -.- and s1r:-i =-
. .
. 8 ubstituting s1nr1
s1nr1 µ
the values, we get => r, =43°
6max ::= 550 => r2 =A-r1 =2° f: r1 +r2 =A}
Again, applying Snell's Law at the emerging face, for violet rays,
13. Given that, i = 60° , A = 30° and B = 30° we get
Since, 6=i+e-A
1.4 2 = s!ne
Substituting the values we get, e = 0° smr2
Now; e = 0°, means that the emergent ray is normal to the face => 0=2.84°
through which it emerges.
=> 6v1o1ei=i+e-A=33.4°
14. Given that A= 30° and i = 0°, so r, = 0°
17. As the angles are small we can take,
Since, r, + r2 =A sine;::: e
=> r2 =A=30° A
Further applying Snell's Law at the plane of emergence, we get
1 _5 =s!ne
s1nr2
Substituting the values, we get
e=-49°
=:> 6=i+e-A=19°

15. For no total internal reflection, when the ray leaves the prism,
r2 =C
1 C
But sinC = .!µ = -1.6
-
Applying Snell's Law for the two emerging rays at AC and AB ,
1 we get
r, =C = sin-•(--) = 38.7° sin(A+1°) sin(4°)
1.6 µ
sinA sin(2A)
Further r1 + r2 =A= 45°
A+ 1° 4°
=> r1 =45°-38.7°=6.3° => µ~~A-=2A
Now, i = sin-• (µsinr,) = sin-' [1.6 x sin(6.3°)] = 10.1° => A=1° and µ=2

16. For reQ ray, we have


e=0°=r2
Since, r, + r2 =A

= c:==================================
1.186

www.puucho.com
Rahul Sardana with www.puucho.com

1. Let us see where do the parallel rays converge (or diverge) on Now applying .&__,& = µ 2 -µ, with the idea that 12 is formed
the principal axis. Let us call it the focus and the corresponding V u R
J:.g_ = 11-a = µ2 - µ1 with at C, because light falls normally on the mirror.
length the focal length f • Using
V u R => µ 1 µ-1
appropriate values and signs, we get
-(R+x) -(2Rµ+x +R) -R
4 i_1
3 1 3 Solving this equation for µ =1.5 , we get
1-~= +10
x :::::,Q.75R
=:- f=40cm=0.4m
Since, the rays are converging, its power should be positive.
Hence,
1 1
4. Applying, µ 2
V
- .& = µ 2 R- µ, , we get
U
i !E
P (in dioptre) = - ( -- ) = - -
f metre 0.4
1 1.6 1-1.6 5cm '•

&T
PI - (-3) = --::S
=> P = 2.5 dioptre= 2.5 D
=> PI =-2.42 cm
2. For first refraction at the unsilvered surface, we have => EI= (5 + 2.42) cm
1.5 1 1.5-1
v,- (-2r) = - - => EI= 7.42 cm 0
:=:,. V1 --tco

i.e., rays become parallel to the principal axis. 5. (a) Applying, µ 2 - µ, = µ 2 R- µ, , we get
Hence the image formed by the curved mirror will lie at the focus V U

of the mirror i.e., a distance i from pole of mirror.


1.5
V -1
1 1.5-1
6
=> V=-90 cm
{from pole of the mirror} So, the distance between object and its image is 80 cm
For second refraction at the unsilvered surface, we have (b) Again applying µ 2 - .& = µ 2 - µ1
1
v, -(-~r
1.5 1-1.5
(-r)
=>
1.5
V - <-u) = - 6 -
1
V
1.5-1
u R

=) V3=-2R
i.e., final image is formed at pole of the mirror.

3. For the 'image of object O to be formed at O , the light should 12


v is negative when >1
fall normally on mirror. First image 11 (after refraction from the u
::::> U<12cm
plane surface) will be formed at a distance of 2R+x from plane
µ
6. Applying, µ 2 _ ,& = µ 2 - µ, twice, we get
surface ' because dapp. = daC\1./al
µ . V u R
1.5 1 1.5-1
:V, - (-2.5) = ~
====================================1.187 =
www.puucho.com
Rahul Sardana with www.puucho.com

Advanced JEE Physics Optics & Modern Physics

30 -----.. +ve
~ v, = --=, cm
,1 1.5 1-1.5
Further,
v, -(20+3;) -10

::::::, v2 =-85cm
i.e., final image is formed at 65 cm from first face on the same
side of the object.

7. First image will be formed by direct rays 1 and 2, etc.


Applying, µ 2 - µ, = µ 2 R- µ, twice, we get
DO 5 V U
D11 =-=-=3.33 cm
· µ 1.5 µ 1 · µ-1
Second image will be formed by reflected rays 3 and 4, etc. AI, - (-2R) = -R
Object is placed at the focus of the mirror. Hence, 12 is formed
=> AI ~ 2µR
at infinity. 1
1-2µ
1 µ 1-µ
Further,
BI, (AI, -R) -R
Solving this equation, we get
2R(4µ-1)
BI,
+
5cm
3µ-1
So, the distance between the final image and the object is

*:t
5cm d= 3R 2R(4µ-1) (µ-1)R
3µ-1 (3µ-1)
·p

8. We have to see the image of O from the other side

= c:::::==========::::;:======================
1.188

www.puucho.com
Rahul Sardana with www.puucho.com

1. When the images of both the sources are formed at the same -1+3
point, then v will be same for both (in value). However for one => --=-
24
case the image will be real and for the other case it will be
virtual. So, ~ f=12 cm
· 1 1 1 Note that you would get the same answer by considering the
For 0 1 , we have-+-=- ... (1)_ othercase(m=-3,for u=-16cm)
V X 9
1 1 1
For 0 2 , we have--+--=- ... (2) 3. The Lens Maker's Formula is given by
V 24-X 9
f=9cm !=(µ-1)
f
(_!_-~)
RI R2
Here, f =30 cm, R1 =10 cm, R2 =oo
o......______...oz
o - - - - - - - - - - - - < •

0, => ~= (µ-1) (_!__.!) = µ-1


30 10 ro 10
=> 30µ-30=10
~X----S--24-x-t-t 4
Adding equations (1) and (2), we get => µ~-
3
1 1 2
-+--~-
x 24-x 9 4. Applying Lens Maker's Formula,
SOiving we get x = 6 cm
Hence, the lens should be kept at a distance of 6 cm from !=(µ-1)(_!_-~)
f R1 R2
either of the object.
Here, µ=1.5, f=60 cm, R1 =+R, R2 =-R

2. Here the image formed can be virtual as well as real, so the => 6~=(1.5-1)(¾+¾)
value of m should be +3 in one case (virtual image) and -3 in
the other (rea! image). Magnification of +3 can be obtained ~ R=60 cm
only when the object is placed within F {I.e., for smaller value of Therefore,· the focal length of the spherical silvered surface, is
the object distance). The magnification of -3 is obtained when given by
the object is kept between F and 2F (i.e., for greater value of R 60
\,, =-=-=+30 cm
object distance). So, 2 2
m=+3,for u=-Bcm (Positive, because it is a converging mirror)·
Therefore, ·from the definition of magnification, we have The equivalent focal length of the lens-mirror combination is then
V
given by
m=- 1 2 2 1
u
-F=l-t:"= 60 - (-30)
=> +3=..!....
-8 ~ F=-15cm
V=3x(-8)=-24 cm The negative sign indicates that the combination behaves as a
=> concave mirror.
Using lens formula, we get
1 11 5. According to the Lens Maker's Formula, we have
-24 -8
1
-=(µ-1) ( -1- ~
1 ) ,where f=10cm ... (1)
f R1 R2

======================================1.189
www.puucho.com
Rahul Sardana with www.puucho.com

Advanced JEE Plzysics Optics & Modem Plzysics

When placed in medium 1, then 1 1

f=(:,-1)(~ -;J ..• (2)


=>
36- -45=1
1=20 cm
When placed in medium 2; then -in the second case, let µ be the refractive index of the liquid,
then
.!.t = (_!:_-1)(_!__
µ
_ _!__)
R,, R
.•. (3)
2 2
From equations (1) and (2), we get 48 -(5+ ~) 20
1= {µ-1) 1.6-1 9
I (:,-1) G:~-1) Solving, we get
µ=1.37
=> \ =91=90 cm 10. The system behaves like a mirror of focal length
i.e., still it behaves as a converging lens given by,
From equations (1) and (3), we get 1 _ 2(µ,/µ,) 2(µ,/µ,-1)"
F--R-,- R, n,
1= (µ-1) 1.6-1 --102
I (_!:_-1) (~-1) . Substituting the values with appropriate signs, we
µ, 1.7 get
=> ~=-10.2f=-102cm
i.e., it now behaves like a diverging lens. {·: R, ->co)
=> F =-7.5 cm
6. Here R1 =+25 cm, R2 =-25 cm, µ 1 =1 and µ 2 =23 So, the system behaves as a concave mirror· of focal length
7.5 cm.
Image coincides with object, hence, u =v =-x (say) -----+ +ve
1 _ 2{3/2) 2(3/2-1) 11. For a convex lens, the distance between an object and its real

0
-x - . - -25 25 image is minimum when· u = 2~ and v = 2~
2 3 1 4 When concave tens is placed in contact, then we have
-=-+-=- 1 1 1
x252525 ---=-
V U fcomb
=> X=12.5cm
Hence, the object should be placed at a distance 12.5 cm in 1+1=1_1_t-l
front of the silvered lens. v'2\\~lt
v' = 2\t
7. Given,\ =+5 cm and t =-10 cm ~-2\
The combined focal length F is given by Shilt of image Is
2
11111 1 tJ.v=v'-v= \t - 2 \ = ~
---+------+- t-2\ t-2\
F I t 5 10 10
Since, ~ » \ , so the shilt of image is
=> F=+10cm
i.e., the combination behaves as a converging lens of focal /J.V~-
4\'
length 1o cm. t
8. According to Lens Maker's Formula, we have 12. The following two cases are possible.
CASE-I:
!f = (µ -11(_!__
R R
- _!__) The mirror is at a distance of d = f + R ,;, 2 m from the lens.
1 2
The path of the beam parallel to the optical axis of the
=> _!__=(1.5-1)(_!__ __
10 R, -10
l_) R, system and the image of object AB are· shown in figure.
Image A'B' (direct and real) is obtainE!d to fu_ll scale with
=> R1 =+10cm the object in any position.
2
Now, using, .!. + 1 2 (µ,/µ,) 2 (µ,/µ, - l)
l+-R-+<
vu R2 R1
Substituting the values, we get
1 1 2(1.5) 2(1.5-1)
v- -15 = -10 - +10
=> v=-2.14 cm F ,--'2F B'
.\-,>l+-1-f-----.. .... --
9. Using lens formula, _! _ _! =! , we get
.
,, ______ ...... ., --
V u I

= ====================================
1.190

www.puucho.com
Rahul Sardana with www.puucho.com

Ray Optics
CASE-II: f,=4a
The mirror is at a distance of d = f =R =1 m from the lens. B
The image of object A'B', also full scale, will be inverted
and virtual with the object in any position.
A

Lens 1 Lens 2 Lens 3


1+- x--,---4a _ __.,.__ 4a - - . i
1 1 1
-+-=- ... (1)
V1 X 4a
B
4ax
v1 = x-4a
=> m1 =V = V1 = ~ ... (2)
u -x 4a-x
Similarly for Lens 2, we get
1 + - - - d - - -.. 1 1 1
13. For a convex lens, distance between an object and its real
v2 v1 -4a a
image is minimum when u =2~ . Hence, 1 x-4a 1
... (3)
V
2
- 16a2 =a
(a) 3\-(1-¾}=2\ 16a2
=> v2 =x+12a
(1-¾}=1 16
( 1a2' ) x-4a
=> m x+ a ... (4)
.1.=1-.\_= 1-\ ,-(16a2 ) x+12a
µ I I
x-4a
I
µ = i""=\ Similarly for lens 3, we get
1 1 1 ... (5)
(b) If a concave lens of very large focal length ~ (i.e., very V
3
- v2 -4a = 4a
small power) is placed in contact with the convex lens, => v3 =-(x+Sa)
then its power and hence the focal length are almost
unaltered. Therefore, there will be no shifting of the image. x+12a
::::, m3 = ~ ...(6).
1 1 1 we get So, we observe from (2), (4) and (6) that
14. Using the lens formula, ---=-,
V U f m1m2 m3 =-1
for the first lens Also, v3 = -(x + Sa) means that the object and its image lie at
1 1 1
-+-=- the same place and m1.m 2m3 = -1 means final image is of the
v, 30 20
same size but inverted.
=> v1 = 60 cm
For the second lens
17. According to the lens formula, we have, .!V _.!U = !f
_.!..._...!_=_.!...
V2 30 10 Since u = -mf , so we get
=> v 2 =7.5 cm 1 1 1
v+ mf = (-fl
2(~:J. 2(~-1J => V=·-(m:Jf
2µ _ 2(µ-1)
15. Since, } So, lateral magnification is given by
R, R, -r
V 1
¾=-( 4µr-2) ml =u= m+1
Hence, the image will be (m + 1) times smaller than the object.
F=-(4/-2]
18. (a) Since a concave lens always forms an erect image
i.e., the lens is equivalent to a concave mirror of focal length whereas the given image I is on the other size·of the optic
r axis, so the lens is convex.
4µ-2 (b) Join O with I . Line 01 cuts the optic axis AB at pole
(P) of the lens. The dotted line shows the position of lens. ·
16. Let x be the distance between first lens and the object AB .
Applying the lens formula, .1.V - .!U = -f1 thrice, we get for lens 1,
r:::::=================================1.191 =
www.puucho.com
Rahul Sardana with www.puucho.com

Adva11ced JEE Physics Optics & Modern Physics

For first reflection, let us use the mirror formula, i.e.,


1 1 1 2
-+-=-=-
v u f R

=> :, +(µµ;1)=:

I => 2-=-(3µ-1)
v1 µR
For second reflection, similarly we get
From point O , draw a line parallel to AB which after
_!_;3µ-1=-2
refraction must pass through the focus F of the lens.
V2 µR R
19. Let parallel rays be incident on first'lens, then
=> v: =-(5~;1)
v,=\=7cm
For the second lens, we have Similarly after nth reflections, we get
1 1 1 _!_ = -[(2n + 1)µ-1]
v, -(7-3)=6 v0 µR

Finally again u·sing the refraction formula, ~ - ~1 = µ 2 ; µ 1 ,


V2 4 6
y 2 =2.4 cm applied at the second curved surface, we get
_!__{(2n+1)µ-1}= 1-r,
20. (a) From Lens Maker's Formula, we get V1 R -R
R
!f = (µ -1)(_!_
R R
- _!_) ~ v,=~--~
1 2 2(µn + µ-1)
\ =µ,-1= 1.7-1=7 =1.4
t µ,-1 1.5-1 5
(b) In this liquid the first lens will be a diverging (as refractive 1
24.
index of liquid > 1.5) and the second a converging one (as \
refractive index of liquid < 1.7 ). 1 2µ 2(µ-1)
21. Since the incident beam is parallel, so we have
\=~--R-
v, =\ =10 cm -R
For the second lens; we have \=2(µ-1)
1 1 1 So, the system behaves as a concave mirror of focal length
V2 -5 -20
-R() . The object will coincide with image when the object is
:::::>v2 =-4cm 2 µ-1
For the third lens, we have placed at centre of curvature. So, we get
1 1 1 R
x, =21\I=- ... ,1)
µ-1
-2µ
;::;:> V
3
-)-00 In the second case, f1 =R
i.e., rays will become parallel to the optic axis.
R
22. Since the focal length is equal to two times the radius of => t=-2µ
curvature, so f =-2r
=> x,= 2ltl=~ ... (2)
(-~) =(µ, +a1-1J(±-n µ
Solving equations (1) and (2), we get
I= 3-2µo X
2a. µ=--'-
x, -x 2
23. The rays will first get refracted, then n ·times reflected and and R = ~
x, -x 2
finally again refracted. So, using µ2 - µ1 = µ2 - µ1 for first
Now, according to Lens Maker's Formula, we have
V u R
refraction, we get ! = (µ -1) (..!) = µ - 1 = 2- {from equation (1)}
.!:_ _ _!_= µ-1 f R R X1
v1 oo R ~ f=X 1

v, = (_.!:._)R
µ-1

= c:::=====================================
1.192

www.puucho.com
Rahul Sardana with www.puucho.com

1. [CJ f
=> m=---

=> m=-
3
1
f-(-2/)

I, I, 4. [DJ
Only one image will be fom,ed by this lens system, because the
optic axis of both the parts coincide. Two images would have
been formed if their optic axis would had been different.
i.--- 8 cm ---+1
5. [BJ
Distance of image from the plane surface is MISCONCEPTION
4
X1 =-=2.5cm
1.6, {.. d =d"ci""}
• app µ Maximum students are gripped by the misconception that when
object lies at focus then image is formed at infinity. This is true
For the curved surface, we have but only for a Concave Mirror (Convex Lens) and is absolutely
1.6 1 . 1-1.6 wrong when applied to Convex Mirror (Concave Lens)
--+-=-- How To Proceed Then?
4 X2 -8 If object placed at focus then just check out the MIRROR. If
=> x2 :::::--3 cm CONCAVE then image is formed at_ infinity and if Convex then
1 1 1
The minus sign means the image is on the side where the object apply-+-=-
. V U f
lies. So,
1,1, =(8-2.5-3) cm=2.5 cm ~ ---=-
V 20 +20
2. [CJ ~ v=10cm
Area of object = 9 cm 2
Also, we know that 6. [CJ
Image will be formed at infinity, when the object is placed at
rea agm.1.
AIM . = mar = -A,v'(f)'
1cat1on =2 = -- focus of the lens i.e., at 20 cm from the lens. So, we have
A0 u f-u
Shift t.x=25-20=(1-t}
A [ -10 ]'
=>
t= -25-(-10)
1
5=(1-- )1
=> A, =rnJ x9
=>

=>
1.5

=>

L W ~ [CJ
The similar thing is extended and applied here too. Here the Critical angle between glass and liquid interface is
answer fabricated by the MISCONCEPTION is 1 (but we must
know this is the answer only for a Concave Mirror (or Convex sinC = ~ = Zµ
Lens). For Convex Mirror we have 3/2 3
f Angle of incidence at face AC is 60°
m=--
f-u For TIR to take place, we have
i>C

::====================================1.193 =
www.puucho.com
Rahul Sardana with www.puucho.com

Advanced JEE Physics Optics & Modern Physics

I 2 I
=>
1-(-15) 1-(-20)
=> 1+20=2f+30
=> l=-10cm

11. [BJ
1
m=+-
B 2
1 20
2 => - = - -
=> sin(60') > ; 2 20-u
=> 40 =20-u
=>
3-13
µ<-- => U=-20 cm
4
12. [BJ
8. [DJ Focal length of mirror is independent of the refractive index of
Since no parallax exists between the images formed by two medium in which it is placed.
mirrors (convex and plane) hence the images for both coincide.
But for a plane mirror an image is as far behind the mirror as the 13. [AJ
object is in front of .it. Hence the image for plane mirror should be Since image formed is erect, hence it must be virtual. So,
30 cm behind it or 1o cm behind the convex mirror. So for
convex mirror. m=-~=3
u
U=-50cm, V=+10cm
Also Jul+lvl=D
Since ..!.+.:!.=!
V U f
lul+3lul=B0
lul=20
1 1 1 ~
=> -+-=- Since object always lies on negative side. So,
10 -50 f
0
' u=-20 cm
5-1
=> 3 I
f 50 => f-(-20)
J+-30 cm---30 cm~
=> I= 50 1 + - - - 50 cm-----+1 => 31+60=1
4 => f=-30cm
=> 1=12.5 cm Negative sign indicates the mirror is concave.
Since R=2f An Advice
=> R=25 cm I would always advice you to write _lul+lvl =D wherever you are
given the distance between object and image as no error will
9. [DJ creep in these because the MOD signs prevent the errors.
Concave lens forms the virtual image of a real object. So, we
have 15. [BJ
Since, o=(µ-1)A=(1.5-1)(4)=2'
1 NOW,I"f
m=-u=4·
V
=> 1=6=2°
u=-4x,then v=-x then 3x=10cm Let the mirror be rotated by an angle 0, then
. 10 i'=(2'+0)
=> x=-cm
3 Since, 6to1a1 = 180°
40 0
=> U=--cm => 0+1so -2i' =180'
0

3
14-3x---+x => 6=2i'
10
and v=-- cm => 2'=2(2+0)
3 ----4x------ 0= -2°
Substituting in ! =_!__.!., we get Here, negative sign implies that i gets decreased or i' =0 . i.e.,
f V U
light should fall normally on mirror.
1 -3 3
-=-+-
f 10 40 16. [BJ
The minimum length of the mirror required for the purpose is-half
I=- 40 the height of ttie person.
9
=> f=-4.4 cm

~L
17. [BJ
A divergent beam appears to Convergent Beam
10. · [CJ converge behind the mirror thus
· Jm.,,J=2Jm..,,,,J giving a virtual image. So, a
convergent beam will give a real
image. - ~

= ::::::===================================
1.194

www.puucho.com
Rahul Sardana with www.puucho.com

Ray Optics

19. [CJ
10 _ 900 0 _ 180 cm
Al .111
ppymg v-u=1,wege
t
=>
55 11
Negative sign with focal length implies that the mirror is a
111 concave mirror.
b+a=1 25. [BJ
f=+20 cm
=> f=~ ... (1)
a+b u=-10 cm
Further in right triangle ACB , we have S" 1 1 1
2
AC + BC 2
=AB 2
mce, v+u=t
1 1 1
=> (a'+c')+(b'+c')=(a+b)' =>
V 10 20
=> a2 +b 2 +2c2 =a2 +b2 +2ab 20
=> V=-Cffi
:::> ab=C2 3
Substituting this in equation (1 ), we get
c' 26. [A]
f=- Covering the lower half will just make the image less bright (not
a+b blurred) as less number of rays will be reflected as compared to
the previous case.
20. [CJ
If the mirror approaches the object or the object approaches the 27. [BJ
stationary mirror with speed v then image approaches object
with speed 2v.

21. [CJ
I
mrea1=-n=f-u
=> -nf+nu=f 20"..• -···
-··20°
=> U=(n:1} 0 : 'b
g:ui
40" 40"
22. [DJ
The incident and the second reflected ray make the same angle
8 with vertical. Hence, they are parallel for any value of 8 .
28. [CJ
A'

23. [D] B B'


1 I
n= f-u
From symmetry we observe that length of mirror is one third of
height of wall.
f-u=nf
=> u=(1-n)f -(n-1)f 0
=> i=!:!3
According to the sign convention used u must always be
negative. 29. [DJ
360
24. [DJ n=-->oo
0
mreal =-4.5
f 30. [A]
m=-
f-u
-4.5 = f
3
:o = 6 which is Even

f-(-20) n= 360 -1=5


=> -4.51-90 =f 0
=> 90 =-5.51
31. [BJ
!=-~ Ray's after reflections from two perpendicular mirrors are always
5.5 parallel to incident ray irrespective of angle of incidence.

================================:::i1.19s=
www.puucho.com
Rahul Sardana with www.puucho.com

Advanced JEE Physics Optics & Modern Physics

32. [CJ 38. [BJ


Power of concave lens must be Jess than that of the convex lens Both concave and convex mirror give virtual image but a
to form.a real image. So, net power will decrease or fqcal length concave mirror gives a magnified virtual image (when object
will increase. Far real image v is- positive, u is negative and f placed between F and P). Since the boy sees his image of
diminished size, so the mirror must be convex.
1 1 1
is positive. Applying lens formula, ---=- (substituting all
V U 39. [A]
values with-sign), we get Let object be placed at a distance x from mirror.
1 , , u=-X, v=-(x+10), f=-12cm
-+-=-
v u I 1 1 1
Since, -+-=-
u is constant, f is iilcreasing. So v will also increase. v u I
1 1 1
34. [D] +-=-
-(x+10) -x -12.
G
x =20 cm
C
==> U=-20cm,

t
'D H
···j······· V=-30 cm

!
d :s
------i------
So, m=~:!.= -(-ao) =-1.5
u -20
Negative sign with magnifi_catiori indicates image is real.
B :E
F
,._ L.., J 40. [BJ
14--2L---+1 At face AB , the ray of light suffers no devicltio11, so applying
Snell's Law at face AC, we get ·
35. [BJ 1 1
Since an elongated- image is formed and it touches one end of µ = sinC = sin(45°)
the rod, so the rod must lie with one end at 2F and other end
between 2F and F (shown in figure). => µ,,,. = -,J2
51 A
Forend A, U=-
3
. , , 1
S mce -+-=-
v u I
1 3 1
v-51= -I
1
.=51-51
3 5
A'
-
0
f/3

A
ROD
F
--1--+-4-~~~-',~/
,
/

-21-
B C
1 2
.=-51 41. [CJ
51 1+-f- 1 1 1
V=--
~ fil ----+I -+-=-
2 3 • v u I
PA'= 51 ==> v-1 + u-1 =r-1 = constant
2 Take derivative w.r.t. time on both sides

0/>/=~-21 !/v·')+!/u·')=O
2
dv ( ) _, du
( - 1) V_, -+ - 1 U -= 0
dt dt

36. [BJ
First _consider two adjacent walls (not the ceiling). If n1 is the
:~ =<'.(:~)
v'
number of images formed due to these perpendicular walls, then Image speed= - u' ( object speed)
n = 360 _ 1 when object moves towards mirror u decreases with passage of
' 90
==> n1 =3
time and hence du =-9 cms-1
dt
Now, when we consider the mirror on the ceiling then it will make
a total of 4 images (one of the original object and th_ree images dv =-(-1-)' du
of the previous arrangement) so, total images formed equals 7. dt f-u dt
However, if the object were the observer himself then total dv ( -24 )' du
number of images is 7 -1 = 6 di=- -24+60 di

37. [CJ
Velocity of light is always normal to the wavefront.

= ==================================
1.196

www.puucho.com
Rahul Sardana with www.puucho.com

Ray Optics

dv =-i(-9) {·: 2sin(½)cos(½) =sini}


di 9
dv ,
-=+4 ems-
di => n =2cos( ½)
Positive value of ~: indicates that v increases with the passage => cos(½)=%
of time i.e. image must be going away from mirror.
=> i=2cos- (%) 1

42. [DJ
A_:::: 60° for equilateral prism.
47. [Bl
i=e=~A The source cannot be seen if angle of incidence in the denser
4 medium is greater than the critical angle.
Since, i+e=A+D Since,
=> 2i=A+D d h
r=--=--
~µ'-1 ~µ'-1
=> 2(¾A )=A+D
=> D=~=30° h=(1)J(%)'-1
2
4
h=- cm
43. [BJ 3
Since, uab > U00
48. [A]
=> Vab<Vcd sinC=..!.
µ

Since, m=-"!... TIR will take place at AC if i > C i.e. 45° > C for
u Red Colour
=> [m,,[<[m.,[ µ, =1.39
. CR =-->-=Sin
Sin 1 1 . 45
44. [D] 1.39 1.41
C
µ=-V sin CR > sin 45

8 => CR> 45°


Z= 3x10
=> (No TIR will take place)
V
Green Colour
=> V=1.5x108 ms-1
µG=1.44
=> v=1.5x10 10 cms-1
1 1
·cG= .4 < .4 =sIn
sm ·45
1 4 1 1
45. [CJ
sin CG< sin45
[v0 [ =[v,[ =~(2)' +(2)' =2-12 ms·' => CG <45°
Relative velocity of image with respect to object is in negative x- ( TIR will take place for Green Colour)
direction as shown in figure.
Blue Colour
y
µ 8 =1.47
v,
, . C =--<-=-Sin
Sin 1 1 . 45
v, v, , 8 1.47 1.41
,,

~
,
,, sinC8 < sin45
45° X
-
VlO
,
,
45°
=> c, < 45' (TIR will take place for Blue Colour)
I 0
'' 45°
'' So, Red is separated from Green and Blue.
''
' 49. [C]
-Vo
46. [CJ
i =2r
A,=(~; =;)A,
. sini
Since, n=-.- => A =(1.54-1) 4
sinr ' 1.72-1
:,. A2 =3o
sini 2sin(½)cos(½)
=> n
sin(½) sin(½)
==================================1.197=

www.puucho.com
Rahul Sardana with www.puucho.com

Advanced JEE Physics Optics & Modern Physics

51. [AJ 59. [BJ


Since rays after passing through the glass slab just suffer lateral
sine= µrarer = vdllnsar displacement hence we have angle between the emergent rays
µdenser vrare, asa.
=> sin8 = vder,,;er = vA
Vrarer Ve 60. [DJ

=> sine=.!. Applying Snell's Law µ = s!ni , we get


v, s1nr

=> sinr = sin I


=> µ

52. [CJ
Since, µv = constant
=> µ 9V9 =~Vw
s
=> ~(2x10')=iv R
2· 3 w
1
:::::> vw =2.25x106 ms-
. sin60° 1
53. [DJ
=> sInr= -/3 =
2
After two reflections from two mirrors placed at right angles, the
=> r=30°
emergent ray will always be parallel to the incident ray for any
value of i. Since, PC= QC
=> LCPQ = LPQC =Lr= 30°
54. [CJ . Angle between reflected ray QR and refracted ray as at the
Since light has to travel from denser to raser medium so, it must
other face Is 180° - r -60' = 90° {·.· r = 30')
be made incident in the dens8r medium at an angle less than
critical angle.
61. [AJ
55. [DJ According to Snell's Law
C oc 1. sini =µw
Since A.Red > AVlolat sinr µ8

==> CRed > CVlolet => sinr=....!_sini


µ.
57. [BJ
=> sinr = ) sin60
According to the problem, we have, u = -(1-1) 43
V=-O+1)
f =+f
r = sin-{
3
;3)
....:
:
l
............... ..
"'" .... 62. [BJ
I
I
' , -.._
._ ,.._
LDBM=45'
: .......... ; ......... => DM = htan45° = h = 32 cm
: ,.,_,_,___ sini A
µ=-.-
'' smr
_______ !! --- ---
F

_,_
.
µsini = 4i'J
sin45° 3

':i; ~~,_ili ~;
smr = = ,/2
4
tanr = sinr _ sinr
=>
cosr .J1-sin2 r
3
I+--- f + 1--+t 4.J2 3 Stone

.
Appymg
I
1 1 1
---=-, we get
=>
tanr= J1- 9 = m
V u I 32
1
--+--=-
1 1
-(1+1) 0-1) f
=> CM=htanr=32x
v23
=20 cm = 3

=> CD =DM-CM = 32-20 =12 cm


=> 12 -21-1=0
=> 1=("2+1) cm 63. (CJ
sinC=_!_=~
µ. 4

-1.198 =================================
www.puucho.com
Rahul Sardana with www.puucho.com

Ray Optics
=> C=49° µA =constant
Apex angle of cone is 2C = 98° => µaAa =·µgA.g
'' . ''
A = µaA.a
- -•:•c'~
: -.;:ec,_c :-'·-~~---:
- • µ,
=====~==-- - -49;-:-~~====
____________'{9: ____________
------- __ -------
-:-:-:-:-:-:-,
- - - -- - - -F: :-:-:-:-:-:-
- - - - - - - -- 75. [BJ
----------------- This happens due to multiple refractions and reflections. The first
image is formed due to reflection at X 1Y1 and is fainter. The
64. [DJ
In 6OAN' second image is formed due to reflection at X2 Y2 and is

sin(i-r)=""
OA
i brightest all other images formed further are faint.

ti
••..•(i-r)
=> 6x=OAsin(i-r)
In 6OAN ••·••••.••.. N' t
ON
cosr=-
OA
1
OA=--
'---'-----"-!_.._.·~--'.....··'-'
N ··
...
l ;{::]''
cosr
tsin(i-r)
6x 76. [CJ
cosr In both A and B, the refracted ray is parallel to the base of prism.

67. [CJ 79. [AJ


For grazing incidence and emergence, we have
. (-
sm A+D.)
-
2
i=e=90°, r, =r2 =~=30° µ
2
sin(~)
According to Snell's Law, we get
µ ·= ~inl =2 .J2sin30 = sin( A+ Dm)
s1nr, "' 2
1
v1z=sIn (A+Dm)
- -
68. [AJ "' 2
4
sine=.!\:.= / 3 60+D. = 45
µ, 3/2 "' 2
_
-
C
=Sin
. -1(8)
Q
"'
D. =15
2
and light must go from denser to rarer medium. Dm = 30°
"'
69. [BJ Further we know that at minimum deviation i = A ~Dm
According to Snell's Law
µA = constant => i=45°
Since A.Red > A.Vlolel
80. [CJ
Normal Emergence implies
e=O
71. [CJ Since i+e=A+D

Critical angle C =sin-


1
(¾) For prism with small A,
D=(µ-1)A
=> i=A+(µ-1)A
=> C = sin-1 (i) =30° "' i~µA
When A > 2C , the ray does not emerge from the prism. So,
maximum refracting angle can be 60° . 81. [CJ
Since, 1 µ 2 x 2 µ 3 x 3µ 4 x 4 µ 1 =1
73. [AJ 4µ3 x3µ2 x2µ1 x1µ,1 =1
According to Snell's Law
µv = constant 4 3
µ3x µ2x µ1=-,-
2 1
µ,
74. [AJ
According to Snell's Law

================================== = 1.199

www.puucho.com
Rahul Sardana with www.puucho.com

Advanced JEE Physics Optics & Modern Physics

82. [CJ 88. [AJ


Let thickness of slab be t and real depth from first side be x.
Then Apparent Depth = !!.
n
µ = i (when viewed from first side)
90. [CJ
µ = t~x (when viewed from second side) From mirror formula, we get
1 1 1
-+-=-
3x X f
(·.-µ=¾) 4
=:> X=9cm => ••. (1)
3X
3 t-9 From mirror formula, we get
=>
2 4 1 1 1
=> t=15 cm ---+--=-
2(6+x) 6+x f
83. [DJ 3 1
=> ... (2)
2(6+x) =1

Since, µ
sin(~)
sin(~)

=> cot(~)- sin(~)


6cm

2 ,....__ X __..,.__ _ 3 X
sin(~)
14-2(6 +X)+i
Equations (1) and (2) give
=> COS (2A) . (A+D )
=Stn ~ 4 3
3x = 2(6+x)
=> 9X =48+8X
=> => x=48cm
=> Shift of screen = 3x - 2 (6 + x)
Shift of screen = 3x48-2(6+ 48) = 36 cm
84. [AJ
i+e=A+D 91. [DJ
=> 60+e=30+30 Apparent Depth = ..i.. + ..i..
2µ, 2µ2
=:> e=0°

85. [BJ 92. [A]


sin(45)=.! In this case, one of the image will be real and the other will be
µ virtual. Let us assume that image of S1 is real and that of S2 is
=> µ=-12 virtual. Then, applyin~ .!+.! = !

=tr
V U f
86. [BJ
f =9cm
sinC1 =E:r_=.&_=..!._
µd µg µg
S, I S,
sinC2 = µw > sinC1 I I I

µ,
....,__ X y -+I
24-x~
for 8 1 , we get
87. [BJ
. d 1 1 1
S mce r= ~ -+-=-
y X 9
.•• (1)
-yµ2 -1
for S2 , we get
4
=>
r-~ 1 1 1
--+--=- ... (2)
y 24-x 9
Solving equations (1) and (2), we get
=> r=4x3=3m x=6cm
4
Diameter = 2r = 6 m

= ==================================
1.200

www.puucho.com
Rahul Sardana with www.puucho.com

Ray Optics
CONCEPTUAL NOTE(S) a 4
-=-
This questi~m may· have following ahsWer· 6 cm from S1 a~ c 3
18cm fr9m .52 and 18cm from.S; or6cm.fram S2 •
98. [BJ

93. [CJ
sin C = µrarer =,&_
µdenser µA
Optical path length = nt
Since, µ,._v,._ = µ 8 V8
So time taken = nt
C => · sinC= VA=_!_
V8 2.4
94. [CJ
=> . C =-
sin 5
To a fish. the outer world is seen in a circle of radius ~ ,d 6
. - "µ2-1
is the depth at which the fish swims. 99. [CJ
According to Snell's Law
12
:::::) r=--- µv =con_stant
~~-1 => µgVg =µtVt

36 => ( 1.5 )x ( 2 x10') = µ, (2.5x1 o')


=> r = ..ff cm
3 30
=> µ,=~=-=1.2
2.5 . 25
95. [AJ
For the diver 100. [DJ

·
1
sinC=_!_=--
µ. (4/3)
(Rememberthatµwatar
~
=±3.) . C
sm
1
= F2
:::::- C = 49° with vertical => C=45°
=> 0=90-49°=41° with the horizon. For i =C in denser medium angle of refraction {n rarer fJ.1edium
is 90°.
96. [CJ
sinC = µmm, = µw 101. [CJ
µdenser µg Real Depth
Apparent Depth
·. 4/3 4 µ
=> sinC=-=-
5/3 5 24
=> Apparent Depth =
413
97. [BJ => Apparent Depth = 18 cm
Applying Snell's Law at the interface sep8rating two media, we
get 102. [CJ
Given that A = Om = 60°
-y

I
__________ J _i;. I). I
µ,=1.5=§.
2
At minimum deviation, we have

i=(A~~m )=60°

1 I 8t+bj I b
' ' x-z plane (i)'
103. [BJ

"'"j'
I
: i c r+i=90
i=90-r
dl,, ••4L"
,I '-f I
I •
For ray not to emerge frOm curved
surface

~ - - - - - y ( o r j)
r-~-·"-, : =·~ =>
=>
i>C
sini>sinC
sin(90-r)>sinC
=> cosr > sinC
From the figure, we get
~ >-
1
=>
n {·: sinC=~}
(¾)(~ )= (-/c'c+~') 2 ... (1)
=>
2
1 _ sin i >_!_
Since a_7 + b} ~nd ci + d} are unit vectors, so we get·
~ n2 n2

-/a' +b' =-ic'+d' =1 => 1 >-\-( 1 +sin' i)


n
Substituting in equation (1 ), ~e get
,_ => n2 > 1+sin2 i

================================== 1.201 =
www.puucho.com
Rahul Sardana with www.puucho.com

Advanced JEE Physics Optics & Modern Physics

I
n>-./2 {sini--->1} CONCEPTUAL NOTE(S)
least value = -./2 Critical angle incre~ses as the relative refractive index • is
decreased.
104. [BJ
110. [BJ
Dispersive Power = :~
dµ 2B
dl. = -)!
So, as B increases, dispersive power i~creas~s.

105. [CJ

For the near end of the rod applying !f = ..:!.U + ..!.V


Since, here u and f are negative, so

lvl=~ 111. [BJ


u-f u=-20cm, f=+20cm, V=?
The farther end of the rod is at infinity, so its image will be
formed at focus. Hence, Since.!+.!=!
V u I
Length of the image is £ = lvl- f
1 1 1
=> - + ~ = -
£=~-f=__f_ v -20 20
u-f u-f
=> V=10 cm
Students generally give (A) as the answer to this problem
· 106. [CJ
because they have in their mind that if .an object is pl8.ced at
For A
centre of curvature C, then image is also formed at C, but this is
(1.5)t true only for the case of a convex lens or a concave mirror. So
Total number of wa:ves = -1.- ... (1)
correct answer is (B).
Total number)= ( Optical path length)
( of waves wavelength 113. [DJ
· Since no parallax exists between the images formed by two
For Band C mirrors (convex and plane) hence the images for both coincide.
Total number of waves But for a plane mirror an image is as far behind the mirror as the
object is in front of it. Hence the image for plane mirror should be
n,(½) (1.s>(¥) 30 cm behind it or 10 cm behind the convex mirror. So for
=---+--~~ ... (2)
l. 1. convex mirror.
Equating (1) and (2) U=-50cm, v=+10cm
:::::) n8 =1.3
Since.!+..!=!
V U f
107. [CJ 1 1 1
Focal length of curved mirrors is independent of the refractive => -+--=-
10 -50 I 0 · 'I
index of the medium in which the mirror is.placed.
1 5-1
=> i=so
108. [BJ i+-30 ,m_..:__30 cm~
C f = 50 14---50cm------------+1
n=- =>
v 4
=> f=12.5cm
Since R=2f
=> R=25 cm

114. [CJ
Since, 6,~. =(µ-1)A=(1.5-1)4°=2°
109. [DJ
1
·01 =1- an d sin
sin ·02 = -
µg µ..,.

Since, µg > µ .... , 01 < 02


The critical angle 8 between glass·water interface is given by
2'
sine=&
µ, 2'

=1.202

www.puucho.com
Rahul Sardana with www.puucho.com

Ray Optics

=> 6,,.,, =(µ-1)A+(180-2i)


=> 6,,.,, =(1.5-1)4°+(180-2x2°)
=> ()total =2°+176°=178° 121. [CJ
sinC = 4000
115. [CJ 6000
1
==> C=sin- (~)

122. [BJ
0 A virtual, erect image is obtained by using the mirror. The mirror
(I) µ can be both concave and convex. But a virtual image obtained
by a concave mirror is always magnified and hence the mirror
i.3 cm+1 / must be convex as we are getting diminished image.
><-a-::-"'<+---10 cm--><
123. [CJ
C f).alr 3
Ax=3[1-(i)f µ=-=--=-
vmad fA.me.d 2
:::::) AX=1Cm 1 V
Now, v=+Sm, m=--=-
==> Object distance = 1O+ 1 = 11 cm 3 u
=> U=-24 m
116. [CJ
Since, ! = .! __!_
Apparent depth = .'! f V U
n 1 1 1 4
2
=> -=-+-=-
f 8 24 24
Hence apparent separation = d
n => f=6 m
For a piano-convex lens
117. [BJ
According to Snell's Law for anisotropic medium f =_I'!_
µ-1
n,=1
n( 0 ) sin ( 90-0 ) = constant Interface
6 =_I!_
or n1 sini=n(8)cos8 0.5
=> (1)sln90 = n(0)ccs0 n(e) => R=3m

=> n(0)cos0 =1 124. [BJ


Since rays after passing through the glass slab just suffer lateral
118. [DJ displacement hence we have angle between the emergent rays
Velocity of approach of man towards the bicycle =(u-v) as a.
Hence velocity of approach of image towards bicycle is
2(u-v). 126. [AJ
As shown in figure the distance between the lenses should be
119. [CJ 30 cm.
Optical path length = µt
Time taken = µt
C

120. [CJ

Optic Axis
11 =20 cm fz= 10 cm

0 127. [CJ
Two piano convex lens of focal length f, when combined using

L optical glue will give rise to a convex lens of focal length ( ½) .


i.e. size of image = size of object.
=> Object at 2F i.e. at a distance f from optical centre.
Optic Axis
128. [BJ
0 ~Hon -+ OO
:,.--.1
:Ax: 1
Since - --=.!+..!-~
L
fcomblriatlon ~ i ~1z

www.puucho.com
Rahul Sardana with www.puucho.com

Advanced JEE Physics Optics & Modern Physics

134. [CJ
·0 =-1-+I __1_0_
-10 t (-10)t 4=Iµ ..
{ · µ
Real Depth }
Apparent Depth
o--1- + I + I
-10 t t and 6=d-t

D
µ
1 2
io = ~ => 6µ=d-4µ
=> d=10µ
t=20cm
=> d=15cm

129. [DJ 135. [AJ


Since, LABO= LOAB = C ', A ~,.,,,
Further,. by definition, we have i Cfi,.c._-+-,c~ PA=('')'.= 4 =0.44
', , Ps rA - 9
sine==.!= _g_ ' ,..,, /
µ 3
0 136. [CJ
Applying Snell's Law at A, we get An 'achromatic combination' is made from a concave tens and a
sini 3 convex lens with greater power of convex lens so as to make the
sinC =2 rays converge at a point.

=> sini = (¾)sinC = (¾)(~) = 1 137. [CJ

=> i =90° ·-. p A 0_••.

130. [CJ
·· ..r1 ··C
For combination to produce dispersion without deviation. µ
0
(n,-1)A,=(n,-1)A,
sini
=> A.2 =(~)A
n2-1 1 -_-=µ (atP) ... (1)
s1nr1
=> A =(1.54-1) 4• and r1 +C.=A ... (2)
' 1.72-1
=> sini=µsin(A-C)
=> A, =¾(4•) => sini=µ(sinAcosC-cosAsinC)

=> sini=µ(sinA~1- µ~ -cosA¾) {·: sinC=¾}


131. [CJ
=> i = sin-• { ()µ' -1 )sin A - cos A}
~+_!_= µ-1
co -u R
138. [DJ
u = ___!!_ from the surface At U=f (focal length}
µ-1
V -too
Total distance frOm centre
Al U=O {i.e .., object is at pole}
x=lul+R=R+~ v= O (image is also at pole}
µ-1
X=R+2R Satisfying these two conditions, only OPTION (D) is correct.
X=3R
139. [CJ
132. [DJ
An air bubble in water always behaves as a concave lens and
hence is always incapable to form a real image.

133. [BJ
Considering pole at P, we have
µ 1 µ-1
f+(-12) f+(-20)
-+-_=-
-2R 00 --R
f I
=> --=--- !'.= µ-1
f-12 1-20 =>
2
=> f-20=-f+12
=> 21 =32 => !'. = 1
2
=> f = 16 cm
=> µ=2

· 1.204 i;::::::::::::::::::::::::::::::::::::::::;:::;::::::::::::::::::::::::::::::::::::::::::::::;::::::::============::;:::::::::::;;:::;:;:::::::::::::::::::::::::::::::::::::::::::=

www.puucho.com
Rahul Sardana with www.puucho.com

Ray Optics
140. [CJ 143. [DJ
The first image is formed due to the reflection from concave The two slabs will shift the image by a distance
mirror M2•
1 1 2 6x=2(1~t}
=> -+--=-
v, (-2R) -R
1 1 4 1
::::::, -=--- 6x = 2(1---)c1.5) = 1 cm
1.5
.V1 2R 2A
Therefore, final image will be 1 cm above point P .
-3
144. [CJ
Since p
PRIIN,
=:- a=2i

m,=-[ !J=-¾ Also A+2a=180°


=> A+2(2i)=180
B T
=> i+4i::::180
::::> Radius of image circle= ¾a, (in magnitude).
=> 5i=180 u
Now the second image is formed by convex mirror M1 • => i::::36°
The second image will be formed because the image formed ~y => A= 36°
the first acts as object for it.
::::> Ob'Jeet d'rsance=
t 2R - 2R = 4R . 145. [CJ
3 3 S'mce tanus =t
h
1 1 1
-+------
v, (-:R)-(~) => f=-h-
tanO
f
2h-----f---)''-5),-;=F-
=>
1 2 3
-=-+-
Further ! ~----+ef,1
V2 R 4R => o=(µ-1)A
4R h
=> V2=11 => f=--
(µ-1)A
4R
m =-v2= -11 =~ 146. [A]
=> 2
u2 4R 11 If no lens had been there the rays would have met at A. On
3 inserting the lens the rays meet at B. Thus A acts as an object
for the lens.
=>
Since.]_ _ _!=!
V U f
3 a a
Sorad'1usof secon d'image= a2 = · = ;..-- 24 cm-----+1
11 3 11
Similarly radius of third ima9e is a3 =~
A
141. [BJ
r+i=90
i=90-r
For ray not to emerge from curved surface i- v -Iii
i>C ~-~.;..----,_ u=+24cm, v=?, f=+24cm
=> sini >sinC
::::> V=12 cm
=> sin(90-r)>sinC
::::::, Distance AB=OA-0B =24-12=12cm
=> cosr > sinC n
=> ~ >-
1 ·: sinC=¾) 147. [DJ
n

=> n2
2
1 _ sin i >_!_
n2
v-u=t I,
1 1
-----1===1 00 =>
1> 1;(1+sin2 i)
v-c-o.3) = 0.2
=> .fi
n => V=0.6m
=> n2 >1+sin2 i 0
A's OC1C2 and 01112 are _similar
=> n>,/2 {sin i---> 1) Hence
=> least value= J2 0.001 l1l2 I,
0.3 0.3+0.6 l+0.3m+!

:::::=================================== 1.205 =
www.puucho.com
Rahul Sardana with www.puucho.com

Advanced JEE Physics Optics & Modern Physics

11 acts as object for mirror M2 and gives an image 1; and


similarly 12 ' acts as object for mirror M1 and gives image 1;.
148. [DJ Since 11 is located at a distance 3b from M2 •
A convex mirror can never form a real image.
So, 1; must be at a distance 3b from M2 •
149. [AJ Similarly 1; must be at a distance 3b from ~.
Given, µ=-,/2, A=60°, 6=30° From above we observe that the first image is at a distance 2b
For the minimum deviation, we have from O.

sin(~) Second image is at a distance 4b from O and so on. So, nth


image is at a distance 2nb from O .
µ sin(i) 153. [CJ
Substituting the values of A and µ we get
1
8=2(2,) (4
1 )' (·.-e=Jut')
6m =30°
i.e., the given deviation 6 = 30° is actually the minimum a= .2;_
16
deviation Sm • At minimum deviation the ray inside the prism is
Reflected ray_ turns through an angle 20 =22.5°
parallel to the base of the prism (in case of an equilateral prism).
154. [CJ
151. [BJ
Distance of first image (I,) formed due to refraction from the
CASEI
plane surface of water is
d ::::: dreal = ..!Q_ =7.5 cm
"'' µ 4/3
Now distance of this image from the plane mirror is
0 x =5 + 7 :5 = 12.5 cm . Therefore, distance of second image
(1 2 ) from the mirror will also be equal to 12.5 cm from it.
Real Depth 4
n=
Apparent Depth 3
155. [DJ
CASE II
p a
n,
j'·· B p"·. •. E _.-0,\
.. 9 F
~./r2
A ·.f1 "·.· o· -{:r r/ .·
. 0 n, n,
R s T
Consider origin to be at the pole P.
o.=90-r, P=90-r2 y=90-r3
~+ "2 = n2-n1
-u V R

~ 1 1-(~) sini = n, sinr,

-(-4)+(-2:r_R_ =-
AIC
sin i =nf sin r1
2 2
... (1)

:::::, R=-25cm n1 sin( 90-r1 ) =llz sinr2


2 2
According to Lens Maker's Formula => n! sin r2 = n~ cos r, ... (2)

~=(n-1)(~ -~J At!l.


n2 sin( 90 - r2 ) = n3 sinr3

=> !=(i-1)(..!.
f 3
__
1)
ro-25
:;,. n! cos 2 r2 = n~ sin 2 r3 ... (3)

:::::, f=75cm

152. [CJ

l', I, tt O
·---:-2b___,:._b b.:b b..:_2b-+<
I, I',
0
=>
n, sin( 90 - r,) = ( 1)sin( 90-i)
2
cos 2 i = n~ cos r3
Adding (1 ), (2), (3) & (4)
=> 1+n!=nf+n~
... (4)

156. [AJ
M1 M2

11 is the direct image of O by mirror M1


P=(µ-1)(..!..-..!..J
R, R 2

12 is the direct image of O by mirror M2

= c::::===================================
1.206

www.puucho.com
Rahul Sardana with www.puucho.com

Ray Optics
158. [DJ 3 5
(90-r )+(90-C)+45 = 180 v"= 51-51
:::::,. r+C=45° 1 2
=> r=45-C v"=-51
Since sini = n V=--
51
sinr 2
sini = n sinr
PA'= 51
sini=n sin(45-C) 2
sini =n(sin45cosC- cos45sinC) OA'=~-21
2
sini= h"(cosC-sinC)
DA'=!_
2
f
DA' 2 3
m=-=-=-
OA .!. 2
3

162. [DJ
For a plane refracting surface, the lateral magnification is 1. So,
the image of the coin will be of the same size as the coin itself.

Further, sinC = ~ 163. [DJ


n 60
6=~= =30
:::::,. sini= ~[~1-~!-~ 1
]
:::::,.
2 2
The ray passes symmetrically through the prism parallel to
its base.
. . = ../2 ~
1 [ vn -n 1 -n 1]
srn1 Hence 0 = 0°

. .-,[.fnCnf-n,J
l=Sln .J2
164. [CJ

RAY2
159. [DJ
PA2
f, +f, =36 cm ... (1)
0: E
.l,_=5 ... (2) RAY1 'A PA1
f,
:::::,. fo=30cm and fe=6cm.
~ f2 -..l

160. [BJ ~ d ----+!


----x---+<
------f,------+I

1 2 1-2 For drawing the above ray diagram, we must consider two rays
v-(-15) = -10 RAY 1 and RAY 2 in such a manner that RAY 1 passes through
optical centre of first convex lens and is parallel to the Principal
1 2 1 Axis for the second convex lens (PA2).
-=---=--
V 10 15 30 So wherever the RAY 1 cuts PA2 is the focal length t of
:::::,. V=-30 cm
second convex lens. Similarly we have RAY 2 parallel to PA1
and let this pass through O' so as to cut PA1 at a distance
161. [CJ
Since an elongated image is equal to focal length ( i) for the first convex lens.
formed and it touches one 6.IDE and LiO'AE are similar
end of the rod, so the rod ,._..
f/3
must lie with one end at ... (1)
2F and other end between 0 A
A' ROD
2F and F (shown in figure). LilCB and AO'B are similar·
F p
51 /1-y (t +d)-x
For end A, u = -
3 /1 t
Since..!+.'!.=~

:::::,.
V
1 3
---=-
V 51
U I
1
-I
14-2f-J>t
1+-----1- fil ----f,I
3

================================== 1.207 =
www.puucho.com
Rahul Sardana with www.puucho.com

Advanced JEE Physics Optics & Modern Physics

171. [A]
... (2)

From (1) and (2)


f =U.:s- 1 )(-¾)
X d \-x =>
1 0.25 2
-=:--X-
i;-i; = \-d f' 1.75 R
=> f'=~(175)
. 2 25
=:- f' = 3.5R
172. [OJ
For refraction at plane Surface.
1 1.5 1.5-1
---+-=--
-(-mR) V oo
=:- v=-1.5 mR
Similarly we can calculate value of y .
For refraction at the curved surface
OBJECTIVE TRICK
If t:. = 0 , y must be zero (satisfied by A, 8, C) · u'=-(1.5mR+R), v'->oo

If d-> \+~, then x->"' (satisfied by B, C, D) 1 1.5 1-1.5


=:,
-(1.5 mR+R) -R
If d-> \, then x-> \ (satisfied by C, D) "'
So, we observe that OPTION (C) is .the only option satisfying -3
=:, 1
both the ,x and y in special situations mentioned above. (1.5 m+1)
=:, 3=1.5m+1
166. [CJ =:, 1.5m=2
Let d' be the diameter of refracted beam. Then
4
d = PQcos(60°) =:, m=-
3
and d' = PQcosr
d' cosr 174. [CJ
:::::,. -=---=2cosr
d cos60° Deviation by a sphere is 20-r)
:::::,. d' = 2dcosr Here, deviation s = 60° = 20- r)
:::::,. i-r=30°
=> r=i-30°=60°-30°=30°
AccOrding to Snell's Law, we have µ = s!ni
s1nr
sin60° ,::;3
µ=--=,~
sin30°

175. [AJ
Fa
. sini 2 1
Since, sm r = = _3_ = Fa µ
2
Also,. cosr = .J1-sin2 r
sinu =.!l
=> . cosr-=-J¾ , sinr n2

cc:,·· d' = (2)(2)JI =4JI cm sina = .!lsinr


n,
... (1)

:::::,. ct··~ 3.26 cm For TIR at other end


sinC= " 2
167. [AJ n,
1 1 1 X Also C=90-r
-=-+---
f I ~ I~
sinu =.!lsin(90-C) {Put r=90-C in(1))
n,
170. [AJ
1 1.5 1.5-1 . =:, sina.=~cosC
--+-=-- n,
-(-x) x R
2.5 0.5
x=R
X=5R

1.208 t:::=:=:::a========;:::=====:::::=========;:;::================::i
www.puucho.com
Rahul Sardana with www.puucho.com

.' Ray.Optics
176. [DJ 181 .. [BJ ·
CD is parallel to AB, so both m$dia--must .have equal refractive
indices. i - · __<!_= tan3O
0.2
177. [CJ d-O.2=_2_
Since all the prisms P , Q arid R are made of same material, - ,/3 10,/3
so the deviation suffered· by the combination remains the same Total number of reflections
as the combination forms a part of the bigger sphere. · I 2,/3
= n=-=---
178. [BJ
Let .R be the radius of cmvature of each surface. Then applying
d Ca~)
Lens Maker's Formula, we get => n=3O

1f =(1.5-1)(.!.R +.!.)
R 182. [CJ
=> R=f . Amu. =2C
For the water lens, again applying ttiB Lens Maker's Formula, We => A== 2(36') ' .
get

183. [CJ
2
=> f= 02-x2
I' 3f
Since, we know that for lenses placed in contact, we have 4D
1 1 1 1 => f (90)' -(20)'
·-=-+-+-
' F. I t t,. 4(90)
1 1 1 1 => f=21.4 cm
-=-+-+-
F f f I'
1 2 2 4 184. [CJ
F=T- 3t = st .
Smeeµ·=-.-
sin·i
smr

179. [BJ => i=45°


sini
sinr n 185. [CJ
Since
tanr= 2 h_=1
2h
=> r=45°
. • h ·1?
srnr = h.JS /
,!"
' ' 1 0 r<--2h---'l ,'; 0 0
srnr = .JS o.,g,;~==~~=
a
1 LOPCl = LOQP =0
1 .JS . => 30 =180'
;; =-1- => 8=60°
-./2
186. [CJ
n=J%

180. [CJ
2=(µ-1)·(-1.._...:!...)
f R1 R2
For no dispersion

d(D=O AB'= Apparent Depth= x


AB=1O cm
-
0 C' C

1 0cm

The face BC appear's to be shifted to B'C' . If x be the apparent


depth, then AB'=x and B'B=(1O-x).

================================== = 1,209

www.puucho.com
Rahul Sardana with www.puucho.com

Advanced JEE Physics Optics & Modern Physics

Also, according to Laws of Reflection an image is formed as far


m -9- . ( - f )'
behind the mirror as the object is in front of it ar - - f+u
=> 20+x=23.2+(10-x)
f
=> 2x=13.2 -=-3
f+u
=> x=6.6cm
{Since we get a real image)
. Real Depth
Smce µ= => _f_=-3
Apparent Depth f-40
10 => -3f+120=f
=> µ=-=1.51
6.6 => 4f=120
=> f=30 cm
187. [AJ
Since µ2t = µl 194. [DJ
Axial Magnification is given by
m = I along PA v'
aK O along PA u'
=> dX=t'-t

=> m., =¾=( f~J'

1
=> i = b ( - )'
f-u
188. [BJ
195. [BJ
By Laws of Reflection From the figure it -is clear that the angle between incident ray
i=r
and the emergent ray is 90° .
Also Emergent
Incident
i+r=90 ray
ray
=> i=45
According to Snell's Law
sini Air
µ=-.-
smr Glass

1_5 =!!_=sin45 =>


2 sinr

196. [BJ
189. [AJ
easel
Shift=i(1-;) away n = Real Depth =±
Apparent Depth 3

190. [BJ
According to Newton's Formula, we have
X1X2 =f
=> (10)(40)=f
=> f2 =400 Case II
=> f=20cm

191. [BJ
A=60°
i=55°' 8=46° 0
Since i+e=A+D Consider origin to be at the pole P.
=> 55+46=60+D ..!:!!.+ "2 = n2-"1
=> D =41° -u V R
So, Dm <D ±
_3_+_1_= _ _3_
1-(±)
192. [AJ -(--4) (- 2:) R
At D =D.
R=-25cm
i=e
According to Lens Maker's Formula
193. [AJ
Areal Magnification = 9 ~=(n-1i(~-~)

= ==================================
1.210

www.puucho.com
Rahul Sardana with www.puucho.com

Ray Optics

=>
1)
!=(±-1)(1-- magnification unity the objeci must be at 2F(or C) and so
f 3 oo-25 image is also formed at 2F ( or C) . H€nce u =-2f = -40 cm
=> f=75cm
203. [DJ
197. [DJ The lens will converge the rays at its focus i.e., 30 cm from the
According to Lens Maker's Formula lens or 20 cm from the refracting surface, so we have
!f =(µ -1i(J..
R R
__!_) PI 1 =20 cm
1 2 3
Now, PI,= µ(PI,) = x20 = 30 cm
=>
1
foc- - 2
(µ-1) ~ 30 cm -----+1
Since µRed < µ~"lolel
=> ted > (.,.,let
=> ted > t1ue
Always keep in mind that whenever you are asked to compare
{greater than or less than) u, v or f· you must not apply sign
conventions for comparison.

198. [DJ 10cm


Hence, the rays will converge at a distance of 40 cm from the
Since, the refractive index is increasing linearly from top to the lens.
bottom, so the light cannot travel in a straight line in the liquid as
shown in options (A) and (B).
204. [CJ
Initially it will bend toward_s normal and after reflecting from the
bottom it will bend away from the normal as shown below. !=(µ-1)(1-J...)
f R -R
! !
i Rarer Rarer i
! j => ~-(µ-1)(¾) ... (1)

II Denser
On cutting
Denser
1=(µ-1)(1-1)
r R "'
199. [AJ => f=2f
U=-(f+X)
205. [CJ
f When a lens is cut parallel to principal axis its focal length
m=--
f+u remains the same. Hence each part will have a focal length 2f.
f
=> m=---
f-f-x
f
=> m=--
x
Negative sign confirms that real magnification is negative.

200. [DJ
For a convex mirror, both (A) and (B) are incorrect.
206. [BJ
201. [DJ
According to Lens Maker's Formula
U=-{f+x,)
.1__.!_ =(µ-1)(1._J...)
v=(f+x,) v u R1 R2

Since..:!._..:!_=~ The lens is made of two materials, so for a single object


V U f distance, two different image distances are obtained i.e. two
images are formed.
1 1 1
=> --+--=-
f+X2 f+X 1 f 208. [BJ
=> 1(2f+x,+x,)=f+(x,+x,)f+x,x, Let ~ and t be the focal lengths of the lenses of refractive
~ f =X1X2 indices µ 1 and µ 2 in water, then

~ ·f=~X1X2

202. [CJ
Two piano-convex lenses of focal length f on combining give a ... (1)

convex lens of focal length -½. To obtain a real image of

r::-:================================= 1.211 =
www.puucho.com
Rahul Sardana with www.puucho.com

Advanced JEE Physics Optics & Modern Physics

... (2)

Agding equations (1) and (2), we get


1 1 2(µ,-µ,)
-+-= ::::::, ill<= 1 cm
\ t µ.R
Object distance = 1O+ 1= 11 cm
=>
=>
...!.. = 2(µ, -µ,)
30 µ.R

=> (µ1-µ2 )
µ.R
=so _.:....... -··
Substituting the .values, we get
.. -···
0 C
4x15 1 (I) µ
(µ,-µ,)= 3x60 =3
14-3 cm+i
209. [AJ <--><<--1 Oc m ~
Since image is formed on the screen, hence it must be real. So, t,x
Lens used must be convex {and canhot be concave)
u=-45cm, V=+90cm 214. [CJ
Since, for a lens
1 1 1
v-u=t
1 1 1
=> -+-=-
90 45 f
=> f=+30 cm 215. [CJ
Further again the positive sign with focal length indicates the A
lens is convex. ·

210. [BJ
a
Size of Image =.":(Size of Obje.ct) o· ih
u
B
=> l=~(5)=-"10cm
-45· .
Here again the negative sign with indicates that it is f0rmed u=40cm v=40cm
below origin i.e. inverted i.e. real.
f=20 cm, u=--40 cm
211. [CJ 1 1 1 .
~+~= µ2-µ1 ,=v-u gives.
-µ V R
1 1 1 1 1
1 1.5 1.5-1 -=-+-=---
vfu2040
=>
-(-50) + v
~ 20 => v=40cm
=> ...!..+12= 0.5 The situation is shown in figure.
50 V 20 From similar triangles PAI and IQB
1.5 1 · 5 40
=> v=4o-5o h 20
1.5 10
=>

=> v = 15x200 =+300 cm


10 216. [CJ
Positive side indicates that image is real. Since 0 2 = 1112
=> 4 = (0.5)!,
212. [CJ
=> l 2 =8cm
r2 =Oo
=> r1 =A=30° 217. [CJ
and i1 =60° According to Newton's Formula
So, from Snell's Law, we get x1x2 =f
µ = sini1 = sin60° = .Js => f=.J16x25 =20 cm
sin~ sin30° .

= ====================================
1.212

www.puucho.com
Rahul Sardana with www.puucho.com

Ray Optics
218. [AJ 222. [DJ
For real image

cos(1B0'-20)
(~)-(~) U=-U1, V=-2U1, f=-20cm

1i+~11i-;13]1 !,
Substituting in ..:!. + .:!. = we get

'' ----=--
-2U U 20
1
11
V U f

1
'
''' => u, =30 cm
=2u2 , f =-20 cm
'''
For virtual image u = -u 2 , v

'
0 ''0
. I . 1 1 1
Aga1nappy1ng -+-=-,we get
V U f
1 1 1
... ' 180'--20. 2U2 U2 20
''
'' => u2 =1Dcm
''
(1-3) ' So, the distance between two positions of the object is u1 - u2
=> u1 -u2 =30 cm-10 cm=20 cm
=> -cos20=-4-
1
-1 223. (AJ
=> -cos20=-
2 1 1 1
-=-+-
f 40 -25
=> cos20=..!
2 f= -40x25
=>
=> 20 =60' 40-25
=> 9= 30° => f =-66.67 cm
p 10
219. [BJ 1.5 D
f(in cm)
!f =(1.5 -1)(..!. _.!.) + (1.2-1)(-oo1--14
14 oo
1
--)
224. [AJ
=> f=20 cm P=P,+P2
=> As object distance = 40 cm => P=10D
=> Image distance = 40 cm 1
f=p=0.1 m
220. [AJ
=> f=10cm
Focal Length of Lens is f1 =20 cm .
Focal length of combination is 225. [BJ
fc =20+5=25 cm Object is placed at a distance of 2f from the lens of focal length
when the space between lens and mirror is filled with a liquid f i.e., the image formed by the lens will be at a distance of 2f
then or 20 cm from the lens. So, if the concave mirror is placed in
this position, the first image will be formed at its pole and it will

MO••~,''°~., oo,_, ~, •• OOj)


1 1 reflect all the rays symmetrically to other side as shown below
- =-+-
f' I, f
1 1 1
-=-+-
25 f, 20
1 1 1
=>
f, 25 20 o I
ft=-100cm
For this liquid lens
I)
14--- 20 cm ---+I+- 20 cm ---....i
f,=(µ-1)(~ -~)
226. [DJ
=> __1_={µ-1)(-1__
100 -33 oo
.!.) Shift=d=t(1-¾)
=> µ-1=0.33
=> l=~
4 n-1
=> µ=1.33=3
227. [AJ
Focal length of piano convex lens using Lens Maker's Formula is

====================================1.213 =
www.puucho.com
Rahul Sardana with www.puucho.com

Advanced JEE Physics Optics & Modern Physics

7=(¾- 1)C~ -~)


235. [BJ

:::::> f=20 cm
If point object O is placed at a distance of 20 cm from the piano
convex lens rays become parallel and final image is formed at
second focus or 20 cm from concave lens which is independent
of y. 0

228. [DJ
Focal length remains unchanged.
I oc Area of Aperture
Since angles opposite to equal sides are equal. So, in MOB
I =kD'
We have LABO =p . In .6.CAB , external angle equals the sum
of internal opposite angles. So,
D=(a-P)+(a-p)

=> I'= 31 D=2(a-p)


4
236. [CJ
229. [BJ When magnification is m = +2 , then we have
U=-X
1'=~
f V=-2X
In moving from air to glass, f remains unchanged while· v f =+20
decreases. Hence, A should decrease.
Applying Mirror Formula, .!.V - .!.U =!1 , we get
230. [BJ
1 1 1
-+-=-
f=--x- (See 'Displacement Method' in SYNOPSIS) -2x X 20
m1 -m2
::::,. x=10cm
To have a magnification of m' = -2, we have·
231. [BJ U=-y
02 -x2
f=-- V=+2y
4D
and f =+20
f= (0.9)' -(0.2)' 1 1 1
=> => -+-=-
4(0.9) 2y y 20
f=0.214 m
=> y=30cm
1=21.4 cm So, the object has lo be moved by
y-x=20cm
232. [CJ
When 8 < C partial transmission and reflection will occur. When
237. [DJ
8 > C , only ieflection takes place. When water is filled in the mirror, a piano convex lens is formed,
so now combination contains a piano-convex lens and a mirror.
234. [BJ The effective focal length of combination is less than the focal
1 2 1 length of above mirror, so image is shifted downwards.
-=-+-
F f, I,,
239. [DJ
1 2 1 The given lens is a convex lens. Let the magnification be m,
;:::} -=.-+-
~ ft 00 then for real image, we have
1 1 1
:::::> F=!t_ -+-=- ... (1)
2 mx x f
Also and for virtual image, we have
1 1 1
.!. =(µ-1)(J_ _ _!_) --+-=- .. ,(2)
t --f 00 -my y f
-r From equation (1) and (2), we get
ft=--
µ-1 I= x+y
2
=> F=---r-
2(µ-1) 241. [BJ
Negative sign indicates that mirror is concave. Let 11 , 12 and 13 be the images formed due to
(i) refraction from ABC
(ii) reflection from DEF and
(iii) again refraction from ABC

-...1.214::=:===================================

www.puucho.com
Rahul Sardana with www.puucho.com

Ray Optics

A D => I,, =180 cm


1 2 2 1
=> -=-+-+-
0 F 60 180 10
B E 1 13
=> -=-cm
F 90
90
C F => F= cm
13
14-5 cm •u •1
2.5cm So, required answer to get asked condition is 2F. Hence
180
Then BI,= (5)µ 9 = (5)(1.5) = 7.5 cm u=-cm
13
Now EI, =(7.5+2.5)=10 cm
246. [CJ
=> EI2 =1O cm behind the mirror For equiconvex lens, we have
Now BI, =(10+2.5)=12.5 cm IR,l=IR,1=1=10 cm

BI = 12.5 = 12.5 = 25 cm Now P=2P, +P•


=> 3
µg 1.5 3
=> _.! = 2(µ-1)(J__...!_)-~
F R1 R2 R2
242. [CJ
02 -x2
f=-- (By Displacement Method)
=> _.! = 2(1.5-1)(J_--
F
1
10 -10 -10
-J-~
4D => F=-2.5cm ,
Therefore, the system will behave like a concave mirror of focal
=> P=~ length 2.5 cm.
02 -x2

=> P=~ 247. [BJ


1-0.16
1 2 1
p =__±__
-=-+-
=> F f, i,,
0.84
Since t, ~ co
=> P=4.76D
=> F=i=15 cm
243. [AJ . 2

Shift=d(1-¾J=1(1- 1 ~J Further
1 1 1
=> Shift= 2 cm (downwards) v-u=F
1 1 1
244. [AJ => -;;- (-40) =15
1 2 1
-=-+-
F f, i,, =>
V 15 40
=> V=24 cm
.!= (µ-1)(J__...!..)
t R1 Rz
248. [DJ
=>
1 · (1 '1J
:r,=(1. 5 -l) 20-60 Applying µ 2 _ _& = µ 2 - µ, , we get
V u R
=> t =60 cm and 1 1.5 1-(1.5)
20 -;;- (-u) = ~
t,=-=10cm obje~t
2 1 3 1
=> -+-=-
1 2 1 v 2u 2R
=> -=-+-
F 60 10 For v to be positive, we have
=> F=7.frcm 1 3
->-
For image to be formed at the same place where object is 2R 2u
situated we have u = 2F = 15 cm => U>3R

245. [DJ 249; [BJ


1 2 2 1
-=-+-+- ~inaUon -), OO
Ff, I,, i,,
1
(Because here we have two refractions at the concave surface Since - --=.:!.+.!-~
of tens and two refraction for water lens). Since fcomblna1ion ~ ~ ~~

t=(t- X6~ -±J .


1
=> 0=-1-+.! __1_0_
-10 ~ (-10)~

==================================== = 1.215

www.puucho.com
Rahul Sardana with www.puucho.com

Advanced JEE Physics Optics & Modern Physics

254. [DJ
=> 0=-1-+.!.+.!.
-10 t t . f
Smce m=--
f+u
1 2
=>

=>
10=1
t=20cm
=> -1= ,:u
==> -f-U=f
==> U=-2f
250. [BJ When the lens is cut, the focal length of the remaining portion is
f f 21.
f+(-12) f+(-20) Applying Lens formula, we get
f-12=-(f-20) 1 1 1
f-12=-1+20 .-(-21) =21
21=32 ==> V--",-oo
f=16cm
255. [BJ
251. [CJ 1 ( R,
1 -R,
1)
10=(1.5-1)
5=(o.5>(~ -~)
=:- ... (1)
=> _!_ _ _!_=10
R1 R2 On immersing it in a liquid of refractive index 3
-1=('µ,-1)10 ~=( 'µ, -1)( ~ -:,)
=> (::-1]=-0.1
~=(\5-1)(¾)
=> ~=0.9
µ, =>
f 10
=> f=-10 cm
=>
Negative sign indicates diverging nature of lens in the liquid.

252. [DJ 256. [CJ


lul+lvl=D Since, we have, at time t
V d = y+ µ(h-y) = µh-(µ-1jy
m=-
u
=> v=mu
lul+mlul=D
f
h-y
D
=> l u l = - !
m+1
f
=> U=-(m~1) r
Since image formed is real, so it must be on positive side. Hence
lvl=+v= mD If A is the area of the tank, then we have
m+1
at
For a lens y =,;:_ {·: Ay = at}
1 1 1
v-u=t => d,;µh-(µ-1)at
A
m+1 m+1 1
=> --+--=- i.e., d- t graph is a straight line with negative slope and
mD D f
positive intercept. But d becomes constant once y =H.
=> f=~
(m+1)'
257. [BJ
On immersing in water f increases and hence P decreases.
253. [AJ
1
-=(1.5-1) (-
1--
1) 258. [DJ
f R eo 1 1
!=(1.5-1)(- - -- )
f 0.5 -0.5
=> _!_ = 0.5 .!_

=>
16
R=8 cm
R
=> !f = o.s(..3..)
0.5
=> P=2 D

= ==================================
1.216

www.puucho.com
Rahul Sardana with www.puucho.com

Ray Optics
259. [AJ 266. [BJ
0=$i; Rays from O must fall normally on the mirror, only then the lens
forms a virtual image at C (the centre of curvature).
0='14x16 =8 cm C
260. [A]
For a lens making real image f
36cm
I
-m=-
l+u
=> -mf-mu=f
=> --mu=l(1+m)

=> U=-{1+¾)

=> lul={1+¾)
1 1 1
For lul to be MINIMUM m must be MAXIMUM i.e. m-, oo => -+
X -(36-12) 40
=> lulmin = f => x=15cm
For lul to be MAXIMUM m must be MINIMUM i.e. m = 1
=> lul.,. = 21 267. [DJ
1 1 1
261. [A] F= (1/2) + (1/3) =5
The ray diagram is as shown in figure => F=0.2 m=20 cm
A Since
1
V u F
0 C 1 1
=> ,-(-30) = 20
=> V=60 cm
- - - 21 _,...;.___ 21 ____,..
268. [BJ
Since triangles CAI and NEI are similar, so we have
Pcomb =P1 +P2 -xPf2
h I
d/2 = 21 0=.!.+..!-~
2h I ~ I~
=> ct= 2 => -X=I+~
=> h=~ 270. [DJ
4
Apparent Separation = 2(Apparent Depth)
265. [AJ
If mirror would have been absent then image is formed on the => Apparent Separation = 2h
µ
other side of lens ( at I').
So 271. [AJ
For distant vision
V -20 15 u = --00
=> v=60cm
=> -2-:;-=7
=> l=-2 m
=> P=-0.5 D
For near vision
u=-D=-25 cm
1 1 1
=> -1- -o.25=f
=> P=+3 D
Hence he must use bifocal lenses with
Since the mirror reflects the ray back, so O' serves as a virtual P = -0.5 D and additional+ 3.5 D
object and forms a real image I in front of mirror. (to give +3 D net)
1 1 1
-+-=-
v -60 15 273. [CJ
=> v=+12cm In the first case. Let x be the distance of object from the mirror.
Then

r::::=================================== 1.211=

www.puucho.com
Rahul Sardana with www.puucho.com

Advanced JEE Physics Optics & Modern Physics

u=-x, v=+2x and f=-f 277. [CJ


. 1 1 1 . vv1025
Usrng v+u=f,weget m=m0 xm =-9..x---2.=-x-
8 U0U82,55
1 1 1 => m=20
2x -x=-1
f 278. [BJ
=> X=- 1
2 (Shift),= 1(1-~J =9(1---) =3 cm
In the second case, let y be the distance of object from the µ, 3/2
mirror. Then
u=-Y, v=-2y and f=-f (Shift),= 1(1- ~:) = 9(1-%) = 1 cm
1 1 1 So, distance between two images is given by
=> -2y -y=-, (Shift), -(Shift), =2 cm
3
v=-t
2 279. [AJ
So, object will have to be moved by a distance of y - x = f .
m = m0 x ffi 8 =m 0 ( 1+{'J
274. [AJ
1=-5
t
Separation =t + t
36 =--4t
=> fa=-9cm
280. [CJ
=> t=45cm
m=1+~
f
275. [DJ
For lenses placed in contact, we have
1 1 1
-=-+-
F \ t 281. [CJ
J=(µ,-1l(±+i)+(µ, -1)(~ -±) f
m=...!!..=10
t,
.!_=µ,-µ2 => fo=50cm
F R
Separation = t +f
8
= 55 cm
F=_R_
µ, -µ2
282. [AJ ,

276. [BJ m=1=5


V8 =-D=-25 cm, t =6.25 cm. t
Since 283. [A]
1 1 1 Separation= t + t
-·=-=--+-
t Ve Ue
Separation= 0.3 + 0.05 = 0.35 m

285. [D]
1 1 1 For an equilateral prism, we have
u, =6.25- (-25) A=60°
Since, the ray inside the prism is parallel to its base, so we have
U8 =5Cm the condition of minimum deviation. So, i = e = 60° and
=> V0 =15-5=10 Cl!1 6m = i+ e-A = (60°+ 60°)-60° = 60°
Also, t =2 cm and sin(A+6m)
1 1 1 Since, µ = 2
-=---
t Vo Ua sinrn)

2 10 U0
=> µ =sin(60') =-J3
sin(30')
=> u0 =-2.5 cm

= ====================================
1.218

www.puucho.com
Rahul Sardana with www.puucho.com

Ray Optics
286. [AJ .
-1)(_!__...!.) =0
f must be maximum
::::,.
.!_ = (µ,
\
\-too
R, R,
{·: R,=R,}

289. [BJ Similarly, t -> oo


. .. . -2 cm So, a hollDw, convex lens of any material will behave like a glass
Lmear magn111cat1on m = - - = -2 plate.
1 cm
So, the image is real and inverted
295. [Dj
lvl=2lul The ray diagram is as shown below
Let lul = x then lvl = 2x .Y
Now lul+lvl=S0-(-40)=90
::::,. x+2x=90·
::::,. x=30cm
So, the distance of object from the lens is 30 cm and of object
is 60 cm, i.e., the lens mt.ist be located at x = -10 cm as
shown in figure.
0
T x=+50cm
x=-40cm X=O
I
•r
. a
S ,nee, x =../2

-
30cm
and Y= ../2
1+---c+<+--60 cm-Jo-1

1 0cm P~c;, 1)
292. [DJ 297. [CJ
Hollow,convex lens is as shown-in figure. Applying Lens Maker's
Formula, we get - 1-16 1-6
=> -1+16=1-6
=> 21=22
::::) f=11 cm

0 =>

~ f1
+

. f2
Hollow glass lens

r::::=================================== 1.219=

www.puucho.com
Rahul Sardana with www.puucho.com

1. [B, CJ µsini·= sinr


The.normal to the interface is along k. => r = sin·' (µsini)
. A-(-1<) +10 . 1
=> 3=sin·'(µsini)-i
COSI= IAlikl =20=+2 ••. (1)
This is a non-linear variation of 6 with i • Also,. we have 6 to be
=> i= 60°
(i is the angle which incident ray makes with maximum when i =·c and hence r =~.So
. 2
-Z-axi~}
Z<O Z=O 6- =6,=~-C
2 ... (2)
Z>O
FIJrther When- i > C ,-then TIA takes place and·thEI incident ray is
reflected back in the denser medium as shown in the tiQure .
•• i .......... .
n"'k :' , /
'L,' ~6=it-2i·
,/

But according to Snell's Law


,/2 sin60 =,/3 sinr
Sp, 6=rc-·2i
=> sinr = v'2 ,/3
,/3 2 • i.e., 6 decreases linearly with i. So,
. 1 6ma.; =02 ·=rc-2C •.. (3) •
s,nr = v'2 From.(2) arid (3), we get
=> r =45° 62 =261
3. [A, DJ 4. [A, BJ
Combl~ed Solution to 2 & 3 sin C = µram, =..&_
For i < C, ·no TIA will take place, so we have deviation (8) µdenier µd
given-by i+r' =90°
6=r-i According to Snelrs Law Reflected Ray
µd sini = µr sin(r')

=> ~=..&.=sine ·o(mser


sin(r') µ, Riirer
sini
=> sinC Refracted Ray
sin(90-i)
µ
=> sin~= tani
.=> C = sin·' ( tani)
=> C = sin·' ( tanr) {Be~use i=r}
Now, according to Snell's Law, we have

= t:::==================================
1.220

www.puucho.com
Rahul Sardana with www.puucho.com

Ray Optics
6. [A, CJ Also at m=O, v=a
According to Snell's Law, we have => O=f-a
~A,= µ2A2 => f=a
=> (1)(6000) = (1.5)).,
14. [A, B, C, DJ
=> i., = 4000 A All are the consequences of the
Since frequency does not change when light goes frOm one "DISPLACEMENT METHOD TO FIND FOCAL LENGTH OF A
medium to another, so· CONVEX LENS"
C
V=-
A, 15. [A, B, C, DJ
8
Objective and eye piece are separated by a distance
3x10
V = 6QQQx1Q-l0
5x1014 Hz (t+t)=16.02m {OPTION {A)}

Angular Magnification= _1_ = - ~ = -800 {OPTION (B)}


9. [B, C, DJ f, 0,02
See Displacement Method. A telescope produces an image which is always inverted.
(OPTION (C)}
12. [A, DJ In a telescope an objective is larger than the eye piece.
Since images are formed at the same place, So one image must {OPTION (D)}
be real and other must be virtual.
lxl+IYl=32 16. [B, DJ
p 1 1 1
Q' -=-+-
f -15 30
t => f = -30 cm (Diverging in nature) {OPTION (D)}
,i,' Since red deviates the least and violet deviates the maximum.
P' So a coloured pattern with red on the outerside is observed.
X----+t+-Y ~
i+- (32-x) --.i 17. [A, C, DJ
ForP Optical path length in passing from 1st medium is r1is1 •
1 Optical path length in passing from 2nd medium is n2:52 and so
y -x 15 on,
1 1 1
=> -+-=- ,,,(1) So total optical path length= I;11s,
_., {OPTION (C)}
y X 15
ForQ . 1 m
~ Total time of flight t = - Ln;s1 {OPTION (A)}
1 1 1 C 1~1
-y -(32-x) 15 For inhomogeneous media optical path length is
B
1
=> - + - - = -
1
-y 32-x 15
1
.,,(2) f
OPL= n(s)ds and the ray must travel along a path in which
A
Adding (1) and (2), we get time taken to go from A to B is minimum. Such paths are called
1 1 2 stationary pathways and this is the statement of Fermat's Least
-+--=-
x 32-x 15 Action Principle or Fermat's Principle of Least Time.
32 2
18. [A, DJ
x(32-x) =:is The final image is formed at infinity when the combined focal
=> 32x-x' =240 length of the two lenses (in contact) is 30 cm i.e.,
=> x' -32x +240 =0 1 1 1
-=-+-
x2 -20x-12x +240 = 0 30 20 f
=>
f=-60cm
=> x(x-20)-12(x-20) = 0
So, when another concave lens of focal length 60 cm is kept in
=> (x-12)(x-20)=0 contact with the first lens.
=> x=12cm,20cm Similarly, if µ be the refractive index of a liquid in which focal
length of the given convex tens becomes 30 cm. Then from Lens
13. [B, CJ Maker's Formula, we have
f-v
m=-- 2~=(¾-1)(~,-;J ... (1)
f
m=1-~ ... (2)
f 3~=(3~2-1)(;, - ; J
1 b From equations (1) and (2), we get
=> Slope=--=-
! C

=> f ="-b
9
µ=a

c::::=================================== = 1.221

www.puucho.com
Rahul Sardana with www.puucho.com

Advanced JEE Physics Optics & Modern Physics

19. [B, DJ
When upper half of the lens is covered, image is formed by the =>
rays coming from lower half of the lens i.e., the image will be
formed by lesser number of rays. Therefore, intensity of image =>
will decrease. However, complete image will be formed.
When the Image is real, then
20. [A, CJ u=-y,so v=-2y
m = +2 , means image is virtual, erect and magnified. A virtual Again applying the mirror formula, we get
and magnified image can be formed only by a concave mirror 1 1 1
and that too when object lies between pole and focus. -2y -y=--,
21. [B, C, DJ =>
When passing from vacuum to a medium, frequency remains
unchanged while speed and wavelength decreases µ times.
=>
22. [A, B, CJ
For convex mirror (having positive focal length) the image is
30. [A, CJ
always smaller in size. For concave mirror (having negative focal When the object moves from infinity to the pole of the mirror, the
length) the image is smaller when the object Iles beyond 2f . virtual image moves from focus to the pole.

23. [A, BJ 31. [B, DJ


Since, for refraction at a plane surface, we have
.&=µ2 ... (1)
U V
If x be the height of the bird above the water surface, then for
the light travelling from the bird to the fish, we have
µ1 =1, µ 2 =µ and u=-x
:' So, from (1 ), we get

''' 1
(-x)=v
µ

'' :::::, V=-µx


l+- f1 - - - -
- - - - 21, - - - - - => lvl = µx
Now speed of the bird is dx
dt
24. [B, CJ So, apparent speed of the bird is
Th~ tube length of an astronomical telescope, in normal
adjustment, is (f0 + ~) and that of Galilean telescope, in normal l~:I=µ ~~
adjustment is (t-t) where t and t are focal lengths of
objective and eye piece respectively. In this case, f0 = f, so
: : :, 1~;1> ~; {·: µ>1}

difference in tube lengths is 32. [A, C, DJ


1,-c, =(t +t)-(t-t,)=2t =21 TIR takes place when ray of light traveis from denser to rarer
medium.
27. [B, CJ
A concave mirror and a convex lens_give virtual magnified.image
Further, sinC12 =&..
µ1
and sinC13 :12
µ1
for a particular object position (i.e. when object lies between F
and P (or Cl) Since, µ2 > µ3
µ, µ1
28. [B, DJ c,2 > C13
A concave mirror can give both real and virtual magnified
Smaller the value of critical angle, more are the chances of TIR.
images. Since nothing is specified,,so
m=±3
33. [A, CJ
=> ±3 = ~
-15-u sin(~)
Since, µ
+u=-10cm .A
sm-
-u= -20 cm 2
For µ =,/2 and A = 60' , we get
29. [C, DJ
When the image is virtual, then om= 30°
U=-X, SO V=+2X Further, at minimum deviation, we have
From the mirror formula, we get A
r, =r2 = =30°
1 1 1
-+-=-
2
v u I Applying Snell's Law. we get

= 1.222 .::::=====================================
www.puucho.com
Rahul Sardana with www.puucho.com

Ray Optics
.,,, sini1 = µsinr1
l+t=-1.Q=~
16 8
=> sini, =(v'2)sin(30°)= ~ Now, from (2) we get
=> i1 =45° 1(~-1)=_1_
8 16
34. [B, CJ => 16\'-10\+1=0
Using µ 2 -&= µ 2 -µ, we get => 16\'-8\-2\+1=0
V I u R '
=> 8\(2\-1)-1(2\-1) = 0
1.5 1 1.5-1
---=-- => (8\-1)(2\-1)=0
V oo 20
;::::, v=+60cm 1
=> \=.!. or \ =-
Since v is positive, the rays actually meet. 8 2
=> P, =8D or P1 =2D
35. [BJ
In air, !=(µ-1)(_1_ _ _1_) ... (1) 37. [A, DJ
f R, R2
Since, .&. + µ 2 = µ 2 -µ,
µ -u R
V
When immersed in a liquid of refractive index
2' where µ 1 =µ, µ 2 =1, u=-R, R=-A

f=C~2- X~. -:J


1

=> ¾=(~. -:J ... (2)


V R
From (1) and (2), we get V=R
1 µ-1
1=-1- 38. [A, DJ
Real image is smaller in size if object lies beyond 2f and it is
=> l=µ-1 larger if object lies between f and 2f .
f
When immersed in a liquid of refractive index 2µ , 39. [A, CJ

i=(:µ -1)(~, -~J A ray can pass undeviated when µ1 = µ 2 or the ray is incident
normally i.e., angle of incidence is 0°.

=> i=-¾(~.-:J .. (3) 40. [C, DJ


For TIA, we have
From (1) and (3), we get i>C
=> sini > sinC
~=(µ-1l(-f)
=:i, sin(45°) > sinC
'=-2(µ-1) Since, sine=..:!.
=> • f
n
1 1
36. [BJ v'2 >,;
When in contact, we have
1 1 1 n > F2
-=-+- n>1.414
F I ~

10=.!+.!.
I t (·.- P=i=10l 41. [A,C]
Applying Lens Maker's Formula, we get
1+~=10\t
When at a separation of 0.25 m, we have
1 1 1 X
... (1)
f~, =(¾-1)(~, -~J
-=-+--- ·1 -(3/2 1)( RI1- R21)
F' I t It \.atar - 4/3 -
6=.!+.!.- 0.25 From these two equations, we get
I ~ I~ \yatar = 4falr = 4f
In air, the image was inverted, real and magnified i.e., the object
6=10-0.25 must be lying between f and 2f . Now the focal length has
It changed to 4f . Therefore, the object now lies between pole and
1 focus and so the new image formed will be virtual and magnified .
It = 16 ... (2)
From (1), we get

==================================== = 1.223

www.puucho.com
Rahul Sardana with www.puucho.com

Advanced JEE Physics Optics & Modem Physics


43. [CJ 45. [B, CJ ,
Let t be the thickness of the watch glass, R be the radi1:1s of A concave mirror can give a virtual image (object lies between F
curvature at inner surface, so (R + t) is radius of curvature of and P) but the image is always magnified and a convex mirror
can never give a real image.
outer surface. If µ be the refractive index of this thin lens (watch
glass), from Lens Maker's Formula, we get 46. [A, B, CJ
Since, the lights used are of different colours, so they have
different frequencies and hence (8) is also correct.
R 47. [B, CJ
!f = (µ -1)(-1- - ...!.)
-2R --R

=> !f = (µ -1)(.!R - ...!.)


2R
-= µ-1
' 2R
.! _.! = ! = (µ -1J(J_ _...!.) .f= 2R
vuf R1 R2 µ-1
The focal length of lens has nothing to do with the direction frOm
.!V = !f = (µ-1)[...!. - _1_]
--R -(R+t) which the light is incident" on it.

l=(µ-1)(-1__ .!)=- (µ-1)t <0 49. [C, DJ


f R+t R R(R+t) For all object positions a convex mirror (or a concave lens) forms
So, the watch glass will have a diverging nature. a virtual and erect image.

44. [A, BJ 53, [B, CJ


For a concave mirror, when object is placed at 2F (or C) real
image of the same size as that object is formed at 2F (or C).
-f
::::) -f-{-2f)
mamcava

For convex mirror


____ ....................... i ::::) m =---=-
f 1
="' 1-(-21) 3
''
'
'''
!+- d-i---2f2----+:'
f, -----+<

= ==================================
1.224

www.puucho.com
Rahul Sardana with www.puucho.com

1. [DJ 11. [AJ

Lateral displace~ent, Ax= tsin{ 1-t) < t . According to Lens Maker's Formula,
!= (µ-1)(J_ _ _!_)
f R1 R2
2. [BJ Since, R1 = R2
Both Statements are true but Statement-2 is not the correct
explanation to Statement-1.
R, R,
6. [CJ 1
So, power =t=O

Y, 16. [BJ
When light goes from one medium to another, its frequency

i 19.
remains unchanged.

[A]
Both the Statements are true, and Statement-2 is the correct
explanation to Statement-1.

L 20. [CJ
Statement-1 is true & Statement-2 is false.

3 3
4
=> Y1/M =Y, + (Y, -y,) ~ Y,
3
3
But Y,ia =2[ :• +(y,-y1)]

=> dy,/8 =2dY2 =2V 21. [BJ


di di O
A=60°

7. [DJ
µ=-12
When the object is virtual, a real image can be formed by a i = A+ Dm = 60 + 30 = 450
plane or convex mirror. 2 2
A
8. [BJ Since, r. = - = 30°
Both Statements are correct but Statement-2 is not correct 2
explanation of Statement-1. So, According to Snell's Law, we have
1
10. [DJ sini = sin(45°) = -12 = -12
For a mirror, m = _f_
sinr sin(30°) .!.
f-u 2
Both Statement-1 and Statement-2 are true but Statement-2 is
not the correct explanation of Statement-1.

====================================· 1.22s=

www.puucho.com
Rahul Sardana with www.puucho.com

Advanced JEE Physics· Optics & Modern Physics

22.
Both Statements.~re t~ue and Statement-2 ·1s correct expIanat·ion
[AJ
· to Statement-1.

23. [Bl '


Both Statements are true and StatemE!nt-2 is correct explanation.
of Statement· 1.

24. [DJ _ -·
Statement-1 false and Statement-2'is true.

25.
[DJ · ·11 . - I 29. '[CJ h ·1· I
When the object is virtual, the convex mirror w1 give a "rea Angle of incidence at any location should be greater t an en 1ca
image. angle.
26. [BJ . . 30. [CJ
After refraction at two parallel faces of a glass slab, a ray of light In search light, we need intense parallel beam of light. When
emerges in a direction parallel to the direction of incidence of
source of light is placed at focus of concave mirror, only paraxial
white light an the slab. As rays of all colours emerge in the _same
rays are rendered parallel due to ·large. aperture of mirro_r.
direction (of incidence of whit~ light), hence there 1s no
Marginal rays give a divergent beam but in case of p~rabohc
dispersion, but only lateral !'.Jisplacement.
mirror, when source is at focus, beam of light produced over th.e
eritire cross-section of mirror is a parallel beam.
28. [CJ

=1.226

www.puucho.com
Rahul Sardana with www.puucho.com

1. [BJ For convex lens :


Different angles by geometry and the given conditions are 1
shown in figure. ---=-
-v -x 20
1 1 1
=> ---=-
x V 20
.•. (1)

For concave lens :


1 1
-(20-v)-(20-x) -10
1 1 '
=> - ----=-
20-x 20-v -10
Salving this equation, we get
20(,13-1)
x v'3 cm ••• (2)

45• => v=20(v'3-1) cm

4. [DJ
µ = sin45" =-./2 Magnification by convex lens is given by
sin30°
m = v, = -v = :!_ 20(,13 -1) v'3
2. [CJ ' LI, -X X 20( ,/3 -1)/,/3
STotal = Sp + Sa + SR Magnification by concave tens is given by
= (45° -30°) + (180° - 2 X 30°) + (45° -30°) = 150° v, -(20-v) 20-v
SToS, m---
,-u,--(20-x) 20-x

3. [BJ m, 40-20,/3 ( 2,13 _ 3 )


Image formed by concave lens is virtual for all positions of object 20- 20( ,/3 -1)
i.e., image by concave lens lies between the two lenses. For ,J3
both the images to coincide, image by convex lens should also
lie in between the two lenses the two lens or image by convex
lens should also be virtual. 8. [AJ
Since refractive index decreases with increase of A and velocity
decreases with increase of refractive index, so we have
V red > Vorang9 > Vyellow

9. [DJ
Colour and frequency remains unchanged as it is property of
source.

l+--X--+>4---20 - X ----11-1 10. [BJ


Dispersion depends on wavelength

====================================· = 1.227

www.puucho.com
Rahul Sardana with www.puucho.com

Advanced JEE Physics . Optics & Modern Physics


11. [DJ 21. [AJ
Since light takes shortest path, so
t = nxo 60°
C /60°
''
12. [CJ '' '
Speed of light changes with refractive index.
''
------~'' -------------
6=180-120=60°
Laws of reflection is universally true

22. [DJ
A=)(6-/3)' +(a-13)' +(-10)'
Ao, A= 20 units

23. [CJ
1 1 1 . id-ii) (6-/3i+a-/3]-1oi<).(-1<)
=> -+-=-
V 20 15 COSl=-ixr--
20
1 1
-=---
1
=> . 10 1 . 60
V 15 20 COSI=-=-· I= 0

1 , 20 2'
=> -=-
V 60 24. [BJ
=> V=60 cm sini -/3 . F2 ..
sinr = J2., sinr = ./3s1n1
14. [AJ
Observer 0 1 cannot see the image because light will be sini = J2 sin60° = F2 x .fa=_!_
absorbed by the blackened portion of the lens. sinr .fa -/3 ./2 ./2
=> r=45°
16. [DJ
According to Snell's Law, we have 25 [DJ
Let the vector representing the refracted ray be
sini =.&.=Ki=60o
sinr µ 2 A' = 6,/31 + a..Jaj + Ck
=> sin(60°)=K, r=.90° . ii' .(-1<)
Smee, cosr=[M
K, = .fa
2 ( 6,/31 + 8-/3] +Ck) ,(-k)
=> cosr
17. [CJ
)(6-/3)' +(a.fa)' +c'
-C
cosr=-;=~~~-=e-
=>
,/108+192+C'
cos45° -C 1 -C
18. [AJ =>
Since, e =·0°
,/aoo + c' ·; F2 - ,/aoo + c'
=> i=r => C =±10-/3
:::::,. K2 =1
Since the refracted ray travels downwards,
=> C=-10-/3
19. [CJ => A'= 6,/31 + a..Ja]-1 O-J3k
Since, f=R-~seca 26. [CJ
R IA'i=)(s..Ja)' +(a-13)' +(-10-/3)' =10v'6
So,for 0>0, f<-
2
27. [AJ
.A' 3, 4,1-
20. [DJ n=rA'r= 5.J2 I+ 5.J2J- .J2k
R .
I,, =R--sec(60°)=0 28. [AJ
2
Optical path length (OPL)= ndx J
= ==================================
1.228

www.puucho.com
Rahul Sardana with www.puucho.com

Ray Optics

=> f'
OPL= (1+x')dx
0 .
=> f
i4
_B_
-2µ .-·

=> x'
OPL= ( x~g )I' =am
O
4
35. [DJ
Focal length of /eris Li and L2 is given by using the Lens
Maker's Formula, so
31. [CJ
.! = (µ, -1)(..!. - ..!.)
peq =2P,ene + Pmirror I R "'
=> \=50cm
where P00,=i=(µ-1)(~ -~,)
Simila~y _1_ = (µ -1)(..!. - - 1-)
' f, ' "' (--R)
and F', =-1=-~
f , R
mrror
=> f=40 cm
So, equivalent focal length is given by
¾=2(µ-1)(~-(..!i))-(-~) 1 1 1
-=-+-
1 4(µ-1) 2 t., I f,
=> -=--+-
I R R => f =200 cm
.. 9
=> .!= 4µ -±+~
I R R R 36. [CJ
1 4µ 2 Image formed by L; lies 50 cm behind it and on principal axis
=> t,=R-R
of Li . This will act as an object for L2 . 86, for L2 ,

=> ¾=¾(2µ-1)
R
=> l-2(2µ-1)
......................... I, PA of L
1

32. [CJ 4.5mm


For a plano-conv~x lens, we have
PA of L,
_1_ = (µ -11(..!. - ..!.)
f, . R "'

=>
we have
=> u=+SO cm, t =+40 cm
Applying Len's Formula, we get
33. [BJ 1 1 1
When plane surface is silvered, then V 50 40
1 2 1 1 1 1
I=,,--~ {·.- ~--> «>} => -=-+-
v 50 40
1 2(µ-1) 200
V=-cm
I=-R- =>
9

R Magnification produced by L, is m, =~ = .±
=> ~ = 2(µ-1) · u 9
However, for ,L2 , the image 11 is at a distance of 4.5 mm above
34. [DJ its principal axis i.e., (PA of L2 ). So, distance ofimage 12 from
When curved surface is silvered, then
PA of L, is
,.-=,,--r
1 2 1
y = m, (4.5 mm)
4
=> 1 (1 1) -(-R)
2 => y= (4.5mm)=2mm
,.-=2(µ-1) ~- --R 9
Hence 12 is at a distance of (4.5-2) mm =2.5 mm from PA of
1 2(µ-1). 2
=> -=--+- L,.
f, R H

=> 37. [CJ
,:=R Applying refraction at cuived surface formula, i.e.,

c::::================================= 1.229=

www.puucho.com
Rahul Sardana with www.puucho.com

Advanced JEE Physics Optics & Modern Physics

EL+h=µ 2 -µ, ,we get


-u V R
1 2 1
-+-=-
x V1 R
2xR
=> V1 = x -R {from pole of curved·surface} O, C
I
I
This image formed will be at a distance v'=(v,-R) from the I 2
I
I
plane surface. Now again applying the above formula, but at the I
plane surface, we get I
I
I
_!_3_=0 {·: forplanesurlace R-->oo} -R---R-
V V'
Since, refractive index of this is 2, so th~ critical ang!B C is
2 given by
=> -=--
v V1 -R
C =sin-•(½)= 30°
2
· Substituting v 1 = xR , we get
x-R Since, i =C-, so the ray grazes the plane surface.
2(x-R)
v= R(x +R) 40. [CJ
Since yellow is the mean colour, so
For the image to be virtual, we have
µb +µ, 1.51+1.49 50
1.
..!.<o
V ••
µ, 2 2

=> 2(x-R) <O 41. [DJ


R(x+R) Again yellow is the mean colour, so
=> X<R µ~+µ; 1.77+1.73 175
µ ',
=~ .
2 2 .
This condition is only sati~fied by ;
42. [DJ
·38. [DJ For no deviation to take place, deviation (given by a=(µ ,-1 )A
. (2x)R by one must be cancelling the deviation due to the other.· For this
Since, v 1 = - -
x-R (a) the prisms must be arranged upside down
So,'for x=2R ,we get
(b) A' =-[µ,-1]
v,=4R A ·µ~ -1
Also, U=-2R
=> A'= -( 1.50-1 )so= -4o
1.7?-1
Since, m, =(~)(~)
43. [DJ
=> m, =(½)(!) Net dispersion = ( a, - a, ) +(a; - a; )
=> m1=-1 =(µ.-µ,)A+(µ;-µ; )A'
However for plane mirror, we have = (1.51-1.49)6°-( 1.77-1.73)4°
m2 =1 =0.02x 6°-0.04x4°
mnat = m1m2 = -1 = 0.12°-0.16° =-0.04°

Since, ml'III, =0 , so the final image formed is real, inverted and 44. [CJ
of same size. Since, fa z + x = 10 z
39. [CJ
-x 10 60°
=> Z=.J3+.J3
sin(90°) =
2
sinr -1
Slope= fa .
=> sinr =¾ -1
=> tan0= fa
=> r=30° 60°
~-----~--+x
In triangle OAC, we have => 0=-30°
OC = 2R , AC= R , so by Pythagora's Theorem, we get So, the ray is incident normally on the face -AB . Thus angle of
0A=.J3R incidence on face AC is 60° .
=> LACP=60° For grazing the face AC, we have
Using geometry, we get µsin(60°) =ix sin(90°)
i=3QO 2
=> µ= Ja

= 1.230 ================================::i
www.puucho.com
Rahul Sardana with www.puucho.com

Ray Optics
45. [BJ Equating equations (1) and _(2), we get
3 dy = y3/4
For µ = the ray will be internally reflected, so, the ray is
2 dx
normal at face BC . Hence finally, the i-efracted ray is parallel to ::::::, y-:314dy=dx

f.
z-axis.
:::::i- f' y-314dy= dx
' '
=> 4y1l4 =X ... (3)
The required equation of trajectory is 4y114 =x
'
=>

46. [BJ 52. [DJ .


If BC is silvered, the ray will retrace its path. Hence equation of At the point of intersection on the upper surface,
ray coming out Ot the prism is .Jaz + x = 10 . y=1m
=> X=(256)''=4m
47. [CJ
The objective lens r11ust form a ·real image for eyepiece to So the co-ordinates are ( 4 m, 1 m)
magnify it.
53. [AJ
48. [AJ As nA siniA = np sinip and as nA = np =1
The image formed by the objective must lie within the· focus of
the eyepiece. Therefore, ip = iA = 90° i.e., the ray Will emerge ·parallel .to the
boundary at P i.e., at grazing emergence.
49. [AJ
To obtain best magnification, the object must be placed just 54. [CJ
beyond the focus of the objective lens. In this case, the first
image distance from the objective is very large. 1 2
F f, I,
50. [CJ
i+9=90°, 9=90°-i, => ~=2(1.5-1)( 2~- 6 ~)-(-!o)
Slope of tangent = tan8 = tan ( 90° - i) = coti 1 1 1 2 8
=> - ----+---
-F 20 60 20 60
51, [DJ
60
=> F=- .=-7.5cm
but tan8 = dy 8
dx
dy = coli ... (1) 55. [BJ
dx Since F =0 , so combination behaves like a·concave mirror.
Applying Snell's Law at A and B
nA siniA = n8sini8 56. [AJ
nA =1because y=O I -7.5 -7.5
m=-=---
1-u -7.5+30 22.5 3

57. [CJ

58. [BJ

59. [BJ
Combine solution of 57, 58, & 59
Each part will work as a separate lens and will form its own
image. For any part, we have u = -0.3 m, f = +0.2 m .
siniA= 1 because iA =90° ( Grazing incidence)
Therefore, from lens formul~,
08 = ~Ky312 + 1 =~ys12 + 1
1
because K=1.0(m)-" ' V U f
1. 1 1
(1)( 1) = ~( y' 1' + 1) sini => -;;-o.3 = 0.2
. . 1 ::::::, V=-0.6 m
=> Slnl
~y3/2 +1 So, each part forms a real image of the point object O at
coti =~or y314 ... (2) 0.6 m from the lens, as sho'Wn in figure.

c:================================= 1.231 =
www.puucho.com
Rahul Sardana with www.puucho.com

Advanced JEE Physics Optics & Modem Physics

64. [DJ
Using Lens-Maker's Formula for the li(luid concave lens,
_1_ =(µ-1)(....!..._..!.) = _µ-1 = _µ-1
_ 50 · -R oo R 10
3
~=µ-1
=>
50 10
=> µ=1+0.6=1.6
,->--v=0.6m---+1
65. [BJ
Since the triangles OL,½ and OI,12 _?re similar. So, we have
Since the ray is incident nonnally, so
1,1 2 _OB_u+v r1 = o
1,L, - OA - -u-
Further, r, + r2 =A= 60°
I,I, _ 0.3 + 0.6 0.9 = ==> r2 =A=60°
=> 3
1,L, 0.3 0.3 A
=> 111, =3(L,L,) =3(2x0.0005) =0.003 m

60 [AJ

f=(µ-1)(~ -~,)
s~-----~
f=(¾-1)(¾- ~)=ix¾=¾ Applying Snell's Law at AC • we get
µsinr2 = µ,sine
=> i =A
. 4 .
=> µ 1 sme= .Jaxstn60°
61. [CJ

i=(¾- 1 X-~ -~)=-3~ =>


.
sine=
4,/32,1312
..f3µ 1 x 2 = ../3 x µ, = µ
1
... (1)

=> f,=-3R Further, deviation is given by


P=e-r2
62. [CJ From the graph, we observe that when µ 1 =k2 , p=O
Now _1_=.!.+_1_=_1_ __!_=~
'Fif,R3R3R So, P=e-r, =0
=> e=r2 =60°
=> F=3R
2 So, equation (1) becomes
Since, image coincides with the object so, clearly rays of light sin(60°)=~
must have retraced their path after reflection. This is possible µ,
only when rays of light must have fallen normally on the plane
mirror. For this, the object is at the focus of the lens system. =>
../3
2=µ1
2
0

i
F
µ,
4
=../3
4
=> k, =../3

66. [CJ
~ F = distance of object =15 cm
When µ 1 <k 1
3
=> :=15cm light will not emerge, it can be seen from graph, so we have
r2 =C when µ 1 =k1
=> R=10cm
=> sinr2 =sinC=~
63 [CJ µ,
If f is the focal length of the liquid concave lens, then
1 1 1 1 -3 => sin(60°)=~ {·: r, =60°)
-=-+- or-=- µ,
25 10 f f 50
,/3 4 1
50 =>
=> f=- cm 2= .,/3xµ,
3
8
=> µ1=3

c:::l 1.232====================================
www.puucho.com
Rahul Sardana with www.puucho.com

1/.ay Optics

k =~ Therefore, second image will be formed at a distance of m6. or


' 3 __ t_)(A) below its optic axis.
( t+\-d
67. [BJ
Therefore, y-coordinate of the focus of system is given by
From the graph, we observe that J3, is the maximum deviation
and j32 is the minimum deviation Y=A-(t+tt-d)

=> Y=(\-d)A
t +\-d
70. [AJ
Wavefronts are parallel in both media. Therefore, light which
propagates perpendicular to wavefront travels as a parallel
beam in each medium.
"
So, P2 = z-60° = 30° and P, =60°
71. [CJ
All points on a wavefront are at the same phase
$,=$,and$,=$,
$, -$, = $, -$,

72. [BJ
In medium-2 wavefront bends away from the normal after
refraction. Therefore, ray of light which is perpendicular to
wavefront bends towards the normal in medium-2 during
refraction. So, medium-2 is denser or its speed in medium-1 is
68. [A] more.
From the first lens parallel beam of light is focussed at its focus
i.e., at a distance \ from it. This image 11 acts as virtual object
for second lens L2 • Therefore, for L2
medium-1

U=+(\-d). f=+t
1 1 1 1 1
-=-+-=-+--
v I u t \-d
V=---
t (\-d)
t +\-d medium-2

y 75. [DJ

•' For both the halves, position of object and image is same,
however the only difference is of magnification. Magnification for
one of the halves is given as 2(> 1). This can be for the first
one, because for this, lvl > [ul . Therefore, magnification,

1ml =]ti> 1. So, for the first half, we have

l~l= 2

=> . lvl = 2lul


L.i
Let LI= -x , then v = +2x
I + - - - d - - - - · f,---d +I
and lul+lvl=1.8 m
- - - - - f, - - - - -
Therefore, x-coordinate of its focal point will be :::::, 3X=1.8m
:::::, X=0.6 m
X=d+V=d+ t(\-d) Hence, u=-0.6m and V=+1.2m
t+\-d
.1111 1 1
It+ d(\-d) Using-=---=----=-
=> X ! V U 1.2 -0.6 0.4
\+t-d :::::, f=0.4m
For the second half, we have
69. [DJ 1 1 1
Linear magnification for L2 , is given by -~--+
f 1.2-d -(0.6+d)

~ -(~~\--d~)(\~d)-t+~-d
1 1 1
=> - = - - +
m 0.4 1.2-d (0.6+d)
Solving this, we get d =0.6 m
i::::::=================================== 1.233=
www.puucho.com
Rahul Sardana with www.puucho.com

Advanced.JEE Physics Optics & Modem Physics

Magnification for the second half will be


V 0.6 1
m, =;;= -(1.2) =-2
12 79. [DJ
and magnification for the first half is m1 = "!.... =-·() =-2
u - 0.6
The distance of the mirror as observed by the eye is H+ !:!µ . So,
76. [AJ the distance of the fish as seen by the eye in the mirror is
For direct observation, the eye E appears to be farther that it
actually is. So, X=H+!:!+..!::!_
µ 2µ
X=µH+i=H(µ+¾)
=> X=H(1+}µ)
77. [BJ
The distance of eye from mirror is 81. [CJ
µH+H 1sin8, =-nsin02
So, distance of eye as seen by the fish in the mirror is
. 9
sm 1 .
= --s1n0
H
X=(µH+H)+ 2 2
n
1

3H
=:- X=2+µH 9,

=> X =H(µ+¾)
9,

78. [CJ
For direct observation by the eye E , the fish appears to be
closer than it actually is. So, we have
H
X=H+-
. 2µ

· · 1.234 ====================================
www.puucho.com
Rahul Sardana with www.puucho.com

1. A-> (r, s)
B-> (p, q, r, s, t) (A) Velocity of fish in air = 8, ,cl= 6 t
4
C-> (q, r, s, t)
D-> (p, r, s) Velocity of fish w.r.t. bird = + 6 6=12t
(A) m < O , means real image, possible for concave mirror and (8) Velocity of image of fish after reflection from mirror in air
convex lens. So, (A) -> (r, s). =8x,:l=6t
(B) m > O, means virtual image, possible for all i.e., plane 4
mirror, convex mirror (always), concave mirror (when w.r.t..bird =-6+6=0
object lies between focus and pole), concave lens (always)
and convex lens (when object lies between optical centre (C) Velocity of bird as seen from water = 6 x .±3 = 8 t
and focus). So, (B)-> (p, q, r, s, t). ·
{C) 1ml < 1 , means diminished image, possible for concave
Velocity of bird w.r.t. fish =8 + 8 =16 t
mirror, convex lens, .convex mirror and concave lens. So, (D) Velocity of bird in water after reflection from mirror = 8 J.
(C) -> (q, r, s, t). . w.r.t. fish = 8-8= O
(D) 1ml ~ 1, means magnified (> 1) and same sized (= 1) A-> (p, q)
image, possible for concave mirror/convex lens (both 5.
B -> (r)
1ml > 1) and plane mirror (1ml = 1) . So, (D) -> (p, r, s).
C-> (s)
D-> (p, q)
2. A-> (s, t)
B-> (p, t) 6. A-> (p, s)
C-> (s, t) B-> (q)
D-> (q, t) C-> (p, q, s)
D-> (r)
vA =1+at= f +{2f +j)<2l=si+2]
7. A-> (q, r)
=> vA. = -si +2} B-> (r)
C-> (p, r, s)
=> "A'A =VA' -VA =-10i
D-> (p, r)
Similarly, v, =(-1 +3]}
8. A-> (p, q, s)
=> v,,=(i+aj) B-; (p, q)
C-> (r)
D-> (p, q, s)

3. A-> (s) 9. A-> (p, s)


B-> (p,"q, r) B-> (p, q, r, s)
C-> (q, r,.s) C-> (p, q, r, s)
D-; (t) D-> (p, s)

4. A-> (r) 10. A-; (p, q, r, s)


B-> (q) B-> (q)
C-> (p) C-> (p, q, r, s)
D-> (p) D-> (p, q, r, s)

================================:::i1.2as=
www.puucho.com
Rahul Sardana with www.puucho.com

Advanced JEE Physics Optics & Modem Physics

11. A-->(p) 14. A-> (s)


B--> (p) B--> (q)
C--> (r. s) C--> (p)
D-> (q. p) D--> (q)

· 12. A-> (p. s)


B--> (p, q, r, s)
·
Ref.on
ground
~y
-~x-
A

B
C

C---+ (p, q, r, s)
D--> (q, s)
(A) For convex mirror : (A) ..'!.(x+y)= dx + dy =5+2=7 ms·•
dt dt dt
1 1 1
-+-=-
v u f (B) dx =3+2=5 ms-1
1 1 dt
V f U (C) d(2x) = 2 dx = 10 ms·•
dt dt
~ V=(u~f) (D) dx =5 ms_,
Now, v may be positive and negative, depending on dt
values .of u and f •
15. A-> (q)
Since, !ml=fu[=(u~ 1)=( ~ ) B--> (r)
7 1 C--> (s)
D-> (p)
Again 1ml can be greater than or less than 1. A µ 2 > µ 1 (towards normal)
(8) For concave mirror :
µ 2 > µ 3 (away from normal)
1 1 1
-+-=- B µ, = µ, (no change in path)
v +u -f

~
Li-0·
V=-(f~u) => Lr·= o on the block
So, v is always negative i.e., image is always real. C µ 1 > µ 2 (away from the normal)

Further, !ml=[~=
.
i-
-+1
µ 2 > µ3 (away from the normal)
1 .
I µ1 x J2=µ 2 s1nr
i.e., m is always less than 1 or image is always
diminished.
=> sin r = }:;1
v2µ 2•
13. A--> (s)
B--> (p) Since sinr < 1
C-> (q) => µ, < ../2µ2
D--> (r) D· ForTIR: 45°>C
A-+ when object lies between pole and focus image is virtual,
=> sin45° > sinC
magnified and erect.
B---+ when object lies between focus and centre of curvature, => _1_>&
·image is real, inverted and magnified. -1?. µ,
C---+ when object lies at centre of curvature, image is real, => µ, > -!?.µ,
inverted and of equal size.
p ---+ when object lies beyond centre of curvature, image is real,
inverted and smaller in size.

= ====================================
1.236

www.puucho.com
Rahul Sardana with www.puucho.com

1. Forward shift of point of incidence due to a single reflection The position of the image of end b can be obtained using mirror
formula, according to which we have
X = 0.2tan(30°) = ~ .
1 1 1
-+-=-
Hence required number of reflections required is v u f
N Mirror Length 2-./3 _ where u = -40 cm , f = -30 cm
30
Forward Shift ( ~) 1 1 1
=> -+-=--
v -40 --30
=> V=-120cm
2. Since, for a concave mirror, we can have both virtual and real Length of the image (rod) is given by
image, so e, =120-60=60 cm
for real image, m =-5 and
for Virtual image, m = +5 So.magnification rn=-dv =-(v2 -v1 )
du U2-U1
Since we know that m =_f_
f-u => m=-(120-60)
60-40
where f = -30 cm and m = ±5 because the image can be real => m=-3
or virtual.
. For real image, m = -5
=> lml=3

=> -5=~ 4. Applying mirror formula, ..!. + ..!. = ! for concave mirror, we get
-30-u
V U f
=> U=-36cm 1
For virtual image, m = +5 v- 60 = (-40)
~30 => v=-120cm
=> 5=---
-30-u
=> u=-24 cm
Hence, the object must be placed at 24 cm or 36 cm in front of
the concave mirror s

3. The image of the end a of the rod which lies at the centre of
curvature C is formed at C .
>+-- x --+1+--60 cm ...........i
Now, for the rays to again converge at S , (after reflection from
the plane mirror)
b' a b Distance of S ) = (Distance of Image formed)
p ( from Plane Mirror
F . by Concave Mirror
C
=> X=120-(X+60)
''
I => x =·ao cm

1+-4Qcm~
,1 So, the desired distance is 90 cm
From reversibility principle, it hardly matters whether the ray of
1 + - - - 60 crn------.i
120 cm _ _ _ _ _ _.,. light is first reflected from the concave mirror or plane mirror.·
1+-------
'
c=================================== 1.237 =
www.puucho.com
Rahul Sardana with www.puucho.com

Adva11ced JEE Physics Optics & Modem Pliysics

5. Since the image formed is real, so for the first case, we have
m=-3
Bird(B)r
=>
''
i
X

=> ''
'
Since.!.+.!=! ' y
V u I u=4cms·'~,

=>
1
--+--=-
1 1 t
(-3u,) (-u,) -I
Fish (F)
=> f = 3u, ... (1)
4 Differentiating w.r.t. time, we get
Similarly, for the second case, we have
m=-2 (-::)=M-:;)+(-:)

=> - V2 =-2 => 16=~(v)+u,where u=4cms-1


u,
=> v=9 cms-1

Since..!.+.!.=! 7. The necessary and sufficient condition for all the rays to pass
V U f around the arc is that the ray with least angle of incidence
1 1 1 should get internally reflected i.e., should sufferTIR.
=> - - + - - = - From the figure, it becomes Obvious that the ray with least angle
-(2u,) (-u,) -I
of incidence is the one which is incident almost grazingly with
f = 2U2 the Inner wall.
=> ... (2)
3
From (1) and (2), we get '
3U1 2u2 '' ,,
7 =3 Ai-di /'R J.

',
~-------
=> ••. (3) 0 "a"
For this ray, let a be the angle of lncidenC:e, then we observe
But according to the problem, we are. given that the shift of the
object is 6 cm , so we get
that
u2 -~=6cm sin a= R; d where d is the diameter of tube

=> !!.u -u =6 For TIA, n ;,.C


8 ' ' => sin a~ sinC
=> u1 =48 cm
A-d 1
--,'2:::-
=>
Since, v1 = 3u1 A µ
=> v1 =144cm
=> A~~
µ-1
So, I= 3u, = ~(48) = 36 cm Since, µ=J.5 and d=4cm,soweget
4 4
A=12cm
Now u, = i = ¾(48) = 54 cm
9
Hence, the least radius required is 12 cm.
and v, = 2u, = 2(54) =108 cm
8. Since, 9 = 90° - i
So, shift of the screen is (v1 -v2 ) => lane= coli
=> (v,-v,}=144-108=36cm => dy = coti • ... (1)
dx
6. Let at some instant bird is at a height of x · from the water Ac_cording to Snell's Law at O _and P ~ we have
surface and it is diving downwards with v cms"-1 µ 0 sin i0 = µP sin ip
At this instant fish is at a depth y· below water surface. Then the Since µ=~1+ay
distance between fish and image of bird at this instance will be, => At y=O, µ=1
S=µ><+y
=> sln(so•) = (~1 + ay )sini
=> s=ix+y.

= 1.230c===========================::;:::=======

www.puucho.com
Rahul Sardana with www.puucho.com

Ray Optics
y -' '
' ,B
A 'I
'
'
' P(x, y) D 0
C
i': y
'}I.,.
_._.""'::'.'.:=-----'':.le!L_;__ _ _ x
0 I + - - - X------+<
. . 1 i=r+r=2r
Slnl= ~ Since, we know that
,1+ay
sini
µ=-.-
coli= .Jay= dy s1nr
dx µ= sin2r ~ 2r = 2
J'ovay
d •
l.=Jdx o
s1nr r

11. After reflecting twice from two plane mirrors at right angles, a ray
=!> X=2l of light gets deviated by 180° , irrespective the angle of
incidence, so the emergent ray is aniiparaliel to incident ray and
Substituting y=2m and a=2x10--a m-1 ,weget hence angle of deviation is 180° .
xmax =2000 m = 2 km
9. The ray diagram for the situation,is shown in figure.
R

\. 4~
I-.· .... --~--~·-~·"·

12. From Archimedes Principle, we know that


Yimmersed = pbody
V101a1 P1;quid

Since, LPCQ=,-2r, LPRO=s-20-r)


"1mmarsed = Psphera = J:...
From the property of a circle, we get
V p~""' 2p
2(LPRQ) + LPCQ = 2s
=> 2s-40-r)+s-2r=2•
V 2
r=2i-!: i.e., half the sphere is inside the liquid. For the image to coincide
2 with the object, light should fall normally on the sphere. Using,
According to Snell's Law, we have
sini s1n1
µ2 _ .&. = µ 2 - µ 1 twice, we have
V u R
µ= -
sinr sin ( 2i-.!:)
2
../3 = sini sini
-cos2i 2sin2 i-1
::::) v1 =12cm
=> 2;/3sin'i-sini-,/3 =0
4 3 4 3
. ' 1±.,11+24 1±5
sm, r,; r,; Further - 3- - _g_ = .L._g_
4v3 4,3 ' h-10 8 -2
Rejecting the negative value, we get Solving this equation, we get
.. Fa
Slnl=-
h=1·s cm
. 2
=> i.= 60° 13. Forthelens, U=-2m, f=+1.5m
1 1 1
10. From the figure, we observe that V -2 1.5
BO=OC ::::) V=6·m
So, i( LOBC = LBCO = r (say) and
in~idence, then
be the angle of
Since, m = ( !
2
) = -3

c:===========================:;:::======== = 1.239

www.puucho.com
Rahul Sardana with www.puucho.com

Advanced JEE Pliysics Optics & Modern Pl,ysics

I1P = I2P and


I1P12 is .L to mirror. 2~ =(1.5-1>(¾- -!i)
,, ==> R=20cm
0.3m Applying ..!:l. _ _& = µ2 - µ 1 twice with the condition that rays
V u R
0 must fall normally on the concave mirror, we get
1.5 1.2 1.5-1.2
1, ... (1)
v, --40 +20
>+----- d _ _ _
' _.., 2 1.5 2-1.5
----=-- ... (2)
>+------6m------~ d-80 v, -20
Therefore, y co-ordinate of image formed by the lens is given Solving equations (1) and (2), we get
by d =30 cm and v1 =-100 cm
Y=m(0.1)=--0.3 m The ray diagram is as shown in figure.
In triangle PNI2 , we have

tanB = 12N
NP
o.3 1, l,
tan 0 = NP =0.3 {·.- lane= 0.3}

=> NP=MP=1m
....,.___ 100 cm _______,..
=> d=6-1=5 m
40
l+-'~c~m.... _ _ 30 cm_____,..
and x co-ordinate of final image 12 is,
X=d-1=4m 1+-----B0cm------+1

17. Using the lens formula.:!._.!.=!, we get


14. Applying lens formula .J-V _.!U =!f twice we get V U f

f=-6cm !=12cm
,/\
,' '

V
0

.,...__ 15 cm _____..
1+- 12 cm ->oi+--- d - - 1 1 1 1
1 1 1 ,=15-10=-30
---=- ••. (1)
v, -12 -6 =:> f=-30cm
1 1 1
••. (2)
oo v,-d 12 18. Using lens formula, _!_ _ ..:!. = ! , we get
Solving equations (1) and (2), we get V U f
v, =--4 cm 1 1 1
v, - (-40) = 30
and d=B cm
=:> V1 =120cm

15. Using lens formula, ..!_.!=!,we get


V U f Shift due to the slab A>c =(1-¾)d =(1- / )98
=4 cm
1 1 1
-+-=- So. u' =-(40-Ax) =-36 cm
v, 40 20
1 1 1
==> V1 =40cm => v, - (-36) = 30

Using mirror formula, .:!.+.!=!, we get =:> Va= 180 cm


V U f Therefore, the screen has to be shifted away from-the lens by a
1 1 1 distance
-+-=-
V2 10 -10 X=Va-V1 =60cm
==> v2 =-5 cm
so, the final image is formed at a distance of 5 cm from the 19. Lateral magnification in first case is -3, so
mirror towards lens. if U=-X then V=+3X

Since.!._..:!.=!
16. From Lens Maker's Formula, we get V U f

= 1.240 ==================================
www.puucho.com
Rahul Sardana with www.puucho.com

Ray Optics
1 1 1 22. Since we know that
=
-+-=-
+3x X f 02 -x2
f=--
4 1 4D
3x =1 Substituting f =16 cm and x = 60 cm , we get
4f D' -(60)'
X=- .•. (1)
3 16 4D
In the second case magnification is -2 , so now we have
=> D' - 3600 =64D
U=-(x+1,5), v=2(x+1.5)
=> D'-64D-3600=0
Since.!_.!=! D' -100D+36D-3600 = 0
V U f
1 1 1 D(D-100)+36(D-100) =0
=> ~--'-~ + - - - =- => (D-100)(O+36) =0
2(x+1.5) (x+1.5) f
3 1 But D ~ 36 cm , because it happens to be less than 60 cm . So,
=> 2(x+1.5) f D=100cm

=> f=3.(x+1.5) ... (2) 23. For the first flare spot, the lens acts as if its right face is silvered .
3 The equivalent focal length is given by
Solving equations (1) and (2), we get
f=9 cm
.!= 2(~:) 2(~-1)
20. Using Lens Maker's Formula for both the cases, we get F R, R,
2x1.5 2(1.5-1)
_1_ =(µ, -1)(..!..-
ti, R1
..!..)
R2
... (1)
F -60 +30
::::::,. F=-12 cm
and ~.. =[~: -1)(~' -:,) ... (2)
Using mirror formula, ..! + ..!. =..! with u ---t co , we get
Dividing equation (1) by (2), we get
v u F
1 1 1
\..,., = (µ, -1) -+-=-
v 00 -12
f,,. [~: -1) ::::::,. v=-12cm

Substituting the values, we get 24. Method 1 :


Ray diagram is as shown
-rn-1) 14---d--+<
r..... -[¾ Jf.,
4-1
3 -···-·-..
=> ~ate,=4'8;r=4x10=40cm
-----··

21. Using the lens formula, ..:!_ _ ..:!_ = we get !, 15 cm


V U f
u=+15cm, f=+30cm ,...____ 40cm ----;~
1 1 1.
=> ---=- d=25cm
V 15 30 Method 2:
=> v =+10 cm Since this combination just behaves as a plane glass plate,
Therefore, the focus of the rays will move 5 cm closer to the so the power of the combination is zero. Since
screen. The ray diagram is as shown in figure. 1 1 1 X
poomS =l.,m, =-i:+t- \t
::::::,. O=_.:!_+ 1 X
40 (-15) (40)(-15)
8------ ....... -----
X
(40)(15) 15 40

F
--- F X 40-15
(40)(15) (40)(15)
X=25cm

25. Applying Lens Maker's Formula, we get


1
-=(1.5-1) ( -1+ -1)
40 120 R,

=================================· 1.241 =
www.puucho.com
Rahul Sardana with www.puucho.com

Advanced JEE Physics Optics & Modern Physics

26. The ray diagram is as shown in figure for first two steps. If the
=> R1 =24 cm
rays reflected from the mirror are parallel after passing through
o: f the lens for the second time, then 12 must lie at first focus of
lens. So, the desired distance is given by

"4>l
f/2 f/2

,6!
1
10cm
0 I,

_x___.,
R, , _ _ 21--+l<I-- 2 1 -

Applying lens formula, for L2 _, we get X=(2f+20-.!.


2
1 1 1 ... (1)
-+-=- x= 31 = 3(30) =45 cm
V1 X 20
2 2
Using, µ 2 - .&. =µ 2 - µ1 for unsilvered side of ~ , we get
V u R
1.5 . 1 1.5-1 ... (2)
(-120)- (v, -10) = ~
Solving equations (1) and (2), we get
X=10Cm

= ====================================
1.242

www.puucho.com
Rahul Sardana with www.puucho.com

Wave Optics

[co~ten~s------------------------~---~
WAVE OPTICS .............................................................................................,...................... 2.1
Solved Practice Problems ..................................................................................................2.41
Practice Exercise Sets
:, Single Correct Choice Type Questions ..................................................................2.52
:> Multiple Correct Choice Type,Questions ...............................................................2.66
:> Reasoning Based Questions (Assertion Reason Type) ........................................2.69
:, Linked Comprehension Type Questions (Paragraph Type) •...•••..•••..•............•••..•••2.71
:, Matrix Match Type Questions (Column Matching Type) ........................................2.TT
:, Integer Answer Type Questions ............................................................................2.80
Answers to lri Chapter Exercise's.(ICE) & Practice Exercise Sets .....................................2.83
Solutions to In Chapter Exercises (ICE) .........................................................................., •• 2.86
Solutions to Practice Exercise Sets ...................................................................................2.92

www.puucho.com
Rahul Sardana with www.puucho.com

INTRODUCTION phase velocity or wave velocity. The energy travels


he phenomenon of interference, diffraction and outwards along straight lines emerging from the source,

T polarisation exhibited by light could not be explained


on the basis of Newton's Corpuscular Theory. In 1678,
Huygen suggested that light propagates in the form of
normally to the wavefront, that is, along the radii of the
spherical wavefront. These lines are called the rays.
For a point source in a homogeneous medium the wavefront
waves. The first historic experiment in favour of wave theory is spherical.
was done by Focault, who in 1850 found experimentally that
velocity of light in denser medium is less than that in the
rarer medium which was contrary to Newton's Corpuscular
Theory.

NEWTON'S CORPUSCULAR THEORY


Newton proposed that light is made up of tiny, light and
elastic particles called corpuscles which are emitted by a
luminous body. These corpuscles travel with speed equal to Spherical wavefront
the speed of light in all directions in straight lines and carry For a linear source of light the wavefront is cylindrical.
energy with them. When the corpuscles strike the retina of
the eye, they produce the sensation of vision. The corpuscles
of different colour are of different sizes (red corpuscles larger
- ," -
than blue corpuscles).
The corpuscular theory explains that light carry energy and s
momentum, light travels in a straight line, Propagation of
light in vacuum, Laws of reflection and refraction. However,
it fails to explain the phenomenon of interference, diffraction
and polarization.
.. -
Cylindrical wavefront

WAVE OPTICS A small part of a spherical or cylindrical wavefront from a


Wave optics is the study of the wave nature of light. distant source will appear plane and is, therefore, called a
Interference and diffraction are two main phenomena giving plane wavefront.
convincing evidence that light is a wave.

WAVEFRONTS AND RAYS


The locus of all the points vibrating in same phase of
oscillation is called a wavefront (WF) i.e. a wavefront is
defined as a surface joining the points vibrating in the same
phase. The direction of propagation of light (ray of light) is
along the normal to the Wavefront. The speed with which
/
Plane wavefront
the wavefront moves onwards from the source is called the

2.1 =
www.puucho.com
Rahul Sardana with www.puucho.com

Advanced ]EE Physics Optics & Modem Physics


----------- CONCEPTUAL
--------·---·--·-
NOTE(S)
Different types of wavefront
Type of wavefront Intensity Amplitude
Spherical

Point source
Cylindrical LAWS OF REFLECTION ON THE BASIS OF HUYGEN'S
Cylindrical THEORY
Light Let AB be the plane wave front incident on a plane mirror
ray M 1M 2 at LBAA' = i, where 1, 2 are th_e cqrrespondin~
incident rays .perpendicular to AB .
Cylindrical 2
WF
Line
:::: -:· ... source

Plane
~1:ne WF
According to Huygen's principle every point on AB is a
''.JJ'J:rJ I cc r
0
Aocr 0 source of secondary wavelets, so
BA'= ct, where c is speed of light
~ rays
J • ·..J··l
••'ft.''
'a

T
-;-+ Light
~
The secondary wavelets from . A will travel the same
distance ct in the same time. So,
AB'=ct
Now, LAA'B=90-i,sothat LA'AB=i, (0<i<90°)
HUYGEN'S PRINCIPLE
Also, LA'AB'=90°-r ,so that LAA'B'=r, (0<r<90°)
This principle is useful for detennining the position of a
From MB'A', we have
given wavefront at any further time if its present position is
known. The principle may be staied in three parts. . AB' ct
smr=----::-'::--=-- ...(1)
a) Every point on the given wavefront may be regarded as AA' AA'
the source of the new disturbance. Fiom ·M'BA., we have
b) The new disturbances from each pon,t spread out in all . . A'B ct
directions with the velocity of light in the same manner
Slfll=-.-=-- ... (2)
AA' AA'
as the original source of light does and ~ese new From equation (1) and (2), we get
disturbances are called secondary wavelets.
sini = sinr
c) The surface of tangency to the secondary wavelets !n
forward 9,irection at any time gives the position of the Li= Lr which is the law of Reflection
new wavefront at that time. This new wavefront is
called the Secondary Wavefront. LAW OF REFRACTION ON THE BASIS OF HUYGEN'S
Titis principle explained successfully, the reflection, THEORY
refraction, total internal reflection, interference and XY is a plane surface that separates a denser medium .of
diffraction but failed to explain the rectilinear propagation refractiVe index µ from a rarer medium. If v 1 is velocity of
light.
light in rarer medium and v, is velocity of light in denser
medium, then by definition

www.puucho.com
Rahul Sardana with www.puucho.com

Wave Optics
V be coherent. The emission of light from any source is from a
µ =---1. ... (1)
. v, very large number of atoms and the emission from each
AB is a plane wave front incident on XY at LBAA' = Li , atom is randoffi and independent of each other. Therefore,
there is no stable phase relationship between radiations from
where 1, 2 are the corresponding incident rays normal to AB
two independent sources. So, for two sources to be coherent,
According to Huygen's principle they must be deriv~_q. fr9II1 the same parent source.
... (2) In practice, coherent sources are obtained either by dividing
The secondary wavelets from A travel in the denser the wavefront (~~,,in the case of Young's Double Slit
medium with a velocity v2 and would cover a distance Experiment, Fresnel's biprism, Lloyd mirror, etc.) or by
AB'= v 2t in the same time. dividing the amplitude (as in the case of thin films, Newton
rings, etc.) of the incoming waves from a single source.
2
A laser discovered in 1960, is different from common light
sources. Its atoms act in a cooperative manner so as to
Incident produce intense, monochromatic, unidirectional and
wave front coherent light. Thus, two independent laser beams can
produce observable interference on a screen.

METHODS OF PRODUCING COHERENT SOURCES

A, DIVISION OF WAVEFRONT
In this method the wavefront is divided into two parts by the
use of mirrors, or lenses or prisms. Well known methods are
2'
Young's double slit arrangement, Fresnel's biprism and
1' Lloyd's single mirror.
So, from MBA' and MB'A'
B. DIVISION OF AMPLITUDE
. . BA' d. AB'.
smz=-- an smr=--
AA' AA' In this method the amplitude of the incoming beam is
divided into two .parts by means of partial reflection of
sini BA AA'1
BA' vt
=> --=--X--=-= 1 = v1 refraction. These divided parts travel different paths and are
sin r AA' AB' AB' v 2t V2
finally brought together to produce interference. This class of
So, from equation (1), we get interference requires broad sources of light. The common
examples of such interference of light are the brilliant colours
sini =.:'!_ = µ which is the Snell's Law of Refraction. seen when a thin film of tr~parent material like soap
smr v2
bubble or thin film of kerosene oil spread on the surface of
water is exposed to an extended source of light. This kind of
INTERFERENCE interference exists in tvvo types.
When two waVes of same frequency, nearly same amplitude a) Interference due to waves reflected from both the front
and constant initial phase difference travel in the same and back surfaces of the film.
direction along same straight line, they superimpose in such b) Interference due to transmitted _waves.
a way that- in the region of superposition, the intensity is
maximum at some points and minimum at some other
INTERFERENCE: MATHEMATICAL TREATMENT
points. This modification in intensity in the region of
superposition is called Interference. The sources having the Two waves (whether sound or light) of equal frequencies
same frequency and .co~tant initial phase difference are travelling almost in the same direction show interference.
called coherent sources. The phenomenon of interference is Consider two waves coming from sources S1 and S2 • These
based on the Law of Conservation of Energy. reach point P with a path difference t,:,c, having amplitude
A 1 and A 2 •
COHERENT SOURCES
Two sources which emit light of the same wavelength with
zero or a constant phase difference are .called coherent
sources.
Unlike sound waves, two independent sources of light
cannot be coherent. Sound is a bulk property of matter. So,
two independent sources of sound can produce coherent
waves. However, two independent sources of light cannot

2.3 =
www.puucho.com
Rahul Sardana with www.puucho.com

Advanced JEE Physics Optics & Modern Physics

y, =A,sin(cot-kx) ... (1) ~ ,j,=(2n+l),r

and y, =A,sin[cot-k(x+LU)] ~ 2
<1>=( :}=(2n+1)1t, where n=O, 1,2,3, .....
~ y, =A,sin(cot-kx-,j,) ... (2)
=>
,.
ll.x=(2n+1) ,where n=O, 1,2,3, .....
2
where ,j, = kLU = ( ; )LU and LU is the path difference. 2
So, intensity will be maximum when phase difference ,j, is
By Principle of Superposition, the resultant wave at P is an odd multiple of ,r or path difference LU is an odd
y = y, + y, = A, sin( cot-kx)+A,sin( cot-kx-,j,)
multiple of ~.
~ y=(A, +A,cos,j,)sin(rot-kx)-(A,sin,j,)cos(rot-kx)
... (3) ~ Imin=I1 +I,-2M=(Jf:-.,/f;)' =K(A,-A,)'
Substituting A1 +A,cos,j,=Acos0 and A,sin,j,=Asin0, we
The ratios
1 (JI,+.,µ,)' (A,+ A,)'
get rnu

A'= A;+ Ai+ 2A,A, cos,j, ... (4)


/min (JI, - .,µ,)' (A, -A,)'
Equation (3), becomes If 11 = 12 = 10 (i.e., A, =A,), we have
y = Asin(cot-kx-0) ... (5) I= = 4I, and Imin = 0
Thus, when interference of two waves of equal intensities
where, A= JA; +Ai+ 2A,A, cos,j, and tan0 A, sin,j,
A, +A,cos,j,
occur, the intensity of maxima becomes four times that of
single wave and that of minima becomes zero.
The intensity of the resultant wave is
In Young's Double Slit Experiment popularly known as
. 1
I =-pvco'A' =KA' =K[ A; +Ai +2A1A,cos,j,] YDSE, usually the intensities 11 and 12 are equal, so
2 I1 =I,=I0
~ I=I1 +I2 +2,Jf;i;cos,j, ... (6)
Since, IR= 11 + 12 + 2,Jf;i; cos~, so we get
Interference term

Thus, when interference of two waves of equal intensities I=2I,(l+cos,j,)


occur, the intensity of maxima becomes 4 times that of single
wave and that of minima becomes zero. ~ I=4I0 cos'(¾)

CONDITION FOR MAXIMA: Constructive Interference


PHASE DIFFERENCE AND PATH DIFFERENCE
From equation (6), I is maximum, when
If two waves travel different lengths of path to reach a point,
cos,j,=+1
they may not be in phase with each other. The phase
=> 4'=0, 2rc, 41t, ....
~ ,j,=2n,r
difference depends on the path difference as ,j, = " LU,,.
2

where LU is the difference in length of path traversed by the


~ <1>=(~}=2n1t,where n=O, 1,2,3, ..... waves.

=>
,.
M=(2n) ,where n=O, 1,2,3, .....
The phase difference between two light waves can change if
the waves travel through different materials having different
2 refractive indices:
So, intensity will be maximum when phase difference ,j, is Suppose, we have two waves having identical wavelengths
an even multiple of 1t or path difference 8X is an even A., initially in phase, in air. One of the waves travel through
medium 1 of refractive index µ 1 and length L and other
multiple of ?: .
2 wave travels through same length L in another medium of
refractive index µ 2 • As wavelength differs in a medium, the
~ I= =11 +I,+2,Jf;i; =(Jf: +.,ff;)' =(A, +A,)'
two waves may not remain in phase.
The path difference after crossing through the medium is
CONDmON FOR MINIMA: Destructive Interference given by
Intensity I_ will be minimum, when LU=(n,-n,)L
cos,j,=-1
where n1 is number of wavelengths in medium 1 and
=> if,=rc, 31t, 51t, ....
n2 is number of wavelengths in medium 2

_.;..,, 2.4

www.puucho.com
Rahul Sardana with www.puucho.com

Wave Optics

two sources was first demonstrated by Thomas Young in


1801.
Light is incident on screen A , which is provided with a
narrow slit, S0 • The cylindrical waves emerging from this
slit arrive at screen B, which contains two narrow, parallel
slits, S1 and S2 . Light emerges from these two slits as
cylindrical waves. In effect, slits S1 and S1 act as individual
Illustration 1 light sources that are in phase as they originate from the
Two light rays, initially in phase and having same cylindrical wavefront. The light from the two slits
wavelength 6 x 10-' m , go through different plastic layers produces a visible pattern on screen C . The pattern consists
of the same thickness, 7 x 10~ m . The indices of refractiqn of a series of bright and dark parallel bands called fringes.
The overall light amplitude at a given point on the screen is
are 1.65 for one layer and 1.49 for the other.
the result of the superposition of the two wave amplitudes
(a) What is the equivalent phase difference between the from S1 and S2 • Two waves that add constructively give a
rays when they emerge?
bright fringe, and any two waves that add destructively
(b) If those two rays then reach a common point, does the
produce a dark fringe.
interference result in complete darkness, maximum
brightness, intermediate illumination but closer to
complete darkness, or intermediate illumination but
closer to maximum brightness?
Solution
(a) ru:=(µ 1 -µ 2 )!=(1.65-1.49)(7xlO-')
lll:=1.12x10_. m

Since, Phase difference ~ =(~" )<ru:) Max

=> $=( } ~_7 )t1.12x10-<)


6 1
=> ~ = 11.72 radian

(b) To discuss this case, two options arise


Option 1 : Waves are in phase, then using

I =I-cos'({), we get
Schematic diagram of Young's double·slit experiment.
The narrow slits act as sources of cylindrical waves.
I= Im~cos '(11.72)
- - =0.81_ Slits S 1 and S2 behave as coherent sources which
2 produce an interference pattern on screen C.
This value is intermediate illumination closer to
maximum brightness. YDSE (QUANTITATIVE TREATMENT): METHOD 1
Option 2 : Waves are out of phase, then
~~, =11.72±1t

=> ~~, = 14.86 rad


14 86
=> I=I_cos'( ; )

=> I =0.1710 Viewing Screen


1bis value is intermediate illumination closer to Geometric construction fordescribingYoung's double-slit
darkness. experiment. Note that the path difference between the
two rays is r2 - r1 =dsin8
THEORY OF INTERFERENCE : MAXIMA & MINIMA Consider a point P on the viewing screen located a
perpendicular distance D from the two identical slits S1
THEORY OF DIVISION OF WAVEFRONT: YOUNG'S
and S2 , which are separated by a distance d . Let us assume
DOUBLE SLIT EXPERIMENT
that the source is equidistant from the hvo slits and is
The phenomenon of interference of light waves arising from
2.5 =
www.puucho.com
Rahul Sardana with www.puucho.com

Advanced JEE Physics Optics & Modern Physics


monochromatic, that is, emitting light of a single wavelength This result shows that the separation between adjacent dark
A.. Under .these assumptions, the waves emerging from slits
fringe~ is.also equal to p = "~ . Since the quantities ; and
S, and S2 have the same freque11cy and amplitude and are
in phase. The light intensity on the screen at P is the d are both measurable, we see that- the double-slit
resultant of light .coming from both slits. Note that a wave interference pattern, together with equation (6), provides a
from the lower slit travels fartl!er_ than a wave from the direct determination of the wavelength ;\. . Youpg used this
upper slit by an amount equal to dsin0. This distance is technique to make the first measurement of the wavelength
called the path difference, x , where · oflight.
x=r,-r,=dsin9 ... (1)
The value of this path difference will ,determine whether or YDSE (QUANTITATIVE TREATMENT): METHOD 2
not the two :waves-are in phase when they arrive at .p-. If the Conside; that two coherent sources•of light s;
and S2 are
path difference is either zero or some integral multiple of the placed at" ; dist~ce ·d · ap:.rt and. a screen is placed· at a
wavelength, the two waves are in phase at P and distance D from the plarie of th~ two sources._ ~t P be a
constructive interference results~ Therefore, the condition for point on the screen at a distance y frorri a point O exactly
bright fr4,ges, or constructive inte~ei-ence, at -P is given by
opppsite to the_ centre of the two sources S1 and S2 • If x .is
x=dsin9=n;\. (n=O,±1,±2,±3,..) · ... (2)
path difference between the light waves reaching point P
The index number n is called the order number of the from the sourCes S1 and S2 , then
fringe. The central bright fringe at 9 = O(n = 0) is called the x;s,P-s,"r
zeroth order maximum. The first maximwn on either side,
when n = ±1, is called the first order maximum, etc. - ,_

Similarly, when the path difference is an odd multiple of ~,


pl
the two waves arriving at P will. be opposite in phase and

~-----~-- ---------- : l
-~ . y
will give 'rise. to. destructive interference. Therefore, the
condition for dark fringes, or destructive·interference, at Pis ~ is,
given by
s d Q~---- --------------- --
x = dsin9 = (2n + 1)2':
2
(n=O,±1,±2...)

It is useful to obtain expressions for the positio~ of. the


... (3)
~ i s,n.-----------------~~- B

bright and dark fringes measured vertically from O to P . LJi...--'-- D ----<• 1Screen
We shall assume that D > d and consider only points .P that 1n right angled t.S,BP, we have
are close to O . In this case, 8 is small, and so we can use the
approximation .sin9 = tan9. From the large triangle ·oPQ in S,P' =52 B' +BP' =D' +(y+f)'
Figure, We see ~t
' Also, in right angled LiS,AP, we have ·
sin9ztan9='; ... (4)

Using this result .together with equation -(2), we see that the S,P' =S;A' +AP' =D' +(y-f)'
positions of the bright fringes measured from O are given
by 2 2
=> S2P -S1P =(D' ;(y+fJ)~(D' ,{y-JJ)
... (5)
=> (S2 P+S,P){S2 P-S1P)=(y+%J-(y-%J
From this expression, we find that the separation between
any two adjacent bright fringes called Fringe Width is equal Since S2 P-S1P=x (the path difference between the two
to
;\.D
d't h .
atts, light waves), th~ abOve equ~tion becomes:
-1..D ;\.D ;\.D (S,P+; 1P)x =4y(%)~ 2yd
"=y
P n~1
-yn =-(n+l)--.
d d n=-d ..... (6)
2yd
Similarly, using equation (3) and (4), we find that the dark X
S2 P+S1P
fringes are located at .
;\.D In practice, the·point .P lies very close to the centre of screen,
Ya.uk =(Zn+ 1)- ... (7) so we have
2d

= 2.6

www.puucho.com
Rahul Sardana with www.puucho.com

Wave Optics
S,P=S1 P=D (b) Phase difference corresponding to the given path
2yd 2yd difference rue is given by
X=--=-
D+D 2D
$=(:")rue
X=yd ... (1)
D
=> $=(2t)(~)
For Maxima, we know that path difference x must be an
even multiple of ~, so

).
x = (2n) , where n = 0 , 1, 2, .....
2
=> ~ =n').., where n =0, 1, 2, .....

=> y.=n().~),where n=0,1,2, .....


Illustration 3
At y = 0 (i.e., for n = 0 ) we get a Central Bright Fringe. In YDSE, the interference pattern is found to have an
For Minima, we know that path difference x must be an intensity ratio between the bright and dark fringes as 9.
Find the ratio of
odd multiple of ~, so (a) intensities.
2
). (b) amplitudes of the two interfering waves.
x = (2n + 1) , where 11 = 0, 1, 2, ..... Solution
2
yd ). In case of interference, we have
D=(2n+1) , where n=O, 1, 2, .....
2 I =11 +I, +2N, cos$
l.D
=> Y. =(2n+l) d, where n=O, 1, 2, .... .
2
(a) r_ =I, +I, +2N, =(JI: +F,)'
and Imm = 11 + 12 -2N, = (F, - F,)'
=> Y.=(n+½)).~ ,where n=0,1,2, .... .

Since, 1max
(F, +F,_)' 9
Illustration 2 [min (F, -F,_)' 1
If the maximum intensity in YDSE is I 0 , find the
intensity at a point on the screen where
F, +.fl, 3

(a) the phase difference between the two interfering F,-F,_ 1


Solving, we get
beams is .'.::,
3
I.i..=±=4
(b) the path difference between the two interfering beams 12 1
. ).
(b) Since, I oc A2 ,
1S - .

Solution
4
=> I.i..=(
I2
~
A2
J
(a) Since, l=lm,,cos'(t)

where Imax is 10 i.e., intensity due to independent


=> (~:J =4

sources is ~ . Therefore, at => ~=2


A,

Illustration 4
The intensity of the light coming from one of the slits
in YDSE is double the intensity from the other slit. Find
the ratio of the maximum intensity to the minimum
I =10 cos '(") =io
3 intensity in the interference fringe pattern observed.
6
2.7 =
www.puucho.com
Rahul Sardana with www.puucho.com

Advanced ]EE Physics Optics & Modem Physics

Solution (c) Position of second minima is obtained by putting n = 2


Since, we know that
in the equation y, = (2n-1) AD, so we get
I-
/min
=(K..p; -.Jr,
+.Jr,)'
2d

y 2 =(2-.!.)1,.D =~P=~(0.2)=0.3mm
2 d 2 2
According to the problem, we have
11 = 210 and 12 = I0
Illustration 7
=> Im~ =
1_
(,/2
,/2 +-11)' = 34 Young's double slit experiment is canied out using
microwaves of wavelength A = 3 cm . Distance between the
slits is d = 5 cm and the distance between the plane of slits
and the screen is D = 100 cm . Find the number of maximas
Illustration 5
and their positions on the screen.
In a Young's double-slit experiment the distance
between the slits is 1 mm and the distance of the screen Solution
from the slits is 1 m. If light.of wavelength 6000 A is used, The maximum path] [Distance between the]
find the distance between the second dark fringe and the difference that = coherent sources
fourth bright fringe. [ can be produced i.e., 5 cm
Solution Thus, in this case we can have only three maximas, one
The position of the second dark fringe is given by central maxima and two on its either side (for a path
difference of 1,. = 3 cm)
y 2 (dark)=(2n-ll"D =(4-ll"D =~(,.D)
2d 2d 2 d
The position of the 4th bright fringe is given by
. nJ,.D 41,.D
y,(bnght)=-a-=-d- f 81:
Therefore, the separation is given by 1s, ...r-_-_--_--:--------1
fly= y4 (bright)-y, (dark)= ( 4-¾Y~
10 For maximum intensity at P, we have
=> fl 5 6000 X 10- X 1 l.S X lQ°" = l.5 mm
y=zx 10"' S,P-S1P=A

Illustration 6 => (y;d)' +D' - (y;d)' +D' =A 2

In a YDSE , the separation between the coherent Substituting d = 5 cm, D = 100 cm and 1,. = 3 cm and
sources is 6 mm, the separation between coherent sources solving the equation, we get
and the screen is 2 m . If light of wavelength 6000 A is y=±75 cm
used, then
Thus, the three maximas will be at
(a) find the fringe width.
(b) find the position of the third maxima. y=0 and y=±75cm
(c) find the position of the second minima.
FRINGE WIDTH & ANGULAR FRINGE WIDTH
Solution
The separation between two consecutive bright (or dark)
(a) Since fringe width is given by p= 1,.~ , so we have fingers is called the fringe width (p), given by
10
(6000x10· )(2) =O.Z mm
P=i,.D
d 6xl0'3
(b) Position of third maxima is obtained by substituting

n=3 in the equation y, =n(A~),soweget


31,.D
y, =d=3P=3(0.2)=0.6 mm
D--~

r:::::l 2.8

www.puucho.com
Rahul Sardana with www.puucho.com

.Wave Optics
:w i.e., 4th minima of 400 nm coincides. with 3rd minima of
f3=Y11+1-Yn·=7
560nm
Note that fringe width p is independent of n. That is, the The IoCation of this minima is.
interference fringes have same width throughout. 7(1000)(400x10_.)
The angular fringe width is given by Y, · 2x0.1
14mm

d~=I=~ Next , 11th minima of 400 nm will coincide .with 8th


D d minima of 560 nm
Location of this minima is
CONCEPTUAL NOTE(S)
21 (1000)( 400 X 10_.)
In VOSE alternate bright and dark bands obtained on the Yn 42mm
screen. These bands are called Fringes. · 2x0.1
Screen So, required separation is
n 1
3_ Bright liiail!liil4 Dark Lly=42-14=28 mm
s LJ · 2 Bng. ht 1lillllliill 3 Dark
. 'Hf
=
:

-~---u1·----
1 Bright lliil/illfiilll2 Dark Illustration 9
----1-~~i~~; ::-:-~:~:---bri~~tn;;i~ge Light from a source consiStS of two wavelength
A-1 = 6500 A and A-1 = 5200 A . If the separation between
s, 2 Bright ~ 2 Dark

~ . ht_.. .' 3Dark the sources from each.' other is 6.5 mm and that from the
3 Brig j,

·. D . ~4.0ark screen is 2 m, find the minimum value of y(;t:O) where


the maxima of both the wavelengths coincide.
d = Distance between slits Solution
D = Distance between slits.and.screen
A.-'=' Wavelength of monochromatic llght emitted from source Let n 1 maxima of A-1 coincides with n2 maxima of A. 2 •
a) ' at Central position
Central fringe is always bright, -because Then, Yn1 ~ Yn:z
the path difference, x =.0 a,nd hence the phase
71iXp = n,,.,D
difference, $ =0° and hence tjle Central Bright Fringe is
d d
,illSo called the zeroth maxima.
b) The nth ·minima_ comes before the'nth maxima'. 5.= ,., = 5200 =±
c) The fringe pattern obtained due to a,slit is more bright than n,_ '-1 6500 5
that due to a point. Thus, fourth maxima of A- 1 coincides with fifth maxima of
d) If the slit widths are unequal, the minima will not be
complete dark. For very large width uniform illumination '-,. The minimum value of y(;, 0) is given by
occurs.
e) If one slit is illuminated with red· light and the other slit is
41.,D 4(0.65x10.. )(2)
y=-d-= 6.5x10.., 0.8mm
illuminated with blue light, n9 interf~rence pattern is
observed on the screen.
f) If the two coherent sources consist of object ?nd ws·
reflected im_age, the central fringe is dark instead of bright Illustration 10
.__ _:o::.n•:c·c____ _- " - - - - - - - - - - - - - - - --~- Two coherent sources are ·o.3 mm apart. They are
0.9 m away from the screen. The second dark fringe is at a
Illustration 8 distance of 0.3 cm from the centre. Find the distance of
fourth bright fringe from the centre. Also find the
In a YDSE bi-chromatic light of wavelengths 400 nm wavelength of light used.
and 560 nm are used.' Th~ distance between the slits is
Solution
0.1 mm and the distance 'between the plane of the slits
and the scree_n is 1 m . Find the minimum distance Second dark fringe n = 2 will be obtained at
between two Successive regions·of complete darkness.
Y=(2n-1)"D = 3'-D (1)
Solution . . . •'2d 2d ...
Let 71i minima of 400 nm coincides with n,_ minima of i..D 2
=> -=-y
560 nm, then d 3
(2n 1 -1)400 = (2n, -1)560 A-D 2
=> P=a= (o.3)=0.2 m
3
2n 1 -1 ='!...= 14 =21
Fourth bright fringe from the centre will be obtained at
2n,_-1 5 10 15
y, =4P=0.8 cm
2.9 =
www.puucho.com
Rahul Sardana with www.puucho.com

Advanced JEE Pl1ysics Optics & Modern Physics

From equation (1), we get n+l 6500 5


=>
1,_ 2yd 2 X 0,3 X 10-,3 X 0.3 X 10-' n 5200 4
3D 3x0.9 => n=4
=> 1,.=6.67xl0- m 7
n1,.D 4x6500xl0-10 xl.2
=> Y, =-d-= 2xl0_,,

Illustration 11 => y, =l.56xl0_,, m=l.56 mm


In a Young's double slit experiment, two narrow slits
0.8 mm apart are illuminated by a source of light of
Illustration 13
wavelength 4000 A. How far apart are the adjacent bright A convergent lens with a focal length of f = 10 cm is
bands in the interference pattern observed on a screen 2 m
cut into two halves that are then moved apart to a distance
away?
of d = 0.5 mm (a double lens). Find the fringe width on
Solution
screen at a distance of 60 cm behind the lens if a point
Since, d=0.BxlO_,, m, 1,.=-4000 A=4000xl0-10 m, source of monochromatic light (,. = 5000 A) is placed in
D=2m front of the lens at a distance·of a= 15 cm from it.
The distance between the adjacent bright bands or the fringe I
Solution
width is given by
P=""D
10
4000xl0- x2 m=lmm Applying lens formula, ,! _,! = .! , we get
d O.BxlO_,, V U f
1
1 1
-+-=-
15 10
v
Illustration 12
A beam of light consisting of two wavelengths, => v=30cm
6500 A and 5200 A, is used to obtain interference fringes , V 30·
Smce, m=-=-=-2
in a Young's double slit experiment. The distance between u -15
the slits is 2 mm and the distance between the plane of the
S,t'
'
~
slits and the screen is 120 cm .
(a) Find the distance of the third bright fringe on the -----1
'' 0.5mm

~
screen from the central maximum for the wavelength 0.25mm
________ _)' 0.25mm
65ooA s '
~ s2!'
0.5mm
(b) What is the least distance from the central maximum
where the bright fringes due to both the wavelengths
15 cm
''
coincide? 30cm D--+1
Solution 1+---- 60cm----+1
According to the problem, we have Distance between two slits is d = 1.5 mm
"-, = 6500 A = 6500 x 10-10 m Distance between slits and screen is D = 30 cm

"-, = 5200 A =5200x10-10 m Fringe width p "-~


(5xrn-')(0.3)
I (l.5x10_,,)
d=2 mm=2xl0_,, m
P=O.lmm
D=l20 cm=l.2 m
n1,.D
(a) Y, =-d- INTERFERENCE EXPERIMENT IN WATER
In water (liquid), of refractive index µ the wavelength
3x6500x10-10 xl.2
=> YJ = 1.17 x 10_,, m = 1.17 mm
decreases from A.
'
to A.' =-
"-
Therefore, if interference
µ
(b) The least distance from the central maximum where the experiment is performed in water the fringe width decreases
bright fringes due to both the wavelength coincide from p to P' , such that
corresponds to that value of n for which
n1,.1D (n + 1)1,.,D P=""D and P'=""'D =""D
d d µd
d d
=> ""-, = (n + 1)1,., => P'=~
µ

= c==================================
2.10

www.puucho.com
Rahul Sardana with www.puucho.com

Wave Optics
Illustration 14
A Young's double slit arrangement produces 41,
interference fringes for sodium light (,. = 5890 A) that are
'
0.20° apart. What is the angular fringe separation if the
entire arrangement is immersed in. water? Refractive index
;-:, ·:''
. 4
'
of water1s -.
3
Solution
D D B B D B --+-y
The wavelength of light in water is '-w = !: Intensity distribution on the screen as a function of yin YDSE
µ
Ima,= 410 for bright fringe and Im1n =0 for dark fringe.
Angular fringe-width in air, e, = !:
d
PROBLEM SOLVING TRICK(S) i
Angular fringe-width in water, ew = '-w a) lntt::rference occurs due to L_aw of Conservation of Energy. /
d Actually, redistribution of energy takes place. ,

So, e - '-w - ,. - 0, - 0.200 - 0,150 b) If w1 and w2 are the widths of the slits and 11 and 12 is l
I
w d µd µ ± the intensity of light (with respective amplitudes a1 and a2 )
3 passing through slits, then
11 _a~_ W 1
FRINGE VISIBILITY (V) -:--7:--
l2 a2 W2
With the help of the concept of visibility, the knowledge
about coherence, fringe contrast and interference pattern is
obtained. Fringe visibility V is defined as
c) Im,.
Imaic =
(a,-a,)'
a1 +a2 = .{vi; +,Fi;
(F.-Jw,)' =(,µ,-,µ,)'
.jI; +.JI;
d) If point source is used to illuminate the two slits, the
v r_-r_ 2N, intensity emerging from the slit is ·proportional to area of
I= + Imm 11 + 12 exposed part of slit. In case of identical slits.
If [min = 0, V = 1 (maximum) i.e., fringe visibility will be the l1 = l2
best. => a1 =a2
Also if I_ =0 then V =-1 e) When white light is used to illuminate the slit, we obtain ,an
interference pattern consisting of a central white fringe
and if I_ =lmin, then V =0 having on both sides symmetrically a few coloured fringes I
and then a uniform illumination. I
f) If qi is the phase difference .between two waves of /
INTENSITY DISTRIBUTION
intensities 11 and 12 , then
When two coherent light waves of intensity 11 and 12 with a
constant phase difference $ superimpose, then the resultant I
IR =11 +12 +2M cosq,
intensity is given by I gl If x is the path difference, then

I =I, +I, +zN, cos$ IR=I1 +12 +2Mcos(~1txJ !


In YDSE, usually the intensities 11 and 12 are equal, so h) In YDSE , if n1 fringes are visible in a field of view with light r
I 1 =I, =I0 of wavelength A1 , while n, witti light of wavelength A2 in \
Since, IR= 11 + 12 + 2.jf;i;·cosq>, so we get
!'
I =2I0 (1+cos$) Separation (file) between fringes '
(a) Between_ nth bright and m 111 bright fringes (n > m)
~ I= 4!0 cos'(¼) ilx =(n-m)~
111
For maxima, . q> = 2mt (b) Between n bright and mth dark fringe
=> I= =410 (i) If n>m then fil< =(n-m+1)~
For minima, $ = (2n + 1lit
I 2 ;

=> In:un =0 i (ii) II n<m then fil<=(m-n-¾)~ I


j) Identification of central bright fringe : To identify central j
[
___ Qrigh!.f!inge, m9n.9-chroro__c!!i.9Jight.isJepJaced_by_whiJ1q_ llgt]_t! :

===================================== = 2.11

www.puucho.com
Rahul Sardana with www.puucho.com

Advanced JEE Plzysics Optics & Modem Physics

I Due to overlapping central maxima will be white with red)


j edges. On the other side of it we shall get a few coloured 1
1 band and then un_iform illumination. j

'
Illustration 15
In YDSE , light of wavelength 60 run is used. The
separation between the sources is 6 mm and between the ~ s,
sources and the screen is 2 m . Find the positions of a point
lying between third maxima and third minima where the
intensity is three-fourth of the maximum intensity on the
Now, NS,= dsin8 and
screen.
Solution MS 2 = 52 P -S,P = yd (as done earlier)
D
Since, ·1 = 410 cos'(¾)
~ Llx=ldsin0-~I
3
where l =
4(41 0) = 310 Once the path difference Llx is known, then
),_
~
cos(¾)= '7 FOR MAXIMA, Llx =(2n)
2
, n =0, 1, 2, 3, .....
),_
FOR MINIMA, Llx = (2n + 1) , n = 0 , 1, 2, 3, .....
~ 1=mt±2:.
2 6
2
~ ~=2nn±-" 3
Illustration 16
A coherent parallel beam of microwaves of
Since, ~ = " Llx where Llx = y,d
2 wavelength ).. = 0.5 mm falls on a Young'~ double slit
).. D apparatos. The separation between the slits is 1.0 mm. The
intensity of microwaves is measured on a screen placed
~
2ny 11 d = 2mt±2:. parallel to the plane of the slits ai a distance of 1.0 m from
).. D 3
it as shown in the figure.
~
y.=(n±¼Y~ y

For the point lying between third minima and third maxima,
we have
n=3 -"'--'3,,,o,_'_,-1-4--d_-_-_1_m_m_ _ _--l--+x

~ Y, =(3-.!_))..D
· 6 d

_ 17()..D) l--D=1m
~
Y,- 6 d
(a) If the incident beam falls normally on the double slit
Substitoting ~=0.6x10_. m, D=2 m, d=6 mm, we get apparatus, find the y-coordinates of all the
17 (0.6x 10.. )(2) interference minima on the screen.
Y, 5.67mm (b) If the incident beam makes an angle of 30° with the
6 6x10""
x-axis (as in the dotted arrow shown in figure), find
the y-coordinates of the first minima on either side of
the central maximum.
YDSE WHEN INCIDENT RAYS ARE NOT PARALLEL TO
Solution
CENTRAL LINE
(a) Given A=0.5 mm, d=1 mm, D=l m
In this case, the r~ys reaching S1 and S2 already have an
initial path difference. So, net path difference between the When the incident beam falls normally
rays reaching the point P is given by Path differenc~ between the two rays S2 P and S1P is
Llx=(NS, +S,P)-S2 P Llx = S,P-S,P ~ dsin8

~ . Llx=JNs, -(s,P-S,P)J=INs, -MS,I For minimum intensity,

dsin0=(2n-1).!:, n=l, 2, 3, ....


2

= ================================:::J
2.12

www.puucho.com
Rahul Sardana with www.puucho.com

Wave Optics

~ ~inS=(2n-1);\. (2n~1)0.5 2n~1 Path difference between the rays after passing through
2d 2x1 4 the slits S1 .and S2 is
S,P-S1P=l1x2 =dsin8
So, net path difference is given by
l1x = INS, +S,P-S,PI = [NS, -(S,P-S,P)[

~ l1x = 111x, -11x, I= 1~·- d sin el


S;S, =d(«D) For first minima, we have

l1x =!:
2
Since, sin 0 :5: 1
~ (2n-1) ,, ~
l~-dsin81 =~
1
4
~ n:,2,5 ~ .'!_-dsin8=±!:
2 2
So, n can·be either·1 or 2
~
1 - 8 =±-
--sm ,.
When n=l,wehave sin91 =..!. 2 2d
4
sin8=!+_.?:_
tans,= =1
·v15
~
2 2d
Since d=1 mm, ;\.=0.5 mm
·.. . . 3
When, n ~ 2 , we have sin 02 =-4 _ . 1 0.5
- => Stn 8 =-+-
2 2
3'
~ tan8, = r.;
- v7 - ~ sin8=!+!
2 4
Since, y=Dtan8=tan8(D=1 m) 1 3
·0 .=-
sm andsm' 8 =-
So, the position of minima will be 4 4
Since the position of first minima on either side of
y, =Dtan81 =
1
=
v15
m central maxima is
3- tan8 =Jl
Y, = Dtan8, = .ff m D
~ y=Dtan8
Since, minima can be on either side of centre O , so
there will be four minimas at positions ± _,;.. m and For sin8=!,wehave tan8= ,;..
v15 4 v15
3 . 1
±.ff m on the scr~e_n. ~ y= ../ts m.

For sin8=~, we have tan8= ~


(b) Path aiffe~~nce between the rays before· entering the
slits S1 and S, is 4 · v7
. d 3
NS1 =l1x1 =dsin(30°)=- ... (1) -~ y= .ff m
2
y
YDSE FOR SOURCE NOT PLACED AT THE CENTRAL
LINE. . .
If the source of light is not placed at the central line but a
little beyond (slightly up or down) the central line, then the
waves reaching S1 and S, will already have an initial path
difference.
Path difference of waves meeting at the point P is
l1x=(SS, +S,P)-(SS1 +S1 P)

================================== = 2.13

www.puucho.com
Rahul Sardana with www.puucho.com

Advanced JEE Physics Optics & Modern Physics

=> tu=(Ss,-ss,)+(S,P-S,P) To calculate the order of interference, we shall calculate

=> Lu=y'd+yd
D' D
n=- ,.
L,x

0.035x10-a m
=> n
5000x10-10 m

---------::?lpfy => n=70


=> tu =701-
_J__ So, 7Qth order maxima is obtained at P .
0
(b) At O, Lu= y,d = 10-2 mm= 0.01 mm
D,
Now, we observe that Lu= 201-
Screen So, 20th order maxima is obtained at O .
where, y' is the distance of the source S above or below the (c) (µ-l)t=0.01 mm
central line and D' is the distance of S from S1 and S2 •
=> t = O.Ol = 0.02 mm = 20 µm
Similarly, here once we calculated the path difference, then 1.5-1
A, Since the pattern has to be shifted upwards, therefore,
FOR MAXIMA, Lu= (2n) , n =0, 1, 2, 3 .....
2 the film must be placed infront of S1 .
A,
FOR MINIMA, Lu=(2n+1) , n=0, 1, 2,3 .....
2 Illustration 18
In the YDSE the monochromatic source of
PROBLEM SOLVING TRICK(S)
wavelength ),, is placed at a distance '!_ from the central
At the position of central maxima, we-have 2
ill(.= 0 axis (as shown in the figure), where d is the separation
between the two slits S1 and S2 •
Illustration 17 n
In the Young's Double Slit experiment a point source
of ),, = 5000 A is placed slightly off the central axis as
sJ1_----:::::7f\1Pi
_shown in the figure.
-----------· O i

1 mm
t ~
t 1+ £ =1.5 m+r<---D =2 m---+'0
(a) Find the position of the central maxima.
(b) Find the order of interference formed at O .
(c) Find the minimum thickness of the film of refractive
. (a)Find the nature and order of the interference at the index µ =1.5 to be placed infront of S2 so that
point P.
intensity at O becomes !th of the maximum
(b) Find the nature and order of the interference at O . 4
(c) Where should we place a film of refractive index intensity.
µ =1.5 and what should be its thickness so that a Given A.=6000 A and d=6 mm.
maxima of zero ·order is placed at O .
Solution
Solution
(a) (Lu)".= 0
(a) The optical path difference between the two waves
arriving at P is y,d = y,d
=>
D1 D,
Lu= y,d + y,d = (1)(10) + (5)(10)
D1 D, 103 2xl0 3 =>
=> tu= 3.5 x 10-2 mm= 0.035 mm

= 2.14 i:::::===================================
www.puucho.com
Rahul Sardana with www.puucho.com

Wave Optics

d 6 d'
~ y=-=-=4mm => "-missing (
) , where n = 0 , 1, 2, 3, ...
1.5 1.5 2n+l D ·
(b) At O , net path difference is given by By putting n = 0, 1, 2, 3 .... Missing wavelengths are
d2
y,d
llX=-=--
(f)<a) (6xl0")
2
d' ,d'
A.=D, 3D' 5D""
D, D, 2xl.5
~ ,ix =12x10-' m Illustration 19
~ lix=120x10- m 7
White light is used in a YDSE with separation
Since, 1,, = 6000 A = 6 x 10- m 1 between the sources to be 0.9 m and separation between
the sources and the screen to be l_ m. Light reaching the
~ lll:=201,,
screen at position y =1 mm is passed through a prism and
So at O , the bright fringe of order 20 will be obtained.
its spectrum is obtained. Find the missing lines in the
visible region of this spectrum.
(c) I=Im.,cos'(¾)
Solution
Path difference is given by
¾rm., =I-cos'(¾)
,ix= yd= (9xl04 )(1x10-') = 900 nm
D
1 = 2'.
2 6
For minima Ill:= (2n + 1)~, where n = 0 , 1, 2, 3, ....
~=~=(2;)tµ-l)t
A,=~= 1800
2000 A (2n+l) (2n+l)
t= A, 6000
6(µ-1) 6(1.5-1) ,_ _ 1800 1800 1800 1800
missing- 1 1
3 1
5 1
7
MISSING WAVELENGTH IN FRONT OF ONE SLIT IN Of these, 600 nm and 360 nm lie in the visible range, so these
VOSE will be missing lines in the visible spectrum.
Suppose P is a point of observation in front of slit 51 as
shown. The wavelengths missing are the ones obtained by SUSTAlNEDINTERFERENCE
using the condition of destructive interference, i.e., The interference pattern in which the positions of maxima
A, and minima remain fixed is called a sustained interference.
iix=(2n+l)-, where n=O, 1, 2, 3, ....
. 2
CONDITIONS FOR SUSTAINED INTERFERENCE
Now iix=.JD'+d' -D
a) The fundamental condition for sustained interference is
that the two sources should be coherent i.e., the initial
iix=D[(1+~:)½-1] ... (1) phase difference between the two interfering waves
must remain constant with time.

Since, D»d, so (1+ ~: l Sel+ ~,


2
b) The amplitudes of the two waves should be equal or
nearly equal. This will give good contrast between
bright and dark fringes.
So, (1) becomes c) The two sources should be very closely spaced,
d' A, otherwise the fringes will be too close for the eye to
iix = - = (2n + 1)-
2D 2 resolve.
d) The sources should be monochromatic, otherwise there
will be overlapping of interference patterns due to
different wavelengths, which will reduce are contrast.

.
l J----- -:..
'~
e) The frequencies of the two interfering waves must

f)
equal.
The sources should be nanow.
Since hvo independent sources cannot be coherent, a
sustained interference pattern can be obtained only if the
,____ D _ _ _..
two sources simultaneously and, therefore, the phase
difference behveen them remains constant.

==================================== = 2.15

www.puucho.com
Rahul Sardana with www.puucho.com

Advanced JEE Physics Optics & Modern Physics

ORDER OF FRINGES
If the slits are verticat as shown in figure, path difference is, n=8
If d =101. (say)
M=dsiri0
n=9
---+---+-----------\ n = 1O

n= dcose
).
n=1Oat0=O'
s,~
Screen CONCEPTUAL NOTE{S)
To calculate ,the .number of maximas or minimas that can be
This path difference increases as 0 increases. obtained on the screen, we use- the fact that value of
The order of fringe H is given by· sine(or case) can never be greater than 1'. For example in the
first case when the slits are vertical.
dsin0 = HA.
sin9=~ {for maximum intensity}
=> n=-- ,.
dsin0 d
$ince, sine'/> 1.
The order of fringe increases as we move away from point
=> ni. ~ 1 => n~-
d
0 on the screen. d i.
d
Suppose in some ,question comes out say 3.6 then total
n=2
t s, ~
).
number of maxihlas on the screen wm be 7. Corresponding to,
n=1
n = O, ±1, ±2, ±3.
d _ _,.,__ _ _ _ _ _ _ ___, n = o
So, highest order of interference maxima is
:t .s,~
nm~ =[P.]
).

n - dsin
- ).
9 where [ ] represerts the greatest integer function. So, total
number ofmaxlmas obtained are ~
n=Oate;,0°
N=2nmax +1
When the slits are horizontat as shown in figure, then the Similarly, highest order Qf interferenc~ minima is
path difference is
ax= dcose nm,, =H:+f]
. nP where [ ] represent~ the greatest integer function. So, total
number of·minirTlas obtained are
N =.2n~;n

Illustration 20
~ c
s, s, II Two coherent narrow slits emitting light of
M-d-+I u wavelength A. in the same phase are placed par.Jlel to each
other at a small separation of 2,. . The light is collected on
· Screen
a screen S which is placed at a distance D(» ,.) from the
This path difference decreases as 0 increases slit S1 as shown in figure. Find the finite distance x such
The order of fringe H is given by that the intensity at P is equal to intensity at O .
dcos0 = HA.
ep f
:::::::>
dcos0
n=--- ,. X

The order of fringe decreases as we move away from point ~1-. - - - ~ 2_ _ _ _ _ _ _ _ _ _ _ _ _ _ _ 0 *


0. f+-2).-+l

>+---- D - - - - < " i i

s
= 2.16

www.puucho.com
Rahul Sardana with www.puucho.com

,, Wave Optics
Solution
Path differen_ce for w9ves reaching.at O is pf
s,o - s,o = 21,. y

i.e. maximum interisity is obtain_ed at O .


1 -----e<"'------+------{J C
S2 S1
i.
Next maxima will be obtained at point P where, i+-nA-tot+--D
S1P-S2 P=1,,

')1/2 "D+L2
Now, S1P=(D 2 +y'iv'=D l+JL, ( D 2D
... (2)
2
~\ Similarly, S2 P = (D + d) + ( y ) ... (3)
2 D+d
S1 -,_ S2
t<t-d°---..(
Substituting (2) and (3) in (1) i.e., S2P-S1P =(n-1)1., we get
i+----D----+IIJ d Y'd (n--1)1,, · ·
s 2D(D+d)
=> dcose = A. Since, d =n'J...

=> (21,,)cos8=~ y' (nA.)


=> n1. n1.-1,,
1 2D(D+n1,,)
=> cos0=-
2 ny'
=> 1
=> 8=60' 2D(D+n1.)
Now in AS1 PO 2D(D+n1.)
=> y=
n
PO =tane
s,o
OPTICAL PATH
=> ~=tan60'=v'3 It is defined as distance travelled by light in vacuum in the
D
same time in which it travels a. given path length in a
=> x=v'3D medium. If light travels a path length d in a medium. at
d .
speed v , the time taken by it will lie t = - . So, the optical
CONCEPTUAL NOTE(S) V
At point O ·, pa,th difference is 2A. i.e. we obtain seco~d order path length is
maxima. At point P , where path difference is l. (so, x = ../3D )
we get first order maxima. The next, i.e., zero order maxima will
be obtained where'path difference, ~cos ff= O
OPL=cl=c(;)=(;}=µd I·--µ=;)
Since, for all media µ > 1, optical path length is always
=> 9=90°
greater than geometrical path length.
=> X4CO
When two light waves arrive at a point by travelling
so; our answer, i.e., finite distance of x should be x=../3D., different distances in different media, the phase difference
Corresponding to first order maxima. between the two is related by their optical path difference 1
instead of simple path difference. So,
Illustration 21 21t .
Phase Difference = T( OPL).
Two point sources are d =nA apart. A screen is held at
right angles to the line joining the two sources at a.distance
D from the nearest source. Calculate the distance of the DISPLACEMENT OR SHIFTING OF FRINGE PATTERN IN
point on the screen, where the first bright fringe (excluding YDSE
the centre one) is obser_ved. Assume D » d.
When a transparent film of thiCkness t and refractive index
Solution
µ is introduced in front of one of the slits, the fringe pattern
At point C path difference is n1,, . Therefore, nth bright shifts in the direction where the fiim is placed.
fringe will be observed. Next bright fringe is observed where How much is the fringe shift?,
path difference is (n-1)1,,, so
. Consider the YDSE arrangem~nt shown in the figure.
S,P-S,P=(n-1)1,, ... (1)

2.17 =
www.puucho.com
Rahul Sardana with www.puucho.com

Advanced JEE Physics Optics & Modern Physics

difference is (µ -1)t.
(µ-i)t .
d) If shi~ is ·equivalent to n" fringes then n =- - or
),,

t=~·
(µ-1)
e) The Sh!ft, tlx is i_ndependent of the order of frihge n., i.e.
µ, t Shift Of.Zero order maxima = Shift of nth order maxima:
Shift is independent of wavelength.
~s,
Illustration 22
l+-----D - - - ~ . , Interference fringes are produced by a double slit
A film of thickness t and refractive indexµ is
arrangement and a piece 1>f plane parallel glass of
placed in front of the lower slit. refractive index 1.5 is interposed in one of the interfering
beam. If the fringes are displaced through 30 fringe widths
·The optical path difference is given by for light of wavelength 6 x 10_. cm, find the thickness of
x = [(S,P-t) + µt]-S,P the plate.
=> x=(S,P-S,P)+t(µ-1) Solution
Path difference due to the introduction of glass slab is
Since S2P-S1P=dsin0
ru:=(µ-l)t
=> x=dsin0+t(µ-1)
Thirty fringes are displaced due to the introduction of slab.
Since sin 0 ~ tan 0 = Y:D So,
Llr=3OA.
d' => (µ-l)t=3OA.
=> x=-1&+t(µ-1)
D
3OA. 3Ox6x10·5
The maxima will be obtained when the path difference-is an => t=--=----
µ-1 1.5-1
even multiple of ,?:. i.e.,
2 => t=3.6x10__, cm

x=(2n)
,.
2 Illustration 23
,. d ' In Young's double slit experiment using
=> (2n)-=-1&+t(µ-1) monochromatic light the fringe pattern shifts by a certain
2 D
distance on the screen when a mica sheet of refractive
=> y;= n~D -(µ-l)t~ index 1.6 and thickness 1.964 microns is introduced in the
path of one of the·interfering waves. The mica sheet is then
In the absence of film, the position of the nth maxima is removed and the distance between the slits and the screen
given by equation is·doubled. It is found that the distance between successive
nAD maxima (or minima) now is the same as the observed
Y, =-d- fringe shift upon the introduction of the mica sheet.
Calculate the wavelength of the monochromatic light used
Therefore, the fringe shift (FS) is given by in the experiment.

FS=y,-y;c'~(µ-l)f=t(µ-l)t \-:~="'~) Solution


Shifting of fringes due to introduction of slab in the
Note that the shift is in the direction where the film is path of one of the slits is given by
introduced. , (µ-l)tD
uy d ... (1)
CONCEPTUAL NOTE(S) Now, the distance between the screen and slits is doubled.
a) 'The "enlife pattern shifts t6wards the side where the Pl.ate
is:intrOduced and there is no other change in the pattern~ Hence, the new fringe width will become
b)'. TO aj~asJ:Jrei this, shift .:,Vhit8 li9ht•·.must be used: bec1;1use P' = ,_(2D) ... (2)
, With, monochrom8.tic light,all the fringes will exactly~ be ·d
slinilar.and hehce- no,Shift.Can"b8 observed. " · '
~ . ~ . ,,
Given, !:!ly = J3'
(.

c) .The·8ffective pat~ in air'is inCf~ased by an a,rnount'' (J..L-:.1)t'


.~due~tojntroduction oMhe ,PlateJe .. jhe :additionai' 'path

= 2.18 ===::::;::============================::::i
www.puucho.com
Rahul Sardana with www.puucho.com

Wave Optics
(µ-l)tD 1.(2D) Illustration 2S
d d A Young double slit apparatus is immersed in a liquid
1,. (µ-1)1 (1.6-1)(1.964x10-<) of refractive index µ 1 • The slit plane touches the liquid
2 2 surface, A parallel beam of monochromatic light of
wavelength 1,. (in air) is incident normally on the slits.
=> 1,. = 0.5892 x 10-< m = 5892 A
(a) Find the fringe width
(b) If one of the slits (say S2 ) is covered by a transparent
Illustration 24 slab of refractive index µ2 and thickness t as shown,
In a YDSE, the two coherent sources are separated find the new position of central maxima.
from each other by 6 mm and from the screen by 2 m . A (c) Now the other slit S1 is also covered by a slab of same
light of wavelength 6000 A is used. A film of refractive thickness and refractive index µ3 as shown in figure
index 1.5 is introduced in front of the lower slit such that
the third maxima shifts to the origin. due to which the central maxillla recovers its position
find the value of µ 3 •
(a) Find the thickness of the film.
(b) Find the positions of the fourth maxima.
Solution -sl
(a) Since third minima shifts to the origin, therefore, the
fringe shift (FS) is equal to three fringe widths i.e., 3P, !-=:-~'i--------" ---- ()--
so we have
*=: s. s
FS=y, =3(1,.~) 1
'-1+------,D----
Since we know that the fringe shift (FS) is given by (d) Find the ratio of intensities at O in the three
conditions (a), (b) and (c).
FS=(µ-1/~
Solution
(µ-l)tD = 3 i..D (a) Fringe ·width is given by
d d
P=i..'D =i..D
d µ1d {-: i..·=~}
t=~
µ-1 (b) Position of central maximum is shifted upwards by a
Since, 1,. = 0.6 x 10-< m , µ = 1.5 distance

t (3)(0.6x10-<) = 3 _6 µm (µ,-l)tD
dY d ... (1)
1.5-1
(b) There are two positions of fourth maxima, one above (c) Downward shift is now given by
and the other below the origin. So, we have
( µ'-1J10
~ d

~
~
•• ... (2)
i Since the central maxima recovers its position, so
f ~ -------------- Yf,
1 O=n=n=4
3 dY=dy'
t ===n=2 So, from (1) _and (2), we get
~ y', n=O
( µµ'1 -1Jm
i.---D---..
l ==:::,n=4
(µ1-l)tD
d d

c:::::::::::::J Bright Fringe => µ3 =µ2


µ,
1'D
y,=1P=d=0.2mm and => µ3 =µ1µ2

y; =-7P=-7(,.~)=-1.4 mm (d) Since, I=I=,cos'(t)

where ~ =(~" ),u:


2.19 =
www.puucho.com
Rahul Sardana with www.puucho.com

Advanced JEE Physics Optics & Modern Physics

1
0 ¼=(f)t1x tan(20°)=-
2.J2
Solution
o I oc cos'(¼) ..
(a) The situation is in .the problem 'is shown in the figure.
In the first and third case, ·/:u = 0 while in second case, So, we observe that separation behveen the source_s is
/:u = (µ2 -l)t • Therefore, the desire•ct"ratio ~I
µ=3/2

'
f 1 :f2 :f3 =1:cos '("(µ,-l)t)·
,. . .1. . I I y

PROBLEM SOLVING TECHNIQUE(S)


=~=7:-'=~-++.:---'·-~·P. Lx
If. two thin plates are also inserted ju,~t after S1 an~ S2 , then_ our
first task is to find the path difference. 1n th!3: figure shown, path
of ray 1 is more.than path of ray 2 by'a distance, X=O
I.
X=0.03 m
I
X=1.03rn
,;x, = d sin a and ,;x, = (µ, -1) t, 1+- D = f m --+I '
and path of ray 2 is greater than path of ray 1 by a distance. d=2x0.9=1.8mm
ill<,=dsin~ and ill<,=(µ,-1)t, , :\. 900
Since, we !mow that :\. = - = = 600 nm
Therefore, net path difference is, µ 3
ill(= (ill<,+ ,;x,)-(,;x, +,;x,) • 2
Applying Snell's Law at the x = 0 interface, we get
3 sin(30°)
2 sina
. 1
sma=-
3
0 a= 20' {Using Trigonometry}
Central
Line
Initial path difference is

l1x =dsina=(1.si(½)=0.6 mm
2
Once, we know the path diff~fence illC , then 2 2
0 $= " l1x " .. x0.6xl0" =-2000n
l. )..' 600xl0
FOR MAXIMA, ill<= (2n) , n = 0, 1, 2, .....
2
0 f=f-cos'(½)=r=
FOR MINIMA, ,;x =(2n +1) l. , n = 0, 1,,2, .....
2 (b) Net path difference at P is now
15
Illustration 26 l1x=(0.6 mm)+( -1)(4.~ mm)=0.3 mm
1.4
A large opaque sheet placed parallel to the yz plane o $=10001t
at x = 0.03 m . The region x ;e 0 is filled with a transparent
:::::) [=[max
liquid of refractive index ~2 . A wide monochromatic
.
beam
Illustration 27
of light of wavelength 900 nm falls on the yz -plane at
In a Young's Double slit Experiment, the light source
x = 0 making an angle of 30° with the x-axis. The sheet is at distance 11 = 20 µm and 12 = 40 µm from the slits. The
has two slits parallel to z-axis at y = ±0.9 mm. The
light of wavelength ;\. = 500 nm is incident on slits
intensity of the wave is measured on a screen placed at
separated at a distance 10 µm . A screen is placed at a
x = 1.03 m parallel to the sheet.
distance D = 2 m away from the slits as shown in figure.
(a) Finci the intensity at a point P on the screen where
(a) Find the values of 0 relative to the central line where
y=z=0.
maxima appear on the screen?
(b) The lower slit is covered by a transparent strip of (b) How many maxima will appear on the screen?
refractiv~ index 1.4 and thickness 4.2 mm. Now find
(c) What should be minimum thickness of a slab of
the intensity at point P.
refractive index 1.5 be placed on the path of one of the
ray so that minima occurs at C ?
.= 2.20 :::=====::::::::::::::::::::::::==:==:;:;::::::::::::::::;:::::::::::::::=::::====::::=:::::=::::::::::;::::::::::::::::::::=::::::====::J

www.puucho.com
Rahul Sardana with www.puucho.com

t - Wave OpHcs

incident at an angle <I>= 30° on a diaphragm having narrow


slits at a· separation d = 2 nim . The screen is placed at a
s I, I distance D = 40 cm from slits. A mica slab of thickness
s, t=5 mm is placed in front.of one of theslits and whole
e, the apparatus is submerged in water. If "the central ·bright
fringe is observed at C, calculate

s,I
Solution ~~f
1", + A, ld _______,, C
(a) The optical path difference between the beams arriving
at P is given by
s,J!
ru: = (1, ;-1,) + dsin8 -/1.,---- D ---->'-"- Screen
The condition for-maxnr{um intensity is,
(a) the refractive index of the slab.
d.l'=nA.,wher~ n=O, ±1, ±2, ...... .
(b) the distance of the first black line· from C. Both
=> sin8 =¾(ru:-(1, -11 )) =¾{n1.-(l2 -4)) wavelengths are in air. Take µw = _!.
3
1
=> sin8 _. (nx500x10-,-20x10~) Solution
lOxlO
(a) To observe bright fringe at C, the mica slab should be
=> sin8=2(; -1) placed in front of S2 • In that case, net path difference at
0 C is,
=> sine =,,':.-2 -
20 ru: = dsin<I>-(µ, - l)t

=> 8=sin·'(; --'2) For central bright at C we have


0
ru: = o
(b) Since we know that 0

lsin0kl => dsin4>=(µ,-1)t


-l) = dsin<I> (2xl0·3 )sin(30°)
=>
0

-1,;(; -2},1- => ( µ, I 0.2


0 5x10-'
=> -20:,(n-40),;20 => µ, =1.2
=> 20,;n,;60
=> µslab =1.2
Heri.ce, number of maxima obtained is • µw
N=60-20=40
=> µ, .. =1.2µ. =1.2(¾)=1.6
2 2
(c) At C,phasediffere~ce, <1>= "~=( ")(1,-4)
1. · 1. _ (b) A black line is formed at the position where both the
2 wavelengths interfere destructively. Distance of nth
=> <!>=( " _,)(20x10-<)=801t dark fringe from C is given by
500x10
Hence, maximum intensity will appear at C . (2n-1)1.D
y
For minimum intensity at C , we have 2d
. (2n,-1)1.;D -(2n,-1)1.;D
(µ-l)t=!:'. For blacklme,
2 2d. 2d ·
- ,.. sooxio... = 500 ~- -where_ t..; .and t..;_ are wavelengths in water.
=> I
2(µ-1) 2xo.s · ,.,
A; · µw A1 4000
=>·-=-=-=--
,.; ~ ,., 5600
Illustration 28
In a Young's double slit experiment a parallel beam µw
containing wavelengths 1.., = 4000 A and 1. 2 = 5_600 A Substituting these values in equation (1), we get

2.21 =
www.puucho.com
Rahul Sardana with www.puucho.com

• Advanced JEE Physics Optics & Modern Physics

2n,:...1=z: y 2 =asin(rot+$)
2n2 -1 S
y, = asin( rot+ 2$)
.. For minimum value n, = "4 and n, = 3
• , r • y, =asin(rot+3$)
Henc!', d,istance of first blacJ< line is given by
(2x4-1)(4000x10-1~)40x10-2 x3 ·
y y. = asin[rot+(N-1)$]
2x2x10-' x4
At angle 0, the path difference b~tween any -two successive
=> y=2.lx10-1 m
slits is tu = d sin 8 . So, the corresponding· phase difference $
=> y=0.21mm is given by
2 2
MULTIPLE SLIT- INTERFERENCE PATTERN $=( ; }~= :(dsin8)
Let us now look at the case where we have a general number
N of equally spaced slits, instead of two equally spaced RESULTANT WAVE AMPLITUDE (USING PHASORS)
slits. AB an assumption, we have. shown in figure. a se.t-up of
The· above set of equations can be represented by phasor ·
six equally spaced slits.
diagram shown in figure (for a set Of six sources generalised
to N sources).
p

.,/ 4>\
r/ i
./ R .. F
.,' -----
:~~-~:::______ t
, 1 ' ........
4'
:
I \
\
\
', ',
......... .
Set up fur 6 Equally Spaced Slits r/ \ ,\ ...... D

Similar to the N = 2 case discussed ,'' '\ '\ ii


' '' '
already, we will make the far-field
assumption that the distance of the
...... 'A' a '
sources from the screen is much larger
than the total span of the slits, which is If R be the amplitude of the resultant of N interfering
(N -1)d. We can then say, as we did in waves, then from above phasor diagram, on extracting
· the N = 2 case, that all the paths to a triangles DAG and· DAB, we get following figures to be
given point P on the screen· have used for evaluation of R .
approximately the same length in a G 0
multiplicative (but not additive) sense, -~ ,1\
which implies that the amplitudes of
-~
~------ I '"
~

y
the interfering waves are all essentially o,-i'.&! B.2 /i:1\
L.)' .. 2 / 2 I2 \
equal and we can also say that all the / : \
paths are essentially parallel (because of :t11'
far-field assumption). A close-up ' 2
r ''I
I '' ''
I
''
\

'' ' ''


. •
.
'''
.
version near the. slits is shown in figure.
Also, each path length is _dsin8 longer
'
: 2
A
A'
' 1 ''
'B.
than· the one just above it. So the lengths take the form of
r. =r1 +(n-l)dsin0.
~I I+- .2. ---+t+- .2. --+I
2
M
2

To find the total wave at a given point at·an angle 8 on the DAG and DAB are isosceles triangles so for triangle OAG ,
screen,. we need to add up the N individual waves. The we have
Procedure is the same as in the N = 2 case, except that now
we simply have more terms in the sum. If a_ be the - ¾=rsin(~$) ... (1)
amplitude due to an individual source, then the equations of Triangle OA~, we have
waves interfering at the point P' are given by
· y1 =asin(rot) ½=rsin(½) - . . .. (2)

= 2.22 ==================================
www.puucho.com
Rahul Sardana with www.puucho.com

Wave Optics _
Dividing (1) and (2), we get

R sm(~)

a sm(½)
~ / [ (1:~.,m:Hl
y=a1~ e'·'
ClC'lJ]- 1

So, the resultant amplitude R is given by

,., [,{~',.m)J[,{-',.m_,-{½)
;!) _e-{1¥)]]
R=-[s:ntT]]
... (3) ~ y=alm (e ) [
If IR be the resultant mtensity, then
IR =R2 ~
_ [ {•••<N-ll½)[sm(~)ll
y-alm e . ($) . sm -
2

~ 1• =•'[sm(~ )]' But Im [ e


{•«(N-1)!))
2 =sm ( rol+(N-1) $)
2
sm(½)
If 10 be the intensity due to an individual source, then

... (4) Agam here, we have directly come across the amplitude of
the resultant wave given by

CHECKPOINT
For N=2,weget
-[sm(~)l
R-a ($) .
sm -
2

la =I,[ s:(iJJ' sm(~)]'


~ 1. 1[
= 0 sm(½) , where I'.= a'
Since sin~= 2sin(½)cos(½)
We observe that at the centre of screen, IR is indeterminate.
=> Ia =4I,cos'(½)=4a'cos'(½)
So, the·maximum intensity at the midpoint of the screen i.e.,
at 0 = 0' is obtamed by taking the limit when $--+ 0'.
RESULTANT WAVE EQUATION 2
(Please note that smce $= i_"(dsm0), so $--+0° when
To find the resultant wave equation, we shall be using the
concept of complex numbers. From our knowledge of 0 --+ 0° ). So,
complex numbers, we know that
e'•' = cos(rot) + ism(rot)
~ Im(e""')=sm(rol) and Re(e""'),=-cos(rot)
Smee the resultant wave equation is obtamed by adding
individual Waves, so we get
y = a[sm(rot)+sm(rot+ $)+ ..... + sm{rot+(N -1)$)]
=> y = aim[e;"''.+ ei(mt+t) + ..... + ei(m1+(N-tlt)]
~ y = a!m[e'"'(l+ e'1 + ..... + ,,cN-1l+ )]

.
Smce, l +r+r , + ..... +r N-1 =1(1-r")
-.-
- 1-r

================================== 2.23 =
www.puucho.com
Rahul Sardana with www.puucho.com

Advanced JEE Physics Optics & Modem Physics

LOCATION OF SECONDARY MINIMA(S) l(a)


I(O)
IR has zero values, when

. '(N$)-o
Slil -
2
- 0.8

0
=> N$ = Integral Multiple of 1t
2 -
4
N$ -
=> T=m1t, where m=0, 1,2,.3, .... .2

=> $ = (2m).':. -5, -2, ·31t -Sn -n -3n -n 0 2:_3,c7t5'1t31t 2" "
N 2 2 4 42 2 4 42 '"
2
N=4
=> $=Even Multiple of .':. ... (5)
N
!(a)
However, one exception to this is when !2 is also an integral l(O)

multiple of 1t, because the denominator in equation (4) is


also zero.

So, f=m'rr.,where m'=0,1,2,3, .....

=> $=(2m')1t
=> $ = Even multiple of 1t ... (6)
So, from (5) and (6), we conclude that f, = 0, when

$ = (2m); excluding $ = 0, 21t, 41t, 6,i, .....


Positions of Primary Maxima ---------------------·
CONCEPTUAL NOTE(S)
a} It is customary· 'not to deal With the resultant intensity
i.e., $=(Even Multiple); , excluding Positions of Primary alone, buJ rather fo deal with the resultant inte!]sity relative
Maxima (located at 0, 21t, 41t, 61t , ....) to the maximuni intensity i.e.,· _!a_
•, - ,lmax

LOCATION OF SECONDARY MAXIMA(S)


·I I. . (N$)]'
sin2
~
To find the locations of the secondary maxima (i.e., small ... (1)
bunips) we have to find the local maxima of I, by taking the [
1, : = 1: = Nsin(½)

derivative I, w.r.t. $(i.e., ~;) and then equating it to zero .. b) lim (J,,__) =+--.o
o--.o- Imax
lim (..!e...) =;
~

So,
dl, =0
c) ?- has"a periodicity of 21t in cl> i.e., repeats itself for

d$ """
integratmultiples of 21t .
d) The nUmber of .zeros between the ~fnain peaks· is, (N-1) ,
=> Ntan(½)=tan(~$) where N is the number of slits used.
This equation has to be solved numerically. However, for e) The number of secondary maxima (little bumps) between.
the main peaks is (N-2), where N is the number of slits
large N , the solutions of $ are generally very close to odd
used. ~ ·
multiples of _2:_ excluding the values of q> = 21t±~, because For N=2 (for two slits)~· ~e gBt,ZERO Secondary Maxima..
N N For N = 3 (for 3 slits), we get One Secondary Maxima. .
these values will be lying well within the primary maxima
For N =4 (for 4 slits), we getTwo Secondary M~ima a~d
region. Just to make you understand, we are plotting
I, so on.
I, Q A point worth noting here is that the height of the
(with$ for N=4 and N=8). se~ondary maxima (~ittle bui'nps).i.e., the bump sizes are
symmetric around cj> = tt (o( in general. any, multiple of n:·).
_ _ Also,_ we know that since -·----------•

= 2.24

www.puucho.com
Rahul Sardana with www.puucho.com

Wave Optics

,, =,, [ s~~n~r]r
Hence, the bump size is shortest at 4' =1t , because then
-I
=1 t
=1
d

d
t
-
the denominator in the equation (1 ), will be having a
maximum value at 4, = re , due to which IR becomes the
i
least at 4' = n . Furthermore, the bump size grows as they D
get closer to the main peaks, as shown for various slits
taken. (a) the distances from P where intensity reduces to zero.
(b) the distances from P where next bright fringe are
Single slit observed.
./.__ (c) the ratio of intensities of bright fringes observed on
the screen.
Solution
N=2 (a) In case of three slits, intensity becomes zero, when
_ _._"-~>-<--'-_,.,_"--i---'"-'--+-'-..,_~-"-<-- $ phase difference between any two waves is,
1' Primary: Secondary
: ______ :maximum: 21t
maximum 4'=2mt+ , where n =0, 1, 2, ....
-- ----- ...
------ 1 I
3
-- -- The corresponding pa~ difference, ~=( :J$
N=3

~ 2
dsin0=U'J( t+zim)=n1.+½

.. ----- ------ ------ -- ........


-- --- -- .... -- sine=~( n+½)

''
$-2•
$' -3
A, A,

-- .......- ------ ------ ---- .....


................ ................ A,
N=5
For small angles, sin 0 "' tan 0 = 1l
D

~ ;=~(n+½)

~ y=,.~(n+½)
Substituting the values, we have
g) As N , the number of slits, is increased, the primary
maxima (the tallest peaks in each graph) become narrower
but remain fixed in position and the number of secondary Y
500x10_,_;
0.5xl0
2(n+.!:.)=
3
2x10-a(n+.!:.) m
3
maxima increases. For any value of N , the decrease in
intensity in maxima to the left and right of the central
maximum, indicated by the blue dashed arcs, is due to ~ y=2(n+½) mm,wh~re n=0,1,2, .....
diffraction patterns from the indfvidual slits, which can be
,__ _n-"e"'gl~cted he~ ________ 2 8
=> y=-mmforn=0,y=-mmforn=letc.
3 3
Illustration 29 (b) Bright fringes are obtained on the screen where
A light wave of wavelength 500 nm falls upon three (i) $=2n1t, n=l,2,3, ....
slits a distance 0.5 mm each from one another. A screen is
placed at a distance 2 m from slits. Find ~=(:")($)=n1.
~ dsin0=n1.

2.25 =
www.puucho.com
Rahul Sardana with www.puucho.com

Advanced JEE Physics Optics & Modem Physics

. 0 =n1.. camera cannot be obtained. To obtain a fixed interference


sm -
d pattern we must have two sources Which either have no
For small angles, phase difference or have a constant difference of phase.
These sources are called coherent sources. It has been
sin8" tan8 =JL generally observed that coherent sources are obtained when
D they are derived from the same parent source. The methods
}!_ = n1.. for obtaining coherent sources (derived from the same
D d parent source) are given below.
n1..D n(soo x 10-,)(2)
y A. DOUBLE SLIT METHOD
d (0.5x10"')
Light from a source S is limited to a narrow beam with the
y=2nx10"' m help of a slit. The emergent light is made to fall upon a
y=(2n) mm {where n =1, 2, 3, .... etc.} screen containing two slits S1 and S2 placed symmetrically
There are called priniary maximas. with respect to the slit. Here, both S1 and S2 are illuminated
(ii) ~=(2n+l)it, n=l,2, .... by the same wavefront. Therefore the beams of light coming
out from S1 and S2 have no phase difference. Thus S1 and
Proc_eeding in the similar manner, we get
S2 can· be treated as the coherent sources. Young used this
y=2(n+½) mm {where n=l,2,3, ....} technique in his famous Young's double slit experiment.

These are called secondary maxim.as.


D
Note that y = 0 is also a secondary maxima
because at P, ~= 1t. \?-------~-·--
(c) At principal maxirnas, we have ~ =21t, 41t, .... , etc.
s,
----.A,
D
----.A, => - - - - - ~ A= 3A, Coherent sources by
----.A, Double Slit Method

Resultant amplitude R = 3A, B. A SOURCE AND ITS OWN VIRTUAL IMAGE


=> I, =910 {·: I oc A') Light from a source S is made to fall on a plane mirror M .
Point of observation P on a screen AB receives direct light
While at secondary maximas, ( ~ = 1t, 3n, 51t..... )
as well as light reflected from M. To an observer, reflected
A , - ----.A, light appears to come from a source S1 (virtual image of S ).
-----A=A,
----. A, So interference at P takes place between waves coming
from S and S' . Since S' is not an independent source,
Resultant amplitude, R' = A,
being the virtual image of S , it will have the same phase as
=> I~ =10 S . Hence the two are taken to be coherent sources. Lloyd
So, the desired ratio is therefore, 9 : 1 made use of this arrangement in Lloyd single mirror
experiment.
COHERENT SOURCES BY DIVISION OF WAVEFRONT A
p
When two or more waves travel through a medium
simultaneously, the resultant intensity at any point, in the (/)
medium depends on whether they interfere constructively or s 2 ~
destructively which, in tum, depends upon the phase rn
difference between them. Resultant intensity, at any point, M "
remains constant with time if the phase difference between
them does not change. Two inP,ependent sources can never
have same phase or a constant phase difference, because if
we try to have interference with two independent sources, C. BIPRISM METHOD
then net intensity at any point undergoes a continuous Light from a source S is made to fall on an assembly of two
change due to a change in the phase difference between right angled prisms A and B joined base to base as shown
them. As a result of this no fixed interference pattern can be in Figure. S1 and S2 are the virtual image of S produced by
observed. The interference pattern of such sources is so
refractiqn through prisms A and B respectively. Being
short-Jived that its photograph with the fastest available

= 2.26

www.puucho.com
Rahul Sardana with www.puucho.com

Wave Optics
virtual images of the same source, S1 and S2 have same a) Determination of D : It is the distan~e between source
phase and henc~ can be treated as the coherent sources. This . and screen. 'It can be ~easured with an ordinary metre
type of arrangement is made use of in Fresnel's biprism rod.
experiment. b) Determination of p : A low power travelling
microscope is used to find the total separation x
between a number of fringes, say 20 and hence p=.=... .
20
c) Determination of d : d can be calculated by using
displacement method. A convex lens is placed in
FRESNEL'S BIPRISM between the biprism and the screen. It is observed that
for two positions I.., and L2 of the lens, the images of
It is one of the convenient laboratory arrangements for
producing interference fringes. It consists of a combination S1 and S2 can be focussed on the screen AB . Let x
of two right angled prisms with their bases joined together and y be the distances between these images when the
so that their faces are inclined to each other at angle of lens is at I.., and L, respectively. Then,
179° 20' . Source of light is taken in the form of a narrow slit
d=..{xy
S, illuminated by the monochromatic light and is held
symmetrically at a distance of about 5 cm from the biprism. P, L, L, A
Light from S gets refracted by prism P1 and P2 , thereby, s,. "
II '
' I
producing virtual images S1 and S2 , which can be taken as ''' I
I

''
two coherent sources })reducing interference. Light beams s,.
from S1 and S2 strike the screen in the regions ED and FG I I Screen
'VI
respectively. EF is the common region where both the P, B
beams can be found. Therefore interference pattern can be
observed in the region EF .
Substituting for p, D ~d d in equation (1), 1,, can be
calculated.

LLOYD'S SINGLE MIRROR


This experimental set-up for producing interference fringes,
was devised by Dr. Lloyd in 1834. Light from a source S1 in
the form of narrow slit is held in such a way that the light is
incident, at almost grazing incidence, upon a mirror MM''
which is blackened at the ba'ck to avoid internal reflections.
S2 is the virtual image of source S1 obtained after reflection
B from MM' . Screen AB is placed to receive light coming
directly from S1 as well as that reflected from the mirror.
The separation ,,between these sources may be found by
using the formula for deviation caused by a thin prism. If u Reflected light can be supposed to be coming from source
is the small angle of- biprism, µ refractive index of material S2 • DF is the common region on the screen where both the·
of biprism and a the separation of source S from biprism, beams are received and hence interference is obtained in
then deviation c~used by prism. region DF.
A' A
o=(µ-l)u
D
From Figure, d = 2ao = 2a( µ-1 )u

s,
DETERMINATION OF 1,, '
'
Biprism method can be used to determine the wave length of ---~-----
d• ,,.
light. The fringe width p for the interference pattern : ,,' t-4----
s, I--,:"-----
obtained is given by,
P=1,,D ,.__ _ _ D------,----
d
'
B' B
A= Pd ... (1) Experimental set up for Lloyd's single mirror
D

2.27 =
www.puucho.com
Rahul Sardana with www.puucho.com

Advanced JEE Physics Optics & Modem Physics

The point C lies symmetrically w.r.t. S1 and S2 and also (AS')(BS') (190+5+5)(0.1)
AP, 4 cm
lies outside the interference region, zero order fringe is not BD 5
visible. It can be seen by moving the screen to position A'B'
so that it just touches the mirror. It will be observed that the
zero order fringe at M' is dark instead of being bright as
demanded by the theory of interference fringes since at M'
path difference is zero. Tilis indicates that the beam which
suffers reflection from MM' undergoes in phase of
1t-radian.
Llyod's single mirror can be used to determine the wave
length of light. Similarly in triangles BCS' and S'P,A, we have
H, a is the height of source S1 above MM', then
AP, AS'
d=2a BS'= BC
If, D is the distance of source S1 from screen AB, then fringe (AS')(BS') (190+5+5)(0.1) cm
AP, 2
width p is given by BC 10
P=i,,o => P,P, =AP, -AP1 =2 cm
d • C 3x10 8 -7
Wavelength of the light 1'.=-=--1-4 =5x10 m
1'.= pd= p(2a) f 6xl0
D D
p can be determined, experimentally, by using a low power Fringe width p = i,,~
microscope. Knowing p, a and D value of 1,, can be
Since, D=S'A =(190+5+5)=200 cm=2 cm,
calculated.
d=SS'=2 mm=2xl0" m
CONCEPTUAL NOTE(S) p (5xl0-')(2)
5xl0-4 m=.0.05 cm
Central spot, in case of Lloyd's single mirror is a dark one 2x10"
instead of being bright. This proves that there is a phase change
of n-radian when a transverse wave (light) is reflected from a
Number of fringes is
denser medium. N= P,P, =40
p
Illustration 30
The arrangement £or a mirror experiment is shown in Illustration 31
the figure. S is a point source of frequency 6 x 10 14 Hz. D Two flat mirrors form an angle close to 180° . A source
and C represent the two ends of a mirror placed of light S is placed at equal distances b from the mirrors.
horizontally and LOM represents the screen. Find the interval between adjacent interference bands on
Determine the position of the region where the screen MN at a distance OA = a from the point of
fringes will be visible and calculate the number of fringes. intersection of the mirror. The wavelength of the light
wave is known and equal to 1,, • Shield C does not allow
the light to pass directly from the source to the screen.
N

190 cm-+: M A
5 cm 5 cm

Solution
M
Fringes will be observed in the region between P, and
Solution
P2 because the reflected rays lie only in this region.
Fringe width is given by
From similar triangles BDS' and S'P2 A AP, = AS' P=1'.D
' BS' BD d
where D=AB~a+b and d=S,S 2

2.28 i::::=====================================
www.puucho.com
Rahul Sardana with www.puucho.com

Wave Optics

B. TRANSMITTED LIGHT
s,r----- , In transmitted light system there is no phase difference or

i l
I
:
di 81 ____ 0'
-- •
..... .._
----a.,..._ s
-----'!.-t--------------- A
path difference due to reflection or transmission as all
reflections take place from rarer medium.
I
'
1 ......
\

--r; -- _-".,. So, the effective path difference is


x=2µtcosr
82~-----
For maxima
I+------ D - - - - - . a 2µtcosr =.n;>,
and for minima
In !1S15B, we have
2µtcosr=(2n-1J.
,. , n=l,2,3, ...
.'!.=2b~ 2
2 2
Obviously, the conditions of interference in reflected and
d=2ba
transmitted lights are opposite to each other, therefore if the
p ;>,(a+b) film appears dark in reflected light, it will appear bright in
2ba transmitted light and vice versa. With the use of white light,
the colours visible in reflected light will be complementary
THEORY OF DIVISION OF AMPLITUDE to those visible in transmitted light, i.e., the colours absent in
one system will be present in the other system; the sum of
two constituting the white light.
A. REFLECTED LIGHT
If µ is the refractive index of material of film of thickness t,
Illustration 32
then path difference between the waves abc and abdef is
2µtcosr
A thick glass slab (µ = 1.5) is to be viewed in reflected
white light. It is proposed to coat the slab with a thin layer
Additional path difference due to reflection at denser
of a material having refractive index 1.3 so that the
medium ( at b) is .'.: wavelength 6000 A is suppressed. Find the minimum
2 thickness of the coating required.
a C
Solution
Optical path difference for the reflected light from
coating and slab is
l1x = 2µ!
For minimum intensity, A/----jBf
,.
2µ,t =- Cl µ,~1.3 iDl
2
t=l:....= 6000
m n 4µ 1 4xl.3
So, effective path difference is

x=2µtcosr+-
,.
2
For maxima or constructive interference to take place, we
--------- - - - - - - . . ·-···------3
.=> t=1154A

CONCEPTUAL NOTE(S)
Both reflected rays (one from AB and the another from CD)
have

2µtcosr+
,. =(2n) ,. -
get a phase change of 1t •
- ------·-·- ------·
2 2
=>
,.
2µtcosr= (2n-1) , n =1, 2, 3, ...
Illustration 33
A parallel beam of white light falls on a thin film
2 4
whose refractive index is equal to - . The angle of
For minima or destructive interference to take place 3
1,, ,. incidence i = 53° . What must be the minimum film
2µtcosr+ = (2n + 1)
2 2 thickness if the reflected light is to be coloured yellow ( ;>,
=> 2µtcosr=nt.., n=l,2,3, ... of yellow = 0.6 µm) most intensively? Given tan53° = i,
3

==================================== = 2.29

www.puucho.com
Rahul Sardana with www.puucho.com

Advanced JEE Physics Optics & Modern Physics

Solution
According to Snell's Law, we have BASED ON INTERFERENCE
sini (Solutions on page 2.86)
µ=-.- 1. In a Yourig1s Double Slit Exp~riment.carried out in a liqµid Qf
smr
retr8.ctive index· µ = 1.3 , a thiri fi1in of air is formed in front
4 37° •
of the lower slit as shown in the figiJre. If a maxima of third
4 sin(53°) 5 5 4
0
order is f0rmed at the oriQin O ; find th_e -
=> 0

3 sinr sinr · (a) thickness of the air film.


(b) positions of the fourth·maxima.
3
=> sinr =- 53' The wavel8ngth Of light i'n air
'is A. 0 _= 0:78 µrn ~ and·
5 3 D .
=> r=37° d=1090.
From Figure (a) :
2 .. sJ µ=1.3 ~
I L--t----- ·:__·. --. _. -;~o
,I
I i s,U ~Airfilm_ · ~
I E, D ~
D
(a) 2. In YDSE , if light of wavelength
0

:sooo A is used, find the


thickness of ~•glass slab (µ .=1,5) ;.,hlch should be piac~d
' . , . ,..
Path difference between 2 and 1 is ru; = 2(AD) . ~ .
befor~. the uppe;' the upper, sli( S1 so that the-' c~r\tral
=> ru:1 = 2BDsecr = 2tsecr
maximum now lies at a point where ·sii bright fringe was
Their optical path corresponding to ru:1 is 2µtsecr lying earl!er (bef_ore inserting the slab);: 1

From Figure (b) : 3. A source S of wavelength A is kept directly behind the ~lit
S1 in a double slit apparatu$. Find.the phase.difference at a
E
point O which Is equidistant- from · ~ 1 and S2 • If D » d ,
2
what will be -~he phase difference ·-at. P if a· liquid Of
refractiv_e index; µ is filled
C (a) between the screen ar1d·the Slits?
(b) , between the slits and th0 s9urce -· s?

fi
fils,j_·· . IP· OP=d/2
1
D
(b)
Path difference between 1 and 2 is given by
-,~r ~-------io
5
'1,__ _ _ D o
ru: 2 = ACsini = (Zttanr)sini ---+1

=> (ru:),,. =ru:1 -ru:2 =2µtsecr-2t(tanrHsini)


'4. _In Solar"cells, a smcdn Solar- c;li'-°{µ =:= 3.5) 18 cocite~·wiih .a
4 5 3 4
=2x-xtx--2xtx-x-
=> .6x
,~ 3 4 45 r thin film "of- silicon- monoxide "SiO(µ =1~45) t9 ITliniITli_?:e
reflective lcisses from the s·urtace. 'Determine the mil1imum
=> L,\xnet
32
= 15 f I thickness .of· SiO that ProduC~s the le_ast reflectiOn ··a( ·a
wavelength of 550' nm , near the .ceflt_re of the- visible
Since reflection takes place at the surface of denser medium, spectrum. • · · ·
so phase difference between 1 and 2 is 1t • I
5. A parallel beam of ·gr·een iighl' ·of waVelerlgth · 546 'nm
So, for constructive interference, we have
passes -'through a slit of width 0.4 mm ., The tran?l"!litted:
32t=~ llgh_t is .. collected on a screen 40 cm .away;· Find the·
15 2 distance between the two first order· ITlinima.

=> I= 151'. = 15x0.6 =O.l 4 µm Calculate the ·minimum. thickness Ct a soap bubble film
64 64 (µ =1.33) that results in construt;;tive interference in the

= ==================================
2.30

www.puucho.com
Rahul Sardana with www.puucho.com

Wave Optics
reflected light if the filiTI is illuminated -With light whoSe ~ (b) The (monochromaticf so\Jrce is· replaced by another
wavelength in free space is A= 600 nm. (monochromatic) source of shorter wavelength
(c) The separation between the two slits is increased
7. Monochromatic light of wavelength 5000 A is used in {d) The monochromatic source is replaced by source of
YDSE , with slit separation 1 mm , distance between white light
(e) The whole experiment is carried out in a medium of '
screen and slits 1 m . If intensity at the two slits are,
refractive index µ
11 =41 0 , 12 =1 0 , find
(a) fringe width ~ . 14. In a Young's double slit experiment, the slits are 2 mm
th
(b) distance of 5 minima from the central maxima on the apart and are illuminated with a mixture of two wavelengths
screen. A, = 750 nm and A. 2 = 900 nm . At what minimum distance :
(c) intensity at y = i mm. from the common central bright fringe on a screen 2 m
from the slits Will a bright fringe from one interterence ,
111
(d) distance of the 1000 maxima. pattern coincide with a bright fringe from the other?
(e) distance of the 5000th maxima.
15. Bichromatic light is used in YDSE having wavelengths ,
8. S1 and S2 are two point sources of radiation that are A. 1 = 400 nm and A= 700 nm . Find minimum order of A,
radiating waves in phase with each other of wavelength
which overlaps with A2 •
400 nm . The sources are located on x-axis at x = 6.5 µm
and x =-6 µm , respectively. 16. In Young's double slit experiment set-up with -light of :
(a) Determine the phase difference (in radian) at the
wavelength A.= 6000 A, distance between the two slits is
origin between the radiation from s, and the radiation
2 mm and distance between the plane of slits and the
from 8 2 •
screen is 2 m. The slits are of equal intensity. When a ,
(b) Suppose a slab of transparent material with thickness sheet of glass of refractive index 1.5 (which permits only a ,
1.5 µm and index of refraction µ = 1.5 is placed fraction T\ of the incident light to pass through) and
between x =0 and x = 1.5 µm . What then is the thickness 8000 A is placed in front of the lower slit, it is :
phase difference (in radian) at the origin between the observed that the intensity at a point P, 0.15 mm above the 1
1
radiation from S, and the radiation from S 2 ? central maxima does not change. Find the value of '1 ·

9. In a Young's double slit experiment using monochromatic 17. In a Young's double slit experiment set up, the wavelength
light the fringe pattern shifts by a certain distance on the of light used is 546 nm . The distance of screen from slits is
screen when a mica sheet of refractive index 1.6 and 1 metre. The slit separation is 0.3 mm .
thickness 1.964 micron is introduced in the path of one of (a) Compare the intensity at a point P distant 10 mm
the interfering waves. The mica sheet is then removed and
from the central fringe where the intensity is 10 •
the distance between the slits and the screen is doubled. It
is found that the distance between successive maxima (or (b) Find the number of bright fringes between P and the
minima) now is the same as the observed fringe shift upon central fringe.
the introduction of the mica sheet. Calculate the wavelength
of the monochromatic light used in the experiment. 18. In a Young's double slit experiment, two wavelengths of
500 nm and 700 nm were used. What is the minimum
10. In Young's experiment a thin glass plate is placed in the distance from the central maximum where their maximas '
path of one of the interfering rays. This causes the central ,
light band to shift into a position which was initially occupied coincide again? Take ~ =103 • Symbols have their usual
by the fifth bright band (not counting the central one). The
meanings.
ray falls onto the plate perpendicularly. The refractive index
of the plate is 1.5. The wavelength is 6 x 10-7 m. What is 19. When a thin sheet of a transparent material of thickness
the thickness of the plate?
7.2 x 10-4 cm is introduced in the path of one of the
interfering beams, the central fringe shift to a position
11. In a double slit pattern (1' = 6000 A), the first order and occupied by the sixth bright fringe. If A= 6 x 10--s cm, find
tenth order maxima fall at 12.50 mm and 14.75 mm from the refractive index of the sheet.
a particular reference point. If A is changed to 5500 A ,
find the position of zero order and tenth order fringes, other
DIFFRACTION : INTRODUCTION & CLASSIFICATION
arrangements remaining the same.
When light waves pass through a small aperture, an
12. In YDSE, the two slits are separated by 0.1 mm and they interference pattern is observed rather than a sharp spot of
are 0.5 m from the screen. The wavelength of light used is light cast by the aperture. This shows that light spreads in
5000 A . Find the distance between 7~ maxima and 11"' various directions beyond the aperture into regions where a
minima on the screen. shadow would be expected if light travelled in straight lines.

13. What is the effect on the interference fringes in a VOSE due


to each of the following operations?
(a) The screen is. moved away from the plane of the slits
2.31 =
www.puucho.com
Rahul Sardana with www.puucho.com

Advanced JEE Physics Optics & Modern Physics

phase of the light at each point in the aperture is the same.


This can be achieved experimentally either by placing. the

r-I--I--J---r---0
-+11).~-1
- - -d--- -- -- ---
observing screen at a large distance from the aperture or by
using a converging lens to focus parallel rays on the screen,
!______ -- --- -- Uniform
as in Figure. Note that a bright fringe is observed along the
axis at 8 = 0, with aitemating bright and dark fringes on
t + - - - D ----+1 intensity either side of the central bright fringe. ·
distribution
Common examples : Diffraction at single slit, double slit and
Light passing through two slits does not produce two diffraction grating.
distinct bright areas on a screen. Instead, an interference
pattern is observed on the screen which shows that the light
has deviated from a straight-line path and has entered the
otherwise shadowed region. Other waves, such as sound
waves and water waves, also have this property of being
10
able to bend around corners.
This deviation of light from a straight-line path is called
diffraction. Diffraction ·results from the interference of light
from many coherent sources. In principle, the intensity of a
diffraction pattern at a given point in space can be computed
using Huygens' principle, where each point on the
11:.'
Incoming
-
Screen
wave
wavefront at the source of the pattern is taken to be a point
source.
The phenomenon of bending of light around the comers of B. FRESNEL'S DIFFRACTION
an obstacle/ aperture of the size of the wave.length of light is When the observing screen is placed at a finite distance from
called diffraction. The phenomenon resulting from the the slit and no lens is used to focus parallel rays, the
superposition of secondary wavelets originating from observed pattern is called a Fresnel Diffraction Pattern.
different parts of the same wave front is define as diffraction Fresnel diffraction is rather complex to treat quantitatively.
of light. Diffraction is the characteristic of all types of waves. Common examples : Diffraction at a straight edge, narrow
Greater the wave length of wave higher will be it's degree of wire or small opaque disc etc.
diffraction.

?, I
o~ -------- ---------

I+--- D ----+1
!j Slit Screen

CONCEPTUAL NOTE($) A Fresnel diffraction pattern of a single slit is


observed when the incident rays are not parallel
Diffraction, can be regarded as a consequence of interference and the observing screen is at a finite distance
from many, coherent wave sources. In other words, t_he from the slit.
phenomena of diffraction and interference are basically
~jv_alc.e_nt_._ _ _ _ _ _ _ _ _ _ _ _ _ _ _ _ _ _
1
FRAUNHOFER DIFFRACTION AT A SINGLE SLIT
Diffraction phenomena are usually classified as being one of Consider that a monochromatic source of light S , emitting
two types, which are named after the men who first light waves of wavelength 1,., is placed at the principal focus
explained them. The first type is called Fraunhofer of the convex lens L,. A parallel beam of light i.e., a plane
Diffraction and the second is called Fresnel's Diffraction. wavefront gets incident on a narrow slit AB of width d as
shown in figure .. The diffraction pattern is obtained on a
A. FRAUNHOFER DIFFRACTION screen lying at a distance D from the slit and at the focal
This occurs when the rays reaching a point are plane of the convex lens L2 •
approximately parallel i.e. when both the source and screen
are effectively at infinite distance from the diffracting device.
In this case, the incident light is a plane wave so that the
C:::::: 2.32

www.puucho.com
Rahul Sardana with www.puucho.com

Wave Optics
SCREEN
SLIT
L,
amount equal to the path difference (f}ine, where d is

J'l------=,..iPt the width of the slit. Similarly, the path difference between
--- y waves 2 and 4 is also equal to (f}ine. If this path
-====::::c=~ ot difference is exactly one half of a wavelength (corresponding
to a phase difference of 180° ), the two waves canCel ·each
other and destructive interference results. This is tr'ue, in fact,
for any two waves that originate at points separated by half
l+-----D------+< the slit width, since the phase difference between two such
According to rectilinear propagation of light, a bright image points is 180° . Therefore, waves from the upper half of the
of the slit is expected at the centre O of the screen. But in slit interfere destructively with waves from the lower half of
practice, we get a diffraction pattern i.e., a central maximum the slit. when
at the centre O flanked by a number of dark and bright,; d . ,_
-sin 8 =-
fringes called secondary maxima and minima on either sid~ 2 2
of the point O . . 8 ,_
The diffraction pattern is obtained on the screen, which lies => Slll =-
d
at the focal plane of the convex lens L2 • It is found that
Similarly, destructive interference (minima) occurs when the
(i) the width of the central maximum is twice as that of a 3 5
secondary maximum and path difference (r!.)sin8 equals 1,., '-, 21,., '-, etc. These
2 2 2
(ii) the intensity of the secondary maxima goes on points occur at progressively larger values of 8. Therefore,
decreasing with the order of maxima. These the general condition for destructive interference is
observations are explained on the basis of the
phenomenon of diffraction using the following ·e =m-,_
sm (m=±l,±2,±3, ... ) ... (1)
mathematical treatment. d

where 1ml,; '!_,_


EXPLANATION & MATHEMATICAL TREATMENT
Consider Frawlhofer diffraction by a single slit as shown in Equation (1) gives the values of 8 for which the diffraction
Figure. Important features of this problem can be deduced pattern has zero intensity. However, it tells us nothing about
by examining waves coming from various portions of the the variation in intensity along the screen. The general
slit. According to Huygens' principle, each portion of the slit features of the intensity distribution along the screen are
acts as a source of waves. Hence, light from one portion of shown in Figure. A broad central bright fringe is observed,
the. slit can interfere with light from another portion, and the flanked by much weaker alternating maxima. The central
resultant intensity on the screen will depend on the bright fringe corresponds to those points opposite the slit for
direction 0 . which the path difference is zero, or 8 = 0 . All waves
originating from the slit reach this region in phase, hence
constructive interference results. The various dark fringes
(points of zero intensity) occur at the values of 8 that satisfy
equation (1). The positions of the weaker maxima lie
approximately halfway between the dark fringes. Note that
the central bright fringe is twice as wide as the weaker
maxima.
2
Angular width of central maxima is '- and. width of central
d
maxima is Z~D , where D is the distance of the screen from

Diffraction of light by a narrow slit of width d. Each portion of the the slit.
slit acts as a point source of waves. The path difference between The intensity distribution of the diffraction pattern is quite
rays 1 and 3 or between rays 2 and 4 is equal to (d/2) sin0
different from the interference pattern produced due to
To analyze the resultant diffraction pattern, it is convenient superposition of light from two coherent sources. The point
to divide the slit in two halves as in Figure. All the waves 0 on the central ax:s is the brightest. The angular position
that origi:rate from the slit are in phase. Consider waves 1 (8) of n"' diffraction minima is given by
and 3, which originate from the bottom and center of the slit, dsin8 = n/,. n=l,2,3,4, ...
respectively. Wave 1 travels farther than wave 3 by an

================================== 2.33 =
www.puucho.com
Rahul Sardana with www.puucho.com

Advanced JEE Physics Optics & Modern Physics


y
I,

Secondary maxima Central


Y, sine= 2/Jd

--lil
+:d'-- -- --- ----· Y, ""Rine - 'i.ld
sine - o ----...,_ I,
--ntralaxis ___ frol~~-~_"_""'
Sec nd
First
__!_,__
.t ce _""'_'---:-~----:,>-ln'-t+ensity (I) 121
y. sin0=-}Jd
lnte nsity 2l. ),_ ). 0 31,_ 2l. 3).
Y2 sine= -2A/d
-er -d d d d d
D-----+<
I
ri r,
I\ '
Plane wavefronts
Screen I l 11
! .. 1
Central

Position of the various minima for the Fraunhofer


diffraction pattern of a single slit of width d. b) As the slit width increases (relative to wavelength) the
width of the control diffraction maxima decreases, that is,
For small angle 0, we have sin 9 .::: 8 . Thus, as shown in the the light undergoes less flaring by the slit. The secondary
figure, the angular position of the 1st, 2nd, 3rd, ... minima are maxima also decreases in width (and becomes Weaker).
2 3 c) If d » A , the secondary maxima dl.Je to the slit disappear;
-'- , -'-, - 1.. , ..... respective
· 1y on e1t
· h er s1"de of the centra1 we then no longer have single slit diffraction.
d d d
d) When ,the slit width is reduced by a factor of 2, the
axis. A maximum is approximately halfway between two amplitude of the wave at the centre of the screen is
adjacent minima. reduced by a factor of 2, so the intensity at the centre is
The central bright fringe is the widest, with width reduced by a factor of 4. ·

P=!:+!:= 21..
d d d
The secondary bright fringes are narrower.
Note that as the slit width d increases, the width of the
central diffraction maximum decreases. That is, there is less
spreading out of the light by the slit. The secondary maxima
also decrease in width and become weaker. When d
becomes much greater than A, the secondary maxima
disappear.
The intensity I of the diffraction pattern as a function of 0 Illustration 34

r
is given as
A slit of width d is illuminated by white light.
[= ro( S~ct
(a) For what value of d, will the first minimum for red
light of 1..=6500A beat 0=15°?
Since sin 0 .=:: ~ , so we get (b) What is the wavelength 1..' of the light whose first
side-maximum is at 8 =15° , thus coinciding with the
ndsin0 "(yd)
first minimum for the red light?
where ct=--,_-"i D Solution
The intensity of secondary maxima is much less. Compared (a) The angular position 00 of nfu minimum is given by
to the intensity of central maximum (I0 ) , the intensity of the dsin0 = ni..
0

first of the secondary maxima is only 4.5%, of the second is Here, n=l, A=6500xl0-10 m, 0=15°
only 1.6%, of the third is merely 0.83% ...... The successive
secondary maxima decrease rapidly in intensity. n1.. lx6500x10°10 ·
=> d sin8n sinl5° ::::: 25
· µm
CONCEPTUAL NOTE(S) (b) This maximum is approximately halfway between the
a) If the intensity of the central maxima is 10 , then the first and second minima produced with light of
intensity of the first and second secondary maxima are wavelength 1..' . Thus, by putting n = 1.5 , we get
found to be ~0 and ~; . Thus diffraction fringes are of
d sin 0 = 1.51..'
1
,., = dsin8 2.5 X lQ-6 X Sin 15°
unequal width and unequal intensities. Hence the ratio of 430 mm= 4300 A
the intensities of secondary maxima to centraJ maxima are 1.5 1.5

-- ~;.!=.i=~---- .---·-------
~ 2.34 r::::=====================================
www.puucho.com
Rahul Sardana with www.puucho.com

Wave Optics
Tilis is the wavelength of violet light. Note that the first
2-_ • Therefore, after travelling a distance D , the diffracted
side-maximum for light of A'= 4300 A will always d
coincide with the first minimum for light of A= 6500 A , b earn acqmres
. . a WI"dh DA
t d.
no matter what the slit-width is.
Geometrical optics is based on rectilinear propagation of
Illustration 35 light, which is just an approximation. We can say that
Angular width of central maximum in the Fraunhofer geometrical optics is valid, if the width ~A of the diffracted
diffraction paHern of a slit is measured. The slit is
illuminated by light of wavelength 6000 A. When the slit is beam is less than the size of the slit, that is
illuminated by light of another wavelength, the angular
DA <d
width decreases by 30%. Calculate the wavelength of this d
light. The same decrease in the angolar width of central
maximum is obtained when the original apparatus is d'
=> D<-
immersed in a liquid. Find refractive index of the liquid. A
Solution
(a) Given ;\, = 6000 A Illustration 36
Let b be the width of slit and D the distance between For what distance is the ray optics a good
screen and slit. approximation, if the slit is 3 mm wide and the
wavelength of light is 5000 A ?
First minima Solution
d' (3x10")' =18 m
D<-
A S000xl0-10
Thus, upto a distance of 18 m , we can assume rectilinear
propagation of light to a good approximation.
----D----i
INTERFERENCE & DIFFRACTION: A COMPARISON
First minima is obtained at bsin0 = A.
INTERFERENCE DIFFRACTION
=> b8 = Asin0 "8
(a) It results from (a) It results from interaction
8 =.?: interaction of light of light coming from
=>
b coming from two different parts of the same
2 different wave fronts wavefront.
Angular width of first maxima = 20 = bA cc A originating from two
coherent sources.
Angular width will decrease by 30% when A is also
(b) Here, the fringes may (b) Here the fringes are
decreased by 30% .
or may not be of the always of varying width.
Therefore, new wavelength same width.

°
A' ={(6000)-(: )6000}=4200 A
00
(c) The
..
nurumum
fringes of (c)
intensity
The fringes of minimum
intensity are not perfectly
are dark (or perfectly dark.
(b) When the apparatus is immersed in a liquid of
dark when waves are
refractive index µ, the wavelength is decreased µ
of same amplitude).
times. Therefore,
(d) All bright fringes (d) The intensity of all the
4200 A = 6000 A possess the same bright fringes is not same.
µ intensity. It is maximum for central
6000 fringe and decreases
=> µ= 4200 sharply for first, second
etc. brightfringes.
=> µ = 1.429 "1.43
(e) An interference (e) In diffraction pattern the
pattern consists a contrast between the
VALIDITY OF GEOMETRICAL OPTICS good contrast bright and dark fringes is
When a slit of width d is illuminated by a parallel beam of between the dark and comparatively poor.
light, the angular spread of diffracted light is approximately bright fringes.

c:================================== = 2.35

www.puucho.com
Rahul Sardana with www.puucho.com

Advanced JEE Physics Optics & Modem Physics

POLARIZATION OF LIGHT PLANE POLARISED LIGHT


According to Maxwell, light possesses electromagnetic It may be defined as the light, in which the vibrations of the
nature. An electromagnetic wave_ consists of varying ele~tric light (vibrations of the electric vector) are restricted to a
and magnetic fields, such that the two fields are mutually particular plane.
perpendicular to each other and to the direction of In a plane polarised light, the vibrations are restricted to a
propagation of waves. The optical phenomena i.e., fixed plane, so that vibrations are perpendicular to direction
phenomena concerning light may primarily be attributed to of propagation of light. Figure (a) represents plane polarised
the vibrations of electric field vector in a direction light having vibrations in the plane of the paper and Figure
perpendicular to the direction of propagation of light. In (b) represents the plane polarised light having vibrations in a
ordinary or unpolarised light, the vibrations of electric field plane perpendicular to the plane of the paper.
vector are regularly or symmetrically distributed in a plane
perpendicular to the direction of the propagation of the light.
111111·
lllll! !
UNPOLARISED LIGHT (REPRESENTATION)
(a)
POLARISED LIGHT (REPRESENTATION)
(b)

In an ordinary ray of light, the electric vibrations are in all ; PROBLEM SOLVING TRICK(S) I
the directions but perpendicular to the direction of The vibrations in plane polarised light are perpendicular to the I
propagation of the light. Such a ray of light is called a ray of
ordinary or unpolarised light. It is schematically
I plane of polarisation. :

represented as shown. The arrows represent vibrations in the , POLARIZATION BY REFLECTION


plane of the paper, while the dots represent vibrations in a
direction perpendicular to the plane of the paper. Polarized light may also be obtained by the process of
reflection. When an unpolarized light beam is reflected, light
The phenomenon, due to which the vibrations of light are
is completely polarized, partially polarized, or unpolarized,
restricted to· a particular plane, is called the polarisation of
depending on the angle of incidence. If the angle of
light.
incidence is either 0 or 90° (normal or grazing angles), the
When ordinary light i.e. unpolarised light passes through a reflected beam is unpolarized. However, for intermediate
tourmaline crystal, out of all the vibrations which are angles of incidence, the. reflected light is polarized to some
symmetrical about the direction of propagation, only those extent.
pass through it, which are parallel to its crystallographic axis
Suppose an unpolarized light beam is incident on a surface
AB . Therefore, on emerging through the crystal, the
as in figure. The beam can be described by two electric field
vibrations no longer remain symmetrical about the direction
components, one parallel to the surface (the dots) and the
of propagation but are confined to a single plane (see
other perpendicular to the first and to the direction of
Figure).
propagation (the arrows). It is found that the parallel
A D component reflects more strongly than the other component,
--------------------------,i and this results in a partially polarized beam. Furthermore,
''
PLANE OF VIBRATION .
,,:~• l
p"'---,---- ' ________ ,S i' PLANE the refracted ray is also partially polarized.
',, i POLARISED Now suppose the· angle of incidence, i , is varied until the
' ',:-- I' ',,: LIGHT
angle between the reflected and refracted beams is 90° . At
I",.... ''',',', .. this particular angle of incidence, the reflected beam is
PLANE OF POLARISATION l
UNPOLARISED '' 17:a_.::.==------------t----- R completely polarized with its electric field vector parallel to
LIGHT '' __________________________ J the surface, while the refracted beam is partially polarized.
'' ''
The angle of incidence at which this occurs is called the
B C
polarizing angle, p .
PLANE OF VIBRATION From figure, we see that at the polarizing angle,
p +90°+ r =180°, so that r = 90°-p. Using Snell's Law, we
The plane (ABCD), which contains the vibrations of plane
polarised light, is called the plane of vibration.· have
sinp
µ=-.-
PLANE OF POLARISATION smr
The plane (PQRS) perpendicular to the plane of vibrations Since sinr = sin(90°-p}= cosp, the expression for µ can be
is called the plane of polarisation. written

= ==================================
2.36

www.puucho.com
Rahul Sardana with www.puucho.com

Wave Optics
sinp polarised light is passed through analyser, the intensity I of
µ=--=tanp
cosp transmitted light varies directly as the square of the cosine of
the angle 0 between the transmission directions of polariser
Incident Reflected Incident Reflected and analyser. Titis statement is known as the Law of Malus.
beam beam beam
Mathematically, according to Malus Law, we have
Ioccos 2 8
~ I =l0 cos2 8
µ, µ,
where I0 is the maximum intensity of transmitted light. It
µ, µ,
may be noted that 10 is equal to half the intensity of
unpolarised light incident on the polariser.
Refracted
beam
EXPLANATION OF THE LAW
(a) (b) Let the planes of polariser and analyser are inclined to each
other at an angle 0 as shown in figure. Let I0 be the
(a) When unpolarized light is incident on a reflecting surface, the
reflected and refracted beams are partially polarized. (b) T.he intensity and a the amplitude of the plane polarised light
reflected beam is completely polarized when the angle of transmitted by the polariser.
incidence equals the polarizing angle ep,· Polariser:
Axis :
This expression is called Brewster's Law, and the polarizing '
angle p is sometimes called Brewster's Angle, after its ,' --............
• • I
fn~~ser
rv..15
discoverer, Sir David Brewster (1781-1868). For example, the ,' ' ....... /
' ' '
Brewster's angle for crown glass (µ = 1.52) is
' ''
p = tan-1 (1.52) =56.7°. Since µ varies with wavelength for a '
' '' a
given substance, the Brewster's angle is also a function of the '
wavelength. ' '' acose
Polarization by reflection is a common phenomenon. ''
Sunlight reflected from water, glass, snow and metallic 0
surfaces is partially polarized. If the surface is horizontal, the asine
electric field vector of the reflected light will have a strong
horizontal component. Sunglasses made of polarizing ""','
material reduce the glare of reflected light. The transmission ' '
Law of Malus
axes of the lenses are oriented vertically so as to absorb the
strong horizontal component of the reflected light. The amplitude a of the light incident on the analyser has
h\To rectangular components,
(i) acos0, parallel to the plane of transmission of the
Illustration 37
analyser, and
A ray of light strikes a glass plate at an angle of 60°.
(ii) asin0, perpendicular to the plane of transmission of
1f the reflected and refracted rays are perpendicular to each the analyser.
other, find the refractive index of glass.
So only the component acos0 is transmitted by the
Solution
analyser. The intensity of light transmitted by the analyser is
Reflected and refracted rays are mutually perpendicular·
only when the angle of incidence is equal to polarising angle, I =k(acos0)'
hence => I=ka 2 cos 2 0
ip =60° => I =I0 cos 2 0
So, refractive index is where 10 = ka 2 , is the maximum intensity of light
µ = tani, = tan60° = ,/3 = 1.732 transmitted by the analyser (when 0 = 0° ). The above
equation is the Law of Malus or Malus Law.
LAWOFMALUS ------- ,, ____________________ _
CONCEPTUAL NOTE(S)
When a plane polarised light is seen through an analyser, the
a) When 0::: 0° or 180°, cos0 =±1
intensity of transmitted light varies as the analyser is rotated
in its own plane about the incident direction. In 1809, E.N. ::::- I =10
Malus discovered that when a beam of completely plane ____ So when the _transmission directions _o_f p9!af'!~er _c~nd
2.37 =
www.puucho.com
Rahul Sardana with www.puucho.com

Advanced JEE Physics Optics & Modern Physics


analyser are parallel or antipar8Jjiil to each other, .the Incident sunlight
maximum intensity of plane polarised light is transmitted by (Unpolarised)
the analyser and is equal' to the Intensity emerging from

b)
the polariser.
When 9=90°, COS9=0 0 --·1-----•-If---+-•t-+•-@ Nitrogen
molecule
=> I= 0
So when the transmission directions of polariser and
analyser are perpendicular to each other, the intensity of
light transmitted through the analyser is zero.
c) When a beam of unpolarised light is incident on the Scattered Light
polariser, then (Polarised)
I= r, (cos' 0)

Since, (cos' 0) = ¾
=> I=&2 "iJ'Eye

Illustration 38
INTENSITY CURVE
Two .,crossed' polaroids A and B are placed in the
As the angle 0 between the transmission directions of path of a light-beam. In between these, a third polaroid C
polariser and analyser is varied, the intensity I of the light is placed whose polarisation axis makes an angle 9 with
transmitted by the analyser varies as a function of cos2 ·8, as the polarisation axis of the polaroid A . If the intensity of
shown in figure. light emerging from the polaroid A is I,, then show that
Intensity I
the intensity of light emerging from polaroid B will be

I, ¼I, sin' (28) ..


Solution
By Malus Law, tl)e intensity of light emerging from the
middle polaroid C will be
L__,.,L..__ ____,,_,L..__ _ _ 8 I1 =I0 cos2 0
90° 180° 270° 360°
This intensity I, falls on the polaroid B whose polarisation
POLARISATION BY SCATTERING axis makes an angle of (90° -8) with the polarisation axis of
the polaroid C . Therefore, the intensity of light emerging
When we look at the blue portion of the sky through a.
from B will be
polaroid and rotate the polaroid, the transmitted light shows
rise and fall of intensity. This shows that the light from the I, =I, cos' (90° -8) =(I, cos' 8)cos' (90° -8)
blue portion of the sky is plane polarised. This is because
sunlight gets scattered (i.e., its direction is changed) when it ~ I, =I,cos 2 8sin2 8=ir,(2sin8cos8)
. 4
2

encounters the molecules of the earth's atmosphere. The


scattered light seen in a direction perpelldicular to the ~ I2 =.!_I,sin2 (28)
direction of incidence is found to be plane polarised. 4
Explanation. Figure shows the unpolarised light incident on
a molecule. The dots show vibrations perpendicular to the Illustration 39
plane of paper and double arrows show vibrations in the
plane of paper. The electrons in the molecule begin to vibrate Two polaroids are placed 90° to each other. What
in both of these directions. The electrons vibrating parallel to happens when N -1 more polaroids are inserted between
the double ar_rows cannot send energy towards an observer two crossed polaroids (at 90' to each other). Their axes are
looking at 90° to the direction of the sun because their equally spaced. How does the transmitted intensity behave
acceleration has no transverse component. The light for large N?
scattered by the molecules in this direction has only dots. It Solution
is polarised perpendicular to the plane of paper. This Transmitted intensity through first polaroid is
.explains the polarisation oflight scattered from the sky. 11 = 10 cos 2 0
where I, is the original intensity. Similarly, the transmitted
intensity through second polaroid will be

= 2.38 c=================================

www.puucho.com
Rahul Sardana with www.puucho.com

Wave Optics
I 2 =11 cos 2 0=10 cos4 0
~-------=-=lC.=E_::11_ __;__ _ _--'---'
If N polaroids are used, then I BASED ON DIFFRACTION AND POLARISATION ·I
IN =I0 (cos0)'N I (Solutions on page 2.89) !
1. A slit of width 0.025 m is placed ·in front of a lens of focal :
As the optic axes of the polaroids are equally inclined, so 1 length 50 cm . The slit is ·. illuminated with light of 1
~gle of rotation e is same for each polaroid. Thus wavelength 5900 A . Calculate the distance between- the !
. .!i;.=(cos0) 2N
I,
centre and first dark band of diffraction pattern obtained on !
a screen placed at the focal plane of the lens. I
. 2. Two spectral lines of sodium D1 and D2 have wavelengths j
Since, angle between successive polaroids is given by !
90° 1t • i of approximately 5890 A and 5896 A. A sodium lamp '
0 = - = - radian sends incident plane wave onto a slit of width 2 micrometre. I
N 2N
A screen is located 2 m_ from the slit. Find the spacing I
For large N , 0 becomes small, so we get between the first maxima of two sodium lines as measured j
on the screen.
( COS 2~ JN =(1- 8~2 + ... JN =(1- 28:~2 + ...) ! 3. In young's double slit experiment, the distance d between
the slits S1 and S2 is 1 mm.. What should the width of ''.
l
which approaches 1 for large N . So, fractional intensity, is
each slit be so as to obtain 10 maxima of the double slit ,
IN =1 pattern within the central maximum of the single slit pattern?
I,
Estimate the distance for which ray optics is a good
:::} IN= Io approximation for an aperture of 4 mm and wavelength
400 nm.

Illustration 40 . 5. Two towers on the top of two hills are 40 km apart. The I
A beam of plane-polarised falls normally on a
I line joining them passes 50 m above a hill half way i
polariser (cross~sectional area 3 x 10---4 m 2 ) which rotates I
between the towers. What is the longest wavelength of i
about the axis of the ray with an angular velocity of radiowaves which can be sent between the towers without \
31.4 rads-' . Find the energy of light passing through the
appreciable diffraction effects? l
polariser per revolution and the intensity of the emergent
I
6. A slit of width d is illuminated·by white light. For what value I
beam if the flux of energy of the incident ray is 10_, W .
I of d will the first minimum for red light (l. =6500 A) fall at !
Solution e
an angle = 30° ? J
2
,
Cross-sectional area of polaroid, A =3x 10--4 rn
A screen is placed 2 m away from a single narrow slit. 1
Angular velocity, ro =3.14 rads-' Calculate the slit width if the firsf minimum lies 5 mm on i
Time taken to complete one revolution, eittier side of central maximum. Incident plane waves have 1
a wavelength of 5000 A.
T= Zit= 2x3.14 =O.Z s
ro 31.4 8. Determine the angular separation between central
(Energy incident/sec)= 10- W 3 maximum and first order maximum of the diffraction pattern
due to a single slit of width 0.25 mm when light of
So, intensity of incident polarised beam is given by wavelength 5890 A is incident on it normally.
Energy incident/sec 10" =10 wm-'
I, Area 3x10-4 3 9. Two polarising sheets have their polarising directions
parallel so that the intensity of the transmitted light is
Since, I= I, cos' 0 where (cos' 0) = ½ maximum. Through what angle must the either sheet be
turned if the intensity is to drop by one-half?

So, average intensity transmitted is 10. A polariser and an analyser are oriented so that the
maximum light is transmitted. What iS the fraction of i
I =10 =~=1.67Wm_, maximum _light transmitted when analyser is rotated through I
<It> 2 3x2
(a) 30° (b) 60°? i
Light energy passing through polariser per revolution is
given by 11. Two pola.roids are crossed to each other. If one of them is
rotated through 60° , then what percentage of the incident
E=IwAT= ~(3x10-4)(0.2)=10-4 J unpolarised light will be transmitted by the polaroids?

12.
Two polaroids are !)laced at 90° to each other and the I
transmitted intensity is zero. What happe.ns when one more .I
L __ pplaroid Js pjaced between these two bisecting_ the_ angle

2.39 =
www.puucho.com
Rahul Sardana with www.puucho.com

Advanced JEE Physics Optics & Modern Physics


-between them? emerging light is 3 Wm_,, what is the angle between the
transmission axes of the first two polarisers? At what angle
13. A polaroid, .examines two adjacerit plane-polarised light
will the transmitted intensity be maximum?
beams A and B whose planes of polarisation are mutually
at right angles. In one position of the polaroid_, the beam B
shows, zero intensity. From this position a rotation of 30°
shows the two beams of equal,inlensities; Find the intensity
ratio ~ of the two beams.
1,

14. Show that when a ray of light is inCident on the surface of a,


transparent medium at the polarising angle, the reflected
and·tral7smltted'rays are perpendicular to each other.

15. Unpolarised light of i~tensity 32 wm--2 pasSes through


I three polarisers such that the transmission axis of the last
L_ upJ)lariser_is .crossed with _the first, it the intensity of t_he.

= t:::=================================
2.40

www.puucho.com
Rahul Sardana with www.puucho.com

Problem 1 Solving, we get


An interference pattern is observed due to two
X =~A.= 1.25A.
coherent sources S1 placed at origin and S2 placed at 4
(0, 3A, 0), where 'A. is the wavelength of the sources. A Hence, the desired x coordinates are
detector D is moved along the positive x-axis. Find the x =1.25A. and x = 4,.
coordinates on the x-axis (excluding· x = 0 and oo) where
maximum intensity is observed. Problem 2
Solution
In given figure, S is a monochromatic point source
At x = 0, path difference is 3,. . Hence, third order emitting light of wavelength A. = 500 nm . A thin lens of
maxima will be obtained. At x-+ oo, path difference is zero. circular shape and focal length 0.10 m is cut into two
Hence, zero order maxima is obtained. In between first and identical halves I., and L, by a plane passing through a
second order maximas will be obtained. diameter. The two halves are placed symmetrically about
y
the central axis SO with a gap of 0.5 mm. The distance
along the axis from S to I., and L, is 0.15 m while that
from ½ and 1 2 to O is 1.30 m. The screen at O is normal
to SO.

s
L,
wi 0.5mm
A

vt
I + - - X ----+I
0
For First order maxima, we have ' '
S2 P-S,P=A- ''' L,
Screen
''
2
.Jx'+9A- -x=A. ''
:....-o.1sm ' 1.30 m
=> .Jx 2 +9A.2 =X+A.
(i) If the third intensity maximum excluding central
Squaring both sides, we get x 2 + 9A 2 =x 2 + A2 + 2xA maximum, occurs at the poin_t A on the screen, find
Solving this, we get the distance OA .
x=4A- (ii) If the gap between I., and L, is reduced from its
For Second order maxima, we have original value of 0.5 mm, will the distance OA
S,P-S,P = z,_ increase, decrease, or remain the same.
Solution
.Jx'-9,.'-x=2A-
(i) For the lens, u = -0.15 m , f = +0.10 m
=> .Jx'+ n' = (x + z,.)
.111
Squaring both sides, we get Thi
ere ore, using - - - =- we get
V U f
xz +9/...2 = xz +4A. z + 4xA.

2.41 =
www.puucho.com
Rahul Sardana with www.puucho.com

Advanced JEE Physics Optics & Modern Physics

1 1 1 1 1
-=-+-=---+--
v u f (--0.15} (0.10)
I
=> V=0.3 m
03
-- tJ:r-·--- ............
~--- 0
----_ C
p
Linear magnification, m = v = · = 2
u -0.15
Hence, two images S1 and S2 of S will be formed at
I.
0.3 m from the Jens as shown in figure. Image S1 due
to part 1 will be formed at 0.5 mm above its optics For central maxima, /J.x = 0

axis (m =-2). Similarly, S2 due to part 2 is formed sin0=(µ-l)t _sin~


d
0.5 mm below the optic axis of this part as shown.
Hence, distance between S1 and 52 is d = 1.5 mm (i-1)co.1)
2
sin0==- ~ - - sin(30°) =!
Also, D = 1.30 - 0.30 = 1.0 m = 10' mm 50x10"" . 2
and ;\.=500nm=5x104 mm => 0=30°
So, fringe width is given by (b) At C, 0=0°,soweget
4 l!vc = dsin~-(µ-l)t
P= ;\.D = (5x10 )(10') mm=! mm
d (1.5) 3
=> l!vc=(50x10-'i(½)-(¾-1)co.1)
Now, as the point A is at the third maxima
=> l!vc.= 0.025-0.05 = --0.025 mm
=> OA=3P=3(½)=1 mm
Substituting, l!vc = n;\., we get
(ii) If the gap between I, and L, is redu,~ed, d will n l!vc --0.025 -50
decrease. Hence, the fringe width p will increase or the A. 500x10~
distance. OA will increase. Hence, at C there will be maxima. Therefore the· order
of minima closest to the C are -49 .
Problem 3 (c) Number of fringes shifted upwards is
Light of wavelength ;\. = 500 nm falls on two narrow
slits placed a distance d = 50 x 10-4 cm apart, at an angle ( i2 -1)co,1J
N=(µ-1)1 ~~.,....,..... 100
~ = 30° relative to the slits shown in figure. ON the lower ;\. 500x10~
slit a transparent slab of thickness 0.1 run and refractive
i~dex ~ is placed. The interference pattern is observed on Problem 4
2 In a modified Young's double slit experiment, a
a screen at a distance D = 2 m from the slits. monochromatic uniform and parallel beam of light of
wavelength 6000 A ~d intensity (1;) Wm_, is incident
normally on two apertures A and B of radii 0.001 m and
0.002 m respectively. A perfectly transparent film of
thickness 2000 A and refractive index 1.5 for the
wavelength of 6000 A is placed in front of aperture A

(a) Locate the position of the central maxima.


(b) Find the order of minima closest to centre C of
screen.
--
(shown in figure).

(c) How many fringes will pass over C, if we remove the


transparent slab from the lower slit?
Solution
(a) Path difference is given by
l!vc = dsin~+ dsin0-(µ-l)t -
~ 2.42 i::::::===================================
www.puucho.com
Rahul Sardana with www.puucho.com

Wave Optics
Calculate the power (in W) received at the focal spot F of Solution
the lens. The lens is symmetrically placed with respect to
the apertures. Assume that 10% of the power received by Applying the lens formula _!:. _ _!:. = l_, we get
each aperture goes in the original direction and is brought V U f
to the focal spot. 1 1 1
-+-=-
Solution v 5 10
~ v=-lOcm
Power received by aperture A is given by
V -lQ
2 ~ m=-=-=2
PA= I( rrr,;} =: (it)(0.001) = 10_, W {: I=:} u -5
i.e. two virtual sources are formed with distance between
1
Power received by aperture_ B is given by
them
10 2
P, = I( itr;) = (it)(0.002) = 4x 10C5 W d=0.Smm
" Lens
Only 10% of PA· and P, goes to the original direction, so
s, P,i"
Portion of PA going to _original direction is P1 = 10-' W
Portion of P, going to original direction is P, = 4 x 10-' W
''
'' •L
1i
s 0
''
Path difference created by slab is given by
1;x =.(µ-l)t = (1.5-1)(2000) = 1000 A
s''
' 5 a,:
!
~
Corresponding phase difference is given by
i+-- 10 cm - - - - - - D. -

~=(:")1'x= 6~; x1000=i F. "dth"p 1'.(D+lO)


0 nngew1 d
Now, resultant power at the focal point is given by
Fringes are observed between the region P and Q (waves
P = P1 + P, + 2JP1 P, cos~
interfere in this region only), where

~ P = 10-< + 4 xlO-' + 2J(10-<)(4xlQ-' )cos(i)


L d
-=-
D 10
P=7x10-' W L=Dd
10
Problem 5 Number of fringes that can be observed on the screen is
A central portion with a width of d = 0.5 mm is cut given by
out of a convergent lens having a focal lengt:ti of L d'D
f = 10 cm , as shown in figure. Both halves are tightly N p 101'.(D + 10) ... (l)
fitted against each other. The lens receives monochromatic Substituting the values, we get
light (,_ = 5000 A) from a point source at a distance of = (0.05) D
2

5 cm from it. 3
10x5x10-5(D+ 10)
Solving this equation, we get

(\
-V
D=15 cm
From equation (1), we have
d'
N
10,.(1+~)
N will be maximum when D-), oo
(a) At what distance should a screen be fixed on the
opposite side of the lens to observe three interference ~ 10--> 0
bands on it? D
2
(b) What is the maximum possible number of d' (0.05)
interference bands that can be observed in this ~ N = 5
- 101'. 10x5x10-5
installation?

2.43 =
www.puucho.com
Rahul Sardana with www.puucho.com

Advanced JEE Physics Optics & Modern Physics

Problem 6
fil = dsinu =_!__ mm
400
A point source is placed at a distance !!.... below ·the
2
Since, ~ = 2,_" fil
principal axis of an equiconvex lens of refractive index ~
2
and radius 20 cm . The emergent light from lens having
~ 2" x_!__xlO_,
5000 X 10-lO 400
wavelength :>. = 5000 A falls on the slits S1 and S2 ~=10n
separated by d = 1 mm which are placed symmetrically
along the principal axis. The resulting interference pattern Since, I =Im,.cos'(t)
is observed ori the screen kept at a distance D =1 m from
the slit plane. => I=Imax

Problem 7
The interference pattern of a Young's · double slit
experiment is observed in two ways by placing the screen
in two possible ways as shown in figure (a) and (b). Th_e
distance between two consecutive right most minima on
the screen of figure (a) using light of wavelength
:>., = 4000 A is observed to be 600 times the fringe width in
(a) Find the position of central maxima and its width
(b) Find the intensity at point O. the screen of figure (b) using the wavelength :>., = 6000 A .

Solution
If D (as shown in figure) is 1 m then find the separation
between the coherent sources S1 and S2 • Given that
Using Lens Maker's Formula, we get
d>31c,_
7=(%-i)(z~ - -~o) 2

~ /=20cm SI' S1 S2 =d sI S,S,=d


Since source lies in focal plane of lens. So, all the emergent
rays will be parallel. So,
d/2 d 1 .
s,I
}-o I
tana.=-=-=-:;::sma
20 40 400 Screen s,I I ""~
Initial path difference (fil)ini.., = dsinu ;,• en
0

(a) (b) "


Solution
Had the screen been perpendicular to S2 P, then P and
Q' would have been the positions of first and second
minima Qast two). .,

'''
s) ''
'
Let the central maxima is obtained at angle 8 . Then ''
----------~o
dsin8 = dsinu ---::,·-"=<·- i d/2
_ __,___....:::,,__=:===,..:'~:!_screen
sin8 =sina. s
'
I ----·--10·
',P
tan8 = tan a.
- - - - x, - - - . i '
'
}f_ =_r!__
D 40 '
Since the angular positions of minima do not depend on the
1

Dd position of the screen, so the second minima is formed at Q


y=40 on the screen.
100 For right most minima at P , we have
y=-d=2.5d=2.5mm
40 dsin01 = A.1
. 2 ... (1)
At O , net path difference is given by

= 2.44 ===============================
www.puucho.com
Rahul Sardana with www.puucho.com

Wave Optics
For small angles, we have (b) Calculate the minimum value of t for which the
2 intensity at point P on the screen exactly in front of
sin el "' tan e, = d/
x, the upper slit becomes maximum.
Substituting in equation (1), we get Solution

d' (a) Net path difference of the waves reaching at Q, is


X2=- ... (2)
A1 l!.x = yd+ y'd
D D'
.For next minima at Q , we have

dsin0 2 =¾J.. 1 ... (3)

For small angles, we have


d
sin92 ~ tan9 2 =_1_
x,
Substituting in equation (3), we get
,.._ D-..l.,1,_s,_ D' _J
d' For central maximum, Ax = 0
X2=- ... (4)
3)..1 , D',
=> y =--y
2d' D
~, PQ=X1-X2=- ... (5)
3)..1

In the second case, fringe width is given by


=> y' = -(f)to.Ssin(1tt))
~=J..,D => y'=-sin(1tl) mm
... (6)
d
Since it is given that PQ = 600~
(b) y 1
=!!... , at point P exactly in front of 51 , so we have
2

=> 2d' = 600 J..,D


3)..1 d l!.x=(yd)+ ( ~)
=> d' = 900J.. 1J.. 2 D D D'
For maximum intensity, we have path difference to be
=> d 3 = 900 X 4000 X 6000 X 10-20 X 1
=> d' =216x10-" an even multiple of !:: , so
2
=> d=6x10-4m=0.6mm
l!.x = (2n)!: = nJ..
2
Problem 8 Substituting the values, we get
In a Young's double slit experiment set-up source S 0.5sin(1tt)+ 0.25 = 0.5n
of wavelength 5000 A illuminates two slits S1 and S2 , . ( rt I) 0.5n - 0.25
=> sm
which act as two coherent sources. The source S oscillates 0.5
about its shown position according to the equation For minimum value of t, we have n =1
y=O.Ssin(nt), where y is in millimetres and t in => sin(1tl)=0.5
seconds.
1t
y => nt =-
6

Is, Lx ~
- ----- - --- --- -- ---- p
=>
1
I= - = 0.167 s
6

s
1 mm Problem 9
i s 2 Two parallel beams of light P an\i Q (separation d )
i+1mm•1• 2mm,-~.,, containing radiations of wavelengths 4000 A and 5000 A
(which are mutually coherent in each wavelength
(a) Locate the position of the central maxima as a function separately) are incident normally on a prism as shown in
of time. figure. The refractive index of the prism as a function of

2.45 =
www.puucho.com
Rahul Sardana with www.puucho.com

Advanced JEE Physics Optics & Modern Physics

(b) For, 4000 A condition of TIR is just satisfied_ Hence, it


wavelength is given by the relation, µ(:\.) = 1.20 +..!', where ,. will emerge from AC, just grazingly_
A is in A and b is posiHve constant. The value of b is A
such that the condition for total reflection at the face AC
is just satisfied for one wavelength and is not satisfied for ,
,,
the other. /
A c - S;;,90°-C
8 ,/
p
r
d
_ sin8=0.B
B For4000 A C

~ go•
So, deviation for 4000 A is given by
~ 6.,.,A =90-i=90-sin-1 (0.8)~37°
B C
(a) Find the value of b For 5000 A , we have
(b) Find .the deviation of the beams transmitted through b 8xl05
µ=1.2+---,=1.2+---, =1.232
the face AC '- (5000)
(c) A convergent lens is used to bring these transmitted A
beams into focus. If the intensities of the upper and
the lower beams immediately after transmission from
the face AC, are 4I and I respectively, find the
resultant intensity at the focus.
Solution
(a) Total internal reflection (TIR) will take place first for
those wavelength for which critical angle is small or µ B For 5000 A C
is large.
siniair
From the given expression of µ, it is more for the Applying, µ
sinimedium
wavelength for which value of :\. is less_
A 1.232 = sin ia1r = sin iarr
sin9 0.8
8
, ,, i., = 80.26°
,,
So, deviation for 5000 A is given by
i-= 8,
,, 65000 A =i.,,,-im<di= =80.26°-sin-1 (0.8)=27_13°

(c) The intensity of the upper beam (4000 A) and lower


B C
beam (5000 A) after transmission are 4I and I
Thus, condition of TlR is just satisfied for 4000 A respectively, then
~ i=C for 4000A. I, =I1 +I, +2.p;i; cos~
~ 9=C
Since no phase change takes place for the waves
~ sin9=sinC refracted from the lens, so qi = 0° .
Since sin 9 = 0-8 and sinC =_!_,so we get ~ r.=4I+I+2.J(4Ilicos(0°)
µ
~ I• =9I
0.8 = _!_ (for 4000 A.)
µ
Problem 10
1
~ 0.8 b A glass plate of refractive indeX: 1.5 is coated with a
1.20+-- thin layer of thickness t and refractive index 1.8. Light of
(4000)2 wavelength :\. travelling in air is incident normally on the
Solving this equation, we get layer. It is partly reflected at the upper and the lower
surfaces of the layer and the two reflected rays interfere.
b = 8_0 x 10' (A)' Write the condition for their constructive interference. If
A= 648 nm , obtain the least value of t for w~ch the rays
interfere constructively.
2.46 ::::===================================
www.puucho.com
Rahul Sardana with www.puucho.com

Wave Optics

Solution Problem 11
Incident ray AB is partly reflected as ray 1 from the Shown in the figure is a prism of refracting angle 30°
upper surface and partly reflected as ray 2 from the lower and refractive index µ, (=-Ji). The face AC of the prism
surface of the layer of thickness t and refractive index
µ 1 =1.8 as shown in figure. Path difference between the two is covered with a thin film of refractive index µ1 ( = 2.2) . A
rays would be monochromatic light of wavelength A =550 nm falls on
1'x =2µ 1 t =2(1.8)t =3.61 the face AB at an angle of incidence of 60° . Calculate

Ray 1 is reflected from a denser medium, therefore, it


undergoes a phase· change of n, whereas the ray 2 gets
reflected from a rarer medium, therefore, there is no change
in phase ofray 2.
Hence, phase difference betweei, rays 1 and 2 would be
6.~ = 7t. Therefore, condition of cOnstructive interference will µ, = 2.2
be
'
1'x=(n-½)A where n=l,2,3, ... (a) the angle of emergence.
(b) the minimum value of thickness t of the coated film
- so that the intensity of the emergent ray is maximum.
=> 3,6t=(n-½)A
Solution
A 1 2
(a) Applying Snell's Law, we get A

B t sini1 = µsinr1

' => sin(60°) = ,J3 sinr1


µ.= 1.8
1
=> sinr1 =2
=> '1 = 300
Least values of t is corresponding to n =1 or Since, r1 + r2 =A
t. =-A- => r2 =A-r1 =30°-30°=0°
= 2x3.6
Therefore, ray of light falls normally on the face AC
648
=> t. =-nm and angle of emergence e = i2 = 0° .
= 7.2
(b) Multiple reflection occurs between surfaces of film:
Intensity will be maximum if interference takes place in
the transmitted wave.
CONCEPTUAL NOTE(S)
For maximum thickness, we have
a} For a wave (whether it is sou_nd or electromagnetic), a
medium is denser or rarer is decided from the speed of 1'x=2µt=A
wave in that medium. In denser medium speed of wave is where t is the thickness of coated film
less. For example, water is rarer for sound, while denser
t =__?:_ =~ =125 nm
for light compared to air because speed of sound in Water
is more than in air, while speed of light is less. =>
2µ 2x2.2
b) In transmission/refraction, no·phase change takes place. In
reflection, there is a change of phase of n when it is
reflected by a denser medium and phase change is zero if Problem 12
it is reflected by a rarer medium.
A vessel ABCD of 10 cm width has two small slits S1
c) If two waves in_ phase interfere having a path difference of
ax; then condition of maximum intensity would be and S2 sealed with identical glass plates of equal
Ax=nA., n=_0,1,2, ... tnickness. The distance between the slits is 0.8 mm. POQ
But if two waves, which are already out of phase (a phase is the line perpendicular to_ the plane AB and passing
difference of n ) interfere with path difference ax, thf;ln through O , the middle point of S1 and S2 • A
Condition of maximum intensity will be Ax·= (n-f}·, monochromatic light source is kept at S, 40 cm below: P
and 2 m from the vessel, to illuminate _the slits as shown in
n=1, 2, ....
~---------- --- - --------------- ------· the figure alongside.

==================================== = 2.47

www.puucho.com
Rahul Sardana with www.puucho.com

Advanced JEE Physics Optics & Modem Physics

(a) Calculate the position of the central bright fringe on 0 Llx, - Llx, = 0
the other wall CD with respect to the line OQ.
• • L • 0 Lll'.2 = Axl
(b) Now, a liquid is poured into the vessel and filled upto
0 dsin0 = 0.16
OQ • The central bright fringe is found to ~e at Q .
Calculate the refractive index of the liquid.
,,, (0.8)sin0 = 0.16

"1·
s,
D 0 sin8= 0.16 =l.
0.8 5
1
0 tan8= .fiA

40cm
t --------------------fo -------- a
S1 . 0 ::::-

0 sm
I.
s
2 m ----+t<-10 cm 5
. . y2 1
s~~--~c 0 tan8R:"sin8=- =-
D, 5
Solution
, 0 D, 10
(a) Given y, =40 cm, D, =2 m=200 cm, D 2 =10 cm y,=-=-=2cm
, . 5 5
Therefore, ·central bright fringe is observed at 2 cm
above point Q on side CD.

Alternate solution for (a)


AX at R Will be zero if Ax 1 = ruc: 2
\
dsiha. =dsin8
'---__JC a.=8 '
tana.::: tan9
40 i"
y,
tana=-=-=- Y..i =Jg_
D, 200 5 D, 02

ex=·tan -1(1)S Y, = (~: } , = (;;0 )<40) cm

Y2 =2 cm .

(b) The central bright fringe will be observed at point Q, if


the path difference created by the liquid slab of
thickness t = 10 cm or 100 mm is equal to &t-1 , so
·that the net path difference at Q becomes zero.

" 5
Path difference between SS, and SS, is
Llx, =ss, -ss,
o Lll:1 =dsina=(0.8 ~J(½)
o · Lll:1 = 0.16 mm ... (1) s
Now, let at point R on the. screen, central bright fringe o (µ-l)t=Llx,
is observed.(i.e., net path difference = 0 ).
o (µ-1)(100)=0.16
Path difference between S,R and._S,R is
o µ-.1 = 0.0016
Lll:2 = S,R-S,R .
o µ=1.0016
o Llx;·= d sin 8 ... (?-i
Central bright fringe will be observ"ed when net paih
difference is zero.

= t:::::================================
2.48

www.puucho.com
Rahul Sardana with www.puucho.com

Wave Optics
Problem 13
The Young's double slit experiment is done· in a => yd =(.&.-1J1
D µ.
medi.;m of refractive index ±
3
.A light of 600 nm
wavelength is falling on the slits having 0.45 mm => y=(~:-1}~
separation. The lower slit S2 is covered by a thin glass Substituting the values, we get
sheet of thickness 10.4 µm and refractive index 1.5. The
=(~-l 10.4x10.,(1.5)
interference pattern is observed on a screen placed 1.5 m
y 4/3 0.45 X 10"3
from the slits as shown in the figure.
y => y=4.33x10·'m

SI => y=4.33 mm

s() l-------------
s, D
o (b) At 0, ~x, =0 and 6x2 =(~: -1}

So, net path difference is


I 6x=6x,
(a) Find the location of central maximum (bright fringe
with zero path difference) on the y-axis. Corresponding phase difference, M = ( ~ )6x
(b) Find the light intensity of point O relative to the
Substituting the values, we get
maximum fringe intensity.
(c) Now, if 600 nm light is replaced by white light of 15
~=M=~( -1)(10.4x10--)
range 400 to 700 nm, find the wavelengths of the light 6 x10·7 4/3
that form maxima exactly at point O .
[All wavelengths in the problem are for the given medium => ~=(~)n
of refractive index _!. Ignore dispersion]
3 Now, I(~)=I-cos'(t)
Solution
Given A=600 nm=6x10·' m, => l=Imaxcos '(13n)
6
d=0.45 mm=0.45x10°" m and D=l.5 m
=> I=~I
SI1 4=

(c) At O ,path difference is 6x=6x2 =(~:-1}

s,--8----------
--- --- For maximum intensity at O , we have

1
M=nA.,where n=1,2,3, ..... .
6x 6x 6x
Thickness of glass sheet, I= 10.4 µm = 10.4 x 10-- m => A.=l, , , ...... and soon
2 3
Refractive index of the medium, µm =¾
=> 6x=(~-1)(10.4x10-'m)
4/3
And refractive index of glass sheet, µg =1.5
3
(a) Let central maximum is obtained at a distance y below => 6x =(~-1)(10.4xl0 nm)=1300 nm
4/3
point 0.
So, maximum intensity will be corresponding to
=> 6x1 =S1 P-S2 P= ~
A=l 300 nm, 1300 nm, 1300 nm, 1300 nm, ...
2 3 4
Path difference due to glass sheet is given by
=> ,- = 1300 nm , 650 mm, 433.33 nm, 325 nm, ....
6x,=(~:-1} The wavelength in the range 400 nm to 700 nm are
650 nm and 433.33 nm .
Net path difference will be zero, when we have
Lll'.1 = M2
2.49 r-1

www.puucho.com
Rahul Sardana with www.puucho.com

Advanced JEE Physics Optics & Modern Physics

Problem 14 Problem 15
In Young's experiment, the source is red light of A double slit apparatus is immersed in a liquid of
wavelength 7 x 10-' m. When a thin glass plate of refractive index 1.33. It has slit separation of 1 mm and
refractive index 1.5 at this wavelength is put in the path of distance between the plane of slits and screen is 1.33 m.
one of the interfering beams, the central bright fringe The slits are illuminated by a parallel beam of light whose
wavelength in air is 6300 A. .
shifts by 10" m. to the position previously occupied by the
51h bright fringe. Find the thickness of the plate. When the (i) Calculate the fringe width.
source is now changed to green light of wavelength (ii) One of the slits of the apparatus is covered by a thin
5 x 10-' m , the central fringe shifts to a position initially glass sheet of refractive index 1.53. Find the smallest
occupied by the 6th bright fringe due to red light. Find the thickness of the sheet to bring the adjacent minimum
refractive index of glass for green light. Also estimate the as the axis. •
change in fringe width due to the change in wavelength. Solution
Solution Given µ=1.33, d=lnun, D=l.33 m and
Path difference due to the glass slab, i,,=6300 A
t.x = (µ-1)1 = (1.5-l)t = 0.51 (i) Wavelength of light in the given liquid is
6300
Due to this slab, 5 red fringes have been shifted upwards. So, 1,,' = 2: = A "' 4737 A = 4737 x 10-10 m
we have µ 1.33
l!u=5Ared . 'dth 1,,'D
~ Frmgewi , P=d
~ 0.51=(5)(7x10-'m)
p (4737x10-10 m)(1.33 m)
~ I = thickness of glass slab = 7 x 10-' m ~
(lxlO-'m)
Let µ' be the refractive index for green light, then
~ Pc"6,3x10 4 m=0.63 mm
t.x'=(µ'-1)1
(ii) Let t be the thickness of the glass slab.
Now the shifting is of 6 fringes of red light. So, we have
t.x' = 61,,red
~ (µ'-1)1=61,,..,
-t--------+o
- ( '-l)= (6)(7x10-') 06
- µ 7x10-' ·
~ µ'=1.6
Since the shifting of 5 bright fringes was equal to 10" m
Path difference due to glass slab at centre O is given by
~ sp.., = 10°" m, where p is the Fringe width
153
t.x = ( µglas, -1)1 = ( -1)1
10" µliq•ld 1.33
~ Pre, =-- m=0.2x10" m
5
~ 't,.x = 0.151
Nowsince P=i,,o Now, for the intensity to be minimum at 0, this path
d
1,,'
~ poci,, difference should be equal to -
2
~ P,,..,. = ,,,,..,. 1,,'
• Pred A.red
~ b.x=-
2
A =A i,,,,..,. =(0.2x10")(5xl0-') 4737
~
P'green 1-'red }..red 7 X 10-7 ~ o.1st = A
2
~ P,.... =0.143x10"m ~ t = 15790 A = 1.579 µm
~ t.p = P""" -p.., =(0.143-0.2)x10" m
Problem 16
~ t,p =~5.71 X 10--' ID
A point source S emitting light of wavelength 600 nm
is placed at a very small height h ;;_bove a flat reflecting
surface AB (shown in figure). The intensity of the
reflected light is 36% of the incident intensity. Interference
= 2.50 ================================:!!
www.puucho.com
Rahul Sardana with www.puucho.com

Wave Optics
fringes are observed on a screen placed parallel to the p p

n~-
reflecting surface at a very large distance D from it.

: D
L~--
!;. . •S
,:-:-:!
•S

'' •S' •S'


'
l'
h
ts' Initial Final
Since the ray is reflected from the surface of a denser
A '
medium, so it suffers an additional path change of .?:_ or
(a) What is the shape of the interference fringes on the 2
screen? a phase change of it .

(b) Calculate the ratio of the minimum to the maximum For maximum at P, path difference equals nA .
intensities in the interference fringes formed near the If AB is shifted by x, then this will cause an additional
point P (shown in the figure).
(c) H the intensity at point P corresponds to a maximum, path difference of 2(x-~) (for object and its image
calculate the minimum distance through which the
taken as coherent sources). Since reflection takes place
reflecting surface AB should be shifted so that the
intensity at P again becomes maximum.
at surface of denser medium, so this will produce an ·
additional phase change of it or a path change of - .
,.
Solution 2
(a) Since there is symmetry about the line SP, so the shape So, we get
of the interference fringes will be circular.
(b) Intensity of light reaching on the screen directly from 2(x-1)=ni..
the source 11 =I, (say) and intensity of light reaching
2x-i..=ni..
on the screen after reflecting from the mirror is
=> 2x =(n + l)i..
I,= 36% of I,= 0.3610 •
=> x=(n+1)
,. where n=0, 1, 2, 3,....
!i_=_I_,-=-1- 2
12 0.3610 0.36
Now, to get minimum value of x, n must be minimum
1 i.e., n = 0
JF,=0 6
=> X=-
,.
2
(.2..-1)'
0.6 1 => X=
600
=300 nm
2
16
1
( -+1 )'
0.6 .
(c) Initially path difference at P between two waves
reaching from S and S' is as shown.

2.51 =
www.puucho.com
Rahul Sardana with www.puucho.com

This section conti.'ms Single Correct Choice Type Questions. Each question has four choices (A), (B), (C) and (D), out of which
ONLY ONE is c~rrect.

1. In ail interference pattern produced by two identical (B) is a geometrical method to find" the position of a
slits, the intensity at the site of the central maximum is wave-front at a later or an earlier instant
I . Th'7 intensity at the same spot when either of two (C) is used to'determine the velocity of light
slits is closed is (D) is used to explain polarization of light
I I
(A) 2 (B) 6. Longitudinal waves do not exhibit
4
I I (A) refraction (B). reflection
(C) (D) ,/2 (C) _diffraction · (D) polarization
2,/2
I
7. The idea of the quantum nature of light has emerged in
2. In YDSE bi-chromatic light of wavelengths 400 run and
an attempt to explain
560 run are used. The distance between the slits is
(A) interference
0.1 mm arid the distance between the plane of the slits
(B) diffraction
and the screen is 1 m . The minimum tjistance bet_ween (C) polarization _
two·successive regions of complete darkness is (D) radiation spectrum of a black body
(A) 4 mm (B) 5.6 mm
(C) 14 mm (D) 28 mm_ · 8. In the spectrum of light of a luminous heavenly body
the wavelength of a spectral line is measured to be
3. In the ideal double-slit experiment, when a glass-plate
(refractive index 1.5) of thickness I is introduced in the 4747 A while actual wavelength of th_e line is 4700 A.
The relative velocity of the heavenly body with respect
path of one of the interfering beams (wavelength 1'. ),
the intensity at the position where the central maximum to earth will be (velocity of light is 3x10 8 ms·')
ocqrrred previously remains unchanged. The minimum (A) 3 x 105 ms·1 mov;,.;g towards the earth
thickness of the glass-plate is (B) 3x10 5 ms·1 moving away from the earth
(B) 21'. (C) 3x10 6 ms·' moving towards the earth
(A) 21'.

(C)
,. (D) ,.
3 (D) . 3 x 10 6 ms·' moving away from the earth

3 9. A grating has 5000 lines cm·'. The maximum order


visible with·wavelength 6000 A
4. Young's double slit experiment is carried out by using
(A) 2 (B) 3
green, red and blue light, one color at a time. The fringe
(C) 4 (D) 0
widths recorded are Pa, P, and P, respectively. Then,
(A) Pa >P, >P, (B) P, >P, >P, 10. A beam of monochromatic light enters from vacuum
(C) P,>P,>Pa (D) P,>Pa>P, into a medium of refractive index n . The ratio of the
wavelengths of the incident and refracted waves is
5. Huygens' conception of seco11dary Waves (A) n:1 (B) 1:n
2
(A) helps us to find the focal length of a thick lens (C)· n :1 (D) 1:n'

2.52 c:===::::::::::::::::;:::::::::::::::::::::::::::::::::=:::::::=::::=======================::::i
www.puucho.com
Rahul Sardana with www.puucho.com

Wave Optics
11. In Young's double slit experiment, 62fringes are seen in (D) scattering is reduced at noon
visible region for sodium light of wavelength 5893 A. If
20. In Young's double slit experiment, carried out with light
violet light of wavelength 4358 A is used in place of
sodium light, then number of fringes seen will be of wavelength 1,. = 5000 A, the distance between the
(A) 54 (B) 64 slits is 0.2 mm and the screen is at 200 cm from the
(C) 74 (D) 84 slits. The central maximum is at x = 0. The third
maximum (taking the central maximum as zeroth
12. Monochromatic light of wavelength 5000 A maximum) will be at x equal to
illuminates a pair of slits 1 mm apart. The separation of
(A) 1.67 cm (B) 1.5 cm
bright fringes in the inteiference pattern formed on a (C) 0.5 cm (D) 5.0 cm
screen 2 m away is
21. Ray optics is valid when characteristic dimensions are
(A) 0.25 mm (B) 0.1 mm
(A) of the same order as the wavelength of light
(C) 0.01 mm (D) 1.0mm
(B) much smaller than the wavelength of light
(C) much larger than the wavelength of light
13. Air has refractive index 1.0003. The thickness of an air
(D) of the order of 1 mm
column, which will have one -more wavelength of
yellow light ( 6000 A) than in the same thickness of 22. In order that a thin film of oil floating on the surface of
vacuum is water shows colours due to interference, the thickness
(A) 2mm (B) 2 cm of the oil film should be of the order of
(C) 2m (D) 2km (A) 1 cm (B) 10 A
(C) 5000 A (D) 10000 A
14. Two coherent monochromatic light beams of intensities
I and 41 are superposed. The maximum and 23. The blue cross on a white background illuminated with
minimum possible intensities in the resulting beam are white light is observed through a red filter. The pattern
(A) 41 and I (B) SI and 31 seen is
(C) 91 and I (D) 91 and 3I (A) a red cross on a black background
(B) a blue cross on a red background
15. There is a wavelength corresponding to each colour. (C) a red cross on a blue background
How many.colours are possible, then (D) a black cross on a blue background
(A) 3 (B) 1
(C) 7 (D) None of these 24. The deflection of light in a gravitational field was
predicted first by
16. Though quantum theory of light can explain a number (A) Einstein (B) Newton
of phenomena observed with light, it is necessary to (C) Max Planck (D) Maxwell
retain the wave nature of light to explain the
phenomenon of 25. Both the particle and wave aspects of the wave aspects
(A) photo-electric effect of light appear to be used in
(B) diffraction (A) photoelectric effect
(C) compton effect (B) gamma emission
(D) black body radiation (C) interference
(D) classical mechanics
17. Which of the following cannot be polarized?
(A) Radio wave (B) X-rays 26. At sunset, the sun seems to be
(C) Infrared radiation (D) Sound waves in air (A) higher than it really is
(B) lower than it really is
18. A blue object on a white background when seen (C) exactly where it really is
through a blue filter will appear (D) lower than it would be at sunrise
(A) blue on a white background
(B) black on a blue background 27. In Huygens' wave theory, the locus of all the points in
(C) blue on red background the same state of vibration .is called a
(D) invisible (A) half period zone (B) vibrator
(C) wavefront (D) ray
19. illumination of the sun at noon is maximum because
(A) the sun is nearer to the earth at noon 28. In Young's experiment, monochromatic light is used to
(B) rays are incident almost normally illuminate the two slits A and B. Interference fringes
(C) refraction of light is minimum at noon are observed on a screen placed in front of the slits.
2.53

www.puucho.com
Rahul Sardana with www.puucho.com

Advanced JEE Physics Optics & Modern Physics

Now if a thin glass plate is placed normally in the path 33. Two coherent point sources s1 and s2 vibrating in
of the beam coming from the slit phase emit light of wavelength ). . The separation
between the sources is 2A . The smallest distance from
s2 on a line passing through s2 and perpendicular to
s1s2 , where a minimum of intensity occurs is

(A)
n
12
(BJ 151.
4
3).
-(CJ ). (DJ
,2 4

(A) The fringes will disappear 34. In a Young's double slit experiment, the fringes are
(BJ The fringe width will increase displaced by a distance x when: a glass plate of
(CJ The fringe width will increase refractive index 1.5 is introduced in· the path of one of
(D) There will be no change in the fringe width but· the beaII1S. When this plate is replaced by another plate
the pattern shifts ·
of same thickness, the shift of fringes is (¾}, The
29. When an unpolarized light of intensity ! 0 is incident on refractive index of second plate is
a polarizing sheet, the intensity of the light which does (A) 1.75 (BJ 1.50
not get transmitted is • (CJ 1.25 (D) 1.00
(A) ZERO (B) ! 0
35. Consider a usual set-up of Young's double slit
(CJ .!1
2 0 (DJ ¼I, experiment with slits of equal intensity as shown in the
figure. Take O as origin and the Y axis as indicated. If
. . . .b 1.D d 1.D
30. In the diagram, CP represent a wavefront and AO and verage mtens1ty etween y1 =- - an y 2 = + -
BP, the corresponding two rays. Find the condition on 4d 4d
equals n times the intensity of maxima, then n equals
0 for constructive interference at P between the ray
(take average over phase difference)
BP and reflected ray OP
0 R s, y

f
d ---------------- 0
t
s,
D

A
B
(A) ½(1+;) (B) 2(1+;)
(A) cos8=-
2d
3).
(BJ cos8=-
4d
).
(CJ (1+;) (DJ ½(1-;)
). 41.
(CJ sec8-cos8=a (D) sec9-cos8=- 36. A plate of thickness ! made of a material of refractive
d
index µ is placed in front of one of the slits in a double
31. In Young's double-slit experiment the separation slit experiment. What should be the minimum thickness
between the slits is doubled and the distance between ! which will make the intensity at the centre of the
the slit _and the screen is halved. The fringe-width fringe pattern zero?
becomes
(A) one-fourth (B) half
(A) (µ-1)~ (B) (µ-1)1.
(CJ double (DJ quadruple ).
(CJ (DJ (µ-1)
2(µ-1)
32. In Young's double slit experiment, the separation
between the slits is halved and the distance between the 37. Two polaroids are placed in the path of unpolarized
slits arid the screen is doubled. The fringe width is beam of intensity ! 0 such that no light is emitted from
(A) unchanged (B) halved the second polaroid. If a third polaroid whose
(CJ doubled (D) quadrupled polarization axis makes an angle 0 with the
polarization axis of first polaroid, is placed between

= ==================================
2.54

www.puucho.com
Rahul Sardana with www.puucho.com

Wave Optics
these polaroids then the intensity of light emerging (A) tum gradually red
from the last polaroid will be (B) tum suddenly red
(C) remains·same
(A) (~}in' 20 (B) (~}in' 20 (D) tum gradually blue

(C) (; )cos' e (D) I, cos' e 45. The transverse nature of light is shown by
(A) interference of light
(B) refraction oflight
38. In Young's double slit experiment intensity at a point is (C) polarization oflight
(¾) of the maximum intensity. Angular position of this (D) dispersion oflight

point is 46. Laser light is considered to be coherent, because it


1 1 consists of
(A) sin- (~) (B) sin- (:a) (A) many wavelengths
(B) uncoordinated wavelengths
(C) sin-'G'a) (D) . -1(-1,.)
sm (C) coordinated waves of exactly the same
4d
wavelength
(D) divergentbeams
39. In the Young's double slit experiment using a
monochromatic light of wavelength 1,. , the path
47. The wavelength. of light observed on the earth, from a
difference (in terms of an integer n ) corresponding to moving star is found to decrease by 0.05%. Relative to
any point having half the peak intensity is the earth the star is
(A) (2n + 1)?: (B) (2n + 1)?: (A) Moving away with a velocity of 1.5x105 ms-1
2 4
(B) Coming closer with a velocity of 1.5 x 10 5 ms-1
(C) (2n +1).1: (D) (2n + 1)_!':_ (C) Moving away with a velocity of 1.5 x 10 4 ms-1
8 16
(D) Coming closer with a velocity of 1.5 x 10 4 ms-1
40. If two slightly different wavelengths are present in the
light used in Young's double-slit experiment, then
48. A beam of electron is used in an YDSE experiment. The
(A) the sharpness of fringes will be more than the case slit width is d . When the velocity of electron is
when only one wavelength is present increased, then
(B) the sharpness of fringes will decrease as we move (A) No interference is observed
away from the central fringe (B) Fringe width increases
(C) the central fringe will be white (C) Fringe width decreases
(D) the central fringe will be dark . (D) Fringe width remains same

41. Two identical coherent sources placed on a diameter of 49. The ratio of the intensity at the centre of a bright fringe
to the intensity at a point one-quarter of the distance
a circle of radius R at separation x( « R)
between two fringe from the centre is
symmetrically about the centre of the circle. The sources
1
emit identical wavelength A each. The number of (A) 2 (B) 2
points on the circle with maximum intensity is (x =SA)
(C) 4 (Q) 16
(A) 20 (B) 22
(C) 24 (D) 26
50. The ratio of intensities of consecutive maxima in the
diffraction pattern due to a single slit is
42. Laser is
(A) 1 : 4: 9 (B) 1 : 2: 3
(A) intense, coherent and monochromatic
(B) only intense and coherent 1-~--4_ 1 9
(C) . 91t2 . 251t2 (D) 1 : ' : '
(C) only coherent and monochromatic n n
(D) only intense and monochromatic
43. Imagine a hypothetical convex lens material which can 51. A beam of natural light falls on a system of 6 polaroids,
transmit all the following radiation. This lens will have which are arranged in succession such that each
minimum focal length for polaroid is turned through 30° with respect to the
(A) ultraviolet rays (B) infrared rays preceding one. The percentage of incident intensity
(C) radio waves (D) X-rays that passes through the system will be
(A) 100% (B) 50%
44. A star emitting yellow light starts accelerating towards (C) 30% (D) 12%
earth, its colour as seen from the earth will

================================== 2.55 =
www.puucho.com
Rahul Sardana with www.puucho.com

Advanced JEE Physics Optics & Modem Physics

52. The maximum number of possible interference maxima intensity of light at the mid-point of the screen in the
for slit-separation equal to twice the wavelength in first case to that in the second case is
Young's double-slit experiment is · (A) 1:2 (B) 2:1
(A) Infinite (B) Five , (C) 4 : 1 (D) 1 : 1
(C) Three (D) Zero
58. In the ideal' double-slit experiment, when a glass-plate
53. A rocket is going towards moon with a speed v , The (refractive index 1.5) of thickness t is introduced in the
astronaut in the rocket sends signals of frequency v path of one of the interfering beams (wavelength 1,, ),
towards the moon and receives them back. on reflection the intensity ai the position where the central maximum
from the moon. What will be the frequency of the signal ,occurred previously remains unchanged. The minimum
received by the astronaut (Take v « c) thickness of the glass-plate is
C C 21,,
(A) --v (B) - - v (A) (B)
c-v c-2v 3.
2v 2c 1,,
(C) -v (D) ~v (C) (D) 1,,
C V 3

54. In Young's double slit experiment the.y-coordihates of 59. In a Young's double-slit experiment, the intensity ratio
central maxima and lQth maxima aie 2 · cm and 5 cm of maxima and minima is infinite. The ratio of the
respectively. When the YDSE,apparatus is immersed in amplitudes of two sources
a liquid of refractive index ,1.5 the corresponding y- (A) is infinity (B) is unity
coordihates will be · · · '· (C) is two , (D) cannot be predicted
(A), 2 cm, 7.5 cm (B) 3 cm, 6 cm
4 10 60. Figure represents a glass plate placed vertically on a
(C) 2cm,4cm. (D) cm, cm
3 3 horizontal table with a beam of unpolarised light falling
-. on its surface at: the polarising angle of 57° with the
,55. Jn·Yoting1s double slit exp°eriment how many maxim.as normal. The electric vector in the reflected light on
can be obtained on a screen (including the central r-ill
, Screen .. S . vib:ate Vfith respect. t~ the plan:e .of
maximum) on, both • sides of" the ·central fringe ·if incidenc~. in- a.
1..=ioooA',,;,d d=7oooA -
(A) 12 (B) 7
(C) 18 (D) 4

56. A monochromatic beam ,of· light falls on YDSE


apparatus at some angle (say 8) as shown in figure. A
thin .sheet of glass is inserted in front ofthe. lower slit
S2 , The central bright fringe (path _difference = O_) will
be obtained

(A) Vertical plane


(B) Horizontal plane ,
(C) Plane makihg an angle of 45° with the vertical
(D) Plane making an angle of 57° with the horizontal

(A) At 0 61. A clear sheet of pol!'}'oid is placed on the top of similar


(B) Above 0
(C) Below 0
sheet so that their axes make,.an ru:ig~e ~~~
1
(¾) with
(D) Anywhere depending on angle 8, thickness of each other. The ratio of intensity of the emergent light
plat_e 't and refractive index of glass_ µ ' · to that of unpolarised incident light is
·(A) 16:25 (B)· 9:25
57. In Young's double slit experiment, the two slits act as (C) 4:5 (D) 8:25
coherent sources 'of equal amplitude A and
wavelength A . In another experiment with the same set 62. Optically active substances are those substances which
up the two slits are of equal amplitude A .and (A) produce polarized light
wavelength i,.. but are
incoherent. The ratio of the (B)
(C)
rotate the plane of polarization of polarized light
produce double refraction ·

2.ss c===================================
www.puucho.com
Rahul Sardana with www.puucho.com

Wave Optics
(D) convert a plane · polarized light into circularly C
(C) (D) 2c
polarized light 5 5

63. If I, is the intensity of the principal maximum in the 72. A star is moving towards the earth with a speed of
single slit diffraction pattern, then what will be its 4.5 x 106 ms-' . If the true wavelength of a certain line in
intensity when the slit width is doubled
the spectrum received from the star is 5890 A, its
(A) . I, (B) ~ apparent wavelength will be about (c = 3 x 10 8 ms-')
(A) 5890 A (BJ 5978 A
(C) 2I, (D) 4I,
(CJ 5802 A (DJ 5896 A

64. The critical angle of a certain medium is sin-1 (¾). The 73. Lights of wavelength 1,,1 = 4500 A, 1,,2 = 6000 A are sent
through a double-slit arrangement simultaneously.
polarising angle of the medium is Then
(A) sin-'(¼) (B) tan-'(¾) (A) no interference pattern will be formed
(B) the third bright fringe of A. 1 will coincide with the
fourth bright fringe of 1,, 2
(C) tan''(¾) (D) tan-'(½)
the third bright fringe of 1,, 2 will coincide with
fourth bright fringe of 1,,1
65. The ray of light is incident on glass of refractive index
1.5 at polarising angle. The angle of deviation of the (D) the fringes of wavelength 1,, 1 will be wider than
incident ray in glass is the fringes of wavelength 1,,2
(A) 57° (B) 33°
(C) 24° (D) 114° 74. Two slits separated by a distance of 1 mm are
illuminated with light of wavelength 6 x 10-' m. The
66. Double refraction of light is shown by interference fringes are observed on a screen placed 1 m
(A) quartz and calcite only from the slits. The distance between the second dark
(B) calcite only fringe and the fourth bright fringe is equal to
(C) calcite and ice only (A) 0.5 mm (B) 1.0 mm
(D) calcite, ice and quartz (C) 1.5 mm (D) 2.0 mm
67. A slit of width a is illuminated by red light of 75. Interference fringes were produced in Young's double-
wavelength 6500 A . The first minimum will fall at slit experiment using light of wavelength 5000 A .
9=30° if a is When a film of thickness 2.5 x 10" cm was placed in
(A) 3250 A (B) 6.5 X 10-4 mm front of one of the slits, the fringe pattern shifted by a
(C) 1.3 µm (D) 2.6 x 10-4 cm distance equal to 20 fringe-width. The. refractive index
of the material of the film is
68. The resolution of the human eye is 1'. The resolving (A) 1.25 (B) 1.35
power of the human eye is nearly (C) 1.4 (D) 1.5
(A) 360 (B) 3600
(C) 36000 (D) 360000 76. In Young's double-slit experiment, an interference
pattern is obtained on a screen by a light of wavelength
69. Colours of thin films are due to 6000 A, coming from the coherent sources S1 and S2 •
(A) dispersion of light (B) interference of light At certain point P on the screen third dark fringe is
(C) absorption oflight (D) scattering of light formed. Then the path difference S1 P - S2P in microns
is
70. A person standing at a distance of 3.6 km can just (A) 0.75 (B) 1.5
resolve two poles. The distance behveen the poles is (C) 3.0 (D) 4.5
(A) 0.1 m (B) 100 m ·
(C) 1m (D) 10m 77. Young's double slit experiment is made in a liquid. The
10th bright fringe in liquid lies where 6th dark fringe
71. A heavenly body is receding from earth such that the
lies in vacuum. The refractive index of the liquid is
fractional change in A. is 1, then its velocity is approximately
(A) C (B) 3c (A) 1.8 (B) 1.54
5 (C) 1.67 (D) 1.2

================================::::- = 2.57

www.puucho.com
Rahul Sardana with www.puucho.com

Advanced JEE Physics Optics & Modern Physics

78. A point source emits light equally in all directions. Two observation be taken from :point P , such . that
points P and Q are at distances 9 m and 25 m
PB - PA = !:. . Then the phase difference between the .
respectively from the source.. The ratio of the 4
amplitudes of the waves P and Q is waves from A and- B reaching P is
(Al 9:25 (Bl 25:9 (Al 156° (B) 140°
(C). ' 9' : 252 (Dl 252 : 9' (C) 136° (Dl 126°

79. In Young's double slit experiment the y-co-ordinates of 86. Two coherent sources S1 and S2 are separated by a
central maxima and 10th maxima are 2 cm and 5 cm
distance four.times the wavelength A of the source.'The
respectively. When the YDSE apparatus is inunersed in sources lie along y-axis whereas a detector moves along
a liquid of refractive index 1.5 the corresponding +x-axis. Leaving the origin and far off points the
y-co-ordinates will be ·
number Of points where ni.axima are observed is
(A) 2 cm, 7.5 cm (Bl 3 cm, 6 cr_n
(Al 2 (B) 3
4 10 (C) 4 (D) 5
(C) 2cm,4cm (Dl cm, ~cm
3
87. The first muumum due to a Fraunhofer diffraction
80. In Young's double ~slit experiment ~ = 10"" (d = using light 'of ·wavelength 500 nm and a slit of )Vidth
0.5 mm will be formed at an angle (in minutes)
distance between slits, D·= distance of,screen from the (Al 2.42 (B) 3.43
slits). At a point P on the screen resulting intensity is (C) 4.84 (Dl 1.71-
equal to the intensity due to individual ~lit I0 • Then the
distance. of point P from the central maximum is 88. Aperture of the human eye is 2 min. Assuming the

(A =6000f) mean wavelength of light to be 5000 A , the ..,;gular


(A) 2mm (Bl 1 mm resolution limit of the eye is nearly
(C)_ .0.5mm (Dl 4mm. (A) 2 minute (Bl 1 min'!}te .
(C) 0.5 minute (D) 1.5 ~ute
81. Th~·yci~g's double-slit experiment is carried out with
89. In the Young's double slit experiment apparatus shown
light of wavelength 5000 A . The distance b~~een the
in figure, the ratio of maximum to minimum intensity
slits is 0.2 mm and the screen is at 200 cm from the slits.
on the screen is 9. The wavelength of light used is A,
The central maximum is at x = 0 . The third maximum
then the v~lue of y is
will be at .. x equal to
Screen
(Al 1.67cm (Bl 1.5 cm

---
(C) 0.5 cm (Dl 5.0 cm
d/2 y
82. in a Young's . double slit experiment,' 12 fringes are
observed to be formed in a certain segment of the screen d/2
when light of wavelength· 600·nm is used. If the
wavelength of light is changed to 400 nm , number of
AD
ho -<-1+--D
fringes obseived in the same segment of the screen is (B) AD
(A)
given by d 2d
(A) 12 (Bl 18 AD (D) ).D
(C)' 24 (D) 30 . (C)
3d 4d
83. The blue colour of the sky is explained by 90. In Young's experiment, using red and blue lights of
(Al refraction (B) reflection
wavelengths 7800 A and 5200 A respectively, the
(C) polarisation (D) scatteriog
value of n for which nth red fringe coincides, with
84. A Young's double slit experiment uses a (n+l)th bluefringeis
monochromatic source. The shape of the interference (A) 2 (B) 3
fringes formed on a screen is_ (C) 4 (D) 5
(A) Straight line (B) Parabola
(C) Hyperbola (D) Circle 91. In Young's experiment for interference of light with two
slits, maxima Occur at angles £Or which sin 8 = m),_ .• Here
85. Among the two interferiog monochromatic sources A d
and B ; A is ahead of B in phase by 66° . If the d is

= ==================================
2.58

www.puucho.com
Rahul Sardana with www.puucho.com

Wave Optics
(A) distance of slits from the screen polaroid is given one complete rotation about the
(B) distance.between dark and bright fringes direction of light
(C) distance between slits (A) the intensity of light gradually decreases to zero
(D) width of mth fringe and remains at zero
(B) the intensity of light gradually increases to a
92. Interference·is observed in a chamber with air present maximum and remains maximum
inside the chamber. The chamber is then evacuated and (C) there is no change in the intensity of light ·
the same light is again used to produce interference. A (D) the intensity of light varies such that it is twice
careful observer will see maximum and twice zero
(A) no change in the pattern
(B) that the fringewidth slightly increases 98. Figure here shows P and Q as two equally intense
(C) that the fringewidth slightly decreases coherent sources emitting radiations of wavelength
(D) no interference pattern 20 m . The separation PQ is 5 m and phase of P is
ahead of the phase of Q by 90° . A , B and C are
93. Finger prints of a piece of paper may be detected by
three distant points of observation equidistant from the
sprinkling fluorescent powder on the paper and than
looking into it under mid-point of PQ . The intensity of radiations at A, B,
(A) yellow light (B) brightness C will bear the ratio
(C) infrared light (D) ultraviolet light •B
''
94. Two nicol prisms (polariser and analyser) have their
''
axes at angles of 30° in between. If I is the intensity of
''
''
• light falling on first nicol, then that of emerging light is ''
{A) 0.125! (B) 0.25! p : Q
(C) 0.3751 (D) 0.5[
•-----------<-
C
I ()-----------•
A
(A) 0 :1 :4 (B) 4 : 1 : 0
95. The Young's double-slit experiment is performed with (C) 0:1 :2 (D) 2 : 1 : 0
blue light and green light of wavelengths 4360 A and
99. In the figure is shown Young's double slit experiment.
5460 A respectively. If X is the distance of 4th
Q is the position of the first bright fringe on the right
maximum from the central one, then
(A) X(Blue)=X(Green) side of O . P is the 11th fringe on the other side, as
measured from Q . If the wavelength of the light used
(B) _ X(Blue)<X(Green)
(C) X(Blue)>(Green) is 6000xl0-10 m, then 51B will be equal to
(D) X(Blue) 5460 Q
X( Green) 4360
s,
D____ B
96. A ray of light of intensity I is incident on a parallel ---------. 0
glass,slab at a point A as shown in figure. It undergoes s1110
partial reflection and refraction. At each reflection 25% s,D
of incident energy is reflected. The rays AB and A'B' p
undergo interference. The ratio Imax / Imm is -- (A) 6x10_. m (B) 6.6x10_. m
I ~
(C) 3.138 X 10-7 m (D) 3.144xl0-7 m

100. The maximum intensity in Young's double slit experiment


is ! 0 • Distance between the slits is d = 51,., where 1,. is the
wavelength of monochromatic light used in the
experiment. What will be the intensity of light in front of
one of the slits on a screen at a distance D = 10d ·
(A)
(C)
4: 1
7: 1
(B) 8:1
(D) 49:1
(A) ~ (B) ¾r,
(C) I, (D) I,
97. Plane polarised light is passed through a polaroid. On 4
viewing through the polaroid we find that when the

2.59.=

www.puucho.com
Rahul Sardana with www.puucho.com

Advanced JEE Physics Optics & Modern Physics

101. The polarising angle of diamond is 67° . The critical


(D) ,/3)..D
angle of diamond is nearest to
(A) 22' (B) 34°
(C) 45° (D) 60° 106. Two waves originating from sources S1 and 5 2 having
zero phase difference and common wavelength A will
102. No longitudinal wave will show
show completely destructive interference at a point P
(A) interference (B) diffraction
if S1P-S2P is
(C) TI R (D) polarisation
3)..
(A) 5).. (B)
103. A beam of light AO is incident on a glass slab 4
(µ =1.54) in a direction as shown in figure. The 2)..
11)..
(C) (D)
reflected ray OB is passed through a Nicol prism on 2
viewing through a Nicol prism, we find on rotating the
107. A thin air film between a plane glass plate and a convex

A~£5B
prism that
· lens is irradiated with parallel beam of monochromatic
light and is observed under a microscope. We see
(A) uniform brightness
(B) complete darkness
I --- -.~- 0 ... ····· . I (C) field crossed over by concentric bright and dark
rings
(D) field crossed over by straight bright and dark
(A) The intensity is reduced down to zero and
fringes
remains zero
(B) The intensity reduces down some what and rises
108. In Young's double-slit experiment, the intensity of light
again
at a point on the screen where the path difference is A.
(C) There is no change in intensity
is I, ).. being the wavelength of light used. The
(D) The intensity gradually reduces to zero and then
again increases intensity at a point where the. path difference is ~ will
. .
104. When one of the slits of Young's experiment is covered be
with a transparent sheet of thickness 4.8 mm, the I I
(A) (B)
central fringe shifts to a position originally occupied by 4 2
the 30th bright fringe. What should be the thickness of (C) I (D) ZERO
the sheet if the central fringe has to shift to the position
occupied by 20th bright fringe 109. In a two slit experiment with monochromatic light
(A) 3.8 mm (B) 1.6 mm fringes are obtained on a screen placed at some distance
(C) 7.6 mm (D) 3.2 mm from the sits. If the screen is moved by 5 x 10-2 m
towards the slits, the change in fringe width is
105. Two ideal slits S1 and S2 are at a distance d apart, and 3 x 10-s m . If separation between the slits is 10-3 m , the
illuminated by light of wavelength ).. passing through wavelength of light used is
an ideal source slit S placed on the line through S2 as (A) 6000 A (B) 5000 A
shown. The distance between the planes of slits and the (CJ 3000 A (D) 4500 A
source slit is D . A screen is held at a distance D from
the plane of the slits. The minimum value of d for 110. In Young's experiment using monochromatic light, the
which there is darkness at O is fringe pattern shifts by a certain distance on the screen
when a mica sheet of refractive index 1.6 and thickness

~
1.964 micron is introduced in the path of one of the
interfering waves. The mica sheet is then removed and
s,~ the distance between the slits and the screen is doubled.
0 It is found that the distance between successive maxima
now is the same as the observed fringe shift upon the
s ~- s,~ introduction of the mica sheet. The wavelength of light
is
1+- D - - - D - + 1 (A) 5762 A (B) 5825 A

(A) l~D (B) ..fijj


(CJ 5892 A· (D) 6500 A

= ====================================
2.60

www.puucho.com
Rahul Sardana with www.puucho.com

Wave Optics
111. The phenomenon of interference is shown by
(A) longitudinal mechanical waves only (A) I=JA'+B'cos 2 ~ (B) r,;;;cos~
(B) transverse mechanical waves only
(C) non-mechanical transverse waves only (C) I=A+Bcosi (D) l=A+Bcos~
2
(D) all the above types of waves
Where A and B depend upon the amplitudes of the
,two waves.
112. In Young's double-slit experiment, if L is the distance
between the slits and the screen upon which the
118. The time period of rotation of the sun is 25 days and its
interference pattern is ob~erved, x is the average
distance between the adjacent fringes and d is the slit radius is 7 x 108 m . The Doppler shift for the light of
separation, then the wavelength of light is .wavelength 6000 A emitted from the surface df the sun
xd xL will be
(A) L (B) d (A) 0.04 A (B) 0.40 A
(C) 4.ooA (DJ 40.oA
(C) Ld (D) 1
x Ldx
119. A flake of glass (refractive index 1.5) is placed over one
113. Interference can take place between . of the openings of a double slit apparatus. The
' interference pattern displaces itself through seven
(A) .transverse waves only,. ]:mt not in longitudinal
waves successive maxima towards the side Where the flake is
(B) lon~tudinal waves only, but not in transverse placed, if wavelength of the diffracted light is
1,, = 600 nm , then the thickness of the flake is
waves
(C) both longi~dinal and transverse waves (A) 2100 nm (B) 4200 nm
(D) light waves only, but not sound waves (C) 8400 nm (D) None of these

114. Young's double-slit experiment is performed with light 120. Two coherent sources separated by distance d are
radiating in phase having wavelength 1,, • A detector
of wavelength ·,_ = 6000 A . A glass plate of thickness
moves in a big circle around the two sources in the
0.01 mm and µ =1.5 is introduced. The number of
plane of the two sources. The angular position of n :a 4
fringes shifting in the system is
interference maxima is given as
(A) 2000 (B) 8
(C) 120 (D) 4910

115. The contrast in the fringes in an interference pattern


depends on
(A) fringe width
(B) wavelength
(C) intensity ratio of the sources
(D) · distance between the slits {A) . _,·(n")
sm - (B) cos·1 ( 4,;')
d
116. In a Young's double slit experiment the source S and
the two slits A and B are vertical with slit A above
(C) tan·'(4d,,) (D) cos·'( :d)
slit B . The fringes are observed on a vertical screen K .
The optical p~th length from S to B is increased very 121. White light may be considered to be a mixture of waves
slightly (by introducing a transparent material of higher with 1.. · ranging between 3900 A and 7800 A . An oil
refractive index) and the optical path l~gth from s to film of thickness 10,000 A is examin~d normally by
A is not changed, as a result. the fringe system on K reflected light. If µ =1.4, then the film appears bright
moves for
{A) Vertically downwards slightly (A) 4308 A, 5091 A, 6222 A
(B) Vertically upwards slightly (B) 4000 A, 5091 A, 5600 A
(C) Horizontally, slightly to the left
:~~~ 1: :: 1: ;~~~ 1_
(C)
(D) Horizontally, slightly to the right
~, . . . . ' (D) 7000 A

·117_ In the Young's double slit experiment, if the phase 122.._The k line of sing!:¥, io,;;sed calcium has a wavelength
difference between the two waves interfering at a point of 393.3 nm as measured on earth. In the spectrum of
is ~, the intensity at that point can be expressed by the one of'the observed galaxies,. this spectral line is located
expression·· at 401.8 nm. The speed with which the galaxy is
nioving away from us, will be

c=================================== 2.s1 =
www.puucho.com
Rahul Sardana with www.puucho.com

Advanced JEE Physics Optics & Modern Physics

(A) 6480 kms-1 (B) 3240 kms-1 (A) 90°+$ (B) sin-1 (µcos$)
(C) 4240 kms-1 (D) None of these
(C) 90° (D) 90°-sin-'( s:$)
123. In a YDSE bi-chromatic light of wavelengths 400 run
and 560 run are used. The distance between the slits is
130. A beam of plane polarized light falls normally on a
0.1 mm and the distance between the plane of the slits
polarizer of cross sectional area 3 x 10-4 rn2 . Flux of
and the screen is 1 m . The minimum distance behveen
two successive regions of complete darkness is energy of incident ray in 10-3 W. The polarizer rotates
(A) 4 mm (B) 5.6 mm with an angular frequency of 31.4 rads-1 • The energy of
(C) 14 mm (D) 28 mm light passing through the polarizer per revolution will
be
124. In a double slit arrangement fringes are produced using (A) 10-< Joule (B) 10-3 Joule
2
light of wavelength 4800 A . One slit is covered by a (C) 10- Joule (D) 10-1 Joule
thin plate of glass of refractive index 1.4 and the other
with another glass plate of same thickness but of 131. A beam with wavelength 1. falls on a stack of partially
refractive index 1.7. By doing so the central bright shifts reflecting planes with separation d. The angle 9 that
to original fifth bright fringe from centre. Thickness of the beam should make with the planes so that the
glass plate is beams reflected from successive planes may interfere
(A) 8 µm (B) 6 µm constructively is (where n =1, 2, ...... )
(C) 4 µm (D) 10 µm

125. Polarisation of light establishes


(A) corpuscular theory of light
(B) quantum nature oflight
(C) transverse nature of light
(D) all the three

126. A ray of unpolarised light is incident on glass plate at


the polarising angle, then
(A) . -1(n:>.)
Sill -
d
(A) the reflected and transmitted rays will be
completely plane polarised (C) • -I -
sm (HA) (D) cos -,(n:>.)
-
2d 2d.
(B) the reflected ray is completely polarised and the
transmitted ray is partially polarised
(C) the reflected ray is partially polarised and the 132. A diffraction pattern is obtained using a beam of re'd
transmitted ray is completely polarised light. If the red light is replaced by blue light, then
(D) the reflected ray and the transmitted ray will be (A) the diffraction pattern remains unchanged
partially polarised (B) diffraction bands become narrower and crowded
together
127. To observe diffraction, the size of the obstacle (C) bands become broader and farther apart
(A) should be of the same order as the wavelength (D) bands disappear
(B) should be much larger than the wavelength
(C) has no relation to wavelength 133. At sunrise or at sunset the sun appears to be reddish
(D) should be exactly half the wavelength while at mid-day the sun looks white. The reason is that
(A) the sun is less hot at sunrise or at sunset than at
noon
128. A slit is illuminated by red light of wavelength 6500 A .
(B) diffraction sends red rays to the earth at these
The first minimum is obtained at 9 = 30°. The width of time
the slit is
(C) refraction is responsible for this effect
(A) 3200 A (B) 1.24 micron (D) scattering due to dust particles and air molecules
(C) 6.5x10-< mm (D) 2.6x10-< cm·
134. Light of wavelength 6328 A is incident normally on a
129. A ray of light is incident on the surface of a glass plate
slit having a width of 0.2 mm . The distance of the
at an angle of incidence equal to Brewster's angle $. If µ
screen from the slit is 0.9 m. The angular width of the
represents the refractive index of glass with respect to
central maximum is
air, then the angle between reflected and refracted rays
(A) 0.09 degree (B) 0.72 degree
is
(C) 0.18 degree (D) 0.36 degree

= 2.62 t:::===============================:::i
www.puucho.com
Rahul Sardana with www.puucho.com '

Wave Optics
135. Two point sources X and Y emit waves of same 142. When light is incident on a transparent surface at the
frequency and speed but Y lags in phase behind X by polarizing angle, which of the following is completely
2nl radian. If there is a maximum in direction D the polarized?
distance XO using ri as an integer is given by (A) Reflected light
(B) Refracted light
(C) Both reflected as well as refracted light
X~D (D) Neither reflected nor refracted light

\~
y
143. An optically active substance
(A) .produces polarized light
A. (B) rotates the plane of polarization of polarized light
(A) -(n-1) (B) ,.(n+I)
2 (C) converts a plane polarized light into circularly
polarized light
!:en+ l) (D) )..(n-0 (D) converts a circularly polarized light into plane
2
polarized light
136. In a Young's double slit experiment, .the slits are 2 mm
144. Diffraction pattern of a single slit consists of a central
apart and are illuminated wii:h a mixture of two
bright band which is
wavelength A.0 = 750 nm and A. = 900 nm . The
(A) wide, and is flanked by alternate dark and bright
minimum distance from the common central bright bands of decreasing intensity
fringe on a screen 2m from the slits where a bright (B) narrow, and is flanked by alternate dark and
fringe from one interference pattern coincides with a bright bands of equal intensity
bright fringe from the other is (C) wide, and is flanked by alternate dark and bright
(A) 1.5 mm (B) 3 mm bands of equal intensity
(C) 4.5 mm (D) 6mm (D) narrow, and is flanked by alternate dark and
bright bands of decreasing intensity
137. If sound waves can be assumed fo be diffracted, which
of the following objects will diffract sound waves in air 145. In Young's experiment with one source and two slits,
from a 384 Hz tuning fork one of the slits is covered with black paper. Then
(A) A sphere of radius 1 cm (A) the fringes will be darker
(B) A sphere of radius 1 mm (B) the fringes will be narrower
(C) A sphere of radius 1 m (C) the fringes will be broader
(D) A sphere of radius 10 m (D) no fringes will be obtained and the screen will
have uniform illumination
138. A beam of light of wavelength 600 nm from a distant
source falls on a single slit 1 mm wide and the resulting 146. The distance between two coherent sources is 0.1 mm.
diffraction pattern is observed on a screen 2 m away. The fringewidth on a screen 1.2 m away from the
The distance between the first dark fringes on either sources is 6.0 mm. The wavelength of light .used is
side of the central bright fringe is
(A) 4000 A (B) 5000 A
(A) 1.2 cm (B) 1.2 mm
(C) 2.4 cm (D) 2.4 mm (CJ 6000 A (D) noo A
139. A nicol prism is based on the action of 147. If three slits are used in Young's experiment instead of
(A) refraction (B) . double refraction two,weget
(C) dichroism (D) both (B) and (C) (A) no fringe pattern
(B) the same fringe pattern as that wit!) two slits
140. A thin sheet of glass (refractive index 1.5) of thickness · (C) a pattern with fringe width reduced to half of that
6 micron,_ introduced in the path of one of the in the two slit pattern
interfering beams in a double-slit experiment, shifts the (D) alternate bright and dim fringes
central fringe to a position earlier occupied by the fifth
bright fringe. The wavelength of light used is 148. When a transparent parallel plate of uniform thickness
(A) 3000 A (B) 6000 A t and refractive index n is interposed normally in the
path of a beam of light, the optical path is
cq 4500 A (DJ 7000 A
(A) increased by nt
(B) decreased by nt
141. Which of the following cannot be polarised?
(C) decreased by(n-1)1
(A) Radio waves (B) p rays
(C) Infrared rays (D) y rays (D) increased by (n-1)1

================================:::: = 2.63

www.puucho.com
Rahul Sardana with www.puucho.com

Advanced JEE Pl1ysics Optics & Modern Pl,ysics

149. In Young's experiment, monochromatic light is used to I I


(A) (B)
illuminate the two slits and interference fringes are 2 ..fi.
observed on a screen placed in front of the slits. Now if I
a thin glass plate is placed normally in the path of the (C) I (D)
4
beam coming from one of the slits, then
(A) the fringes will disappear 157. Interference pattern is obtained ol1 a screen due to two
(B) the fringe-width will decrease identical coherent sources of monochromatic light. The
(q the fringe-width will increase intensity of the central bright fringe is I . When one of
(D) there will be no change in the fringe-width the sources is blocked, the intensity become I0 • The
150. If the Young's double slit-experiment is performed with intensity in the two situations are related as
white light, then the colour which will have maximum (A) I =I0 (B) I= 2I0
fringe width is · (q I =3I0 (D) I =4I0
(A) Blue (B) Green
(C) Yellow (D) Violet , 15B. The phase difference between two wave trains giving
rise to a dark fringe in Young's double-slit experiment
151. In the interference pattern, energy is is ( where n is an integer )
(A) created at the positions of maxima
(B) destroyed at the positions of minima (A) ZERO (B) 2,m+2:
2
(C) conserved but is redistributed
(D) not conserved (C) 2nn + 1t (D) 21tn +.!:
4
152. Fluorescent tubes give more light than a filament bulb
of same power because 159. A Young1s double-slit set-up for interference is shifted
(A) the tube contains gas at low temperature from air to within water. Then the
(B) ultraviolet light is converted into visible light by (A) fringe pattern disappears
fluorescence (B) fringewidth decreases
(C) light is diffused through the walls of the tube (C) fringewidth increases
(D) it produces more heat than bulb (D) fringewidth remains unchanged

153. Energies of photons of four different electromagnetic 160. Two interfering beams have intensities in the ratio of
radiations are given below. The energy value 9 : 4 . Then the ratio of maximum to minimum intensity
corresponds to a visible photon is equal to iri the interference pattern is
(A) 1 eV (B) 2 eV (A) 25:1 (B) 13:5
(C) 5 eV (D) 1000 eV (C) 5:1 (D) 3:2

154. Atomic spectrum should be 161. In interference with two coherent beams of light, the
(A) pure emission line spectrum fringe width is proportional
(B) emission band spectrum (A) to wavelength
(C) absorption line spectrum (B) to inverse wavelength
(D) absorption band spectrum (C) to square of wavelength
(D) to inverse square of wavelength
155. If a torch is used in place of monochromatic light in
Young's experiment 162. The fringe pattern observed in Young's double-slit
(A) fringes will appear as for monochromatic light experiment is
(B) fringes will appear for a moment and then they (A) a diffraction pattern
will disappear (B) an interference pattern
(C) no fringes will appear (C) a combination of diffraction and interference
(D) only bright fringes will appear patterns
(D) neither a diffraction nor an interference pattern
156. A beam of unpolarized light of intensity I is passed
first through a tourmaline crystal A and then through 163. In Young 1s interference experiment with one source and
another tourmaline crystal B oriented so that its two slits, on~ slit is covered with a cellophane sheet
principal plane is parallel to that of A. If A is now which absorbs half the intensity. Then
rotated by 45° in a plane perpendicular to the direction (A) no fringes are obtained
of the incident ray, the intensity of the emergent light (B) bright fringes will be brighter and dark fringes
will be will be darker
(C) all fringes will be dark
= 2.64 ==================================
www.puucho.com
Rahul Sardana with www.puucho.com

Wave Optics
(D) bright fringes will be less bright and dark fringes ~69. In the double-slit experiment, the distance of the second
will be less dark dark fringe from the central line is 3 mm. The distance
of the fourth bright fringe from the central line is
164. The distance between sources in a biprism _of angle a (A) 6mm (B) 8mm
and refractive index µ , if the source is placed at a (C) 12mm · (D) 16mm
distance a from it is
(A) 2(µ-l)cx {B) 2(µ-l)aa 170. In Young's double-slit experiment, we get 60 fringes in
(C) (µ-l)cx (D) (µ-l)cxa the field of view if we use light of wavelength 4000 A .
The number of fringes we will get in the same field of
165. To obtain a sustained interference pattern, we require view if we use light of wavelength 6000 A is
two sources which emit radiation of (A) 40 (B) 90
(A) the same frequency. (C) 60 (D) 50
(B) nearly the same frequency.
(C) the same frequency having a definite phase 171. With a monochromatic light, the fringe-width obtained
relationship. in a double-slit experiment is 1.33 mm. If the whole set-
(D) different wavelengths. up is immersed in water of refractive index 1.33, the
new fringe-width will be
166. A thin mica sheet of thickness 2 x 10-< m and refractive {A) 1.33 mm (B) 1mm
index (µ =1.5) is introduced in the path of the first 1.33
. (C) 1.33x1.33 mm (D) --mm
wave. The wavelength of the wave used is 5000 A . The 2
central bright maximum will shift
(A) 2 fringes upward (B) 2 fringes downward 172. Two waves having amplitudes in the ratio 5: 1 produce
(C) 10 fringes upward (D) None of these interference. The ratio of the maximum to the minimum
intensity is
167. In Young's double slit experiment, the slits are {A) 25:1 (B) 6:4
0.5 mm apart and interference pattern is observed on a (C) 9: 4 (D) 3:2
screen placed at a distance of 1 m from the plane
containing the slits. If wavelength of the incident light is 173. If the intensities of the two interfering beams in Young's
6000 A ,.then the separation between the third bright double-slit experiment are I1 and 12 , then the contrast
fringe and the central maxima is between the maximum and minimum intensities is
(A) 4 mm (B) 3.5 mm good when
(C) 3 mm (D) 2.5 mm (A) [I, - I, [ is large
168. Interference fringes are obtained due to the interference (B) [11 -I,[ is small
of waves from hvo coherent sources of light with
(C) either J1 or 12 is zero
amplitudes a1 and a,_ ( a,. = 2a,). The ratio of the
(D) I,=I,
maximum and minimum intensities of light in the
interference pattern is
(A) 2 (B) 4
(C) 9 (D) 00

2.65 =
www.puucho.com
Rahul Sardana with www.puucho.com

This section contains Multiple Correct Choice Type Questions. Each question has four choices (A), (B), (C) and (D), out of which
ONE OR MORE is/ are correct.

1. In an interference arrangement similar to Young's < i,_,


(B) If ).. d < at least on~ more maximum (besides
double-slit experiment, the slits S1 and S2 are the central maximum) will be observed on the
illuminated with coherent microwave sources, each of screen
(C) If the intensity of light falling on slit 1 is reduced
frequency 106 Hz . The sources are synchronized to
so that it becomes equal to that of slit 2, the
have zero phase difference. The slits are separated by a
intensities of the observed dark and bright fringes
distance d = 150.0 m. The intensity I(8) is measured as
will increase
a function of 8, where 0 is defined as shown. If ! 0 is (D) If the intensity of light falling on slit 2 is increased
the maximum intensity, then I(8) for O,; 8,; 90° is so that it becomes equal to that of slit 1, the
given by ·1 intensities of the observed dark and bright fringes
will.increase ;
Os,
f
d/2
3. In a modified YDSE ·. experiment if point source of
monochromatic light O is placed in such a manner that
QS1 -OS, = ~, where ).._ is the wavelength of light and
f
d/2
-- 8 ---------

S1 , S2 are the slits separated by .a distance 2).. . Then


value(s) of 8 for which a maxima is obtained is/ are
*
(A) I(8)= I, for 8=30° - (!_ _____
2
(B) I(8) =.!._. for 8 = 90°
4 s,
(C) I(8)=I0 for 8=0°
(D) I(8) is constant for all values of 8 (A) sm . -1(1)
-
8
(B) . -1(--1)
sm
4
2. In a Young's double slit experiment, the separation
between the two slits is d and the wavelength of the (C) . -1(5)
sm -
6
(D) . -1 ( 7)8
SID --

light is )._ . The intensity of light falling on slit 1 is four


times the intensity of light falling on slit 2. Choose the 4. Two coherent waves represented by
correct choice(s).
2
-rot+i) and
(A) If d =).., the screen will contain only one y1 =Asin( ';:'
1

maximum
. (2nx,
y, = Asm T - rot +6")
= 2.66 ================================::i
www.puucho.com
Rahul Sardana with www.puucho.com

Wave Optics
are superposed. The two waves will produce (D) the amplitude ratio is 2
(A) constr.uctive
. _inter
. ference a t x - x = M
ll:\.
1 2
9. Four coherent light waves are represented by
. mter1.erence
. ' (i) y, = a, sin(rot)
(B) constructive at x1 - x2 = -23,.
24 (ii) y 2 =a,sin(rot+~)
(q destructive interference at x1 - x 2 = ~: (iii) y, = a1 sin(2rot)
(iv) y, =a2 sin(2rot+~)
(D) d estructive
. mter
. £erence a t x - x =~
ll:\.
1 2 Interference fringes may be observed due to
superposition of
5. (A) (i) and (ii) (B) (i) and (iii)
To obse~e a stationary interference pattern formed by
(C) (ii) and (iv) (D) (iii) and (iv)
two light waves, it is not necessary that they must have
(A) the same frequency
10. If one of the slits of a standard Young's double slit
(B) the same amplitude
(q a constant phase difference experiment is covered by a thin parallel slit glass so that
(D) the same intensity it transmits only one half the light intensity of the other,
then
6. Two point monochromatic and coherent sources of light
(A) The fringe pattern will get shifted towards the
covered slit
of wavelength A. are placed on the dotted line in front
(B) The fringe pattern will get shifted away from the
of a large screen. The source emit waVes in phase with
covered slit
each other. The distance between S1 and S2 is d while
(C) The bright fringes will become less bright and the
their distance from the screen is much larger. Then for dark ones will become more bright
(D) The fringe width will remain unchanged

11. If the first minima in a Young's slit experiment occurs


S1 S2 directly in front of one of_ the slits, (distance between
•----•------------- 0 slits and screen d = 5 cm ) then the wavelength of the
1+-d-+t
radiation used can be
Screen (A) 2cm (B) 4cm
2 4
(q -cm (D) - cm
3 3
3
(A) d = ,. , 0 will be a minima
2
12. Two coherent sources A and B emitting light of
(B) d = 3,. , there will be a total of 6 minima on· screen
wavelength A. are placed at positions (-D, 0) and
(q d = A. , there will be one maxima on the screen
(D) d = 2A , there will be two maxima on the screen (-D, 3A.) respectively D»1,,
y
7. If white light is used in a Young's double-slit
experime~t, then
(A) bright white fringe is formed at the centre of the
screen
(B) fringes of different colours are observed clearly
only in the first order
(q the first-order violet fringes are closer to the centre o~-----•x
of the screen than the first order red fringes (A) number of minima on y-axis is 6
(D) the first-order red fringes are closer to the centre of (B) number of minima is more than nurnbef of
the screen than the first order violet fringes maxima on y-axis
(C) number offfiaxima on x-axis is 3
8. In the Young's double slit experiment, the interference (D) number of maxima on x-axis is more than number
\ pattern is found to have an intensity ratio between of minima on x-axis
bright and dark fringes as 9. This implies that -
(A) the intensities at the screen due to the two slits are 13. In the figure A , B and C are three slits each of them
5 units and 4 units respectively individually producing the same intensity 10 at P
(B) the intensities at the screen due to the two slits can
be 4 units and 1 unit respectively when they are illuminated by parallel beam of light of
(C) the amplitude ratio is 3

2.67 =
www.puucho.com
.., Rahul Sardana with www.puucho.com

Advanced JEE Physics Optics & Modern Physics

14. White light is used to illuminate the two slits in a


wavelength 1,. • it is given that BP -
'
AP =!:.2 . Also given Young's double-slit experiment. The separation
between the slits is b and the screen is at a distance
that d « D, then wavelength 1,. and resultant intensity
I at P will be d(» b) ·from the slits. At a point on the screen directly
in front of one of the slits, certain wavelengths are
missing.. Some of these missing wavelengths are
b' 2b'
(A) 1..=~ (B) 1..=-
d d
b' 2b'
(C) 1..=- (D) 1..=-
3d 3d

'
15. The fringe width in Young's double-slit experiment can .
----------------- p
t + - - - D ~---<~ be increased by decreasing "
(A) · separation of the slits
(B) frequency of the source of light
2d' • 4d' (C) distance between slit and screen
(A) 1,.=- (B) 1..=-
D D (D) wavelength of the source of light
(C) I= 2I," (D) •I=I,

,I

.,

...

= 2.68 ================================::i
www.puucho.com
Rahul Sardana with www.puucho.com

This section contains Reasoning type questions, each having four choices (A), (B), (C) and (D) out of which ONLY ONE is
correct. Each question contains STATEMENT 1 and STATEMENT 2. You have to mark your answer as
Bubble (A) If both statements are TRUE and STATEMENT 2 is the correct explanation of STATEMENT 1.
Bubble (B) If both statements are TRUE but STATEMENT 2 is not the correct explanation of STA,TEMENT 1.
Bubble (C) IfSTATEMENTl is TRUE andSTATEMENT2is FALSE.
Bubble (D) If STATEMENT 1 is FALSE but STATEMENT 2 is TRUE.

Statement-I:
Statement-2:
If the phase difference between the light waves
The fringe width is inversely proportional to the
emerging from the slits of the Young's experiment is
distance between the two slits in simple YDSE :
1t -radian, the central fringe will be dark.

Statement-2: 5. Statement-1:
2n The minimllffi slit separation d for interference to
Phase difference is equal to times the path
1,. produce at least one maxima other than central maxima is
difference. 31,..
Statement-2:
2. Statement-1: For a maxima, path difference equals n'J... . The maximum
When a thin transparent sheet is placed in front of both value of path difference is d .
the slits of Young's experiment, the fringe width will
increase. 6. Statement-I:
Statement-2: Two slits in YDSE are illuminated by two different
In Young's experiment the fringe width is proportional sodium lamps emitting light of saffie wavelength. No
to wavelength of the source used. interference pattern is observed.
Statement-2:
3. Statement-I: To obtain interference pattern, source must be coherent.
In Young's double slit experiment, we observe an Two different light sources can never be coherent.
interference pattern on the screen if both the slits are
illuminated by two bulbs of same power. 7. Statement-I:
Statement-2: In Young's double slit experiment interference pattern
The interference pattern is observed when source is disappears when one of the slits is covered by
monqchromatic and coherent. transparent slab.
Statement-2:
4. Statement-I: . Interference occurs due to superimpoSition of light wave
No interference pattern is detected when tw'o coherent from two coherent sources.
sources are infinitely close to each other in sim_ple
YDSE. 8. Statement-I:
When a thin transparent sheet is placed in front of both
the slits of Young's experiment, the fringe width will
increase.
2.69 =
www.puucho.com
Rahul Sardana with www.puucho.com

Advanced JEE Physics Optics & Modern Physics

Statement-2: Statement-2:
In Young's experiment the fringe width is proportional In Young's double slit experiment, fringe width is given by
to waveiength of the source used. re1a· " :w .
!ion e =d .
9. Statement-I:
Total number of maxima obtained over screen ren:iains 16. Statement-I:
same whether Young's Double slit experiment is Interference pattern is obtained on a screen due to two
performed in air or in water with same setup. identical coherent sources of monochromatic light. The
Statement-2: intensity at the central part of the screen becOmes one-
fourth if one of the source is blocked.
f3watcr = f3air (in Young's double slit experiment).
Statement-2:
µwater
The resultant intensity is the sum of the intensities due
to two sources.
10. Statement-I:
Interference obeys the Law of Conservation bf Energy.
17. Statement-1:
Statement-2: Thin films such a soap bubble or a thin layer of oil on
The energy is redistributed in case of interference. water show beautiful colours when illuminated by
monochromatic light.
11. Statement-I:
Statement-2:
Geometrical optics can be regarded as the limiting case
Colour in film are obtained due to interference between
of wave optics.
reflected light from the upper & lower layer of film.
Statement-2:
When size of obstacle Or opening is very large 18. Statement-1:
compared to the wavelength of light then wave nature The fringe obtained at the centre of the screen is known
can be ignored and light can be assumed to be as zeroth order fringe, or the central fringe.
travelling in straight line.
Statement-2:
Path difference between the wave from S1 and S2 ,
12. Statement-I:
Light from two coherent sources is reaching the screen. reaching the central fringe (or zero order fringe) is zero.
If the path difference at a point on the screen for yellow
3 19. Statement-1:
light is : , then the fringe at the point will be The phase difference between any two points on a
wavefront is zero.
coloured.
Statement-2:
Statement-2:
Light from the source reaches every ·point of the
Two coherent sources always have constant phase
wavefront at the same time.
relationship.
20. Statement-1:
13. Statement-1:
In Young's double slit experiment interference pattern
The maximwn intensity in interference pattern is four
disappears when one ofthe slits is closed.
times the intensity due to each slit. '
Statement-2:
Statement-2:
Interference occurs due to superimposition of light wave
Intensity is directly proportional to square of
from two coherent sources.
amplitude.

14. Statement-1:
21. Statement-1:
If a clean glass slide is observed under white light, one
Interference can be obtained by using two different
does not observe any colours. However, if this slide is
lamps.
. touches with oily hands, coloU)"ed fringes appear on the
Statement-2: slide.
Two different lamps are incoherent sources as constant
phase difference cannot be maintained between them.
Statement-2:
These fringes are due to interference of reflected light,
15. Statement-1: reflected from the upper and lower surfaces of the thin
Interference pattern is made by using blue light instead oil film.
of red light, the fringes becomes narrower.

= 2.70

www.puucho.com
Rahul Sardana with www.puucho.com

This section contains Linked Comprehension Type Questions or Paragraph based Questions. Each set consists of a Paragraph
followed by questions. Each question has four choices (A), (B), (C) and (DJ, out of which only one is correct. (For the sake of
competitiveness there may be a few questions that may have more than one correct options)

Comprehension 1 3
5. If intensity at center is of maximum intensity then
In a Young's experiment the upper slit is covered by a 4
thin glass plate of refractive index µ =1.4. The interference minimum value of 1 is
pattern is observed using light of interference 5000 A. Based (A) t = 2777.7 A (B) t=3188 A
on above information, answer the follOwing questions.
(C) · t = 4188.8 A (D) t = 2122.9. A
1. It is observed that
(A) the central maxima shifts upwards. Comprehension 2
(B) the central maxima shifts downwards. A thin film of a specific material can be used to decrease
(C) fringe pattern will change after introduction of the intensify of reflected light. There is destructive
thin plate. interference of waves reflected from upper and lower
(D) · none of the above phenomenon is observed. surfaces of the film. These films are called non-reflecting or
antireflection coatings. The process of coating the lens or
2. Now a thin plate of refractive index 1.7 is placed in surface with non-reflecting film is called blooming as shown
front of lower slit then in the figure. The refracting index Of coating ( n1 ) is less than
(A) central maxima will be obtained above centre. that of the glass (n2 ). 'Based on above information, answer
(B) central maxima will be obtain at centre. the following questions.
(C) there will .not be any change in central maxima 2
after introduction of thin plate.
(D) no conclusion can be withdrawn without knowing
the thickness of the plate. Air

3. Assume the thickness of both thin glass plate is t , the


·path difference between waves incident at center is R.1. =n, Glass
(A) 0.5! (B) 0.3!
(C) 0.8! (D) l.7 t 6. If the light of wavelength 1,. is incident normally and
1.5
the thickness of film is t then optical path difference
4. If minima is to be obtained at center then minimum between waves reflected from upper and lower surface

,. ,.
value of t is (source wavelength is 1,.) of film is

(A) 2n1t
·

(B)
·

2n t--
,.
(A) ! = - (B) t=- 1 2
0.6 0.3
. (C) (D) 2!
(C) (D) t = __!,:__
0.15

2.71. =
www.puucho.com
Rahul Sardana with www.puucho.com

Advanced JEE Physics Optics & Modern Physics

7. Magnesium fluoride (MgF,) is generally used as anti- 14. The phase difference introduced by the film is
reflection coating. If refractive index of MgF, is 1.38 (A) radian (B) radian
2"
1t
then minimum thickness of film required for
1-=550 nm is (C) .': radian (D) 2: radian
(A) 112.4nm (B) 78.2nm 3· 4
(C) 99.64 run (D) 225nm.
15. The power in watt received at the focal point F of the
8. Assuming that the thickness of film in above problem is lens is
not technically possible to manufacture, then next (A) 2 µW (B) 5µW
thickness of film required is (approximately) (C) 6 µW (D) 7µW
(A) 298.9 nm (B) 271.7 nm
(C) 304.7 run (D) 550 nm Comprehension 4
A monochromatic beam of light of wavelength
Comprehension 3 1- = 600 nm falls on Young's double slit experiment
In a modified Young's double slit experiment, a apparatus as shown in figure. A thin sheet of glass is
monochromatic uniform and parallel beam of light of inserted in front of lower slit S, . Based on above
10
wavelength 6000 A and intensity wm-2 is incident information, answer the following questions.
. "
normally on two circular apertures A and B of radii
0.001 m and 0.002 m respectively. A perfect transparent s,
film of thickness 2000 A and refractive index 1.5 for the
wavelength of 6000 A is placed in front of aperture A . The ~ ---------- 0
lens is symmetrically placed with respect to the apertures. s, D
Assume that 10% of the power received by each aperture µ, t
S,S, = d(«D)

--
goes in the original direction and is brought to the focal spot.

-A II
D

16. The central bright fringe can be obtained

-- F
(A) at 0
(B) at O or below 0
(C) at O or above 0

-
_s

Based on the a~ove facts, answer the following questions.


(D) anywhere on the screen

17. If central bright fringe is obtained on screen at O, then


we have
9. The power received at A is (A) (µ-l)t=d sine (B) (µ-l)t = dcos8
(A) 10.., W (B) 4x10_, W d
(C) µt=de (D)
(C) 10-< W (D) 4x10-< W µ-1 sine

10. The power received at B is 18. The phase difference between central maxima and fifth
(A) 10-5 W (B) 4 x 10-5 W minima is
(C) 10-< W (D) 4x10_. W
(A) "6 (B) 91t

11. The power transmitted through A is (C) 31t


(A) 10-5 W (B) 4 x 10-s W (D)
2
(C) 10-< W (D) 4x10-< W
19. Fringe width for the pattern obtained on screen, if
12. The power transmitted through B is 1-=600run, µ=1.5, d=3mm, D=2m and 8=30°
(A) 10-5 W (B) 4x10-5 W is
(C) 10-< W (D) 4 X 10-< W (A) 2x10 5 run (B) 4x10 5 nm
4
(C) 10 run (D) 3x106 nm
13. The path difference introduced by the film is
(A) 10"" m (B) 10.., m
(C) 10-' m (D) 10_, m

= 2.72 i:====================================
www.puucho.com
Rahul Sardana with www.puucho.com

Wave Optics

20. Assume if a(<~) is increased then for a given value of 25. Minimum thickness of film for. destructive interference
in transmitted light system is
µ (A) 150 nm (B) 200 nm
{A) central maxima will move downwards. (q 250 nm (D) 100 nm
(B) central maxima will move upward.
(q fringe width will increase. Comprehension 7
(D) fringe width will decrease. A narrow tube is bent in the form of circle of radius R
as shown. Two small holes S and D are made in the tube at
Comprehension 5 the positions right angles to each other. A source placed at S
In the arrangement shown in the figure, the distance D generates a wave of intensity 10 which is equally divided
is large compared to the separation d between the slits. into two parts. oi,e part travels along the longer path; while
Monochromatic light of wavelength 1.. is incident on the slit, the other travels along the shorter path. Both the part waves
based on the information provided answer the following meet at point D where a detector is placed. Based on above
questions. information, answer the following questions.
I

I . (D»d)
I

- du 0
=:
1+-
~
o---- 2D ---+i
~Screen

21. The minimum value of d for which there is a dark :o


fringe at the point O is
26. Maximum intensity produced at. D is given by
{A) 4I0 (B) 3I0
(A)"
(q 2I, (D) I,
(B) ff \ 27. The maximum value of wavelength 1,. to produce a
(q ,ffJ5 maximum at D is given by
(D) not possible t~ be calculated (A) 1tR (B) 21tR
(C) 1tR (D) 31tR
22. The position of first bright fringe for the minlmum 2 2
value of d is
d 28. The maximum value of wavelength 1,. to produce a
(A) below (B) d above
2 minimum at D is given by
3d 3d (A) 1tR (B) 27tR
(q below (D) - above
2 2 (C) 1tR (D) 31tR
2 2
23. The fringe width is
(A) 3D1.. · 3D1.. Comprehension 8
(B) When Fi'esnel 1s biprism experiment is performed in air
4d 2d
2D1.. then distance between coherent sources is 0.5 mm and
(q D1.. (D)
d distance between source and screen is 1 m . Fringe width
d
obtained in air is 1 mm . Refractive index of biprism is 1.5.
Now the experiment is performed in water having refractive
Comprehension 6
If light incident ·on a thin film has wavelength as index µ. =±.If the refractive index of the biprism is µ = ~ .
900 nm and refractive index of film is 1.5. Based on above 3 2
information, answer the following_questions. Based on above information, answer the following questions.

24. Minimum thickness of film needed for constructive 29. The distance between coherent sources in water is
interference in reflected light system is 1
{A) 1mm ,(B) -mm
(A) 100 nm (B) 150 nm 2
(C) 200 nm (D) 250 nm 1 1
(C) -mm (D) -mm
4 8
2.73=

www.puucho.com
Rahul Sardana with www.puucho.com

Advanced JEE Physics Optics & Modern Physics

30. The fringe width in. water is


(A) 1 mm (B) 2mm
¾is plac~d. The interference pattern i~ observed on a screen
(C) 3mm (D) 4mm at a distance D = 2 m from the slits. Based on above
information, answer the following questions.
31. The wavelength of light in air is
(A) 4000 A (B) 4500 A
(CJ 5000 A (DJ 6000 A I
Comprehension 9 .
~-f
-·d-------------------- C
In a YDSE experiment, the two slits are covered with
transparent membranes of negligible thickness which allow ~J ,.__ _ _ D - - - - . , I I
light to pass through it but does not allow water. A glass slab
of thickness t = 0.41 mm· and refractive index µ, = 1.5 is
placed in front of one of the slits as shown in figure. The 35. The angular position of the central maxima w.r.t.
separation between the slits is d = 0.30 mm. The entire space central line is
to the left of the slits is filled with water of refractive index (A) 60° (B) 45°
µ."= ½- A coherent light of intensity I and absolute (C) 30° (D) 15°

wavelength ;. = 5000 A is being incident on the slits making 36. The order of minima closest to centre C of screen is
an angie of 30° with horizontal. Screen is placed at a (A) 50 (B) 49
distance D = 1 m from the slits. Based on above information, (C) 48 (D) 47
answer the following questions.
37. The number of fringes that will pass over C , when the
transparent slab from the lower slit is removed is
ES:._,---;;;"'IP +
y
(A) 100
(C) 96
(B) 98
(D) 94
-------- 0 t
Comprehension 11
In a Young's double slit experiment set-up source S of
water
Screen wavelength 5000 A illuminates two slits S1 and S2 , which
S,S, = d( «D)
t+----- D ___., act as two cOherent sources. The source S oscillates about its
shown position according to the equation y = 0.5sin(1tt),
32. At point O, equidistant from slits we get where y is in millimetres and t in seconds. Based on above
(A) 9th dark fringe (B) 10th dark fringe
(C) 11th bright fringe (D) 10th bright fringe information, answer.the following questions.
y
33. Central maxima is located at
5
(A) y=+- cm
6
(B)
5
y=--cm
6
Is. Lx
----------------- p
5 5 s
(C) y=+-cm (D) y=--cm
3 3

34. The ratio of intensity at point P at y =! cm on screen


8
and maximum intensity is
(A) 0 (B) 1 38. The position of the central maxima as a function of. time
is best represented by the equation
(C) _!_ (D) 1 (A) -cos(1tt) (B) cos(1tt)
..fi. 2
(C) -sin[(1t+l)t] (D) -sin[1t(l+t)]
Comprehension 10
Light of wavelength ;. = 500 nm falls on two narrow 39. The minimum time t at which the intensity at point P
on the screen exactly in front of the upp~ slit becomes
slits placed a distance d =50 x 104 cm apart, at an angle
maximum is
cj, =30° relative ~o the slits shown in figure. On the lower slit
1 1
a transparent slab of thickness 0.1 nm and refractive index (A) - s (B) - s
2 3
= 2.74 ;:;:===================================
www.puucho.com
Rahul Sardana with www.puucho.com

Wave Optics
1 y
{C) - s (D) 1 s
6
SI
40. The minimum time f at which the intensity at point P
on the screen exactly in front of the upper slit becomes s() }------------- o
minimum is
1
s,, D
(A) 1 s (B) - s
2
1 1 43. The central maximum formed on the y-axis is located at
(C) - s (D) -s
3 6 11
{A) y= mm above O
3
Comprehension 12 11 .
(B) y= mm below 0
A vessel ABCD of 10 cm width has two small slits S1 3
and S2 sealed with identical glass plates of equal thickness. 14
(C) y = mm above 0
The distance between the slits is 0.8 mm. POQ is the line 3
perpendicular to the plane AB and passing through O , the (D)
middle point of S1 and S2 • A monochromatic light source is
kept at S , 40 cm below P and 2 m from the vessel, to
44. The ratio of light intensity of point O to the maximum
illuminate the slits as shown in the figure. Based on above
information, answer the following questions.
fringe intensity is
{A) 1 1
(B) 2

1
s,
D

(C)
4
3
4
(D) 1

f---------------------f o -------- 0 45. Assuming the 600 nm light to be replaced by white


40 cm S1
t...., --- 2 m ---+t<- 10cm
s
light of range 400 to 700 nm, the wavelengths of the
light that form maxima exactly at point O are
B~------'C
(A) l300 nm, 500 nm (B) 1400 nm, 600 nm
3 3
41. The position of the central bright fringe on the other
13 0
wall CD with respect to the line OQ is (C) ; nm, 650 nm (D) l:OO nm, 650 nm
{A) 2 cm below Q (B) 2 cm above Q
(C) 4 cm belo~ Q (D) 4 cm above Q
Comprehension 14
In the Young's double slit experiment a point source of
42. It is observed that when a liquid is poured into the
vessel and filled upto OQ , the central bright fringe is 1c = 5000 A is placed slightly off the central axis as shown in
the figure. Based on the information provided, answer the
found to be at Q . The refractive index of the liquid is
following questions.
(A) 1.0008 (B) 1.0004
(C) 1.0016 (D) 1.0012
Is ttp
Comprehension 13 t 1· . :t5f mm
The Young's double slit experiment is done in a 1'-7m"--~
-'r' _10-+-1m+-----:li10
medium of refractive index
.
±.A light of 600 nm wavelength
3
is fallirig on the slits having 0.45 mm separation. The lower
,s, ~

slit S2 is covered by a thin glass sheet of thickness 10.4 µm 1+-- 1 m--1----- 2 m---...
and refractive index 1.5. The interference pattern is observed 46. The order and nature of the interference at the point P
on a screen placed 1.5 m from the slits as shown in the is
figure. Assume that all wavelengths in the problem are for
(A) SO" order, maxima (B) SO" order, minima
the given medium of refractive index ½ and ignore (C) 70" order, maxima (D) 70" order, minima
dispersion. Based on above information, answer the
following questions.
================================== 2.75 =
www.puucho.com
Rahul Sardana with www.puucho.com

Advanced JEE Physics Optics & Modern Physics

47. The order and the nature of the interference at O is from each other by a distance 6 mm and 1.5 mm in the
(A) 50., order, minima (B) 50., order, maxima other. Now lens is removed and interference pattern is
obtained on the screen. Based on the information provided,
(CJ 20., order, minima (D) 20., order, maxima
answer the following questions.
48. If the zero order maxima is formed at O , then
(A) we should place a film of refractive index µ =1.5,
thickness 10 µm in front of 52 •
(B) we should place a film of refractive index µ =1.5, s rcreen
thickness 20 µm in front of 52 •
(CJ we should place a film of refractive index µ =1.5,
thickness 10 µm in front. of 51 . t+- 1 oo cm ------+1
(D) we should place a film of refractive index µ =1.5,
49. Focal length of the lens is
thickness 20 µm in front of 51 • (B) 20cm
(A) 16cm
(CJ 36 cm (D) 40cm
Comprehension 15
A narrow slit S allows monochromatic light of so. Fringe width of the_pattem on the screen is
wavelength ). = 6000 A to fall on a prism of very small angle (A) 0.1 mm (B) 0.2 mm
as shown in figure. A screen is placed at a distance (CJ 0.3 mm (D) 0.4 mm
l = 100 cm from the source to obtain an interference pattern.
51. If screen is displaced slightly away from prism, then
To determine the distance between the virtual images
(A) No interference pattern is observed
formed by the prism an experiment is done. The prism and
(B) fringe width remains same.
screen are kept fixed and a convex lens is moved between
the prism and the screen. For two positions of the lens
(CJ fringe width decreases.
(D) fringe width increases.
(between the prism and the screen) we get two sharp point
images on the screen in each case. 'fl1;e images are separated

= 2.1s c=====================================
www.puucho.com
Rahul Sardana with www.puucho.com

Each question in this section contains statements given in two Columns which have to be matched. The statements in
1

COLUMN-I are labelled A, B, C and D, while the statements in COLUMN-II are labelled p, q, r, s (and t). Any given statement
in COLUMN-I can have correct matching with ONE OR MORE statement(s) in COLUMN-II. The appropriate bubbles
corresponding to the answers to these questions have to be darkened as illustrated in the following examples:
If the correct matches are A --> p, s and t; B --> q and r; C --> p and q; and D --> s and t; then the correct darkening of bubbles will
look like the following :
P q r s t
A@@)(D@@
B@@@®CD
c.® ®©®CD
D@@)(D@@)

1. COLUMN-I shows the effect on the fringe pattern in YDSE corresponding to the changes mentioned in COLUMN-II.
Match the effects in COLUMN-I with the corresponding causes in COLUMN-II.
i COLUMN-I COLUMN-II
(A) Angular fringe width remains same (p) Screen is moved away from the plane of the slits
(B) Angular.fringe width changes (q) Wavelength of light used is decreased
(C) Fringe width (linear separation between two (r) The separation between the slits is increased
consecutive fringes) changes (s) The width of the source slit is increased
(ID) The fringe pattern may disappear

2.
'
Figure shows a set-up to perform Young's double slit experiment. A monochromatic source of light is placed at S. S1 and
52 act as coherent sources and interference pattern is obtained 011 the screen. Match COLUMN-I with COLUMN-II
keeping in mind the Young's double slit experiment.
Screen

J~ ____·i------
s,~
0

2.77=

www.puucho.com
Rahul Sardana with www.puucho.com

Advanced fEE Physics Optics & Modem Physics

C01'UMN-I CQLUMN-Il,
(A) S is removed and two real sources emitting light of (p) Interference fringes disapp~ar.
same wavelen~ are placed at S1 and S2 • (q) There is uniform iUumination on a large part of the
(B) Width of S1 is two times the width of s2·• screen.
(r) The zero order fringe will not form at O .
(q S1 is closed.
(s) Intensity of a dark fringe will ~e non-zero, but less
(D) A thin _transparent plate is placed in front of S1 • than the intensity of bright fringe.
Assuming negligible absorption by the plate.

3, Match the contents of COLUMN-I with the respective phenomenon in COLUMN-II.


COLUMN-I . COLUMN-II ./
(A) Shining of diamonds. (p) interference
(B) Light waves projected on oil surface shows seven (q) total internal reflection
colours. I (r) origin of spectra
(q Huygen's wave theory of light cannot explain. (s) photoelectric effect
(D) Phenomena which is not Explained by Huygen's
construction of wavelength.
'.J•

4. In Young's Double Slit Experiment, if .distance between slits is· d , distance between slit and screen is D , wavelength of
light used is. i.. • Then match COLUMN-I with COLUMN-II. .
[ . COLUMN-I COLlJMN:-Il
(A) For bright fringe, path difference. Di..
· (B) For dark fringe, path difference. (p) 2d
(q Displacement of fringe when, glass plate of thickness I
(q) 'D(µ-1)1
is placed. · d
(D) Distance between central maxima and first dark fringe (r) ni..
when glass plate of thickness I is used. ·
i..
(s) (2n-1)-
2

5; Match the contents of COLUMN-I with the respective contents of COLUMN-II.


I , COLUMN-I ·COLUMN-II
(A) Sources of variable phase difference,- (p) Incoherent sources
(B) Point on a wavefront behay~s as a light source. (q) Coherent sources
(q Net displacement is the vector sum of individual (r) Superposition principle .
displacement. (s) Huygen's principle
(D) Young's double slit experiment uses. ·
- - ' ~ . ,- .
6. In the YDSE appratus shown in figure, rue is the path difference between S2 P and S1P. H, now a glass slab is introduced
in°front of S~, then match the contents of COLUMN-I with the respective match"': in COLUMN-II.

-++-------ii' 0

s,~

= t::::===================================
2.78

www.puucho.com
Rahul Sardana with www.puucho.com

Wave Optics

COLUMN-I COLUMN-II
(A) Fringe width will (p) increase
(B) Fringe pattern will (q) decrease
(C) Number of fringes between O and P will (r) remain same
(D) t;x at P will (s) shift upward
(t) shift downward'

7. Match the quantities in COLUMN-I with their respective matches in COLUMN-II,


COLUMN-I COLUMN,11
(A) Point source of light (p) Spherical wavefront
(B) Limit of resolution of telescope (q) Amplitude division
(C) Interference (r) Superposition of waves
(D) Coherent sources (s) Radius of lens

8. In the light of possibility of occurrence of phenomena listed in COLUMN-I match the ·listings in COLUMN-I to the
corresponding waves in COLUMN-II.
[ COLUMN-I COLUMN-II
(A) Reflection (p) Non-mechanical waves
(B) Interference (q) Electromagnetic waves
(C) Diffraction (r) Visible light waves
(D) Polarisation (s) Sound waves

c:::::=================================== = 2.79

www.puucho.com
Rahul Sardana with www.puucho.com

In this section the answer to each of the question is a four digit integer, ranging from Oto 9999. The appropriate bubbles below
the respective question numbers in the ORS have to be darkened. For example, if the correct answer to question number X (say)
is 6092, then the correct darkening of bubbles will look like the following :
X.@e@@
CDCDCDCD
®®®·
@@@@
@@@@
@@ @@

~~~~
@®®®
®@e@
1. Interference pattern with Young's double slit 1.5 mm (a) central maxima (where path difference is zero) is
apart are formed on a screen at a distance 1.5 m from obtained.
the plane of slits. In the path of the beam from one of (b) third order maxima is obtained.
the slits, a transparent film of 10 micron thickness and
the refractive index 1.6 is interposed while in the path 4. An equiconvex lens of focal length 10 cm (in air) and
of the beam from the other slit a transparent film of
refractive index ~2 is put at a small opening on a tube of
.
15 micron thickness and a refractive index 1.2 is
interposed. Find the displacement of the fringe pattern, length 1 m fully filled with liquid of refractive index
inmm.
½. A concave mirror of radius of curvature 20 cm is
2. Two Coherent radio point sources that are separated by cut into two halves m, and m, and placed at the end of
2 m are radiating in phase with a wavelength of
the tube. m1 and m, are placed such that their
0.25 m . If a detector moves in a large circle arowtd
their midpoint, at how many points will the detector principal axes AB and CD respectively are separated
show a maximwn signal? by 1 mm each from the principle axes of the lens.

3. In the figure shown the distance between the slits is


d = 201., where ). is the wavelength of light used. Find
the angle 0, in degree, where s•-e-----

-------- I t+20c

'~ A slit S placed in air illuminates the lens with lighi of


frequency 7.5 x 1014 Hz . The light reflected from m,

= ==================================
2.80

www.puucho.com
Rahul Sardana with www.puucho.com

Wave Optics
and m, forms interference pattern on the left and EF
1 ------------ p
of the tube. 0 is an opaque substance to cover the hole
created by the placement of m, and m,_ • Find :
(a) The position of the image, in cm, formed by lens-
water combination.
(b) The distance, in mm, between the images formed
by m. and m,. Screen
D>>d
(c) Width of the fringes on EF, in µm .
10. In a YDSE (young double slit experiment) screen is
5. A ray of light is incident on the left vertical face of the placed 1 m from the slits wavelength of light used is
glass slab. If the incident light has an.intensity I and on 6000 A . The fringes formed on the screen are observed
each reflection the intensity decreases by 90% and on
by a student sittingdose to the slits. The student's eye
each refraction the intensity decreases by 10%, find the
can distinguish two rieighboring fringes, if they subtend
ratio of the intensities of maximum to minimum in the
an angle more than 1 minute of the arc. Calculate the
reflected pattern.
maximum distance between the slits, in nun,. so that
fringes are clearly visible. Give your answer to the
nearest integer.

11. A parallel beam of white light falls from air on a thin


film in air whose refractive index is ,J3 .The angle of
incidence is i = 60° . Find the minimum film thickness
(in nanometer), if the reflected light is most intense for
6. Two slits are separated by 0.32 mm. A beam of 500 nm i..=6oooA.
light strikes the slits producing an interference pattern.
Determine the number of maxima observed in the 12. In a modified YDSE the region between screen and
angular range -30° < 8 < 30° . slits is immersed in a liquid whose refractive ind~x
5
· w1'th'
vanes time as µ1 = - t , un til"1t reaches a stead y
7. Interference fringes were produced by Young's double 2 4
slit method, the wavelength of light used being
6000 A . The separation between the two slits is 2 mm .
state value ¾.A glass plate of thickness T = 36 µm and

The distance between the slits and screen is 10 cm . refractive index µ = ~ is introduced in front of one of
When a transparent plate of thickness 0.5 mm is placed 2
over one of the slits, the fringe pattern is displaced by the slits. If the separation between the sources and the
5 mm . If µ be the refractive index of the material of screen is 1 m and the separation between the sources is
the plate, then find 5µ . 2 mm, then calculate the speed ·of the central maxima,
in mms-1 , when it is at O,
8. In Young's double slit experiment mixture of two light y
wave having wav~engths 1,. 1 = 500 nm and
i.., = 700 nm are being. used. Find the position next to
central maximal where maximas due to both waves

~ = 1000)
X
coincides. ( Given 0

9. Consider the interference at P between waves µ,


emanating from three coherent sources in same phase
located at 51 , S2 and S3 • If intensity due to each source 13. In a Young's double slit experiment, 12 fringes are
observed to be formed in a certain segment of the
is ! 0 =12Wm-' at P and ~=!::, then find the screen when light of wavelength 600 nm is used. If the
2D 3 wavelength of light is now changed to 400 nm , what
resultant intensity at P , in wm-2 • will be the new number of fringes observed in the same
segment of screen.

'

==================================== = 2.81

www.puucho.com
Rahul Sardana with www.puucho.com

Advanced JEE Physics Optics & Modern Physics

14. A glass wedge of angle 0.01 radian is illuminated by 15. In Young's double slit experiment the two slits act as
monochromatic light of wavelength 6000 A falling coherent sources of eq"i'1 amplitude A and
normally on it. Find the distance from the edge of the wavelength :>.. • In another experiment with the same set
wedge, in mm where the lQth fringe will be observed up the two slits are source of equal amplitude A and
due to the reflected light. wavelength :>.. but are incoherent. Find the ratio of
intensity of light at the mid-point of the screen in the
first case to that in second cas.e.

www.puucho.com
Rahul Sardana with www.puucho.com

• ICE!
1. (a) 7.8 µm 5n
8. (a)
2
(b) 0.6mm

2. 5 µm 25n
(b)
4
2"[(,/d' + !' -d) + µd']
3. (a) 9. 5890 A
A 2D
10. 1.2 µm
2
(b)
A
"(µ(Jd' +I' -1)+~)
2D 11. Zero order maxima will remain unchanged.

4. 94.8 nm Tenth order will now be at 4.55 mm.

5. 0.546 mm 12. 8.75 mm

6. 112.78 nm 14. 4.5mm

7. (a) 0.5 mm 15. 7

(b) 2.25 mm 16. 0.21

(c) 31, 17. (a) 3x10-4l 0

1 (b) 5.49
(d) ,/3 meter
18. 3.5 mm
(e) n = 5000 is not possible 19. 1.5

• 1.

2.
ICE II
11.8mm

9x10-4 m
9,

10.
±45° I ±135°

(a) 0.75
.BAS_l=D ON Dli=FRACTJ.0N &!f'()~_filS~IOl;fj

3. 0.2mm (b) 0.25

4. 40 m 11. 37.5%

5. 12.5 cm 12. !..


4
6. 1.3µm
13. 1:3
7. 0.2mm
16. 30°' 45°
8. 3.534 x 10.., rad

2.83 =
www.puucho.com
Rahul Sardana with www.puucho.com

Advanced JEE Physics Optics & Modern Physics

1. B 2. D 3. A 4. D 5. B
6. . D 7. D 8. D 9. B 10. A
L11. 0 12. D 13. A 14. C· 15. D
16. B 17. D 18. D 19. B 20. B
21.
26.
C
A
22.
27.
C
C
23. D 24. B 25. D I
28. D 29. C 30. B
31. A 32. O· 33. A 34. A 35. A
---- - - -
36. C 37. A 38. C 39. B 40. B
41. A 42. A 43. 0 44. D 45. C
----
46. C 47. B 48. B 49. A so. C
51. D 52. B 53. B 54. C 55. B
56. D 57. B 58. A 59. B 60. A
61. D 62. B 63. D 64. B 65. C
66. D 67. C 68. B 69. B 70. C
r--n. A 72. C 73. C 74. C 75. C
76. B 77. A 78. B 79. 'C 80. A
[- 81. B 82. B 83. D 84. C 85. A
86. B 87. B 88. B 89. C 90. A
91. C 92. B 93. D 94. C 95. B
96. D 97. D 98. D 99. A 100. A
~- A 102. 0 103. D 104. D 105. C
106. D 107. C 108. B 109. A 110. C
------
[ 111. D 112. A 113. C
··-------------- ------- 114. B 115. C
116. A 117. D 118. A 119. C 120. B
! 121. A 122. A 123. . D 124• A 125. C
----------
126. B 127. A 128. B 129. C 130. A
L 131. C 132. B 133. D 134. D 135. A
136. C 137. • C 138. D 139. D 140. B
141. B 142. A
-----
143. B 144. A 145. D
----
146. B 147. D 148. D 149. D 150. C
151. C 152. B 153. B 154. A 155. C
156. D 157. D 158. C 159. B 160. A
161. A 162.
-·------
C 163. D 164. B 165. C
166. A 167. B 168. C 169. B 170. A
171. B 172. C 173. D

, __1:.:..__:_A:.:C:.....
~ - - - - - -2. AB 3. AD 4. BD 5. BO
----------------------~=-__:c=----....J
6. ABC 7. ABC 8. BD 9. AB 10. AGO
L__1c...1:.:.·__:_A:.:C:.__ _ _ ____::12~-~-c_______1_3_._B_D_ _ _ _ _ _14_._A_c_ _ _ _ _ _1_5_._·,_A_B_ _ _. - . J

= c:===================================
2.84

www.puucho.com
Rahul Sardana with www.puucho.com

Wave Optics

- - - - - - - - - - - - - - - - ----- - - - - I
L _ ~_ _ _ _ _ _ _ _2.:___D_____ _ _ _ --~'.-~-- _________4:_ _B____ ---------~- D J
6. A 7. D 8.D 9.D 10.A
[ 11. A
16. C 17. D 18. D 19. A 20. A
21. A ______ _ ________________________________________________ j

I
~
1.
- ---------~-- - - - - - - - - - - - - - - - -
A _ _ _ _ _ _ _ 2_._D_________ 3·--~---------- -- 4. -- A ----- ------- 5.__ A ___________ I
_
- --1

6. A 7. C 8. A ____ 9. A 10. B
11. C - - - - - - 1 - 2 . D _________ 13. __ C ________ 14. __ C _ _ _ ______ 15. __
D_______- _ I
16. D 17. A 18. B 19. B 20. B .
--------- ---- ---- - - - - ---- --------- ----. -- --------
01._~_ 22. C ____ 23. D __________24. ___~-- ____________ 25:_!-________ )
26. C ~A ~B ___ aD me
31. -------:n:--D~_-_-_--_-_-
C __-_-_-_~.:__--~ ____ - ____ 34._ C _____________ 35. C________;

OT
36.

46.
B
B
C
~ A
42_. _ ~------ _____ :13_._ __[?__~
__
47. D 48. D
~ D
=---_-_-=:: _--_4_4._-_c_-_-__-_--_--_----~--= ~s-=-_--c~===~--~-J
~

49.
C

A
-

50.
A

B
51. D __-_: ----~--~-----~--=------_-_ ·=-:==-:-:-:=-----]
IANSWER----
r
I
1. A-; (p, s)
B--> (q, r)
2. A--> (p, q)
B-; (s)
3. A--> (q)
B--> (p)
4. A--> (r)
B--> (s)
- 5.--: =: l~!--·-1
C--> (p, q, r) C--> (p, q) C--> (s) C--> (q) C--> (r)

I 6.
D--> (q, r, s)

A-; (r) 7.
D--> (r)

A--> (p)
----------·-
8.
D-; (r)

----
A--> (p, q, r, s)
D--> (p)

-----
____ D-->(q,~

B--> (t) B--> (s) B--> (p, q, r, s)


C--> (r) C--> (r) C--> (p, q, r, s)
D-; (p) D--> (q) D--> (p, q, r)

--·--- --- -- ---------


I_
1. _3_ _ _ _ _ _ _ _ _
2. _32
_ _ _ _ _ _ _ _ _3_. (a)45,(b)59 _
------·-- -- ------------,
4._(a)B0,(b)4,(c}60 _ _ j
5. 361 6. 739 7. 6 8. 3

~~ - - - - - - - - - - - - - - - - - - - - - --------- ----'=--=----- ____


10. 2 11. 100 12. 3
...J
13. 18 14. 3 15. 2

2.85 =
www.puucho.com
Rahul Sardana with www.puucho.com

Therefore, net path difference t:.x. = (..Jd2 + t 2 -t) + i;:;


So, phase difference Is given by
$= 2•[(../d' +l' -d)+ µd']
(b) Upwards ,_ 2D
(b) When liquid is filled between slits and source S , then
yd -(1-.!)1= 4,_
D µ µ
t,x =µ(t../d' + I' -t)+ ~)
=> y=4.2mm
Downwards
=> $=~(µ(../d'+t'-t)+i~)
. 1(1-.!)+yd='4,.
µ D µ
=> y=0.6 mm 4. The reflected light will be minimum when rays 1 and 2 (shown in
figure) meet the conditi6n of destructive interference.
Rays 1 and 2 'both suffer an ad~itional phase change of 1t
2. Since.we are given that
(or 180°) after being reflected at surface of a denser medium.
Shift= 5 (fringe width)
The net change In phase due to
(µ-1)tD SW 1
reflection is therefore zero and the 2
=> d d condition for a reflection minimum
,. Air
=>
s,- 2sooo
l=-=--=50,000 A =5 µm requires a path difference of
µ-1 1.5-1 2
Hence,
2µ1=2
,.
3. f>x=SS,0-SS,O=..Jd'+l'-l
,- 550
=> => I= 4µ = 4 (1 _45 ) = 94.B nm

=>
5. The desired distance is
546x10-9 x0.4
y= I'nngew1'dh
t =d
AD
0.4x10-3
=> y = 0.546 x 10"' m = 0.546 mm

, ~,I ·First order minima f


--s-------il y =0 y'
(a) Geometric path difference between S2P and Sf is
1 ~ First order minima i
YJ~)dD
=~
2D
1
6. For constructive interference In case of soap film,
Their optical path difference = ,:;;

= ==================================
2.86

www.puucho.com
Rahul Sardana with www.puucho.com

Wave Optics

2µt=(n-i} n=1,2,3, .... 1' = (µ-1)t = (1.6-1)(1.964 x 10"')


2 2
For minimum thickness t, n =1 => A= 0.589x10-e m
1'
=> 2µ1=2 => 1'=5890A

1=~=~=112.78 nm 10. Shift = 5 (fringe width)


4µ 4x1.33
=> (µ-l)t~=1'~

7. · )a) . P= 1'D = (5ooox10·")(1) 0.5 mm 1' - 6x10·' -12 X 10"' m


t-
________
d 1x10-:i µ-1 (1.5-1) .
(b) y=(2n-1)w t=1.2µm
d
For n=5
11, P= (14.75-12.50) mm=0.25 mm
y=2.25 mm 10-1
1 , ),.' 5500
(c) At y= mm, y«D p = -i:-P =
3 6000
x 0.25 mm~ 0.23 mm

=> ,Ix= yd Zero order maxima will remain unchanged. Earl/er it was at
D 12.5 mm-12.25 mm.
Tenth order will now be at 12.25 mm+ 10P' = 14.55 mm.

Now resultant intensity, is given by 12. Given, d=0.1mm=10""'m, D=0.5m and


1=11 +I2 +2Mcos.6.4, · 1'=5000A=5x10·' m

=> I= 41 0 + 10 + 2 g cos.6.4' =510 +4l 0 cos( ~1t) =31 0 t.y=(Y,,).,. -(y,),,..,


(2x11-1)1'D
2d
?AD
d

d 10"' 2000 l:i.y= 7AD 7x5x10-~xO.S 8.75x10-3 m=S.75 mm


(d)
A 0.5x10-v 2d 2x10
n = 1000 is not very much less than 2000
Hence now .ruc =dsin0 must be used, so we get
dsine = n1' = 10001'
13. (a) Angular separation ( =~) of the fringes remains constant.

But the linear separation or fringe width increases in


proportion to the distance (D) from the screen.
=> 0=30° (b) As 1' decreases, fringe width (P oc 1') decreases.
Since, y =Dtane
(c) As d increases, fringe width (Pa::{) decreases.

(d) The interference pattern due to different component


(e) Highest order maxima colours of white light overlap (in-coherently). The central
bright fringes of different colours are at the same position.
n- =[f]=2000 Therefore, the central fringe is white. Since blue colour has
the lower A, the fringe closest on either side of the central
Hence, n = 5000 is not possible. white fringe is blue; the farthest Is red.

8. (a) .1>c =0.5 µm =5 x 10-1 ·m (e) Since in a medium the wavelength of light is 1'' = !:µ ,
So, phase difference is given by therefore the fringe width is given by
$ = (~)(,Ix)= ( 400~·10_, )<5 X 10-7) P=1''D=W.
d µd
=> $= 5n Thus, fringe width decreases by µ .
2
(b) t.X'=(µ-1)1=(1.5-1)(1.5)=0.75 µm 14. For bright fringes to coincide, we have
(.6.)c)ne! =_rue+ flx' =1.25 µITT= 1.25 X 1Q-6 ITT n1"-P n2A2D
-d-=-d-
2
$'=( " )(1.25x10"')
~~~ =~
400x10_,
=) ~: = ~: =
$' = 25n
4 i.e., 51t> order maxima of larger wavelength A. 2 will overlap with
6°' order maxima of smaller wavelength A. 1 •
9. Since, shift =2(Fringe Width)
SAD 6x750x10-9x2
(µ-1)1D 1'(20) So, Yrrin = d 2x10--3
=> d =-d-

c::::=================================== = 2.87

www.puucho.com
Rahul Sardana with www.puucho.com

Advanced JEE Physics Optics & Modern Physics

=> Ym1n=4.5x10-3m 2
$'=.!:+ " (1.5-1)(8000+10-")
2 6QQQ X 1o-lO
=> Ymrn =4.5 mm
=> $' =11.
6
15. Let n1 bright fringe of A1 overlaps with n2 bright fringe of A. 2 •
The intensity at P is now given by
Then,
I= 11 +1 2 +2~ cos~
n1A1D = n2 A2 D
d d 1
=> I'=l0 +11I0 +2~cos( ;1t)=2l0 {·: I'= 21,}
=> n1A1 =n2 A2
Solving this equation, we get,
=> .!1='-2 = 700 =!... ~ = 0.21
n, l., 400 4
4 7
17. (a) Since, file ~ (o. 3 xl0-,;(10xl0_,) _3x10-' m
6
3 5 A,<~
=> $=(~")(file)=(546~"10-• )(3x10-<)
4
P, <P2
2 => $ = 34.52 radian= 1978°
3
=> 1=989'
1 2 2

y=O
1 Since, Ip = I0 cos
2
(!)
Y=O
l., l., => 1p=3x10-"'I0

The ratio .!:!!. = !.... implies that 7th ' bright fringe of A. 1 will overlap
l.D (546x10-,)(1)
(b) Fringe width p d (0. x _,) 1.82x10--3 m
n, 4 3 10
with 4" bright fringe of A. 2 • Similarly 140, of A1 will overlap with 811'1 => P=1.82mm
of A2 and so on. Therefore, number of fringes between point P and the
central fringe are given by
So the minimum order of A.1 which overlaps with A. 2 is 7.
N=~=5.49
1.82
16. Without inserting the slab, path difference at P is given by
.6.x yd 0.15x10-a x2x10-a 18. Let n1 bright fringe co"!"esponding to wavelength i..1 = 500 nm
D 2
coincides with n2 bright fringe corresponding to wavelength
=> ~X=1.5'x10-7 m
i..2 =700 nm.

=>

=>n1A.27
n2 =i;=s
This implies that 7"' maxima of i..1 coincides with 5th maxima of
i..2 • Similarly 14~ maxima of i..1 wilf coincides with 10~ maxima
of A.2 and so.on.
Corresponding phase difference at P is So, minimum distance is

$=( 2: )<file) Ym1n = n1~p = 7x5x10-7 x10 3

=> Yrrin =3.5x10--3 m=3.5 mm


=>

=> 19. Here i..=6x10--s cm, t=7.2x10-4 cm


Displacement of the central bright fringe, file = sp
0

Since, intensity at P is given by I= 410 cos 2


(%) Since file= f(µ -1)t
l.
=> 1=4l COS (¾)=21
0
2
0 => f(µ-1)t=6P
l.
Phase difference after placing the glass sheet is
=> -l=6l. 6x6x10_. =0, 5
$'=$+~(µ-1)t µ t 7.2x10-4
=> µ=0.5+1=1.5

= ==================================
2.88

www.puucho.com
Rahul Sardana with www.puucho.com

For first dark band, sine = ~


1
1. For 10 bright fringes, we get, 0, 0 = gJ..
As the diffraction pattern is obtained in the focal plane of lens, The angular width of the central maximum in the diffraction
therefore pattern due to slit of width a is

tane =f 201 = 2A
a
For small e, tane = sine Since, 10 maxima of the double slit pattern within the central
X A maximum of the single slit pattern
f d => 10!:<2?:.
d a
x=(~}
=> a::;~
Since, l.=5900A=59x10-' m, f=50cm=0.50m 5

d=0.025 m=2.5x10-s m a=..!. mm=0.2 mm


5
59 x10-axO.SO 11.8x10-a m=11.8 mm
X 2.5x10-5
. d'
4. Smee, DF =T
2. Given, A, = 5890 x 10-10 m , J..2 = 5896 x 10-10 m ,
_where d = 4 mm =4x 10-a m, A= 400 nm =4x10-1 m
d=2µm=10--6m, D=2m
_(4x10"')' m
Distance of first secondary maximum from the centre of the =)
Df 4x1Q-J 40
1
screen is
3 DJ. Thus ray optics is valid upto a distance of 40 m from the
X=-- aperture.
2 d
For the two wavelengths, we have
5. Here size of Fresnel zone DF at the middle hill must be less that
X1=~DA, and X2 =~DJ.. 2
2 d 2 d 50 m.
Spacing betweeQ the first two maximum of sodium lines is given Distan\:e of either of the two hills from the middle hill is
by 40
D= km=20,000 m
3D 2
t.x =X, -x, = 2d(l., -J.,)
Since size of Fresnel's zone, DF = .Jw
<l.x 3x2x(5896-5890)x10-' -SxlO~ m => ..fw « 50
2x2x10--6 AD«2500
2500 2500
3. The linear separation between n bright fringes in an i.« = =0.125 m=12.5 cm
interference pattern on the screen is given by
D 20,000
nW Thus wavelengths longer than 12.5 cm will undergo serious
Xn=d diffraction effects.
Since, xn « D , the angular separation between n bright fringes
6. Given, A = 6500 A = 6500 x 10-10 m , 0 =30°
should be Forfirstminimum, dsin0=A
8 =x"=nA 6500 x10-10 6500x 10-10
" D d d= l.
sine sin30° 0.5
2.89 =
www.puucho.com
Rahul Sardana with www.puucho.com

Advanced JEE Physics Optics & Modern Physics


'
=> d=1.3x10-a m=1.3 µm If ais the angle between the transmission directions of the two
polaroids, then the intensity of light emerging from second
polaroid is
7. Given, D=2m, X1 =5mm=5x10-3 m,
I2 = I1COS29 =2COS
Io 29
l.=5000A=5x10-' m
Width of central maximum is Initially the two polaroids are crossed to each other i.e., 8 = 90° .
Po =2X 1 =2x5x10-3 m=10-2 m When polaroid is rotated through 60° , the angle between their
So, slit width is polarising directions will become
2D).. 2x2x5x10-7 8=90°-60°=30°
d= 2x10-4 m=0.2mm 2
Po 10-2
=> 12 =1-cos2(30°)=1-x(J3) =~1 0
2 2 2 8
8. From figure, it is clear that the angular separation between
central maximum and first order minimum is
Central
=>
,,
.!,_ =0.375

Maximum So, transmitted percentage is 1-x100 = 0.375x100 = 37.5%


I,

12. Obviously, the axis of P3 is inclined at 45° to the axes of P1


First order
Maximum and P2
Let amplitude of linearly polarised light emerging from P, be E0

). 8=0 The amplitude of light emerging from P3 is E0 cos45° = ]


-2
Next the amplitude of light emerging tro'm P2 is
0=3)._0=3).
2d 2d 1
~cos45°=~x--= Eo
3x589Ox10-10 J2 ,/2 ,/2 2
0
2x0.25x10-13 Let intensity transmitted by P1 = 10 oc E~
e = 3.534 x 1o-3 rad So, intensity after passing through P2 and P3 is

9. Since, I = ~

Using Malus Law, I= 10 cos 2 8 I=~


4
I
=> ; =loCOS 2 8 Thus the intensity becomes one~fourth of the maximum
transmitted intensity.
1
C0S8=±,n_ 13. The planes of polarisation of light beams A and B are mutually
at right angles. Initially, the beam B shows zero intensity.
=> a= ±45°, ±135°
Therefore, 8 = 90° for beam B and 8 = 0° for beam A . When
The same effect occurs no matter which sheet is rotated or in
which direction it is rotated. the polaroid is rotated through 30° , we have
0 = 60° for beam B and
10. According to Mal us Law, I= 10 cos 2 8 e= 30° for beam A
In this position, we have
(a) Here e =30°

=> l=I0 cos (30°)=1 x(


2
0 :J =¾l 0
(

=>
Intensity of emerging)= (Intensity of emerging)
beam A beam B
IA cos 2 (30°) = le cos 2 (60°)
=> __r__=0.75
I,
(b) Here e =60° 1_ cos 2 (60°) G)' =.!=,,3

l=l0 COS (60°)=1 0 ( 1)'


2
z' =-t
I
le cos 2 (30°)
(~J 3

__r__=0.25. sini
1, 14. From Snell's Law, ___ P = µ
s1nrP

11. Let 10 be intensity of incident unpolarised light. Then the sini


From Brewster Law, taniP =--~ = µ
intensity of light emerging from the first polaroid will be COSIP

I _1_
' - 2

= 2.so c===================================
www.puucho.com
Rahul Sardana with www.puucho.com

Wave Optics

::::> sinrP = cosiP The intensity of light emerging from P3 is given by


=> sinr,=sin(90°-~) 2
13 = 12 cos (90° -0) =¾1 0 cos2 0sin2 0 = ¾1 0 sin2 (20)
::::> rP =90°-iP
. '( 2e)-BI,_8x3_3
::::> iP+rp=90° Sin -------
10 32 4
Hence the reflected and transmitted rays are perpendicular to
each other. => sin(20) = ../3
2
15. Let P1 , P2 , P3 be the three polarisers and 0 be the angle => 20 = 60°
=> 0 =30°
between the transmission axes of P, and P2 • As P1 and P3 are
crossed, the angle between P2 and P3 is (90° -0) . So, 13 = ~sin2 (20)
Let 10 be the intensity of the unpolarised light falling on P1 • 13 will be maximum when sin 2 (20) = 1 (maximum), i.e.,
Then the intensity of light emerging from P1 will be sin(28) = 1 = sin(90°)
I, ::::> 0=45°
l1=-
2
By Malus Law, the intensity of light emerging from P2 is given
by

================================:::i 2.91 =
www.puucho.com
Rahul Sardana with www.puucho.com

1. [BJ
2
lmax=I=(a+a)
:::::> 1=4a2 =4l 0
lU·C 47x3x10 9
When either of the two slits is covered then => vs =-i.-= 4700
I'=(a+o)' =•' =¼ => v& =3x106 ms~1 away from earth

2. ~ ~ [BJ
Let nlh m1mma Ot 400 nm coincides with mth minima of The relation among angle of diffraction 8 , order n and number
560 nm, then of lines per cm of the grating N is sine= NnA.. The maximum
4 0 5 0
(2n- 1i( ~ ) =(2m-1)( : ) value of sine= 1. Hence maximum value for n = N~ .

2n-1 7 14 1.=6000x10_,, cm, N=5000 lines/cm, we get n=3.33. The


=> --=-=-
2m-1 5 10 order of spectrum is an integer. Thus we cannot see the fourth
i.e., 41h minima of 400 nm coincides with 3rd minima of 560 nm. order, but can see the third order.
Location of this minima is,
y (2x4-1)(1000)(400x10_.) 10. [AJ
1 2x0.4 14 mm Since µ).. =·constant
Next 11 th minima of 400 nm will coincide with B" minima of => µ1)..1 =µ2)..2
560 nm => (1)1., =ni.,
Location of this minima is,
y _(2x11-1)(1000)(400x10_.) => ~=n
2 42mm i.,
- 2x0.1
Required distance =Y2 - Y1 =28 mm 11. [DJ
n,A1 =n2A2
3. [AJ
Path difference due to slab should be integral multiple of A or =) 62 x5893 = n2 x4358
,Ix= ni. . =) n2 =84
=> (µ-1)t=n1. n=1,2,3•..._..
12. [DJ
=> t=~
µ-1 p = ).D
d
For minimum value of t, n =1
P= (5ooox10-")(2)
=> t = -2:._ =_i._ = 2). =>
µ-1 1.5-1 (10"')
=) P=10--3 m=1 mm
4. [DJ
As AR>AG>'-e 13. [AJ
Let the required thickness be t A. So, number of wavelengths,in
:::::> PR >J3o >Pa as j3ocA
vacuum is _t_.
6000

= =============:;:=====================
2.92

www.puucho.com
Rahul Sardana with www.puucho.com

Wave Optics
=:, Number of wavelengths in air is Einstein's photoelectric equation appears to be interlinking of the
1.0031 two aspects.
6000
According to the problem 26. [AJ
This is the effect of refraction. Rays from the sun are passing
_t_= 1.0031 +1 from vacuum to air when they enter·into the earth's atmosphere.
6000 6000 They bend towards the normal making the sun to appear at a
=:, t=2mm higher altitude as shown in figure.

14. [CJ
I-,

lm; 0
=(A +F,)' =91
=(F,-F,)' =[
N

1./
,
....
Image
Sun

15. [DJ
Theoretically infinite colours are possible, hence infinite
wavelengths.

17. [DJ N'


Because sound waves are longitudinal in nature.
28. [DJ
18. [DJ In the presence of thin glass plate, the fringe pattern shifts, but
The blue filter will allow only blue light to pass through. So light no change in fringe width.
from the object passes through filter. Similarly, the whi_te
background will also look blue through the filter. Thus we have a 29. [CJ
blue object under a blue background, which makes it If an unpolarised light is converted into plane polarised light by
indistinguishable. passing through a polaroid, it's intensity becomes half.

19. [BJ 30. [BJ


Intensity of illumination at a point is decided by three factors; (i)
power of the source P (ii) distance from the source r and (iii)
angle of incidence of rays cos e
0
=:, I= Pcos 2
r
At noon the sun's rays are normally incident making e= 0 .
Hence I is maXiJ'l'.IUm at noon.
A
20. [BJ
Distance of third maxima from central maxima is
3l.D 3 x 5000 x 10-" x (200 x 10-')
1.5 cm B
X d 0.2x10-s PR=d
PO~dsecB
22. [CJ
and CO= POcot20 = dsec0cos20
For observation of colours, the thickness of the film should be of
the order of the wavelength of visible light. path difference between the two rays is,
6x = CO+ PO= (dsec0+ dsec0cos20)
23. [DJ phase difference between the two rays is
The red filter will pass only red light. Hence a blue cross will Li~= 1t (one is reflected, while another is direct)
appear as black. The white background will appear as red when Therefore, condition for constructive interference should be
seen through a red filter. Thus the observer will see a black
cross under a red background.
i'.x-!: 3l.
- 2' 2

24. [BJ l.
=> dsec0(1+ cos20) =
Newton's concept of light is that it is made of corpuscles or 2
particles. All particles are deflected by earth's gravitational field.
Hence they are also deflected by gravitational field. In the => ( _d_)(2cos'
case
B) =!:
2
general" theory of relativity. Einstein predicts deflection of light by
the gravitational field. The angle of deflection predicted by both => case=_!:_
are however not the same. Einstein's prediction agrees with 4d
experimental results.
31. [AJ
25. [DJ l.D
In photoelectric effect, the photon is treated as a particle having ~ = - and
d
a quantum of energy hv. It emits one electron. Here the particle
and wave aspect of light appear in two sides of equation. P' ~ l.{D/2) =.!!_
2d 4

2.93 =
www.puucho.com
Rahul Sardana with www.puucho.com

Advanced JEE Physics Optics & Modern Physics

32. [DJ 36. [CJ

~='° Intensity at the centre will be zero if path difference =~


d
d is halved and D is doubled
(µ-1)t=!:
So, Fringe width p will become four times. 2

1=-'-
33. [AJ 2(µ-1)
Path difference at s 2 is 2/,,. . Therefore for minimum intensity at
p 37. [AJ
No light,.is emitted from the second polaroid, so P, and P2 are
perpendicular to each other

P,
0
90°-0
P,e-------':.L='-=-"----P,

2l.
Let the initial intensity of light is 10 • So Intensity of light after
Let x be the minimum distance from s2 • Then
transmission from first polaroid =1-.
31. l. 2
s 1P-s 2P=-'F- ... (1)
2 2 Intensity of light emitted from P3
~- X = 3l. 11 = 1-cos2 0
2 2
Solving this equation we get Intensity of light transmitted from last polaroid i.e., from
7l.
X=-
12 P2 = 11 cos2 (90° -0) =~cos2 0sin 2 0
l.
NOTE: If we substitute s,P-s2P= in equation (1) we get 2
P2 =1..(2sin0cos0) =1..sin 2 20
2 8 8
x =15A which is greater than TA .
4 12 38. [CJ

34. [AJ
I= 41 cos'(½)
0

Shift = (µ - 1) ID ~= 2n
d 3
(1.5-1)tD
X= ..• (1) .6.xx(2,.,7')= 231t =,i
d
and !l.x_ (µ- 1)t0 ••. (2) sin0= ~
2 d
Dividing equation (1) by (2)
sine=..?:.
2 0.5 3d
3 µ-1
2µ-2=1.5 39. [BJ
2µ =3.5 I= 1_ cos'(½)
µ = 1.75

35. [AJ
¾=cos'(½)
=> COS$=0
Phase difference corresponding to y, =- " and that for
2
Y2 =+2"
=> Average intensity between y1 and y2

=> I,. =.!n_~1' I= cos' (±)d~


2
=I= (n2n+ 2)
2 41. [AJ
Path difference at P is
So, the required ratio is ,½(1+;)
.6.X = 2(icos8) = xcos0

= 2.94 ==============;;;:====================
www.puucho.com
Rahul Sardana with www.puucho.com

Wave Optics
For intensity to be maximum .
ax= nl. {n=0,1,2,3, ...)
=> XCOS9=nA ••. (2)
p
=> cos a= nA Using (1) and (2), .!t = 410 = 2
X
, , '' l2 2Io
Since, case 'fo 1 ,/ '''
=> nl.t,1 ,/ ' 50. [CJ
.,/ a· :
=>
X

n;t,~
l.
Substituting x = SA , we get
-x-
s,o--~-.. s, l=lo[si:a.r ,where a.=!

2 1
For nth secondary maxima dsin8 = ( n + )A
2
nt,5
=> n=1,2,3,4,5, ..... => a =.P.2 =.'.:.[dsinS]
l.
=(2n+l)•
2
Therefore, in all four quadrants there can be 20 maximas. There
are more maximas at 8 =0° and e = 180° .
. (2n+1)
Sin -- • ]'
But n =s· corresponds to e=90" and 8 =270° which are I =I 0 2 I,
coming only twice .while we have multiplied it follr times. [ (2n/1)n
Therefore, total number of maximas are still 20 i.e., n =1 to 4 in
four quadrants (total 16) plus four more at a= 0°, 90°, 180°
and 270°.

42. [AJ 51. [DJ


Laser is intense, coherent (all photons in phase) and
If I is the final intensity and 10 is the initial intensity then
monochromatic (only single wavelength).
1=1-(cos2 30°)5
43. [DJ 2
The focal length increases with increases of wavelength.
ioc (n-1) nearly cc : .where A is wavelength. In the given => _I_=.!.x(./3)" =0.12
2 10 2 2
set of radiations. X-rays have the minimum wavelength. Hence
focal length is minimum for X-rays. · 52. [BJ
For maxima 11 = d sin 8 = nA
44. [DJ ::::) 2Asin8=n1
As the star accelerates towards the earth, its velocity will
increase. By Doppler effect in light, a source approaching an ::::) sine=%
observer shows violet shift. So the colour of the star will shift
toward~ the shorter wavelength region, that is blue. since value of sine cannot be greater 1.
n=0,1,2
47. [BJ Therefore only five maximas can be obtained on both side of the
6). V screen.
l. C
53. [BJ
In this case, we can assume as if both the source and the
observer are moving towards each other with speed v. Hence
::::) V=1.5x10 5 , c-u c-(-v) c+v
(Since wavelength is decreasing, so star coming closer)
v = - -0 v=---v=--v
c-us c-v c-v

48. [BJ , (c+v)(c-v) c'-v'


v ~-,-~~~v=~-~~-v
2 2 2
Momentum of the electron will increase. So the wavelength (c-v) 0 +v -2vc

(A=~) of electrons will decrease· and frinQe width decreases c' C


Since v « c , therefore v' = - , - - =- - v
c -2vc c-2v
as ~a:l..
54. [CJ
49. [AJ Fringe width p oc A. • Therefore, A and hence p decreases 1.5
times when immersed in liquid. The distance between central
1=41,cos'(!) 111
maxima and 10 maxima is 3 cm in vacuum. When immersed in
At central position 11 = 410 .:.(1) liquid it will reduce to 2 cm . Position of central maxima will not
111
Since the phase difference between two successive fringes is change while 10 maxima will be obtained at y = 4 cm .
2x , the phase difference between two points separated by a
distance equal to one quarter of the distance between the two, 55. [BJ
For maximum intensity on the screen
successive fringes is equal to 6 = (2n>(¾) =% radian dsin8 =nA

2.95 =
www.puucho.com
Rahul Sardana with www.puucho.com

Advanced JEE Physics Optics & Modern Physics

sine= n1. = n(2000) =_I!_ the central maximum in the diffraction pattern you add N
d 7000 3.5 phasers, all in the same direction and each with the same
Since maximum value of sin 8 is 1 amplitude. The intensity is therefore N2 • If you double the slit
So n = O , 1, 2, 3 only. Thus only seven maximas can be width, you need 2N phasers, if they are each to have the
amplitude of the each to have the amplitude of the phasers you
obtained on both sides of the screen.
used for the narrow slit. The intensity at the central maximum is
56. [DJ proportional to (2N>2" and is, therefore, four times the Intensity
If d sin 8 = (µ -1)t , central fringe is obtained at 0 for the narrow slit.

If d sin 8 > (µ -1)t , central fringe is obtained above O and 64. [BJ
If d sin 8 < (µ -1) t , central fringe is obtained below 0
sin C = ¾. Hence n= %. If i is the polarising angle tan i = n .

57. [BJ
Resultant intensity I = 11 + 12 + 2Jrj; cos qi
i=tan-1 (n)=tan-1 (%).
At central position with coherent source and 11 = 12 = 10 -
65. [CJ
Icon = 410 ... (1)
In case of incoherent at a given point, lj, varies randomly with Reflected
Incident
time so (cos$)~=0 ray ray
I,nco11 = l 1 + 12 =210 ... (2)

Hence 1coh = _g_


llncoh 1

58. [AJ
According to given condition
(µ-1)t = nA. for minimum t, n =1
So, (µ~1)tm;, =1.
Refracted
1. 1. ray
t, =--=--=21.
ma µ-1 1.5-1
For glass of refractive index 1.5, polarising angle is 57°,
tan i = 1.5 . At this stage the angle between the refracted ray and
the reflected ray is 90° . The required angle from figure is
8=57-33=24°.

66. [DJ
The property of double refraction is shown by quartz, calcite and
ice also.
60. [AJ
When unpolarised light is made incident at polarising angle, the
reflected light is plane polarised in a direction perpendicular to 67. [CJ
the plane of incidence. For first minimum
A.=asine
Therefore E in reflected light will vibrate in vertical plane with
respect to plane of incidence. => 6.5 x 10-1 = a sin 30
=> a =13x10-1 m
61. [DJ => a=1.3µm
If I is the intensity of the incident unpolarised light, the intensity

transmitted by the firs~ is ½. This is the intensity of incident light 68. [BJ

on the second polaroid. Intensity transmitted by the second The resolving power of the human eye is is the smallest
dB
2
polaroid is ( ½)cos 8, where 8 is the angle between the axes. angle subtended by two objects just distinguishable,

d8 = 1' = ~0 .,Converting this into radian and taking 1 radian as


Here sine=¾, case is therefore¾. 6
nearly 60° (Actually 57° 18') we get the resolving power as
2
2
..!.cas e=..!.x(i) =~I 3600.
2 2 5 25
8 70. [CJ
is the required ratio.
25

63. [DJ ~ x
If you divide the original slit into N strips and represents the light
D
from each strip, when it reaches the screen, by a phaser, then at

= 2.96

www.puucho.com
Rahul Sardana with www.puucho.com

Wave Optics
If x is the distance between poles and D is the distance of the => llx:=1.5 mm
eye from the poles as shown in figure we have d0 = tand9 = ~.
D 75. [CJ
Here D = 3.6 km= 3600 m and d0 = ~ radian. Hence Shift, ilx=[(µ-1)t]~
36 0
X=1m. tD
=> 20~=(µ-1)-
d
71. [AJ
1
M V => 20().~)=(µ-1) ~
). C
=> 201.=(µ-1)1
=> 1=~
C => µ= 20). +1
=> V=C t

=> µ
20 X 5QQQ X 1 o- 1
0

72. +1
[CJ 2.5x10-5

).' = 1.(1-f) = 5890(1


45
· xio')
3x10 8
= 5802 A =>
4
µ=-+1
10
=> µ=1.4
73. [CJ
i., _ 4500 _ 3 76. [BJ
i., - 6000 -4 For dark fringe at P
::::) 4A.1 = 3A.2
i.e. the fourth bright fringe of A.1 coincides with the third bright
Here n = 3 and A =6000
fringe of A. 2 • 5). 6000 .
So, 11= =5x--=15000A=1.5m1cron
2 2
74. [CJ
From here, we must take a note that central maxima ( n = O) is 77. [AJ
flanked by first minima (n = 1) and then first maxima (n = 1).
Fringe width ro = A~ -~ When the apparatus is immersed in a
Position of the ( n + 1) th dark fringe is
1-D liquid, A and hence w is reduced µ (refractive index) times.
x,., =(2n+1) d
2 10oo'=(5.5)oo
So, position of second dark fringe is
=> 101.·(%) =(5.5l~
x, =[2(1)+1]~~
1. 10
=> 'i:= 5.5 =µ
=>
=> µ=1.8

78. [BJ
Second Maxima {n = 2) The intensity of light received is inversely proportional to the
=~----i
""------< = 1) square of distance. Hence the amplltude of the light wave
._ ____ ___,
Second Minima (n
First Maxima (n = 1)
received is inversely proportional to the distance.

~v,· First Minima (n =0) 79. [CJ


l"'""''-------1
Central Maxima (n = O) Fringe width ~ oc A. Therefore, A and hence w will decrease
'===~----1
a@ft- First Minima (n = 0) 1.5 times when immersed in liquid. The distance between central
i=="--------1 th
maxima and 10 maxima is 3 cm in vacuum. When immersed in
First Maxima (n = 1)
=~~---, liquid it will reduce to 2 cm. Position of central maxima will not
change while 10 maxima will be obtained at y = 4 C1"!1 .
th
"-------< Second Minima (n = 1)
._ _____
_, Second Maxima (n = 2)
80. [AJ

Similarly, pbsition of nth bright fringe is I=4I,cos'(!)

x0 =n(i.~) 10 =4l0 cos


2
(¾)
4W
=> X4=d ::::) cos(!)=¾
So, separation is
::::) !=2:
ilx=x,-x,=%(1.~) 2 3

2.97

www.puucho.com
Rahul Sardana with www.puucho.com

Advanced JEE Physics Optics & Modern Physics

16A.2 -4l2
=> ~=23•=(2;)AX x, - 41- 3).

16A.2 -9l2 7 ,._


=> .!=(-!}i
6). 6
3 I- D X3-

A 6x10-7 X4 =0
=> y= ;----er
x-
=3 X 1 Q-4
So, Number of points for maxima becomes 3.
D
=> y=2x10-a m=2 mm 87. [BJ
If a is the width of the slit, the minimum of Fraunhofer diffraction
81. [BJ pattern appears for asin8 = nA, where n is an integer. For the
For third maximum
first minimum n=1.Then sin8=~=10-.'l rad; 1rad=57°.
a
X=n(A~) Hence 8 = 0.057° = 0.057 x 60' = 3.42' .
X=( 3 )5000x10·"x2
0.2x10-3 88. [BJ
Angular limit of resolution of human eye is
X=1.5cm
- = R( say )
1.221-
-
82. [BJ a
7
n,A.1 =nzA 2 R 1.22x5x10~ rad
2x10-3
1-, 600
=> n2 =n1 x-=12x-=18 R 1.22x5x10"' 180 d
1-, 400 x --3 x egree
2 10 7
83. [DJ R = 0.0175 degree
Blue of the sky is explained by scattering. Scattered light obeys R=0.0175x60 minute
Rayleigh's law. I oc A~ . Hence shorter wavelengths are R=1 minute

scattered with more intensity. In visible light, violet and blue have 89. [CJ
the minimum wavelength. Of the two, blue is mare intense.
I~=( ~I,/1, +1J 9
84. [CJ I"" ~I,/1, -1 =1
The interterence fringes for two slits are hyperbolic.
x+1=
=>
x-1
3 {·: X=~I,/1,}
85. [AJ
Total phase difference => X=2
= Initial phase difference+ Phase difference due to path ·
360° 360' I- - => .!i. = 4
::::::, ~= 66°+-A-xruc =66°+-A-x = 66°+90=156° I,
4
=> 11 = 412
86. [BJ i.e., if 12 = 10 then 11 = 41 0
From .6.S1S 2O,

(S,D)' =(S,S,)' +(S,D)' 10 = 410 cos


2
(½)
(S,P+PD)' =(S,S,)' +(S,D)' => ~= 2•
3
y
=> (2;)(~)= ~
AD
=> Y=-
3d

90. [AJ
(n+1)5200 =n(7BOO)
X
=> n =2
92. [BJ
Here S 1P is the path difference =nA. for maximum intensity. Air has a refra"ctive index slightly greater than one. So, when the
chamber is evacuated, the refractive Index decreases and hence
(nl-+x,)' =(41-)' +(x,)' wavelength increases as a result of which fringe width increases
slightly.

93. [DJ
16A.2-A2 Ultraviolet radiations, when fall on a fluorescent material, are
2). 7.51. converted into visible radiations. Hence we should use ultraviolet
radiation.

= 2.98 r::::==================================
www.puucho.com
Rahul Sardana with www.puucho.com

Wave Optics
94. [CJ

i ~
Using the same argument as in the SPLUTION 104.
I, 1331
-cos 30=-x-=-=0.3751 s, -------- , / /
2 2 4 8

1.c,--+1
95. [BJ d ,,
ARed > /.Violet
and all other colours have wavelength value lying in this region.
Further, we have X cc A , so Screen
X(Blue) < X(Green)

96. [DJ Hence corresponding phase difference $ = 21t x!: = ~


I 91 , 4 2
From figure 11 = - and 12 = - Resultant intensity at P
4 64

=> _!g_=~ I= I~ cos


2
½= $ cos ¾J =~
0
2
(
1, 16

101. [A)
If i is the polarising angle, n = tani (Brewester's law).

n = tan67° is greater than 2. We also know 1


- - = n.
sine
sine=.!, where e is the critical angle, sine is less than 0.5.
n
Hence C is less than 30° . The only possible answer is 22° .

By using -""- =
I
lmin
rx;_+,l = [ J¼
VI,
Jf'
[ ~- 1
11
Vtti+
/9
16
~-1
16
l
1
49
=-
1
102. [DJ
Only transverse waves can be polarised, while the other
properties are common for all wave motion.

103. [DJ
In the arrangement shown, the unpolarised light is incident at
polarising angle of 90° -33° = 57° . The reflected light is thus
plane polarised light. When plane polarised light is passed
97. [DJ
through Nicol prism (a polariser or analyser), the intensity
Since, according to Malus Law
gradually reduces to zero and finally increases.
I= l 0 cos 2 ~
104. [DJ
98. [DJ If shift is equivalent to n fringes then
Since P is ahead of Q by 90° and path difference between P

and Q is ¼. Therefore at A , phase difference is zero, so


n=-- ,
(µ-1)t

=> noct
intensity is 41 . At C it is zero and at B , the phase difference is _!g__= n2
=>
90° , so intensity is 21 . 11 n1

99. [AJ
P is the position of 111h bright fringe from Q. From central
position O, P will be the position of 10th bright fringe. 20
t2 = x4.8=3.2mm
Path difference between the waves reaching at 30
P = S 18 = 10A =10x 6000x10-10 = 6x10--o m
105. [CJ
Path difference between the waves reaching at P, A = .6.1 + .6.2
100. [AJ
Suppose P is a point infront of one slit at which intensity is to be where .6. 1 = Initial path difference
d
calculated from figure it is clear that x = . Path difference
2
between the waves reaching at P

,\=xdJ~)ct =_i_=S,=_1:
D10d20204

Screen
-0,--,--0-
2.99 =
www.puucho.com
Rahul Sardana with www.puucho.com

Advanced JEE Physics Optics & Modem Physics

~=Path difference between the waves after emerging from'slits.

6, =SS, -SS, =~D2 +d' -D


and 6 2 =S,0-S,O=~D'+d' -D 113. [CJ
Interference is a physical effect.of superposition of wave motion.
2 This can happen for any kind of wave motion, i.e., light waves,
=> 6=2((o'+d )Lol=2{(0'+:~)-o} sound waves, matter waves, etc.
d' {From Binomial expansion} 114. [BJ
6=-
D The extra path difference produced by the glass plate of
thickness t and refractive index µ is ( µ -1) t . Due to this if n is
For obtaining dark at 0, 6 must be equals to (2n -1)~ i.e.,
the number of fringes which shift, (µ-1)t=n1..
d' ).
-=(2n-1)- t=O.O1x1O...:a m, A.=6OOOx10- 10
m, µ=1.5. This gives a
D 2
value for n = 8 .
=> d=/2n-21)1.D
117. [DJ
For minimum distance n =1 so d = ~ I= a~ + a! + 2a 1a 2 cos4>
Put a~ +a! =A and a 1a2 =B
106. [DJ ~ l=A+Bcos4>
For destructive interterence, path difference must be an odd
118. [AJ
multiple of ~.
l!J..=A.'!.. and v=rro
C
107. [CJ 2
These concentric bright and dark fringes are called Newton's V=7x1O 6 x rc , C=3x1O8 ms-1
25x24x3600
Rings.
=> 61. =o.o4 A
108. [BJ
I= 10 + 10 + 2l 0 cos(21t) 119. [CJ
=> 1=410 Shift =.!!.(µ-1)t
).
A. 21t 1t
For X=-. ~=-X=-
4 ). 2 => 7p =.!!.(µ-1)t
).
I'=l0 +10 +0

1'=210 =.!.2
120. [BJ
109. [AJ
. P=w
d
=> J3ocD
131 D,
132 = D2
P, -P2 = D, -D2
132 D2 Here path difference at a point P on the circle is given by
6x=dcose ... (1)
6P P, 1.,
For maxima at P
ao=o2 =d2
6x=~ ·--~
3x10-5 From equation (1) and (2)
=A.2 = Sx _2 x10--3 =6x10-7 m=6000A
10 nA. = dcos0

110. [CJ ~ 1
Scos- ( n;) = cos-1( ~)

[(n-1)t]~= 1.(2D)
d d 121. [AJ
1.= (n-,lt =5892A The film appears bright when the path difference
=>
2 (2µtcosr) is equal to odd multiple of!:
2
112. [AJ (2n-1)).
i.e., 2µtcosr ~-~ where n=1, 2, 3 .....
).L 2
X=-
d
= 2.100 ================================::i
www.puucho.com
Rahul Sardana with www.puucho.com

Wave Optics

A 4µtcosr 5 X 48QQ X 1o-lO


=> => I=~ 8x10.. m=8 µm
(2n-1) (µ, -µ,) (1.7-1.4)
A, 4x1.4x10,000x10"'10 xcoso 56000 A
=>
(2n-1) . (2n-1) . 126. [BJ
At the polarising angle, the reflected ray is fully polarised while
=> i. = 56000 A, 18666 A, 0000 A, 6222 A, 5091 A,
the transmitted ray is partially polarised. In fact a method to
4308 A, 3733 A. produce plane polarised light is by reflection at the polarising
The wavelength which are not within specified range are to be angle.
refracted.
128. [BJ
122. (AJ ).
a=-
M V B
C 6500 x10-10
=> a
(401.8-393.3)
=>
393.3
V
3x1Q8 (i)
=> v =6.48 x 106 ms-1 =6480 kms-1 :::, a=1.24x10~ m
=> a=1.24µm
123. [DJ
Let nth minima of 400 nm coincides with mth minima of 130. [AJ
560 nm then Using Mal us Law, I= 10 cos 2 8
(2n -1)400 = (2m -1)560
As here polariser is rotating i.e., all the values of 8 are possible.
=> 12:rr 1 2.
Iav =-Jid8=-= J10 cos 2 8d8
21to 21t o
i.e., 4th minima of 400. nm coincides with 3rd minima of
560nm On integration we get Iav =~
The location of this minima is
7(1000)(400x10.. ) where I Energy p 10"' 10watt
- 14 mm 0
3x10-4 -3rfi2
2x0.1 Area x Time A
Next, 11th minima of 400 nm will coincide with 8th minima of
I =.!x.!Q=E, Watt
560nm ~ av 2 3 3
Location of this minima is
21(1000)(400 X
2x0.1
10. )
42 mm
. 2n: 2x3.14 1
andTimepenod T=-=---=- sec
ro 31.4 5
So, energy of light passing through the polariser per revolution
Required distance =28 mm
=I xAreaxT=~x3x10"""x..:!.=10--4J
~ 3 5
124. [AJ
Shift rue= t (µ-1)1 131. [CJ

11+-- o, ___.,. Screen


Path difference = 2dsin8
For constructive interference
w B'e
'

2dsinB = ni.

'(n:>.)
Shift due to one plate rue, = t (µ;-1)
=> .
B =Sin- 2d

Shift due to another path rue, =f(µ, -1)t 133. [DJ


During sunset and sunrise we see the unscattered part of the
sunlight. Sun's rays have to travel long distance and hence are
Nets~ift rue=rue,-rue,=.!1.(µ,-µ,)~ ... (1) .
). . incident obliquely. All shorter wavelengths are scattered by
Also it is given that rue = 5P ... (2) Rayleigh's law for scattered intensity I a: h.~ . Only the red-

Hence 5P = t(µ, -µ,)t orange rays reach us. During noon the sun's rays are normally
incident on the earth. Hence a portion of both the shorter

i:==================================2.101 =
www.puucho.com
Rahul Sardana with www.puucho.com

Advanced JEE Physics Optics & Modern Physics


wavelengths and longer wavelengths reach us. The sun looks 139. [DJ
white. The action of a nicol prism is based on double refraction and
dichroism. When a ray of light enters into a calcite crystal, it is
134. [DJ split into two rays O and E. Both are plane polarised in mutually
perpendicular planes. One of them suffers total reflection and
P='°
d absorption whil_e the other comes out as plane polarised.
Selective absorption is called dichroism.
So, angular width, .20 is
29=2p=21 140. [BJ
D d
Shift, ,1x =[(µ-1)t]%
6328x10-10
9
- 0.2x10"" Here ,1x = sp
0 = 3164x10-a radian
=> s(1"~)=[(µ-1)t]%
180
=> e = 3164x10-e x degree

9 = 0.18 degree
• => 51"=(µ-1)t
=>
=> 29 = 0.36 degree => 1"=(µ-1)½

1"= (0.5)(6x10"')
135. [AJ =>
5
For maxima 2,n = ~"(X0)-27tl ~ A.=6x10-7 m

2 => 1"=6000 A
=> ;cxo)=2a(n+I)
141. [BJ
=> (XO)=1"(n+l)
Only transverse waves can be polarised. (A) and (C) and (D) are
electromagnetic radiations, which are transverse waves while
136. [CJ (B), that is, p rays are electrons.
From the given data, note that the fringe width · (P,) for
i, = 900 nm is greater than fringe width (P,) for 146. [BJ
A. 2 =: 750 nm . This means that .at though the central maxima of P='°
d
the two coincide, but first maximum for l 1 = 900 nm will be
s M1.2)
further away from the first maxima for A.2 =750 nm and so on. =>
1000 =( 0.1)
A stage may come when this mismatch equals (32 , then again 1000
maxima of A1 = ~00 nm , will coincide with a maxima of => 1" = 5000 A
A. 2 =750 111
nm , let this correspond to n order fringe for A.1 • Then it 150. [CJ
will correspond to (n + 1t order fringe far A. 2 •
Using the equation p = ~). , p ~A. Of the given colours, yellow
Therefore n~D - (n + ~J.. 2D has the maximum wavelength and hence the maximum fringe
width.
=> n x 900 x 10-<> = (n + 1)750 x 10·'
::::) n=S 152. [BJ
Minimum distance from The tubelight has coating of fluorescent material which converts
9 ultraviolet light into visible light. Hence the light is more intense.
C t I . nA.p 5x900x10- x2
en ra maxima = -d- = x a-a
2 1 153. [BJ
= 45 x 10-4m = 4.5mm We take one wavelength in the visible region and calculate the
energy just to know its order. Let us take 6000 A (red) for
137. [CJ
One of the essential condition to observe diffraction is that the
convenience of a rough calculation. Energy E = ~. Taking
dimension of the object should be of the order of wavelength of
the wave. The velocity of sound in air as nearly 350 ms-1 • h=6.6x10---3-4, C=3x108 , A=6000x10-10 m we get :c in
Hence the wavelength of sound waves produced by a tuning fork
joules. Dividing this by 1.6x10-19 to convert to eV, we get
of frequency 384 Hz is A= f = 1 m near1y. Hence the diameter E = 2 eV. For energy 1 eV, wavelength will be 12000 A.
of the sphere should be of the same order. Energy 5 eV means wavelength f of 6000, that is 2400 A.
138. [DJ Both these ar8 invisible, recalling that the visible range is
Distance between the first dark .fringes is 4000 to 1000 A .
21D
Ax=d=2.4mm

=2.102c:===================================

www.puucho.com
Rahul Sardana with www.puucho.com

Wave Optics
154. [A]. 167. [BJ
The energy levels of atoms are well defined. Hence transition
between them should give a well defined frequency, which Separation n111 bright fringe and central maxima is xn =n~D
means a single frequency, and hence a single line.
3 X 6000 X 1o-lO X 1
3.5mm.
155. [CJ 0.5x10-3
The torch produces light waves which are non-coherent in
nature. 168. [CJ
lmax =(81+82) 2=9
156. [DJ
For unpolarised light 1nm, a a 1- 2

I'= .!P..cos 2
~ 169. [BJ
2
For second dark fringe, n = 1
I ' =2COS
I '(n)
4
Since, x = (2n+1)~~
=> I'=.!.
4 => 3 = 3W ... (1)
2d
158. [CJ For fourth bright fringe, n =4. Since
The path difference of two waves producing destructive
X=n(l.~)
interference or darkness is (2n + 1)( ½) where n is an integer 0,
::, X=4(l.~) .•. (2)
1, 2, 3, etc. A path difference of A corresponds to a phase

difference of 2v. rad and a path difference of ½corresponds to =>


X
3=(3/2)
4

a phase difference of 1t. Hence the corresponding phase => X=B mm


difference is (2n + 1)x .
170. [A]
159. [BJ '. For same field of view

P'=l<p n1P1 = n2P2


µ
=> n,(7)=n,(l.~D)
160. [A]
=:> n1A1 = n2A2
11 a~ 9
½=a~=4 => 60(4000) =n,(6000)
=:> n2 = 40
=>
171. [BJ
S.ince -
I== (a,+a,)'
- - = (3+2)'
-- P'=£=1mm
, Im;n a1 -az 3-2 µ

=> 172. [CJ


5-=~
161. (A] a,

Using the equation ~ = ~l. ; p « l. =:> lmax


lmin
=(a,+a2)'
81 - 82

164. [BJ =>


In a biprism, the distance between sources is due to deviation
produced by two small angled prisms. The deviation produced
by each prism is (µ-1)o:. The total deviation is 2(µ-1)o:. 173. [DJ
Separation of sources 2(µ-1)aa. I= =(Jf; +,/I;)'
166. [A] I.., =(Jf;-,/1;)'
For contrast to be good 11 = 12 and hence
Shift = i(µ-1) t ( p ")(1.5)x2x10 .. =2P
l. 5000x10- Imax =411
i.e., 2 fringes upwards.
Im1n =0

r========================================== 2.103 ~

www.puucho.com
Rahul Sardana with www.puucho.com

1. [A, C]
For, n=0, sin8=¾
The intensity of light is !(9) = 10 cos'(%)
81
__:_--7lp
n=±1,·sin8=-!,+!
8 8
where ii=~"(,i,c)=(~)cdsin9)
. •3 5
(i) For 9 = 30'
n=±2_, s1n8=-a· 8
C 3x1Q 8 ri=±3, sin8=-~.?...
l.=;,=10'=300m and d=150m,weget 8 8 s,
n=4, sin8=-~
0=(~)(150)(1)=-" 8
300 2 2
ii • 6. [A, B, Cl
=> 2=4 Path difference at O is d

=> I(8)=1 0 cos


2
(~)= ~ {option (A)}
if d = A, 2~, 3A, .... ., then O is a maxima.
3" 5 '-
"
1,fd
="2' 2' 2' .... th en 01s a·m1mma
··
(ii) For 9 = 90'
The path difference on screen is > 0
ii= ( :• )<150)(1) =.
00 For d = 3A , we have

i=.': and 1(0)=0 0;5 l.


=>
2 2 1.5 l.
(iii) For 9=0', ii=O or i=o 2.5 l.
2
0 - - - - + - - - - - - - t , i ) ( = 3l.
!(9) =10 {option (C)}
2.5 l.
3. [A, DJ 1.51.
The phase difference corresponding to 0S1 -0S2 = ¼iS i 0.5 l.
Hence, the net phase difference is given by
2
8. [B, DJ
~=-"- "(2l.sin9)
2 l. I= (,µ, +,fi;)' g·
=> if,=2:-41tsin8
I,,., =(,µ,-,[I;)'
2
For maxima, we have 4' = n1t , where n = o, ± 1, ± 2 ... => ,ff; +F, =3(,Jf; -F,)
=> 2:-4nsin8=n1t
=> 2,µ, =4,fi;
2 => 11 =41 2
1
=>
--n
2-
sln0=-
4
Since, I =(A)'
....1.
12
---1.
A2

=2.104==================================
www.puucho.com
Rahul Sardana with www.puucho.com

Wave Optics

,/o'+4d' -O=~
2
. 4d 2
10. [A, C, DJ gives, A=D
I1 =1 2 =I
C

1_ +F,)' =(.JI +J½J


=(,if, <41
45°
B +----,-~-•A

I"'=(./J-J½J >0
11. [A,CJ Intensity is due to C only
Path difference = ..Jo 2
+d 2
- D = 1 cm =lo

Also, UD 2 +d 2 -D)=(2n-1)½ 14. [A, CJ


x = Path Difference = ..Jb2 + d2 - d
=> l.=~
2n-1
=> X=d(n-1)
2 2
=> A=2cm, cm, cm ........ .
3 5
=> X=d(1+£-1)
2d'
12. [A, B, CJ
00
'' { b'
Since d2 « 1, Hence +d2 =1+ 2b'}
v1rt1 d
2

y ''
b'
~ X=-
3 l. 2d
(-D, 3 l.) 2.5 l. For wavelengths to be missing we must find positions of minima.

'''' 1.5 l.

''
•----------+------+x
0.5 l.
!s, \" d
....·· P
(-0, 0)
--0.5 l. !s, _;.:,...···~

-d-
-1.5l.
-2.5 l. J
--3 l.

'' At minima path difference is an odd multiple of ½


co'' b' l.
.rue at co on y-axis is => X= d =(2n+1) ; n =0, 1, 2, 3, ..... .
3). and -3A. there will be 6 minima and 2 2
Smaxima
b'
y => l. (2n+1)d;n=0,1,2, ............ .
b2 b2 b2
l.=d. 3d' 5d'"""""·"·
(-0, --3 l.) ..........
15. [A, BJ

Ax= 3l.
(-0, 0) o-----+-
~-------,
2.5l. 1.5l. 0.5l.
No. of maxima = 3
Ax=0
P= l.D
d
For increasing J3 , d must be decreased and A must be
increased (i.e. frequency must be decreased).
13. [B, DJ

================================== 2.105 =
www.puucho.com
Rahul Sardana with www.puucho.com

2. [DI
1 5. [DI
Fringe width p= ~ shall remain the Same as the waves travel Ax=d=nA, for n=1, d=A and here we will have three
in air only, after passing through the thin transparent sheet. Due maxima.
to .introduction of thin sheet, only. path difference of the wave is
changed due to .which there is shift of position of fringes only, 9. [DI
Statement-1 is false but Statement-2 is tn.ie.
1
which is given as .rue D(n; )t , where n is refractive index of
14. (DI
thin sheet arid t is its thickness. Statement-1 is false, Statement-2 is true.

3. [DI 15. [A]


If both the slits are illuminated by two bulbs of same power, no Statement-1 Is true & Statement-2 Is true and Statement-2 .is
interference pattern will be observed on the screen. This- is correct explanation. of the Statement-1.
because waves reaching at any point on the screen do not have
a constant phaSe difference. as phase difference from two non
coherent sources changes randomly. Therefore, maxima and
minima would also change their positions randomly and in quick
succession. This will result in gen·eral illumination of .the screen.
1

www.puucho.com
Rahul Sardana with www.puucho.com

1. [AJ
It will be shifted upward

2. [DJ
10. [BJ
. .
Cannot be predicted without knowing the thickness of thin plate.
Power received at 8,
3. [BJ P8 =.!_CJ_n(0.002)' =4x10" W
(µ, -µ,)t =rue= o.3t n

11. [CJ
4. [AJ
Power transmitted through A,
l.
(µ,-µ,)l=rue=2 p~ =~xrn-'W=1o-«,i,v
100 ·
=> t=-l.-=J:...
2(0.3) 0.6 12. [DJ
Power transmitted through B,
5. [A]
P~ =_:!_Q_x4x10-o W=4x10-6 W
3 ,-~ ,/3 100
-I0 =I0 COS - = -
4 2 2
13. [CJ
~="' 3
Path difference,
A=(µ-1)t=(1.5-1)x2000x10·" m
=> rue="~,.=~
2n 6 .6.=10-7 m
;. ;_ 5000 A
(µ, ~ µ,)t = 6 t = (6l(o_3 ) = ----:i:a- = 2777.7 A 14. [CJ

· 6. [A] .
Phase difference, qi=
2
t .6.
2n: X 10-7
6000x10-10
Optical Path Difference (OPD) =2n,t
=> qi=~ radian
3
7. [CJ
For destructive interference, we have 15. [DJ
l.
{·: n, < n,) We know that, P cc a2 or P =ka 2 , where k is any arbitrary
2n1tmln =2 positive constant.
550
=> tmln =- - =99.64 nm
4_x1.38
Now, aA =~1~---e 'aa =~4x~o-e

Resultant aniplitude,
a. [AJ 8., = ~ra'""!
-+-~'""+_2a_A_a._c_O_S_~
2n,t = 3l. Substituting values and simplifying,
2
7x10..a
t = 3tm1n =298.9 nm
2
::::) ar=-k-

Resultantpower, P, =k~ =7x.10--6 W =7 µW


9. [AJ
Power received at A,
====================================2.101=

www.puucho.com
Rahul Sardana with www.puucho.com

Advanced JEE Physics Optics & Modem Physics

16. [DJ d2 nyd


Anywhere on screen because there is no relation between 8 , => -+-=nA. ... (1)
D 2D
µ.
Also, ~ + ny:; 1 =(n+1)1. ... (2)
17. [AJ
Total path difference is given by (2)-(1) we get !P=1-
dX = (µ-1)1-dsine
For central bright fringe to be obtained at O , we have 2D
=> P=-1-
dX=O d
=> (µ-1)t=dsine
24. [BJ
In reflected light system, for constructive interterence, we have
18. [BJ
&~=(2n-1h,where n=5 2µt = (2n - 1).1:
2
For minimum thickness, we have
19. [BJ n =1
Fringe width P=~ =4x105 nm ;,_
=> 2µ1=2

20. [BJ => t = 900


4x1.5
Since, filC=dsin9-(µ-1)t, so if 9 increases then dsin9 will
=> 1=150nm
increase and hence b.x increases, so that the central maxima
will shift upward.
25. [AJ
In transmitted light system, for destructive interference, we have
21. [BJ
Net path difference is given by 2_µ1 = (2n - 1).1:.
2
dsin91 + dsin82 =½ {for minimum d } Again, for minimum thickness, we have
n =1
900
I Is, D>>d => I=
4x1.5
=150 nm

26. [CJ
__ J~e,,._d'--=-1-..-..=-----=io
I Is, Path difference between the waves reaching D is
AX =3nR _ nR = 2nR = nA
2 2 2
-0-----20---- Since, rR =11 +12 +2$j; cos~

=> drn)+d(!) =½<sine~ lane) => In=(,µ,+.µ,)' when ~=2n•, n=0,1,2,3, ....
Since the wave generated is divided equally in two parts, so we
3 d' 1- have
=> 20=2

=> d=n
22. [CJ

• For central maxima, we have


dsin01 +dsin82 =0 27. [AJ
Since &x = xR
=> ~+dL=o For maxima, we have
D 2D
=> Y=-2d &$=(2n)n, where n=O, 1, 2,3, ....
So, the POS:ition of first bright fringe is given by
=> (~)<nR)=(2n)n
d 3
y=-2d+2=-2d
=> ,._ =~= nR
nlT&ll 1
So, the first brightfringe is located ¾d below O max

Amax= nR
=>

23. [DJ 28. [BJ


dsln81 + dsin92 =nA. Similarly, for minima, we have
&$=(2n+1)n,where n=0,1,2,3, ....

= 2.108 ==================================
www.puucho.com
Rahul Sardana with www.puucho.com

WaveOpHcs

=> ( 2; )<nR) =(2n + 1)n yd= -10).


D

=> ). = 2nR => Y=-10().~)=-%x10 4 m


max 2n+1
=> Arna~= 2nR
34. [CJ
29. [DJ At P, we have
Since, d., =2a(µ-1)a ~= i +(µ, -1)t-µ.dsin0
and dwater =2a(:w -1)a => d>C= 10-2x3x10--4 + 1 0A= 10).+ 3A
8x1 4
l:__1
dwator=~
,I~= 2•(10i.+ 3l.) = 20•+ 3,
=> l. 4 2
dalr µ.-1
So, Ip 1t)
= 12 + 12 + 212 cos ( 201t + 3
2
=>
Since cas(201t+ ~n)=cos(3;)=o
=> I, =.!21
=>
Further I™ =½+½+2~(½)(½)cos(oo)
30. [CJ
=> lmax =1+1=21
Since, Pair
2a(µ-1)a 1, ./21 1
=>
DA"
I™ =21= ./2
and Pwaier
2a(µ-µ.)a
35. [CJ
~-1 Path difference is given by
=> Pwatar = ~ = _2_ ~ = dsin~+dsin0-(µ-1)t
P., µ-µ. 3 4
2 3
=> Pwaier=3x1=3mm

31. [CJ
-- LI~--- ~
=>
" w
S"mce, p=d

1x10-a - 1 xA.
0.5x10-3
//1" ------ e ----

=> ).=5x10·' m=5000A For central maxima, ax= o

32. [DJ => sin8=(µ- 1)t_sin$


At O , optical path difference is d
~ = (µ, -1)t-µ.dsin0 (~-1)<0.1)
4 1 => sin8- ;Ox10-a sin(30°) = .!
=> ffi( = Q.5 X Q.41 X 1 Q-3 -3X 3 X 10-4 X2 2
=> 0=30°
=> .1X=5x10..f.l m
Since, A =5000 A , so we observe that 36. [BJ
~ =10). At C, 8=0°,soweget
So, 1011\ bright fringe, will be formed at 0. ~=dsin~-(µ-1)1

33. [DJ => ~=(50x10"")(½)-(¾-1)<0.1)


For central maxima, we have
~=0 => ~ = 0.025 - 0.05 = -0.025 mm
Substituting, AX= n). , we get
where, ~=i+(µ,-1)t-µ.dsin0 n = LIX -0.025 _50
A
500x10-6
=> i+(µ,-1)t-µ.dsin0=0 Hence, at C there will be maxima. Therefore the order of
minima closest to the C are --49.
SubstitutinQ values, we get

====================================2.109 =
www.puucho.com
Rahul Sardana with www.puucho.com

Advanced TEE Physics Optics & Modern Physics

37. [A] => 1t1:=7t


Number of fringes shifted upwards is => 1=1 s

N= (µ-1)1 (¾-1)(0.1) 100 41. [BJ


). 500x10-< Given Y1 =40 cm, D1 =2m=200 cm, D2 =:=10 cm

38. [DJ
Net path difference of the waves reacting at Q , is
Ax=yd+y'd
~
s,
D D'
--------------------
B'----'C

=-
For central maximum, l1x =O

=- y'=--y'
D' =-
D

=- y' = -(~)(o.5sin(,rt))

=- y' = -sin(,rt) mm

=- y' = sin(n+ ,rt)= -sin(,rt)


a
39. [CJ 5
. d
y' = , at point P exactly in front of S1 , so we have Path difference between S81 and SSz is
2 AX: 1 = S81 -8S2

t;x=(yd)J ~) =- Ax,= dsina = (0.8 mm)(¾)


D D'
For maximum intensity, we have path difference to be an even => dX: 1 =0.16 mm ... (1)
Now, let at point R on the screen, central bright fringe is
multiple of ½,so observed (i.e., net path difference =0 ).
). Path difference between SiA and S1R is
Ax= (2n)- = ni.
2 ru<: 2 =S2R-S1R
Substituting the values, we get
~ .1.x 2 =dsin8 •.. (2)
0.5sin(,rt) + 0.25 = 0.5n
Central bright fringe will be observed when net path difference is
=> sin(,rt) = 0.5n-0.25 zero.
0.5 => ID<2 - ID<, = 0
For mir:iimum value of t, we have n =1
=- sin(,t) = 0.5 =- Ax 2 = ru( 1

=- dsin0 = 0.16
=- nt=~
6 =- (0.B)sin0=0.16
0 16
t=.!. =- sin0= · =.!.
=- 6 0.8 5

40. [A] =- tan0= ~


24
For minimum intensity, path difference is an odd multiple of ½, =- sine:::,..!
5
so
dX=(2n+1)½
=- tan8::::, sin8 =Yi.=..:!.
D, 5
D2 10
=- 05
0.5sin(,rt)+0.25= ~ {·: n=1} =- Y2=-=-=2cm
5 5
Therefore, central bright fringe is observed at 2 cm above point
=- sin(,rt) = 0
Q on side CD.

= 2.110 ==================================
www.puucho.com
Rahul Sardana with www.puucho.com

Wave Optics

j Alternate solution for (a) => y= (.&_ - 1)1~d


! ax at R will be zero if Ax:1 =dX:2 µ_
Substituting the values, we get
=> dsina=dsin8
=(~- 1)10.4x10.. (1.5)
=> a=0
Y 4/3 0.45x10"
=> tan a= tan0
=> y=4.33x10-am
11. = _lg_ 13
D1 D2 => y=4.33 mm= mm
3
=> y, = D, ·Y, =(_!Q_)(40) cm 44. , [CJ
D,, 200
=> y2 =2 cm At O,· dl<,=0
So, net path difference
42. [CJ ax= .ruc:2
The central bright fringe will be observed at point Q , if the path
difference created by the liquid slab of thickness t =10 cm or
100 mm is·equal to Ax:1 , so that the net path differe~ce at Q
Corresponding phase difference t
~4' = ( 2 )Ax

Substituting the values, we get


becomes zero.
$=A$=~(~-1)(10.4x10_.)
6x10-' 4/3 .

=> $=(~)n
a
Now, I(~)=I=cos'(f)

s
:::::,. I= !max cos 2 (1!1t)
=> (µ-1)t=dl<, =>
=> (µ-1)(100)=0.16
=> µ-1=0.0016 45. [CJ
=> µ=1.0016 At O , path difference is Ax= Ax 2 = (:: -1}

43. [DJ For maximum intensity at O , we have


Given A=600 nm=6x10-7 m, ~x =nA, where n=1, 2, 3, ......
d=0.45 mm=0.45x10"m and 0=1.5 m A=
dX dX dX
, , , ...... and so on
=>
1 2 3
sl1
dX =(!i:-1)t10.4x10 .. m)

=> dX =(!i:-1)(10.4x10' nm)=1300 nm


s,--s---------------- So, maximum intensity will be corresponding to
1 1300
A=1300 nm - nm, -1300
- - nm, -1300- nm, ...
Thickness of glass sheet, t =10.4 µm =10.4 x 1a.fl m 2 3 4

Refractive index of the medium,. µm = i => i. = 1300 nm , 650 nm, 433.33 nm, 325 nm, ....
The wavelength in the range 400 nm to 700 nm are 650 nm
And refractive index of glass sheet, µg =1.5
Let central maximum is obtained at a distance y below point 0.
and 433.33 nm(=
13
:o nm)

yd 46. [CJ
:::::, Ax 1 =8,P-S 2 P=[)
The optical path difference between the two waves arriving at P
Path difference due to glass sheet is given by is
y,d y,d (1)(10) (5)(10)
Ax,=(~: -1} dl<=-+-=--+---
D1 3
D2 3
10 2x10
Net path difference will be zero, when we have :::::,. Ax=3.5x10- 2 mm=0.035 mm
6.x1 =.1.X2 To calculate the order of interference, we shall calculate

=> yd
D
=(!1-1)1
µ.
n=-
1-
dX

================================:::2.111 =
www.puucho.com
Rahul Sardana with www.puucho.com

Advanced JEE Physics Optics & Modern Physics

=> n ~~00~5xx1~~: ~ Since, m = _f_


f+u
=> n=70 => 100-x = 4
=> 6x =70l. X
So, 70111 order maxima is obtained at P . => X=20 cm
So,weget U=-20, v=+80
47. [DJ
Since.!_.!=!
At O, 6x = y,d =10_,, mm =0.01 mm V U f
D, => f=16 cm
Now, we observe that fix= 20A.
So, 20111 order maxima is obtained at O . 50. [BJ
0
l.D _ 6000x10-" x1 0
48. [DJ 2
'"' d 3x10-a · mm
(µ-1)1 = 0.01 mm
51. [DJ
I =o.o,
- - = 00 . 2 mm=20 µm
1.5-1 P= l.D
Since the pattern has to be shifted upwards, therefore, the film d
must be placed lnfront of S, . · When D is increased, then p is increased.

49. [AJ
Using the concept of Displacement Method, we get
0=$i; =3mm

=2.112 ==================================

www.puucho.com
Rahul Sardana with www.puucho.com

1. A-> (p, s) 6. A--> (r)


B--> (q, r) B--> (!)
C --> (p, q, r) C --> (r)
D--> (q, r, s) D --> (p)
If glass slab is introduced across s, this effective path
2. A-> (p, q) increases so central maxima will be shifted downward but fringe
B--> (s) width remains same.
C--> (p, q)
D--> (r) 7. A--> (p)
B--> (s)
3. A--> (q) C--> (r)
B--> (p) D--> (q)
C--> (s)
D--> (r) 8. A4' (p, q, r, s)
B--> (p, q, r, s)
4. A--> (r) C --> (p, q, r, s)
B--> (s) D --> (p, q, r)
C--> (q)
D--> (p) .

5. A-> (p)
B--> (s)
C--> (r)
D--> (q, r)

i:====================================2.113 =
www.puucho.com
Rahul Sardana with www.puucho.com

Substituting d = 201. , we get


1. Shift dY =I(µ, -1)1, -(µ, -1)t,I~

=> dy=i(1.2-1)x15x10 .. -(1.6-1)x10x10_.1( l.S __,)


sinB=sin(45°)+ ::i.
1.5x10 => 9::::59°
3
,.1.y=3x1ff" m
',
=>
=> dY =3 mm 4. (a) 1~ =(¾-1)(¾+¾)
2. Since, 1- =0.25 m and d =2 m = 81. => A=10cm
At A ·and C, AX=d=81- Now applying µ 2 -~ = µ2 - µ, twice, we get
Since this is an even multiple of A, so maximum intensity is V u R
obtained. 3/2 __1_=3/2-1 and
•.• (1)
At Band D, AX=0 v, -20 10
Since.this is an even multiple of A, so again maximum intensity
will be obtained. ·
4/3 _3/2 4/3 -3/2
V2 V1 -10
.•. (2)
B
Adding equation (1) and (2), we get
'' v2 =80cm .
'
'''
(b) The Image formed by (lens + water) system will act as an
object for the mirror.
'
C ------•--•---- A This is below the axis of m1 and at the same distance as
S1 1 Sa
..,._~-... the -centre is, therefore, its image will be fanned vertically
' above at 1 mm from AB . Similarly m2 will fonn an image
'' of 11 , 1 mm below CD .
D
Further, between A and B seven maximas corresponding to
AX = 7A , 61. , 51. , 41. , 31. , 21. and i. are obtained.
Simila~y between B and C, C and D , and D and A .
So, total number of maximas is N = 4 x 7 + 4 =32 ----------+-------
3. Ray 1 has a longer path than that of ray 2 by a distance
c---------1-------- ----·D
dsin(45°), before reaching the slits. Afterwards ray 2 has a
path longer than ray 1 by a distance dsin8. The net path i+-20 cm......
difference is therefore, dsin9- dsin45°
(a) Central maximum is obtained where, net path difference Is
=> 1112 =1+1+1+1=4 mm
zero, so we have (c) d=1112 =4 mm, D =80 cm
dsin8-dsin(45°)= 0 . V
=> 8=45° Since, 1c =f
(b) Third order maxima is obtained where net path difference
is 3"- , so we have i.---"--( 3x10' x3)
=>
dsin8-dsin(45°) = 31. - µ.f- 7.5x10 14 4
3
=> sine"= sin(45°) + di.

=2.11411:::===================================
www.puucho.com
Rahul Sardana with www.puucho.com

Wave Optics

=> P=).D =~( 3x10'14 J(80x10""J m= 6 x 10_. m =>


d 4 7.5x10 4x10-3
=> P=60 µm
-3d] 3d2
5.
S 3P-S,P=dsine, = d [ ~ =w=l.
The intensities of the rays due to successive reflections and
refractions are shown in figure. So, => ,i~ = 2,
11 =0.11 0 , 12 = 0.08110
s, pl
3d
2
--
I ._
--- 0.09!, s,

I ---u
0.11,

s,
- e, -------- !

If IR be the resultant intensity of the wave at the point P , then


IR=l1 +1 2 +2Mcoscji
=>
=> r, = 1, + 41 0 + 2J(1,)( 41,) cos(120')
=> l,=31,=3(12Wm"")=36Wm""
1
=> =[Jf +ll' =(19)' =361
!
m~
I.,,. {I_1 1O. Angular fringe width e = =~
vi;
According to the given condition
!:, __
._
6. Fringe width p = AD and y = ';, d-180x60
d v3
Therefore_, number of fringe widths in a distance y are given by d < 6x10-7 x180x60
=>
n= r = __E.__ -
P ,/al,
0.32 X 10_,
(,/3)(500x10_,)
369.5
dmax =2.06 X 1"Q-3 m =2.06 mm

11. According to Snell's Law, we have


1sin(30') = F3 sinr
:) r=30°

2
air
~-__c,.""'-----JL--

film
Therefore, total number of maxima obtained in the angular range
-30° < 0 < 30° (including the central one) is
N=2x369+1=739 air
The Optical path difference is given by
7. Here, d=2mm=2x104 m, 0=10 cm=0.10 m, llX. = 2µtsecr-2ttanrsini
t= 0.5 mm =0.5x10-a m, llX =5 mm=5x10-a m,
=> AX =2,/a(tsec30')-2ttan(30°)sin(6D')
A=6x10-7 m

Since, AX= ~(µ-1)1 AX= 4t-2{ lJ( ~) =31


Since the Ray 1 is reflected at the surface of the denser medium
µ- 1 = .1Xd 5x10-3x2x10: =0. 2 so it suffers an additional phase change of re or a path change
Dt0.10x0.5x10
µ=1+0.2=1.2 of ~ . So, for constructive interterence, we have
=> 5µ=6
3t=!:
2

9. => t=!:=1000 A=100 nm


6

c:=================================== 2.115=

www.puucho.com
Rahul Sardana with www.puucho.com

AdvancedJEE Physics Optics & Modem Physics

12. Since the wavelength is decreaf?ing from 600 nm to 400 nm ,

so fringe width ~Isa decreases by a factor of ~ = ~ . So the

number of fringes will Increase by a factor of ¾. Hence the new


number of fringes formed ls
N=_:3_(12)=18
2

The path difference at O Is 14. 0 = 0.01 radian


AX=(S,P-T).,., + T- -(S,P).,., n =10
J..=6000x10~ cm
AX=yd+(µT _T) - AX=2t=n).
D µt 1quld

At the central maxima, we have path difference Since, 8 =.!.


X 0
AX=O
=> l=0X
=> Y=_DT(!1=11,)=-TD(4:J)
d µ, d 10-t
=> 2Bx = n, l+-X-

1
Cen~ maxima will be at O when y = O => x=" =3x10-1 m=3mm
20
~ t=4s
Now, the speed of the central maxima is given by 15. When coherent, then IDC at centre is zero, so, we have the
lcl)tJ 6DT resultant intensity to be
V=[dt]= (10-t)'d 1, =41
When·incpherent, then the sources will not interfere and it wi11 be
6DT OT
At t=4s,wehave v= d =6d a general illumination at the point, so we have the resultant
36 intensity to be, C

1 3 I, =21
=> v- ( )( exrn-<>) 3x10'° ms-' =3 mms-'
(6)(~x10_,)
=> .!i.=2
1,

13. Fringe width, p = '~ «).

=2.116

www.puucho.com
Rahul Sardana with www.puucho.com

Dual Nature of
Radiation & Matter

IContents ---- ----- ---- -----------


------- - _______ _,
DUAL NATURE OF RADIATION & MATTER-----------···············································............ 3. 1
Solved Practice Problems ..................................................................................................3.15
Practice Exercise Sets
:> Single Correct Choice Type Questions ..................................................................3.21
:> Multiple Correct Choice Type Questions ...............................................................3.33
:> Reasoning Based Questions (Assertion Reason Type) ........................................3.36
:> Linked Comprehension Type Questions (Paragraph Type) ...................................3.38
:> Malrix Match Type Questions (Column Matching Type) ........................................3.40
:i Integer Answer Type Questions ......................••..•••...•••..••...•.................................3.43
Answers to In Chapter Exercises (ICE) & Practice Exercise Sets .....................................3.45
Solutions to In Chapter Exercises (ICE) .............................................................................3.48
Solutions to Practice Exercise Sets ...................................................................................3.52

www.puucho.com
Rahul Sardana with www.puucho.com

MATTER WAVES
1..=-h- ... (3)
Light possesses dual nature i.e. it behaves both as a wave ~2mEK
and as a particl~. In some phe~omena e.g., interference,
diffraction and polarisation, it behaves as waves because For charged particles accelerated through a potential
they are explained on the basis of Wave theory while in difference of V volts,
some other phenomena e.g. photoelectric effect, Compton Kinetic energy i.e. EK = qV
effect, it behaves as particles (photons). ,. h
Since nature demands symmetry, therefore de-Broglie ... (4)
~2mqV
thought that matter must have dual nature. The particle
nature of matter is well known and hence de-Broglie thought For electrons accelerated through a potential difference of V
that material particles must possess wave-nature. volts
h
de-Broglie wavelength ).. ,/2tneV
de-Broglie's POSTULATE
According to de-Broglie a material particle in motion must Substituting m=9-lxl0°"1 kg, h = 6 •62 X lO""'Js ,
have a wave like character and the wavelength associated e=l-6x10"19 C,weget
with it is given by
,. =!!. ... (1) A. =J1~Q X 10-10 ... (5)
. p
where, h is the Planck's constant whose value is given by
h =6.63x10°"' Js and
=> i..=J1~o A=1'% A
p = momentum of the particle This is expression for de-Broglie wavelength associated with
electron accelerated through a potentjal difference of V .
de-Broglie assumed this expression in analogy with photon
For neutral particles (like neutrons, atoms) at temperature
because momentum of photon is
T , kinetic energy of most of particles,
h
p=- ,. EK= k8 T, where k8 is the Boltzmann's constant.

1,, h ... (6)


=> A.=!!. ~2mk,T
p
H m is the mass of particle and v the velocity, then The wave nature is possessed by all particles neutral of
momentum of particle is p =mv . charged. The wave nature was first verified by Davisson and
Germer for slow electrons.
So, de-Broglie wavelength ).. =.!!.._ .•. (2)
mv
Illustration 1
If E1: is kinetic energy of particle, then Obtain the de-Broglie wavelength associated with
thermal neutrons at room -temperature (27 °c) . Hence
p=~2mEK. {·:EK=;:} explain why a fast neutron beam needs to be thermalised

====================================· 3.1 =
www.puucho.com
Rahul Sardana with www.puucho.com

Advanced JEE Physics ' Optics & Modern Physics

with the environment before it can be used for neutron de-BROGLIE WAVELENGTH ASSOCIATED WITH THE
diffraction experiments. CHARGED PARTICLES
Solution The energy of a charged particle accelerated through
Average kinetic energy of a neutron at absolute potential difference V is E =.!.mv' = qV
temperature T is 2
Hence de-Broglie wavelength
.!.mv' =!l_k T
2 2 ' 'l.=!!.= h h
p .J2mE )2mqV
=a, L =!3.k T {·: p=mv)
2m 2 ' Using the above formula, we get
=a, p=~3mk,T ,. = 12.27 A
Eectnm .Jv
de-Broglie wavelength is given by
'/,. h h ,. = 0.286 A
Proton .Jv
p ~3mk8T
Given m, = 1.675x10-" kg, k, = 1.38x10-23 J mol-' K-1
T=27+273=300K, h=6.63x10-" Js
6.63x10""'
'/,.
.J3 X 1.675 X 10-27 X 1.38 X 10-23 X 300
6.63x10-20 de-BROGLIE WAVELENGTH ASSOCIATED WITH
=a, '/,. ----m=1.45x10-" m UNCHARGED PARTICLES
4.56
For Neutron de-Broglie wavelength is given as
=a- 'l.=1.4sA
10
As this wavelength is comparable to interatomic spacing ,. 0.286 x 10- m 0.286 A
N~<ron ~E(in eV) ~E(in eV)
(- 1 A) in a crystal, so thermal neutrons can be used for
diffraction experiments. A high energy neutron beam should Energy of thermal neutrons at ordinary temperature is given
be first thermalised before using it for diffraction. by
E=k,T
CHARACTERISTICS OF MATTER WAVES
'l.=-h-
a) Matter wave represents the probability of finding a .J2mkT
particle in space.
where T is the Absolute temperature, k8 is the Boltzmann's
b) Matter waves are not electromagnetic in nahlre.
constant given by k 8 = 1.38 x 10-23 JK-1
c) de-Brogile or matter wave is independent of the charge

=-;=--~;;;;;~~----
on the material particle. It means, matter wave of de- 34
6.62x 10- 30.83 A
Broglie wave is associated with every moving particle So, AThermalneutro
27
• .J2x1.67x10- xl.38x10- T 23
.ff
(whether charged or uncharged).
d) Practical observation of matter waves is possible only
HEISENBERG's UNCERTAINTY PRINCIPLE
when the de-Broglie wavelength is of the order of the
size of the particles. According to Heisenberg it is impossible to measure the
e) Electron microscope works on the basis of de-Broglie position and momentum of a particle simultaneously with
waves. 100% accuracy. This is called Heisenberg's uncertainty
principle. Uncertainty principle successfully explains the
£) The phase velocity of the matter waves can be greater
than the speed of the light. (i) Non-existence of electrons in the nucleus
g) Matter waves can propa·gate in vacuum, hence they are (ii) Finite size of spectral lines.
not mechanical waves. If /;,x and ilp are uncertainties in determining the position
h) The number of de-Broglie waves associated with nth and momentum of the particle simultaneously, then
orbital electron is n .
/;,xilp ;e ..!:._, where h = 6.63 x 10->1 Js
i) Only those circular orbits around the nucleus are stable 41t
whose circumference is integral multiple of de-Broglie h
wavelength associated with the orbital electron. =a> m!;,xilv ?. -
41t

= 3.2

www.puucho.com
Rahul Sardana with www.puucho.com

Dual Nahtre of Radiation & Matter


This principle is universal and holds for all microscopic and 6.63x10-"
macroscopic particles. The principle is also unaffected by ). 3.32 X 10-27 X 1911 ill
experimental techniques.
If ~ = 0 , then 11p ----> oo and if 11p = 0 then ~ ----> oo ,i.e. if we ). = 1.04; 10-rn m = 1.04 A
are able to measure the exact position of the particle (say an
electron) then the uncertainty in the measurement of the QUANTUM NATURE OF LIGHT
linear momentum of the particle is infinite. Similarly, if we Some phenomena like photoelectric effect, Compton effect,
are able to measure the exact linear momentum of the Raman effect could not be explained by Wave theory of light.
particle i.e., tlp = 0 , then we cannot measure the exact Therefore quantum theory of light was proposed by Einstein
position of the particle at that time. who extended the Planck's hypothesis to explain Black Body
radiation. According to quantum theory of light or radiation,
Viewertj tJ ti the energy of an electromagnetic wave is not continuously
distributes over the wave front Gust like the energy
lncide~ · \ / Reflected possessed by water waves). Instead Plank proposed that an
photo~: \ a ~ photon electro-magnetic wave travels in the form of discrete packets
or bundles of energy called Quanta.
Original momentum
of electron
~a,. So, according to Plank, "light is propagated in bundles of
small energy, each bundle being called a photon and
Final momentum
possessing en~rgy", given by
of electron
he
An electron cannot be observed without changing it's E=hv=-
,_
momentum
This principle is also applicable to energy and time, angular where, v is frequency, 1,. is wavelength of light, h is
momentum and angular displacement. Hence, Planck's constant whose value is 6-63xl0_-"-Js and
c=3x10 8 ms-1
!'.EM-;,;!!_
4,r
PROPERTIES OF PHOTONS
M,/10 -;,; .!!_
4,r 1. In the interaction of radiatio~ with matter, the radiation
behaves as if it is made up of particles called Photons.
This fundamental is also called as Corpuscular Theory
PROBLEM SOLVING TRICK(S)
of Light. So, light behaves both as a particle and a wave.
a) For numerical problems, we shall use
2 All photons emitted by any source travel through free
Ax.l.p ~ h t>EAt z h &t.8 ~ h
space with a speed equal to the. speed of light i.e.
b) If the radius of the nucleus is r then the probability of
c=3x108 ms-1 •
finding, .the. electron inside the nucleus is L\x =2r and
3. Each photon has a definite energy depending upon the
uncertainty in momentum is t\p =..!!.... . frequency v of the radiation and this energy is
4.r
independent of the intensity. So,

Illustration 2 ,.
E=hv=hc (injoule)

Find the de Broglie wavelength corresponding to the


root-mean-square velocity of hydrogen molecules at room 4. · A , then E =-,_-
u,.·IS m 12375
eV .
temperature (20 °C) •
5. If the intensity of the light of given wavelength is
Solution increased, then there is an incr~ase in the number of
photons incident per second per unit area on a surface.
.
Smce, Vrms=
~3RTM However, energy of the photon remains the same as
long as the frequency or the wavelength is unchanged.
v = 3x8.3lx293 =1911 ms-' 6. The speed of the photon changes as it travels through
nns 2x 10-3
different media due to the change in its wavelength.
h h 7. The frequency of the photon does not change when it
Now, A=-=--
p mvrms goes from one medium to the other.
Mass of one hydrogen molecule is given by 8. In the situations wizen a photon collides witlz a
2
material particle, the total energiJ and momentum
m kg = 3.32 X 10-27 kg remains conserved. However, the number of photons
6.02x1026 may not be conserved in a collision because during the

3.3 =
www.puucho.com
Rahul Sardana with www.puucho.com

Advanced JEE Physics Optics & Modern Physics

collision photon(s) may be absorbed or new photon(s) 14. Intensity of light (1)
may be created. Energy crossing per unit area normally per second is
9. A photon is an electrically neutral particle which is not called intensity or energy flux ,i.e.
deflected by electric and magnetic field.
10. Rest mass of photon is zero. I= !t =: (f = P = radiation power)
Since the mass of a particle m moving with a speed v At a distance r from a point source of power P
is given by intensity is given by
mgm, I=-
41tr2
p

where m,
'
is the rest mass of the particle.
=> I oc ]:_
r'

m,=m g 2
15. Number of photons falling per second (n)
If P is the power of radiation and E is the energy of a
p
Now, for a photon, v = c, hence photon then n = - .
E
m0 =0
So from here we can conclude tl:iat the rest mass of a MOMENTUM.AND RADIATION PRESSURE
photon is always zero, i.e. we cannot have a frame of
reference where the photon is at rest. An electromagnetic wave consists of photons capable of
transporting linear momentum. The linear momentum P
11. Dynamic or kinetic mass of a photon, is determined by
possessed by an electromagnetic wave is related to the
using the Einstien's Mass-Energy Equivalence, i.e.,
energy E it transports according to the relation
E=hv=mc2
E
h hv P=- ---W
=> m=-=- C
c2 cJ...
If the wave is incident in the direction perpendicular to a
12 The Linear Momentum of a photon is found by using surface and gets completely absorbed by the surface, then
the de-Broglie relation according to which, we have equation (1), tells us the linear momentum imparted to the
h C surface.
:\.=-,where :l.=-
p V If surface is perfectly reflecting, the momentum change of
the wave is doubled. Consequently the momentum imparted
hv h
=> p=-=-
c ,.
to the surface is also doubled.
According to Newton's Second Law, the force exerted by an
However, this resuit is also obtained by using the fact electromagnetic wave on a surface is given by the equation,
that the total energy of a subatomic particle of rest mass
m0 , moving with a velocity v , having momentum p is F=t.p
M
given by
E2 =p2c2 + ni;c4 .
Fromequation(l), t.p= - -
- l(t.E)
M c M
Now, for a photon, m0 =0, so we have from above
expression that => F=!(t.E)
M C
... (2)
E=pc
Intensity (I) of a wave is the energy transported per unit
Since, E=hv=: area per unit time.

=> I= (2-) t,.E


=> pc=- ,.
he A M
=> t.E =IA
hv h M
=> p =~=i
13. The number of photons N , each of energy E , emitted Substituting in equation (2), F = IA
C
from a source of monochromatic radiation of
F I
w~velength ;\. and energy W and power P => - =pressure =-
A C
N=w =w =Pt I
E hv hv => prad =~

www.puucho.com
Rahul Sardana with www.puucho.com

D11al Na hire of Radiation & Matter

!_ is also equal to the energy density (energy per unit p I(4itR')


C (b) n=-
E E
volume) u.
2
Hence, P,., = u ... (3) (1400)(4,r)(l.49 X 1011 )
=> n l.18xl0 45
3.3xl0-19
The radiation pressure is thus equal to the energy density
(Nm-2 = Jm"). At a perfectly reflecting surface the pressure
Illustration 5
on the surface is doubled. Thus, we can write,
I A small plate of a metal is placed at a distance of 2 m
prad=-=U {wave totally absorbed} from a monochromatic light source of wavelength
C
4.8 x 10-7 m and power 1 watt. The light falls normally on
21 the plate. Find the number of photons striking the metal
and P.., =-=2u {wave totally reflected}
C plate per square meter per second.
Solution
Illustration 3
(6.6 X 10-34 )(3 X 10 8 ) .1 X _ J
Find the energy, the mass and the momentum of a 4 25 10 19
(4.8x10-')
photon of ultraviolet radiation of 280 nm wavelength.
Number of photons striking the metal plate per square meter
Solution per second is
Given, ,. = 280 x 10-" m
. E =l!c n=(f)(4:,,)
Smce, ,_-
1 1
=> n=( 19 ) =4.82xl016 m-2s-1
E (4.316x10-15 eV sec)(3x108 ms-1 ) . 4.125xl0- (4it)(2)2
=> 4.6eV
(280x10-" m)

Mass of photon is m = ~ Illustration 6


C Find the number of photons entering the pupil of our
4.6 X 1.6 X 10-19 eye per second corresponding to the. minimum intensity of
=> m 8.2 X 10-J6 kg
(3x10')
2
white light that we humans can perceive (-10-" wm-2 ) •
Momentum of a photon is Take the area of the pupil to be about 0.4 cm' and the
average frequency of white light to be about 6 x 1014 Hz.
E
P=- Solution
C

4.6 X 1.6 X 10-19 Minimum intensity, I =10-10 wm-2


=> p 2.45 X 10-27 kg ffiS-l
3xl0 8 Area of pupil, A=0.4 cm' =0.4x10--< m
Average frequency, v = 6 x 10-14 Hz
Illustration 4 Energy of one photon
The intensity of sunlight on the surface of ear\h is E=ltv=6.63xl0-34 x6xl014 J=4x10-19 J
1400 wm-2 • Assuming the mean wavelength of sunlight to Let n be the nllmber of photons crossing per square metre
be 6000 A, calculate area per second.
(a) the photon flux arriving at 1 m' area on earth Now Intensity = Energy incident per square metre area per
perpendicular to light radiations, and second
(b} the number of photons emitted from the sun per => I= Total Energy of 11 photons
second assuming the average radius of Earth's orbit is => I = n x Energy of one photon
1.49 X 1011 ffi ,
Intensity
Solution => 11
Energy of one photon
(a) Energy of a photon 10-10 wm-2
12400 => I =--:,',,,CC.-
19
2.5xl08 m-2 s-1
E = ltc = = 2.06 eV =3.3xo-" J 4xl0- J
,_ 6000
So, the number of photons entering the pupil of our eye per
Photonflux IA (l400)(l), 4.22x10 21 second is
E 3.3x10-19
N = n (Area of the pupil)
=> N = 2.5 x 108 x 0.4 x 10-1 .-, =104 s-'
3.5 =
www.puucho.com
Rahul Sardana with www.puucho.com

Advanced JEE Physics Optics & Modern Physics

Illustration 7 Illustration 9
Find the number of photons emitted per second by a A cylindrical rod of some laser material 5 x 10-2 m
25 W source of monochromatic light of wavelength long and 10-2 m in diameter contains 2 x 1025 ions per
6600 A . What is the photoelectric current assuming 3% m 3 • ff on excitation all the ions are in the upper energy
efficiency for photoelectric effect? Given h = 6.6 x 10"' Js . level and de-excite simultaneously emitting photons in the
· same direction, calculate the maximum energy contained in
Solution
a pulse of radiation of wavelength 6.6 x 10-' m. If the pulse
Energy of each photon is
lasts for 10-7 second, calculate the average power of the
Eh: 66 10
· ::~:;;_: '
19
3x10- J laser during the pulse.
Solution
Total energy emitted per second by 25 W source is Total number of ions in the rod is
E=25J
N =(Number of ions)x(Volume of)
Number of photons emitted per second is per unit volume the rod
25
n= 8.33 X 1019 N=(2x10 25 m--3)x(3.14x(0.005)2 x5x10-2 m 3 )
3 x10-19
Photoelectric current (I) is N=7.85x10 190
As all the ions de-excite simultaneously, the number of
I=( 3 % of photons ) x (charge on)
emitted per second electron photons emitted in the same·direction is also 7.85x10 19 •
So, the energy contained in a pulse of radiation of
3 25 _,,
I=-x xl. 6 xl 0 -=0.4A wavelength 6.6 x 10-7 m is
100 3x10-19

Illustration 8
A source emits monochromatic light of frequency E = he x7.85x1019
A,
5.5 x 1014 Hz at a rate of 0.1 W . Of the photons given out,
·0.15% fall on the cathode of a photocell which gives a E 6.6x10-34 x3x10 8
23.55 J
current of -6 µA in an external circuit. 6.6 x10-7
(a) Find the energy of a photon. Energy
Average power P=-.--
(b) Find the number of photons leaving the source per Tune
second. 2355
(c) Find the percentage of the photons falling on the P= J = 23.55 x 107 W = 235.5 MW
10-' s
cathode which produce photoelectrons.
Solution ,A: .... ,, _, .,., .. ,.,,,,,,. . --1~~~1e·1 "
20
~--·-~v~:-: :'~-~:.-:~·,,,t1H:,'" -~,,>~~-;:a.lSEt~i5m4flN 1.- ...
(a) Since, E=hv=(6.6x10"')(5.sx10")=36.3x10- J BASED ON PHOTON PROPERTIES /
=>
E=2.27eV (Solutions on page 3.48) !
, 1. The intensity of direct sunlight before it passes through the ;
(b) Number of photons leaving the source per second is
earth's atmosphere is 1.4 kWm 4 • If it is completely :
n=!'._=
O.l 2.75x1017 absorbed find the corresponding radiation pressure. \
I
E 36.3x10-20
(c) Number of photons falling on cathode per second is ! 2. According to the maxwell theory of electrodynamics an I
electron going in a circle should emit radiations of frequency ]
n, = O.l 5 x2.75x1017 =4.125x1014 equal to its frequency of revolution. What would be the
100 wavelength of the radiation emitted by a hydrogen atom in
ground state if this rule is followed?
Number of photoelectrons emitting per second .is
6x10_. \ 3. An electron is accelerated by a potential difference of 25
n2 - - ~ ~ 3.75x1013 volt. Find the de-Broglie wavelength associated with it.
l.6x10-19
I ;
%age of Photons ) = n2 x 100 . 4. Find the number of photons emitted per second by a MW !
So, ( falling on Cathode n, transmitter of 1O kW power emitting radiowaves ofj
13 wavelength 500 m .
%age of Photons ) = 3.75 x 10 x = %
( falling on Cathode 4.125 x 1014 100 9 15. lf 5% of the energy supplied to an incan9escent light bulb Is
radiated as visible light, how many vi.Sible _light photons are
.. .!31J1itted_by 1O_O_'!V~tt b_ulb~b~sume. waY.E!l~i:i91Ji. of_~IJ _visible
,.____... 3.6

www.puucho.com
Rahul Sardana with www.puucho.com

Dual Nature of Radiation & Matter


photons to be 5600 A . Given h = 6.625 x 10--" Js . The minimum energy required for the electron emission
from the metal surface can be supplied to the free electrons
6. Calculate the number of photons in 6.62 J of radiation by any one of the following physical processes.
energy of frequency 1012 Hz. Given h = 6.62 x 10-34 Js.
THERMIONIC EMISSION
7. Monochromatic light of frequency 6 x 1014 Hz is produced
By suitably heating a metal, sufficient thermal energy can be
by a laser. The power emitted is 2 x 10-3 W .
imparted to the free electrons to enable them to come out of
(a) What is the energy of each photon in the light?
(b) How many photons per second, on the average, are
the metal. The free electrons so emitted are called as
emitted by the source? Therm ions.

8. Show that a free electron at rest cannot absorb a photon FIELD EMISSION
and thereby acquire kinetic energy equal to the energy of
the photon. Would the conclusion change if the free electron When the metal surface is subjected to very strong electric
was moving with a constant velocity? field of the order ranging from 10 3 vm-1 to 108 vm-1 , the
9. An electron microscope uses electrons accelerated by a
electrons (beyond a certain limit) start coming out of the
voltage of 50 kV . Determine the de-Broglie wavelength
metal surface. This method of emission is dangerous and less
associated with the electrons. If other factors (such as efficient. This method of emission is also called the Cold
numerical aperture etc.) are taken to be roughly the same, Cathode Emission.
how does the resolving power of an electron inicroscope
compare with that of an optical microscope which uses
PHOTO-ELECTRIC EMISSION
yellow light?
When light of certain minimum energy (or m1rumum
10. An electron and proton are possessing the same amount of frequency or maximum wavelength) illuminates or falls on a_
kinetic energy. Which of the two have greater wavelength?
metal surface, electrons are emitted from the metal surface.
11. An electron and a photon have same de Broglie wavelength TI1e emitted electrons are called pllotoelectrons. In case of
(say 1 A ). Which one possesses more kinetic energy?
Photoelectric emission, the rate of emission of photoelectrons
is very low.
12. An electron and a proton have same wavelength. Which
one p_o~sesses more energy? SECONDARY EMISSION
When fast moving electrons strike a metal surface, then some
EMISSION OF ELECTRONS of their energy is transferred to the free electrons of the
As we are aware of the fact that metals have free electrons metal. Due to this, when free electrons gain energy more
(negatively charged particles) which are responsible for their than the work function, then they are emitted from the metal
conductivity. However, the free electrons cannot normally surface. These emitted electrons are called the secondary
escape out of the metal surface. If an electron attempts to electrons.
come out of the metal, the metal surface acquires a positive
charge and pulls the electron back to the metal. The free PHOTOELECTRIC EFFECT
electron is thus held inside the metal surface by the attractive The phenomenon of emission of electrons from a metallic
forces of the ions. Consequently, the electron can come out of surface by the use of light (or radiant) energy of certain
the metal surface only if it is supplied some minimum
minimum frequency (or maximum wavelength) is called
energy to overcome the attractive pull of the metal.
photoelectric effect. The emitted electrons are called as
This minimum energy required by an electron to escape photoelectrons. The phenomenon was discovered by
from the metal surface is called the work function of the metal. Hallwach in 1888. For photoelectric emission the metal used
It is generally denoted by W or $0 and is measured in eV must have low work function e.g., alkali metals. Cesium is
(electron volt). One electron volt is the energy gained by an assumed to be the best metal for photoelectric effect. To
electron when it has been accelerated by a potential escape from the surface, the electron must absorb enough
difference of 1 volt, so energy from the incident radiation to overcome the attraction
1 eV=1.6x10-19 J of nucleus of the atom of the metal surface. The explanation
to the photoelectric effect given by Einstein is based on the
This unit of energy is commonly used in atomic and nuclear Law of Conservation of Energy. Before discussing the effect
physics. further, we must understand the following terms.
The work function, W depends on the properties of the
metal and the nahrre of its surface. The work function for A. WORK FUNCTION (OR THRESHOLD
platinum is the highest WP, = 5.65 eV, whereas it is the ENERGY) (W,)
lowest for caesium i.e., W0 = 2.14 eV. The minimum energy of incident radiation, required to eject
the electrons from metallic surface is defined as work

3.7 C=:J

www.puucho.com
Rahul Sardana with www.puucho.com

Advanced JEE Physics Optics & Modern Physics

function of that surface. It is the characteristic of a metal a) For each emitting metal, there is _a certain minimum
surface. frequency v 0 (or maximum wavelength 'l.0 ), called the
threshold frequency of the incident radiation, below
W0 =hv0 = he (in joule), where (above which) which no emission of photoelectron
'I.,
takes place, no matter how great is the intensity. The
v O = Threshold frequency and
value of v0 ( or 1..0 ) is different for different emitting
'/.
0
= Threshold wavelength surfaces.
Work function in electron volt is given by b) The process of emission of photoelectrons is an
W.{eV)=~ 1237~ instantaneous process. There is no time lag ( < 10..,, s)
e'l.0 '/.
0
(in A) between the incidence of radiation and the emission of
photoelectrons.
B. THRESHOLD FREQUENCY (v0 ) c) Photoelectric effect is a one photon-one electron
phenomenon i.e. even if photon has an energy sufficient
The minimum frequency of incident radiations required to to strike off 3 electrons (say) it can only strike off one
eject the electron from metal surface is defined as threshold electron with the excess energy being imparted to the
frequency. struck off electron as kinetic energy.
If incident frequency v < v0 => No photoelectron emission. d) The number of photoelectrons emitted per second, that
is, photoelectric current is directly proportional to the
For most metals the threshold frequency is in the ultraviolet
intensity of the incident radiation but is independent of_
(corresponding to wavelengths between 200 and 300 nm),
the frequency (or wavelength) of light.
but for potassium and cesium oxides it is in the visible
spectrum ( 'I. between 400 and 700 nm) e) The velocities (or the energies) of the emitted
photoelectrons vary between zero and a definite
C. THRESHOLD WAVELENGTH ('1.0) maximum (vmax). The proportion of photoelectrons

The maximum wavelength of incident radiations required to having a particular velocity is independent of the light
eject the electrons from a metallic surface is defined as intensity.
threshold wavelength. f) The maximum velocity, vmax, and hence the maximum
If incident wavelength 'I.> 'l.0 , then No photoelectron kinetic energy is independent of the intensity of the
emission will take place. incident light, but depends on its frequency, increasing
linearly with the increase of the frequency of the
incident light.
Illustration 10
In an experiment on photoelectric effect light of EINSTEIN's EXPLANATION OF PHOTO-ELECTRIC
wavelength 400 nm is incident on a caesium plate at the EFFECT
rate of 5 W . The potential of the collector plate is made
The wave theory of light could not explain the observed
sufficiently positive with respect to emitter so that the
characteristics of photoelectric effect. Einstein extended
current reaches the saturation value. Assuming that on the
Planck's quantum idea for light to explain photo-electric
average one out of every 106 photons is able to eject a effect.
photoelectron, find the photocurrent in the circuit.
According to his idea, the energy of electromagnetic
Solution radiation is not continuously distributed over the wave front
E = 12375 = 3.1 eV like the energy of water waves but remains concentrated in
4000 packets of energy content hv, where v is frequency of
Number of photoelectrons emitted per second radiations and h is universal Planck's constant

n= (-!.)(
10
5
3.lxl.6x10- 19 ) = 1 x 1013 per second
(=6-625x10""" Js). Each packet·of energy moves with the
speed of light. The assumptions of Einstein's theory are

. q Ne (N)
a) The photoelectric effect is the result of collision of two
.
smce, t=-;=- f e=ne
-= particles, one of which is a photon of incident light and
1
the other is an electron of photo-metal.
=:, i=(ne)=lxl013 xl.6xl0-19 =1.6x10-' A=l.6 µA b) The electron of photo-metal is bound with the nucleus
by Coulomb attractive forces. The minimum energy
LAWS OF PHOTOELECTRIC EMISSION required to free an electron from its bondage is called
We thus have the following laws of photoelectric emission, work function, W = $0 = hv 0 •
derived from the experimental observations. c) The incident photon interacts with a single electron and
loses its energy in two parts
= 3.8

www.puucho.com
Rahul Sardana with www.puucho.com

Dual Nature of Radiation & Matter.


(i) Firstly, in getting the electron released from the EXPERIMENTAL SETUP FOR PHOTOELECTRIC EFFECT
bondage of the nucleus.
It consists of two conducting·electrodes, the anode (Q) and
(ti) Secondly, to impart kinetic energy to emitted
electron. cathode (P)· which are enclosed in an evacuated glass tube
as shown in figure. ,,.~
d) The efficiency of photoelectric effect is less than 1 %, i.e.
0,0-~o
less than 1 % of photons are capable of ejecting <?-"' ,
v~'"''-~,,.
o:
photoelectrons.
Accordingly, if hv is the energy of incident photon,
--=,,_ -"----
then
r-fll-p-1 ·-~ ··:.:~
hv=W+K_ ~-~~
~ K==hv-W ... (1)

~---'W.Mr-----1, IIIt----'

The battery or . some other source of potential difference


creates an electric field in the direction from anode to
cathode. Light of certain wavelength or frequency falling on
the surface of cathode causes a current to flow in the external
. Metal circuit. This current is called the photoelectric current.
When the potential difference increases, the photo
This is Einstein's photoelectric equation, where W is electric current also increases till saturation is reached. As
work function and ~ the polarity of battery is reversed (i.e. plate Q is at negative
. 1 2 potential w.r.t. plate P) the electrons start moving back
Kmax = zmv= = eva towards the cathode. It is observed that at a particular
negative potential of plate Q , no electron reaches the plate
is the maximum kinetic energy of photo-electrons
emitted. Q i.e. the current becomes zero. This negative potential for
which the photo-electric current is zero is called the
Equation (1) is referred as Einstein's photo electric
stopping potential denoted by V0 • Maximum kinetic energy
equation that explains all experimental results of photo-
electric effect and is based on the Law of Conservation (in eV) of photo electrons in terms of stopping potential will
of Energy. therefore be K_ =IV, I eV

Illustration 11 Illustration 12
Ultraviolet light of wavelength 2000 A causes When a beam· of 10.6 eV photons of intensity
photoemission from a surface. The stopping potential is · 2 wm·2 falls on a platinum surface of area lxto-< m 2 and
2V. work function 5.6 eV, 0.53% of-the incident photons eject
(a) Find the work function in eV photo electrons. Find the number of photoelectrons
(b) Find the maximum speed of the photoelectrons. emitted per second and their minimum and maximum
Solution energies (in eV). Given that 1 eV =1.6xto·" J.
(a) Using Einstein relation Solution
he Number of photoelectrons emitted per second
W=-i:--eV0
N (Intensity) (Area) 0.53
x--
~ W= 12400 -2=4.2 eV (Energy of each photon) 100
2000
~
N (2)(1x10-<) x 0.53 = 6.2Sxl0 11 s-•
.
(b) Smce ·1 ·, =eV. •
~mv_ t (10.6xl.6x10·19 ) 100
O
2
Minimum kinetic energy of photoelectrons is
~ v = J2 eV = 0
19
2(1.6xl0- )(2) K""' =0
max m 9.lxl0-:31
and maximum kinetic energy is,
=> vmax =8.4x10 5 ms-1 K_ =E-W=(l0.6-5.6) eV=5 eV

3.9 =
www.puucho.com
Rahul Sardana with www.puucho.com

Advanced JEE Physics Optics & Modern Physics

Illustration 13 Bap
V=-- ... (1)
A monochromatic light source of frequency 1!
illuminates a metallic surface and ejects photoelectrons. (2,5 X10" )(1.76X lQU )(2.7 X10-3 )
The photo electrons having maximum energy are just able V
3.14
to ionize the hydrogen atoms in ground state. When the
entire experiment is repeated with an incident radiation of => v=0.38x10 6 ms-'

frequency ~ f,
the photoelectrons so emitted are able to Since, KE=.!..mv 2 =E-W
6 2
excite the hydrogen atom beam which then emits a
radiation of wavelength 1215 A . => W =E-.!.mv' ... (2)
2
(a) What is the frequency of radiation. Substituting value of v from equation (1) in equation (2), we
(b) Find the work function of the metal. get
Solution 2
W = _ _ 1 (9,1 X10-" )(0.38 X10 6 )
(a) Using Einstein's equation of photoelectric effect i.e. 49
2 1.6x10-19
Km~=hf-W
=> W =(4.9-0.4) eV
where K= =13.6 eV => W=4.5eV
=> h/-W=13.6eV ... (1)
So, when the experiment is repeated, then Illustration 15

h(~
6
t)-
W=
12375
1215
= 10.2 eV ... (2)
If the wavelength of the incident radiation is
increased from 3000 A to 3010 A, find the corresponding
Solving equations (1) and (2), we get change in the stopping potential V .
Solution
hf =3.4 eV
6 According to Einstein's Photo-electric equation, we
have
f (6)(3.4)(1.6x10-19) 4.92x10" Hz
eV1 =E1 -W ... (1)
(6.63 X10-34 )
(b) From equation (1), we have eV,=E,-W ... (2)
W=hf-13.6 Subtracting (2) from (1), we get
=> W=6(3.4)-13.6 e(V, -V,) = (E, -E,)
=> W=6.8eV => V, - V, = he (2-_2-)
e A.1 A. 2
Illustration 14 6.6x10""' x3x10 (-1___
8
1_)-o 012 V
A photon with an energy of 4.9 eV ejects
V,-V, 1.6 X10-19 X10-lO 3000 3010 - .
photoelectrons from tungsten. When the ejected electron
enters a constant magnetic field of strength B = 2.5 mT at
Illustration 16
an angle of 60° with the field direction, the maximum
pitch of the helix described by the electron is found to be A light beam of wavelength 400 nm is incident on a
2.7 mm . Find the work function of the metal in electron- metal of work function 2.2 eV . A particular electron
volts. Given that specific charge of electron is absorbs a photon and makes 2 collisions before coming out
1.76 x10" Ckg-'. of the metal
(a) Assuming that 10% of extra energy is lost to the metal
Solution in each collision find the final kinetic energy of this
Pitch of helical path is electron as it comes out of the metal.
(b) Under the same assumptions find the maximum
p =(vcos8)T = vT {·: 8 = 60°) number of collisions the electron should suffer before
2
it becomes unable to come out of the metal.·
where, T= 2mn = 21t I·-- !l
Cl= Solution
qB Ba
12375
1tV (a) Since, E(in eV)
p=- 1.(inA)
Ba

= 3.10 ==================================
www.puucho.com
Rahul Sardana with www.puucho.com

Dual Natllre of Radiatio11 & Matter

E= 12375 =3.1 eV Current~)


4000
Energy of electron after first collision is
E, =(90% of E)=2.79 eV [·: 10% is lost}
Energy of-electron after second collision
Vs.JVs'2 ¼, 0
E, =(90% ofE,)=2.51 eV
(-JV (+JV
Hence, KE of this electron after emitting from the metal
surface = (2.51-2.2) eV = 0.31 eV Thus, the increase of frequency increases maximum kinetic
(b) Energy after third collision, energy of photoelectrons but leaves the photoelectric current
unchanged.
E, = (90% of E,) = 2.26 eV
Similarly, E, = (90% of E,) = 2.03 eV < W C. EFFECT OF PHOTO-METAL
So, after four collisions the electron will not be able to When frequency and intensity of incident light are kept fixed
come out of the metal. and photo-metal is changed, we observe that stopping
potential (V,) versus frequency ( v) graphs are parallel
CHARACTERISTICS OF PHOTO ELECTRIC EFFECT straight lines, cutting, frequency axis at different points. This
The following observations were made to study the effect of shows that threshold frequency are different for different
changes in various factors while studying the Photo Electric
Effect. metals, the slope ( : ) for all the metals is same and hence
universal constant.
A. EFFECT OF INTENSITY
Since we know that
Intensity of light means the energy incident per second per eV,=hv-W
unit area. For a given frequency, if intensity of incident light

~ v, =(~}-(~)
is increased, the photoelectric current increases and with
decrease of intensity, the photoelechic current decreases, but
the stopping potential remains the same.
Comparing with the line y = mx + c, where m is the slope
In photoelectric effect current (;) is directly proportional to and c is the intercept on the y axis.
intensity (I) of incident light.
Then, we observe that the graph is a straight line with slope
Current (i)
~ (a universal constant) and negative intercept W
e e
31 (depending on the nature of the metal).
21
I In figure threshold frequency and work function are greater
for Metal 2 as compared to Metal 1.
o~-----+ V,
Intensity (I)
Metal 1
This means that the intensity of incident light affects the Metal2
photoelectric current but leaves the maximum kinetic energy
of photoelectrons unchanged.

B. EFFECT OF FREQUENCY
When the intensity of incident light is kept fixed and
frequency is increased, the photoelectric current remains the
same but the stopping potential increases.
If the frequency is decreased, the stopping potential
decreases and at a particular frequency of incident light, the
stopping potential becomes zero. This value of frequency of D. EFFECT OF TIME
incident light for which the stopping potential is zero is There is no time lag between incidence of light and the
called threshold frequency v0 • If the frequency of incident emission of photo-electrons.
light ( v) is less than the threshold frequency (vo) , no
photo-electric emission takes place.

3.11 =
www.puucho.com
Rahul Sardana with www.puucho.com

Advanced JEE Physics Optics & Modern Physics

I PROBLEM SOLVING TRICK(S) I DETERMINATION OF PHOTOELECTRIC CURRENT


FORMULAE FOR WORKING THE PROBLEMS ON PHOTO· j Let P be the power of a point. source of electromagnetic
ELECTRIC EFFECT ' radiations, then intensity I at a distance r from the source is
Maximum Kln~tic Energy, of photo-electrons given by
EK = eV0 =Jmv!a,. I=..!_2 wm·2
I 41tr
If A- 0 is the threshold wavelength and v, the threshold I If A is the area of a metal surface on which radiations are
1 frequency, '
I
I incident, then the power received by the plate is
i Work function of photo-metal, i
I. . he
4io =hvo = -
P'=IA=(_I'_,)A(W)
4itr
i.,
If v is the frequency of radiation, then the energy of photon
Threshold frequency Is minimum frequency and Threshold
wavelength Is maximum wavelength of incident light to cause is given by
photoelectric e~ect. E=hv
Einst~in's ~hoto-electric Equation may be expressed as or I The number of photons incident ,on the pla~e per second
EK=hv-$0 (called photon flux) is given by
I· _ , -. I
!
$=E= [_I'_,xA]
~=hc_hc
' 1- ~ . p•
4,,~
· cc;mdition for photoelectric emission is hv ~ 4>0 or equivalently
I . -I ,
!v ~ v 0 or equivalent y As; A0 • Area (A)

If v > v, (threshold frequency) and


GRAPH BETWEEN K_ and V

Let us plot a graph between the maximum kinetic energy


Km~ and the frequency of the falling photon v or the
photon efficiency of the metal plate is.
T\% , the_n the number
photoelectrons emitted per second is
given by
of

-
-.-
-, I
I \
-r-
incident· light. According to Einstein's Photo-Electric
equation, we have
hv=W+E, -
n=.h-[6xA]_B_
100- hv 100 ·
=> K==hv-W Finally, the photocurrent i is given by
Comparing with the line y = mx + c, where m is the slope i =ne
and c is the intercept on the y axis. where e is the charge of an electron (e = 1.6 x 10-19 C)
Then, we observe that the graph is a straight line with slope
h (a universal constant) and negative intercept W
Illustration 17
(depending on the nature of the metal).
Light of wavelength 180 nm ejects photoelectrons
For Metal 2, we observe that
from a plate of a metal whose work function is 2 eV . If a
W2 >W1 and hence (v,),>(v,), uniform magnetic field of 50 µT is applied parallel to the
v. plate, what would be the radius of the path followed by
Metal 1 electrons ejected normally from the plate with maximum
Metal 2 energy.
Solution ,
1,.=180 nm=18oo A
12375
E= =6.875 eV
1800
Since, Km~ = E - W = 4.875 eV

Since, r = mv = ./2mK
qB qB
Substituting the values, we get
Also, 1we observe that when v = v0 , the threshold frequency,
then, K= =0 r=
Jz X 4,875 xl,6 xlQ-l9 X 9,1 X 10-Jl
0.15 m=l5 cm
5x10-5 x1.6x10'=19

= 3.12

www.puucho.com
Rahul Sardana with www.puucho.com

Duai Nature of Radiation & Matter

Illustration 18 •-~~=·'"":,i'--.>-"'x/<i_,t_•._,c;'c.·_ ---"---".'""'"tJ


_:_IC'--E=llc.>cc:·,.··~·~,.>~;='_'_.
In a photoelectric effect set-up, a point source of light ! BASED ON PHOTOELECTRIC EFFECT :
of power 3.2 x 10-3 W emits mono energetic photons of
!' 1.
{Solutions on page 3.50} [
energy 5 eV . The source is located at a distance of 0.8 m What will be the maximum kinetic energy of the l
I photoelectrons ejected from magnesium (for which the work ,
from the centre of a stationary metallic sphere of work
j function W =3.7 eV) when irradiated by ultraviolet light of ·
function 3 eV and of radius 8 x 10-a m. The efficiertcy of
frequency 1.5 x :1015 sec-1 • ' i
photoelectron emission is one for every 106 incident
photons. Assume that the sphere is isolated and electrons 2. A 40 W ultraviolet light source of wavelength 2480 A 1
are instantly swept away after emission. illuminates a magnesium (Mg) surface placed 2 m away.
(a) Calculate the number of photoelectrons emitted per Determine the number of photons emitted from the source 1
second. per second and the number incident on unit area of the
(b) Find the ratio of the wavelength of incident light to magnesium surface per second. The photoelectric work :
the de Broglie wavelength of the fastest functlo_n_ for magnesium is 3.68 eV .. Calculate the kinetic 1
photoelectrons emitted. energy of the fastest electrons ejected from the surface.
'I Determine the maximum wavelength for which the ,
(c) It is observed that the photoelectron emission stops at l
photoelectric effect can be·observed with a Mg surface.
a certain time t after the light source is switched on. lI I
Why? i 3. The hydrogen· atom iii fts ground state is excited ·by m8ans !
(d) Evaluate the time t.
I of monochromatic radiation. Its resulting spectrum has six :
different lines. These radiations are incident on a metal :
Solution plate. It iS observed that only two of them are responsible .
for photoelectric effect. If the ratio of maximum kinetic \
(a) Number of photoelectrons emitted per second is energy of photoelectro_ns in the two cases is 5 then find the
work function of the metal.
n=(1~' )(f)(4:;,) l
[Take ground state energy of H -atom = -13.6 eV ].

1 )( 3.2xl0_, )( 1 ) i 4. A metallic surface is irradiated with monochromatic light of I


II= ( lo' 5 X 1.6 X 10-19 4it X 0,8 X 0.8 : variable wavelength. Above a wavelength of 5000 A , no '·
photoelectrons are emitted from the surface. With an ;
(nxSxlO_, x Sx 10-3 ) unknown wavelength, stopping potential of 3 V is ,
necessary to ·eliminate the photocurrent. Find the unknown '
=:> n =10 5 sec-1 wavelength. j
(b) Km~ =E-W=2 eV I
A,
Since, for an electron, we have · 15. A light source, emitting three wavelengths
6000 A and 7000 A , has a total power of 1er' W and a
5000 1

1
I
:>..
2
= 150 A beam diameter of 2 mm . The power density is distributed
KE(in eV) equally amongst the three wavelengths. The beam shines '
1
·normally on a metallic surface of area on 10-4 m2 and
~ 0
:>..,=J1~ =s.66A having a work function of 1.9 eV . Assuming that each
photon liberates an electron, calculate the charge emitted
,
,
Further, Wavelength of incident photon is per second from the metal surface. :

:>.., = 12375 = 2475 A !' 6. A beam of light consists of four wavelength 4000 A , !
5 4800 A , 6000 A and 7000 A , each of intensity i
!':!_ ~ 286 1.5 x 1o-:3 wm-2 •The beam falls normally on an area 1
:>.., 10'-4 m2 of a clean metallic surface of work function [ ·
(c) Photoemission will stop when potential on the sphere 1.9 eV . Assuming no loss of light energy calculate the ,
becomes equal to the stopping potential. .,' number of photoelectrons liberated per second.
• i
(d) K= = 2 eV0 • Therefore, the stopping potential V, is i In . an experiment on photo electric emission, following 1
'7.
2 volt. Let I be the desired time. Then
I observations were made :
{i) Wavelength of the incident light = 1.98 x 10-1 m
V, =-l_!/.=__!!.!_= (nt)e (ii) Stopping potential =2.5 V .
4ne0 r 41te0r 41te0 r Find the
(a) threshold frequency.
t= V,r 2xsx10-3 (b) work function and
llls
9xlQ X 105 Xl.6 xlQ-l9
9
(c) energy of photo electrons with maximum speed.
_ l )<ne)
( 41te0 I
\ a. Radiation of wavelength 5461 A falls on a photo cathode /
L-- ~q_~~ct~~n~ .!"i!h -~ !!1~1'!!ury,_·~ine!i~ ~e~gy of _!U ~ ~v_ j

================================== = 3.13

www.puucho.com
Rahul Sardana with www.puucho.com

Advanced JEE Physics Optics & Modem Physics

are emitted. When radiation of wavelength 1849 A falls on 11. The photoelectric work function of potassium is 2.3 ei7:7f
the same surface a (negative),potential of 4.6 V has to be light having a wavelength of 2800 A falls on potassium,
applied to the collector electrode to reduce the photoelectric find c-
current Is zero. Find the value,of h 8.nd cutoff wavelength. (a) the. kinetic energy in electron volt of the mqst
energetic electrons ejected.
9. Illuminating the surface of a certain metal alternately with (b) the stopping potential in volt.
light of wavelengths l., = 0.35 µm and l. 2 = 0.54 µm , It
was found that the corresponding maximum velocities of
photo electrons differ by a factor ~ =2. Find the work
function of that meta1.

1O. When a sunace is irradiated with light of l. =4950 A a


photocurrent appears.which vanishes if a retarding potential
0.6 V is applied. When a different source of light is used It
is found that critical retarding potential Is changed to 1.1 V .
Find the work function of emitting surface and wavelength of
second source. If photoelectrons after emission from
surface are subjected to a magnetic field Ot 1OT , what
· changes will be observed in the above two retarding
potentials?

= 3.14

www.puucho.com
Rahul Sardana with www.puucho.com

Problem 1 3.552x10-19
A mercury arc lamp provides 0.2 watt of ultraviolet 20
( ~-: )(4it X 10- )
radiation at a wavelength of A. = 2537 A .
Assume no other wavelength to be present. The cathode of . E 12375 V
photoelectric device consists of potassium and has an
(b) Smce, ,_ (in A) e
effective area of 4 cm2 • The arc lamp is at a distance of
12375
1 m from the cathode. Given that work function for => E= = 4.87 eV
2537
potassium is W = 2.22 eV.
(c) Let N be the number of photons reaching the cathode
(a) According to classical theory, what time of exposure to
the radiation should be required for a potassium atom per second. Then intensity at cathode is,
(radius 2 A) to accumulate sufficient energy to eject a I'=NxE
photoelectron? · But energy falling per second on the cathode is
(b) What is the energy of a single photon from the source?
(~: )(4x10_,), so
(c) What is the flux of photons (number per second) at
the cathode? To what saturation current does this flux
N 0.2x4x10_, 0.2x4x10_,
correspond if the photo conversion efficiency is 10%.
Photo conversion efficiency is the probability of a 4itE 4itx4.87xl.6x10-19
photon being successful is knocking out an electron. => N = 8.12 x 1012 photon sec-1
(d) Find the cut-off potential. Since, 10% of these photons are able to eject electrons.
Solution => i=0.lxNxl.6xl0-19 A=65nA
(a) Energy required to just eject a photoelectron from the
(d) K= =E-W=(4.87-2.22) eV=2.65 eV
potassium surface is equal to the work function, i.e.,
2.22 eV = 3.552 x 10-19 J => Stopping potential = 2.65 V
As per the classical theory, energy flow is a continuous
process and a photoelectron will be ejected if a Problem 2
potassium atom receives this amount of energy over a
In a photocell the plates P and Q have a separation
length of time.
of 10 cm , which are connected through a galvanometer
The potassium atom is at a distance of 1 m from the
without any cell. Bi-chromatic light of wavelengths
source. Hence, intensity of ultraviolet radiation on the
potassium surface is, 4000 A and 6000 A are incident on plate Q whose work
02 function is 2.39 eV . If a uniform magnetic field B exists
I=_J:_=~= · wattm-'
4itr 2
4it(1) 2
4it parallel to the plates, find the minimum value of B for
which the galvanometer shows zero deflection.
Cross-sectional area of the potassium atom is,
2
Solution
A=it{2x10-10 ) =4itx10-20 m 2 Energy of photons corresponding to light of wavelength
Hence, the exposure time is given by, ,., = 4000 A is

3.15 =
www.puucho.com
Rahul Sardana with www.puucho.com

Advanced JEE Physics Optics & Modern Physics

£i =·12375 =3.1 eV Solution


4000 (a) Intensity of light at a distance 0.8 m from the source
and that corresponding to A2 = 6000 A is, = (3.2 x 10-" Js·1 ) = x 104 watt
1 4
4it(0.8)2 m 2 m'
E = 12375 =2.06 eV
2
"6000 Energy incident on the metallic sphere in wtlt time is
Given that the work function for the metal is W = 2.39 eV, E =1t(8xl0·') (4x104 )=8.04x10-' W
2
1
so we observe that
Energy of one single photon is
E,<Wand£i>W
E2 = 5xl.6 x10·19 J = 8x10·19 J
Hence photoelectric emission is possible with A1 only.
Therefore, total number of photons incident on the
Photoelectrons experience magnetic force and move along a
sphere per second is
circular path.
n = 5_ 8.04 X lQ-' = 1011
1
E2 8x10-19
Since, the efficiency of photoelectric emission is one for
every 10 6 • Hence, total number of photoelectrons per
second is
n
2
= 1 6 = 10116 = 10s
10 10
The galvanometer will indicate zero deflection when the (b) Maxi":um kinetic energy of photoelectrons
photoelectrons complete semi-circular path before reaching K==E-W=2eV
the plate P.
So, stopping potential V0 = 2 V
Thus, d=2r =10 cm
=> r=5cm=0.05m · Total positive charge on·the sphere after time t is,

mv .J2Km
q = (n2 e) t = (10 5 )(1.6 x 10·")t
Further r = - = - -
Bq Bq => q=(l.6x10·")t

=> B . = .J2Km Potential on the sphere V = _q_


- rq 4ni:; 0r

where K=E1 -W=(3.1-2.39)=0.71 eV Photoemission will stop when this potential becomes
the stopping potential, so
Substituting the values, we have
1 {1.6x 10·14 ) t
B. =·.J2x0.71xl.6xl0-19 x9.109x10"1 2
41tE 0 r
- (0.05)(1.6x10-19 )
(9 X 10 9 Hl.6 X 10·14 )t
=> B""' = 5.68 x 10·5 Tesla 2
(BxlO")
=> 1=111 s
Problem 3
In a photoelectric effect setup, a point source of light
Problem 4
of power 3.2 x 10-3 W emits mono energetic photons of
energy 5 eV. The source is located at a distance of 0.8 m When photons of energy 4.25 eV strike the surface of
from the centre of a stationary metallic sphere of work a metal A, the ejected photoelectrons have maximum
function 3 eV and of radius 8 x 10-3 • The efficiency of kinetic energy, TA expressed in eV and de-Broglie
photoelectron ·emission is one for every 10 6 incident wavelength A. A • The maximum kinetic energy of
photons. Sphere is initially neutral and that the photoelectrons liberated from another metal B by photons
photoelectrons are instantly swept away after emission. of energy 4.70 eV is T8 =(T. -1.50 eV). H the de-Broglie
(a) Calculate the number of photoelectrons emitted per wavelength of these photoelectrons is A. 8 = 2AA, then find
second.
(a) the work function w. of metal A and the work
(b) It is observed that the photoelectrons emission stops
at a certain time t after the light source is switched function W8 of metal B .
on. Evaluate time t. (b) the maximum kinetic energy TA of the electrons
ejected from metal A.

= 3.16 :::;:=====================================
www.puucho.com
Rahul Sardana with www.puucho.com

Dttal Nahtre of Radiation & Matter

Solution 3.6x10.., Wm-' equally distributed amongst the three


K==E-W wavelengths. The beam falls normally on an area 1 cm2 of
Therefore, TA ~ 4.25- WA ... (1) a clean metallic surface of work function 2.3 eV . Assume
that there is no loss of light by reflection and that each
T, =(TA -1.50)=4.70-W, ... (2) energetically capable photon ejects one electron. Calculate
the number of photoelectrons.liberated in two seconds.
Equations _(1) and (2) gives,
W,-W, =1.95 eV ... (3) Solution

de-Broglie wavelength is given by .


Smce, E('meV) = 12375
( •)
1- inA
h
1-=~-
..J2Km Energy of photon having wavelength 4144 A
1 12375
=> 1-cc- K = KE of electron E1 = eV = 2.99 eV
4144
.fi.
Similarly,
=> ~: = ffe,. E, =
12375
4972
.
eV = 2.49 eV and

2=~. TA 12375
=> E, = eV =1.99 eV
TA -1.5 6216
=> TA =2 eV Since, only E, and E, are greater than the work function
From equation (1 ), we get W = 2.3 eV, only first two wavelengths are capable for
ejecting photoelectrons. · Given intensity is equally
W, =4.25-TA =2.25 eV
distributed in all wavelengths. Therefore, intensity
From equation (3), we get co'rresponding to each wavelength~
W, = WA + 1.95 eV = (2.25 + 1.95) 3.6x10."' 1.2x10"' w~-2
=> W, =4.20 eV 3
or energy incident, per second in the given area
=> T,=4.70-W,=4.70-4.20=0.50 eV
(A=l cm' =104 m') is
I =1.2x10"' xl0 4 =,1.2xl0-7 Js-1
Problem 5
Let '1t be the number of photons incident per unit time in
Ultraviolet light of wavelengths 800 A and 700 A
when allowed to fall on hydrogen atoms in their ground the given area corresponding to first wavelength. Then,
state is found to liberate electrons with kinetic energy I 1.2xl0-7
n1 = - = - - - - - - 2.5x10 11
1.8 eV and 4.0 eV respectively. Find the value of Planck's E1 19
2.99xl.6xl0-
constant.
Similarly,
Solution
I ' 1.2x10-'
When 800 A wavelength falls on hydrogen atom (in n, 3x1011
E, 2.49 X 1.6 X 10-!9
ground state) 13.6 eV energy is used in liberating the
electron. The rest is given as kinetic energy to the electron. Since, each energetically capable photon ejects electron,
hence total number of photoelectrons liberated in 2 s is
Hence, K=E-13.6(ineV)
N=t(11t+n,)
(1.8xl.6x10-") he 13.6xl.6x10-" ... (1)
B00xl0- 10
=> N =2(n1 +n,)
Similarly for the second wavelength, we. have => N =2(2.5+3)x10 11
(4xl.6x10-19 ) he 13.6xl.6xl0-19 ... (2) => N=l.lxl0 11
. 700x10-10
Solving these two eqtiations, we get
Problem 7
h=6.6x10""' Js A monochromatic point source S radiating
wavelength 6000 A , wlth power 2 watt, an aperture A of
Problem 6 diameter 0.1 m and ·a large screen SC are placed as shown
A beam of light has three wavelengths 4144 A, in figure. A photoemissive detector D of surface area
4972 A and 6216 A with a total intensity of 0.5 cm2 is placed at the centre of the screen. The efficiency

================================== 3.17 =
www.puucho.com
Rahul Sardana with www.puucho.com

Advanced JEE Plzysics OpHcs & Modern Plzysics

of the detector for the photoelectron generation per => n3 = 1.052 x 1016 s-1
incident photon is 0.9.
This aperture ~ become new source of light.
SC
Now these photons are further distributed in all
A. directions. Hence, at the location of the detector,
photons incident per unit area per unit time is
L ,~----------'- " n
4
= n3
2
1.052x 1016
2
41t(6-0.6) 4,r(S.4)
0.6m => n4~=2.87x10 13 m-2s-1
6m _ _ _., This is the photon flux at the centre of the screen.

(a) Calculate the photon flux at the centre of the screen Area of detector is 0.5 cm2 or 0.5 x 10-4 m 2 . Therefore,
and the photocurrent in the detector. total number of photons incident on the detector per
unit time is
(b) If the concave lens L of focal length 0._6 m is inserted
in the aperture as shown, find the new values of n, = (0.5x10-<)(2.87xl013 d) = l.435x10' s-1
photon flux and photocurrent. Assume a uniform The efficiency of photoelectron generation is 0.9. Hence,
average transmission of 80% from the lens. total photoelectrons generated per unit time is
(c) If the work function of the photoemissive surface is
n, = 0.9n5 = 1.2915 x 10' s-1
1 eV , calculate the values of the stopping potential in
the two cases (without and with the lens in the Hence, photocurrent in the detector is
aperture). i = (e)n, = {l.6xl0-19 ){1.2915x109 ) = 2.07x10-10 A
Solution
(b) Using the lens formula, we get
(a) Energy of one photon,
1 1 1
----=--
E he (6.6x10-34 )(3x10') v --0.6 --0.6
A. 6000 X 10-tO => V =--0.3 m
=> E=3.3x10-1' J i.e., image of source (say S', is formed at 0.3 rn) from
the lens.

s s·
'' L
'''
·'' 0.3m

Screen ----5.7 m----+1


Total number of photons incident per unit time on the
Power of the source is 2 W = 2 Js-1 . Therefore, number lens are still n3 or 1.052xl016 s-1 • Since, 80% of it
of photons emitting per second is
transmits to second medium, therefore, at a distance of
2 5.7 m from S' number of photons incident per unit
= 6.06 X 1018 S-I
nl 3.3 X lQ-19 area per unit time will be
At distance 0.6 m, number of photons incident per unit 1
area per unit time is ( ~)(1.os2x10
100
')

n, = (4,r)(S.7)2
n, 2
4>,(0.6) => n, =2.06x1013 m-2 s-1
Area of aperture is,
This is the photon flux at the detector.
2
2
S1 =~d =~(0.1) =7.85x10°" m 2 New value of photocurrent is given by
4 4
i' = (2.06 x10 1')(0.s x 10-< )(0.9)(1.6 x 10-1' )
So, total number of photons incident per unit time on
the aperture, => i' =1.483 X 10-10 A
n, =n2 S1 =(1.34x10 18 )(7.85x10°") s-1 (c) Energy of incident photons (in both the cases) is

~ 3.18 =================================

www.puucho.com
Rahul Sardana with www.puucho.com

Dual Nature of Radiation & Matter

E(in eV) 12375 Problem 9


,.(in A) When a beam of 10.6 eV photons of intensity
1237 2 wm-1 falls on a platinum surface of area lxl0-1 m 2 and
=> E = ~ = 2.06 eV
6000A work function 5.6 eV. 0.53% of the incident photons eject
Work function W = 1 eV photoelectrons. Find the number of photoelectrons emitted
per second and their minimum and maximum energies
Maximum kinetic energy of photoelectrons in both
cases, (in eV). Take 1 eV =1.6 x10·19 J.
K_ =E-W=l.06 eV Solution
or the stopping potential will be 1.06 V . Energy of incident photons,
E; =10:6 eV
Problem 8 => E; = 10.6 X 1.6 X 10-19 J
Assume that the de·Broglie wave associated with an => E; =16.96 x 10-19 J
electron can form a standing wave between the atoms
arranged in a one dimensional array with nodes at each of Energy incident per unit area per unit time (intensity)= 2 J
the atomic sites. It is found that one such standing wave is So, number of photons incident on unit area in unit time is
formed if the distance d between the atoms of the array is
n 2
2 A . A similar standing wave is again formed if d is 1.18xl018
A 16.96 X 10-!9
increased to 2.5 A but not for any intermediate value of d. Therefore, number of photons incident per unit time on
Find the energy of the electron in e V and the least value of
given area (1 x 10-4 m 2 ) is
d for which the standing wave of the type described above
can form. "' n = (1.18x lO")(lxl0-1)
Solution => n =1.18x1014
From the figure it is clear that But only 0.53 % of incident photons emit photoelectrons, so
1+---2A--+
number of photoelectrons emitted per second (n) is
So, p{~)=2A and
N~ N
,. .
(p+l)·-=2.5A p-loops
n = (0.53)(1.18xl0")
100
2 => n = 6.25 X 1011

!:'.=(2.5-2) A=o.sA
K.,., =0
=> (p+1) loops
2 and Kinax = Ei - W
,__ _ 2.s A - - -
=> '-=1A=10-" m => K_ =(10.6-5.6) eV =5 eV
de-Broglie wavelength is given by
=> K-=5eV
,_='!_=_h_
and Kmm =0
p ../2mK

,,,
where K is the kinetic energy of electron
Problem 10
=> K=-- A small plate of a metal having work function of
2mA2
1.17 eV is placed at a distance of 2 m from a
2
(6.63 X 10·" ) monochromatic light source of wave length 4.8 x 10-7 m
=> K
2(9.1x10-31 )(10-rn)' and power 1 W. The light falls normally on the plate. Find
the number of photons striking the metal plate per
=> K = 2.415 x 10-17 J m 2 per sec . If a constant · uniform magnetic field of
=> K =(2.415x10·") eV strength 10-1 T is applied parallel to the metal surface,
1.6x10·" find the radius of the largest circular path followed by the
=> K=150.8 eV emitted photo electrons. Given /1 = 6.6 x 10""' Js,
6 1 19
The least value of d will be, when only one loop is formed. c=3x10 ms- , e=l.6x10- C and electron mass
So, we have

d. =-
,. m=9.lx10"31 kg.
Solution
= 2
Energy of photons of wavelength 4.8 x 10-19 J is
=> dmin =o.5 A
3.19 =
www.puucho.com
Rahul Sardana with www.puucho.com

Advanced JEE Physics Optics & Modern Physics

6.6xl0-" x3xl0' (b) the magnitude of the electric field between the plates
E he 4.125xl0-19 J
;, 4.8x10-7 A and Bat t=l0s and
(c) The kinetic energy of the most energetic
Power of source = 1 W = 1 Js-1
photoelectrons emitted at t = 10 s when it reaches
So1 rate of emission of photons from the source is plate B.
1 J5 -1 Neglect the time taken by the photoelectron to reach
n 2.424x1018 s-1
19
4.125 X 10- J plate B.
These photons move in all directions randomly. At a (Take e0 =8.85x10-u C'N-m-2 )
distance r from the source, the photons fall normally over a
Solution
spherical surface of area 41tr 2 • The plate is at a distance
r = 2 m . Hence the number of photons striking the surface Area of plates A= 5 x 10 4 m 2
per m 2 per second is Distance between the plates d = 1 cm = 10-2 m
(a) Number of photoelectrons emitted upto t = 10 s are
n 2.424xl0"' 4.82xl0"
4x3.14x(2) number of photons falling) ( . )
( in unit area in unit time x area x time
The maximum KE of a photoelectron emitted from the plate n=
is 10'

Kmax=T-Wo
he ~ n =~[(10) x(Sx10 4 )x(10)]
16

10
~ K- = 4.125 X 10-19 -1.17 X 1.6 X 10-19 = 2.253 X 10:,, J ~ n=5x107
Hence the maximum velocity of the photoelectron is (b) At time I= 10 s , charge on plate A
q, =+ne=(5x10')(1.6x10-19 )
V = /2IC:
V~.
max o q, =8x10-12 C

-
19
2 X 2.153 X 10- = 7_03 X lQS ffiS-t and charge on plate B,
~ V =
9.lxl0-31 q8 = (33.7x10-12 -8x10-12 )
Radius of the largest circular path of the photoelectrons in
~ q8 =25.7x10-12 C
the magnetic field is
mVm~ 9.1 X 10-31 X 7.036 X 105 Since, electric field between the plates is
r = - - = - - - - ~19- ~ -4 ' - 4xl0-' m = 4 cm
eB 1.6 x 10- x 10- E (q,-q,)
2Ae,
Problem 11 (25.7 -8) X 10-t2
~ E ---,--C----c-c---= 2x103 NC-1
T~o metallic plates A and B each of area 2 X (5 X 10-4 )(8.85 X 10-t2)
5 x 10-4 m 2 , are placed parallel to each other at separation (c) Energy of photoelectrons at plate A is
of 1 cm. Plate B carries a positive charge of 33.7 x 10-12 C.
K=E-W=(S-2) eV=3eV
A monochromatic beam of light, with photons of energy
Increase in energy of photoelectrons is
5 eV each, starts falling on place A at t = 0 so that
106 photons fall on it per square meter per second. (eEd) joule= (Ed) eV
Assume that the photoelectrort is emitted for every 10 6 ~ (!ncreaseinEnergy)=(2x10')(10-2 ) eV=20 eV
incident photons. Also assume that all the· emitted
photoelectrons are collected by plate B and the work Energy of photoelectrons at plate B is
function of plate A remains constant at the value 2 eV . (20+3) eV =23 eV
(a) the number of photoelectrons emitted up to t = 10 s

3.20

www.puucho.com
Rahul Sardana with www.puucho.com

This section contains Single Correct Choice Type Questions. Each question has four choices (A), (B), (C) and (D), out of which
ONLY ONE is correct.

1. Consider a source emitting 100 W of green light at a 5. Which of the following graphs represents the variation
wavelength of 500 nm . The number of photons of particle momentum and the associated de Broglie
emerging from source per second is wavelength?
(A) 2.5 x 1019 photon per second
p p
(B) 25 x 10 20 photon per second

2.
(C) 25 x 10 19 photon per second
(D) 25 x 1017 photon per second

The distance d of a 100 W lamp is continuously


increased from a photocell. The photoelectric current I
(A) IL_, 0) ~ >

L. ~,LL.
varies with distance d as p p
(A) I a: d2 (B) I a: d
1 1
(C) I a: d' (D) I a:-
d
(Q
3. When a metallic surface is illuminated with
monochromatic· light of wavelength 1,,, the stopping
potential is 5V0 • When the same surface is illuminated 6. If Planck's constant is denoted by /z and the electronic
with light of wavelength 31,, , the stopping potential is charge by e, experiments on photoelectric effect allow
V0 . Then the work function of the metallic surface is the determination of
(A) only h (B) only e
(A) he (B) ~
61,, 51,, h
(C) both h and e (D) only -
he 2hc e
(C) (D)
41,, 41,,
7. The maximum energy of the electrons released in a
photocell is independent of
4. An electron of mass m , when accelerated through a
(A) frequency of incident light
potential difference V has de-Broglie wavelength A..
(B) intensity of incident light
The de-Broglie wavelength associated with a proton of
(C) nature of cathode rays
mass M accelerated through the same potential (D) None of these
difference will be

(A) 1..(:) (B)


8. If the energy and wavelength of electron are E and A,
then the graph between logE and log),, will be

(C) 1..( ~) (D)

3.21

www.puucho.com
Rahul Sardana with www.puucho.com

Advanced JEE Physics Optics & Modern Physics

log l. log l. 2x10·' T. If(;.) of electron is 1.76xl0 11


Ckg·1 , then
the diameter of the circle is nearly
(A) 1.1 m (B) 1.1 mm
(A).·~ e, (C) 1.1 cm (D) 11 cm

14. The radiation emitted, when an electron jumps from


log E n = 3 to n = 2 orbit is a hydrogen atom, falls on a metal
log E
to produce photoelectron. 'IJ1e electrons from the metal
log l. log l. surface with maximum kinetic energy are made to
move perpendicular to a magnetic field of _!_ T in a
· 320
radius of 10-3 m. The work function of metal is
(D)
(A) 1.03 eV (B) 1.89 eV
(C) 0.86 eV (D) 2.03 eV

log E log E 15. The radiation force experienced by a body exposed to


radiation of intensity I assuming surface of body to be
9. The total energy E of a sub-atomic particle of rest mass

-
perfectly absorbing is

·=~
mo moving at non-relativistic speed v is
(A) E=m0c2 (B) E= 1 m0 v'
2
(C) E = m0 c' +-m
1 0V 2
2
1
(D) E = 1"0C2 -2mov2

10. An electron is 2000 times lighter than a proton. Both are


(A)
1tR'I .
C
- (B)
27tR 2I
C
moving such that their matter waves have a length of 41tR'I
(C) (D) None of these
1 A . The ratio of their kinetic energy in approximation C

is
(A) 1 :1 (B) 1: 2000 16. A radiation of energy E falls normally on a perfectly
(C) 2000 :1 (D) 1:200 reflecting surface. The change in momentum of
radiation is
11. The de Broglie wavelength of a particle is 2E
(A) E (B)
approximately the same as that of a photon with the C C
same energy.
(A) The energy of the particle is much greater than its (C) Ee (D) ~
rest energy.
(B) The energy of the particle is much less than its rest 17. An electron, accelerated by a potential difference V,
energy.
(C) The energy of the particle equals its rest energy.
has de Broglie wavelength ).. . If
the electron is
accelerated by a potential difference 4V, its de Broglie
(D) Data insufficient to arrive at a conclusion.
wavelength will be
(A) 2).. (B) 4)..
12. Two lumps of clay each of rest mass m0 , collide with a
4 (C) 2:. (D) A.
speed of -c head on and stick together. The mass of 2 4
5
the composite lump thus formed is 18. The photoelectric work function of a.metal is 1 eV. Light
10 5
(A) m, (B) m, of wavelength ).. = 3000 A falls on it. The
3 3 photoelectrons will come out with approximate speed
5
(D) -m equal to
12 ° (A) 10 ms·1 (B) 10 2 ms·1
4
(C) 10 ms· 1
(D) · 106 ms·1
13. An electron moving with velocity 2x107 ms-1 ·

describes·. a circle in a magnetic field of strength

= 3.22 c=====================================
www.puucho.com
Rahul Sardana with www.puucho.com

Dttal Nahtre of Radiation & Matter


19. A proton and an a -particle are accelerated through the
same potential difference. The ratio of their de Broglie
(C)
2(.!!E..-•')
w, 8,rn0d
wavelengths is (D)
n hceW,
1
(A) .fi. (B) .fi.
26. Aperfectly reflecting body sphere in shape of radius R
(C) 2.fi. (D) 2 is placed on a path of parallel light beam of intensity I
shown in figure. One end of a spring is attached to
20. In an electron microscope if the potential is increased
centre of sphere and the other end to a rigid wall as
from 20 kV to 80 kV, the resolving power R of the shown in figure. Assuming the sphere to be in
microscope will become
equilibrium, the spring constant of spring to be K , then
(A) R (B) 2R compression in spring is

=. r-~
(C) 4R (D) R
2

21. Boron has two- isotopes 5 B10 and 5B11 . If the atomic
weight of boron is 10.81, the ratio of 5 B10 to 5
B11 in ""'~~~---"-------"'
nature is (Neglect any thermal effect and friction is absent)
(A) 19 20 (A) E_nR2 • (B) _I_nR'
(B)
81 53 KC KC
(C) 15 (D) 10 faR'
(C) C (D) None of these
10 11

22. In photoelectric emission the number of electrons 27. The de-Broglie wave present in the fifth Bohr orbit is
ejected per second is proportional to the
(A) I \ .f\
(A) intensity of light
(B) wavelength oflight v
(C) frequency of light
.(D) work function of the material
I\ I\
(B)~

23. For a photon, the de Broglie relation is given by


(C) / \
(A) ,.=.!!__
me
(B)
p
A=!!_ - ~/ \v/~
\

v/ \v/ \~0 v/v


\
(C) ,. -> infinity (D) Data Insufficient .(D) / \

24. A desklamp illuminates a desk top with light of


wavelength ").. . The amplitude of this electromagnetic
28. The interatomic distance between atoms in a crystal is
wave is E0 • Assuming illumination to be noi-mally o~
2.8-A, Then if such a crystal is used in Davisson-
the surface, the number of photons striking the desk per Germer experiment, the maximum order of diffraction
second per unit area N is that can be observed for a beam of electrons accelerated
(A) N =J.-e,E! (B).
N = 2J.-eh0E! by 100 V shall be
h (A) n=l (B) n=2
(C) N = f...t 0 E! . (C) n=10 '(D) n->oo
(D) Data Insufficient
2h
29. The energy of a photon of wavelength J.- is
25. A small metal plate (work-function W0 ) is kept at a (A) he").. (B)
he
distance d from a singly ionised fixed ion. A J.-
monochromatic light beam is incident on the metal° J.- (D) hJ.-
plate and photoelectrons are emitted. The maximum (C) he c
wavelength of light so that the photo-electrons may go
round the ions along a circle is 30. The work function of a metal in 4 eV. Foi- the emission
81te0 W0 d+e2 8nhce0 d of photoelectrons of zero velocity from the metal
(A) (B) surface, the wavelength of the incident radiation should
8nhce0d
be

3.23 =
www.puucho.com
Rahul Sardana with www.puucho.com

Advanced JEE Physics Optics &.Modern Physics

(A) 1700A
(CJ 3100A
(B). 2700 A
(DJ 5900 A
(C) less than vl//4
3
(D) greater than v l
38. The de-Broglie wavelength of a molecule of thermal
31. Photoelectric effect is the phenomenon in which
energy k,T ( k, =Boltzmann constant and T = absolute
(A) photons come out of a metal when it is hit by· a
beam of electrons temperature), is
·(B) photons come out of the nucleus of an atom·under h
.(A) l ~ (B)
the action of an electric field. = 2mk8T ~2mk8T
(C) electrons come out of a metal with a constant
It
velocity which depends on the frequency and (C) h~2mk8T (D)
intellSity of incident radiation . 4m 2 kiT 2
(D) electrons come out of a metal with different
velocities not greater than a certain value which 39. A proton is accelerated through a potential V . The de
depends only on the frequency of the incident light Broglie wavelength associated with it is
and not On its inte_nsity. (A) 12.27 A (B) 0.287 A
.Jv .Jv
32. The photoelectric effect is the ejection of electrons from
the surface of a metal when
(C) 12.27 frn (D) 0.287 frn
(A) it is heated to a high temperature. .Jv .Jv
(B) electrons of suitable velocity strike it.
(C) radiation of suitable wavelength falls on it. 40. In Davisson-Germer experiment Ni crystal acts as
(D) it is placed in a strong electric field. (A) an ideal reflector
(B) three dimensional diffraction grating
33. Matter waves are (C) an ideal absorber
(A) electromagnetic waves. · (D) two dimensional diffraction grating
(B) transverse mechanical waves.
(C) longitudinal mechanical waves. 41. The angle between the incident and the diffracted
(D) neither electromagnetic nor mechanical waves. electron in the Davisson-:Germer,experiment is called as
(A) angle of incidence
(B) angle of diffraction
34. A photon of wavelength 1000 A has energy 12.3 eV. If
(C) angle of scattering
light of wavelength 5000 A , having intensity I., falls (D) none of the above
on a metal surface, the saturation current is 0.40 µA
· · 42. In Davisson-Germer experiment maximum intens_ity is
and· the stopping potential is 1.36 V. The work function
observed at "-,
of the metal is
(A) 2.47eV (B) 1.36 eV (A) ·50° and 54 volt (B) 54° and 50 volt
(C) 1.10 eV (D) 0.43 eV (C) 50° and 50 volt (D) 65° and 50 volt

35. In PROBLEM 34, if the intensity of light is made 4I, the 43. In Davisson-Germer experiment, an electron beam of
stopping potential will become 60 eV energy falls normally to the surface of the crystal
(A) 1.36 V (B) 2.72 V and maximum intensity is obtained at an angle of 60'
(C) 5.44 V . (D) 21.76 V to the direction of incident beam. The inter-atomic
distance in the lattice plane of the crystal is
36. In PROBLEM 34, if the intensity of light is made 4I , the (A) 18 A (B) 3.6 A
saturation current will become (C) 1:8 A (D) 0.18 A
(A) 0.4 µA (B) 0.8 µA
(C) 1.6 µA (D) 6.4 µA 44. The incorrect statement in connection with Davisson
and Germer experiment is · .
37. In a photoemissive cell with exciting wavelength l, the (A) The inter-atomic distance in nickel crystal is of the
fastest electron has a speed v . If the exciting order of the de-Broglie wavelength.
3 (B) Electrons of constant energy are obtained by the
wavelength is changed to l , the speed of the fastest electron gun.
4 . .
(C) Nickel crystal acts · as a three dimensional
emitted electrons will be
diffracting grating.
. /3 (D) Davisson-Germei- experiment is an interference
(A) v\{4
experiment.

= 3.24

www.puucho.com
Rahul Sardana with www.puucho.com

Dual Nature of Radiation & Matter

45. In Davisson-Germer experiment the relation between 2.5 eV . The stopping potential for radiation of
Bragg's angle ~ and diffraction angle 0 is wavelength 1500 A will be
90°-$ (A) 2.5V
(A) 0=90'-$ (B) 0=--
2 (B) 5.0 V
(C) 0=180°-$ (D) ~~ --(1802'-0.) (C) less than 5.0 V but more than 2.5 V
(D) more than 5.0 V .

46. The ionization chamber used is Davisson-Germer 53. The de Broglie wavelength of a particle of mass m
experiment, acts as moving with a kinetic energy E is

~
(A) emitter (B) collector
(C) source (D) radiator (A) (B) ..J:mE
47. The distance between two consecutive atoms of the (C) h (D) .Jij
2mE 2mE
crystal lattice is 1.227 A. The maximum order of
diffraction of electrons accelerated through 10 4 V will 54. Two· photons of energy 2.5 eV each are incident on a
be metal plate whose work function is 4.0 eV, then the
1 number of electrons emitted from the metal surface will
(A) 10 (B) 10
be
1 (A) one (B) two
(C) 100 (D)
100 (C) None of these (D) more than two

48. The human eye can barely detect a yellow light 55. Of the following moving with same momentum, the
(6000 A) that delivers 1.7 x 10-" watt to the retina. one which has largest wavelength is
(A) an electron.
Nearly how many photons per second does the retina
(B) a proton.
receive?
(A) 50 (C) an CI-particle.
(B) 5
(D) all have same de-Broglie wavelength.
(C) 500 (D) More than 5 million
56. The maximum velocity of an electron emitted by light
49. The ratio of the specific charge of a proton to that of an
of wavelength ). incident on the surface of a metal of
CI-particle is
._ work-function$ is
(A) 1:4 (B) 1:2
(C) 4 :1 (D) 2:1 (A) 2(hc+1'.$) (B) 2{hc-1'.$)
m). m
SO. A photosensitive surface is receiving light of (C) 2(h:~).$) (D) J2(h~-$)
wavelength 5000 A at the rate of 10-7 Js-1 • The number
of photons received pe,r second is where h =Planck's constant, m = mass of electron and·
(A) 2.5x10 12 (B) 2.Sxl011 c = speed of light ·
(C) 2.5x10 10 (D) 2.Sx.109
57. An CI-particle is accelerated through a potential
51. Photons of frequency v fall on a metal surface for difference of 200 V. The increase in its kinetic energy in
electron volt will be ·
which the threshold frequency is v O • Then,
(A) 100 eV (B) 200 eV
(A) all ejected electrons have the same kinetic energy (C) 400 eV (D) 800 eV
h(v-v,).
(B) the ejected electrons have a distribution of kinetic 58. Figure represents the graph of photo current i versus
energy from zero to h ( v - v0 ) • applied voltage ( V) . The maximum energy of the
(C) the most energetic electrons have kinetic energy emitted photoelectrons is
hv
(D) the average kinetic energy of ejected electrons is
hv0 •

52. When radiation of wavelength 3000 A is incident on a


photosensitive ~urface, the kinetic energy of electrons is -4 -3 -2 -1 0 1 2 3 4
~
==================================== = 3.25

www.puucho.com
Rahul Sardana with www.puucho.com

Advanced JEE Physics Optics & Modern Physics

(A) 2eV (B) 4 eV nR2 I l.7itR 2I


(C) 0eV (D) 4J (A) (B)
C C

0.3itR2I
59. Which one of the following graphs represents correctly (C) - - - (D) None of these
C
the variation of photoelectric current (i) with intensity
( I) of incident radiations 65. Two electrons are moving with the same speed v . One

Jt_ '"' iL
electron enters a region of uniform electric field while
the other enters a region of uniform magnetic field, then
after sometime if the de-Broglie wavelengths of the two
are 11. 1 and A. 2 , then
(A)
(A) 1.1 = 1. 2 (B) 1.1 > 1. 2
Intensity (!) Intensity (I)
(C) 1.1 < 1.2 (D) 1.1 > i.2 or 1.1 < i.2

(C) ~lL_
~ '
~
()

Intensity (!)
~
(D) ~r=_-················
~
()

Intensity (I)
66. An electromagnetic radiation of wavelength i. has the
same moffientum as an electron moving with a speed
2x10 5 ms-1 .
(A) 1. = 2.64 nm (B) 1.=1.64 nm
(C) i. = 3.64 nm (D) 1. = 4.64 nm
60. The potential difference between the cathode and anode
in a cathode ray tube is V . Then the speed acquired by 67. Let K1 be the maximum kinetic energy of
the electrons is proportional to· photoelectrons emitted by light of wavelength 1.1 and
(A) V (B) .Jv K2 corresponding to wavelength 1.2 • If 1.1 = 21.2 then
(C) V2 (D) V¥2 (A) 2K1 = K, (B) K1 = 2K2

61. The duration of a laser pulse is 10_. s. The uncertainty (C) K1 < :, (D) K; > 2K2
in its energy will be
(A) 6.6 x 10·" J (B) 6.6 x 10·" J
68. Solar constant of the sun is cr=8.106xl0 4 Jmm·1m·2
(C) 6.6 X 10--U J (D) _!_xl0 26
J and average Sun earth. distance is 1.5 X 10 8 km , The
6.6
yearly loss in the mass of the sun is
62. Which one of the following statements about photons is -(A) 13.8xl017 kg (B) 1.38x1019 kg
incorrect? (C) 1.38xl017 kg (D) 13.8xl0 20 kg
(A) Rest mass of a photon is zero
(B) Momentum of a photon of frequency v is hv 69. A photon strikes a free electron at rest and is scattered
C straight backward. if the speed of electron after collision
(C) Energy of a photon of frequency v is hv is ac , where a«: 1 then,
(D) Photons exert no pressure (A) electron's kinetic energy is a fraction a of
photon's initial energy.
63. A parallel beam of uniform, monochromatic light of 1
wavelength 2640 A has an intensity of (B) electron's kinetic energy is a fraction of
a
100 wm·2 • The number of photons in 1 mm' of this photon's initial energy.
radiation are , (C) electron s kinetic energy is a fraction a' of
1

(A) 222 (B). 335 photon's initial energy.


(C) 442 (D) 555 1
1
(D) electron s kinetic energy is a fraction of
64. Consider a sphere of radius
R exposed to radiation of
-- photon's initial energy.
a'

intensity I as shown in
figure. If surface of sphere
is partially reflection and
reflection coefficient is 0.3,
then radiation force
--
T
70. Which one of the following does not fit into the group?

71..
(A) Photon
(C) Proton
(B) Graviton
(D) Meson

Two photons approach each other. The relative velocity


experienced is
of approach is

= 3.26 i::::=================================
www.puucho.com
Rahul Sardana with www.puucho.com

Dual.Nahtre of Radiation & Matter


Plate
(A) .:. (B) C
2
----+ l--'mKirJl'_ __j
(C) 2c (D) ic ----+

72. A particle of mass 3m at rest decays into two particles


of masses m_ and ·2m having non-zero velocities. The
=: l--'imrK,oo._ _J;f Fixed
---+- K f
ratio of the de-Broglie wavelengths of the particles ----+ L__r== ,'·

(,.,,.,) .
- lS

IA
- - - - - ~ ~ -*'-~~· _.._
Smooth surface
-,.._'--.... ..,

2IA
(A) 1 1 (A) (B)
(B) KC 3KC
4 2
3IA 4IA
(C) 1 (D) 2 (C) (D)
KC 3KC
73. A small ball is projected with initial speed u and at an
angle 8 with horizontal from ground. The de-Broglie 79. The work function of a metallic surface is 5.01 eV.
Photoelectrons are emitted when light of wavelength
wavelength of ball at the moment its velocity vector
becomes perpendicular to initial velocity vector is 2000 A falls on it. The potential difference required to
(A) h . (B) h stop the fastest photoelectrons is
mu musin8 (h=4.14x10-15 eVs}
h (A) 1,2 V (B) 2.4 V
(D)
mucosa (C) 3.6 V (D) 4.8V

74. The force exerted by a photon of intensity 1.4 kWm-2 if 80. A proton, accelerated through a potential difference V
it falls on a perfect absorber of radius 2 m is has a certain de Broglie wavelength. In order to have
(A) 2.35 x 10""' N (B) 10 8 N the same de Broglie wav:elength, an a -particle must be
accelerated through a potential difference
(C) 8.35xl0 4 N (D) 8.8x10 .. N
·(A) 4V (B) 8V
75. Photoelectrons are emitted with maximum kinetic (C) V (D) V
energy E from a metal surface when light of frequency 4 8
v falls on it when light of frequency v' falls on the
same metal, the maximum kinetic energy of emitted 81. Ultraviolet light wavelength 300 nm and intensity
photoelectrons is found to be 2E , then v' is 1.0 wm-' falls on the surface of a photoelectric
(A) v'=v (B) v'=2v material. If one percent of the incident photons produce
~ ~>h • ~ ~ch photo electrons, then the number of photoelectrons
emitted per. second from an area of 1.0 cm2 of the
76. A material particle with a rest :mass m0 is moving with surface is nearly
speed of light c . The de-Broglie wavelength associated (A) 9.61 x 1014 (B) 4.12 X 1013
is given by (C) 1.51 X 1012 (D) 2.13x1011
(A) h (B) m0c
m,c h 82. A proton and an a-particle are injected into a uniform
(C) ZERO· (D) 00 electric field at right angles to the direction of field with
equal kinetic energy. Then
77. Light of two different frequencies whose photons have {A) the proton trajectory will be less curved than a-
energies 1 eV and 2.5 eV successively illuminate a metal particle trajectory.
of work :function 0.5 eV. The ratio of the maximum (B) the a-particle trajectory will be less curved than
speeds of the emitted electrons will be proton trajectory. ·
"(A) 1:5 (B), 1:4 (C) both the trajectories will be equally curved.
(C) 1:2 (D) 1:1 (D) both trajectories will be straight.

78. Light of intensity I is incident normally on a perfectly · 83. de Broglie waves are associated wj.th
reflecting plate of area A kept in a gravity free space. If (A) moving charged particles only.
the photons strike the plate synunetrically and initially (B) moving neutral particles only.
the springs are in their natural lengths, then the (C) all moving particles.
maximum compression in the springs is (D) all particles whether in motion or at rest.

=============================== = 3.27

www.puucho.com
Rahul Sardana with www.puucho.com

Advanced JEE Physics Optics & Modern Physics

84. The maximum kinetic energy (E.) of photoelectrons (D) Planck's constant
varies with the frequency ( v) of the incident radiation
89. A photon of frequency v is incident on a metal surface
as
whose threshold frequency is vO • The maximum kinetic
energy of the emitted electron :will be
(A) h{v-v 0 ) (B) h(v+v,)
1 1
(C) h(v-v0 ) (D)
(A)~• ~) ~ • 2 2h(v+v 0)

90. A stream of photons impinging normally on a


completely absorbing screen in yacuum exerts. a

~
pressure Pl'. If I is the irradiance then,
2
(A) til'= I (B) til'=Ic
C
~• (D)~•
(D) til'=2Ic

85. Stopping potential for photoelectrons 91. An electron and a photon have same wavelength. It p
(A) .does not depend on the frequency of the incident is the momentum of electron and E the energy of
light.
(B) does not depend on the nature of cathode material. photon. The magnitude of f in SI unit is
(C) depends on both the frequency of the incident light
and the nature of the cathode material. (A) 1 (B) 1
(D) depends on the intensity of the incident light. 2c C

2
(C) (D) None of these
86. A particle of mass 10:.,,. kg is moving with a speed of C

10 5 ms-1 • The de Broglie wavelength of the particle is·


92. The maximum kinetic energy of photoelectrons emitted
(A) 6.63 x 10-s m (B) 6.63 A from a surface when photons of energy 6 eV fall on it
(C) 66.3A (D) 6.63 X 10-7 ffi is 4 eV. The stopping potential in volt is
(A) 2 (B) 4
87. A moving i,article is associated with a wave packet' or (C) 6 (D), 10
group of waves. The group velocity is equal to
(A) velocity oflight 93. A sensor is exposed for time I to a lamp o( power P
(B) velocity of sound placed at a distance l . The sensor has an opening that is
(C) velocity of particle 4d in diameter. Assuming all energy of the lamp is
1 given off as light, the number of photons entering the
(D)
particle velocity sensor if the wavelength of light is 1,. is
P1..d 2I 4P1..d2I
{A) N = hcl' (B) N =~
88. The maximum kinetic energy ( E= ) of photoelectrons
emitted in a photoelectric cell varies with frequency (C) N = P1..d't (D) N = PM't
( v) as shown in the graph. The slope of the graph is 4hcl 2 16hcl2
equal to ·
94. When a metallic surface is illuminated by a
monochromatic light of wavelength 1,. , tl1e stopping

lL.
potential for photoelectric current is 3V0 • When the
same surface is illuminated by light of wavelength 21,.,
the stopping potential is V, . The threshold wavelength
v, for this surface for photoelectric effect is
(A) charge of the electron (A) 61.. (B) 41..
3
(B) ~ of the electron (C) 41,. (D) 81..
m
(C) work function of ~e emitter

= ==================================
3.28

www.puucho.com
Rahul Sardana with www.puucho.com

DuafNature of Radiation & Matter


95. The momentum of a photon of an electromagnetic
radiation is 3.3 x 10-" kgms-1 . The frequency of the
associated waves is ( h = 6.6 x 10-34 Js , c = 3 x-108 ms-1 )
(A)
(A) 3:0 x 103 Hz (B) 6.0 x 10 3 Hz
(C) 7.5 X 1012 Hz (D) 1.5 X 1013 Hz

96. If the wavelength of incident radiation in a


photoelectric experiment is decreased theri
(A) the photoelectric current will decrease.
(B) the photoelectric current will increase.
(C) the stopping potential will decrease.
(D) the stopping potential will increase.

97. The threshold wavelength for a photosensitive surface


103. An electron is accelerated through a potential difference
is 6000 A and the wavelength of incident light is of 100 V . Its kinetic energy will be
5000 A . Then the maximum energy of emitted (A) 100 J (B) 100 erg
electrons would be (C) 100 eV (D) 100 MeV
(A) 0.041 eV (B) 0.41 eV
(C) 4.1 eV (D) 41 eV 104. If 5% of the energy supplied to a bulb is radiated as
visible light, the number of visible quanta emitted per
98. An electron of mass m and charge e initially at rest second by a 100 W bulb, assuming the wavelength of
gets accelerated by a constant electric field E . The rate visible lightto be 5.6 x 10-s cm , is
of change of de-Broglie wavelength of this electron at (A) 1.4xl019 (B) 1.4xl0 20
time t , ignoring relativistic effects (C) 2x1019 (D) 2x1020
h eht
(A) - eEt' (B) E
105. A neutron is confined to a nucleus of size 10-14 m . The
(C) - .;;, (D) ! minimum momentum of the electron may be
(A) 6.6x10-20 kgms-1 (B) 3.3x10-"' kgms-1
99. Of the following, the one which has the largest de (C) 3.3x10 4 kgms-1 (D) 6.6x10..., kgms-1
Broglie wavelength for the sarn~ speed is
(A) electron (B) proton 106. The stopping potential ( V, ) versus frequency plot of a
(C) a -particle_ (D) oxygen atom substance is shown in figure The threshold wavelength
is
100. A small photocell is placed at a distance of 4 m from a
V0 volt
photosensitive surface. When light falls. on the surface
the current is 5 mA. If the distance of cell is decreased to 2 -------------------- i
1 m , the current will become
(A) 1.25mA (B) (:6)mA I
(C) 20mA (D) 80 mA 4 5 6 7 8
14
vx10 Hz
101. Einstein's photoelectric equation is E, = hv- ~ . In this (A) 5xl014 m
equation EK refers to (B) 6000 A
(A) kinetic energy of all the emitted electrons.
_(CJ 5000 A
(B) mean kinetic energy of emitted electrons.
(C) maximum kinetic energy of emitted electrons. (D) cannot be estimated from given data
(D) minimum kinetic energy of emitted electrons.
107. The energy of incident photon is 12.375 eV while the
102. In photoelectric effect, the graph showing the variation energy of scattered photon is 9.375 eV . Then the
of cut-off voltage (V0 ) with the frequency of incident kinetic energy of recoil electron is
(A) 3 eV (B) less than 3 eV
radiation ( v) is (C) more than 3 eV (D) 21.75 eV

c::::================================= = 3.29

www.puucho.com
Rahul Sardana with www.puucho.com

Advat1ced JEE Physics Optics & Modem Physics

108. When electrons are accelerated through potential


2
difference of V volt, the de-Broglie wavelength (A) Moc2 (B) M 0c [1-(~J]
associated is given by

(A) ).=t~o A (B) ).=t~o m (CJ M 0 c'(1- ~) (D) M 0 c'[1-(~J]

(CJ ).=~A (D) ).=ffso A 116. The number of red photons {). = 663 nm) that must
V
strike a. totally reflecting screen per second at normal
109. An electron and a proton are accelerated through the incidence so that a force of 1 N is exerted on the screen
same potential. If their masses are me and is
m, respectively, then the ratio of their de Broglie (A) n=5xl0 23 (B) n=5x10 24
wavelength is (C) n=5xl0 25 • (D) t1=5x10 26

(A) 1 (B) ~ 117. A 20 arnu atom emits photon of 6.6 A while making a
transition from excited state to ground state. The recoil
(CJ m,
m, (D) ~ energy of the atom will be
(A) 1.5 x 10·23 J (B) 3.5 x 10-23 J
(CJ 5.lxl0-23 J - (D) 7.5xl0-23 J
110. A particle of mass 1 g is located in a box of size 2 cm.
The uncertainty in the momentum of thE:_ electron will 118. How many red photon (wavelength ). ) must strike a
be totally reflecting screen per second at normal incidence,
(A) 3.3 x 10""2 kgms' (B) 6.6 x 10-02 kgms' if the exerted force is to be 1 N ?
(CJ 3.3 X 10-33 kgms·t (D) 6.6 X 10->1 kgms·l (A) !: (B) 2).
h h
111. If E1 , E2 , E3 are the respective kinetic energies of an ).
(CJ 2h (D) infinity
electron, an alpha-particle and a proton, each having
the same de-Broglie wavelength, then
(A) E, >E, >E, (B) E, >E3 >E, 119. The surface of a metal is illuminated with the light of
~ E,>E,>E, ~ E,=E,=E, 400 nm . The kinetic energy of the ejected
photoelectrons was found to be, 1.68 eV. The work
112. The kinetic energy of electron is E, when the incident function of the metal for he= 1240 eV-nm is
light has wavelength ). . To increase the K.E. to 2E, the (A) 3.09 eV (B) 1.41 eV
incident light must have wavelength (C) 1.51 eV (D) 1.68 eV

(A) he (B) he).


£).-he &+he 120. A cathode of a photoelectric cell is changed such that
the work function changes from W1 to W, ( 1'½ < W2 ) •
(C) h). (D) he).
E). +he &-he If the current before and after changes are 11 and 12 all
other conditions remaining unchanged, then (assuming
113. We wish to observe an object which is 2.5 A in size. hv>W,)
The minimum energy photon that can be used is (A) I,= I 2 (B)
(A) 5 keV (B) 8 keV (C) I,> I, (D)
(CJ 10 keV (D) 12 keV
121. The momentum of a photon of frequency v is
114. A certain mass of ice at 0° C melts into water at 0° C
and there by gains 1 kg mass. If initial mass of ice is (A) ~~ (B) hev
m0 then, (C) ltvc (D) hve2
(A) m 0 = 2.69 x 10 kg
11
(B) m0 =1 kg
122. The work functions for three different metals A, B and
(CJ '"o = 2.69 X 1010 kg (D) m0 =9xl0 16 kg
C are $A, $, and $c respectively with $A>$,> $c-
115. A star of mass M 0 , radius Ro contracts to radius R. The graphs beiween stopping potential (V0 ) and
Energy radiated by the star assuming uniform density frequency v of incident radiation for them would look
in each case while temperature remains unchanged is - like

= 3.30

www.puucho.com
Rahul Sardana with www.puucho.com

Dual Nature of Radiation & Matter

field needed so that the rays describe the same


approximate circular path ( voltmetr~-1 ) is
(A) {A) 2x (B) 3x
(C) 4x (D) 6x

129. Which of the following graphs gives the variation of


photoelectric current (I,) with the voltage (V) applied
to the electrodes of a photo cell?

(C)

123. In order to increase the kinetic energy of ejected


photoelectrons, there should be an increase in
(A) intensity of radiation. ·
(A)
LL, .,r=_,
,(B) wavelength of radiation.
·(CJ frequency of radiation.
(D) both wavelength and intensity of radiation.

124. Which of the following arrangements corresponds to


decreasing order of specific charge?
(A) Electron, proton, a -particle
(C)
Jc_,~, lL_,
(B) Proton, a -particle, electron 130. An electron with speed v and a photon with a speed c
(C) a-particle, electron, proton have the same de-Broglie wavelength. If the K.E. and
(D) Electron, a -particle, proton momentum of electrons is E, and P, and that of photon
is EP11 and Pp1, respectively, then the correct statement
125. A photoelectric cell is illuminated by a small bright
is
source of light placed at 1 m. If the same source of light
is placed 2 m away, the electrons emitted by the (A) s_=2c {B) ~=~
cathode E,, V Eph 2c
(A) each carries one quarter of its previous Pe 2c
(C) - = - (D) None of these
momentum. Pp1, V
(B) each carries one quarter of its previous energy.
(C) are half the previous number. 131. The energy of a photon corresponding to the visible
(D) are one quarter of the previous number.
light of maximum wavelength is approximately
(A) 1 eV (B) 1.6 eV
126. Light of frequency 1.5 times the threshold frequency is
(C) 3.2eV (D) 7eV
incident on a photo-sensitive material. H the frequency
is halved and the intensity is doubled, the photoelectric
132. Planck's work was connected with
current becomes
(A) wave nature of matter
(A) four times (B) double (B) photoelectric effect
(C) half (D) ZERO (C) structure of atom
(D) quantum nature of radiation.
127. The ratio of the de Broglie wavelengths of a proton and
an a-particle will be 1: Z- if their 133. The graph shows stopping potential versus
(A) kinetic energies are in the ratio 1: 8 1
(B) kinetic energies are in the ratio 8 : 1 - - - - for three metals,. then
wavelength
(C) velocities are in the ratio 1: 8
Stopping
(D) velocities are in the ratio 8 : 1 potential 2 3

128. Cathode rays moving with same velocity v describe an


apprqximate circular path of radius r metre in an
electric field of strength x volt metre-' . If the speed of
the cathode rays is doubled to 2v, the value of electric wavelength

================================== 3.31 =
www.puucho.com
Rahul Sardana with www.puucho.com

Advanced JEE Physics Optics & Modern Physics

(A) Planck's constant for metal (1) is greatest (A) 0.48xl0-10 m (B) 0.24 X 10-10 A
(B) Work function for metal (3) is greatest (C) 0.24 X 10-!0 m (D) l.24xl0-10 m
(C) Threshold frequency for metal (1) is greatest
(D) Threshold wavelength is maximum for metal (3) 138. The kinetic energy of the body is twice the rest mass
energy. The ratio of the relativistic mass of the body to
134. The work function of a substance is 4 eV. The longest its rest mass is
wavelength of light that can cause photoelectron (A) 1 (B) 2
emission from this substance is approximately (C) 3 (D) infinite
(A) 540 nm (B) 400 nm
(C) 310 nm (D) 220 nm 139. The number of complete de-Broglie wavelengths
associated with the electron in nth orbit of hydrogen
135. A radio transmitter operates at a frequency of 880 kHz atom is
and a power of 10 kW. The number of photons
(A) n (B) n2
emitted per second is
(A) l.7lxl0 31 (B) 1327 x 10" (C) .! (D) n'
n
(C) 13.27 X 10" (D) 13.27 X 1044
140. In a photoelectric cell, the current "stops when the
136. The largest momentum we can expect for a microwave collecting plate is one volt negative with respect to the
photon is emitting metal. The maximum kinetic energy of the
(A) 6.6xl0-27 kgrns-1 (B) 6.6xl0_,. kgrns-1 photoelectrons is
(C) 6,6 X 10-31 kgrns-l (D) 6,6 X lQ-"" kgrns-l (A) 1 erg (B) 1J
(C) 1,6 X 10-1' J (D) 1.6 x 10-" eV
C
137. An electron is moving with a velocity of . The de-
10
Broglie wavelength associated with it is

= 3.32 ================================::::;
www.puucho.com
Rahul Sardana with www.puucho.com

1his section contains Multiple Correct Choice Type Questions. Each question has four choices (A), (B), (C) and (D), out of which
ONE OR MORE is/ are correct.

1. When photons of energy 4.25 eV strike the surface of a Irnax when averaged over many pulses is 80 µA . Then
metal, the ejected photoelectrons have maximum which of the following statements is/ are true.
kinetic energy TA eV and de Broglie wavelength :\.A. (A) The charge carried by one pulse is 8 x 10-13 C
The maximum kinetic energy of photoelectrons (B) Number of photoelectrons emitted per light pulse
liberated from another metal B by photons of energy is 5
4.70eV is T,=(TA-1.50)eV. If the de-Broglie (C) Number of photons in one light pulse is 50
wavelength of these photoelectrons is :\., = 2:\.A, then (D) Number of electrons carried by one pulse is 5 x 105
(A) the work function of A is 2.25 eV.
(B) the work function of B is 4.20 eV . 4. Light from a monochromatic source is incident
(C) TA= 2.00 eV. normally on a small photo sensitive surface S having
work function ~. If power of the source is W afld a is
(D) T, =2.75 eV.
the distance between the source and S , then
(A) the number of photons striking the surface per unit
2. A collimated beam of light of flux density 30 kWm-' is
incident normally on a 100 mm' completely absorbing .
timew illb(WASJ
e ---,
41chca
screen. If 'fYJ is the pressure exerted on the screen and (B) the maximum energy of the emitted electrons will
Ap is the momentum transferred to the screen during a
1000 s interval then, be(~-~)
(A) 9" = 10-3 Nm-2 (B) 9" = 10.., Nm_, (C) the stopping potential needed to stop the most
(C) ~p = 10.., kgms-1 (D) ~p = 10-5 kgms-1 energetic photons will be !:..(l,c-:\.~)
• A.
3. A photomultiplier tube is to be used to detect light (D) p h oto erruss1on lzc
· · occurs onl y 1"f O,;; :\. ,;; ~
pulses each of which consists of a small but fixed
number of photons. The average photoelectric efficiency
is 10%. That is photon has 10% probability of causing 5. When ultraviolet radiation is incident on a surface, no
the emission of a detectable photoelectron. Assume the photoelectrons are emitted. If another beam causes
photomultiplier gain is 10 6 and that the output current photoelectrons to be emitted from the surface, it may
as a function of time can be approximated as shown in consist of
figure. (A) radio waves (B) infrared rays
(C) X-rays (D) gamma rays

----------------~--· 6. When the intensity of a light source is increased,


(A) the number of photons emitted by the source in
1- unit time increases

_J _________ _ (B) the total energy of the photons emitted per unit
time increases
3.33 i::::J

www.puucho.com
Rahul Sardana with www.puucho.com

Advanced JEE Physics Optics & Modern Physics

(C) more energetic photons are emitted (A) Mirror will start to oscillate.
(D) faster photons are emitted (B) Wavelength of reflected rays will be greater than
that of incident rays.
7. Light rays are incident on an opaque sheet. The correct (C) Wavelength of reflected rays may be less than that
statement(s) is/ are of incident rays
{A) Light rays exert a force on the sheet (D) None of these
(B) Light rays transfer an energy to the sheet
(C) Light rays transfer momentum to the sheet 14. For a 75 W point light source assuming all the electric
(D) Light rays transfer impulse to the sheet power consumed goes Into emitted light of wavelength
600 nm, then
8. According to Heisenberg's Uncertainty Principle, (A) frequency of the emitted light is 5 x 1014 Hz
h h (B) number of photons emitted per second is 2.3 x 10 20
(A) lu!J.p ?. - (B) ti.EM ;, -
4rr 4rr (C) this emitted light on falling on a metal surface of
h h work function 1.07 eV, will emit photoelectrons
(C) ti.OM.;,- (D) lu!J.v ? . -
4rr 4mn having kinetic energy between Oand 1 eV
where the symbols bear the usual meaning. (D) on doubling the distance of this metal surface from
the point source maximum kinetic energy of
9. In which of the following situations, the heavier of the photoelectrons emitted becomes 0.25 eV
two particles will have a smaller de-Broglie wavelength.
(A) The particles move with the same speed 15. Photoelectric effect supports the quantum nature of
(B) The particles move with the same linear light because
momentum (A) there is a minimum frequency of light below
(C) The particles move with the same kinetic energy which no photoelectrons are emitted. ·
(D) The particles have fallen.through the same height (B) the maximum kinetic energy of photoelectrons
depends only on the frequency of light and not on
10. The threshold wavelength for photoelectric emission its intensity.
from a material is 5200 A . This material when (C) even when the metal surface is faintly illuminated,
illuminated with .................... monochromatic radiation the photoelectrons leave the surface immediately.
emits photoelectrons. (D) electric charge of the photoelectrons is quantized.
(A) 1 WUV (B) 50WUV
16. If the wavelength of light In an experiment on
(C) 1 WIR (D) 50WIR
photoelectric effect is doubled,
(A) the photoelectric emission will not take place
11. Light rays are incident on a metallic sheet. Then, (B) the photoelectric emission may or may not take
(A) the force exerted is Independent of frequency of - place
light Incident (C) the stopping potential will Increase
(B) the force depends on the direction of light Incident (D) the stopping potential will decrease
(C) the pressure is Independent of frequency of light
incident 17. A metallic surface ejects electrons when exposed to
(D) the pressure is proportional to the area of the plate green light of Intensity I but no photoelectrons are
emitted when exposed to yellow light of Intensity I . It
12. The momentum of a single photon of red light of
is possible to eject electrons from the same surface by
frequency 400 x lOu Hz moving through free space is (A) yellow light of some Intensity which is more than
(A) ZERO (B) 8.8 x 10-" kgms-2 I.
(C) l.65x10 .. MeV/c (D) Data Insufficient (B) green light of any Intensity.
(C) red light of any Intensity.
(D) violet light of any Intensity.
13. A small mirror is suspended by a thread as shown in
figure. A short pulse of monochromatic light rays is
18. In an experiment of photoelectric effect, light from a
Incident normally on the mirror and gets refiected. point source of monochromatic light of wavelength
Which of the foliowlng statements is/ are_ correct?
3000 A is Incident on a metal surface. The kinetic
energies of photoelectrons range from zero to 4x10-19 J,
then ·
(A) stopping potential for this light is 2.5 V
(B) threshold wavelength for the material is 7590 A

= 3.34 i:::::===================================
www.puucho.com
Rahul Sardana with www.puucho.com

Dual Nature of Radiation & Matter

(C) stopping potential will be doubled on reducing the (C) if the wavelength of source of radiation is doubled
distance and the wavelength of light source to half then KE of photoelectrons is also doubled
(D) saturation current will be doubled on reducing the (D) photoelectrons will be emitted from the surface of
distance of source to half all the three metals if the wavelength of incident
radiations is less than 200 nm
19. The threshold wavelength for photoelectric emission
from a material is 5200 A . Photoelectrons will be 23. When a rnonocliromatic point source of light is at a
emitted when this material is illuminated with distance of 0.2 m from a photo-electric cell, the cut-off
monochromatic radiation from a voltage and the sahµ'ation current are respectively
(A) 50 watt infrared lamp 0.6 V and 18.0 mA . If the same source is placed 0.6 m
(B) 1 watt infrared lamp away from the photoelectric cell, then
(C) 50 watt ultraviolet lamp (A) the stopping potential will be 0.2 V .
(D) 1 watt ultraviolet lamp (B) the stopping potential will be 0.6 V.
(C) the saturation current will be 6.0 mA .
20. Light of wavelength 496 nm is incident on a metal (D) the saturation current will be 2.0 mA .
surface causing ejection of photoelectrons for which
stopping potential is 1.5 V, then 24. Figure represents a graph of kinetic energy ( K ) of
(A) the work function of the surface is 1 eV photoelectrons (in eV) and frequency ( v) for a metal
(B) de-Broglie wavelength of fastest photoelectron is used as cathode in photoelectric experiment. The work
100nm function of metal is
(C) to move the fastest electron in a circle of radius
3K
1 m, perpendicular magnetic field B required is
4µT 2
(D) this fastest electron when strikes zinc target can
produce X-rays 0+---,,---+v

21. Which of the following statements about photoelectric -~ ...............


effect is/ are false?
(A) It exhibits the particle nature of radiation -3
(B) Electrons are emitted only if the radiation has a (A) 1 eV {B) 1.5 eV
frequency above a certain value (C) 2eV (D) 3 eV
(C) All the electrons emitted by radiation of a
particular frequency have the same energy 25. In PROBLEM 24, the threshold frequency is nearly
(D) Changing the intensity of radiation changes the (A) 5xl0 14 Hz {B) 10xl014 Hz
maximum energy with which the electrons can be
emitted (C) 2.5 x 1014 Hz (D) cannot be estimated

22. Radiations of monochromatic waves of wavelength 26. It is necessary to consider light as a stream of photons
400 run are made incident on the surface of metals Zn , to explain
Fe and Ni of work functions 3.4 eV, 4.8 eV and (A) Photoelectric effect
(B) Compton effect
5.9 eV respectively (take he= 12400 eVA)
(C) Polarization of light
(A) maximum KE associated with photoelectrons from
(D) Diffraction oflight
the surface of any metal is 0.3 eV
(B) no photoelectrons are emitted from the surface of
Ni

================================== 3.35 =
www.puucho.com
Rahul Sardana with www.puucho.com

'I

This section contains Reasoning ,type questions, each having four choices (A), (B), (q and (D) out of ,which ONLY ONE is
correct. Each question contains STATEMENT 1 and STATEMENT 2. You have to mark your answer as
Bubble (A) If both statem~ts are TRUE and STATEMENT 2 is the correct explanation of STATEMENT 1.
Bubble (B) If both statements are TRUE but STATEMENT 2 is not the correct explanation of STATEMENT 1.
Bubble (C) If STATEMENT 1 is TRUE and STATEMENT 2 is FAISE. ,. ,, . . . -~ ,
. Bubble (D) IfSTATEMENTl isFAISEbutSTATEMENT2isTRUE.
,.
1. Statement-I: Statement-2:
' I - ••

Threshold wavelength of certain metal is 1..0 • Light of Higher th~ ,accelerating potential, s~aller is the
wavelength ~lightly less than 1.. 0, is incident on the de-Broglie wavelength.
plate. It is found that after some time the emission of
electrons stops. 5. Statement:I: _ _ ' ,
Work function of copper is greater than that of sodium.
Statement-2: But both will have saine value of the threshold
The ejected electro~ experience force of attracti~n due ·frJquency and threshold wavelength. :
to development of positive c],arges OI) plate which after . '. \ ! . . 1. ' ••

certain time is adequate enough to hold .them to plate Statement-2:


itself. · The frequency is inversely. proportional to wavelength.

2. Statement-I: 6. Statement-I:
A photon ,has no rest mass, yet it, carries definite In case of .an, electron 'and a "photon having same
momentum. momentum, wavelength associated with electron is
smaller.
Statement-2: . .
Momentum of photon is due to its energy and heilce its Statement-2:
equivalent mass. ' ·' ' Electron.cannot move with a· speed of photon.

3. Statement-I: 7. Statem~t-1: r . _ ,
In a photoelectric effect, the current increase~_ when The photoelectrons produced by a monochromatic light
positive potential of . q:>llector is increased, before · beam incident on a metal surface, have a spread in
saturation of current. their kinetic energies.

Statement-2: Statement-2:
Ute number of emitted photoelectrons increases. The work ·function of the metal varies as a function of
depth from the surface.
4. Statement-I:
The de Broglie wavelength of an electron accelerated B. Statement-I:
through 941 volt is 0.4 A. A proton, a deutron and an a. ~})article are accelerated
by the same potential difference. Their ;elocities will be
in the ratio of 1 :1 :-./2..

= 3.36

www.puucho.com
Rahul Sardana with www.puucho.com

Dual Nature of Radiatio11 & Matter


Stntement-2: 14. Statement-1:
Kinetic energy, E =qV =!:.mv 2 A photon and an electron, both of energy 1 MeV has
2 same wavelength.
Statement-2:
9. Statement-1:
Photoelectric effect demonstrates the wave nature ·of E = 22mec 2 =10 6 eV
light.
15. Statement-1:
Statement-2:
The number of photoelectrons is proportional to the The velocity of body of rest mass m0 is ~ c (where c
frequency of light.
is the velocity of light in vacuum) then mass of the body
10. Stntement-1: is 2m0 •
A photon has no rest mass, yet it carries momentum.
Statement-2:
Statement-2: m,
Momentum depends more on velocity than that of
mass.
Moving mass is given as m
g'
11. Stntement-1:
16. Statement-1:
The threshold frequency of photoelectric effect supports
the particle nature of sunlight. In the process of photoelectric erruss1on by
monochromatic light, all the emitted photo-electrons
Statement-2: possess the same kinetic energy.
If frequency of incident light is less than the threshold
Statement-2:
frequency, electro~ are not emitied from metal surface.
In photoelectric effect a single photon interacts with a
12. Statement-1 : single electron and electron is emitted only if energy of
Effective mass of photon varies with wavelength. each of incident photon is greater than _the w9rk
function.
Statement-2 :
E = mc2 is the relation between mass and energy. 17. Statement-1:
de-Broglie wavelength of an electron accelerated
13. Statement-1: 1 2
through a potential difference of V volt is 1,. = :/(; A .
Though light of a single frequency (monochromatic
light) is incident on a metal, the. energies of emitted
photoelectrons are different. Statement-2:
Statement-2: de-Brogle wavelength of an electron is given by 1,. =..!!____
mv
The energy of electrons just after they absorb photons
incident on metal surface may be lost in collision with
other atoms in the metal before tne electron is ejected
out of metal. ·

3.37.=

www.puucho.com
Rahul Sardana with www.puucho.com

This section contains Linked Comprehension Type Questions or Paragraph based Questions. Each set consists of a Paragraph
followed by questions. Each question has four choices (A), (B), (C) and (D), out of which only one is correct. (For the sake of
competitiveness there may be a feW questions that may have more than one correct options)

Comprehension 1 4. The kinetic eriergy of the electrons is


A photocell is operating in saturation mode ·with a (A) 0.56 eV (B) 1.32 eV
photo current 20 µA when a monochromatic radiation of (C) 0,86 eV (D) 1.76 eV
wave length 3000 A and power 1-MW is incident." When 5. The work function of the metal is
another p1onochromatic radiation of wave length 1500 A (A) 1.89 eV (B) 1.03 eV
and power 5 MW is incident, it is observed that the (C) 1.58 eV (D) 2.32 eV
maximum velocity of photo electron is doubled. Assuming
efficiency of photoelectron generation per incident photon to 6. Wavelength of radiation is nearly
be same for both the cases. Based qn above information, (A) 6565 A (B) 5555 A
answer the following questions. (CJ 4545 A (DJ 3535A.
1. The threshold wavelength for the cell is
Comprehension 3
(A) 3500 A (B) 4000 A·
de-Broglie suggested that every moving particle has a
(CJ 4500 i:,. (D) 5000 A
wavelengih associated with it, which is given by 1,, =!: or
, - p
2. The saturation current in second case is
h - . - . - -
(A) 50 µA (B) 40 µA " =..JZmK . With the help of these formulae we know that
(C) 60 µA (D) 45 µA
wavelength for a charged particle accelerated through a
3. The efficiency of photoelectron generation per incident potential V is 1,, ~. Based on above information,
photon is V
(A) 8.5%. (B) 8.25% answer the following questions.
(C) 8% (D) 8.75%
7. Proton and alpha particle are accelerated through same
Comprehension 2 . potential difference. Then ratio of their wavelength is
The radiations emitted when an electron jumps from (A) 1: ./2 (B) 2./2: 1
n = 3 to n = 2 orbit of hydrogen atom falls on a metal to (C) 2:1 (D) 1:2./2
produce photoelectrons. The electrons emitted from the
metal surface with maximum kinetic energy are made to 8. When an electron is accelerated through 150 V
mov_e p~endicular to a magnetic field of strength ~ T potential difference, then the wavelength associated
3 0 with it is approximately
in a raciills of 10-3 m . Based on above information, answer (A) 1 A (B) 2 A
the following questions. (CJ 3 A (DJ 4 A

= ==================================
3.38

www.puucho.com
Rahul Sardana with www.puucho.com

Dual Nature of Radiation & Matter


9. If electron and alpha particle have same momentum the 15. The threshold wavelength for photoelectric emission to ·
ratio of their wavelength is occur is nearly
(A) 1840: 1 (BJ 1: 1840 (A) 1250nm (B) 2000nm
(CJ 1: 1 (D) No!'e of these (CJ 2250nm (DJ 3000nm

Comprehension 4 Comprehension 6
A large number of identical balls each having a mass of A parallel beam of monochromatic light (,. = 663 nm)
66.3 g are thrown with speed of 5 ms-• into a house of intensity 30 kWm- 2 is incident normally on a 100 mm'
through two tall, narrow, parallel windows spaced 0.6 m completely absorbing screen for 10 s. Based on above
apart, the choice of window as target being random at each information, answer the following questions.
throw. Fringes are formed on a wall 12 m behind the
windows. Assuming the Plank's constant to have a 16. Pressure exerted by beam on the surface is
hypothetical value of h = 6.63 x 10" Js . Based on above (A) 10-5 Pa (BJ 10-< Pa
information, answer the following questions. (CJ 2x10-< Pa (DJ Sxl0-5 Pa

10. de-Broglie wavelength of the balls is 17. Momentum transferred to the screen· during the interval
(A) 0.2m (B) 0.02m is
(CJ 2m (D) 20m (A) 10_. kgms-• (B) 5 x 10_. kgms-1
(CJ 10-7 kgms-• (D) 2 X 10"7 kgms-l
11. Fringe width obtained on the wall is
(A) 0.01 m (BJ 0.2 m
(CJ 0.5 m (DJ 0.4 m 18. Number of photons striking the screen during the
interval is
12. If electrons are used in place of balls, moving with (A) 5x1019 (BJ 10'°
speed 107 ms-1 in the experiment then (CJ 2x10 20 (D) 5xl0 20
(A) Fringe width will decrease
(B) No interference pattern is obtained Comprehension 7
(CJ Fringe width will not increase A metallic surface, when illuminated by light of
(DJ Interference pattern is obtained but it is not frequency 8x10 14 Hz and 12xl014 Hz emits
possible to observe it photoelectrons of maximum kinetic energy 0.5 eV and
2.0 eV . Based on the information provided answer the
Comprehension 5
following questions.
A surface has light of wavelength ,. = 496 nm incident
on it, causing the ejection of photoelectrons for which the 19. The value of Planck's constant is
stopping potential is found to be 1.5 V . Based on above
(A) 6.0 x 10"' Js (B) 6.2 x 10--" Js
information, answer the following questions. 34
(CJ 6.4x10- Js (DJ 6.6x10"' Js
13. The de-Broglie wavelength of the fastest photoelectron
emitted is 20. The work function of metal is
(A) 75nm (B) 100nm (A) 0.5 eV (BJ 1.5 eV
(CJ 135nm (D) 235nm (CJ 2.5 eV (DJ 3.5 eV

14. To move the fastest electron in a circle of radius 1 m, 21. de-Broglie wavelength of electron when its energy is
the value of perpendicular magnetic field B is 0.5eV
(A) 4.lxl0_. T . (B) 2.1x10_, T (A) 8.68A (BJ 17.35 A
(CJ 3.2x10_, T (D) 5.2x10_. T (CJ 21.2SA (D) 24.54A

3.39 =
www.puucho.com
Rahul Sardana with www.puucho.com

Each question in this section contains statements given in fwo columns, which ·have to be matched. The ~~tements in .
COLUMN-I are labelled A, B, C and D, while the statements in COLUMN-II are labelled p, q, r, s (and t). Any given statement
in COLUMN-I can have correct matching with ONE OR MORE statement(s) in COLUMN-II. The appropriate bubbles
- corresponding to the answers to these questions have to be darkened as illustrated in the following examples:
If the correct matches are A-> p, sand t; B-> q·and r; C-> pand q; and D-> sand t; then the correct darkening of bubbles will
look like the following : ·
P qr s I·
A@@)(v@@)
B@@@®CD
c@@©®CD
D@@)(v@@)

1. In_ a photoelectric effect experiment, if f is the frequency of radiations incident on the metal surface and I is the inte~ity
of incident radiations, then match the quantities in COLUMN-I with.their matches in COLUMN-II.
f COLUMN-I COLUMN-II ·
(A) If distance between cathode and anode is increased. (p) . Stopping potential increases.
(B) If I is increas':d keeping f and work . function (q) Saturation current increases.
Constant. (r) Maximum kinetic energy of photo electron
(q Work function is decreased keeping f and I constant. increases.
(s) Stopping potential remain same.
(D) If f is increased keeping I and work function
constant.

2. In a photoelectric effect experiment, if the following changes are made, then match the COLUMN-I with COLUMN-II.
I COLUMN-I COLUMN-II
(A) If intensity of incident light is increased keeping its (p) Stopping potential will increase.
frequency constant. (q) Current through circuit may stop.
(B) If work function of photo sensitive electrode is increased. (r) Maximum kinetic energy of ejected
(q If frequency of incident light is increased and its intensity photoelectrons will increase.
is deCTeased. · (s) Saturation current will increase.
(D) If frequency of incident light is increased keeping its
intensity constant. ·

3.40 ======================================
www.puucho.com
Rahul Sardana with www.puucho.com

Dual Nature o/Radiatio11 & Matter


3. Match the properties in COLUMN-I with their respective phenomenon in COLUMN-II ..
I COLUMN-I COLUMN-II
(A) Photon character of radiation. (p) Photoelectric effect.
(B) Wave character of radiation. (q) Compton effect.
(C) Interaction of a photon with an electron, such that photon. (r) Diffraction.
energy is much greater than the binding energy of (s) Interference.
electron, is more likely to result in. '
(D) Interaction of a photon with an electron, such that
photon energy is equal to or slightly greater than the
binding energy of electron, is more likely to result in.

4. Match the wavelengths in COLUMN-I to the respective matches in COLUMN-II.


I COLUMN-I COLUMN-II
(A) 0.1A (p) de-Broglie wavelength of electron in X-ray tube.
(B) 1A (q) Photoelectric threshold wavelength.
(r) X-ray wavelength.
(C) 10A
(s) de-Broglie wavelength of most energetic
(D) soooA photoelectron emitted from metal surface in
photoelectric effect.

5. Some quant;ties related to photoelectric effect are mentioned under COLUMN-I and COLUMN-II. Match each quantity on
COLUMN-I with the corresponding quantity.in COLUMN-II on which it depends.
I COLUMN-I COLUMN-II
(A) d.e-Broglie wavelength of photoelectron. (p) Frequency oflight.
(B) Force due to radiation falling on metal plate. (q) Work function.
(C) Stopping potential. (r) Area of photo sensitive plate.
(D) Saturation current. (s) Intensity of light (at constant v ).

6. In a photoelectric effect experiment, if / is the frequency of radiations incident on the metal surface and I is the intensity
of incident radiations, then match the following.
I 'COLUMN-I COLUMN-II
(A) Work function is decreased keeping f and I constant. (p) Stopping potential increa_ses.
(B) If I is increased keeping / and work function (q) Saturation current increases.
constant. (r) · Maximum kinetic ene~gy of photoelectron
increases.
(C) If distance between cathode and anode is increased.
Stopping potential remain same.
(D) If f is increased keeping I and work function (s)
constant.

7. With respect to photoelectric effect experiment, match the entries of COLUMN-I with the entries of COLUMN-II.
I COLUMN-I COLUMN-II
(A) If I is increased keeping / and ~ constant. (p) Stopping potential increases.
(B) If f (frequency) is increased keeping I (intensity) and (q) Saturation photocurrent increases.
~ (work function) constant. (r) Maximum K.E. of the photoelectrons increases.
(s) Stopping potential remains the same.
(C) If ~ is.decreased keeping f and I constant.
(D) If the distance between anode and cathode increases.

================================== 3.41 =
www.puucho.com
Rahul Sardana with www.puucho.com

Advanced JEE Physics Optics & Modern Physics

8. If radiation of energy E, intensity I falls on different kinds of surfaces mentioned, then match the quantities in
COLUMN-I with their respective answers in COLUMN-IT.
COLUMN-I COLUMN-IT t
(A) Radiation pressure for a perfectly absorbing surface. I
(p) (l+p)-
(B) Radiation pressure for a perfectly reflecting surface. C

(q Radiation pressure for a surface of reflection coefficient 2E


(q)
(p). C

(D) Impulse on a perfectly absorbing surface.


(E) Impulse on a perfectly reflecting surface.
(r) -I
C

(F) Impulse on a surface of reflection coefficient (p) . (s)


21
•C

E
(t) (l+p)-
C •

E
(u)
C

= ====================================
3.42

www.puucho.com
Rahul Sardana with www.puucho.com

In this section the answer to each of the question is a four digit integer, ranging from 0 to 9999. The appropriate bubbles below
the respective question numbers in the ORS have to be darkened. For example, if the correct answer to question number X (say)
is 6092, then the correct darkening of bubbles will look like the following:
X.@e@@
(]) G) G) (])
®®®·
®®®®
@@@@
® @ ® ®
~~~~
@®®@
®®·®
1. A metal plate is placed 5 metre from a monochromatic followed by electrons ejected normally from the plate
light source whose power output is 10-3 W . Consider with maximum energy.
that a given ejected photoelectrons may collect its
energy from a circular area of the plate as large as ten 5. The maximum kinetic energy of photoelectrons emitted
from a certain metallic surface is 30 eV when
atomic diameters (10'° m) in radius. The energy
monochromatic radiation of wavelength A, falls on it.
required to remove an electron through the metal
surface is about 5 eV. Assuming light to be a wave, When the same surface is illuminate_d with light of
how long, in hour, would it take for such a 'target' to wavelength 2A. , the maximum, kinetic energy of
soak up this much energy from such a light source. photoelectrons is observed to be 10 eV. Calculate the-
wavelength A. and determine the maximum
2. Energy from the sun is received on the earth at the rate wavelength of incident radiation (both in A) for which
of 2 cal cm-' min-1 • If average wavelength of solar light photoelectrons can be emitted by this surface. Given
h=4x10-15 eVs and c=3xl08 ms-1 •
be taken as 6600 A, then xx 10 18 photons are received
on earth per cm2 per minute, find x Take 1 cal= 4.2 J,
6. On a certain metal light of frequency v = 5v 0 falls then
c=3xl08 ms-1 .
maximum velocity of electrons emitted is 8x106 ms-1 ,
3. Compute the typical de-Broglie wavelength of an where v0 is threshold frequency of metal. If v = 2v 0
electron in a metal at 27 °C and compare it with the then the maximum velocity of photoelectron is
mean separation between two electrons in a metal xx10 6 ms-1 .Find x.
which is given to be about 2 x 10-10 m:
7. A metallic sphere of radius 21 cm is kept in the path of
4. Light of wavelength 180 nm ejects photoelectrons from a parallel beam of light of intensity _!_ Wm_,. The
a plate of metal whose work-function is 2 eV. If a 110
uniform magnetic field of 5 x 10-s T be applied parallel force exerted by beam on the sphere is xx 10-13 N . Find
to the plate, what would be radius of the path, in mm, x.

================================:::::; = 3.43

www.puucho.com
Rahul Sardana with www.puucho.com

Advanced JEE Physics Optics & Modern Physics

8. The wavelength of light incident on a metal surface is 10. If photons of ultraviolet light of energy 12 eV are
reduced from 300 nm to 200 nm (both are less than incident on a metal surface of work function of 4 eV,
threshold wavelength). Find the change in the stopping then find the stopping potential (in eVJ.
potential, in volt, for photoelectrons emitted from the
surface. Take h = 6.6 x 10"' Js. 11. A proton and an a -particle are fired through the same
magnetic fields which is perpendicular to their velocity
9. In the photoelectric experiment; if we use a vectors. Both move such that radius of curvature of
monochromatic light, the 1-Vcurve is as shown. If work their path is the same. Find the ratio of their de-Broglie
function of the metal is 2 eV, estimate the power of wavelengths.
light used (in W ). Assume efficiency of photo emission
to be 10-3%, i.e., number of photoelectrons emitted are
10-3% of number of photons incident on metal.
I

10 µA -------~--

_ _-:::___J__________ v

-5 volt

= 3.44

www.puucho.com
Rahul Sardana with www.puucho.com

"- _·-t"f4s
• _Cif.i· ·1~-.·: .i 'F:·"
E!_ASElfQN P.HOfit:fNjP,ROP.ER'TIES
,
1. 4.7x10-e Nm---2 7. (a) 3.9Bx10·" J

2. 453A (b) 5x1Q15

3. 2.5A 9. 5.5x10-12 m

4. 2.51x1031 10. AG> Ap

5. 1.43x1019 11. E, >E,


6. 10" 12. EP >Ee

4 •1Jfeg!1(~ii~t;;:-,;:~( \ii/._~-:~zt?~:i:?~tt1:~~F~:-.. ;:-' ;>;':./ -~_-:... ; ~ ·/?.:.~~:tt~~~:~~-B~S.~DfQ~-!ift(C:?:h9tUECiR1¢lf:FF.Ee]i


1. 2.BeV a. 6.6 x 1O'" Js , 5990.25 A
2. aasaA 9. 1.9eV
3. 11.93eV 10. 1.9 eV, 4125 A, No change is observed
4. 2260A 11. (a) 2.1 eV
5. 9.28 me (b) 2.1V
12
6. 1.12x10

7. (a) 1015 Hz

(b) 6.25 eV

(c) 2 eV

c:::================================== = 3.45

www.puucho.com
Rahul Sardana with www.puucho.com

Advanced JEE Plzysics Optics & Modem Physics

• ANSWERS SINGLE CORRECT CHOICE TYPE QUESTIONS I


- - - ---
1. C 2. C 3. A 4. B 5. C
-
6. D 7. B 8. A 9. C 10. C
11. A 12. A 13. C 14. A 15. A
16. B 17. C 18. D 19. C 20. B
21. A 22. A 23. B 24. C 25. B
- -- -
26. B 27. D 28. B 29. B 30. C
31. D 32. C 33. D 34. C 35. A
36. C 37. D 38. B 39. B 40. B
- -
41. B 42. A 43. C 44. D 45. D
46. B 47. A 48. B 49. D 50. B
51. B 52. D 53. B 54. C 55. D
56. C
-
57. C 58. B 59. C 60. B
61. A 62. D 63. C 64. A 65. D
66. C 67. C 68. C 69. A 70. C
-
71. B 72. C 73. C 74. A 75. C
--
76. C n. C 78. D 79. A BO. D
81. C 82. A 83. C 84. D 85. C
86. A 87. C BB. D 89. A 90. C
91. B 92. B 93. A 94. C 95. D
96. D 97. B 98. A 99. A 100. D
101. C 102. B 103. C 104. A 105. B
--- - - --
106. B 107. A 108. A 109. D 110. A
- -
111. A 112. B 113. A 114. A 115. D
116. D 117. A 118. C 119. B 120. A
121. B 122. B 123. C 124. A 125. D
126. D 127. D 128. C 129. C 130. B
131. B 132. D 133. B 134. C 135. A
136. C 137. C 138. C 139. A 140. C

• 1·
L
ANSWERS

1.
6.
ABC
AB
2.
7.
BD
ABCD
3.
8.
ABC
ABCD
MULTIPLE CORRECT CHOICE TYPE QUESTIONS

4.
9.
ABD
ACD
5.
10.
CD
AB
I

11. ABC 12. BC 13. AB 14. ABC 15. ABC


- -- -- -
16. BD 17. BD 18. AB 19. CD 20. AC
21. CD 22. BD 23. BD 24. C 25. A
26. AB

= ==================================
3.46

www.puucho.com
Rahul Sardana with www.puucho.com

Dual Nahtre of Radiation & Matter

• ANSWERS REASONING BASED QUESTIONS j


:- ---- ------- - - - --------------· -- ------------ ---·
,_ -~- -~---·-· · - - - · -~-~- ____________ 3.__ C _________ 4.:_ __ ~---. _ _ _ _ _5_._D_ _
6. D 7. C 8. D 9. D
13. A 14. A
10. C
r- ------·-·--------------- -- --·-- -·--- ---· --· ···- .. - ----- --- -·- ----------
1_ ~~--~ _ ·- - · . - - __1_2._~ _ ... 15. A
-------
-------- - -------·--··----- -------------·--------------~
16. D 17. D

LINKED COMPREHENSION TYPE QUESTiC>NS j


·-------------- ----------------- ---7
1. C 2. A 3. B 4. C 5. B
1 •. - ----- - - - - ---------- ------- - - - ---- --- . --- ----------- ----- -- - ------- ---- - - -------· - - - - - • .J
6. A 7. B 8. A 9. C 10. B
[_·11._0 ____________ 12._o _____________ 13._B _______________ 1<i. __ A__ 15. A __ I
16. B 17. C 18. B 19. A 20. C
, 21. B _________ J'
·---------- --------·-- - ··--··- - --------- ·------------

• ANSWERS MATRIX MATCH TYPE QUESTIONS I


' ---- -- ---- - --- - ---- --- ----
1. A-> (s) 2. A-> (s) 3. A --> (p, q) 4. A-> (p) 5. A --> (p, q)
B--> (q, s) B --> (q) B--> (r, s) B--> (r) B--> (p, r, s)
C--> (p, r) C--> (p, r) c, .... (q) C--> (s) c .... (p, q)
D--> (p, r) .• ··- ···- ___ D--> (p, r) _ -- ·- _____ 0--> (Pl - ·--- -· ____ o ...._(q) --····- - __D- ~ ~ -
6. A --> (p, r) 7. A--> (q, s) 8. A--> (r)
B--> (q, s) B--> (p, r) B--> (s)
C--> (s) C--> (p, r) C--> (p)
D-->(p.r) D--> (s) D--> (u)
E--> (q)
F--> (t)

• ANSWERS INTEGER ANSWER TYPE QUESTIONS ·j


------ ··----- - - --- -- - ------- - - . -- -- --
i 1.
, ___
20 ---
..
2. 28
-- ---- -- ,. --
3.
-
31
-
-- -- - ·-··· - 4. 148
------------- -------- _,
/
5. 1200 6. 4 7. 42 8. 2
1-·------·-- - - - - - ------------- - -·-- - ·-· --- - - - ----~-- - - -- - - - - ---i
, 9. 7 10. 8 11. 2
,_ ------------ ---------------------- ---------- ----------- -· - - . --· - ·-- ----- --- - ________ j

================================== = 3.47

www.puucho.com
Rahul Sardana with www.puucho.com

1. For completely absorbing surface, 6. Energy of each photon is


I 1.4x103 4.7x10~ Nm--2 E = hv = 6.62x 10--:w x1012 = 6.62x1042 J
Pra<1=c 3x108 Number of photons present in 6.62 J of radiation energy is
calculated by using
E=N(hv)
2. f~~=£
21tr1 A· N=_§_ 6.62. 10""
hv 6.62x10""'
). 2ncr1 (2,)(3 x 10 )(0.529 x10-")(1 o")
8
A
·v, (2.2" 10 8 ) 7. (a) Energy of each photon, •
E =hv = 6.63x10--34 x6 x1014 =3.98x10-19 J
(b) If N is the number of photons emitted per second by the
source, then
3. For an electron, de-Broglie wavelength is given by, Power transmitted in the beam is
P =N (energy of each photon)
l.=J150 =J150 =,/6
V 25 => P=NE
=> l.~2.5A N=!'_- 2x10" W
E 3.98 x 10-" J
=> N = 5 x 1015 phoions per second.
4. Power of transmitter =10 kW = 1o• W
So, total energy emitted per s_econd is- 10• J 8. The total energy of an electron with rest mass m0 and
he 6.63x10--34 x3x10 8
Energy of one photon, E = hv =- ------- J momentum p is
l. 500
E = ~,..m"":-c•""+-p~'""c=•
Number of photons emitted per second is
n = Total energy emitted per second · where c is the speed of light in free space. Law of'Conservation
Energy of one photon of Energy requires,
Rest mass)+ (Energy of) = ( Total energy of)
=> n= 104 x500 -2.51x1031 ( of electron photon moving electron
6.63x10--34x3x108 •
2 4
=> m0 c :+ hv = ~m~c + p2 c2 ••• (1)
5. · Energy of one photon is Law of Conservation of Momentum requires.
E=hc Initial momentum)+ (Momentum) = (Final momentum)
l. ( of electron of ~hoton ~ of electron
8
=> E 6.625x10--34x3x10 -S.Sx 10_19 J h
5600x10" 10 => O+i;:=P
Since a 100 Wbulb supplies 100 J of energy per second.
hv
So, energy released per second as visible photons is c=P 1--- l.=~l
E'=100x5= 5 J => hv=pC ... (2)
100
Squaring both sides of equation (1), we get
Number- of photons emitted per second as visible light is
·m~c + h v 2 + 2m0 c 2hv =m~c4 +p2 c2
4 2
E' 5
1.43x1019
E 3.5 x10"18 => 2(m,c')(hv) = 0 {using (2)}
This is impossible. In a frame, where the initial electron is
moving with uniform velocity, the same conclusion must hold

= 3.48·==================================
www.puucho.com
Rahul Sardana with www.puucho.com

Dual Nature of Radiation & Matter


because if a process is forbidden in one inertial frame, it is also 11. Kinetic energy of electron is
forbidden in another inertial frame.
E - 1mv2- 1 _(mv)2 - p2 = h2
9. KE of an electron is e 2 2 m 2m 2mA2 (·.- P=£)
Energy of a photon is totally kinetic and is given by
EK= 50 ev·=1.6x10-19 x5x 104 J = Bx 10-15 J
he
de-Broglie wavelength of electrons is Ep =hv=-
l.
l.=-h- Ep he 2mA2 2mcA
~2mEK => Ee ::: T. h2 = -h-
=> l. ----p----ee=--=m
6.63x10--34
'12 9.11 x1
X Q--31 X
8 1 X Q-lS
=- Ep mp
Ee= me
=> A 6.63x10-11 -55 10-12 Thus the KE of a photon is greater than that of an electron of
12.07 m- · x m same wavelength.
For yellow light, wavelength A =5.9 x 10-1 m
As resolving power is inversely proportional to wavelength, so
12. The total relativistic energy of a particle is
5 E = ~moc4 + p2c2
the resolving power of an electron microscope is about 10
times greater than that of an optical microscope. As wavelength A. is same for both electron and proton,

-10. The kinetic energy of a particle of mass m can be expressed in


terms of its momentum p as follows
Since, momentum, p = £ is same for both particles and hence

p 2c2 is same for both.


1 2 1 (mv)2 p 2
KE=-mv = - · - - = - But rest mass m0 of a proton is greater than that of an electron,
2 2 m 2m
therefore, the energy of a proton is more than that of an electron
As de-Broglie wavelength, A=~ of same wavelength.
p
=> ~=p' ~J2m,-KE = /m,
Ap Pe 2m0 • KE vm::
As m0 < mp , therefore, i..0 > AP
Thus the electron has greater de-Broglie wavelength.

3.49 =
www.puucho.com
Rahul Sardana with www.puucho.com

.1. Since, E=hv=(4.316x10-" eV'sec)(1.5x10" sec-•)_=6.5 eV 12375


5. Sl·nce. E("1n eV) -
=> K= =E-W=(6.5-3.7) eV=2.8 eV - i.(in A)

=> E, = 12375 = 2.475 eV


12375 5000
2. E= ;, 5 eV =8x10-" J
2480-' E _,12375_206 V d
Number of photons emitted per second from the source is given 2-·6000 :- . e an
by
E ·= 12375=1.77eV
p 40 19 3
n, =-=---=5x10
19
7000
E Bx.10-
Since, W is 1.9 eV, photons of energy ~ and E2 can only
Number of photons incident ·per unit area per second on
magnesium surface is emit photoelectrons. Charge emitted per second is
19
n = n1 = 5x10 ~ 1016 .'!=(16x10-")(.!) (1D"")(lO_.)
2 2
• 4nr 2
4rt(2)2 t · 31tx(10-3) x2.475x1.6x10-19
From Einstein's Photoelectric equation, we have +(1.6x10-")(.!) <;o"")(rn-•)
K= =E-W=5-3.68 =1.32 eV 3 nx(10"") x2.06x1.6x10-"
s·ince,_ i. (· A)
om
12375
W(in eV) => 7=9.28x10"" C=9.28 mC

=> 1. = 12375 = 12375 =3363 A


' W 3.68 E, = 12375 =3.1 eV
6.
4000
3. .!S_ =5 12375
K, E, = = 2.57 eV
4800
AE,-W = 12375
=> 5 ..• (1) E, = = 2.06 eV and
AE,-W 6000
E,=-0.85 eV E, = 12375 = 1.7? eV
.1 E,=-1.51 av 7000
• • E,=-3.4 eV Therefore, light of wavelengths 4000 A , 4800 A and 6000
can only emit photoelectrons.
A
So, number of photoelectrons emitted per second is
n = l 1A1 + I2 A2 + l 3A3
E, E2 E3
E, =-13.6 eV ;;;::), n=IA(E1E2+E2E3+E,EaJ
Here, .6.E1 =E,-E1 =12.75eV and .6.E2 =E3 -E1 =12.09eV E,E,E,
Substituting in equation (1) and solving we get _ (1.5x10->)(10~) 3.1x2.57+2.57x2.06+3.1x2.06]
W=11.93 eV
;;;::), n- ,s
. 1.6x10- 3.1x2.57x2.06
1
=> n=1.12x1Q 2-
4. K= =E--W
=> 7. (a) ~v 0 ~W=3.75x1.6x10- 18 J
3 eV=12375_12375
1. 5000
=>
v = 3.75x1.6x10-19 :::,'
10,s Hz
=> 1-=226oA 0
6.6x10-34

= ===================================
3.50

www.puucho.com
Rahul Sardana with www.puucho.com

Dual Nature of Radiation & Matter


12375 hc _w
(b) E= eV = 6.25 eV
1980
4=-1.,-
=> Kmax = 2.5 eV hc_w
(c) Kmax =2 eV i.,
4x12375 12375 _ eV
=> 5 63
K =E-W=hc_hc 5400 3500
8.
mrut A Ao => W=1.9eV
_ x _ x _,, h(3x10') h(3x10')
0 18 1 6 10 ... (1) 10. From Einstein's Photo-~lectriC Equation, we have
5461x10-10 A0
0. 6 = 12375 -W •.. (1)
he he 4950
Further, eV,=E-W=T-"i:"""
0
1.l= 12375 -W ... (2)
).
=> 4.6x1.6x10-" h(3 x 1o') h(3x10')
•.. (2)
1849x10-10 A. 0 Solving above two equations, we get
Solving equations (1) and (2), for h and 1.0 , we get W=1.9 eV and 1.=4125 A
No change is observed.
h=6.6x10.,.. Js
and 1.0 = 5990.25 A 11. Since, W=2.3 eV and 1.=2800 A
E{in eV) = 12375 = 12375 = 4.4 eV
9. From Einstein's Photo-Electric Equation, we have i. (in Al 2800
.!mv/ = he - W ... (1) (a) From Einstein's Photo-Electric Equation, we have
2 i., Kmax =E-W
.!mv/ = he -W ... (2) => K= =(4.4-2.3) eV
2 i.,
=> Kmax=?:.1eV
Dividing equation (1) with equation (2), and using the given fact
(b) Since, K= = ev,
that v, = 2v2 , we get
=> 2.1eV=eV0
=> V0 =2.1V

3.51 =
www.puucho.com
Rahul Sardana with www.puucho.com

1. [CJ
E=nhv => E=m,e'(1+ :0,)
=> _§_ = P = (-"-)hv . 1
t . t => E=m 0 c2 +-m
2 0v
2

3. [AJ 10. [CJ


he Since both have same de Broglie wavelength, hence both must.
s ev, =T-w ... (1)
have equal value of momentum.
he '
=> ev, = 3;. -W ... (2) Since, E=_E__
2m
Solving equation (1) & (2), we get
=> Ee= mp =1840=2000 nearest possible )
he -W= 5he _ 5W ( approximation to answer
EP me
). 3).

=> 4W=2he 11. [A]


3).
E2 = p2c2 + m~c"
=> W= he
6). => E2 =p2 Cf+E!

4. [BJ
l.=-h-
=> P=~J1-(~ J
=~ (tor E»E,)

../2meV =>
h_
"A.'= __
For a photon
../'2MeV
E=hv=he
=> ).' = ;.Jfj l.,
=> l. =he=l.
8. ' E

12. [AJ
, Energy of each lump before collision is
=> A.= kE2 m0 c2 5
E=mc2 = mc2
=> log, l. = logk -½logE - FITT3'
The energy of composite lump after collision will be Mc2 •
9. [CJ
By energy conservation principle we get
E=mc2 Total initial energy= Total final energy
=> _§_m .c2 +_§_m c2 =Mc2
=> 3 0 3 0

=>
Since ~«1
e

= 3.52

www.puucho.com
Rahul Sardana with www.puucho.com

Dual Nature of Radiation & Matter


13. [CJ 18. [DJ
mv Energy of falling light
r=-
qB E=hc (4.14x10·")(3x10')
2x107 A 3x10-7
r = 71.-=7:::6-x71o"''"'x--=-2-x710,-_.,., => E=4.14eV
r=0.0055 m So, electrons are emittec;t with a kinetic
D=2r=0.011m energy= 4.14-1 = 3.14 eV
1
=> D=1.1cm => zmv 2
=3.14x1.6x10-19

14. [AJ 2 X 3.14 X 1.6 x1Q-19


V= 9.1x10-31
E, -E, =13.6[;- ~]
3 v=1.2x10 6 ms-1
13 6 5
AE= · x =1.89 eV
36 19. [CJ
Photoelectrons with Kma)[ are moving on circular path, so 1.=~
mv mv
r=-
qB => 1.=--h-
mv =qBr mJ2~V
1 19
p = qBr =1.6x10- x --x10-3
3200 => 1.=--h-
. ~2rnqV
p=..:!.x10-24 =5x10-25 kgms- 1
2
, => i.,
i..
= Jm·q• = 2-,/2
mpQp
{·:m. =4m,. q
0
=2q,)
Energy of photoelectron is K =L 0

= 2m
K = 25x10-so eV
20. [BJ
max 2x9.1x10-3 1 x1.6x10-19 Roc.! and 1.= ~ h oc ;.,
K= =0.86 eV 1. 2mqV -.,V
Now using Einstein equation, we get => Roc.Jv
hv = cj> + Kmax
=> 1.89=0.56+$ => ~ =J~~~~ =2
=> $=1.03eV => R,=2R=2R
15. [AJ 21. [AJ
F = .!.(Effective Area) A NiA, +N2A2
C
N1+N2
_ _ N,(10)+N,(11).
10 81
N1+N2
Solving·, we get
16. [BJ
N, 0.19 19
N, = 0.81 = 81

23. [BJ
h he
Fora photon A=-=-
P E
For a particle of mass m moving with a velocity v de Broglie
relationship is given by
Let p=-E 1.=~
c mv
=> Ap=_E,_-(__E,_)=2E 24. [CJ
C C C

E=n(~)
17. [CJ
1.=-h~ Also ..!1: Ea = Energy
-hmeV 2 °0
Volume

So, when V becomes 4 V, A. becomes !:


2
~ Energy= (ie 0 E~ )(Volume of cavity)

================================:::i 3.53 =
www.puucho.com
Rahul Sardana with www.puucho.com

Optics & Modern Physics


Advanced JEE Physics
38. [BJ
Energy= ( i,,E; )(Area( ct.t))
-mv
1 ' = k8 T
2
=> n~ =i<,E;(Act.t)

n _ (Number of photons striking the desk)


v=l~T
Mt - (Area)( time) => l. h
- ~2mk,T
=> ~ = N =~ e0E~
Mt 2 h
39. [BJ

26. [BJ ..!.m v2 =eV


2 p

Kx, = !(sR')
=> V= teV m,
=> x0 = K~ (nR')
Since, '),.. =~
m,v
30. [CJ
According to Einstein's Photoelectric ~quation => l.=--h-
EK =hv 0 -4> ~2mpev
=> 0=hv0 -«j>
A=
o 287
· r.; A
(is de-Broglie wavelength)
for a proton accelerated
vV t~rough a potential V
12 27 ( is de-Broglie wavelength
A.= ---+.,;A for an electron accelerated
J
(6.626x 10_,. )(3x10')
=> .;4 X 1.6 X 10-19 vV through a potential V
l.,
48. [BJ
P= nhc
34. [CJ l.t
Stopping potential of 1.36 V implies EK = 1.36 eV . n X 6,6 X 10-34 X 3 X 108
=> 1.7x10-18
Since hv = $0 +EK 6000 X 10-lO X 1
( 4.14 X 10-" )( 3 :10•) => n=S.15=5
=> ~, +1.36
5000 x10-10
49. [DJ
2.46=~, +1.36
(e/m,) =
2
=> ~,=1.1eV (2e/4m,)

35. [AJ 50. [BJ


Stopping potential is independent of intensity and depends upon According to Plank's Quantisation Law.
frequency.
E=nhv=n(h:)
36. [CJ
Saturation current a: (Intensity)
=> Saturation current= 4( 0.4 µA) => f=(~)(~)
=> Saturation current=1.6 µA _, = (!l_ (6.626 X 10_,. )( 3 X 10')
10 t 5000 X 1o-10
37. [DJ ~=2 11
t .5x10
..!mv2 = he -W ..• (1)
2 l.
52. [DJ
..!:mv'2 =~-W ... (2)
he
2 3l./4 -=~,+2.5
l.
,
2
4hc_w
Dividing,(~) = ~~ -W he
(l./2)

=,o +
E'
K

l.
E~-2.5=hc,
l.
E; =2-5+4-14
E~ = 6.64 eV
So, stopping potential is 6.64 V > 5 V

,---, 3.54 c-================================


www.puucho.com
Rahul Sardana with www.puucho.com

Dual Nature of Radiation & Matter


53. [BJ 64. [AJ

•=~ mv
F = .!.(Effective Area)
C
If Eis kinetic energy of electron, then
=> F=.!.(,R')="R'I
E=.!mv2 C C
2
65. [DJ
V=~ Speed of first electron may increase or decrease, depending on
the direction of electric field. However in the case of electron
,=_h_ entering the magnetic field speed remains constant. Since, from
,l2mE de·Broglie relation

54. [CJ
Energy of falling photon< Threshold energy. So, no photoelectric
•=~ mv
effect takes place. => A1 > "-2 OR '-2 > A1
are both the possibilities.
55. [DJ
66. [CJ
Since A=!!
p h
i= mev
For identical p's, A are identical
=> ,=3.64nm
56. [CJ
hv=EK +~ 67. [CJ

hv-~=.:!.mv2
2
,,
K1=hc_w ... (1)

V=~
,,
K2=hc_W ... (2)

Since, A1 = 2A 2 , so (1) becomes

m,
V= • /-2~(_hc~--·~~)
K,=~-w=.!(h )-w
2A.2 2 A2
0

58. (BJ 1 .
Stopping potential is 4 V. So, maximum K.E. is 4 eV. => K, = (K, +W)-W
2
59. [CJ K = K, _ W
Photoelectric current oc Intensity ' 2 2
=> iccl K < K2
=> i=kl ' 2

60. [BJ 68. [CJ


1 2
Total energy radiated per minute from sun is
mv =eV
2 Eradiated = cr( 41tR!e)
=> v=~2eV Energy radiated annually is given by
, m Etotal =24 X 60 X 365 X Erndia!ed
61. [AJ ~ Annual loss of mass= Am= E10; 11 = 1.38 x 1017 kg
C
Since .6.E.6.t 2:: h
=> t.E( 10.,,) = 6.6 x 10.,., 69. [AJ
=> .6.E=6.6x10-2fi J Since V=uc {where a« 1)
the electron is moving at non-relativistic speed.
63. [CJ By Law of Conservation of Momentum
h h
-+-=mv

1mmx1 mm2
' ,'
he he mv2
~ -+-=mCV=-- ... (1)
C A A.' u
100x10-9 x2640x10-10 By Law of Conse,vation of Energy
N
(3 X 10')' X 6.63 X 10.... ..!mv2 = he_ he ... (2)
2 A A'
=> N = 442 photonsmm-3
Adding (1) & (2), we get
2hc=.!mv'+.!(mv')
, 2 a

==================================== 3.55 =
www.puucho.com
Rahul Sardana with www.puucho.com

Advanced JEE Physics Optics & Modern Physics

he mv 2 mv2 74. [A]


=> -=E=-+-
A. T 4 2a F =_f'_= IA
ox C C
Since, a« 1
where I= 1.4 x10 3 Wm 4
::::) ..!.»1 2
(J.
=> A= 41tr2 =4x3.14x(2) m2
mv 2 mv 2 => F0 • =2.35x10-4 N
=> -«-
4 2a
75. [CJ
=> E,=-Hm;') KE=hv+~ ... (1)
2KE=hv'+~ ..• (2)
=> !mv2 =aE
2 ' => 2{hv+$)=hv'+~

70. [CJ => v'=2v+f


h
Photons are exchange particles for electromagnetic interactions.
=> v'>2v
Gravitons are exchange particles for gravitational interactions.
Mesons are exchange particles for nuclear interactions.
Whereas protons are not any sort of exchange particles. 76. [CJ
h
1-=- where,
71. [BJ mv
Relativistic relative velocity of approach is given by
m=g-
'
As V-> C
c-(--c) m-;oo
=> v,
1-(c)(--c) => 1--> 0
c'
77. [CJ
=> EK =hv-$0
=> (E, ), = 1-0.5 = 0.5 eV
=> Similarly (E, ), = 2._5-0.5 = 2 eV
Never expect the relative velocity to grow beyond the speed of
light. (E,),=_!
=>
{E, ), 4
72. [CJ
By Law of Conservation of Momentum, we have v'
=> _!_=-
0 =mv, + 2m{-v,) v' 4
'
=> mv1 =2mv2 ... (1)- => ~=.!
Now, according to de-Broglie relation, we have v, 2

1-=!1. 78. [DJ


p
21A
Force=~
=> C
=;, Fx=(iKx')xa
73. [CJ
The vector is perpendicular to initial velocity vector at t = _u_ => (2t)x=¾Kx'
gsm8
and at this instant its speed is
v =ucot0
=> X=i(~)
Since, A= J!...
mv 79. (AJ
KE.=EK =hv-hv0 =hv-lj,0
=> l.=--h-
mucot0 he
=> EK =--lj,o
1-
( 4.14 X 10"" )( 3 X 10')
=> A=(!u}an9 => E, - 5.01
2x10-7
=> E, = 6.21-5.01
=> E,=1.2eV
So, stopping potential required is 1.2 V

= 3.56 ================================:::i
www.puucho.com
Rahul Sardana with www.puucho.com

Dual Nah,re ofRadiatio11 & Matter


BO. [DJ 90. [CJ
For ti.yo partiCles to have same de-Broglie wavelength The time rate of change of momentum equals force.
~mqV =constant => f=6p
~

::::;, mpqpV=m.. q V' 6t


0
Force per unit area is pressure
V'=mp Qp V
m.. Qo
=> 9" =.! 6p
A 61
V'='!_ But for a photon E =pc , where
8 p = momentum of photon
E = energy of photon
81. [CJ 6E= e6p
Energy of each photon,
E he 6.6x10-34x3x10 8 => go=_l_6E=_:t_(~)=.!{I)
Ae6t eMt e
l. 300x10·'
Because irradiance I is defined as energy per unit area per unit
E=6.6x10-19 J time.
Power of source
P=IA=1.0x1.0x10-' =10-' watt 91. [BJ
So, number of photons per sec is A., =AP
N P 10"' h he
E 6.6x10-19 mv=E
Number of electrons emitted is
1 10-' =>. .e.=-
E e
N'- x--'--'----
-100 6.6x10-19
::::) N'=1.51x1012 per second 92. [BJ
Stopping Potential V5 = Kmax
82. [AJ e
Transverse deflection is

1 E '
_ 1at'- 1(qE)(x)'
y2 2mv
_2'1"
mv 2
93. [AJ
E= he
!qEx2 l.
y 2 Number of photons emitted is

2-(Jm:1') (htrn,
Pt

For same electric field E, kinetic energy EK and length x, y a:: q ,


q is smaller for proton, so y is smaller for proton. So proton's P,t
trajectory will be less curved than a-particle's trajectory. => no =t,c
Since the radiation is spherically symmetric, so total number of
84. [DJ photons entering the sensor is n0 times the ratio of aperture
EK= hv-hv0
area to the area of a sphere of radius l .
EK ::::::::y, v::::::::x, hv0 =constant(k)
=n n(2d)' =Pl.Id'
=> y =hx-k (Equation of a straightline) N
=> o 41t£2 he £2

86. [AJ 94. (CJ


A=~ he
mv "i"=$,+3V, ... (1)

=> l.- 6.626x10_,_, he


21' =$,+V, ... (2)
( 10---11 )( 105 )
::::;, A.=6.63x10-a m Subtracting
he
88. [DJ
2V0 =n
EK =hv-hv0 V _ he
0 - 4/o.
is the equation of a straight line with slope h.
he
89. [AJ $, = 41'
EK=hv-hv 0 he he
l-0 = 41

3.57 =
www.puucho.com
Rahul Sardana with www.puucho.com

Advanced JEE Physics Optics & Modern Physics

which is again equation of a straight line with slo_pe (;) .

95, [DJ
104. [AJ
p = hv (for a photon)
C 5% of 100 Wis 5 Js-1
:::::, 3 _3 x 1 0-29 = 6.6x10-34v
8
3x10
=> 5=~(~)
v=·1.5x10 13
Hz n 5l.
=> 1= he
96. [DJ n 5(5.6x10-7 )
=>
Decreasing the A. of incident photon means energy of incident t = 6.626 ;10--" x 3 x 10 8

light is increased, so E1t increases and hence stopping potential


=> !!=1.4x1019
increases. t
97. [BJ 105. [BJ
E-- he_ he Since .1x.1.p - h
K - A Ao
=> (2x10-")ap=6.6x10"'
E = 4.14x10-15 x3x10 8 (_!_.:!_) => ~P =3.3 x 10-20 kgms-1 •
K 10-7 5 6
=> E, = 0.414 eV 106. [BJ
Wherever necessary we take h = 6.626 x10-34 Js v 0 = 5x10 14 Hz
15
:::::, h=4.14x10- eVs Since, c=va').
6
A.= 3x10
98. (AJ =>
5x1014
l.=~ => l. = 6000 A
mv
=> dl.=~~(v-')=~(-1)~-,dv 107. [A]
di mdt m dt Kinetic energy of recoil is
dl. h EK =hv-hv'
=> dt=- mv2 -a
=> E,=12.375-9.375
Also V=U+at
=> Et<=3eV
=> V=at {·: U=O}
dl. h h 108. [AJ
:::::, dt - ma2 t2 a - mat2
Also ma =eE l.=-h-
../2meV
dl. h Putting values of h, m, e, we get
=> dt=- eEt2
=> l.=~A
99. [AJ
=> l. = 12.21 A
l.=~
mv .Jv
Since electron is the lightest particle of these four. So A. 8 is
maximum.
{·:q,=q,)
100. [DJ
I a:: _!_
d' 110. [AJ

when d becomes f, I becomes 161 l>x =-1..._ m


100
Since
102. [BJ t,xt,p = h
hv=hv 0 +eV0 6.6x10--34
=>
=> eV0 = hv-hv0 (2/100)
h h => .6.p=3.3x10-32 kgms-1
=> V0 =-v--v0
e e

= ====================================
3.58

www.puucho.com
Rahul Sardana with www.puucho.com

Dual Nature of Radiation & Matter


111. [A]
. -h => N ;= _Total number of photons striking the totally reflecting

l. = ,/2mE screen = (~hr 2h


F F)..

=> mE = constant
me <mp <mca ::::) N=5x1026
=> E1 >E 3 >E2
117. [AJ
112. [BJ ,/2mE =~
).
E=hc_~ ... (1)
).
h' (6.6 x 1O""' )'
E=--
2E=hc_,_ .. ,(2) 2mA2 2x20x1.66x10--21x(6;6x10-10 )2
)! '
10 --48+27 1
E--'-'-- x10-"J
=> 2E - E = hc[1- _
).'
_!] = E
).
40x1.66 4x1.66
E=1.5x10""" J
E+hc=hc
). ).'
118. [CJ
E>. + he he
---=- 2nh =1
). ).' ).

l.'=(~)
E>.+hc n=-
).

2h

113. [AJ 119. [BJ


In order for scattering to occur, the wavelength of the waves
must be of the same order of magnitude or smaller than the size Since, ~ =-½mv +cl> 2

of the object being observed. Hence the largest possible


he 1
wavelength we can use in the present problem is Amax =2.5 A .
2
=> ~=T-2mv
Hence minimum energy is
1240
he => ~= -1.68=1.41 eV
Em1n=,:-- 400
=
E _ 12.40x10' vA 120. [AJ
mill- 2.SA Both are independent of each other.
Emln = 4.96x103 eV 121. [BJ
Emin = 5 keV For a photon
E=pc=hv
114. [A] hv
=> P=-
mc2 =m 0L c

(1)(3x1o')' =m0 (80x4200) 124. [AJ


=> m0 =.2.67 x 1011 kg s =~
• m,
115. [DJ
s =~
Density of star of mass M0 , radfus R0 is ' m,
M
p=--·- S=2e=e=1s
i1tR3 a4mp2mp2p
3 0

On contracting the new mass becomes Se is maximum, then comes SP and then S"

M = .'!_nR'p = M0
3 R0
(_!!._)' 125. [DJ

Loss in mass due to contra"ction = -AM I oc _!_


d'
=> Loss =M0 -M On doubling the distance the intensity becomes one fourth i.e.
=> Energy radiated = c2 AM only one fourth of photons now strike the target in comparison to
the previous number. Since photoelectric effect is a one photon-
one electron phenomena, so only one-fourth photoelectrons are
116. [DJ -
emitted out of the target hence reducing the current to one fourth
the previous value.
F=~ = ~t[i-(-i)]=N(~)
{ ·.- Photons rebound with the same initial value of momentum}

3.59 =
www.puucho.com
Rahul Sardana with www.puucho.com

Advanced JEE Physics Optics & Modern Physics

126. [DJ V =he_ W { ·: W = work function}


Let threshold frequency be v0 • s eA. e
v=1.5v 0
134. [CJ
When the new frequency is halved
v' =0.75v 0 < v0 i.(in A) 12400
E(in eV)
So, no photoelectric effect takes place.
12400
1.= =3100A=310 nm
127. [DJ 4
Ap=m,.va
135. [AJ
A,. mp Vp
E=nhv
=> _!=4v"
2 v, => f =Unhv

=> Vp
v.
=8
=> 10000 = T( 6.6 x 10-" )( 880 x 1000)
128. [CJ => ~=1.71x1031
t
2
Y
=..:!.at2 1 eE l
2 =2m v2 136. [CJ
For same. (near1y) circular path y is same Maximum microwave frequency= 3 x 1011 Hz
E
=> 2 =constant . E hv
Smce, p=-=-
V
c C
Here E=x
( 6.626 X 10°"' )( 3 X 1011 )
=> =>
p_ . (3x10')
=> x'=4x => Pma,c = 6.626 x 1o-31 kgms-1

130. [BJ 137. [CJ


Given that !1. = he = ...!1... A.=·~ 6.62x10-34
p E mv mv 9.1x10--31 x3x10 7
=> E=mvc
=> A=0.24x10-10 m
1 mv2
Ee =i__=_!_ 138. [CJ
EP mvc 2c
(m-mo)c2 =2moc2
131. [BJ => m=3m0
Amax= 7500 A
140. [CJ
So, E = he Stopping potential= 1 V
1.
=> EK=1eV
E= (4.14x10-" eVs)(3x10' ms-1 )
7500x10-10 m => i mv 2 =1.6x10-19 J
E=1.6eV

133. [BJ
eVs =KEmax =hv-W

= 3.60 ==================================
www.puucho.com
Rahul Sardana with www.puucho.com

1. [A, B, CJ Q
hv =KE.(T) + Work function (W) n=-
e
=> hv=T+W ---2ons--._
=> 4.25 eV = TA + WA (for Metal A) ----------------~---.
=> 4.70 eV = T, + W, (for Metal B)
Since T, =(TA -1.5)eV r=
h
Also A=-
p
J_________ _
8 x10-13
=> l.=-h- :::::i, n 1.6x10_,g Sx10s
-J2mT
Then the number of photoelectrons emitted per light pulse is
=> ).A = f t n ' = ~ =5
Aa TA 10'
s·ince 1 and hence the number of photons in one light pulse is
AA =2A 8
N=~=SO
=> TA =4T8 0.1

=> T, = TA -1.50 gives 8. [A, 8, C, D]


=> T8 =4T8 -1.5 According to Heisenberg's Uncertainty Principle the product of
(a) uncertainty in position and uncertainty in momentum
=> T, = 0.5 eV
h
=> TA =2 eV cannot be greater than - .
4s
=> WA =2.25 eV (b) uncertainty in energy and uncertainty in time cannot be
=> W, =4.20 eV h
greater than - .
4s
2. B,D (c) uncertainty in angular position and uncertainty in angular
h
Since 'PP=.!.=104 Nm-2 momentum cannot be greater than - .
C
4s
(d) uncertainty in generalised coordinate and generalised
h
momentum cannot be greater than - .
4,

10. [A, BJ
3. [A, B, CJ Wavelength of UV radiation is less than 5200 A where as
The total quantity of charge carried by one pulse of current is wavelength for IR radiation is greater than 5200 A. Hence
f
Q = Idt photoelectric effect will be shown by UV radiation irrespective of
its intensity.
which is the area of the triangle in figure. Thus
Q =!(2ox1o"')(sox10-<>) = Bx10-" C 12. B, C
2 For a photon
and the number of electrons carried by one pulse is pc-= hv

3.61 =
www.puucho.com
Rahul Sardana with www.puucho.com

Adva11ced JEE Physics OpHcs & Modern Physics

hv mv'
=> p=- Since, qvB = - -
c r
::::> p=8.8x10..-28 kgms-2
::::> B = mv =~2mKmax
=> p=1.65x10.. MeV/c qr qr

=> 8
= .J2 x 9.1 x 10-a1 x ex 1.5
14. [A, B, CJ ex1
(A) f=~ 3x108 sx10-14 3x91x10-31
l. 600x10.. => B= .
1.6x10-19
(B) N=£_ => B = 4.13x10-' T ~ 4 µT
hf
X-rays are of order of 0.1 nm= 12.4 KeV
(C) KE = 1240 -1.07=1
max 600
22. [B, DJ
(D) KEmax depends upon frequency of incident photons and Energy of photon incident
not distance of source.
hv = 12400 eV
4000
17. [B, DJ
=> hv=3.1 eV
A.red > Avio1et Since, hv < work function of all metals
VIBGYOR pattern shows that VIBG all have l. less than that of Hence no electron will come out
yellow colour and hence can initiate photoelectric effect If 1.. = 200 nm
irrespective of intensity.
12400
then hv =he= = 6.2 eV
18. [A, BJ 1.. 2000
Since, 6.2 eV > work function of all metals, hence photoelectron
K =4x10-19 J= 4x10-19 2.5 eV
max 1.6 X 10-19 will be emitte:)d.

=> Stopping potential = 2.5 eV 23. [B, DJ


Cut off voltage is independent of intensity and hence remains
Since, Km,. =hc(l_..!..)
l. l.,
the s~me. Since distance becomes 3 times, so I becomes ½.
=> 2.5 = 1240 _ 1240
Hence photocurrent also decreases by this factor i.e. becomes
300 A. 0
:::::> A0 =759nm ~=2mA
9

19. [C, DJ 24. [CJ


Infrared light has wavelength greater than 5200 A and UV has Work function is the intercept on K-axis i.e. 2 eV
A< 5200 A. So UV lamp will be able to get the photoelectrons
25. [AJ
emitted by a surface irrespective of intensity.
2 = 4.14 x 10-15 v
20. [A, CJ => v=4.8x1014 Hz
Kmax =1.5 eV
26. [A, BJ
l..=-h- Photoelectric effect and Compton effect are explained on particle
,J2mK nature of light i.e. light is considered to be made up of a stream
::::::> A=1nm of photons.

E of incident photon = ~~!


1
::: 2.5 eV

=> ~=2.5-1.5=1 eV

= ====================================
3.62

www.puucho.com
Rahul Sardana with www.puucho.com

6. [DJ 15. [AJ


l. =-
h
p
, Same for both m=Hm,
---,-
8. [DJ
m,
Kinetic energy; E =.:!.mv2
2
=qV
2m
,=H 1--
c'
v ex: ~ because V is constant
V=,/3C
2

16. [DJ
K= =hV=~
KE of emitted photoelectrons varies from ze_ro to ~

=> v,:v,:v.=1:~:~="2:1:1 17. [DJ


,. =12};1 A
13. [AJ
Energy of photoelectron emitted is different because after
absorbing the photon electrons within metals collide with other
atom before being ejected out of metal.

14. [AJ

~=J E 2 =J1o'eV =1
A.ph 2rn8 c 1MeV

==================================== = 3.63

www.puucho.com
Rahul Sardana with www.puucho.com

1. [CJ Efficiency of photoelectron generation per incident photon is


. ·
Smee, Kmax =T- 1he he
~ =(~x100%) = 1s xhc x100
n ex PA

..!mv2 =he_ he 2Q X 1 a-e·X 6.6 X 10-34 X 3 X 1 Q6 X 1QQ
... (1)
2-max. ).1 ,..Ill f\= 1.6x10-19 x1x10-3x3000x10-10

..!mv2 =he_ he ••• (2) ~=(~)(66)= 66 =8.25%


2 maxz A2 A111 · 160 10 8
Dividing (2) by (1 ). we get
4. [CJ
he he

[::J ~~:~~ =
).1 ).!h
mv'
-=qvB
r
=> ~2mEK = qBr
E =(qBr)'
1500 '• K 2m
(2)'
1 1 E. =0.86 eV
3000 '•
4 4 1 1 5. [BJ
3000-,. =1500- ••
4
1x2 3
E3 -E, =13.6( ; -
2 3
1,)
------=-
3000 1500x2 '• => ,1.E=1.B9 eV
Since, if,=E-Kl!IIUI
2 3
=> --=- => ~=1.89-0.86=1.03eV
3000
'•
=> '• =4500 A 6. [AJ

2. [AJ 1.=~
. ,1.E
l 8 oc PA
=> , = 6565 A
=> I~ = P2 A2
r, P1A1 7. [BJ
1
~= 5x1500 1.oc,=
=> ;1mq
20 1x 3000
=> I~ =50 µA
8. [AJ
3. [BJ
i. ·= 12.21 A
For case I, we have
•.. ..Jv
P,
n=-· 9. [CJ
he
Since, A=~
Since, n
'
=1..e p

= =================================
3.64

www.puucho.com
Rahul Sardana with www.puucho.com

Dual Nature of Radiation & Matter


Further, Pe1ee1rcn = pat>ha 17. [CJ
. :::::,. Wavelength will be same L\p=PA.6.t=10--4x10--4x10=10-7 kgms· 1

10. [BJ 18. [BJ


J. = ..!!_ = 6.63 X 10" Nh
-i:-=t.p
mv (66.3x10")(5)
=> l. = 0.02 m N 10-7 x663x10..g
1020
6.63 X 1Q--:W
11. [DJ
Fringe width p is given by 19. [AJ
P=l.D 12xo.02 Since,~ =hv-W
d 0.6 If KITIII)( and W both are in eV , but hv is in joule, then we have
=> p = 0.4 m

12. [DJ
K
= = hve -W
So for both the frequency values, we have
de-Broglie wavelength of electron is
_ h(Bx10") W
l.- 6.63x10" 05 ... (1)
9.1x10--31 x10 7 e

A. ::::,-1021 m h(12x10") W
:::::i- 2 ... (2)
e
21 22
Fringe width p = AD :::: (~) x 10 z 10 m Subtracting equation (1) from (2), we get
d 0.6
h(4x10")
Fringe width is so large that it is not possible to observ~ it. 1.5
e
13. [BJ h 1:sx1.ex10·19
4x1014
l.= ~ = /150"=10A=100nm
v2mE Vii h = 6x10-34 Js

14. [AJ 20. [CJ


mv' From equation (1 ), we get
qvB=- _ = (6x10"')(Bx10") W
r 05
1.6x10·19
=> B= mv =4.1x10-a T "=> 0.5=3-W
qr
=> W=3-0.5"=2.5 eV
15. [AJ
21. [BJ
Photon energy = ~ = :::: = 2.5 eV
1
l.=-h~= 12.21 A
Since, Va = 1.5 V
,/2rnK ,/v
1
So, work function is given by => l.= ~ A=12.27../2 A
~=2.5-1.5=1 eV -.,0.5
12375 => l.=17.35A
=> l.'""=--=12375A=1237nrn
1
:::::,. =
"-min 1250 nm

16. [BJ
_I_ 3x104 _ ...i p
P-c-3x108 -
10 a

3.65 =
www.puucho.com
Rahul Sardana with www.puucho.com

1. A--> (s) (C) de-Broglie wavelength of most energetic photoelectrqn


B--> (q. s)
A 1·227 x 10-9 1.227x10-s =12.2x1~-io m
C--> (p, r) -Fi
D--> (p, r)
).=10A
2. A--> (s) (D) . E=h:
B--> (q)
C--> (p, r) 12.4x10·1 m
=> ).=hc=hc
D--> (p, r) V .
E eV
3. A --> (p, q) V=2.5 V
B--> (r, s) E=
12.4x10·'=5000A
C :-> (q) 2.5
D--> (p)
5. A--> (p, q)
4. A--> (p) B --> (p, r, s)
B--> (r) C--> (p, q)
C--> (s) D--> (s)
D--> (q)
de-Broglie wavelength of electron in X-Ray tube 6. A--> (p, r)
).=-h-=_h_ B--> (q, s)
,/2mKE ,J2mev C--> (s)
D--> (p, r)
). = 1.221x10·' 12.21 A
=>
-Fi -Fi 7. A-> (q, s)
(A) Accelerating potential B-->(p,r)
V =1o• eV in X-ray tube .C-->(p,r)
12 7 D--> (s)
A= - ~ x10-a =1.227x10·11 m
10
8. A--> (r)
).=0.1A B--> (s)
(B) Wavelength associated with X-rays C--> (p)
).. 12.4x:o-7 -12.4x10·11 m D--> (u)
10 E--> (q)
A=1.2x10·10 m F--> (t)

).=1A

= 3.66 ==================================
www.puucho.com
Rahul Sardana with www.puucho.com

1. The target area is S,=n(10-9Y2"=1ex10-18 m 2 ~- 6.2x10-9


The area of a 5 metre sphere centred on the light source is,


r- 2x10·10 31

s, =4,(s)' =100 ,m'. 4. The maximum kinetic energy of the emitted electron is
Thus, if the light source radiates uniformly in all directions the K =.!.mv 2 = he -W,
rate P at which energy falls on the target is given by, max 2 """' A

P = (10-" watt)(§.)= (10-"


S2
>("' 10-") = 10-zi Js-•
100xn
=>
6.62x10-34 x3x10 8
180x10·9
2x1.6x1Q·19 = 7.8x10·19 J

Assuming that all power is absorbed, the required time is, => V =~Kmax = 2x7.8x10·19 =1.3x106 ms-1
max m 9.1 X10--31
t=( SeV )(1.6x10-"J)~ 2ohr The magnetic field provides the centripetal force to the electron,
10-23 Js-1 1 eV so
2. Rate of energy received from the sun
Q = 2 cal cm-2 min-1 = 2 x 4.2
Bev
= = mv2"""'
r
1 6
r= mvmaK = 9.1x1_0...a x1.3x10
=> Q = 8.4 J cm-2 min-1 => 0.148m
Be 5x10-sx1.6x10·19
Energy of a photon received from the sun is
9 => r=148mm
E he 6.6x10--34x3x10 x _, J
10 3 10 9
A 6600x10- 5. The Einstein's photoelectric equation for the first case can be
If n is the number of photons reaching the earth per cm 2 per written as

minute, then their total energy will be (3 x 10-19 n) joule.


he
-i:-=W+K, ... (1)
=> (3x10-")n = 8.4
When illuminated with light of wavelength 2A. ,
8 18
=> n = x ·~_19 = 28 x 10 photons per minute per cm 2 he
3 1 2A =W+K2 ... (2)
=> X=28 Subtracting (2) from (1), we get
3. Mass of an electron is hc_hc=K -K
A 2A , 2
m=9.11x10-" kg and T=27+273=300K
=> l. he (4x10-")(3x10')
de-Broglie wavelength of electrons is
2(K,-K,) 2(30-10)
1-~h~
- ./3mkT => A=300x10·10 m=300 A
If "-max
--,.----------===
J3
6.63x10-"'
X 9.11X10--3I X 1.38 X 10·23 X 300
m
is the maximum wavelength of the photons with which
photoelectrons can be emitted, then

6.63x10-s 6.63x10-s ~=W =hc_K (4x10-")(3x10') 3 0eV


=> 1 0 1 10
A/1\alC A 300x10·
./3x9.11x1.38x3 10.64
=> E=40-30=10eV
=> A=6.2x10-9 m
Mean separation between two electrons in a metal is => 1 =~ (4 x10-")( 3 x10') 12oox10-" m
10 eV
r=2x10· 10 m 10
rnax

=> "= =1200 A


3.67 i:::::J

www.puucho.com
Rahul Sardana with www.puucho.com

Advanced JEE Physics Optics & Modern Physics

6. Since Kmax = hv-hv 0 => hv=7eV


So, total number of photons incident is
=> ¾ m(8x10 6 ) ' =h(Sv,-v,) ... (1) p
n=-
hv
For the second case, we have
Since, Tl= 1O...'.!%
KE= ¾mv' =h(2v, -v,) ... (2)
10"
=> nemltted = n X 1QQ
(8x10')' -4
v'
V=4x10 6 ms-1
=> i=7=("~ )e
where, from the graph
1=10x10"' A
7. F =(Area).!. here effective Area = nR2
C 10x10-e =~x 10-3 x1.6x10-19
=> F = (nR').!. hv 100
C
10 10
x -e = Px10-3 x1.6x10-19
=> 22
F=-x(21x10--2 )' x 1- x -1- , 7x1.6x10"19 x100
7 110 3x10 P=7W
=> F (22){21x10")(21x10")
7x110x3x108 10. According to Einstein's Photo-Ele~tric Equation, we have
=> F =42x10- 13
N Kmax =eV, =hv-W
=> X =42 => eV0 =12 eV-4 eV
=> eVs = 8 eV

8. Since, eV, = he - ~ and => V, =8 V


i.,
he 11. Magnetic force experienced by a charged particle in a magnetic
eV,=--~ field is given by
i.,
Subtracting, we get Since, r= mv
qB
e(V, -V,) =hc(I., -1.,) The de Broglie wavelength is given by
i..,)..2
I.=_!!_=~
V,-V,=hc(l.,-1.,) mv qBr
e A.1A2
~ = qaro.
V. -V =6.6x10-" x3x10 x ~
34 8 =>
=> 2 1
1.6x10-19 6x10-o
).o. QlP

=> v - v = es x 1o-34•8+-2+19+s Given, _!'g__=1 and q0 =2


rP q.,
2 1 32
· 33
=> V2 -V1 =-;::;2V
16

9. Since, eVa =hv- W


=> 5eV=hv-2eV

= 3.68

www.puucho.com
Rahul Sardana with www.puucho.com

- }

Atomic & Nuclear


Physics

(a) ATOMIC PHYSICS ........................................................................................................4.1

(b) NUCLEAR PHYSICS ..................................................................................................4.21


Solved Practice Problems ..................................................................................................4.48
Practice Exercise Sets
:> Single Correct Choice Type Questions ..................................................................4.58
:> Multiple Correct Choice Type Questions ...............................................................4.73
:> Reasoning Based Questions (Assertion Reason Type) ........................................4.78
:> Linked Comprehension Type Questions (Paragraph Type) .................................. .4.81
:> Matrix Match Type Questions (Column Matching Type) ........................................4.89
:> Integer Answer Type Questions ............................................................................4.92
Answers to In Chapter Exercises (ICE) & Practice Exercise Sets .....................................4.94
Solutions to In Chapter Exercises (ICE) .............................................................................4.99
Solutions to Practice Exercise Sets .................................................................................4.106

www.puucho.com
Rahul Sardana with www.puucho.com

STRUCTURE OF AN ATOM : AN INTRODUCTION spectral line. Experimentally, hydrogen is found to give


All matter is made up of tiny particles known as atoms. several series, each series consisting of several lines. This
There are about 105 different kinds of atoms, and they indicated that Thomson's atom model needed modifications
combine with each other in different ways to form groups which was modified by Rutherford.
called molecules. All matter has been found to be composed
of atoms or molecules, and the basic knowledge of atoms RUTHERFORD'S ATOMIC MODEL
and their constitution gives us valilable information about The correct description of the distribution of positive and
the behaviour of matter. negative charges within an.atom was made in 1911 by a New
Zealander while working at Manchester University in
THOMSON EMPIRICAL MODEL / THOMSON PLUM England. This was Ernest Rlftherford, who was later called
PUDDING MODEL as Lord Rutherford for his many scientific achievements. He
J.J. Thomson gave the, first idea regarding structure of atom. entered into physics during that crucial period of its
The model is known after him as Thomson's atom model. development when the phenomenon of natural radioactivity
According to this, entire positive charge is distributed had just been discovered, and he was first to realize that
uniformly in the form of a sphere. Negatively charged radioactivity represents a spontaneous disintegration of
electrons are arranged within this sphere lying here and heavy unstable atoms.
there. The model is popularly known as plum-pudding Rutherford realized that important information about the
model. Every electron is attracted towards the centre of inner structure of atoms can be obtained by the study of
uniformly charged sphere while they exert a force of coll!sions between the rushing a particles incident on the
repulsion upon each other. The electrons get themselves atoms of various materials that forni. the target on which the
arranged in such a way that the force of attraction and that of a. particle beam is incident.
repulsion balance each other. When disturbed, electrons
vibrate to and fro within the atom and cause emission of EXPERIMENTAL ARRANGEMENT
visible, infra-red and ultra-violet light.
The basic idea of the experimental arrangement used by
Positively Charged Rutherford in his studies was explained as follows:
Matter
A piece or speck of a-emitting radioactive material is placed
in a lead shield with a hole that allows a narrow beam of the
a-particles to pass through it. In front of this arrangement is
placed a gold thin metal foil to deflect or scatter the a
particles. After passing through the gold foil the deflected
particles are incident on a pivoted fluorescent screen with a:
magnifier through which the tiny flashes of light were
observed whenever an ex-particle struck the screen. The
whole apparatus placed in an evacuated chamber, so that the
Thomson's atom model
particles would not collide with air molecules.
Thomson's atom model satisfied the requirements of the
atom and the demands of electro-magnetic theory.
According to this model, hydrogen can give rise to a single

4.1 =
www.puucho.com
Rahul Sardana with www.puucho.com

Advanced JEE Physics Optics & Modern Physics

Zinc Sulphate~
Screen
\ Microscope
/_____/-
:~:---7-_~
C••----
:=J©
~----- ~:: :::::::;-------
\~
Gold Different deviations for different a particles
a-Particle Source
Foil Also during the experiment following conclusions were
made
OBSERVATIONS
(i) If $ is the.angle made by a scattered particle with
Rutherford performed experiments on the scattering of alpha
its original direction of motion and N is the
particles by extremely thin metals foils and made the
number of particles available in that direction, it
following observations.
was found that,
a) Most of the a-particles, either passed straight through 1 .
the metal foil or suffered only small deflections. Titls - - ~ oclog,N
could not be explained by Thomson's atom model. sin 4 .!.
2-
b) A few particles were deflected through angles which
were less than or equal to 90° . (ii) If t is the thickness of the foil and N is the
number of a-particles scattered in a particular
c) Very few particles were deflected through angles
greater than 90° . It was observed that about only 1 in direction ($= constant) it was observed that ·
8000 particles was found to be deflected greater than N
-=constant
90° . Sometimes a particle was found to be deflected t
through 180° . In other words, it was sent back in the
=> N1 =!i_
same direction from where it came. The large angle of
N2 t2
scattering came as a greater surprise. It could not be
explained by Thomson's atom model. It was one of the
main reasons for rejecting Thomson's atom model. RUTHERFORD'S ATOM-MODEL POSTULATES
On the basis of the conclusions drawn from Rutherford's
CONCLUSIONS experiment, a new atom model was proposed. Titis atom
a) The fact that most of the a-particles passed undeviated model, known as Rutherford's atom model, had the
led to the conclusion that an atom has a lot of empty following characteristics.
space in it. a) An atom consists of equal amounts of positive and
b) a-particles are heavy particles having high initial negative charge so, the atom, as a whole is electrically
speeds. These could be deflected through large angles neutral.
only by a strong electrical force. Titls led Rutherford to b) The entire positiv_e charge of the atom and practically its
the conclusion that entire positive charge and nearly the entire mass is concentrated in a small region which
entire mass of the atom were concentrated in a tiny forms the core of the atom, called the nucleus.
central core. Rutherford named this core as Nucleus. c) The negative charge, which is contained in the atom in
c) The difference in deflection of various particles can be the form of electrons, is distributed all around the
explained as follows: nucleus, but separated from it.
a-particles which pass at greater distances away from d) In order to explain the stability of electron at a certain
the nucleus, shown as A and A' in figure, suffer a small · distance from the nucleus, it was proposed by
deflection due to smaller repulsion exerted by the Rutherford that the electrons revolve round the nucleus
nucleus upon them. The particles like B and B' which in circular orbits. The electrostatic force of attraction
pass close to the nucleus experience a comparatively between the nucleus and the electron provides the
greater force and hence get deflected through greater centripetal force to the electron to revolve in the orbit.
angles. A particle C which travels directly towards the e) The nuclear diameter is of the order of 10·" m . Titls
nucleus is first slowed down by the repulsive force. can be calculated as follows.
Such a particle finally stops and then, is repelled along Let an a-particle having velocity v approach a nucleus
the direction of its approach. Thus, it gets repelled back (head-on) having a charge +Ze . The velocity of
after suffering a deviation of 180° . a-particle decreases till it comes to rest at a distance r0

= 4.2

www.puucho.com
Rahul Sardana with www.puucho.com

Atomic Pl,ysics
from the nucleus. It is, then, repelled back along the So it must radiate energy continuously. Bohr calculated
direction of approach r0 gives the radius of nucleus.
1 that this emission of radiation would cause the
electrons in an atom to lose all their energy and fall into
Initial K.E. of a-particle =.!.mv' the nucleus within a hundred - millionth of a second
2
following a spiral path. Thus the whole atomic structure
Initial P.E. of a-particle =0 should collapse. Since matter composed of atoms exists
Final K.E. of a-particle =0 permanently as far as we know, there was obviously
1

something wrong here.


Final P.E. of a-particle =-l_q,q, b) According to Rutherford model, electron can revolve in
4ru:o To
any orbit. So, it must emit continuous radiations of all
By Law of Conservation of Energy frequencies. But elements emit spectral lines of only
(U +K).,;._ =(U+K)....,, definite frequencies.

=> .!.mv' =-l_q,q,


2 41te0 r0 Illustration 1
An isolated hydrogen atom emits a photon of
r,
0
= _3.__(q,q,)
2 10.2 eV.
4ne 0 mv
(a) Determine the momentum of photon emitted
For a-particle, % = 2e
(b) Calculate the recoil momentum of the atom
2 (2Ze') (c) Find the kinetic energy of the recoil atom. [Mass of
=>
To = 41tEo mv2 proton, ,nP =1.67 x 10-21 kg ]
Solution
=> r,0 =-4-(Ze' )=4x9x109 ze2
41te0 mv 2 mv2 (a) Momentum of the photon is
In one of the experiments, a-particles of velocity E 10.2 x 1.6 x 10-" 5.44 x 10-21 kgms-'
2 x 10 7 ms_, were bombarded upon gold foil with P, =-;; 3x10'
Z=79 (b) Applying the Law of Conservation of Linear
So, for Z = 79 Momentum, we get

e=l.59x10-" C
m=4xl.67x10-27 kg
P,-0 ~
photon
P,
atm
v=2x107 ms-1 P, = p, = 5.44 x 10-27 kgms-'
we get,
(c) K = ..!..mv2 ( v = recoil speed of atom, m = mass of
79 X (1.59 X 10-,0 )' 2 /
r0 =4x9x109 x 2 hydrogen atom)
4xl.67xl0'27 x(2x10') 2

=> r0 = 2.69 x 10-14 m => K =.!m(.E..)


2 m
=]!_
2m
This gives the order of the radius of nucleus. Substituting the value of the momentum of atom, we
get
FAILURE OF RUTHERFORD MODEL
·(5.44x10-21 )
2

a) According to laws of electro-magnetic theory, a charged K = 2 X 1.67 X 10-27 8.86 X 10-27 J


particle in accelerated motion must radiate energy in
the form of electro-magnetic radiation. As the electron
revolves in a circular orb!t, it is constantly subjected to BOHR'S ATOMIC MODEL
2
centripetal acceleration ~.
r BOHR'S THEORY OF THE HYDROGEN LIKE ATOMS
To rectify the drawbacks of Rutherford Model, Bohr
proposed a theory which applies to hydrogen atom and
species like He+, Lr-+ etc. Here a single electron revolves
around a stationary nucleus of positive charge Ze where
Z =1 for hydrogen atom, Z = 2 for He+ etc. Bohr in
defiance of the well-established laws of classical mechanics
Electron spiralling inwards

4.3 =
www.puucho.com
Rahul Sardana with www.puucho.com

Advanced JEE Physics Optics & Modem Physics

and electrodynamics, proposed the following postulates to mv' 1 (Ze)(e)


... (1)
support his atomic model. r 41t&o ,2
nh
A. CIRCULAR ORBITS and mvr=- ... (2)
2n
The atom consists of central nucleus, containing the entire
positive charge and almost all mass of the atom. The
electrons revolve around the nucleus in certain discrete
From(2), v=--
nh
2nmr
circular orbits. The necessary centripetal force for circular Put in (1), we get
orbit is provided by Coulomb's attraction between the nhe 2 2
electron and nucleus. So, r
II
=--'
1tme2 Z
mv2 1 (Ze){e)
n' •
r 4ne0 r
2
=> r, =(0.53) A
2
where, m = mass of electron, So, for H-like atoms
r = radius of circular orbit, n'
v = speed of electron in circular Orbit, r oc-
' z
Ze = charge on nucleus,
Z = atomic number, H. VELOCITY OF ELECTRON IN NTH ORBIT
e = charge on electron =-1 · 6 x 10~19 C . nh
Smce, v = - -
2nmr
B. STATIONARY ORBITS
n 2 h2 s
The allowed orbits for electron are those in which the and r=--'
. 1tme2 Z
electron does not radiate energy. These orbits are also called
stationary orbits. => ( e'
v= 2hs
0
)z --;;

C. QUANTUM CONDITION
(BOHR'S QUANTISATION RULE)
The stationary orbits are those in which angular momentum
=> v=( 2::,c )( c:)
of electron is an integral multiple of _!:_( = h)
2n
=> v=a(c:)
e'
i.e., mvr = n(;1t), n being integer or the principle quantum where a= - - is the fine structure constant (a pure
21Js 0c
number . 1
number) whose value is - .
137
D. STATIONARY NUCLEUS
The nucleus is so heavy, that its motion may be neglected. => v=(1~7 )c:
E. CONSTANCY OF MASS i.e. velocity of electron in Bohr's First Orbit is _c_ in
137'
The mass of the electron in motion is assumed to be constant.
Second Orbit is _c_ and so on.
274
F. BOHR'S TRANSITION RULE
When an electron jumps from one stationary orbit to I. KINETIC ENERGY OF ELECTRON (EJ
another, a photon is emitted or absorbed having energy
Since, we have
equal to the difference of energies between initial and final
states and being given by mv' 1 (Ze)(e)
E;-E1 =hv r 41tso ,2
E;-E1 1 2 Ze2
=> v=--- => -mv = - -
h 2 81ts 0 r
1 2 Ze 2
G. RADIUS OF ORBIT => EK=-mv = - -
2 81ts0 r
Since, we have

,--, 4.4

www.puucho.com
Rahul Sardana with www.puucho.com

Atomic Physics
J. POTENTIAL ENERGY (U) OF ELECTRON IN nth level (n = 1) is known as the ground state. The energy level
ORBIT
corresponding to n = 2 is called the first excited state and so
u 1 (Ze)(e) · on . In this diagram zero energy level corresponds to n = oo
41te0 r which is the ionized state of the atom.
Ze 2
=:, U=---
41te0r Illustration 2
Using Bohr's theory show that when 11 is very large
K. _ TOTAL ENERGY (E) OF ELECTRON IN nth ORBIT the frequency of radiation emitted by hydrogen atom due
to transition of electron from n to (n-1) is equal to
Total Energy= K.E. + P.E.
frequency of revolution of electron in its orbit.
=:, E= Ze' _ Ze' Solution
81te0 r 4m;0 r
FrequenCj' of revolution electron in nth orbit is given by
Ze2
=:, E=---
8m:0r
So, we conclude that
l V (2::,c)~
lre,ol,tioo =r= 21tr = 21t(n'h_'•o)
1 1tme2z
Total Energy= -K.E. = (P.E.)
2 me' ) z
2

f revolution = ( 4e~h3 n3 ... (1)

Further, frequency of transition from n to (n -1) is

E=-( 87,::i) !: me'z'[ 1 1]


lif=-- - - -2 - -
Bh'e: (n -1) n'
Z' 4 2
E=-(13.6), eV me z [ 2n-1 ]
n /if= Bh'•i. n 2 (n-1)2
~so E=-( "!e4 )ch 2:
8e0 dr 3 . n
When n is large then
2n-lee2n and n-leen
Z' me 4 z2 2n
=:, E = -(Reh)-;;, =:, /if-=.,,
2 4
Bh e~ 11

where R=Rydberg'sconstant = ,~e


Be,ch3
4
=1.097xl07 m-1 and =:,
f transition =( 4
me'
E~h3
)z'~ ... (2)
Rch=Rydberg's Energy =2.17xl0-18 J=l3.6eV is the
So, from (1) and (2), we observe that for large n,
electron energy in first orbit of H atom.
lrevolutJ.on =!transition (between adjacent levels)
An energy level diagram for the hydrogen atom (Z = 1) is
shown in the figure. This Principle is also called "BOHR'S CORRESPONDENCE
E PRINCIPLE".

Free Electron l . Illustration 3

oevl ==~r-;:;:;-;:;:;;:rt
n s
n 4 Bound Electron
I
Assume a hypothetical hydrogen atom in which the
potential energy between electron and proton at separation
-1.51 eVt-----n=3
--3.40 av n= 2
r is given.. by ~ =k( log,. r-½), where k is a constant. For
such a hypothetical hydrogen atom, calculate the radius of
nth Bohr's orbit and energy levels.
Solution
Force of interaction between electron and proton is
given by
-13.6 av 1 - - - - - n = 1
The vertical axis represents energy. The (arbitrary) zero of
F=- dU = -k
dr r
energy is taken as the energy of a stationary electron,
infinitely far from the positive nucleus. The lowest energy
4,5 =
www.puucho.com
Rahul Sardana with www.puucho.com

Advanced ]EE Physics Optics & Modem Physics

Force is negative. It means there is attraction between the M =~) n/1 )nhBe = nhe
particles and they are bound to e~ch other. Titls force 2 2nBe 2nm' 4nm
provides the necessary centripetal force for the electron. So,
Since, U = -MBcos180°
we have
mv 2 k ~ U=nheB
... (1) 4nm
r r
According to Bohr's ass~ption, we have The angle between M and ii will be 180° because
instead of taking electronic current, we have to take
mvr=n(::) ... (2) conventional current which moves opposite to
electronic current.
Solving equations (1) and (2), we get
(d) E=U +K= nheB
2mn
r= 2" ~ and v=Jf h
(e)- [$[ = Bnr 2 = "
2e
Since, E = U + ½mv'

k k Illustration 5
E = klog, r-
2 + 2 =klog,r Light from a discharge tube containing hydrogen

So, r. = nh
21tvmk
=and E. =klog, ( nh )
2xvmk
= atoms falls on the surface of a piece of sodium. The kinetic
energy of the fastest photoelectrons emitted from sodium
is 0.73 eV . The work function for sodium is 1.82 eV .
Find
Illustration 4 (a) the energy of the photons causing the photoelectrons
An electron is orbiting in a circular orbit of radius r emission.
under the influence of a constant magnetic field of strength (b) the quantum numbers of the two levels involved in
B • Assuming that Bohr's postulate regarding the the emission of these photons
quantisation of angular momentum holds good for this (c) the change in the angular momentum of the electron
electron, find in the hydrogen atom, in the above transition, and
(a) the allowed values of the radius r of the orbit. (d) the recoil speed of the emitting atom assuming it to be
(b) the kinetic energy of the electron in orbit. at rest before the transition. (Ionization potential of
(c) the potential energy of interaction between the hydrogen is 13.6 eV).
magnetic moment of the orbital current due to the

l
Solution
electron moving in its orbit and the magnetic field B .
(a) From Einstein's equation of photoelectric effect,
(d) the total energy of the allowed energy levels.
(e) the total magnetic flux due to the magnetic field B Energy of :
passing through the nth orbit. · photons causing =[.J<ln.etic
Maximum
energy ( work )
the photoelectric of emitted + function
(Assume that the charge on the electron is -e and the [
emission photons
mass of the electron is ,n ).
Solution ~ E=K-+W=(0.73+1.82) eV
. mv ~ E=2.55 eV
(a) Smce,
. r=-
Be
... (1)
(b) In case of a hydrogen atom,
From Bohr's Quantisation Rule, we have E1 =-13.6 eV
nh E, =-3.4 eV
mvr=- ... (2)
2x E, =-1.5 eV
Solving these two equations, we get
E4 = -0.85 eV
r=) nh and v=)nhBe2 Since, E, - E2 = 2.55 eV
21tBe 2nm
Therefore, quantum numbers of the tw'o levels involved
(b) K= ~ niv' = nhBe in the emission of these photons are 4 and 2 i.e., from
2 4xm
4->2.
(c) . (e)( ') =-(
M=IA= .-
T "' ) "' =-
e ( ') evr
21tr 2
(c) Change in angular momentum transition from 4 to 2
will be ·
V

,___;, 4.6

www.puucho.com
Rahul Sardana with www.puucho.com

Atomic Physics
L>.L=L2 -L4
v=i=Z'R( _!_ __!_)
=> l>.L = z(!!__)- 4(_1!__) A ln/ n:
· 2n · 2n 1bis relation ·holds for radiations emitted by hydrogen-like
=> AL=_]!. atoms i.e. -
n H(Z=l),He'(Z=2),Li.. (Z=3) and Be... (Z=4).
(d) By Law of Conservation of Linear Momentum, we have
Momentum of) ( Momentum of) HYDROGEN SPECTRUM
( hydrogen atom = emitted photon
Sinc_e, wave number (v) is given by
E
=> mv = - ( m = mass of hydrogen atom) v=~=Z'i _!_~_!_)
C
A l n} n:
E
=> v=-, where R is th~ Rydberg constant.
me
The various transitions for the hydrogen atom are shown in
(2.55i(1.6 X 10-19 ) the following figure. All transitions starting from n = 2
=> V
(1.67 x10-27 )(3 X 108 ) onwards and ending at n = 1 belong to the Lyman Series.
=> v=0.814 ms-1 Likewise, all transitions starting from n = 3 onwards and
ending at n = 2 belong to the Balmer Series. The other
FREQUENCY OF EMITTED RADIATION spectral series' names are mentioned in the figure.
If electron jumps from initial state ni, to a final state n1 , then 0eV 0 =CO

' 11
' n=5
frequency of emitted or absorbed radiation v is given by II ••• Bracket n=4
-1.51 eV HJ n=3
E1 -E1 =hv. Pasch"eil
-3.40 eV Balmer
n=2
or V= E,-Er =Z'RJ _!_ __!_)
h lnJ n:
If c is the speed of light and '.l. the wavelength of emitted or
absorbed radiation, then

v=E.= ZRc(_!_ __!_)


2

A n} n: -13.6 e V - ~ ~ - - - - _ : _ _ - - - - - - - - n = 1
Lyman
So, Wave number (v) is given by

.
FIRST MAXIMUM
WAVELENGTH MEMBER- SERIES LIMIT, LINES
INITIAL STATE FINAL STATE WAVElENGTH FOUND ,
FORMULA SECOND n1 , - oo TO n,
MEMBER
(n,+1) TO n, IN

From oo to 1 From2to 1
z
< n;=2 to n,=1
UV
::;; n; =2, 3, 4, 5, 6, ... "1 =1 _!=R(_!_-2_)
2
n~
).=_!
R
).=..±_
3R Region
:'.i A 1 n;=3ton1 =1
i.=911 A A=1216A
a: From co to 2 From 3 to 2
w n1 =3 to n1 ;::2
::;; n; =3', 1,5, 6,,7, ... _!=R( _!2 _ _2_) I.=± ). = 36 Visible
.J n, =2
< A 2 n~ n1 =4 to n1 =2· R SR Region
Ill
i.=3646A i.=6563A
.-
z From oo to 3 From4h3
w
:c ni=4ton,=3
0
U)
n1 = 4; 5, 6, 7, 8, ... n, =3 _!=R(_1_2 _ _1_) ). =~ ).= 144 IR
<- .- A _3 n~ n;=Ston,=3 R 7R Region
a.
i. = 8204 A i.=18753A

4.7 =
www.puucho.com
Rahul Sardana with www.puucho.com

Advanced JEE Physics Optics & Modern Physics

From oo to 4 From 5to4


~
~
0
<
ni =5 , 6, 7, 8, 9, ... n,=4
A.
_l __l)
.i.=R( 2
4 n~
I\ =5 ton, =4'
n,=6ton,=4
1-= 16
R
1- = 400
9R
IR
Regiori
a:
Ill 1-=14585 A I i. ~ 40515 A
From oo to 5 From 6 to 5
0
z
::,
IL
n; = 6, 7, 8, 9, 10, :·· n,=5 .i.=R(
).
_l __l)
52 "i2
n;=6ton,=5
n1 =7 jo n1._=5
1-= 25
R
1- = 900
11R
FarlR
Region
"-
1- = 22790 A 1-=74583 A

Illustration 6 Illustration 7
A single electron orbits around a stationary nucleus of The wavelength of the first line of Lyman series for
charge +Ze, where Z is a ·constant aµd e is th~ hydrogen is identical to that of the second line of Balmer
magnitude of electronic 'charge. It requires 47.2 eV to series for some hydrogen lik~ ion x . Calculate energies of
0
excite the electron from secol\d Bohr orbit to the third Bohr the first four levels, of x , ·
orbit. Solution
(a) Find the value of Z Wavelength of the first line of Lymen series for
(b) Find the energy required to excite the electron fro~ hydrogen atom will be given by the equation
n=3 to n=4
(c) Find the wavelength of radiation required to remove :, =R(1l2-::{,)=3: ... (1)
electron from first Bohr's Orbit to infinity.
(d) Find the kinetic energy, potential energy and angular The wavelength of second Balmer line for hydrogen like ion
momentum of the electron in the first Bohr orbit. X is
3 2
Solution 2 ... (2)
:,=Rz (;,-;, )= :
(a) Given i'.E23 = 47.2 eV
Given that A1 = A2
Since, i'.E=l3.6Z 2 (J,--J,-) eV
n; n,
,,,1 ,,,1
~
· 47.2=13.6Z '( 1, - 1,
2 3
J ~
3R
-=--
3Rz2
4 16
~ Z=5
~ z=2
(b) To find i<.E,., n1 =3, n,=4.
Hellce the ion x is actually He+.
i'.E=i3.6Z'( 1,-J,-)
n; n,
eV The erte:r:gies of first four levels. of ion x are,
E, =-(13.6)z2 =-54.4 eV
~ i'.E=l3.6x52(;, - ;, ]=16.53 eV ~ E1
..., = ( ), = -13.6 eV
2
(c) Ionization energy is the energy required to excite the
electron from n = 1 to fl = co E,.= E), 2 =-6.04 eV and
(3
2
Thus, i'.E=;3,6x5 (;, - ~ 2]=340 eV E1
E, = ( )2 =-3.4 eV
4
The respective wavelen~ is
,, =~ = 12400 = 12400 =36 _47 A Illustration 8
i'.E i'.E 340
A doubly ionised lithium atom is hydrogen-like with
(d) K=-E=+340 eV atomic number 3.
U=2E=-680 eV (a) Find the wavelength of the radiation required to
excite the electron in Lf+ from the first to the third
L=~= 6·63 xl0""' l.056xl0""' Js
21t 21t Bohr orbit. (Ionisation energy of the hydrogen atom
equals 13.6 eV ).

= 4.8

www.puucho.com
Rahul Sardana with www.puucho.com

Atomic Physics
(b) How many spectral lines are observed in the emission
spectrum of the above excited system? · => ~;, 10.2
Solution => K;,20.4 eV
Given, Z=3 Therefore, the minimum kinetic energy of moving hydrogen
. Z' is 20.4 eV.
Smee, En ex: 2
n
(a) To excite the atom from n = 1 to n = 3 , energy of Illustration 10
photon required is , A moving H-atom makes a head on perfectly inelastic
(-13.6)(3)2 (-13.6)(3)'] collision with a stationary Li++ ion. Before collision H-
£,43 =E,-E, ( atom and Li++ ion are both in their first excited states.
(3)' (1)'
What is the velocity of the moving H atom if after
=> E,43 = 108.8 eV
collision H is found in its ground state and Lr-+ ion in its
Corresponding wavelength will be, second excited state. Take mass of hydrogen atom,
· ,. (in Al 12375 12375 ns-_74 A '"u =1.66 x 10-27 kg and mass of Li++ to be '"zr =7,nu .
' E(in eV) 108.8
Solution
(b) From nth orbit total number of emission lines can be For u++, we have Z=3
n(n-l) . So, mnnb er of erruss10n
· · 1·mes
· 1s
· -13.6(3)
2

2 => E, = -30.6 eV
(2)'
-13.6(3)'
=> E, -13.6 eV
(3)'
Illustration 9 Energy required for Li++, ion to go from first excited state
A moving hydrogen atom- makes a head on collision (n = 2) to.second excited state (n = 3) is
with a stationary hydrogen atom. Before collision both bE=-13.6-(-30.6)=17 eV
atoms are in ground state and after collision they move
together. What is the minimum value of the kinetic en~rgy Energy released by hydrogen atom to go from first excited
of the moving hydrogen atom, such that one of the atoms state to ground state is
reaches one of the excitation state. M.' =-3.4-(-13.6)=10.2 eV
Solution. So, bE-bE'=17-10.2=6.8eV is the energy that should
Let K be the kinetic energy of the moving hydrogen come from loss in KE in collision.
atom and K' , the kinetic energy of combined mass after From Law of Conservation of Linear Momentum, velocity of
collision. combined mass is

(m,=7m,)
--~-----n=2
K'
=> AKE=-m 1 u, - -
1(m, +m ) v ,
o----!S--. 0 ~
=!>
dE = 10.2 eV
2 1 1 2 2
m m 2m
Since, Af< = 6.8 eV
--..;;-----n=1
Solving these equations, we get
By Law of Conservation of Linear Momentum, we have u, =3.9x10 4 ms-1
p=p'
=> ..f2Km = ,J2K'(2m) Illustration 11
=> K=2K' ... (1) Find the largest and shortest wavelengths in the
From Conservation of Energy, K = K' + bE ... (2) Lyman series for hydrogen. In what region of the
electromagnetic spectrum does each series lie?
Solving equations (1) and (2), we get
Solution
bE=K The transition equation for Lyman series is given by,
2
Now minimum value of bE for hydrogen atom is 10.2 eV, so ¾=R(;,-:,), n=2,3, .....
we have
bE2'.10.2 eV The largest wavelength is corresponding to n = 2 , so

4.9 =
www.puucho.com
Rahul Sardana with www.puucho.com

Advanced JEE Physics Optics & Modern Physics

_l_ = 1.097 X 107


,._ 1
(.!.-.!.)4 Illustration 12
The nucleus of a deuterium has a mass of
3.34 x 10-27 kg as compared to 1.67 x 10-" kg for the
=> ,._
_l_ = 0.823 x10' hydrogen. Calculate the wavelength difference between
the first Balmer line emitted by hydrogen and the first
=> ,__ = 1.2154 X 10-7 ill Bw,;er line emitted by deuterium. Given that the mass of
,._ =1215 A electron is m, = 9.109 x 10-31 kg .
=>
The-shortest wavelength corresponds to n ~ oo, so ' Solution
The first Balmer line corresponds to the transition from
7
1
-=1.097xl0 (1
. - -1)
- n = 3 to n = 2 . In case of hydrogen atom, we have
Amin 1 oo
t.E=E,-E,
=> '-mm =0.911xl0;'. m=911 A
Both of these wavelengths lie in ultraviolet (UV) region of => t.E={-
1
!;6-(-~~·6 )}eV=l.89eV
electromagnetic spectrum. '
12375 .
So, wavelength 1,. = ( )
ABOUT RYDBERG CONSTANT . t.E ineV
Rydberg Constant is given by => ,. ;,, 12375 = 6547.6 A
me' 1.89
R=-- ---~----n=3
8e~ch3
--~~----·n=2
where m is mass of electron.
It app"ears that Rydberg constant is same for all elements, but
it is not so. Actually Rydberg constant is different for
different elements. The reasC?n is that in Bohr's ~eory
nucleus is assu_med infinitely heavy as compared to electron. --------n=1
But if the mass of nucleus is taken-into account then electron
For ordinary hydrogen reduced mass of.proton and electron
mass m must be replaced by reduced mass (µ), where is,
mM · (1.67x10°")(9.109x10"'31 )
µ= m+M µ, (1.67 X 10-2' + 9.109 X lo'"')
where M is mass of nucleus. => µ 1 =9.10408xl0-31 kg
mM m
=> µ=--=-- For deuteriulll atom reduced mass of nucleus and electron is,·
m+M l+."!.
M· (3.34x 10-27 )(9.109 X 10·31 )
µ, = (3.34 X 10-2' + 9.109 X 10""1 )
If M-->oo(infinity), µ---->m(Mass of electron) ·
=> µ, =9.10654~10""1 kg
Therefore
µe' me' All energies are proportional to µ , whereas the wavelengths
R are inversely proportional to µ . The wavelength of photon
8s~ch3
emitted in case of hydrogen_ is given by
H for a heavy nucleus, Rydberg Constant R is represented (6547.6)(9.109x10""1 ) . A
6551
by '-, (9.10408xl0""')
me' Similarly in case of deuterium, wavelength of photon
R =-.-
8E~Ch3
<O
emitted is,
Then Rydberg constant for an element is given by ,. _ (6547.6)(9.109xlo-") A
2 6549
- (9.10654x10-31)
R=__&_
1 +."!. => ,'.1,.=A,-A,=2A
M
Oearly this depends on mass of nucleus M and so Rydberg
.Illustration 13
constant is different for different -elements. Greater is M,
larger is the value of Rydberg constant R. Thus, Rydberg Calculate the separation between the particles of a
constant increases with increase of mass of nucleus. system in the ground state, the corresponding binding
energy and wavelength of first line in Lyman series if such

= i::::====================================
4.10

www.puucho.com
Rahul Sardana with www.puucho.com

Atomic Physics
a system is positronium consisting of an electron and Ritz made this discovery empirically (1908) long before Bohr
positron revolving round their common-centre of mass. proposed his theory and is known as Ritz combination
Solution principle. •
(m)(m) m Generalising, we may write, labelling the photon frequency
Reducedmass µ=-(--)=- by appropriate integers, as follows:
m+m 2
hv6111 =Es -E111
.
Smce, 1
rcx::-
·m
For, H atom, we have => hvsm =hvsn +hvnm (m<n<s) · ... (3)
,1 =0.53 A Since all combinations predicted by (3) are not actually
=> r=(2)(0.53)=1.06 A observed, tli.ere has been an hnposition of some rules, the so-
called selection rules, to eliminate certain combinations.
Since, E cc m {binding energy)
Bohr's theory provides, as discussed above, a proper
For, H atom, we hav:e explanation of the combination principle.
E1 =13.6 eV

E=
13 6
· =6.8 eV --~--~ -a~-~::t:o~~~~c!i~~Aa~o:
0;uE~:~ to a-:v:::~:~~-,
2
principal quantum number n , then the maximum number
For, H atom, we have of photons (emitted) will be (n -1) .
E,~i =10.2 eV {First line of Lymen series) b) If in a hydrogen like sample the electrons of many atoms
jump to a level having principal quant1;1m number n, then
10 2
E,, = · = 5.1 eV the maximum number of photons emitted =nC2 = n(n - ) .
1
2 2
c) For example if many atoms are in third excited .state
(n = 4), then the number of ·photons emitted for spectral
4 3
linesseen = ; =6
RITZ COMBINATION PRINCIPLE
~) If an atom goes to excited state by absorbing certain
If an electron is initially in an excited state with say n = 3 , energy, then it may emit a number of photons in
then it may transit downward from n =3 level to n =1 level succession; the sum of the energies of all emitted photons
directly. Alternatively, it may first transit from n = 3-> n = 2 will be equal to the amount of energy absorbed. For
and subsequently from n = 2-> n =1 . In the first case if v 31 example if an atom absorbing energy E reaches to an
excited level and it returns to ground state by emitted
be the frequency of the photon emitted wavelength A1,A2 and A3 , then
... (1)
E=hc+hc+hc
-~--~----n=3 ).,1 "-2 ll.3
v~ e) If an energetic electron strikes an electron (target electron)
_ _,___ _...,__ _~-n=2 of an atom in ground state then electron jl.lmps to an
excited state by absorbing energy equal to the difference
of ground state energy and excited state energy and the
remaining energy is still carried by the incident (or striking)
v,. electron.
f) The de-Broglie quantum condition: According to de-
Broglie, only those orbits are allowed as stationary orbits in
_ i _ __ _..___ _ _ _ n=1
which circumference of orbit is equal to the Integral
In the second case, two different spectral lines (photons) of multiple of de-Broglie wavelength.
frequency v32 and v21 respectively would be emitted given i.e. 2xr =ni..

by => 2xr=n(!v)
hv 32 =E3-E2 and hv21 =E2 _,....£1 ... (2)
(1) can be rewritten as => mvr =n( : )
2
hv31 = (E, -E2 )+(E,-E,) This is same as Bohr's quantum condition.
g) "An atom possesses discrete levels" was verified by
__
Franck Hertz Exp en_·m_e_n_t._ _ _ _ _ _ __
=> Y31=Y32+Y21

4.11 =
www.puucho.com
Rahul Sardana with www.puucho.com

Advanced JEE Physics Optics & Modern Physics

MERIT AND DEMERITS OF BOHR'S THEORY CRITICAL POTENTIAL


The merits of Bohr's theory can hardly be overestimated. It The resonance potential, the excitation potentials and the
saved physics at a time when it was in the grip of severe ionisation potentials are all included in the wider term
crisis. But, like every physical theory, this theory also has critical potentials.
drawbacks. We enumerate below some of its merits and
demerits. RESONANCE POTENTIAL
A minimum potential V is required to accelerate the
MERITS bombarding electron to an energy V (in electron-volt) in
a) The determination of the ratio of the mass of an electron order that an atom may be excited from its ground state to
to that of a proton in terms of the reduced mass concept the next higher state. This potential is called the resonance
of Bohr's theory agrees excellently with the value potential. ·
obtained by other methods.
b) The general principle used by Bohr has also been EXCITATION POTENTIAL
successfully applied lo a great number of phenomena
The various values of the potential required to impart the
such as the excitation and ionisation of atoms, X-ray
necessary energy to excite an atom to different higher states
spectra etc.
are known as excitation potentials.
c) The validity of the theory is further confirmed by the
fact that the theory predicts new undiscovered series IONISATION POTENTIAL
lines (spectral) which have later been actually observed.
d) The theory has been instrumental to the discovery of The minimum potential necessary to supply the required
energy to ionise an atom is called the ionisation potential or
· heavy hydrogen (deuterium) by H.C. Urey.
the first ionisation potential.
e) It gives a convincing explanation and a very simple and

f)
elegant picture of the origin of spectral lines.
The agreement between the empirically determined
' PROBLEM SOLVING TRICK(S) I
: Let us illustrate the above definitions by taking the case of :
value of the Rydberg constant and that evaluated by ! hydrogen atom. For H atom we have : 1
Bohr in terms of fundaqiental constants offers an 18
1
excellent proof of the truth of Bohr's theory. E = me4.. 2.17x10- J=-13.6 8 V
8c~n2h2 n2
11
n2

DEMERITS So the energy of the 1st, 2nd, 3rd,. ......, oo-th orbits are
respectively -13.6 eV, --3.4.eV, -1.15 eV ,.... , o eV.
a) There is an ad hoc nature in the assumptions of Bohr in
that the quantum idea of the stationary orbits is mixed Hence,
up with the classical idea of coulomb force. ; Resonance potential = --3.4-(-13.6) = 10.2 eV
b) The assumption of only circular orbits is utterly '. First excitation potential =resonance potential = 10.2 eV
unjustified. In fact, Bohr in his original paper suggested Second excitation potential =-1.51-(-13.6) =12.09 eV
that the orbit might be an ellipse instead of a circle. I Ionisation potential =0-(-13.6)=13:6 eV
c) The spectral series, though agree excellently in case of
hydrogen, are at variance with the theory for multi-
electron atomic systems, e.g. the helium, singly ionised Illustration 14
lithium etc. In these cases, it becomes necessary to Find the quantum n.umber n corresponding to excited
introduce a magnetic quanhlm number. state of He+ ion if on transition to the ground state, the ion
d) It cannot suggest any explanation whatsoever for the emits two photons in successi9n with "'?avelengths
origin of the fine structure of the spectral lines. 108.5 nm and 30.4 nm . The ionization energy of H atom
e) Bohr's theory is also unable to account for the multiple is 13.6 eV.
structure of spectral lines. For example, the doublet of Solution
sodium, triplets of magnesium etc. cannot be explained The energy transitions for the given wavelengths are
from Bohr's theory.
f) It cannot make any calculation about the transitions or t.E = 12400 = 12400 =ll.43 eV
I A1 1085
the selection rules which apply to them.. ·
g) It could not explain the splitting up of spectral lines M,, = 12400 = 12400 = 40 _79 eV
when an atom is subjected to electric field i.., 304
(phenomenon called Stark Effect) or magnetic field Total energy emitted t.E = t.E, + t.E2 = 52.22 eV
(phenomenon called Zeeman Effect).
=> .l.E=13.6Z'(..!__..!_) eV
Hi ni
,--. 4.12 C-================================
www.puucho.com
Rahul Sardana with www.puucho.com

Atomic Physics
where, AE is the energy emitted (take the mass of the nucleus to be infinite). Assuming that
the Bohr model of the atom is applicable to this system:
~ 52.34=13.6x22 ( 11, - :, ) (i) Calculate the radius of nth Bohr orbit
Thus, n=5 (ii) Find the value of 11, for which the radius of orbit is
approximately the same as that of first Bohr orbit for
the hydrogen atom;
Illustration 15 (iii) Find the wavelength of radiation emitted when the
A hydrogen atom in a state of binding energy 0.85 eV µ -meson jumps from the third orbit to first orbit
makes a transition to a state of excitation energy of (Rydberg's constant =1.097x107 m-' ).
10.2 eV. Find the energy and wavelength of photon
Solution
emitted.
(i) Radius of the nth Bohr orbit for hydrogen atom is
Solution
n'
Since the binding energy is always negative, therefore, r, =0.53
E, =-0.85 eV
2
. 1
Let n, be the initial binding state of the electron, then Smce, roc-
m
2' Radius of nth orbit for µ -meson is
E, =-13.6-
n,2 0.53n 2
r=---
2' ' (208)2
~ --0.85 = -13.6,
n, ~ r, =(8.Sx10 ... )n 2
(ii) (8.SxlO ... )n' =0.53
13.62 2 ~ n2 =623
Binding energy is En
n' :::::) n~25
2
-13.6(1) (iii) In case of hydrogen like atom,
~ 0.85 eV
n'2
LI.E=E,-E, =13.62'(1-;, )=12.08 eV
=> n2 =4
Let the electron now goes to an energy level n whose Since, .1E oc m
excitation energy is 10.2 eV . Since the excitation energy .6.E So for µ -meson, we have
is defined with respect to ground state, therefore LI.E = (12.8)(208) = 22.6 keV

E=13.62'(J..-J..) eV ,. 12400 12400 •


~ 0.548 A
n1 n2 LI.E 22•6 X 103

~ 10.2=13.6x1'( ~ -J..)
1 n1 Illustration 17

~ n1 =2 A hydrogen like atom of atomic number z 1s m an


excited state of quantum number 2n . It can emit a
So the electron makes a transition from energy level ni = 4 to maximum energy photon of 204 eV . If it makes a
n1 =2 transition to quantum state n, a photon of energy 40.8 eV
Thus, the energy released is .6.E = £4 - E2 is emitted. Find 11, z and the ground state energy (in eV)
for this atom. Also, calculate the minimum energy (in eV)
~ LI.E = 13.6[;, - ;, ]= 2.55 eV that can be emitted by this atom during de-excitation.
Ground state energy of hydrogen atom is 13.6 eV.
12400 Solution
Since, 1. = .!!E._ = = 5511 A
LI.E 2.25 eV Given, E211 - E1 = 204 eV

Illustration 16 ~ 2
(13.6)2 (1- 4:' )=204 ... (1)
A particle of charge equal to that of an electron, -e Also, E211 - E11 = 40.8 e V
and mass 208 times the mass of electron (called a µ -meson)
moves in a circular orbit around a nucleus of charge +3e ~ 13.6z'(J..--.!,)=40.8
n 4n
... (2)

4.13 =
www.puucho.com
Rahul Sardana with www.puucho.com

Advanced.JEE Physics Optics & Modern Physics


2 2
Solving equations (1) and (2), we get n hE
r=---"2
n=2 and z=4 zmnp e
. E
Smce, n = (-13.6)z'
2
eV ,sowe h ave Substituting m, = 100 m
n
where m = mass of electron and z = 4
2
E1 =(-13.6)(4) eV n2/t2E
0
wegetr=
~ E, =-217.6 eV ' " 400nme2
During de-excitation, minimum enei-gy emitted is, (b) As we \<now, E~ = -13.60 eV
Emm = E2n - E2n-t = E4 - E3

E _ = -217.6 _(-217.6)=l0 58 eV
and E11 oc (-:: )m
mm 42 32 · For the given particle,

E = (-l3 -60)( 4)' xl00=-1360 eV


Illustration 18 4 (4)'
Calculate the energy of a H ~atom in the first excited
state, if the potential energy is assumed to be zero in the and E2 = (.:_13 ·60~( 4 )' xl00=-5440 eV
ground state. (2)

Solution t.E=E 4 -E2 =4080 eV


Since, we know that, in ground state, n =1, we have ,.(in A)= 12375 12375 = 3 A
t.)o(in eV) 4080
TE=-KE=PE
2
~ PE=2(TE)=2(-13.6 eV) Illustration 20
~ PE=-27.2eV Electrons in hydrogen-like atom (Z = 3) make
However, we have assumed this energy to be zero i.e., transitions from the fifth to the fourth orbit and from the
potential energy is increased by 27.2 eV . Since, kinetic fourth to the third orbit. The resulting radiations are
energy in all energy states will remain unchanged whereas incident normally on a metal plate and eject
potential energy and hence, the total energy in all states will photoelectrons. The stopping potential for the
increase by 27.2 eV . Further, first excited state means n = 2, photoelectrons ejected by the shorter wavelength is
3.95 V. Calculate the work function of the metal, and the
so
stopping potential for the photoelectrons ejected by the
E2 = -3.4 eV (previously)
longer wavelength (Rydberg constant = 1.094x107 m·1 ) ·
~ E; =-3.4+27.2=23.8 eV (now) Solution
The stopping potential for shorter wavelength is 3.95 V
Illustration 19 i.e., maximum kinetic energy of photoelectrons
An imaginary particle has a charge equal to that of an corresponding to shorter wavelength will be 3.95 eV.
electrori and mass 100 times the mass of the electron. It Further energy of incident photons corresponding to shorter
moves in a circular orbit around a nucleus of charge +4e • wavelength will be in transition from n = 4 to n = 3 .
Take the mass of the nucleus to be infinite. Assuming that 2 2
the Bohr's model is applicable to this system. E =E -E = -(13.6)(3) (-(13.6)(3) )
,~, 4 3 (4)' (3)'
(a) Derive an expression for the radius of nth. Bohr orbit.
(b) Find the wavelength of the radiation emitted when ~ E,~, =5.95 eV
the particle jumps from fourth orbit to the second Now, from the equation,
orbit. K_=E-W
Solution
we have W = E - Kmax = E4 -+ 3 - K=
m v'
(a) Wehave ' ... (1) ~ W=(5.95-3.95) eV=2eV
r.
Longer wavelength will correspond to transition from n = 5
The quantization of angular momentum gives, to n = 4 . From the relation, we get
nh
mPvr,1 = -
21t
... (2) ~=
A
Rz'(-1N__ __!_)
12 N;z
Solving equations (1) and (2), we get
The longer wavelength,

= ===========================================
4.14

www.puucho.com
Rahul Sardana with www.puucho.com

Atomic Physics

]:_ = (1.094 X 10 7 )(3)3


1.. 16
(_!_ __!_)
25
Coolidge modified the Roentgen tube. A modem X-ray
tube consists of
=> 1..=4.514x10-' m=4514.A a) An electron source, preferably a filament heated by the
passage of an electric current which may be varied.
Energy corresponding to this wavelength is
b). A heavy target of high melting point inclined at 45° to
E = 12375 e~ A 2.74 eV the path of electron beam, kept cooled by circulating
4514A cold water internally.
So, maximum kinetic energy of photo-electrons is c) A source of high potential difference applied across the
K==E-W=(2.74-2) eV=0.74eV filament and the target, keeping target positive with
respect to filament. When the filament is heated, a fine
Hence, the stopping potential is 0.74 V. beam of electrons strikes the target to produce X-rays.

Illustration 21 CONTROL OF INTENSITY


Hydrogen atom in its ground state is excited by means The intensity-of incident electrons determines the intensity of
of monochromatic radiation of wavelength 975 A . How X-rays , i.e., greater is the number of electrons striking the
many different lines are possible in the resulting target, more intense are the X-rays produced.
spectrum? Calculate the longest wavelength amongst them.
You may assume the ionization energy for hydrogen atom CONTROL OF PENETRATING POWER
as 13.6 eV.
The potential difference across the filament and target
Solution
determines the energy and hence the penetrating power of
Energy corresponding to given wavelength is X-rays.
E(in eV) 12375 12375 = 12.69 eV
1..(inA) 975 NEED OF COOLING DEVICE
Now, let the electron excites to nth energy state. Then, Only about 1 % of incident electron's energy is converted into
E, - E1 = 12.69 X-rays and the remaining 99% is converted into heat,
therefore cooling device is essential with an X-ray tube.
=> ((~·t-(-13.6)=12.69
HARD AND SOFT X-RAYS
=> ns,4
i.e., electron excites to 4th energy state X-rays upto 4 A have high penetrating power and are
Total number of lines in emission spectrum would be called hard X-rays while those of 1,. > 4 A are called soft
N n(n-1) 4x3 X-rays.
6
22
Longest wavelength will correspond to the minimum energy X-RAY SPECTRA CLASSIFICATION
and minimum energy is released in transition from· n = 4 to In X-ray tube, when high speed electrons strikes the target,
n=3. they penetrate the target. They loses their kinetic energy and
-13.6 (-13.6) comes to rest inside the metal. The electron before finally
E,~ 3 =E4 -E3 = ( ') - ( ), =0.66eV being stopped makes several collisions with the atoms in the
4 3
target. At each collision one of the following two types of
Hence, longest wavelength will be, X-rays may get formed.
1,. = 12375 12375 A = 1.875 x 10 _. m A. Continuous X-rays
-E(in eV) 0.66 B. Characteristic X-rays
=> 1,._ = 1.875 µm
A. CONTINUOUS SPECTRUM
X-RAYS It consists of radiations of all possible wavelengths within a
definite wavelength range having a definite short
PRODUCTION wavelength limit. These are produced due to deceleration of
electrons near heavy nucleus. These decelerated electrons
When fast moving electrons strike a target of high melting
emit radiations which lie in X-ray region. As the electrons
point and high atomic weight (like tungsten, platinum
molybdenum), X-rays are_ produced. X-rays were suffer collisions at all angles, right from the glancing
discOvered by Roentgen, therefore, they are also known as collision to the direct hit, they suffer varying decelerations
Roentgen rays. Roentgen discovered X-rays in 1895.

==================================== = 4.15

www.puucho.com
Rahul Sardana with www.puucho.com

Advanced JEE Physics Optics & Modem Physics

and hence radiations of all possible wavelengths within a Solution


certain range are emitted, forming the continuous spectrum. de Broglie wavelength (i.,,) when a charge q is
Intensity
accelerated by a potential difference of V volt is given by
Characteristic X-Aays h

I, \ \ For cut-off wavelength of X-rays, we have


... (1)

he
Continuous qV=-
X-Rays ··-.__ i.,,.
wavelength l. i., =.!!E._ ... (2)
m qV
The maximum limiting frequency vmax or minimwn limiting
wavelength "-mm is obtained when entire kinetic energy of From equations (1) and (2), we get
bombarding electron is converted to X-ray energy. If V0 is
the accelerating potential, then ~= Jtf·
eV0 =hvmax

=,, V =-
ev, For electron, we have !L =1.Bx10 11 Ckg-1
m
- h
11
1.8 X 10 X lQ X 103
In terms of """"
2 0.1
he =,,
eV. =-- 3x10 8
o "-m1n
i., . =.!!E_ B, CHARACTERISTIC SPECTRUM
= eVo
The minimum wavelength depends on the electron energy,
This relation gives the short wavelength limit of continuous but not on the target material. The line spectrum depends on
X-ray spectrum. the element used as target. These characteristic X-rays are
If f is the fraction of kinetic energy of electrons converted produced when an electron knocks out an atomic electron
from one of the inner levels. The ejected electron leaves a
into X-ray, then wavelength of emitted X-ray photon is vacancy, which is then filled by an electron falling from a
given by higher level. When an electron jumps from a higher energy
he orbit Ei to lower energy orbit E2 , it radiates energy
f(eV0 )=-;:-
( E, - E,) . Thus this energy difference is radiated in the form
=,, A=~ of X-rays of very small but definite wavelength which
feV, depends upon the target material. The X-ray spectrum
consists of sharp lines and is called characteristic X-ray
spectrum.
Illustration 22
If the transitions are to the n =1 level, the X-rays are
Find the cut-off wavelength of the X-rays emitted by
labelled K. , K, .. ., If they are to the n =2 level, they are
an X-ray tube operating at 30 kV ,
labelled Ln, L/J , .... etc. In the figure shown, the energy level
Solution
diagram for an atom is drawn. The arrows indicate the
For minimum wavelength, the total kinetic energy
transitions that give rise to the different series of X-rays.
should be converted into an X-ray photon,
=0_41 A
-----------N
Thus ,_ =he =12400 = 12400
' E E 30x103 ---.---+-1-1r-+-i -M

Illustration 23
If au X-ray tube operates at the voltage of 10 kV, find
the ratio of the de-Broglie wavelength of the incident
electrons to the shortest wavelength of X-rays produced.
=ii=L·=~:
K. K, K,
In 1913; Moseley noted that the characteristic lines shifted
The specific charge of electron is 1.8 x 1011 Ckg _, . systematically as the target material was changed. He
plotted the square root of the frequency of the K. line

4.16 :::::============:::;:=====::;::::::==:::::===================
www.puucho.com
Rahul Sardana with www.puucho.com

Atomic Physics

versus the atomic nwnber Z for many elements. The where a and b are constants for a given transition. The
straight line he obtained is shown in the figure. constants a and b do not depend on the nature of target.
Different values of b are as follows
b =1 for Ku - radiations of all substances.
b = 7 · 4 for L. - radiations of all substances.
b =19.2 for Mo: - radiations of all substances.
If the bombarding electron knocks of an atomic electron from
0 Atomic number Z
K shell and if the vacancy is filled in from L, M, N, ....... .
Moseley's plot did not pass through the origin. Let us see, shells the characteristics X-rays form K -series.
why?. Once one of the two electrons in the
For Ka-line, EL -EK =hvK..
n = 1 level is ejected, an electron in the next highest level
will drop to the lower state to fill the vacancy and in the For K11 -line, EM -EK =llvKp
process it emits the Ku frequency. For this electron the
Both these transitions are highly probable. Similarly, if
electric field due to the nucleus is screened by the remaining bombarding electron knocks off atomic electron from
electron in the n = 1 level. Moseley estimated that the L-shell and if the vacancy is filled in from M, N, ..... shells,
effective nuclear charge for the K. transition is (Z-l)e. the characteristics X -rays form L -series and so on.
Thus Moseley's Law for the frequency of the Ku line is
,[v;;: =a(Z-1) MOSELEY'S LAW: CONCLUSIONS REVISITED
a) Mosley's Law supported Bohr's theory.
where a= J¾ Re in which R is the Rydberg constant c is b) It experimentally determined the atomic number (Z) of
elements.
the speed of light.
c) Titis law established the importance of ordering of
The wavelength of K - lines is given by elements in periodic table by atomic number and not by
2 1 atomic weight.
f=(Z-1) [1- n ,] where n=2,3,4, .....
d) Gaps in Moseley's data for A= 43, 61, 72, 75 suggested
existence of new elements which were later discovered.
Illustration 24 e) The atomic numbers of Cu, Ag and Pt were
Show that the frequency of K, X-ray of a material established to be 29, 47 and 78 respectively.
t) When a vacancy occurs in the K-shell, there is still one
equals to the. sum of frequencies of Ku and Lu X-rays of
electron remaining in the K-shell. An electron in the
the same material.
L-shell will feel an effective charge of (Z-l)e due to
Solution +Ze from the nucleus and -e from the remaining
K-shell electron, because L-shell orbit is well outside the
K-shell orbit.
g) Wave length of characteristic spectrum is

The energy level diagram of an atom with one electron _! = R(Z-b)'(..!__2_)


knocked out is shown above.
A tfi ni
Energy of Ku X-ray is EKu =EL-EK of KJJ X-ray is h) Energy of X-ray radiations is given by

E,:,. =EM-EK t;E = hv =he= Rhc(Z-b)'(..!._2.)


A n~ ni
and, of Lu X-ray is Eta.= EM -EL
i) If transition takes place from n2 = 2 to n1 = 1 we get
thus, EK~ =EKu +ELu or vK~ =vK.. +vL~
the Ku line, for which we have

MOSELEY'S LAW: GENERAL TREATMENT


According to Moseley, for a given transition, the frequencies
(i) a= J3~c = 2.47 x 1015 Hz

(ii) Frequency is given by


of characteristic lines emitted by different elements obey the
relation 2 3 2
v =RC(Z-1) (1-..!.)= RC (Z-1)
2
Ko: 22 4
v=a(Z-b)
2
also called Moseley's Law. => vK
0
=2.47x10 15 (Z-1) Hz

c==================================== 4.17 ,-----,

www.puucho.com
Rahul Sardana with www.puucho.com

Advanced JEE Physics Optics & Modern Physics

(iii) E•• =10.2(Z-1) eV


2 Solution
Energy associated with the electron in the K shell is
(iv) In general the wavelength of all the K-lines are
approximately
given by 2._=R(Z-1)'(1-J,.) 2
E, =-(74-1) (13.6 eV)=-72474 eV
AK ~ n
An electron in the M shell is subjected to an effective
where n=2,3,4, ....
nuclear charge that depends on the number of electrons in
- 1216 •
While for Ka line AK« =---, A the n = 1 and n = 2 states because these electrons shield the
(Z-1) M electrons from the nucleus.'
Since, there are eight electrons in the n =2 state and one
Illustration 25 remaining in the n = 1 state, hence roughly nine electrons
In Moseley's equation, we have :J1 =a(z-b), where shield M electrons from the nucleus, so we have
a and b are constants. Find their values with the help of zeH=z-9
thfll
e o owine: d ata. Hence, the energy associated with an electron in the M shell
Element z Wavelength of K. X-rays. is
2
Mo 42 -o.71A -13.6z!, V -13.6(z-9) V
27 1.7ssA 32 e 32 e
Co
Solution EM= (13.6)(74-9)' eV=-6384eV
Since, According to Moseley's Law we have 9
..fl =a(z-b) Therefore, emitted X-ray has an energy equal to
EM -EK =[--6384-(-72474)] eV = 66090 eV
where. f=- ,.
C

So, for the first element, we have Illustration 27

Jt =a(z,-b) ... (1)


X-rays are incident on a target metal atom having
30 neutrons. The ratio of atomic radius of the target atom
and :He is (14)1/'.
and for the second element, we have
(a) Find the atomic number of target atom.
{t=a(z,-b) ... (2) (b) Find the frequency of K. line emitted by this metal.
R=l.lx10 7 m-1 , c=3x10 8 ms-1
From equations (1) and (2), we get
Solution
Jc(_},- kJ=a(z1 -z2 ) ... (3)
(a) From the relation r oc A'
1

Now, we know that c = 3 x 108 ms-1 and further it is given to


us that
A1 = 0.71 X 10-10 m

i )'
1
A2 = 1.785 X 10-lO m
=> ( =(14)½
z, =42 and
=> A, =56
z, =27
Solving above three equations, we get
(b) z, = A, - numberof neutrons
a=5x107 (HzJV' and b=l.37
=> z, =56-30=26
.
Smee, f,.=Rc (Z-1 l'(l 1)
3Rc( Z-1),
21 - 22 =--4
Illustration 26
Substituting the given values of R, c and Z, we get
Determine the energy of the characteristic X-ray (Kil)
emitted from a tungsten (z = 74) target when an electron
!,a = 1.55 X 1018 Hz
drops from the M shell (11 =3) to a vacancy in the K shell
(11=1).

= 4.18 ==================================
www.puucho.com
Rahul Sardana with www.puucho.com

Atomic Physics
=
Illustration 28 where n is an integer, called order of maxima. Equation (1)
Characteristic X-rays of frequency 4.2 x 1018 Hz are is called Bragg's equation
produced when transitions from L-shell, l.= 2dsin8
K-shell take place in a certain target material. Use Mosley's n
Law to determine the atomic number of the target material. For ma;ximum wavelength
Given Rydberg constant R =1.lxl0 7 m-'.
nm,. =1 and (sine)_ =1
Solution
:::::} "-mu =2d
AE = hv = Rhc(Z-b)'(J___J__) Hence equation (1) has solution only for l. 5 2d .
ni ni
For K-series, b =1 INTENSITY OF TRANSMITTED X-RAY

=> v = Rc(Z -l)'(J___J__)


n; ni
The intensity of monochromatic X-ray beam after
penetrating a thickness x of a target material is given by
Substituting the values, we get I= I 0 e-µx, where µ is a constant called Absorption
Coefficient. Its value depends upon nature of material. µ
4.2x 1018 = (1.1 x 107 )(3x10')(Z-1)'G-¾)
increases with increase of A and atomic number Z of
absorbing material. µ is maximum for lead.
=> (Z-1)' = 1697
=> Z-1=41
=> Z=42
BASED ON ATOMIC STRUCTURE & PROPERTIES
(Solutions on page 4.99)
PROPERTIES OF X-RAYS 1. A doubly ionised lithium atom is hydrogen like with atomic
a) X-rays are electromagnetic waves of very short number 3.
(a) Find the wavelength of the radiation require to excite
wavelength of order of 1 A . Therefore, they can exhibit
the electron in Li++ from the first to the third Bohr orbit
properties of reflection, refraction, interference, (Ionisation energy of the hydrogen atom equals
diffraction, polarisation like ordinary light. Due to this 13.6 eV).
property they help in the study of crystal structure. (b) How many spectral lines are observed in the emission
b) They travel in vacuum with speed of light i.e. spectrum of the above excited system.
c =3x10 8 ms-1
2. Find the Ionization energy of a doubly ionized lithium atom.
c) They are electrically neutral, hence cannot be deflected
by electric and magnetic fields. 3. An electron and a proton are separated by a large distance
and the electron approaches the proton with a kinetic
d) They do not possess magnetic moment. energy of 2 eV . If the electron is captured by the proton to
e) They have ionising power. Therefore, when they pass form a hydrogen atom in the ground state, what wavelength
through a gas, the gas is ionised. photon would be given off?
f) They have penetrating power. They can penetrate light
4. Electrons in hydrogen like atoms (Z = 3) make transitions
substances like wood, flesh, thick paper, thin sheets of
from the fifth of the fourth orbit and from the fourth to the
metals, but cannot penetrate heavy substances like lead, third orbit. The resulting radiation are incident normally on a
calcium, barium sulphate etc. metal plate and eject photoelectrons. The stopping potential
g) When incident on certain metals, they liberate electrons. for the photoelectrons ejected by the shorter wavelength is
This effect is called photoelectric effect. 3.95 volts. Calculate the work function of the metal and the
stopping potential for the photoelectrons ejected by the
h) They cause fluorescence in many substances like longe~ wavelength.
barium, cadmium, zinc sulphide etc.
i) They have destructive effect on living tissues. Therefore 5. Use Moseley's Law with b = 1 to find the frequency of the
the persons working with X-rays· often wear lead K.
clothes. X-rays of La(Z = 57) if the frequency Ot the K" X-rays of
Cu(Z=29) is known to be 1.88x1018 Hz.
BRAGG'S LAW
6. When the voltage applied to an X-ray tube is increased
When an X-ray beam of wavelength A is incident on a TJ = 1.5 times, the short wave limit of an X-ray continuous
crystal of inter planar spacing d at grazing angle 8, then the spectrum shifts by M = 26 pm . Find the initial voltage
directions of diffraction maxima are given by applied to the tube.
2dsin8 = nl. ... (1)

4.19

www.puucho.com
Rahul Sardana with www.puucho.com

Advanced JEE Physics Optics & Modern Physics


7. Find the .cutoff wavelength for the continuous X-rays ~(d) Find wave number of the p~oton emitted for ihe
coming from an X-ray tube operating at 40 kV . transition n =3 to 11 =1.
(0) What Is the minimum energy that an electron will have
8. Certain gas of Identical hydrogen like atoms has all .its after interacting with this atom- in the .ground state if
atoms in a particular upper .energy level. The atoms make the initial kinetic energy of the electron is
transition to a higher energy level when a monochromatic (i) 6 eV (ii) 11 eV
radiation, having wavelength 1654 A, is incident upon It.
Subsequently, the atoms emit radiation of only three: 14. Using the known values for hydrogen atom, calculate
different photon energies. (a) . radius of third orbitfor Li"'
(a) Identify the atom . (b) speed of electron in fourth orbitfor He· .
(b) Obtain the ionization energy for the gas atoms•.
(c) If the atoms of the gas are to be excited to such a 15. Find an 8xpression for the magnetic dipole ·moment and
level which gives radiation ·of only six different photon magnetic field Induction at the center of a Boh(s
energies, what should be energy of incident radiation.
hypothetical hydrogen atqm in the nth orbit of the electron in
terms of universal constants. 0
9. Electrons are emitted from an electron gun at alniOst zero
velocity and are accelerated by an-electric field E through a
16. A small particle.of mass m moves In such a w_ay that the
distance of 1 m . The electrons are now_ scattered by an_
atomic hydrogeri: sample in grounds state. What shoulg be potential energy U = ar2 where ~ iS a, ·constant and r is
the minimum value of E so that red light of wavelength the distance of the particle from the origin. Assuming Boh(s
model of quantization of angular momentum and circular
6563 A may be emitted In the hydrogen?
orbits, find the.radius of nth allow~d orbit. .

1 0. A hot gas emits radiation of wavelengths 460 A, 831 A 17. (a) If two times the 1-,,,. of continuous X-ray spectra 61
and 1035 A only. Assume that the.atoms have only two target atom A at 34.3 kV, is same as the wavelength
excited states and the difference between consecutive
energy levElls decreases aS energy IS increased. Taking the of K0 line of target atom B at 40 kV , then determine
energy of the highest energy state to be zero. Find the the atomic number of the atom B . _
energies of the ground state and the first e~cited state. (b) Stopping potentials of 24, 100, 11 O and 115 kV are
measured for photoelectrons emitt~d from a certain
11. A mixture of hydrogen atoms (in 'their ground state) and ele_ment when it is radiated with mo~nochromatlc X-ray.
.hydrogen like Ions (in their first excited state) are being If ·Uils element is used as a target in an X-ray tube,·
excited by electro!'ls which have· been accelerated by same what will be the wavelength of K0 line?
potential difference V volts. After excitation when they
come directly into ground state, the wavelengths of emitted
light are fotind in the ratio f5 : 1. Calculate the minimum 18. If potential energy In first orbit is taken to be zero, then find
the kinetic energy, potential energy and total energy in first
value of V for which both the ,atoms get excitea after and second orbit of hydrogen atom.·
collision with electrons. Also find the atomic number of 0,ther
ion and the energy of emitted llghtby hydrogen atoms and 19. Wavelengths belonging to Balmer 'Series lying in the range
ions.
of 450 nm to 750 nm Were used to eject photoelectrons
from a metal surface whose work- function is 2 eV . Firid
12. A stationruy .He• emitted a photon corresponding to the
(il1 eV) the maximum kinetic energy of the emitted
first line of Lyman series. This photon liberated a photo
photoelectrons. Take h9 = 1242 eV nm.
elecJron from a stationary· hydrogen atom in the gl'ound
state. Find the velocity of the photoelectron.
20. The potential energy of a particle varies as :
130 The energy levels of a hypott,elical ·one electron atom are U(x)=[E,, o,;x,;1
shown In the figure. 0 X>1
"'-------~o ev For o,;x,;1, the de-Broglie'wavelength Is J..1 -and for
n = 5 - - - - - - - - - - 0 . 8 0 eV
x> 1 , the de-Broglie wavelength is }.2 • Total energy of the
n=4 -1.45eV ' ,

particle is 2E, . Find !:,_.


n =3 - - - - - - - - - 3 . 0 8 eV J..,

21. An electron·in a hydrogen like atom Is In an excited state. It


n=2 -----,,l.-----5.30eV has a total energy of -3.4 eV , Calculate.
(a) the kinetic energy,
(b) the de-Broglie wavelength of the electron.
10 3

n = 1 °'---- ·_.1_e_v_ _ _ -15,6 eV

(a) Find the Ionization potential of this atom.


(b) Find the short wavelength llmit of the series
terminating at n = 2 .
Cc) Find the excitation ootential for the state n = 3 .

= ==================================
4.20

www.puucho.com
Rahul Sardana with www.puucho.com

NUCLEAR STRUCTURE
The nucleus of an atom consists of two types of particles, 1 u = ~ ( mass of one atom of 6C12 ) = 1 · 66056 x 10-21 kg
1
protons and neutrons. A proton has a positive charge equal
to 1 · 6 x 10-" C and a mass equal to 1 · 6726 x 10-27 kg . A NUCLEAR RADIUS
neutron has no charge and its mass is 1-6749 x 10-21 kg. Assuming that the nuclei are spherical, their radii are well
Thus a neutron is slightly heavier than a proton. Protons or represented by the empirical formula
neutrons, being the particles present inside nucleus, together R=R,A1/'
are called Nucleons.
The total number of protons in the nucleus is called its where R, =1-lxl0-15 m=l·lfermi(fm)
atomic number (Z) . The total number of nucleons in the
nucleus is called its mass number (A) . If N is the number NUCLEAR DENSITY
of neutrons, then, The density of a nucleus of mass M and mass number A
A=Z+N can be written as
No electrons are present inside the nucleus. M Aamu (Ax1.67x10-") kg
If X is the chemical symbol for an element then its nucleus p=--
is represented as:x or as 2 XA ¾7tR' fx(R,A1/3)' ±x(l-lxl0-15 m) A
3
3

=> pce2.9x1017 kgm.,,


ISOTOPES
1bis comes out to be -10 17 kgm-3 , which is extremely large
Nuclides having the same charge number {Z) but different as compared to the density of ordinary matter which is
mass number (A) are called isotopes. All the isotopes are -10 3 kgm-,3.
chemically similar and hence they occupy the same position
in the periodic table. CONCEPTUAL NOTE(S)
Since density is independent of mass number A, so all nuclei
ISOBARS same density. So, whether two nuclei are isobars, or isotopes or
isotones. They must possess same density.
Nuclides having the same mass number (A) but different
atomic number ( Z) are called isobars. THE NUCLEAR FORCE
The force which binds the protons and neutrons inside the
ISOTONES nucleus is neither electrical nor gravitational. It is an entirely
Nuclides having the same neutron number ( A-Z) but different kind of force called the strong nuclear force. This
force is extremely complex in nature. Some of its main
different mass number (A) are called isotones. characteristics are mentioned below.
a) Nuclear forces are attractive in nature. Their
ATOMIC MASS UNIT (u OR AMU) magnitude, which depends upon inter nucleon distance
is of very high order.
Atomic and nuclear masses are generally expressed in terms
of atomic mass unit (a.m.u.).

================================== = 4.21

www.puucho.com
Rahul Sardana with www.puucho.com

Advanced JEE Physics Optics & Modem Physics

b) Nuclear forces are charge independent. Nature of force P------>N'+1t+


remains the same whether we consider force between
two protons, between two neutrons or between a
proton and a neutron. b) Due to exchange of "- meson.
c) These are short range forces. Nuclear forces operate 1t"· meson is emitted by the neutron whiclt is absorbed
between two nucleons situated in close neighbourhood by the proton.
only.
N-->P'+1t·
d) Nuclear forces decrease very quickly with distance
between two nucleoI\S. Their rate of decrease is much P+1C------>N'
more rapid than that of inverse square law forces
(Coulombic forces). The forces become negligible when DOG-BONE ANALOGY
the nucleons are more than 10-14 cn:i apart.
The above interactions can be explained with the dog bone
e) Nuclear forces are spin dependent. Nucleons having analogy according to which we consider the two interacting
parallel spin are more strongly bound to each other nucleons to be two dogs having a common bone clenched in
than those having anti-parallel spin. between their teeth very firmly. Each one of these dogs
wants to take the bone and hence they cannot be separated
YUKAWA THEORY OR MESON THEORY OF NUCLEAR easily. They seem to be bound to each other with a strong
FORCES attractive force (which is the bone) though the dogs
According to this theory, a nucleon consists of a core themselves are strong enemies. The meson plays the same
surrounded by a cloud of mesons, which may be charged or role of the·common bone in between two' nucleons.
neutral. The mesons constantly get exchanged, back and
forth, between two neighbouring nucleons. In this process RADIOACTIVITY
the two nucleons remain bound to each other. The phenomenon 0£ Spontaneous emission of radiations
(o., p, r etc.) oy certain nuclei is called radioactivity.
A. PROTON-PROTON INTERACTION
The phenomenon of radioactivity was discovered by
It is the force between two neighbouring protons. It is due to
Bacqrierel.
the exchange of rr, 0 meson between them. It is represented in'
the form of a reaction as folloWs.
LAWS OF RADIOACTIVE DISINTEGRATION
Proton P1 emits rr,0 and gets coi:verted into a proton P/,
having different co-ordinates. So, A. RUTHERFORD-SODDY LAWS

P1------>P/ +rr:o a) Radioactivity is nuclear disintegration phenomenon. It


is independent of all physical and chemical conditions.
This 1t0 is absorbed by P, which also gets converted into a b) The disintegration is random and spontaneous
new proton P2'. ·Hence, statistical process. It is a matter of chance for any atom
to disintegr_ate first.
P2 + 7to------> P2' c) The radioactive substances emit o. or P particles along
with r -rays. These rays originate from the nuclei of
B. NEUTRON-NEUTRON INTERACTION disintegrating atoms and form fresh radioactive
It is the force between two neutrons. It is also due to products with different physical and chemical
exchange o~ 1r0 between_ them propertief?.

N 1 -----+.N/ + n°
• •
The rate of decay of nuclei (- dN) is directly proportional-t~

the number of undecayed nuclei (N) in the sample at time


N2 + 1to-----+-N2'
_t.
C. PROTON-NEUTRON INTERACTION dN '
=> --ocN
It is the force between a proton and a neutron situated close dt
to each other. It can be take place in following two ways. "
=> dN =-1.N
a) Due to exchange of :r,+ meson. dt
Proton e~ts 1r+ meson and gets c'?nverted· to a where A is constant of proportionalitr called Decay_
neutron. Another neutron absorbs this 1t+ meson_ to get Constant or Disintegration Constant.
converted to a proton. So,

= ========::;:=========================
4.22

www.puucho.com
Rahul Sardana with www.puucho.com

Nuclear Physics
N
CONCEPTUAL NOTE(S)
N, So, at t =T,~
N
,. , N =--.!!...
2

N fTw • N~ '2Tt:2 • No 3Tw • N0 nT1., , N0


- 0----:---:-,, 2 ~ 22 -.....,........ ?:3 - ~ ..... - 2"

0.368 N,_ where 11= t Time Lapsed


T,, Half Life
·---·----------------
T2T3T4T5T
NdN ,
~ f-=-Af dt
N N 0

~ log,( ( ) = _:,.t ~ e
-Uiµ.
=-21
=> N=Noe-At ~ AT1/, = log, 2 = 0.693
where, N is the number of un-decayed nuclei in the sample 1: _ 0.693
~, l/2 - - , . -
at time t and N O is the number of un-decayed nuclei in
the sample at time t = 0 (initially). Table : Fraction of active/decaved atom at different time
So, we coriclude that the number of an-decayed nuclei in the : Re~1.aining fraction . Fraction•of atoms
sample decays exponentially with time. of active atoin.s N -N
Time (t) decayed -·-0-·- ..-
(N/N,) probability N,
B. DISPLACEMENT LAWS •.
, of survival probabilitv of decav
a) When a nuclide emits ex -particle, its mass number is
t=0 1 (=100%) 0
reduced by four and atomic number by two, i.e.,

b)
z
XA-----+ z-2 yA-4+ 2 He4+Energy

When a nuclide emits a p-particle, its mass number


I t=Tv, ½(=50%) _!, (=50%)
2 I
remains unchanged but- atomic number increases by t = 2T1/, .!.4 (=25%) ~ (=75%)
one, i.e., 4
zXA-----+:+1 yA +_1 13° + V + Energy
where V is the antineutrino
I 1=3T1/L, ½(=12.5%) 7 C
- (=87.5%)
8 . I
l

The P-particle is not present initially in the nucleus but


is produced due to disintegration of neutron into a
t=10(r1/,)
(1)"
2 =0.1% ~99.9%

proton. i.e., •.
0
1
n ----->1 H' +_1 p + v (antineutrino)
0 t=nT1/,
(½J . {1-(½J}
When a proton is converted into a neutron, positive
P-particle or positron is emitted.
Illustration 29
1 H 1 --->0 n1 +1 13°+v (neutrino) At a given instant there are 25% undecayed
c) When a nuclide emits a gamma photon, neither the radioactive nuclei in a sample. After 10 seconds the
atomic number nor the mass number ~ges. number of undecayed nuclei reduces to 12.5%. Calculate
(a) mean life of the nuclei
HALF LIFE (T,J (b) the time in which the number of undecayed nuclei
The half life period of a radioactive substance is defined as. will further reduce to 6.25% of the reduced number.
the time in which one-half of the radioactive substance is Solution
disintegrated. If N 0 is initial number of radioactive _atoms (a) In 10 s, number of nuclei has been reduced to half (25%
present, then in ~ half life time T1/,, the number of to 12.5%).
Therefore, its half-life is
undecayed radioactive atorru; will be N, and in next iu;lf life'
2 11/2 = 10 s
N0 Relation between half-life and mean life is
- andsoon.
4

4.23 =
www.puucho.com
Rahul Sardana with www.puucho.com

Advanced JEE Physics Optics & Modem Physics


number of atoms.
I =_i_ =~ S . T 0.693
m= log,(2) 0.693 c) S mce, 112 = - -
.· :>.
=> tmean =14.43 5
1 1
(b) From initial 100% to reduction till 6.25%, it takes four => °i:"' ' = 0.693 (T,,,) = l .44 (T,,,)
half lives. i.e. mean life is about 44% more than that of half life.
100%~50%~25%~12.5%~6.25% Which gives us 't > T,t/2l
So, we conclude that
=> 1=41v2 =4(10)s=40s
Tv2 . = 0.693't
=> 1=40 s
:::::,. Tv2 <-r

MEAN LIFE(<)
Illustration 30
The mean life or average life of a radioactive substance is
equal to the average time for which the nuclei of atoms of the The mean lives of a radio·active substances are 1620
radioactive substance exist. years and 405 years for a emission and p emission
The average life of a sample can be calculated by finding the respectively. Find out the time during which three-fourth
total life of all the nuclei of the substance and then dividing of a sample will decay if it is decaying both by a emission
it by the total number of nuclei present in the sample and P emission simultaneously.
initially. Mathematically Solution
Let at some instant of time t, number of atoms of the
radioactive substance are N . It may decay either by a
emission or by f3 emission. So, we can write,

=> 't=-
,.1 If the effective decay constant is A , then
1'.N =1'..N +1'.,N
CONCEPTUAL NOTE(S) 1 1
a) From N = N0 e-11 , we get => 1'.=1'.. +1'., = 1620 + 405
1
log,(~)= -:>.t => 1'. = - - year-1
324
So, the slope of the line shown in the graph i.e. the

magnitude 91 inverse of slop~ of log,(~) vs t curve is


=> No =Noe-u
kn~wn as mean life ('t).
4

=> -1'.t = log,(¼) =-1.386

=> ( _!_}=1.386
324
=> 1=449 year

Illustration 31
b) From N=N0e-u,if
.
t=..!.=t
:>. A radioactive nucleus can decay by two different
processes, the half life for the first process is t 1 and that
=> N = N0 e-1 = N0 ( ¾) =0.3iN 0 = 37% of N0 • for the second process is t 2 • Show that the effective half-
i.e. mean -life is the time inteival in which number of
life t of the nucleus is given by ~ = .!. + .!. .
undecayed atoms (N) becomes .!a times or 0.37 times or f tl t2

37% of original number of atoms. or Solution


It is the time in which number of decayed atoms (N0 - N) Let at any instant t, number of nuclei in radioactive
sample be N . Then it may decay by either of two different
becomes ( 1-¾) times or 0.63 times or 63% of original
pro~esses. So,
= 4.24

www.puucho.com
Rahul Sardana with www.puucho.com

Nuclear Physics

-d: =-(d:), -( d:), dN d


A=--=--(N0e-').t)=AN0 e-A1 =Aae->-1 where
dt dt
=> '/,.N = '/,. 1N + '/,. 2N A, = ('/,.N0 ) is the activity at time t = 0 .
=> '/,.='/,. 1 +'/,. 2 So, activity of a radioactive sample decreases exponentially
with time.
1 1 1 "/ -t-0.693)
~ -=-+- ( . v,- --,.-
t t, t, UNITS OF RADIOACTIVITY

Illustration 32 A. BACQUEREL
Prove mathematically that mean life or average life of In S.I. system the unit of radioactivity is bacquerel.
a radioactive substance is t,m =..!. . 1 bacquerel = 1 disintegration/sec= 1 dps.
'/,.

Solution B. RUTHERFORD
Let N be the number of atoms that exist at time t . It is defined as a unit of activity equal to 10 6 dps.
Between t and t+ di let dN atoms are decayed, then
Mean or average life is C. CURIE
0
f tdN The traditional unit of activity is curie. It is defined as
3, 7 x 1010 dps which is also equal to the radioactivity of 1 g
tw =.!:!!.__
0
of pure Radium.
N,
f dN
Illustration 33
.
Smce, dN
--='/,.
N
dt The half-life of Cobalt 60 is 5.25 years. How long after
its activity have decreased to about one-eight of its original
=> dN=-'/,.Ndt value?
-f t'/,.Ndt
0
Solution
The activity is proportional to the number of undecayed
-N, atoms.
Since, N =N 0 e-u, so In each half-life, half the remaining sample decays.

Since, (½)(½)(½)=½, therefore, three half-Jives or

N, 15.75 yr are required for the sample to decay to !th its


8
This integration is done by parts to get the result original strength.

Illustration 34
A count rate-meter is used to measure the activity of a
ACTIVITY OF RADIOACTIVE SUBSTANCE (A) given sample. At one instant the meter shows 4750 counts
The activity of a radioactive substance means the rate of per minute. Five minutes later it shows 2700 counts per
decay (or the number of disintegration/sec). This is denoted minute.
by (a) Find the decay constant
A (b) Also, find the half life of the sample
Solution
A, ='/,.N,
Initial velocity A;= dNI ='/,.N0 = 4750 ... (1)
dt 1=0

Final velocity A 1 = dNI = '/,.N = 2700 ... (2)


dt 1=5
Dividing (1) by (2), we get
0

==================================== = 4.25

www.puucho.com
Rahul Sardana with www.puucho.com

Advanced JEE Physics Optics & Modern Physics

4750= N0 743x101•,-0) 2 xl0' (1-e-(';:°:'""})


2700 N, . ( 0.693 )
The decay constant is given by 14.3x3600
Solving this equation we get t = 14.3 hr.
A = 2.~03 log, ( ~:)
Illustration 37
=> A= 2.303 10 (4750) = 0_113 min-1 There is a stream of neutrons with a kinetic energy of
5 g, 2700 ·
0.0327 eV. If the half life of neutrons be 700 s, what
Half life of the sample is fraction of neutrons will decay before they travel a distance
T = 0.693 = 0.693 = 6_14 min. of 10 m? Mass of neutron equal.to 1.675 x10·" kg.
A 0.113
Solution
. K 1 ,
Illustration 35 Smce, =-mv
2
The half-life of radium is 1620 years. How many 19
radi~ atoms decay in 1 s in a 1 g sample of radium. The v= {2k = 2x0.0327x1.6x10· = 2_5 xlO' ms·'
atomic weight of radium is 226 gmoi- 1

=>
v-;;; 1.675 X 10'27
s 10 4x10-' s
Solution =>
V 2.5xl03
Number of atoms in 1 g sample is
N N,{1-,-") _,,
26 21 Fraction decayed is 1 -e
N = ( O~~l )( 6.02 x 10 ) = 2.66 x 10 atoms N, N,

The decay constant is => !:!_ =1- e-0.6930dxlo-3/700


N,
A 0.693 0.693 1 _35 x 10-u s-'
T1/2 (1620)(3.16x10') => ~=3.96x10-'
7
N,
Taking one year = 3.16 x 10 s

Now, t.N = iN = (1.35 x 10-11 )( 2.66 x 10 21 ) = 3.6 x 1010 s-1 Illustration 38


. l!.t
A radionuclide X is produced at constant rate a . At
Thus, 3.6 x 1010 nuclei decay in one second.
time t = 0 , number of nuclei of X are zero. Find
(a) the maximum number of nuclei of X .
Illustration 36 (b) the number of nuclei at time t .
A F" radio nuclide with half life· T =14.3 days is Decay constant of x· is A..
produced in a reactor at a constant rate q = 2 x 109 nuclei
Solution
per second. How soon after the beginning of production of
(a) Let N be the number of nuclei of X at time t .
that radio nuclide will its activity be equal to R =10 9
disintegrations per second? Rate= a Rate =AN
Solution :
10'
7.43 x1013 Rate of formation of X = CL {given}
0.693
14.3x3600 Rate of disintegration =AN
dN Number of nuclei of X will increase until both the rates
Now, -=q-iN will become equal.
dt
Therefore;
=> 1~=f
o q-AN o
dt a=A.Nrrur,(
CL
N=i(1-,-") => Nmax=i
=>
A
(b) Net rate of formation of X at time t is,
Substituting the values, we get
dN
-=CL-AN
dt

= 4.26

www.puucho.com
Rahul Sardana with www.puucho.com

Nuclear Physics

_!!'!_=dt
o.-1.N
=> i X 1QQ% =0.648 =64.8%
Integrating with proper limits, we have Similarly, share of U235 is 0.016%

f_!!'!_
oa-J..N
=fdt
o
and of U238 is 35.184%

Illustration 40
=> N=~(1-,-")
1. Uranium ores on the earth at the present time
This expression shows that number of nuclei of X are typically have a composition consisting of 99.3% of the
isotope 92 U238 and 0.7% of the isotope 92 U:z.;s • The half
increasing exponentially from Oto ~ .
1. Jives of these isotopes are 4.47x10'y and 7.04xl0 8 y
N respectively. If these isotopes were equally abundant when
the earth was formed, estimate the age of the earth.
o/1-. --------------- Solution
Let NO be number of atoms of each isotope at the time
of formation of the earth (t = 0) and N 1 and N 2 the number
of atoms at present (t = t). Then
N1 =Noe-;,11 ... (1)
and N 2 = N 0e-;, 21
... (2)
Illustration 39
=> N1 = i"2-1"iJ1 ... (3)
Natural uranium is a mixture of three isotopes ~U, N,
~U and ~U with mass percentage 0.01%, 0.71% and Further it is given that
99.28% respectively. The half life of three isotopes are
N, = 99.3
2.5 x 10 5 years. 7.1 x 108 years and 4.5 x 109 years ... (4)
N, 0.7
respectively. Determine the share of radioactivity of each
isotope into the total activity of the natural uranium. Equating (3) and (4) and taking log both sides, we get
Solution
(1. 2 -1. 1 )t=log,(::)
Let R1 , R2 and R3 be the activities of U234 , U 235 and
U238 respectively. => t=(-l-)log,(99.3)
1.2 -1. 1 0.7
Total activity R = R1 + R2 + R,
Substituting the values, we get
Shareof U"' ,-=
R, 1.1 N1
R 1.1N 1 + 1.2 N 2 + 1.,N, 1 lo (99.3)
0.693 0.693 g, 0.7
Let m be the total mass of natural uranium.
7.04 X lo' 4.47 X 109
Th 0.01 0.71 d 99.28
en m1 = m, m2 = m an m3 = m => t=5.97xl09 y
100 100 100
m1 m2 m3
Now, N 1 = - , N2=- and N 3 = - Illustration 41
M1 M2 M3
In the chemical analysis of a rock the mass ratio of
where M 1 , M 2 and M 3 are atomic weights.
hvo radioactive isotopes is found to be 100 : 1 • The mean

=>
R, = (;:;J;, lives of the hvo isotopes are 4 x 109 years and 2 x 109 years
respectively. H it is assumed that at the time of formation
the atoms of both the isotopes were in equal proportional,
R ~l_ + ~_!_ + ~_!_
M 1 T1 M 2 T2 M3 T3 calculate the age of the rock. Ratio of the atomic weights of
the two isotopes is 1.02 : 1.
0.01/100
-~-x---
1 --
Solution
R, 234 2.5 x 10 5 years
=> At the time of observation i.e., at time t, we have
R 0.01/100)( 1 )+(0.71/100)( 1 )
( 234 2.5xl0 5 235 7.lxl0 8 3.= 100 {given}
m2 1
+ (9928/100)( 1 )
238 4.5 X 109 Further it is given that

================================== = 4.27

www.puucho.com
Rahul Sardana with www.puucho.com

Advanced JEE Physics Optics & Modem Physics

A,= 1.02 masses of the final neutral atom and the neutral
A, 1 helium- atom.
b) P-decay : There are different simple type of P-decay
Number of ato'ms N = m p-, P' and electron capture.
A
N, m, A, 100 (i) A beta minus particle (p-) is an electron. Emission
-=-x-=-- ... (1)
N2 m, A, 1.02 of Jr involves transformation of a neutron into a
Let N 0 be the number of atoms of both the isotopes at the proton, an electron and a third particle called an
time of formation, then antineutrino ('ii) .
N1 Noe-'-i' ().2-"-1)1 (ii) P- decay usually occurs with nuclides for which
-=---=e ... (2)
N2 Noe-~t
the neutron to proton ratio ( ~ ratio) is too large
Equating (1) and (2), we get ·
for stability.
e('-,-A,)1 =100
-- (iii) In p- decay, N decreases by one, Z increases by
l.02
one and A doesn1t change.
(1.2 -1.,)t = log, (100)-log, (1.02)
(iv) p- decay can occur whenever the neutral atomic
log, (100)- log, (1.02)
mass of the original atom is larger than that of the
final atom.
(2x~O' - 4x~O')
Substituting the values, we have (v) Nuclides for which N is too small for stability can
z
t =1.834 x 1010 years emit a positron, the electron's antiparticle, which is
identical to the electron but with positive charge.
NUCLEAR RADIATIONS The basic process called beta plus p• decay
According to Rutherford's experiment when a sample of p-> n+P' +v (v = neutrino)
radioactive substance is put in a lead box and allow the
emission of radiation through a small hole only. When the
(vi) W decay can occur whenever the neutral atomic
radiation enters into the external electric field, they splits mass of the original atom is at least two electron
masses larger than that of the final atom
into three parts (a-rays, P-rays and r -rays)
(vii) The mass of v and 'ii is zero. The spin of both is

a -rays
y-rays
+
){
a. -rays
y-rays x x
Magnetic
!2 in units of
21t
!!.__ The charge on both is zero. The
+ ){ field
+
X
spin of neutrino is anti.parallel to it1s momentum
p-rays
·+
' ' ' while that of antineutrino is parallel to it's
+ momentum.
(viii) There are a few nuclides for which f3+ emission is
not energetically possible but in which an orbital
electron (usually in the k-shell) can combine with a
proton in the nucleus to form a neutron and a
(A) (B)
neutrino. The neutron remains in the nucleus and
the neutrino is emitted.
p+p+-+n+v
a) a -decay : Nearly 90% of the 2500 known nuclides are
radioactive; they are not stable but decay into other c) y -decay : The energy of internal motioil of a nucleus is
nuclides quantized. A typical nucleus has a set of allowed
(i) When unstable nuclides decay into different energy levels, including a ground state (state of lowest
nuclides, they usually emit alpha (a) or beta (P) energy) and several excited states. Because of the great
strength of nuclear interactions, excitation energies of
particles.
nuclei are typically of the order of 1 MeV, compared
(ii) Alpha emission occurs principally with nuclei that
with a few eV for atomic energy levels. In ordinary
are too large to be stable. When a nucleus emits an
physical and chemical transformations the nucleus
alpha particle, its N and Z values each decrease
always remains in its ground state. When a nucleus is
by two and A decreases by four. placed in an excited state, either by bombardment with
(iii) Alpha decay is possible whenever the mass of the high-energy particles or by a radioactive
original neutral atom is greater than the sum of the transformation, it can decay to the ground state by
= ==================================
4.28

www.puucho.com
Rahul Sardana with www.puucho.com

Nuclear Physics
emission of one or_ more photons called gamma rays or The intensity of y -decay after passing through x
gamma-ray photons, with typical energies of 10 keV to
thickness of a material is given by I= I,,-'"
5 MeV. Titls process is called gamma (y) decay. All
(µ=absorption co-<efficient)
the known conservation laws are obeyed in y-decay.

Table: Properties of a, p and y -rays


Features
! a -particles P-particles y-rays
I
Helium nucleus or doubly Fast moving electron
1. Identity Photons (E.M. waves)
ionised helium atom ( 2 He4) (-P' or P-)
'
12 Charge +2e -e ZERO
I
3. Mass 4mP (m, =mass of
4mP m, Massless
proton = 1.87 x 10-"

14. Speed ::::: 107 ms-1 1 % to 99% of speed of light Speed of light
I
All possible values between a
5. Range of kinetic energy 4MeV to 9MeV minimum certain value to Between a minimum value to
l.2MeV 2.23 MeV

6. Penetration power 10,000


'100
(r, P, a) 1 (Stopped by a paper) (100 times of p upto 30 cm
(100 times of a )
I of iron (or Pb ) sheet
7. Ionisation power
·10,000 100 1
(a>P>r)
I
18. Effect of electric or·
Deflected Deflected
I
' magnetic field Not deflected '
' !
9. Energy spectrum Line and discrete Continuous Line and discrete

I10. Mutual interaction with Produc~ heat Produces heat


Produces,. photo-electric
effect, Compton effect, pair
I
I
'
I
matter

XA a-decay
production I
z
zXA ~Z+lyA + _1eo+v
z-2 yA-4 + 2He-1
11. Equation of decay
XA ~
na yA' zXA ~ z.XA zXA~zXa+"(
Z Z'
-
A-A 1 => n,=(2n.-Z+Z')
=> 1l =---
" 4

RADIOACTIVE SERIES Table · Four radioactive series


a) If the isotop~ that results from a radioactive decay is Inumber
Mass Series
Parent
Stable end
Integer
itself radioactive then it will also decay and so on. (Nature) product
11
I
b) The sequence of decays is known as radioactive decay Thorium
4,z
(natural)
., Th'" a2Pb200 52
series. Most of the radio-nuclides found in nature are
members of four radioactive series. These are as follows
Neptunium
I 411+1 (Artificial) 93Npw B""' 52 II 83 I

c:================================= 4.29 =
www.puucho.com
Rahul Sardana with www.puucho.com

Advanced JEE Physics Optics & Modern Physics


-
Uranium Find the time at which NY is maximum and determine the
4n+2 92 U"' 82 Pb 206 51
(Natural) population of X and Z at that instant.

I 4n+3
Actinium
s9Ac221 Pb'"' 51 I
'
(Natural)
-
82
I Solution
c) The 4n+1 series starts from 94 Pu 241 but commonly Let at time t =t , number of nuclei of Y and Z are NY
known as neptunium series because neptunium .is the and N 2 • Then
longest lived member of the series.
d) The 4n + 3 series actually starts from U235 •
z
92
t=O N0 =10w Zero Zero
t=t Nx= N 0 e. . .x1 N, N,
SUCCESSIVE DISINTEGRATION AND RADIOACTIVE
EQUILIBRIUM
Rate
Suppose a radioactive element A disintegrates to form
another radioactive element B which intern disintegrates to
still another element C ; such decays are called successive
~ :
l.,N,
~ : ~ l..,N,
disintegration. Rate equation of the populations of X, Y and Z are
l.,
A B ( d~x) = -AxN x ... (1)
C

( d~y) = AxNx -1..yNy ... (2)


Rate of disintegration of A= dN, = -1.. 1N 1 (which is also the
dt
rate of formation of B) and ( d:z )=AyNy ... (3)
Rate of disintegration of B = dN, = -1..2 N 2
dt Given Ny(t) = N,1..x [e-'"' -e·',']
Ax - Ay
Net rate of] (. Rate of Rate of
formation = disintegration - disintegration
J( J . . dN (t),
For Ny to be maxrmum __Y _ = 0
( dt
ofB of A ofB
i.e., AxNx = AyNy {from equation (2)} ... (4)

=> A (N e-i.xt) = ').. No'A.x [e-Ayl _ e-Axt]


X O y A, -'}..
EQUILIBRIUM X y

In radioactive equilibrium, the rate of decay of any A -A e-Ayt


=> _X_ _ Y=---1
radioactive product is just equal to it's rate of production A.y e-Axt
from the previous member.
=> Ax= e<Ax-Ay)t
i.e.~ A1/'{1 ='A. 2N 2
Av
=> !:1.- N2 - '!2 _·(TtJ2)2
'A.2 - N1 - '!1 - (Tt/2\ => (Ax -1..y )tlog, (e) = log,G:)

1
Illustration 42 => I log,(,.x)
(Ax-Av) 1..y
A radioactive nucleus X decays to a nucleus Y with
Substituting the values of Ax and Ay, we have
a decay constant Ax = 0.1 s-1 • Y further decays to a stable
0
nucleus Z with a decay constant Ay =2. s-1 Initially t ( · ~_!_log,( /)=15log,(3)
30
O.l 30 30
there are only X nuclei and their number is N 0 =10 20 •
Setup the rate equations for the populations of X , Y and => !=16.48 s
Z . The population of the Y nucleus as a function of time The population of X at this moment
is given by Nx = Noe-i.xi = (1020)e-Co.1JC16.48)
Ny(t) = {(,.:~~Y) }{exp(-1..vt)-exp(-1..xt)} => Nx =1.92xl019

= 4.30

www.puucho.com
Rahul Sardana with www.puucho.com

Since, Ny= NxAx {From equation (4))


A.y
BASED ON NUCLEUS PROPERTIES &
RADIOACTIVITY
=> Ny =(1.92xlO")(O/) =5.76xl019

30
,. {Solutions on page 4.102)
At time t = O, number of nuclei of a radioactive substance
are 100. At t = 1 s these numbers become 90. Find the ,·
=> Nz=N,-Nx-Ny ' number of undecayed nuclei at t = 2 s .
=> 20
Nz=l0 -l.92xl0 -5.76xl0 19 19 II 2. Find the amount of heat generated by 1 mg of Po210
!
=> Nz =2.32xl019 i preparation during_ the mean life period of these nuclei if the 1
' emitted alpha particles are known to possess kinetic energy :
5.3 MeV and practically all daughter nuclei are formed I
Illustration 43 directly in the ground state.
Consider a radioactive disintegration according to the
equation A -), B -), C . Decay constant of A and B is same 3. The radioactivity of a uranium specimen with mass number !
and equal to A. Number of nuclei of A, B and C are N 0 , 238 Is 2.5x1-04 s-1 , the specimen's mass is 2 g. Find the 1
1
half-life.
0, 0 respectively at t =0 . Find
'
(a) Number of nuclei of B as function of time t . 4. Ac'l2. 7 has a half life of 21.8 years with respect to 1'
(b) Time t at which the activity of B is maximum radioactive decay. The decay follows two parallel paths, one I
and the value of maximum activity of B . leading the Th 227 and the other leading to Fr223 • The !
percentage yields of these two daughters nuclides are 1.2%
Solution .
and 98.8% respectively. What is the rate constant in yrs-1 ,
(a) Let the number of nuclei at any instant be shown in the for each of the separate paths?
table
: 5. The disintegration rate of a certain radioactive sample at
A B C I
any instant is 4750 disintegrations per minute. Five minutes
At t=O N, 0 0 I
later the rate becomes 2700,per minute. Calculate
I (a) decay constant and
At t N, N, N,
... (1)
I
I
(b) half-life of the sample.

I 6. In an agricultural experiment, a solution containing 1 mole J


of a radioactive material (T\12 =14.3 days) was injected into j
Now, dN, =1,.(N, -N,)
dt ' the roots of a plant The plant was allowed 70 hours to 1
I settle down and then activity was measured in its fruit. If the ,
=> dN, =1,.N e·" -fN
I
activity measured was 1 µCi , what percentage of activity is f
dt a 2 iI transmitted from the root to the fruit in steady state?
=> dN, +1,.N,dt=1,.N0 e·" I
I 7. A sample of 1 g of 1:ai with a half life of 2.7x107 year
=> e"dN2 + 1,.N,e"dt = 1,.N0 dt ' decays into a stable isotope .of thallium by emitting a
particles.
=> d(N,e")=1,.N0 dt (a) What is the activity of the sample?
Integrating, we get {b) What will be the activity of the sample after 2 years?
(c) After-what time does the activity reduces to 25% of the
N, ' i
f d(N,e")= J1,.N,dt I
original activity?
I
0 0
I a. A number N0 of atoms of a radioactive element are placed \
~ N 2 =AN0 te-u inside a closed volume. The radioactive decay constant for I
(b) Activity of Bis, R, =1,.N2 =)..'N,te·" the nuclei of this element is A1 • The daughter nuclei that j
form as a result of the decay process are assumed to be l
For maximum activity, we have radioactive, too, with a radioactive decay constant A2 • \

dR, =0 Determine the time variation of the number of such nuclei. ,


dt Consider two limiting cases, when A.1 » A2 and A1 « A2 • j

=> 1=2.),_ 9. Calculate the probability that a radioactive atom having a


I
f
mean life of 10 days decays during the fifth day. I
=> R_ = 1,.N,
e 10. Old wood from an Egyptian tomb, 4500 years old has C-14
activity of 7.3 dis. min- 1 g-1 • 'Old wood known to be 2500 1
years old has a C-14 activity 9.3 dis. min-1 g-1 • j
================================::::J = 4.31

www.puucho.com
Rahul Sardana with www.puucho.com

Advanced JEE Physics Optics & Modern Physics

(a) · What is half life !or C-14? l!.Jn = [zm, +(A-Z)m.J-m( 2 XA)
(b) What is the activity of fresh wood? _
I/ 11. Determine the amount of 84 Po
210
(polonium) necessary to
This mass defect exists in the form of binding energy of
nucleus, which is responsible for binding the nucleons into a
l provide a source of a particles, of 5 millicurie strength. If small nucleus. So,
10
hail life ot.polOnium is 138 days, given 1 curie =3.7x.10 Binding energy of nucleus = (!!.m)c' = (931.5)!!.m (in MeV)
disintegrations/sec.
andb"m d"~g energy per nuc1eon (t.m)c'
=-A--
12. The specific activity of a preparation consisting of
l radioactive Co58 and non-radioactive Co59 is equal to If the masses are taken in atomic mass unit, the binding
j 2.2x10 12 dissec·1gm. The half 'life of Co58 is 71:3 days. energy is given by
! Find the ratio of the mass of radioactive cobalt in that
1 preparation to the total mass of the preparation. B.E.=[(zm, +(A-Z)m,,)-m( 2 xA)]931.5 MeV
Dividing the binding energy by the number of nucleons A ,
13. The half Jives of radioisotopes P32 and P33 are 14 days
and in the nucleus, We obtain the binding energy per nucleon.
25 days respectively. These radio isotopes are· mixed in the The stability of a nucleus is measured by the binding energy
ratio of 4: 1 of their atoms. If the initial activity of the mixed per nucleon.
sample ·is 3 mCi, find the activity of the mixed ·isotopes I The more the binding energy per nucleon of the nucleus the
_ after60days. ------ - ----- -- _ _ __J more stable a nucleus is.
MASS ENERGY EQUIVALENCE Packing fraction (f) is the mass defect per nucleon. So,

According to Einstein the mass and energy are equivalent f = i'.m


i.e., mass can be converted into energy and vice-ve_rsa. The A
mass energy equivalence relation is
l!.E = c't.m CONCEPTUAL NOTE(S)
Accordingly, .annihilation of 1 kg· mass is equivalent to a} It is not the binding energy which accounts for th8 stability
of nuclells.
energy given by b) The stability of nucleus is governed by binding energy per
2 nucleon. The more the binding· energy per nucleon, the
/1E =1 X (3 X 10 5 )
more stable a nucleus is.
=> l!.E=9x1016 J
Energy corresponding to annihilation of 1 amu of mass is Illustration 44
l1E=(1.67x10·")(9x1016 ) J Find. the binding energy of 1
;c? Also find the
21
(1.67 x 10- )(9 x10 ) 16 binding energy per nucleon. Given mass of !H =1.0078 u,
l!.E -'-----'--'c~-~ eV
1.6x10-" :11 = 1.0087 u, :'C = 12.00004 u .
l!.E = 931.5 MeV Solution
One atom of 1;c consists of 6 protons, 6 electrons and
MASS DEFECT AND BINDING ENERGY
6 neutrons. The mass of the rm-combined protons and
It has been observed that the mass of a nucleus is always less electrons is the same as that of six ~ H atoms (if we ignore
than the mass of its constituent nucleons (i.e., protons +
the very small binding energy of each proton-electron pair).
neutrons). The difference between the total mass of the
nucleons and the mass of the nucleus is called the mass Mass of six ) H atoms = 6 x 1.0078 = 6.0468 u
defect (i'.m) . This is due to the fact that when nucleons Mass of six neutrons = 6x1.0087 = 6.0522 u
combine to form a nucleus, the binding energy of nucleons is Total mass of component particles = 12.0990 u
liberated. The binding energy is equal to the work that must Mass of 1;c atom = 12.00004
be done to split the nucleus into particles constituting it.
. Mass defect ·= 0.0990 u
Let m( XA) be the mass of nucleus,
2
Binding energy = (931)(0.099) =92 MeV
mp = the mass of proton and
Binding energy per nucleon = ~~ = 7.66 MeV
mn =mass of neutron
then, the mass defect is given by
81n = mass of constituent nucleons - mass of nucleus

= c::==================================
4.32

www.puucho.com
Rahul Sardana with www.puucho.com

Nuclear Physics
Illustration 45 peaks corresponding to nuclei 2 He4 , 6 C12 and 8 Q 16 •
A neutron breaks into a proton and electron. Calculate The peaks indicate that these nuclei are more stable
the energy produced in this reaction in MeV . Mass of an than those in their neighbourhood. It confirms the
electron = 9 x 10-31 kg, Mass of proton = 1.6725 x 10-27 kg , reason for extraorciiuary stability of a-particle .
Mass of neutron 1.6747 x 10-27 kg . Speed of light c) After mass number 20, binding energy per nucleon
=3x10 8 ms-1 • increases gradually and for mass number between 40
and 120, the curve becomes more or less flat. The
Solution
average value of binding energy per nucleon in this
region is about 8.SMeV. For A=56( 26 Fe 56 ), the
Mass ) ( mass of proton ) binding energy per nucleon is maximum and it is equal
Mass defect (~m) = of - + to 8.8 MeV.
( neutron mass of electron d) After mass number 120, binding energy per nucleon
~m = [(1.6747x 10- )-(1.6725x 10- +9xIO-'" )]21 27 starts decreasing and drops to 7.6 MeV for uranium.
This low value of binding energy per nucleon in case of
~In= 0,0013 X 10-27 kg heavy nuclei is unable to ha:ve control over the
Energy released Q = ~c' repulsion between the large number of protons. 'Such
nuclei are unstable and are found to disintegrate by
2
Q=(0:0013x10-27 )x(3xl08 ) =1.17xl0-13 J emitting a-particles . The emission of cx-parti~e not
1.17x10-13 only decreases repulsive force .inside the nucleus but
Q --~~
19
0.73 x IO' eV = 0.73 MeV also increases the value of B.E./ A of the nucleus due to
1.6xl0-
its extraordinary stable structure ( a-particle has large
VARIATION OF BINDING ENERGY PER NUCLEON WITH binding energy). It is call~d a-decay.
MASS NUMBER A Sometimes, the heavy nuclei increase the value of their
The graph represents the average binding energy per B.E.f A by emitting an electron. It is called P-decay .
nucleon in MeV against mass number A . It is observed that Inside the nucleus, an electron does not exist. It is
the binding energy for nuclei (except ,He', ,c 12 and 8 0 16 ) created at the time of f3-decay due to conversion of a
rises first sharply, reaches a maximum value 8.5 MeV at neutron into proton. The f3-decay leads to increase in
A=S0 and then falls slowly, decreasing to 7.6MeV for Coulomb's repulsive force, but it increases B.E./ A and
elements of higher mass number A = 240 . Following facts also improves the neutron-proton ratio.
can be concluded from this curve. All such nuclei, which undergo a and P-decay are
Region of Greatest Stability called radioactive nuclei.
,_..._______
9.0
-,• e) The binding energy per nucleon has a low value for
both very light and very heavy nuclei. In order to attain
O" / ' .. higher value of binding energy per nucleon, the lighter
~ c·;,
7.0
;'·'
.
nuclei may unite together to form a heavier nucleus
(process of nuclear fusion) or a heavier nucleus may
split into lighter nuclei (process of nuclear fission). In
6.0 both the nuclear processes, the resulting nucleus
.. acquires greater value of binding energy per nucleon
5.0 along with the liberation of enormous amount of
energy.
4.0

3.0 NUCLEAR STABILITY


Among about 1500 known nuclides, less than 260 are stable.
I.
2.0 I· The others are unstable that decay to form other nuclides by
I I I I I I I I I I I I
emitting a, P-particles and y -EM waves. (This process is
1.0 I I I I I I I
0 50 100 150 200 250 called radioactivity). The stability of nucleus is determined
by many factors. Few such factors are given below :
a) The binding energy per nucleon for light nuclei, such as
2
1 H ,isverysl)1all (=1 MeV).
a) Neutron-proton ratio ( ~ Ratio) : The chemical
b) The binding energy per nucleon increases rapidly for _ properties of an atom are governed entirely by the
nuclei up to mass number 20 and the curve possesses number of protons (Z) in the nucleus, the stability of

i:::::=================================== 4.33 =
www.puucho.com
Rahul Sardana with www.puucho.com

Advanced JEE Physics Optics & Modern Physics

an atom appears to depend on b9th the number of (iii) Only five stable odd-odd nuclides are known :
protons and the number of neutrons. 1
H 2 , 3 Li 6 , 5 Be10 , 7 N 14 and 75Ta18/J
(i) For lighter nuclei, the greatest stability is achieved c) Binding energy P.er nucleon : The stability of a nucleus
when the number of protons and neutrons are is determined by value of it's binding energy per
approximately equal (N ~ Z) i.e., ~ =1 . nucleon. In general, higher the value of binding energy
per nucleon, the more is the stability of the nucleus.
(ii) Heavy nuclei are stable only when they have more
neutrons than protons. Thus heavy nuclei are ARTIFICIAL TRANSMUTATION - NUCLEAR REACTIONS
neutron rich compared to lighter nuclei (for heavy A radio-active substance breaks up by emitting radiations. ·
nuclei, more is the number of protons in the The daughter nucleus, left behind, has different physical and
nucleus, greater is the electrical repulsive force chemical properties and is assigned a new place in the
between them. Therefore more neutrons are added periodic chart. Thus radio-activity is the phenomenon by
to provide the strong attractive forces necessary to which a substance gets converted into another one. Titls
keep the nucleus stable.) change can be brought about by artificial method, by
(iii) Figure shows a plot of N verses Z for the stable bombard~g a given nucleus with some radiation. The
nuclei. For mass nurn0er upto about A= 40. For particles constituting the incident radiation must possess
larger value of Z the nuclear force is unable to sufficient kinetic energy so as to penetrate into the given
hold the nucleus together against the electrical. nucleus. As they enter the given nucleus, a compound
repulsion- of the protons unless the number of nucleus (an intermediate state) is formed which is generally
neutrons exceeds the number of protons. unstable. The compound nucleus then breaks up to produce
product nucleus by emitting radiation. The process is,
schematically, represented as
1-::·o 4 < - - - - - - - - - - - - - - - I + T - > C* -> P + 0
96it-~,---,, __ -;,_
~~i':
e::~J (!:~J (~~J (~::J (~J
.~gi __:_j;_..__-+ This is a reaction in which only the nuclei take part. Orbital
electrons have no contribution to it. Such reactions are
known as nuclear reactions.
~
:;; LAWS GOVERNING NUCLEAR REACTIONS
.c
E
a) Law of Charge: The electric charge involved in nuclear
"
C:
C: reactions must be same before and after the reaction. So,
i charge number is conserved in nuclear reactions.
z"' b) Law of Number of Nucleons: The total number of
nucleons involved in a nuclear reaction must be same
before and after the reaction. So, mass number is also
conserved in nuclear reactions.
c) Law of Conservation of Energy: The total energy (rest
.32 40 4815.6J6~ft;ff~80·_···aa!-96~ mass energy + K.E.) of the reacting particles must be
Atomic number (Z) equal to the total energy of the product particles.
d) Law of Conservation of Linear Momentum: The total
At Bi(Z = 83, A= 209), the neutron excess in linear momentum of the reacting particles must be
equal to the total linear momentum of the produCt
N -Z = 43. There are no stable nuclides with particles.
2>83. e) Law of Conservation of Angular Momentum: Total
b) Even or odd numbers of Z or N : The stability of a angular momentum of nuclei before and after reaction
nuclide is also determined by the consideration whether must be the same.
it contains an even or odd number of protons and
neutrons. Q-VALUE OF A NUCLEAR REACTION
(i) It is found that an even-even nucleus (even Z and Consider a nuclear reaction, schematically represented by
even N) is more stable (60% of stable nuclide have equation
even Z and even N ). I+T->P+O
(ii) An even-odd nucleus (even Z and odd N ) or
Let E1 , Ep and £0 be the kinetic energies associated with I,
odd-even nuclide (odd Z and even N) is found
to be lesser sable while the odd-odd nucleus is P and O respectively while the target Tis at rest initially.
found to be less stable. Q-value of a nuclear reaction is given by,

· 4.34 .:======================================
www.puucho.com
Rahul Sardana with www.puucho.com

Nuclear Physics
Q=E,+E0 -E1 Reactants Products
Let m11 my., mp, m0 , respectively,·bethe masses of I, T ,- ;u 7.01600-3m, ; Be 7.01693 - 4m,
P and 0
BEFORE REACTION AFTER REACTION
: H 1.0783 - lm, !n 1.0866
Total 8,02383- 4m, Total 8.02559-4m,
Energy of I= m1c2 + E1 Energy of P = mpc 2 + Ep
The Q -value of the reaction
Energy of T = m,c' Energy of O = m0 c2 + E0
Q = -0.00176 u = -1.65 MeV
Total energy of the Total energy of the
The energy is supplied as kinetic energy of the bombarding
system= m1c2 + E1 + "'rc2 system= mpc2 + Ep + m0 c2 + E0 proton. The incident proton must have more than this
By Law of Conservation of Energy, energy because the system must possess some kinetic energy
m1c + E, + 1tlrC2 = mpc 2 + Ep + m0 c2 + E0
2 even after the reaction, so that momentum is conserved.
With momentum conservation taken into account~ the
o Q = E, + E0 -E, =[(m1 +m, )-(m, +m0 )]c' minimum kinetic energy th~.t the inciderit particle can be
where t.m is the mass defect between initial and final found with the formula.
particles.
E., =-(1+ :)Q=-(1+f)(-1.65)~1.89 MeV
CASE I Exo-ergic reactions: A reaction is said to be exo-
ergic if Q is positive.
o (m,+m0 )<(m1 +m,) Illustration 47
The part of mass which disappears gets converted A neutron with kine ti~ ~nergy K = 10 MeV activates a
into the energy in accordance with- Einstein's nuclear reaction n+ 12C-----.+ 9 Be+a. Find the kinetic
Mass-Energy equivalence. energy of the alpha particles outgoing at right angle to the
CASE II Endo-ergic reaction: A reaction is said to be endo- direction of incoming neutrons.
ergjc if Q is negative. Take ll = 931.5 MeV and threshold energy of reaction
o (m, +m0 ) > (m, +m,) (E,.)= 6.17 MeV. .
i.e., the sum of the masses of product particles is Solution
greater than that of reactant particles. For this
reaction to proceed, the incoming particle must
possess an energy equivalent to the mass defect.
C
The minimum amount of energy that a n
bombarding particle must have in order to initiate
an endoergic reaction, is called Threshold Energy
Q------+
m, Qm,
E,,,.
Since,
Q+K, =K, +K4 ..• (1)
where, mp is the mas~ of the projectile i.e. the Applying Law of Conservation of Linear Momentum
nucleus used to hit the ,target and Along x-axis
111r is the mass of the target nucleus .J21n,K, = .J2m,K, cos8 ... (2)
Along y-axis
Illustration 46
.j2m,K3 = .J2m,K, sin8 ... (3)
How much energy must a bombarding proton possess
to cause the reaction. Squaring and adding equations (2) and (3), we get_
iLi+iH-+:Be+~n 111tK1 + m3 K3 = m4 K4

Solution => K4=m,K1+m3Ks


Since the mass of an atom include the masses of the m4 m4
atomic electrons, the appropriate number of electron masses Substituting value of K, in equation (1) and rearranging, we
must be subtracted from the given values. get

r:::================================== = 4.35

www.puucho.com
Rahul Sardana with www.puucho.com

Advanced JEE Physics Optics & Modern Physics

. -E. --o:l 7 - 5 69 M V Suppose the parent nucleus ~X be at rest before decay. Let
w h ere, Q ( ) -- 1 -- . · e
1+"" 1+- -va and Vy be the· velocities· of the a-particle and the
_m, 12 daughter nucleus. Applying the Law of Conservation of
Momentum, we get
~ (1+¼)K, =-5.69+(1-¼)(10) myVy = ma Va •.• (1)

~ K, =2.21 MeV As the energy Q -released in the decay process appears in


the form of kinetic energy of a- particle and the daughter
ALPHA DECAY nucleus, So :we have
'Alpha decay is a process in which an unstable nucleus 1 2 1 2 Q
2mttva +2myVy =
transforms itself into a new nucleus by emitting an alpha
particle (a helium nucleus, ~He). Substituting the value of Vy from equation (1), we get-
Since an a -particle has two Protons and two neutrons, so 1 . v2 +-
-m
1 2 2
mo.Va =Q
after an a. -decay, the parent nucleus is transformed into .a 2 a a 2 m~
daughter nucleus with mass number smaller by 4 and atomic
number sg,aller_ by 2. 1 21.22
2mymQv(l +2mava =myQ
An alpha decay can be expressed by the equation
:x---+ ~jY + !He+Q ½(my+m(l)mav! =myQ
Here Q is the energy released in the 'pi-ocess and is
determined from Einstein's mas~-energy relatioii. which K =.!:.m
Cl • 2 a_«.
v'=(- my
my
+m(l
)Q
gives
- -
Q= ( mx·-mr-mHt ) 2
c Since, my = (A - 4) amu and ma = 4 amu, theref~re,
where mx, my ._and mHt are the masses of the parent nucle1;1s
X, daughter nucleus Y and the a -particle respectively.
The energy Q is shared by the daughter nucleus X and the
a -particle. As the parent nucleus is at rest before its
a -decay, the a -parti~les are emitted with fixed energy and " '
hence are mono-energetic. This energy can be determined by For example, in the a.- decay of a randon nucleus ~Rn, we
Applying the Laws of Conservation of Energy and have
Momentum. Q=5.587 MeV
For example, uraru'!'Il (238) on emitting an u -particle
changes into thorium (234) as ~ . K. =( A~4)Q ( 222 - 4 ) x5.587 MeV
222
~u~~Th+ !He+Q
~ K. = 5.486 MeV = 5.486 x 1.6 x 10-19 J
Similarly, polonium (208) is transmuted into lead (204) as
Since, m. = 4 arnu = 4 x 1.66 x 10·21 kg
'!Po---,--),~Pb+ !He+Q
Generally, the nuclei with mass number 210 or more 2 X 5.486 X 1.6 X 10-19 -1
Va = 4 X 1.66 X 10-2'
ms
undergo a -decay. In such nuclei, the long range repulsive
forces between the protons dominate Over the short range ~ v. =1:62xl01 ms·1
nuclear forces which bind the various nucleons together. By
emitting a. -particles, these nuclei achieve greater stability.
An a -particle has a high value of binding energy THEORY OF a-DECAY (TUNNELLING EFFECU
(= 28 MeV) . After the emission of an a -particle, the The a.- particles emitted by different raclioactive nuclei have
binding energy per nucleon of the emitting nucleus increases kinetic energy ranging from 4 to 9 MeV. The nucleus of an
and the residual nucleus·becomes more stable. a- emitter posses a barrier of height about 25 MeV. Figure
shows a plot of the potential energy U of the system
SPEED OF EMITTED a-P.ARTICLES consisting of the a- particle and the residual nucleus. The
a- particles are short of about 16 to 25 MeV of energy,
Consider the alpha decay process equation i.e.,
needed for the emission. There~ore, classically, we cannot
1X------+t~Y + ;He+Q explain the· emission of a.- particles by radiOactive nuclei.
The speed of the emitted a- particles can be calculated by
using the Laws of Conservation of Energy and Momentum.

= 4.36

www.puucho.com
Rahul Sardana with www.puucho.com

Nuclear Physics
u Given : my :::: 228.03 amu '"• = 4.003 amu
Potential barrier
m(;n) = 1.009 amu m(!H)=l.008 amu
K =KE of a-particle 1 amu=l.66x10-" kg=931.5 Mevc-2

2
Solution
2Ze
r=--
4ne0K .(a) The given equation is,
~x~~Y +!He
A=228+4=232
Plot of potential energy U of an a-particle as a function of and 92=z+2
distance r from the centre of the residual nucleus.
~ z=90
In 1928, Gamow, Congdon and Gurney explained the
emission of a.- particles in terms of the penetration of the (b)
nuclear potential barrier on the basis of Quantum Theory.
According to this theory, we have
a) An ct- particle may exist as an entity (already formed)
inside a nucleus before it escapes from the nucleus.
l.lxl0 2 x2xl.6x10-19 x3x10 3
•=
b) The ct- particle is in a state of constant motion inside ~ V
the nucleus with a speed of about 10' ms-1 • 4.003 x1.66 x 10-"
~ v(l =4x106 ms-1
c) Quantum mechanically, even an ct- particle having
insufficient kinetic energy has a small but . finite Applying Law of Conservation of Llnear Momentum, we get
probability p of its crossing the potential barrier.
As the size. of the nucleus ~ 10-14 m and speed of a- particle
(4.003){4x106 )
~ 10 ms-1 , the a- particle takes about 10-21 s to move
7 ~ v,
m, (228.03)
across the nucleus. Thus a- particle .Presents itself before the
potential barrier 10 21 times in a second. The probability of ~ vy =7x10 4 ms-1
escape of an ex- particle from a nucleus will be Therefore, energy released during the process is
P=pv
21
As v is large (10 s-1 ) , so P is sufficiently large and the
ex- particle can tunnel through the energy barrier which is (1.66x10-")
classically impossible. Hence a.- decay occurs as a result of ~ E
{2xl.6x10-13 )
barrier tunnelling.
2
The barrier tunnelling explains why every ~U nuclide in a [C4.003){4x10') +(228.03){7x10')'] MeV
sample of ~U atoms does not decay at once, even when its 34
~ E = 0.34 MeV = 0. amu = 0.000365 amu
decay process has a positive Q value. Consequently, the 931.5
half-lives for a.- decay of most of the alpha unstable nuclei · Therefore, mass of ~ X = m, + m. + 0.000365 = 232.033365u
are very long. For example, the half-life of ~U for ct- decay So, mass defect is
is 4.5 x 109 year . l'.m = 92(1.008) + (232-92)(1.009)-232.033365
~ l'.,n = 1.962635 amu
Illustration 48 ~ Binding energy =1.962635x931.5 MeV =1828.2 MeV
A nucleus X-initially at rest, undergoes alpha-decay,
according to the equation BETA DECAY
~x~~Y:ra The process of spontaneous emission of an electron (e-) or a
(a) Find the value of A and z in the above process. positron (e•) from.a nucleus is called beta decay.
(b) The ct -particle in the above process is found to move Like a- decay, j3-decay is a spontaneous process, with a
in a circular track of radius 1.1 x 10 2 m in a uniform definite disintegration energy and half-life. It is also a
magnetic field of 3 x 103 T. Find the energy (in MeV) statistical process, obeying the Law of radioactive Decay.
released during the process and binding energy of the
In beta minus (p-) decay, the m~s number of the
parent nucleus X .
radioactive nucleus remains m1changed but its atomic

4.37 =
www.puucho.com
Rahul Sardana with www.puucho.com

Advanced JEE Physics Optics & Modem Physics

number increases by one. An electron and a new particle However in j3- decay, the disintegration energy is shared in
antineutrino (V) are emitted from the nucleus, as in the all proportions between the three particles i.e., daughter
decay. nucleus, electron (or positron) and antineutrino (or
~P----+ ::s + e- + V neutrino). As a result, the kinetic energy of the electrons (or
positrons) is not fixed. Their energy varies from zero to a
In general, the beta minus decay may be represented as maximum value K=. Thus j3- rays have. a continuous
:x----+ 2 :iY + -~ e + V energy spectrum, as shown in Figure.
OR 1X----+ 2 jY+Jr_+V

~r
.,.-'-~
The electron emitted from the nucleus is called a beta
particle, denoted by p- .
In beta plus (P') decay, the mass number of the radioactive
.0 ,,,
E "
"C;t;:
e
<D,,, I
I
"' "\,
nucleus remains unchanged but its atomic number deCI'eases >o I

"' "'
:;; a.
by one. A positron (e') and a new particle neutrino ( v) are ,;
a:
emitted from the nucleus, as in the decay. /
' ./
'fiNa----+~Ne+e+ +v 0 0.2 0.4 0.6
In general, the beta plus decay may be represented as Kinetic energy (MeV)-
1X----+ 2 ~Y++~e+v The distribution of the kinetic energies of positrons
emitted in the decay of :cu.
OR ~X----+ 2 jY +_p+ +v
The maximum kinetic energy or end Point energy K= must
The positron so emitted is called·a beta plus particle (P') be equal to disintegration energy Q. When the electron (or
The positron iS an antiparticle of electron. It has a positive positron) has maximum energy, the energy carried by the
charge equal in magnitude to the charge on an electron and dau'ghter nucleus and neutrino is nearly zero.
has a mass equal to the mass of an electron. Similarly,
neutrino and antineutrino are antiparticles of each other. --- . --~-
-- ------·-- --·CONCEPTUAL NOTE(S)
Both are massless, chargeless particles having spins ±.!..
2
a) Since neutrinos (or antineutrinos) ·are massless and
chargeless, they interact so weakly with matter that it
Although a nucleus contains no electrons, positrons and becomes very difficult to detect them. They can penetrate
through earth without being absorbed. By ingenious
neutrinos, yet can it eject these particles. It is believed that experiments, neutrinos have been' detected and their mass
electrons, positrons and neutrinos are qeated during the and spin or intrinsic angular momentum have been
process of beta decay. If the unstable nucleus has excess measured.
neutrons than needed for stability, a neutron converts itself b) Electroa:i capture. Some proton rich nuclei capture one of
into a proton. So in a beta-minus decay, an electron and an the atomic electrons (usually from the K shell). A proton in
antineutrino are created and emitted from the nucleus via the nucleus combines this electron forming a neutron. A
neutrino created in the process is emitted from the
the reaction given by
nucleus.
n---+p+e-+V p+e- ...+n+v
If the unstable nucleus has excess protons than that needed The entire process may be represented as
for stability, a proton converts itself into a neutron. So in a
~X+e-----+l,Y+v
beta-plus decay, a positron and a neutrino are created and
emitted from the nucleus via the reaction given by This process is called electron capture or K-capture. The
vacancy created in the K shell is filled by trans"ition of
p---+n+e++v electrons from the outer shells. This results in the emission
Clearly, a beta decay process involves the conversion of a ---~_l_i:~aracteristic X-rays.
neutron into a proton or vice versa. These nucleons have
nearly equal masses. That is why the mass number A of a Illustration 49
nuclide undergoing beta decay does not change. Neon - 23 beta decays in the following way:
~Ne----=,~Na+ _~e+V
CONTINUOUS ENERGY SPECTRUM
FOR BETA RAYS Find the minimum and maximum kinetic energy that the
In both ex- and p-decays, the disintegration energy Q beta particle -~ e can have. The atomic masses of 23 Ne and
depends on the nature of the radionuclide. In the ex-decay of "Na are 22.994511 and 22.989811, respectively.
a particular radionuclide, every emitted a- particle has a
definite amount of kinetic energy.
= 4.38

www.puucho.com
Rahul Sardana with www.puucho.com

Nuclear Physics
Solution Usually, y- rays are emitted after a- or p- decay, but there
Reactants Products are long lived radioactive nuclei that emit only y- rays.
~Ne 22.9945 - lOm, ~Na 22.9898-11 m,
CLASSIFICATION OF NUCLEAR REACTIONS
-~e-me
Nuclear reactions can be classified into the following
Total 22.9945 - lOm, Total 22.9898- lOm, categories.
Mass defect = 22.9945 - 22.9898 = 0.0047 u
Q = (0.0047)(931) = 4.4 MeV A. ELASTIC SCATTERING

The P-particle and neutrino share this energy. Hence the The incident particle gets deflected without any change in its
energy, i.e.,
energy of the P-particle can range from 0 to 4.4 MeV .
iHe+ 1~Au-------+ 1;:Au +~He
GAMMA DECAY The bombarding particle passes sufficiently at large distance
away from the target nucleus so as to get repulsion which
The process of emission of a y- ray photon during the changes its direction of motion without any change in its
.radioactive disintegration of a nucleus is called gamma energy.
decay.
As the emitted r- ray photons have zero rest mass and carry B. INELASTIC SCATTERING
no charge, so in a y- decay the mass number and atomic If the bombarding particle passes close to target it gets
number of the nucleus remain unchanged and no new deflected. Due to strong repulsion, the target particle also
element is formed. A y- decay can be expressed as acquires some energy. So the energy left with the scattered
:x
(Excited state)
-----+ ;x
(Ground state)
+ y particle is less than that it had initially.
1
l H + 73 Li-'>.7Li+
3
1
1H
A nucleus does not contain photons, yet it can emit photons.
These photons are created during the emission process. We ~Li means existence of ~Li in one of its excited states.
know that a nucleus can exist in different energy states. After
an a or a P- decay, the daughter nucleus is usually left in C. SIMPLE CAPTURE
the excited state. It attains the ground state by single or The incoming particle is captured by the target nucleus. The
successive transitions by emitting one or more photons. As product nucleus which is generally in the form of excited
the nuclear states have energies of the order of MeV, state decays to the ground state by emitting y-ray of energy
therefore, the photons emitted by the nuclei have energy of
hv.
the order of several MeV. The wavelength of such high
energy photons is a fraction of an angstrom. The short
wavelength electromagnetic waves emitted by nuclei are
called y- rays. D. DISINTEGRATION
An example of y- decay is shown through an energy level (NUCLEAR TRANSMUTATIONS)
diagram shown in Figure. Here an unstable ;co nucleus is The intermediate compound nucleus breaks up and ·results
transformed via a J3- decay into an excited :Ni nucleus, in a product nucleus and an outgoing particle. The product
nucleus has different chemical properties as compared to the
which in tum reaches the stable ground state by emitting target particle. Majority of nuclear reactions belong to this
photons of energies 1.17 MeV and 1.33 MeV, in two category. Such nuclear disintegrations are called Nuclear
successive y- decay processes. Transmutations.
a) Disintegration by a-particles
(i) (a, p) reactions

E,= 1.17 MeV


:x+~He-)>::~c* -)>~:iY +~H
{C • is Compound Nucleus}
E,= 1.33 MeV EXAMPLES
The historical experiment of Rutherford is an a
~Ni induced transmutation, an ( a, p) reaction.

Energy-level diagram showing the emission of 7N14 + ;ie4-------+ eF1s·-------+ i!O + iH1
r-rays by a ~Co nucleus subsequent to beta decay. and is exoergic in nature (Q > 0). Other useful
(a, p) reactions are

4.39 =
www.puucho.com
Rahul Sardana with www.puucho.com

Advanced JEE Physics Optics & Modern Physics

1 1
~B + !He-----+ ;N *-------+
1
:c + ~H
~Na+ !He-------+~AI *-----+:Mg+ :H EXAMPLES
11 8 +1H-----+ ,2C*-----+ 11c+1n
~Al+ !He-----+ ~;p *-------+ ;!Si+ :H 6 I 6 6 0

{ t~c~ ,~B
:~sc + !He-----+ :~v *------+:Ti+ :H
(a= -1. 763 MeV) (Half-life = 2.5 min)
(ii) ( u, n) reactions
"N 'H -----+ 12
11 a+,. "M 9 • -----+ 12
"M 9 + 'on
; X+ ~He-------+ 1:;c *-> ~;iY + ~n { ~Mg~;:Na
{C* is Compound Nucleus)
(Q = -4.84 MeV) (Half-life= 12.3 s)
EXAMPLES 1 54 1
54
241 Cr + H---t 55
25Mn * ----t 2 5+
Mn 0n
~Li+ !He-----+ ~8 •------+ ~8 + 6n
1 1
{
~Mn~ :er
:se + !He-----+ :c •------+ :c + 6n
1 1
(0=-2.16 MeV) (Half-life=310 days)
1
!F + !He-----+ ~Na•------+ ~Na+ ~n 63
~
Cu+ 11H----t 64
~
Zn *-----+ 63
~
Zn+ 10n
~Na+ :He-----+ ~Al*-------+ :Al+ 6n · :zn~ :cu
{

b) Disintegration by protons
(Q=-4.15 MeV) (Half-life=38.5 min)
(i) (p, u) reactions: When the reactions yield
The (p, n) reaction is always endoergic.
a-particles. The (p, a.) reactions are usually
(iii) (p, y) reactions: When the reactions yield
exoergic and have the general form
y-photons . The compound nucleus formed by
; X + ~H-> ~:ic *-------+ ~~iY + ~He absorption of proton by the target nucleus does not
{C • is Compound Nucleus) emit any nuclear particle but goes down to the
EXAMPLES
ground state emitting one or more y-photons . The
~u+:H->!Be*-------+:He+iHe (0=4 MeV) (p, y) reaction is the radiative capture of proton.
;u+:H------+!Be*->!He+iHe (0=17.35 MeV) The general equation of (p, y) reaction is
1 1
~s+:H------+ :c·------+!Be+iHe (Q-8.59 MeV) :x +~H-----+::ic * -----+t:~c + r
The product nucleus !Be is highly ustable and } {C • is Compound Nucleus}
decays almost immediately as !Be-------+!He+iHe,
{ EXAMPLES
so that the final reaction gives three a-particles
;u + iH-----+ :se •-----+!Be+ r
19
9 F 11H-------+ 10
+ 20
Ne *-------+ 16
6
4
+
0 2He (a -- 812 MeV)
1
;N +~H----t 1!0 * ----t 1!0 + y
~1Mg + iH-----+ ~Al* ----t ;';Al+ y
(Q = 2.38 MeV) gAI +iH----t ;!Sr* ----t ~Sr +y
:cu +:H-------+ :zn *-------+:Ni+ !He (iv) (p, d) reactions: When the reactions yield
(Q = 3.76 MeV) deuterons. The general equation of this type of
(ii) (p, n) reactions: When the reactions yield reaction is
neutrons. The general equation of this type of : X + !H ----t A-£x + ~ H
reactions is This is an example of direct reaction without any
:x+!H----t1:iC*----t 2 .'.iY +~n formation of the compound nucleus.
EXAMPLES
{C * is Compound Nucleus}
the product nucleus being isobaric with the target
~Li+ ;H-t :u + ~H
nucleus. :se + ; H ~ :se + ~H
Since two isobars differing in Z by unity cann~t c) Disintegration by neutrons
both be stable, the product nucleus is p+ active, (i) {n, a) reactions: When the reactions yield
decaying by p• emission (or electron capture) into a-particles. The general equation for (n, u)
: X (the same as target nucleus): reactions is

= 4.40

www.puucho.com
Rahul Sardana with www.puucho.com
,

. N11clear Physics

: X + ~n-------+ A+iC *-------+ :=;y + ~He The radiative caph.µ'e raises the target nucleus to
an excited isomeric state and by releasing the
{C* is Compound Nucleus) excitation energy as y photons, the product
The (n, a)· reactions are usually exoergic, i.e. Q is nucleus becomes an isotope of the target nucleus.
positive, particularly for medium heavy nuclei. The isotopic product nuclei are generally p-
EXAMPLES active, as it has a higher neutron-proton ratio
compared to the original one. In fact, this method
:u+~n-Jo;u·--t:H+:He (0·=4.785 MeV)
of inducing P-activity is used extensively with
followed by :H-4 ~He+ -~e + V0 copious supply of neutrons from reactors.
1
~B+~n-J, 1~B*->;U+!He (0=2.79 MeV) EXAMPLES

fiCt+~n-Jo:;c1•------4-~p+:He (0_=0.935 MeV)


The first two reactions are utilised in the followed by ~H ~rr , :He+-~e+V0
12
construction of neutron detectors as they have fairly d) Disintegration by deutrons
large cross-sections. The first reaction also gives a
method of .producing tritium which is _useful in (i) (d, a) reactions: When the reactions yield
nuclear fusion.
a-particles. The general equation of ( d, a)
(ii) (~' p) reactions: When the reactions yield protons.
reactions is
The general equation for ( n, p) reacti~ns is
1x+ iH-> ~::c *-> t:Y + ~He
:x +~n-------+A+;c *-------+ 2 ~Y +!H {C * is Compound Nucleus)
{C * is Compound Nucleus) The Q-values are usually positive and the
The product nucleus Y is an isobar of the target reactions exoergic. Some example of ( d, a)
nucleus X with Z-value one unit lower and is
reactions are given as under.
thus p- active decaying to the target nucleus. So, EXAMPLES
2 ~Y ~ ~X
So, the (n, p) reactions are such that the initial and (O =22.4 MeV)
final nuclides are identical. The process therefore
appears to be a conversion of neutron into a proton (0=13.57 MeV)
and an electron. 1 1 1
EXAMPLES :0 +~H--+ :F *------+ jN+;He
1
jN+~n------+ 1~N*------+ 1:c+:H (Q = 0.627 MeV) ~~Na+ ~H--+:iMg *------+ ~riNe + ;He
(o =6.9 MeV)
21 Al+ 2H------+ 29Si * ___,,.·2sMg + 4He
~Al+~ n------+:!Al*------+~Mg+iH (a= -1.83 MeV) 13 1 ~'" 12 2

1
(0=6.7 MeV)
followed by ~Mg 10~ :iAl+~ e+Ya
~;Mg+ ~H------+:AI *-----+~Na+ !He
Only the first reaction is induced by thermal
neutrons. (0=1.96 MeV)
(iii) (n, y) reactions: When the reactions yield Since the a-particles ejected from the compound
y-photons . This is the most important neutron- .nucleus are to cross high potential barrier, the
(d, a) reactions occur at fairly high energy of
induced transmutation, known as radiative
deuteron and for low Z target nuclei.
capture of neutrons, and has the following general
equation. (ii) ( d, p) reactions: When the reactions yield protons.
~x +~n------+ A+_ic * ------+A+1Y +r The general equation of ( d, p) reactions is
{C * is Compound Nucleus} AX+2H->A+2C*->A+1X+1H
Z .1 Z+l Z 1

The product nucleus is thus the same as the {C * is Compound Nucleus)


compound nucleus in the ground state. The ( n, 'Y)
The Q-values are usually positive and the
reaction is always exoergic (Q > 0) and can be reactions exoergic. For ~ome light nuclei, however,
induced by almost zero energy neutrons. Q may be negative. Som~. examples of ( ii, p)
reactions are as under.
================================~ 4.41 =
www.puucho.com
Rahul Sardana with www.puucho.com

Advanced JEE Physics . Optics & Modern Physics

EXAMPLES (iv) (d, t) reactions: When the reactions yield tritium.


;u+:H-------+!Be*->:Li+~H (Q=-0.193 MeV)
The general equation of (d, t) reactions is
1
:c + fH-------+ jN *-------+ :c +~H
1 1
(Q = 2.72 MeV)
~X+iH-),~:;c*-------+A-~y +~H

{C * is Compound Nucleus}
(Q =4.74 MeV)
The product nucleus Y is an isotope of the target
~~P+;H-------+:8*-------+fsf+~H (0=5.71 MeV) nu~leus X . The cross-section of such reactions is
1 1
~Ag + ;H-------+ ~Cd *-------+ ~Ag +~H
1 low. Some examples are as under.
(0=4.6 MeV) ~Li +iH-----+ !Be* ---+~Li +!H
The products of (d, p) reactions are usually radioactive ( Q = -0.996 MeV)
(iii) ( d, n) reactions: When the reactions yield
neutrons. The general equatioil. of ( d, n) reaction (Q = 4.59 Me\T)
is NOTE : At higher energies (> 20 MeV) of
:x +iH-------+ :::c * ------+:!~Y + ~n deuterons the (d, 2n)! (d, 2p), (d, 3n) etc.
{C * is Compound Nucleus) reactions in Which more than one particle (two or
more) is emitted from the compound nucleus
With some exceptions, the ( d, n) reactions are become important, e.g.
exoergic and the Q-values are positive. The 1
:;Ag + iH-----+ 11';Cd *-----+ 1:ca + 2 ~n
product nucleus Y is an isotope of the compound
e) Photo disintegration ( y-induced transmutations)
nucleus. Some examp"ies of ( d, n) reactions are
This type of reactions, called photo disintegrations or
given below.
photonuclear reactions, occur when sufficiently high-
EXAMPLES
energy photons enter into a nucleus. The energy ol the
;u+;H-------+ !Be*-------+ !Be+~n (a= 15.024 MeV) incident photon must be greater than the binding
1
!Be+fH-------+ ~8 *-------+ 1~B+~n (Q = 4.36 MeV) energy of a nuclear particle (separation energy) like
neutron, proton, a-particle etc. to produce
1
:c+;H-------+ ;N *-------+ ~N+~n (Q =--0.283 MeV)
1 1

(y, n), (y, p), (y, CI) etc. reactions.


1 1 1
:0 +.:H-------+ :F *-------+ ~F + ~n (a= -1.625 MeV) The photo disintegration of deuteron, discovered by
~c1+;H-------+:Ar "-------+\1:Ar+~n (Q = 6.28 MeV) Chadwick and Go!dhaber, deserves special mention, for
it was from this reaction that they evaluated neutron
When deuterons bombard deuterons both (d, p)
mass.
and ( d, n) reactions may be observed because of
i H +y-----+~H *-------+ ~H +~n ( y, n reaction)
the two alternative decay schemes for the
compound nucleus. Another example of ( y, n) reaction is
~H·+~H-------+ !He•-------+ ~H +~H (a= 4.03 MeV)
!Be+r-----+!Be*-----+!Be+~n
~H +~H-------+ :He•-------+ :He+ ~n (a= 3.26 MeV)
This reaction is used for preparing the source for photo-
The product ~H is tritium; an isotope of hydrogen. neutrons.
Its nucleus Is called triton which is p-active . Other y-induced reactions are:
(Half-life =12.4 yr)
1
~c +y-----+ ~c *-----+ !Be+!He ( y, CI reaction)
The other product ~He is a stable isotope of 1
;B + y-----+ ;n * -------io !Be+ ~H
1
( y, I reaction)
helium. 1 1
The tritium may be bombarded with deuterons to
~B + y-------+ ~B *-----+!Be+ ~H (y, d reaction)
produce (d, n) reaction !Be+r-----+:Be*-------+!Be+~n ( y, n reaction)
~H+~H-------+:He*-------+!He+~n (0=17.6 MeV) !Be+ y-----+ !Be*-----+ ~Li+ !H (y, p reaction)
The reactions with beryllium and deuterium serve
as sources of neutrons. A thick beryllium target if The last ( y, n) reaction serves as a convenient source of
bombarded with 1 MaV deuterons (accelerated in a neutrons.
cyclotron or Van de Graaff generator) yields about
Neutral 1t-mesons can be artificially produced by the
108 neutrons per sec per µA deuteron-current
absorbed in the target.
interaction of y-rays with hydrogen and deuterium.

= ==================================
4.42

www.puucho.com
Rahul Sardana with www.puucho.com

Nuclear Physics

1H +y~rc 0
+!H CONCEPTUAL NOTE(S)
0
iH +y~1t +~n+!H When slow neutrons are bombarded on 92 U235 , the fission takes
place according to reaction
iH+y~1t0 +iH
92 U235 +0 n1 - i ,56 Ba141 +36 Kr 92 + 3( 0 n1 )+200 MeV
The first reaction can occur only if the energy of the
a) In nuclear fission the sum of masses before reaction is
y-photon is not less than the threshold value equal to greater than the sum of masses after reaction, the
the mass-energy of the neutral pion mass. Steinberg difference in mass being released in the form of fission
could produce rt0 -mesons by bombarding light targets energy.
such as hydrogen or beryllium with high energy b} The phenomenon of nuclear fission was discovered by
Otto Hans and F. Strassmann in 1939 and was explained
x-radiation from an electron synchrotron. by N. Bohr and J.A. Wheeler on the basis of liquid drop
r+!Be~!B*~!Be+~n model of nucleus.
{Photon) c) It may be pointed out that it is not necessary that in each
fission of uranium, the two fragments Ba1 ' 1 and Kr 92 are
Illustration 50 formed but they may be any stable isotopes of middle
weight atoms. The most probable division is into two
A proton is bombarded on a stationary lithium fragments containing about 40% and 60% of the original
nucleus. As a result of the collision two a particles ·are nucleus with the emission of 2 or 3 neutrons per fission.
produced. If the direction of motion of the u particles So, average number of neutrons produced per fission is
with the initial direction of motion makes an angle 2.5.
d) Most of energy released appears in the form of kinetic
cos_, ( ¼), find the kinetic energy of the striking proton. energy of fission fragments.
e) The fission of U238 takes place by fast neutrons.
Given binding energies per nucleon of Li7 and He4 are
5.60 and 7.06 MeV respectively. (Assume mass of proton
:;::: mass of neutron). CHAIN REACTION

Solution If on the average more than one of the neutrons produced in


each fission are capable of causing further fission, the
Q value of the reaction is given by
I\umber of fissions taking place at successive stages goes on
Q=(2x4x7.O6-7x5.6) MeV =17.28 MeV increasing at a rapid rate, giving rise to self sustained
sequence of fission known as chain reaction. The chjtin
reaction takes place only if the size of the fissionable material
Li7 a is greater than a certain size called the critical size.
p 0
a-- 0 a
0
There are two types of chain reactions.

A. UNCONTROLLED CHAIN REACTION


In this process the number of fissions in a given interval on
Applying Law of Conservation of Energy for Collision, we the average ·goes on increasing and the system will have the
get explosive tendency. This forms the principle of atom bomb .
K, +Q=2K 0
. . . (1) If a nuclear reaction is uncontrolled then in about 1 µs,
where KP and Ka. are the kinetic energies of proton and a energy of order of 2 x 103 J is released.
particle respectively.
B. CONTROLLED CHAIN REACTION
Applying Law of Conservation of Linear Momentum, we get
(AS IN A NUCLEAR REACTOR)
~2m,K, = 2~2m0 K0 cos 0 ... (2) In this process the nwnber of fissions in a given interval is
maintained constant by absorbing a desired number of
= K, = l6~0 cos 0 = (16K.)(¼)'
2
{-:- m. =4m,} neutrons. This forms the principle of nuclear reactor,
consisting of the following parts:
= ~=~ ... ~ a) Fuel: The fuel is U235 or U233 or Pu 23<J
Solving equations (1) and (3) with Q = 17.28 MeV, we get b) Moderator: A moderator is a suitable material to slow
K, = 17.28 MeV down neutrons produced in the fission. The best choice
as moderators are heavy water (D20) and graphite
NUCLEAR FISSION (C).
The splitting of heavy nucleus into two or more fragments of c) Controller: To maintain the steady rate of fission, the
comparable masses, with an enormous release of energy is neutron absorbing material known as controller is used.
called nuclear fission. The control rods are made of Cadmium or Boron-steel.

i:::=================================== 4.43 =
www.puucho.com
Rahul Sardana with www.puucho.com

Advanced JEE Physics Optics & Modem Physics

d) Coolant: To remove the considerable amoW1t of heat 0.693


produced in the fission process, suitable cooling fluids Also, "- = - - = 0.005 per day
11/2
known, as coolants are used. The usual coolants are
water, carbon-dioxide, air etc. ~ (- dN)=m (0.005)(6x10")rn per day
e) Reactor shield: The intense neutrons and gamma di 210
radiation produced in nuclear reactors are hannful for So, electrical energy produced per day is
human body. To protect the workers from such
radiations, the reactor core is surronnded by concrete
E (0.005)(6x10")m x _ x _,.
8 4 10
J
wall, called the reactor shield. • 210
Since, E=l.2xl07 (given}
CRITICAL MASS ~ m=l0g
If the amount of uranium is too small, then the liberated Activity at the end of 693 days is
neutrons have large scope to escape from the surface and the 23 21
0.005x6xl0 xl0
chain reaction may stop before enough energy is released for
explosion. Therefore, in order for explosion to occur, the
R
= 210 710 d ~(1)"
per ay=." 2
mass uranium has to be greater than some minimum value, where, n is the number of half lives
called the critical mass. 693
~ n=--=5
138.6
REPRODUCTION FACTOR
1021
~ R, =R(2) =32x-=4.57x10 21 per day
5
It is the ratio of the rate of neutron production and the rate at
7
which the neutrons disappear. ·
Whether a mass of active material will sustain a chain
reaction or not is determined by the reproduction factor (K). Illustration 52
If K ;, 1 , the chain reaction will be sustained. If K -1 , the In a nuclear reactor, fission is produced in 1 g for
mass is said to be critical. U"' (235.0439 u) in 24 hours by a slow neutron (1.0087 u).
Assume that 35 Kr" (91.8973 u) and 56 Ba141 (140.9139 amu)
Illustration 51 are produced in all reactions and no energy is lost.
· Polonium {~Po) emits !He particles and is (a) Write the complete reaction
(b) Calculate the total energy produced in kilowatt hour.
converted into lead {~Pb). This reaction is used for
Given 1 u=931 MeV.
producing electric power in a space mission. Po 210 has half Solution
life of 138.6 days. Assuming an efficiency of 10% for the
The nuclear fission reaction is
thermoelectric machine, how much 210 Po is required to
92 U235 + on1 ~ 56Ba141 + 36Kr92 + 3 on1
produce 1.2 x 107 J of electric energy per day at the end of
693 days. Also find the initial activity of the material. Mass defect ,:'.rn = [ ( m, + rn,) -( m,, + rnK, + 3m, )]
Given: Masses of nuclei
0
mn = 256.0526 - 235.8373 = 0.2153 u
" Po= 209.98264 amu, ""Pb= 205.97440 amu, Energy released, a = 0.2153 x 931 = 200 MeV
~ He = 4.00260 amu .
6 02 10
lamu=931MeV and Number of atoms in 1 g · x " 2.56 x 1021
235
Avogadro number =6 x 1023 moi-1 Energy released in fission of 1 g of U 215 is
Solution
Q = 200 x 2.56 x 1021 = 5.12 x 10" MeV
Since, 2~Po-------+::Pb+iHe
~ Q = (5.12 X 10 23 ) X (1.6 X 10-13 ) = 8.2 X 1010 J
~ £\rn = 0.00564 amu
Energy liberated per reaction = (£\rn)931 MeV = 8.4x 10-13 J Q = 8•2 X lQto kWh= 2.28 X 10 4 kWh
3.6x106
Electrical energy produced = 8.4 x 10- 14
J
210
Let m g of Po is required to produce the desired energy, NUCLEAR FUSION
then The phenomenon of combination of two or more light nuclei
to form a heavy nucleus with release of enormous amount of
N=.!:!....x6x10" energy is called the nuclear fusion. The sum of masses before
210
fusion must be greater than the sum of masses after fusion,

= 4.44 ================================:::i
www.puucho.com
Rahul Sardana with www.puucho.com

Nuclear Physics
the difference in mass appearing as fusion energy. The He3 nuclei.
2
fusion of two deuterium nuclei into helium is expressed as
Net result is
1 H 2 + 1H 2 ----.+ 2 He" +23-8 MeV
,H' + 1H1 +,H1 + 1H1 - > 2He4 +2 1~ 0 +
It may be pointed out that this fusion reaction does not
2v+2y+Energy (24-6 MeV)
actually occur. Due to huge quantity of energy release, the
helium nucleus 2 He4 has got such a large value of excitation (ii) Carbon-Nitrogen cycle
For the main sequence stars with extremely high
energy that it breaks up by the emission of a proton or a temperatures, Bethe suggested an alternative to
neutron as soon as it is formed, giving rise to the following proton-proton cycle called the Carbon-Nitrogen
reactions. cycle. The cycle is

1 H 2 + 1H 2 - > 2 He3 + 0 n1 +Q(=3-26 MeV) H + 6 C12 ~ 1 N13 • + r + 1.95 MeV


1

1 H 2 + 1H 2 - > 1H 3 + 1H 1 +Q(=4·04 MeV) ,N13" - ) . 6 C13 + ,~0 + v(neutrino) + 2.22 MeV


13
The fusion process occurs at extremely high temperature and 8 C +,H1-> 7 N14 +r+ 7.54 MeV
high pressure just as it takes place at sun where temperature 14 150
7N + ,H1-> 8 0 + y + 7.35 MeV
is 107 K. So, fusion reactions are also called Thermo-nuclear
reactions. 80
150
- > 7N15
+tP'°' +v +2.7 MeV
Nuclear fusion has the possibility of being a much better ,N + 15 1
1H - } ,
12
sC + 2He4 + 4.96 MeV
source of energy than fission due to the following reasons.
Net result is
a) In fusion there is no radiation hazard as no radioactive
material is used. ,H +,H1 + 1H1 +,H1-> 2He4 +2,f3° +
b) The fuel needed for fission (U-235 etc.) is not available 2v + 3y Energy (26 · 7 MeV)
easily whereas hydrogen needed for fusion can be For sun, both the cycles occur with equal probability.
Stars with masses between 0.4 to 2.5 solar mass
obtained in huge quantity.
produce energy mainly by C-N cycle rather than P-P
c) The energy released per nucleon is much more in fusion cycle. Stars with masses 0.4 solar mass or lower
than in fission. (which ·constitute the bulk of stellar population in our
galaxy) mainly derive their e~ergy frorJ'! P-P cycle.
However, the very high temperature and pressure required
for fusion cannot be easily created and maintained and as
such it has not been possible as yet to use fusion for power Illustration 53
generation. It is proposed to use the nuclear fusion reaction:
tH1+1H2 =2He"
CONCEPTUAL NOTE(S)
a) For the fusion to take place, the component nuclei must in a nuclear reactor of 200 MW rating. If the energy from
be brought to within a distance of 10·14 m . For this they above reaction is used with a 25% efficiency in the reactor,
must be imparted high energies to overcome the repulsive how many grams of deuterium will be needed per day.
force between nuclei. This is possible when temperature is (The masses of 1 H 2 and ,He' are 2.0141 and 4.0026 u
enormously high. respectively).
b) The principle of hydrogen bomb is also based on nuclear
fusion. To start a fusion bomb very high temperature is Solution
required. This is achieved by incorportating an atom bomb Energy released in the nuclear fusion is
within the nuclear bomb.
c) The source of ene_rgy of sun and other stars is nuclear Q=tJ.mc' =ti.m(931) MeV
fusion (or thermo-nuclear reactions). There are two => Q =(2x2.0141-4.0026),r931 MeV = 23.834 MeV
possible cycles:
(i) Proton-Proton cycle => Q=23.834xlO'eV
In 1938, Hans Bethe suggested that the stellar Since efficiency of reactor is 25 %
energy is produced by thermonuclear reactions in
which protons are combined and transformed into
helium nuclei. This is known as proton-proton cycle
So effective energy used =~ x 23.834 x 106 x 1.6 x 10-19 J
100
and is applicable for relatively low stellar
temperature. The cycle is =9.534x10-13 J
1 H1 + 1H ~ 1H2 + ,~ + v (neutrino)+ 0.42 MeV
1 0 Since the two deuterium nucleus are involved in a fusion
reaction, therefore, energy released per deuterium is
... (1)
2 3
9.534 X 10-13
,H +,H1->2He +r+S.5 MeV ... (2)
2
3 3 4 1 1
2 He + 2 He - > 2 He +,H + 1H + 12.8 MeV ... (3) For 200 MWpowerperday number of deuterium nuclei required
The reactions (1) aQd (2)_ occur _twice to yi~ld two

4.45 r=:::

www.puucho.com
Rahul Sardana with www.puucho.com

Advanced JEE Plzysics Optics & Modern Physics

200xl0 6 x86400 c) Radioactive isotopes are used to locate the position and
3.624xl0 25 extent of cancer.
9.534 xlO-n
2 d) Radioisotopes are used in locating tumours within the
23 brain.
Since 2g of deuterium constitute 6 x 10 nuclei, therefore
e) Radioactive Cr 51 is used to locate the exact position
amount of deuterium required is
where the haemorrhage might have taken place inside
2 X 3.624 X 1025 the body.
120.83 g/ day
6xl023 f) Water contents of the body are measured by using
deuterium and tritium as tracers.
Illustration 54 g) Radio gold is being used for the treatment of leukaemia.
In the fusion reaction ~ H + ~H-)- ~He+ ~11, the
masses of deuteron, helium and neutron expressed in amu B. IN INDUSTRY
are 2.015, 3.017 and 1.009 respectively. If 1 kg of deuterium a) Radioactive Carbon-14 is used to study wear and tear of
undergoes complete fusion, find the amount of total the position of an engine. C-14 is mixed with the ring.
energy released 1 amu = 931.5 Mevc-'. After some time, the engine oil is analysed to detect the
presence of any radiation in it. In case of wear and tear
Solution the radiations are found to be there.
i'.n! =2(2.015)-(3.017 +1.009)=0.004 amu b) Radio Cobalt is used for testing fields and castings by
So, energy released is 6E = (0.004x931.5) MeV =3.726 MeV taking their photographs with y-rays.
3.726
Energy released per deuteron = - - = 1.863 MeV
2 C. AS TRACERS
. k 6.02xl0
26 The radioactive isotope has identical chemical properties as
N umb er ofd euterons m 1 g ===-=- 3.0lxlO" another stable isotope. Therefore, by mixing it With stable
2
isotope we can trace the presence or distribution of the
So, energy released per kg of deuterium fusion is
element in a biological or physical system by detecting the
E = (3.0lxlO" x 1.863) = 5.6x 1026 MeV radiation emitted by radio-isotope of that element. The
radioisotope in such a case is said to be a Tracer.
Thus, a radioactive tracer is a radioisotope which, when
NUCLEAR HOLOCAUST mixed with a chemically similar element or artificially
attached to a biological system, can be traced by radiation
The estimate of after effects of the atomic (or nuclear) detecting devices.
explosion is termed as nuclear holocaust. If a fusion bomb
Following are the few examples of radioisotopes acting as
explodes, then a nuclear holocaust will not only destroy
tracers:
every form of life on earth but will also make this planet
unfit for life for all times. The radioactive waste will hang a) Phosphorous-32 mixed with phosphorous manure has
like a cloud in earth's atmosphere and will absorb snn' s been used to study the process of extracting food from
radiations, thus causing a long nuclear winter. One can soil by various plants.
imagine this only by the mathematical figures quoted, b) Radio carbon is being used for research in
according to which energy liberated by fission of 50 kg of photosynthesis in plants.
U235 is equal to 4xl015 J which is the energy available from · c) The progress and absorption of sodium chloride in the
20,000 tons of Trinitrotoluene ('TNT). body can be studied by feeding the person with radio-
isotope sodium-24 alongwith sodium chloride.
USE OF RADIOISOTOPES d) With the aid of radioisotopes, the rate, place and
sequence of formation of the organic constituents of a
living body can be studied.
A. IN MEDICINE
Radioisotopes are extensively used in medicine:
a) Radio iodine is used to determine the condition of
human thyroid gland. lodine-131 is administered orally
to the patient. After a sufficient time, the activity is
{Solutions on page 4.104) I
measured. From the observations it can be interpreted . 1. Consider two decay reactions . I
I
whether the gland is over-active, normal or under- I
(a) :_u----+ 2~~Pb + 10 protons+ 22 neutrons
2
'
active.
b) Amounts of sodium and potassium in the body is (b) ::U----,, 2:Pb + 8 !He+ 6 electrons
2

measured by using Na-24 and K-42 as tracers . Are both the r~actions posslble?.

.._:_.. 4.46 ================================:::;


www.puucho.com
Rahul Sardana with www.puucho.com

Nuclear Physics
r-- Given : Average binding ;n~y of 2:U = 7.57 MeV-, that ofl /9.~ssuming. the splitting otU215 - ;~cle~~b·;~t;; MeV 200 l
1 j energy, find i
2:Pb = 7.83 MeV and that of !He= 7 MeV per nucleon.
II i (a) the energy liberated in the fission of 1 kg of U"' and
I 2. Find the minimum kinetic energy of an a -particle to cause 1 (b) the mass of coal with calorific value of 30 kJgm-1
14 I' which is equivalent to 1 kg of U235
the reaction N + 4He-+ 170 + 1H. The masses of 14
N, •

4 1
He, H and 17
0 are respectively 14.00307u, 4.00260u,
l
1.00783u and 16.99913u . I ,o. In a nuclear reaction a+ 7 N14 ------+ 8 017 + p when a-!

! 235
I particles of KE 7.7 MeV were bombarded on nitrogen atom ,
protons were ejected with a kinetic energy of 5.5 MeV. ;
'3. In a neutron induced fission of 92 U nucleus, usable ! (a) Find the Q -value of the reaction
235
energy of 185 MeV is released. If a 82 U reactor is
(b) Find the angle ~ between the direction of motion of
continuously operating it at a power level of 100 MW , find proton and a -particle.
the time it takes for 1 kg of uranium to be consumed in this /
Given that atomic mass of 1H1 =1.00814 amu, atomic mass
I
reactor. j
of 7 N14 = 14.00752 amu, Atomic mass of 8 0 11 = 17.00453
I: 4. Find the increase in mass of water when 1 kg of water
3
and atomic mass of 2 He4 =4.00388 amu .
absorbs 4.2 x 10 J of energy to produce a temperature
riseof1K. 11. 8 protons and 8 neutrons are separately at rest. How much 1

2
~Bi decay as per following equation.
energy will be released if we form :0 nucleus?
1
'
5.
2
~Bi-----+ a:,Tl + !He
Given: Mass of :0 atom =15.994915u
1

Mass of neutron =1.008665u ,


The kinetic energy of a particle emitted is 6.802 MeV.
Calculate the kinetic energy of Tl recoil atoms.
------ ____ __,I
Mass of hydrogen atom =1.007825u

6. Neon-23 decays in the following way


~Ne----+ ~Na+ -~e + V ,
Find the minimum and maximum kine1ic energy possessed
by the beta particle ( _~e) . The atomic mas~es of 23 Ne and
"Na are 22.9945u and 22.9898u, respectively.

7. The radionuclide 11
c decays according to
11 11
C------+ 8 + e... + v. The maximum energy of the emitted \
·positrons is 0.961 MeV . I
Given that atomic mass of 11
C=mc:=11.011434u, atomic I
11
mass of 8=m~=11.009305u, and the mass of positron
=mP = 0;0005486u, calculate disintegration energy Q and
compare it with the maximum energy of the emitted positron
given above. (1u-931 MeV).

8. It is proposed to use the nuclear fusion reaction


1
H2 + fi 2 ------+ J-ie4 in a nuclear reactor of 200 MW rating.
If the energy from the above reaction is used with a 25%
efficiency in the reactor, how many gram of deuterium fuel
will be needed per day. Given that the masses of 1H2 and

2
He4 are 2.0141 atomic mass units and 4.0026 atomic
mass unit respectively.

================================== = 4.47

www.puucho.com
Rahul Sardana with www.puucho.com

that the emitted wavelengths are either less than or


Problem 1
greater than or smaller than the wavelength of absorbed
A gas of hydrogen like ions is prepared in such a way photon. Which is possible only in the second case, i.e.,
that the ions are only in the ground state aild .the first
when electron jumps from n =2 fo n = 4 .
excited state. A monochromatic light of wavelength
· 1216 A is absorbed by the ·ions. The ions are lifted t~ Hence, E4 -E, =i0.177 eV
higher excited states and emit radiations of six
=> -13.6z'- -(-13:6z')=l0.l77
wavelengths, some higher, some •ower or some greater 42 22 .
than the incident wavelength.
Solving !his, we get z = 2
(a) Find the principle quantum number of the final
excited state. (c) Maximum wavelength corresponds to rmmmum
(b) Identify the nuclear charge on the ions. ertergy, i.e., a trarisition from n = 4 to .n =3 . Thus,
(c) Calculate the value of the maximum and minimum t.Emln =E,-E,
wavelengths. ( .:13_6)(2) 2 ·[-(13.6)(2)']
Solution => t.Emin (4)' . (3)'
(a) Energy corresponding to ;\. = 1216 A is, t.Emln = 2.64 eV
=>
E = ~ eV=l0.177 eV 12375
1216 => ,. =~= =4687 A
max 8Euun 2.64
Now, total six wavelengths are obtained in the erniSsion
spectrum, hence from Minimum wavelength Corresponds to maximum
n(n-l)
=
ener_gy: i.e., a transition from n 4 to n =1 .
. 2 6 Hence, ~Emax = E4 - Ei
2
We have n = 4, i.e., after exCitation the single electron (-13.6)(2) _ [(-13.6)(2)']
=>
junips t? 3rd excited State or n = 4 . (4)
2
, Ct)'
(b). Now it may jump either from n = l or n =.2 . => t.E...;. =51 eV
-i~-----,m----n=4
--+------"'"4-l-~--n=3 ,. . = 12375 = 242 A
=>
- + - - - - - -~'4--''Lh--n=2 = 51

Problem 2
- - - - - - ___y__n_n=1 A gas of hydrogen like·atoms can absorb radiations of
(a) (b) (c) 68 eV. Consequently, the atoms emit radiations of only
three different wavelengths. AU the wavelengths are equal
If it jumps frOm n =1, then in emission spectrum all the or smaller than that of the absorbed photon.
six photons haVe energy equal to or less than the energy
(a) Find the initial state of the gas atoms.
of absorbed photon or the wavelength of emitted
photon is either equal to O! greater than the wavelength (b) Identify the gas atoms.
of absorbed photon While in the question it is given (c) Calculate the minimum wavelength of the emitted
radiations.
= 4.48

www.puucho.com
Rahul Sardana with www.puucho.com

Atomic & Nuclear Physics


(d) Find the ionization energy and the respective Solution
wavelength for the gas atoms.
1
Solution (a) Since, E" =- ~ eV, so we have
n
(a) s·mce, N n(n-l) 3 -18
2 E =-=-18eV
l (1)'
=> n=3
-18
i.e., after excitation atom jumps to second excited state. E =-=-4.5eV
2 (2)'
Hence n1 =3.So ni canbelor2
-18
E,=--2 =-2eVand
(3)
-18
E, =--, =-1.125 eV
(4)
The energy level diagram is shown in figure.
- - - - - - - - E4 =-1.125 eV
n1 = 1 - - - - - - - - E , =-2 av
If ni =1 -then energy emitted is either equal to, greater - - - - - - - - E2 =-4.5eV
than or less than the energy absorbed. Hence the
emitted wavelength is either equal to, less than or
greater than the absorbed·wavelength. Hence n1 ,;t:. l.
If n; = 2, then E,, ~ Ea and hence Ae :5 Ab - - - - - - - - E , =-18 eV

So, n1 =2 (b) The excitation potential of stage n = 2 is


(b) Since, E3 -E, = 68 eV E,-E, =18-4.5=13.5 V
(c) Energy of the electron accelerated by a potential
=> (13.6)(z'i(¼-¼)=68 difference of 16.2 V is 16.2 eV. Since we observe that

=> Z=6 E, -E1 =-1.125-(-18) = 16.875 eV > 16.2 eV


12375 and E3 -E1 =-2-(-18)=16 eV <16.2 eV
(c) 28.43 A
(13.6)(6)'(1-½) With this energy the electron will be able to excite the
atom from n = 1 to n = 3 , so we have the possible
(d) Ionization energy is wavelengths corresponding to the· transitions from
2 3-->1, 3--;2 and 2-->1. Hence
IE= (13.6)(6) = 489.6 eV
,_ _ 12375 12375
4950
A
A= 12375 =25.3 A 32
-E,-E, -2-(-4.5)
489.6
,_ = 12375 = 12375 = 773 A
31 E3-E1 16
Problem 3
The energy levels of a hypothetical one electron atom 12375 917 A
1 -4.5-(-18)
are given by En =- ~ eV, where n =1, 2, 3, .....
n (d) The energy corresponding to 1. = 2000 A is given by
(a) Calculate the four lowest energy levels and construct
12375
the energy level diagram. E= =6.1875 eV
2000
(b) Find the excitation potential of the stage 11 = 2.
whereas the minimum excitation energy is 13.5 eV
(c) Find the wavelengths (A) which can be emitted (n=l to n=2).
when these atoms in the ground state are bombarded Hence this is not possible.
by electrons that have been accelerated through a
potential difference of 16.2 V . (e) Threshold wavelength for photoemission to take place
from such an atom is,
(d) Assuming these atoms to be in the ground state, will
12375
they absorb radiation having a wavelength of 1. mm. = 18 = 687.5 A
2000A?
(e) Also calculate the photoelectric threshold wavelength
of this atom.
4.49 =
www.puucho.com
Rahul Sardana with www.puucho.com

Adva11ced JEE Physics Optics & Modern.Physics

Problem 4 = · dE=76xl0 eV 3

Two hydrogen like atoms A and B are of different The wavelength of K. line (transition of electron from L
masses and each atom contains equal number of protons shell to K shell) is,
and neutrons. The difference in the energies between the
first Balmer lines emitted by A and B is 5.667 eV. When i. (in A}= 12375
the atom A and B, moving with the same velocity, strike '· ,iE(in eV)
a heavy target they rebound back with the same velocity. i. = 12375 = o.163 A
In the process, atoms B imparts twice inomentum to· the K, 76xlQ3
target than that A imparts. Identify the atoms A and B .
Solution . Problem 6
For ~ certain hypothetical one-electron atoni, the·
5:667=13.6(2!-Z!)(;, -;, )
wavelength (in A) for the spectral lines for transitions
= ~-~=3 .,.w originating at n = p and terminating at 11 = 1 are given by

Applying Law ·of Conservation of Linear Momentum on , = 1500p'


,. , where p=2,3,4 .....
atom A and heavy target, we get p -1
mAu=Mv~ -mAu {a) Find the wavelength of the least energetic and the
most energetic photons in this series.
=> 2mAu=Mv1
(b) Construct an energy level diagram for this el_ement
Similarly for atom B and heavy target, we get showing _the ene_rgies of the lowest three levels.
2m8 u=Mv2 • (c) Calculate the ionization potential of this element.
Given Mv2 =2Mv1 Solution
=> m8 =2mA ... (2) . .,
(a) Smce, l\,=-,--
1500p'
Since, both A and B contain equal number of protons and p -1
neutro~, so we ha'Y·e
mA =2;lA =ZA ... (3)
= 1. = 1500[----½-)
1--
'"s 2Za Zs p'
From these equations we get So, "-max corre:sponds to least energetic photon with
z, =land z, =2 p=2 . , ..
i.e., A is 1 H' and Bis ,He' (both having single electron).
i.= =1500[~)=2000 A
1--
Problem 5 4

Stopping potential of 24 kV, 10 kV, 110 kV and Amin -corresponds to most energetic photon with p -+ oo
115 kV are measured for photoelectrons emitted from a = i.""" = 1500 A
certain element when it is radiated with monochromatic X-
ray. H this element is used as a target in an X-ray tube, (b) i.-, =1500 A
what will be the wavelength of K. line? 12375
= E.c:E1 =
1500
·eV=8.25eV
Solution
Stopping potentials are 24 kV, 10 kV, 110 kV and = E1.=-8.25eV
115kV - - - - - - - E , . = -0.95 av
If the electrons are emitted from conduction band then the - - - - - - - E2 =-2.05 eV
maximum kinetic energy of photoelectrons would be
115xl03 eV.
li they are emitted from next mner .shell maximum kinetic
energy of photoelectrons would be 110 x 103 eV and so on.
- - - ~ ~ - ~ E 1 = -8.25 eV
For photoelectrons of L shell it would be 100xl03 eV and
for K shell it is 24x103 eV. Therefore, difference between Further, A. 2 ....1 = 2000 A
energy of L shell and K shell is, 12375
E,-E, = _eV=6.2 eV
M= E, -E, =(100-24)x103 eV 2000
E, =-2.05 eV
'
= ==================================
4.50

www.puucho.com
Rahul Sardana with www.puucho.com

Atomic & Nuclear Physics

Similarly ). 31 =1?.°o[
1
~¼) = 1687.5 A
~ E=6.63xl0 6 MeV

Problem 8
12375 It is proposed to use nuclear reaction
E,,-E, = eV = 7.3 eV
16875 5-1Po 210 -----; 82 Pb 206 + 2 He 4 to produce 2 kW electric power
E, =-0.95 eV in a generator. The half life of polonium (Po'") is
(c) Ionization energy is 138.6 days . Assuming efficiency of the generator be 10%,
E,~. =8.25 eV calculate
So, Ionisation Potential equals 8.25 V (a) how many grams of (Po 210 ) are required per day at
the end of 1386 days.
(b) initial activity of the material
Problem 7
A radionuclide with half life 1620 sec is produced in Mass of nuclei : Po'" = 209.98264 amu ,
a .reactor at a constant rate 1000 nuclei per second. During Pb"" = 205.97440 amu ,
each decay energy 200 MeV is released. If production of ,He'= 4.00260 amu (1 amu = 931 MeV)
radio nuclides started at t = 0 , calculate
Solution
(a) rate of release of energy at t = 3240 sec. (a) Llm= 0.00564 amu 25.25 MeV =8.4x10-13 J
(b) total energy released upto t = 405 sec.
0 693
Solution Since, ). = · = 0.005 per day
Iv,
(a) Let N be the number of nuclei at time I, then net rate
of increase of nuclei at instant t is, Let m g of Po210 are r~quired per day for the reactor,

dN then
-=o.-).N
di (6.02xl023 )m
n
{where o. = rate of production of nuclei} 210
23
dN) 'N 0.005x6.02xl0 xm per day
1_.:!!'1._=
a-AN
Idi
0
( -dt
- =A
210
0
So, energy produced per day is
~
,.
N=~(1-,-")
.
... (1) E
210
23
0.005x6.02xl0 xm xS.4 x 10_13 J
Rate of decay at this instant
R=1'.N=a(1-,-") ~ E=(l2xl06 )mJ
Hence, rate of release of energy at this time = R (energy Now, 10% of (12xl06 )m equals 2xl03 x24x3600J
released in each decay) ~ m 2xl03 x24x3600
1 144
=cx(l-,-")(200) MeVsec- l.2xl0 6 g
Substituting the values, we get 144
(b) R =AN= (o.oos)( )(6.02x10")
0.693 ~3240) 210
Rate of release of energy = 1000 ( 1- e1: 20 (200)
~ R = 2.064 x 1021 per day
Rate of Release)= l.5 x 10, MeVsec_1
( ofEnergy ·
Now, R=R,Gr /.,. n = 1386 = 10 )
(b) Total number of nuclei decayed upto time I is ext- N \ 138.6
10

Total number of)= o.t~~(l- ,-") R, = (2) R = 2.11 x 10 24 per day


( decayed nuclei 1'.
Hence, total energy released upto this instant is Problem 9
Suppose a nucleus initially at rest undergoes a. decay
E=[ o.t-f(1-,-")]<200) MeV ~c_cording to equation
Substituting the values, we get ~X----+Y+a
At t = 0 , the emitted a. particle enters in a region of space
_ (1- e '-"'·"')]
E = 1000 x 405- 1000 1620
x 200 MeV
0 693 where a uniform magnetic field ii = B01 and electric field
[
1620

==================================4.51 =
www.puucho.com
Rahul Sardana with www.puucho.com

Advanced JEE Physics Optics & Modem Physics

E= E0 i exist. The a particle enters· in the region with (c) When an a -particle is emitted with velocity v, from a
stationary nucleus X, decay product (nucleus Y)
velocity V = v0 J from x = 0 . At time t = ../3 x 107 ,n!J, sec, recoils. Then by Law of Conservation of Linear
qaEo
Momentum, we have
the particle was observed to have speed twice the initial
speed v0 , then find myvy =mavo
(a) the velocity of a particle at time t . v = m.v, =( 4.003 )(lO')
(b) the initial velocity v 0 ofthe a particle ' m, 221.03
(c) the binding energy per nucleon of a particle. v, =1.81xl0 5 ms-1
Given that m(Y) = 221.03r,, m(a) = 4.003u, m(n) = 1.009u, Total energy released during a -decay of nucleus X is
m(p)=l.00Su mass on a particle m. =~x10-" kg charge E = K.E. of nucleus Y + K.E. of a -particle
3
1 2 1 2
on a particle q =3.2x10-19 C and 11t=931 Mevc-2
0
=>- E=2,myvy+zm(lv0
Solution 1 ·66 xl0-" [(221.03)(1.81x10 5 ) 2 +
E
(a) Magnetic force on a particle, (at t = 0) 2xl.6x10-"
F. = q(vx ii)= q.[( v,j}x(B 1)] 0 (4.003)(10')
2
]

=> fm =-qavoBok ~ E=2.11MeV


Force due to electric field (at any time I) Hence, Mass lost during a -decay is !~~ u = 0.0023u
F, =qE =q E,i 0
Mass of nucleus X is
Hence, the particle will move in a c!rcular path in y-z
m, = ( m, +m. +0.0023)u
plane due to magnetic field and at the same time it will
µ,.ove along x -direction. The resultant path is therefore, ~ m, = 225.0353u
a helix with increasing pitch. Mass defect in nucleus
y
X, ti.m=92m, +(225-92)m. -m, =1.898u
v, So, binding energy per nucleon is
BE 1.898x931.5 MeV = 7_86 MeV
A 225
{;)-------+-+e.-Z

Problem 10
A radionuclide consists of two isotopes. One of the
isotopes decays by a. -em1ss1on and other by
Hence., velocity of particle at any time t can be written f3-emission with haH lives T1 = 405 s , T2 = 1620 s,
as,
. respectively. At t = 0, probabilities of getting a and P-
ii=( q~~o 1)1 +v cos8]-v,sin8k
0
particles from the radionuclide are equal. Calculate their
respective probabilities at t = 1620 s . If at t = 0 , total
number of nuclei in the radio nuclide are N O • Calculate
where 0 =rot= Baq,:i. t
m. the time t when total number of nuclei remained
N
(b) Speed of particle at any time t is undecayed becomes equal to - 0 •
2

( m.Et)'
Given, log10 2 = 0.3010, log 10 5.94 = 0.7742 and
V = ~ + v! {·.· sin2 0+cos2 9=1}
x'+4x-2.5=0, x=0.594.
Solution
. Given v=2v0 at t=(J3X107 ) '"u. ,so,weget At t = 0, probabilities of getting a and p particles are
qaEo
same. This implies that initial activity of both is equal, say
2
2
(2v,) ={"3x107 ) +v; R,.
Activity after t = 1620 s is

= 4.52 ==================================
www.puucho.com
Rahul Sardana with www.puucho.com

Atomic & N11c/earPhysics


1620
1 (n+l)
R=R,(2)"'
2
1 =16.
R, ~ -=--
).. 21tr
... (2)
Equating equations (1) and (2), we get
and R2 =R,(½):: =~ (n+21trl)=RZ'(n(n+2;) ... (3)
(n + l)
Total activity R = R, + R, = _2._ R, n'
16 Since, r oc-
Z
Probability of getting a particles is R, =2 and 2
R 9 (n + 1)
=> r=---r,
11
Probability of getting p particles is R, =_ll_
R 9 Substituting in equation (3), we get
Since, R, =R 1 02
I
11 R(ll)'(n)(n+2)
=> Not= N.,_ (n+ 1)
T, T, => (n + l) = (1.09x107 )(11)(21t) x (0.529 x10-10 )(n' + 2n)
=> Not =2
N., 4
Solving this equation, we get
n=24
Let N 0 be the total number of nuclei at t = 0 then,
Problem 12
_N,
Nm - d N _ 4N0
- an "' - - -
5 5 A radioactive element decays by )}--emission. A
detector records n beta particles in 2 seconds and in next
Given, that N 1 + N 2 = No
2 2-seconds it records ~n beta particles. Find mean life
4
' '
=> N,
5 2
(2)"' + 4N,5 (2)'"'
2
= N,
'2
... (1)
correct to nearest whole number. Given log, 121 = 0.6931,
log, 131 =1.0986 .

Let (½)';'° =x, then the equation (1) becomes Solution


Let n0 be the number of radioactive nuclei at time
4
x +4x-2.5=0 I= 0 . Number of nuclei decayed in time I are given by
=> X=0.594 n0 (l - ,-><), which is also equal to the number of beta
particles emitted during the same interval of time. For the
=> (½)1620' = 0.594 given conditio_n.
n = n0 h- e-") ... (1)
Solving, we get
t=l215 s (~+¾n )=n,(1-e-") ... (2)

Problem 11
Dividing equation (2) by (1), we get
1-e4 }.
If the wavelength of the nth line of Lyman series is 1.75 =------::,,:-
equal to the de-Broglie wavelength of electron in initial 1- e
orbit of a hydrogen like element (Z =11). Find the value => l.75-l.75e-" =l-e-4>
of n.
=> 1.75e-n-e--4,. =~ ... (3)
Solution 4
n line of Lyman series means transition from (n + it Let us take e-21.
th =X

state to first state. Then the above equation becomes

2-RZ'(1--1-) ... (1)


x' - l.75x + 0.75 = 0
).. - (n+l)' 2
1.75± ~(1.75) -(4)(0.75)
de-Broglie wavelength in (n + 1t orbit is => X 2

,_ h hr (21t)(hr) 21tr 3
=> x=l and -
mv mvr (n + l)h = (n +1) 4

================================== = 4.53

www.puucho.com
Rahul Sardana with www.puucho.com

Adva11ced JEE Physics Optics & Modern Physics

e-21• =1 or e-n =~ Problem 14


4 A nucleus at rest undergoes a decay emitting an
But e-2l. =1 is not accepted because which means A= 0 . o.-particle of de-Broglie wavelength, ,. = 5.76 x 10-15 m . If
the mass of the daughter nucleus is 223.610 amu and that of
Hence, e-2;.. = ~ the a-particle is 4.002 amu. Determine the total kinetic
4
energy in the final state. Hence obtain the mass of the
=> -21dog.(e) = log, (3)-log, (4)·= log,(3)-21og, (2)
parent nucleus in amu. (1 amu = 931.470 MeVe-')
1 Solution
=> A=log,(2)- 1og,(3)
2 Given mass of a-particle, m = 4.002 amu and mass of
Substituting the given values, we get
dau"ghter nucleus,
,. = 0.6931-½x(l.0986) = 0.14395 s-1 M = 223.610 amu
de-Broglie wavelength of a-particle,
So, mean life
A.= 5.76 X 10-lS ID
1
!w =-i;:=6.947 S
So, momentum of a-particle is
h 6.63x10°"' k _1
Problem 13 p=-i;:= 5.76x10-1' gms
Nuclei of a radioactive element A are being produced => p = 1.151 X 10-l9 kgms-l
at a constant rate a. . The element has a decay constant A,.
By Law of Conservation of Linear Momentum, this should
At time t = 0, there are N 0 nuclei of the element. also be equal to the linear momentum of the daughter
(a) Calculate the number N of nuclei of A at time t . nucleus (in opposite direction).
(b) if a= 2N0A., calculate the number of nuclei of A after Let K1 and K2 be the kinetic energies of a-particle and
one half-life of A and also the limiting value of N as' daughter nucleus. Then total kinetic energy in the final state
t --4- 00, is
Solution p' p'
K=K1 +K2 = - + -
(a) Let at time t, number of radioactive nuclei are N 2m 2M
Net rate of formation of nuclei of A
=> K =i._(~+..!_)
2 m M
dN =a.-A.N
dt => K=p'(M+m)·
2 Mm
~=dt
a.-,N Since, 1 amu = 1.67 x 10-21 kg

f ~=fdt
Noa-AN o
Substituting the values, we get
K =10-12 J
Solving this equation, we get 10-u
K · 6.25MeV
1.6 xl0-13
N =.!(a-(a- ..N 0 )e-") ... (1)
A. K=6.25MeV
Substituting a= z,.N, and t --1/,-
t - log.(Z) in , 6.25
(b) (i) A. Mass d e1ect, 1'.m = - - - = 0.0067 amu
931.470
equation (1), we get,
3
N=-N,
2 ( ;::::J=(
nucleus
m:;
a-particle
J+(d::;~~!rJ+[=::tJ
nucleus (.6.m)
(ii) Substituting a= 2W0 and t-> oo in equation (1), => m,.,..,, =(4.002+223.610+0.0067) amu
we get
a.
~ mparent = 227.62 amu
N=-=2N,
A. Hence, mass of parent nucleus is 227.62 amu.
=> N =2N0

= ====================================
4.54

www.puucho.com
Rahul Sardana with www.puucho.com

Atomic & Nuclear Physics

Problem 15 1,. = 0.693 = 0.693 h-1


A hydrogen-like atom (described by the Bohr model) t,12 15
is observed to emit six wavelengths, orginating from all
=> A.=0.0462h-1
possible transistions between a group of levels. These
levels have energies between -0.85 eV and -0.544 eV Let R,, be the initial activity then
(including both these values). R,, = 1 microcurie = 3_.7 x 10' dps
(a) Find the atomic number of the atom.
Let r be the activity in 1 cm3 of blood at I= 5 hr, then
(b). Calculate the smallest wavelength emitted in· these
transistions. r = 296 d'1smtegration
60 . ' per second
(Take /Jc=1240 eV-mn, ground state energy of hydrogen
atom =-13.6 eV) => r = 4.93 disintegration per second, and
Solution R be the activity of whole blood at time I= 5 hr
(a) Total 6 lines are emitted. Therefore, Total volume of blood should be
n(n-_1) V=!
6 r
2
=> n=4 V = R,,e-"
S0 transition is taldng place between ,n th energy state r .
1

and (m + 3 )"' energy state. Substituting the values, we get


4
E. =-0.85 eV V =(3.7 x10 )e-<o.ot62J(s) cm3
4.93
=> -13.6(:: )=-0.85 => V = 5.95 x 10 3 cm3
=> \1 = 5.95 It.
=> ~=0.25 ... (1)
m
Problem 17
Similarly, E••, =-0.544 eV
The element curium ;!8Cm has a mean life of 1013
z'
=> -13.6 2 -0.544 seconds. Its primary decay modes are spontaneous fission
(m+3) and ex-decay, the former with a probability of 8% and the
z latter with a probability of 92%. Each fission releases
-(-)=0.2 ... (2)
m+3 200 MeV of energy. The masses involved in decay are as
Solving equations (1) and (2) for z and m , we get follows:
m=12 and z=3 :;'Cm= 248.072220 u,
(b) Smallest wavelength corresponds to maximum ;!' Pu = 244.064100 u and : He = 4.002603 u . Calculate the
difference of energies which is obviously (Em+J -Em). power output from a sample of 10 20 <:;m atoms.
=> t.E= =-0.544-(-0.85)=0.306 eV (1 u=931 mevc-1 )
1240 Solution
1,.min =_!!I;__= = 4052.3 nm
t.E= 0.306 The reaction involved.in a -decay is
:!Cm ---+ 2!! Pu +; He
2

Problem 16 Mass defect


A small quantity of solution containing Na" radio ·ll.m = mass of 2;cm - mass of 2
,: Pu - mass of ; He
nuclide (half-life = 15 hour) of activity 1.0 microcurie is => llm = (248.072220-244.064100 -4.002603)u
injected into the blood of a person. A sample of the blood
of volume 1 cm3 taken after 5 hour shows an activity of => llm = 0.00551711
296 disintegrations per minute. Determine the total v~lume Therefore, energy released in a -decay will be
of the bond in the body of the person. Assume that the E. =(0.005517x931) MeV =5.136 MeV
radioactive solution mixes uniformly in the blood of the
person. Similarly, E&sinn = 200 MeV {given)
(1 curie = 3.7 x 1010 disintegrations per second) Mean life is
Solution 1
tmean =1013 s=-
A,
A. is the disintegration constant, then

================================== 4.55 =
www.puucho.com
Rahul Sardana with www.puucho.com

Advanced ]EE Physics Optics & Modern Physics

So, disintegration constant A.= 10-13 s-1 ~2K1m = ~2 (4m) K, sin 0 ... (2)
Rate of decay at the moment when number of nuclei are
Squaring and adding equations (1) and (2), we get
1020
K+K1 =4K, ... (3)
_dN =AN=(10-")(10 20 ) ~ 4K2 -K1 =K=65eV ... (4)
dt
Now, during collision, electron can be excited to any
=> dN =10
-dt 7 d"ISmtegration
· . per. second
higher energy state. Applying Law· of Conservation of
Energy,.we get
Of these disintegrations, 8% are in fission and 92% are in
K =K1 +K, +'1E
a -decay.
-3.4 e V - - - - - - - - - n = 4
Therefore, energy released per second is
-6.04eV n=3
P=~=(0.08x107 x200+0.92xl0 7 x5.136) MeV -13.6 e V - - - - - -.......--1---n = 2
t
P=~=2.074x10' MeV
t
So, power output (in watt) is
-54.4 eV-------4>--+-..,__ n =1
P = energy released per second (Js-1 ) He (Z = 2)
0

~ P=(2.074x10')(1.6xl0-13 ) ~ 65=K1 +K, +'1E ... (5)


So, power output is P = 3.32 x 10-s W 6.E Can have the following values,
'1E1 ={-13.6-(-54.4)} eV=40.8 eV.
Problem 18
Substituting in (5), we get
A neutron of kinetic energy 65 eV collides K, +K, =24.2 eV ... (6)
inelastically with a singly ionized helium atom at rest. It is
scattered at an angle of 90° with respect of its original Solving (4) and (6), we get
direction. K, = 6.36 eV and K, = 17.84 eV
(a) Find the allowed values of the energy of the neutron Similarly, when we put AE = '1E,
and that of the atom after the collision.
(b) If the atom gets de-excited subsequently by emitting ~ '1E={-6.04-(-54.4)} eV
radiation, find the _ frequencies of the emitted ~ '1E=48.36 eV
radiation. Put in equation (5), we get
[Given:Massof He atom =4x(massofneutrons) K1 +K, =16.64 eV ... (7)
Ionization energy of H atom = 13.6 eV ] Solving (4) and (7), we get
Solution K1 =0.312 eV and K, =16.328 eV
(a) Let K, and K2 be the kinetic energies of neutron and Similarly, when we put
helium atom after collision and l!..E be the excitation
'1E=AE3 ={-3.4-(-54.4)} =51 eV
energy.
K, Put in equation (5), we get
K1 +K, =14 eV ... (8)

~ K=65eV,"
4m
LX Now, solving (4) and (8), we get
K, = -1.8 eV and K, = 15.8 eV
But since the kinetic energy cannot have the negative
values, the electron will not jump to third excited state
K, i.e., n=4.
Applying Law of Conservation of Linear Momentum Therefore, the allowed values of K1 (KE of neutron) are
along x-direction, we get 6.36 eV and 0.312 eV and of K, (KE of the atom) are
P;=P1 17.84 eV and 16.328 eV and the electron can jump
upto second excited state only (11 = 3).
~ ./2Km = ~2(4m)K2 cos9 ... (1)
(b) Possible emission lines are only three as shown in
Similarly, Applying Law of Conservation of Linear figure. The corresponding frequencies are
momentum along y-direction, we get
= 4.56

www.puucho.com
Rahul Sardana with www.puucho.com

Atomic & Nuclear Physics


----.-,---n=3 Hence, mv' =-1__ (Ze)(e)
11 2 r 4ne0 r2
... (1)
--------'L-+--.-n=2
Further, it is given that Bohr model is applicable to .this
system also. Hence,
3
Angular momentum in nth orbit is L = nh
-----...L..----'-n=1 21t
(E, -E,) h
v, :::;,mvr=n- ... (2)
21t
h
nh
{-6.04-(-13.6)} xl.6 xl0-19 :::;, V=--
=> v, 2mnr
6.63x10-34
Substituting in (1), we get
=> v1 =1.82xl015 Hz
n2h2 e
E3-E1 r=--'
2
V2=--h- nme Z
Substituting Z = 3 and m = 208m,, we get
{-6.04-(-54.4)} xl.6 x10·19
=> v, n2h2eo
6.63x10·34 r,
624mnee2
=> v2 =11.67xl0 15 Hz and v3 =E,-E, (b) The radius of the first Bohr orbit for the hydrogen atom
h is
{-13.6-(-54.4)} X 1.6xlQ·l9
=> v,
6.63x 10·34
=> v 3 =9.84x10" Hz
Equating this with the radius calculated in part (a), we
Hence, the frequencies of emitted radiations are get
1.82 X 1015 Hz, 11.67 X 1015 Hz and 9.84 X 1015 Hz n' z624
~ n::::i25
Problem 19
(c) Kinetic energy of atom is
A particle ·of charge equal to that of an electron, -e,
1 Ze2
and mass 0.208 times of the mass of the electron (called a K=-mv 2 = - -
mu-meson) moves in a circular orbit around a nucleus of 2 8m:0 r
charge +3e. (Take the mass of the nucleus to be infinite).
Assuming that the Bohr model of the atom is applicable to t e potenti.al energy ·u
an dh IS
Ze'-
=- -
41t£0r
this system.
(a) Derive an expression for the radius of the nth Bohr Ze'
~ Total energy E11 = - - -
orbit. 81tt0r
(b) Find the value of n for which the radius of the orbit Substituting value of r, calculated in part (a), we get
is approximately the same as that of the first Bohr 4
orbit for the hydrogen atom. E =1872( m,e )
n n2 8t~lt2
(c) Find the wavelength of the radiation emitted when
the mu-meson jumps from the third orbit to the first
orbit. But (- m{:) is the ground state energy of hydrogen ·
8e0h
(Rydberg's constant =1.097xl0' m·1 )
atom and hence is equal to -13.6 eV
Solution
If we assume that mass of => E, = -lS,72 (13.6) eV= 254 9 2
~ · eV
nucleus is very much mass of mu- --- -,,
',
n n
meson, then nucleus will be
assumed to be at rest, only mu- '' => E3 -E, =-25459.2(¼-¼)=22630.4 eV
meson is revolving round it.
(a) In nth orbit the necessary
g '
<$J m=208m,
,'
So, the corresponding wavelength, is

centripetal force to the mu- ,, ,.(in A)= 12375


/ '- 22630.4
meson will be provided by
the electrostatic force between
--· ,'

=> ,. =o.546 A
the nucleus and the mu-meson.
4.57 =
www.puucho.com
Rahul Sardana with www.puucho.com

This section contains Single Correct Choice Type Questions. ,Each question has four choices (A), (B), (C) and (D), out of which
ONLY ONE is correct.

1. . A radioactive substan~e is -being pi-o_duced at a constant (C) 126.2 days (D) 1262 days
rate' of 200 nuclei. The decay coilstant of the'. subst~ce is
1 s-1 • Assuming that initially there are no nuclei 6. a-particle emitted during various radioactive processes
present, the time (in second) after which the number of have same
nuclei will become 100 is (A) speed (B) momentum
· (A) 1 s (B) . log,(2) s
(C) kinetic energy (D) specific charge
1 7. In the beta decay, the inass number and atomic number
(C) - - - s (D) 2 s
log.{2) of the daughter nuclei respectively
(A) increase by 2
2. Radioactivity is (B) decrease by 1
(A) spontaneous process (C) do not change at all
(B) irreversible process (D) mass number remains· same but atomic number
(C) self disintegration proc~ss n:tcreases by 1
(D) all the above
8. Half life of a radioactive substance A is two times the
3. There are two radio nuclei A and B out of which A is half life of another radioactive substance B . Initially the
an alpha emitter and B is a beta emitter. Their number of nuclei of A and B are N, and N,
disintegration constants are in the ratio of 1: 2 . The respectively. After three half lives of A number of
ratio of number of atoms of A and B at any time t so
that probabilities of getting alpha and beta particles are nuclei of both are eq~~- Then the ratio N" is
. N,
same at that instant is
(A) 1 1
-(A) 1:2 (B) · 1:e (B)
3 6
(C) 2: 1 (D) e: 1
1 (D) 1
(C)
4. The magnetic field at the centre (at nucleus) of the 4 8
hydrogen like atoms (atomic number = z) due to ·the
motion of electron in nth orbit is proportional to 9. The angular momentum of an electron in an·. orbit is
n3 , n4 quantized because it is a necessarj., condition for the
(A) -;_,· (B) z compatibility with
(A) the wave nature of electron.
z' z' (B) particle nature of electron.
(C) n' (DJ ,,,
(C) Pauli's exclusion behaviour.
(D) None of these.
5. The half life of radon is 3.8 days. After how many days
will _!_ th of the radon sample remain behind 10. In a sample of a radioactive substance, the fraction of
10 the initial number of nuclei will remain undecayed after
(A) 1.262 days (B) 12.62 days

= ==================================
4.58

www.puucho.com
Rahul Sardana with www.puucho.com

Atomic & Nuclear Physics


T 16. The total energy E of a sub-atomic particle of rest mass
a time t =-, where T is the half life of radi9active
2 m0 moving at no·n-relativistic speed v is
substance is 1
1 1 {A) E =m0c2 (B) 'E=-mov2
{A) .fi. (B) 2
4
(C) 1 v2
E = m0c + 2m 2'
(D) E·=m0 c' - 1 m0 v '
1 0
2
(C) 2.fi. (D) 1
.fi.-1
17. An electron is lying initially in the n = 4 excited state.
11. In a characteristic X-ray spectra of some atom The electron de-excites itself to go to n =1 state directly
superimposed on continuous X-ray spectra emitting a photon of frequency v 41 • If the same electron
first de-excites to n = 3 state by emitting a photon of
frequency v 43 and then goes from n =3 to n =1 state
by emitting a photon of frequency v 31 , then
W ~=~+¾ 00 ~=~-¾
{C) v43 ·=v 41 +2v31 (D) Data Insufficient

18. The transition from the state n = 4 to n = 3 in a


(A) P represents K. line hydrogen like atom results in ultraviolet radiation.
(B) Q represents K, line Infrared radiation will be obtained in the transition
{C) Q and P represent K. and K, lines respectively w 2~1 00 3~2
(C) 4~2 (D) 5~4
(D) Position of K. and K0 depend on the particular
atom 19. When an electron has a transition from the state ( n + 1)
to state n where n is quite large, then the frequency of
12. In X-ray tube when the accelerating voltage V is
the emitted radiation ( v) will vary with n as
halved, the difference between the wavelengths of K0
1 1
line and minimum wavelength of continuous X-ray {A) voc- (B) voc-
n3 n2
spectrum
(A) remains constant 1
(C) voc- (D) vocn
(B) becomes more than two times n
(C) becomes half
(D) becomes less than two times 20. The spin angular momentum of an electron is equal to
(A) h (B) _!i_
13. The minimum kinetic energy of an electron, hydrogen n .fi.n
ion, helium ion required for ionization of a hydrogen (C) h (D) 2h
atom is E1 in .case el~ctron is collided with hydrogen 2n .fi.n
atom. It is E, when hydrogen i?n is collided and E3
when helium ion is collided. Then 21. In Hydrogen spectrum the wavelength of H 0 line is
(A) E1 =E,=E, (B) E,<E,<E, 656 nm , whereas in the spectrum of a distant galaxy,
H. line wavelength is 706 nm . Estimated speed of
g'!laxy with respect to earth is
14. The half life of ·,,, At is 100 µs . The time taken for the (A) 2xl0 8 ms-1 (B) 2xl07 ms-1
215 1 (C) 2xl06 ms-1 (D) 2xl05 ms-1
radioactivity of a sample of At to decay to -th of
16
its _initial value is 22. If R be the Rydberg's constant, the energy of an
(A) 400 µs (B) 6.3 µs electron in the ground state of the H atom is given by
(C) 40 µs (D) 300 µs (A) Rh (B) 1
c Rhc
15. The rate of decay of a radioactive element (C) -Rhc (D) he
(A) increases with increase in time R
(B) decreases with increase in time
(q remains constant with increase in time 23. The activity of a radioactive substance i~ R1 at time t1
(D) decreases exponentially with time and R2 at time 12 {> ~1 ). Its decay constant is 1,.. Then

================================:::] = 4.59

www.puucho.com
Rahul Sardana with www.puucho.com

Advanced JEE Physics Optics & Modern Physics

(A) R1t1 =Rit2 (B) R, = R, ,'(•,-•,l respectively. A light beam containing wavelel)gths of
550 nm , 450 nm and 350 run with equal intensities
R - ,,
(C) - 1 - " 2- = constant (D) R, = R,l(', -•,) illuminates each of the plates. The correct I-V graph
t2 -tl for the experiment is
I
24. In the PROBLEM 23, number of' atoms decayed between
time interval t1 and t2 are
2 (A) (B)
(A) Jog/ \R, -R2 ) (B) R,e-"• -R,e-"'

(C) 1,.(R,-R,) (D) (R, ;R,)


25. Half-life of a radioactive substance A is 4 days. The
probability that a nucleus will decay in two half-lives is
(C) (D)
(A) 1 (B) 3
4 4

(C) 1 (D) 1
2

26. An artificial satellite moving in a circular orbit around 32. Two radioactive materials X1 and X2 have ·decay
the earth has a total (kinetic + potential) energy -E0 • Its constants 101,. and 1,. respectively. If initially they have
potential energy is the same number of nuclei, then the ratio of the number.
(A) -E, (B) 1.5E0
of nuclei of X, to that of X, will be :!. after a time
(C) 2E0 (D) E, e
1 1
(A) (B)
27. The de-Broglie wavelength of electron in ground state 101,. 111,.
of an hydrogen atom is 11 1
(A) 0.53A (B) 1.06A (C) (D)
101,. 91,.
(C) 1.s2A (D) 3.33A
33. A potential difference of 20 kV is applied across an
28. The difference between nth and (n + l)th Bohr's
X-ray tube. The minimum wavelength of X-rays
radius of H atom is equal to it's (n-l)th Bohr's generated is
radius. The value of n is
(A) 0.26A (B) 0.62A
(A) 1 (B) 2
(C) 3 (D) 4 (C) 0.16A (D) o.so A

29. The angular momentum of an electron in the hydrogen 34. White X-rays are called 'white' due to the fact that
3h (A) they are electromagnetic radiations having nature
atoms is - , where, h is the Planck's ·constant. The .
2it same as that of white light.
kinetic energy of this electron is (B) they are produced most abundantly in X-ray tubes.
(A) 1.51 eV (B) 3.4 e'v (C) they have a continuous wavelength range.
(C) 4.35 eV (D) 6.8 eV (D) they can be converted to visible light using coated
screens and photographic plates are affected by
30. A y -ray photon is emitted them just like light.
· (A) after ionization of an atom
(B) due to conversion of a neutron into a protonjn the 35. The half life period of a radioactive element X is same
nucleus as the mean life time of another radioactive element Y .
(C) after de-excitation of a nucleus Initially both of them have the same number of atoms.
(D) due to conversion of a proton into a neutron in the Then,
nucleus (A) X and Y have the same decay rate initially.
(B) X and Y decay at the same rate always.
31. Photoelectric effect experiments are performed using (C) Y will decay at a faster rate than X.
three different metal plates p, q and r having work (D) X will decay at a faster rate than Y.
functions ~, = 2 eV, ~, = 2.5 eV and ~. = 3 eV,

· 4.60 ====================================
www.puucho.com
Rahul Sardana with www.puucho.com

Atomic & Nuclear Physics


36. It has been observed that the most stable nuclei have 45. In a hydrogen atom, the binding energy of the electron
generally the neutron to proton ratio as in the ground state is E, , then the frequency of
(A) 1:2 (B) 2:1
revolution of the electron in the nth orbit is
(C) 1:4 (D) 1:1
(A) E1n (B) .!i_
37. Select the wrong statement. h nh
(A) Radioactivity is a statistical process (C) 2E,n (D) 2E,
(B) Radioactivity is a spontaneous process h nh
(C) Radioactivity is the neutral characteristics of few
elements ~6. Magnetic moment due to the motion of the electron in
(D) Radioactive elements cannot be produced in the n"' energy state of hydrogen atom is proportional to
laboratory (A) n° (B) n
(C) n' (DJ n'
38. Which of the following statements is correct?
(A) Beta rays are same as cathode rays.
47. A radioactive isotope is being produced at a constant
(B) Gamma rays are high energy neutrons.
rate X . Half life of the radioactive substance is Y .
(C) Alpha particles are singly ionised helium atoms.·
After some time the number of radioactive nuclei
(DJ Protons and neutrons have exactly the same mass. become constant. The value of this constant is
•.

39. The approximate value of quantum number n for the (AJ XY (B) X
y
circular orbit of hydrogen 0.0001 mm in diameter is
(A) 1000 (B) 60 XY
(C) log,(2) (DJ (XYJlog, (2)
(C) 10000 (D) 31

40. Which of the following is its own antiparticle? 48. Which one of the following statements is wrong in the
(A) photon (B) electron context of X-rays generated from an X-ray tube?
(C) proton _ (D) it-meson (A) Wavelength of characteristic X-rays decreases
when the atomic number of the target increases
41. In Bohr's Model of hydrogen atom. The ratio between (B) Cut-off wavelength of the continuous X-rays
the period of revolution of an electron in orbit of n = 1 depends on the atomic number of the target
to the
period of revolution of the electron in the orbit (C) Intensity of the characteristic X-rays depends on
n =2 is the electrical power given to the X-ray tube
(A) 1: 2 (B) 2:1 (D) Cut-off wavelength of the continuous X-rays
(C) 1: 4 (D) 1:8 depends on the energy of the electrons in the X-ray
tube
42 K. wavelength emitted by an atom of atomic number
Z =11 is A . Find the atomic number for an atom that 49. Assuming that all laws of thermodynamics can be
applied, to a nucleus, the a. decay of a nucleus may be
emits K 0 radiation with wavelength 4;.
regarded as an
(A) Z=6 (B) Z=4 (A) isothermal process (B) isobarric process
(C) Z=ll (D) Z=44 (C) adiabatic_process (D) isochoric proces~
43. X-rays are produced in an X-ray tube operating at a 50. Consider the fission reaction
given accelerating voltage. The wavelength of the 236u------,,xn7 + y111 +n+n
continuous X-rays has values from " .
i.e. two nuclei of same mass number 117 are found plus
(A) 0 to oo
(B) ,._ to oo where Amin > 0 two neutrons. The binding energy per nucleon of X
and Y is 8.5 MeV whereas of U236 is 7.6 MeV. The total
(C) 0 to Am~ where A- < oo
energy liberated is
(D) Amin to Amax where O< "-mm < Amax < oo (A) 2000MeV (B) 200MeV
(C) 20MeV (D) 2MeV
44. The half-life of the radioactive radon is 3.8 days. The
time, at the end of which l/20th of the radon sample 51. The activity of a radioactive sample is measured as
will remain undecayed, is (given Iog 10 e = 0.4343) 9750 counts (minute)-1 at I= 0 and 975 counts
(A) 3.8 day (B) 16.5 day (minute)-1 at t = 5 minute. The decay constant is nearly
(C) 33 day (D) 76 day (A) 0.922 min-' (B) 0.691 min-1
(C) 0.461 min-' (D) 0.230 min-1

4.61 =
www.puucho.com
Rahul Sardana with www.puucho.com

Advanced JEE Physics Optics & Modern Physics

52. Magic numbers are (q. -(N11.1,-,/ +N11.2,-,,,)


(A) 2, 8, 18, 28, 50, 164... . 1. N 1. ,-<>-,•>,l•
(B) 2, 8, 18, 50, 82, 164... . (D) -N. 1 1 2 2

(C) 2, 8, 20, 28, 50, 82, 126 ... .


(D) 2, 20, 50, 82, 164, 224 ... . 59. If an electron has, orbital angular momentum quantum
number I= 7, then it will have an orbital angular
53. The half life of radium is 1600 years. The fraction of the momentum equal to
sample of radium that would remain after 6400 years is
(A) 7(;n) (B)
1 1 42(;n)
(A) (B)

(C)
2
1
(D)
4
1
(C) J7(;J (D) "56(;n)

8 16
60. The radius of the first orbit of hydrogen is 0.528 A. The
54. A and Bare isotopes.Band Care isobars. ff dA, d, and radius of the second orbit of hydrogen is
de be the densities of nuclei A, B and C respectively (A) 4.752 A (B) 2.112 A
then
(C) o.on
A (D) 0.142 A
(A) dA>d,>dc (B) dA<d,<dc 61. The speed of the electron in the first orbit (ground state)
(C) dA =d, =de (D) dA =d, <de of the hydrogen atom in te~ of velocity of light c is
C C
55. If u denotes 1 atomic mass unit. One atom of an (A) 2 (B) 11
element has mass exactly equal to Au, where A is C C
mass number of element. (C) 137 (D)
274
(A) A=l
(B) A=12 62. A hydrogen atom emits a photon corresponding to an
(C) A=16 electron transitio_n from n =5 to n = 1 . The recoil speed
(D) A can take up any integral value from 1 to 110 of hydrogen atom is almost
(A) 10-< ms-1 (B) 2x10-2 ms-1
56. The ac~vity of a sample of radioactive material is R1 at
(C) 4 ms- 1
(D) 8x102 ms-1
time ! 1 and R2 at time t, ( !2 > !1 ) : H mean life of the
radioactive sample is T , then 63. A beam of electron is used in an YDSE experiment. The
(A) (B) R, - R, =constant slit width is d . When the velocity of electron is
t2 -fl increased, then
(A) no interference is observed
3/
(C) R,=R1 exp ( t -t ) (D) R, = R1 exp(..!L)
Tt, (B) fringe width increases
(C) fringe width decreases
(D) fringe width remains same
57. The electron in a hydrogen atom makes a transition
from an exited state to the ground state. Which of the
following statements is true? 64. Masses of two isobars and ,.Zn 61 are 63.9298 u
29 Cu"

(A) its kinetic energy increases and its potential and and 63.9292 u respectively. It can be concluded from
total energies decrease these data that :
(B) its kinetic energy decreases, potential energy (A) both the isobars are stable
increases and its total energy remains the same (B) Zn" is radioactive, decaying to Cu" through
(C) its kinetic and total energies decrease and its P-decay
potential energy increases
(C) Cu" is radioactive, decaying to Zn 61 through
(D) its kinetic, potential and total energies decrease
y-decay
58. At time t=O, N 1 nuclei of decay constant 1.1 and N 2 (D) Cu" is radioactive, decaying to Zn" through
nuclei of decay constant A. 2 are mixed. The decay rate P-decay
of mixture is
65. The ratio of molecular mass of two radioactive
(A) -N1N 2,-<,-,•>,J•
substances is 3/2 and the ratio of their decay constant
(B) -( ~:),-(',•',)<
is ½. Then the ratio of their initial activity per mole will
be
= 4.62

www.puucho.com
Rahul Sardana with www.puucho.com

Atomic & Nuclear Physics

8 73. A radioactive sample S1 having an activity of 5 µCi


(A) 2 (B) 9
has twice the number of nuclei as another sample S2
(C) 4 (D) 9 which has an activity of 10 µCi . The half lives of S1
3 8
and s2 can be
(A) 20 years and 5 years, respectively
66. The electronic transition in Li 2+ ion that emits radiation
(B) 20 years and 10 years, respectively
of wavelength same as the wavelength of second
(C) 10 years each
Balmer line of H -atom is
(D) 5 years each
W 4~3 ~ 6~3
(C) 12~6 (D) 12~9 74. In the options given below, let E denote the rest mass
energy of a nucleus and n a neutron. The correct
67. The ratio between total acceleration of the ele~tron is option is
singly ionized helium atom and hydrogen atom (both in
ground state) is (A) E(';;u) > E('i,iI) + E(;Y) + 2E(n)
(A) 1 (B) 8 (B) E(';;U)<E('i_iI)+E(;Y)+2E(n)
(C) 4 (D) 16
(C) E(';;U) < E(':Ba) + E(:;Kr) + 2E(n)
68. A radioactive substance X decays into another (D) E(~U) < E(':Ba)+ E(:;Kr) + E(n)
radioactive substance Y. Assuming that initially oniy
X was present, 11._l" and AY be the disintegration 75. The potential difference across the Coolidge tube is
constants of X and Y, N, and N, be the number of 20 kV and 10 mA current flows through the voltage
nuclei of X and Y at any time t , then the number of supply. Only 0.5% of the energy carried by the
nuclei NY will be maximum when electrons striking the target is converted into X-rays.
The power carried by X-ray beam is P .
(A) ~. '-, (B) N, '-, {A) P=0.1 W (B) P=l W
Nx -Ny "-x -Ay N,: -Ny "-x -Ay
(C) P=2 W (D) P=lO W

76. For large principal quantum number n the frequency


69. The ratio of the maximum wavelength of the Lyman of revolution of electron is equal to the frequency of
series in hydrogen spectrum to the maximum transition of the electron between two adjacent orbits.
wavelength in the Paschen series is This frequency f is proportional to
3 (B) 6
(A) 105 15 (A) :, (B) :,
52
1
(C) 7 (D) 1~8 (C) 1
n
(D) -;;o
70. The SI units of decay constant is
77. A metal of atomic number Z is used as a target in a
(A) m-1 {B) mm-1
Coolidge tube. Let v be the frequency of the K. line.
(C) s-1 (D) year
For a number of values of Z and v which plot gives a
straight line.
71. The velocity of an electron in the first orbit of H atom
1
is v . The velocity of an electron in the 2nd orbit of He' (A) v vs Z (B) V VS
is
z
(A) 2v (B) V (C) Jv vs z (D) V VS ..fz
(C) V (D) V
2 4 78. Solar constant of the sun is a=8.106x10 4 Jmin-1m-2
and average sun earth distance is 1.5 x 10s km . The
72. Electrons with de-Broglie wavelength ),. fall on the target yearly loss in the mass of the sun is
in an X-ray tube. The cut-off wavelength of the emitted (A) 13.8 x 1017 kg (B) 1.38 x 1019 kg
X-rays is ·
2 (C) 1.38x1017 kg (D) 13.8x10 20 kg
(A) ,_ _ 2mc),.
0
(B) '-o = 2h·
- h me
79. Which of the following processes ~epresents a y -decay?
(A) •xz+r--+AXz_, +a+b

4.63 =
www.puucho.com
Rahul Sardana with www.puucho.com

Advanced JEE Physics Optics & Modern Physics

(B) AXZ +lnO~A-3XZ-2 +c (A) 4 (B) 5


(q 6 (D) no comments
(q AXz------>AXz+ f
(D) A Xz +e_1 ~AXA-1 + g 87. In the above decay, how many p particles are given out?
(A} 4 (B) 5
80. Binding energy per nucleon vs mass number curve for (C) 6 (D) no comments
nuclei is shown in the figure. W , X , Y and Z are
four nuclei indicated on the curve. The process that 88. An electron jumps from the fourth orbit to the 2nd orbit
would release energy is of the hydrogen atom. If R=lD' m-1 , the frequency of
the emitted radiation will be
·c
y
j 8.0
8,5 __,_____X w (A) 1-x105 Hz
16
(B) l_x1015 Hz
16
~>7.5 --~-----1------
, '
E' ~
~ .5 5.0 '' ''' _2_x10 15 Hz (D) ~x10 15 Hz
w- '' 16 4
~
C '' '''
'6
ffi '' '' 89. The recoil speed of a hydrogen atom after it emits a
0 30 60 90 120 photon in going from n = 5 state to n = 1 state is
Mass number of nuclei
(A) 4.718 ms-1 (B) 7.418 ms-1
(A) Y------>22 (B) W------>X+Z (C) 4.178 ms-' (D) 7.148 ms-1
(C) W------> 2¥ (D) X------> Y + Z
90. A hydrogen atom emits a photon of energy 12.1 eV. Its
81. Packing fraction is given by the relation orbital angular momentum changes by /iL . Then t,.L
A A-M equals
(A) (B)
M-A A (A) 1.05 X 10-;,.& js (B) 2.11 X 10-;,.i js
M M-A
(C) - - (D) (C) 3.16x10-;,.i Js (D) 4.22x10-;,.i Js
·M-A A

91. The Bohr radius of the fifth electron of phosphorus


82. In the following nuclear reaction, Y stands for
atom (atomic number 15) acting as a dopant in silicon
~AI+1 He----7~P+ Y (relative dielectric constant 12) is
(A) ;n (B) ~, e (A) 380.9 pm (B) 390.8 pm
(C) \H (D) '.,e (C) 930.8 pm (D) 830.9 pm

83. If 1 mg of U"' is completely annihilated, the energy 92. A photon collides with a stationary hydrogen atom in
liberated is ground state inelastically. Energy of the colliding
(A) 9xl010 J (B) 9x1019 J photon is 10.2 eV . After a time interval of the order of
macro second another photon collides with same
(q 9x1018 J (D) 9x1017 J
hydrogen atom inelastically with an energy of 15 eV .
84. The fraction of the initial number of active nuclei which
What will be observed by the detector?
remains undecayed after half of a half-life of the (A) 2 photon of energy 10.2 eV
radioactive sample is (B) 2 photon of energy 1.4 eV
1 1 (q One photon of energy 10.2 eV and an electron of
(A) 4 (B) 2./2 energy 1.4 eV
1 (D) One photon of energy 10.2 eV and another photon
(C) .f5. (D) ./2-1 of energy 1.4 eV

131 131
85. The instantaneous concentration N, the initial 93. The half-life of I is 8 days. Given a sample of I at
concentration N 0 , the radioactive constant A can be time t = 0 , we can assert that 1

(A) no nucleus will decay before I= 4 days


written as
(B) N =N,e_,., (B) no nucleus will decay before I =8 days
(A) N =N,e"
(C) all nuclei will decay before I = 16· days
(q N=N,e-' (D) N=N,e' (D) a given nucleus may decay at any time after I= 0

86. How many a. particles are emitted in the decay


1:X-),Yiat9

4.64 ====================================
www.puucho.com
Rahul Sardana with www.puucho.com

Atomic & Nuclear Physics


94. The radius of second orbit of an electron in hydrogen 18
atom is 2.116 A . The de-Broglie wavelength associated (C) SR (D) .!
R
with this electron in this orbit would be
(A) 1.058 A (B) 2.116 A 101. The binding energy of the electron with n --> oo in H
(C) 6.64 A (D) 13.28 A atom is
(A) ZERO (B) infinite
95. During a negative beta decay (C) 13.6 eV (D) 10.2eV
(A) an atomic electron is ejected
(B) an electron which is already present within the 102. Energy levels A, B, C of a certain
nucleus is ejected atom correspond to increasing ~ ' C
(C) a neutron in the nucleus decays emitting an values of energy i.e., B
electron EA< Ea< Ee. If A.1, "-21 A.3 are "'2
(D) a part of the binding energy of the nucleus is the wavelengths of radiations . A
converted into an electron corresponding to the transitions C to B, B to A and C to
A respectively, which of the following statements is
96. Consider the spectral line resulting from the transition correct?
n =2 --> n =1 in the atoms and ions given below. The
(B) 1.., = 1..,1,.,
shortest wavelength is produced by
(A) hydrogen atom
"-1 + A.2
(B) deuterium atom (D) 1..,'=1..,'+1..,'
(C) singly ionized helium
(D) doubly ionized lithium IO~. If elements of quantum number greater than n were
not allowed, the number of possible elements in nature
97. The angular momentum (L) and radius (r) of a would have been
hydrogen atom are related to each other as 1
(A) n(n+l)
(A) Lr= constant (B) Lr'= constant 2
1
1 n ( n+1)(2n+l) .
(C) Lr 4 = constant (D) Lr-, = constant (C)
6
98. The energy _of a photon is equal to the kinetic energy of
104. The radius R of a nucleus changes with the nucleon
a proton. The energy of the photon is E . Let 1..1 be the
number A of nucleus as
de-Broglie wavelength of the proton and 1,.2 be the 2 1
(A) RocA' (B) RocA3
wavelength of the photon. The ratio ~ is proportional (C) RocA0 (D) RocA
1..,
to
105. The net force between two nucleons 1 fm apart is F,_ if
(A) E' (B) E'1'
both are protons, F2 if both are neutro:15, and f 3 if one
(C) E-1 (D) E-'
is a neutron and the other is a proton.
99. The total energy of the electron in the hydrogen atom in (A) F, <F, <F3 .(B) F, <F, <F,
the ground state is -13.6 eV. The kinetic energy of the (C) F1 <F2 =F3 (D) F,=F,<F,
electron is
(A) 0 (B) 13.6 eV 106. The half life of the radioactive substance is 40 days. The
(C) 6.8 eV (D) -13.6 eV substance will disintegrate completely in
(A) 40 days (B) 400 days
100. Imagine .an atom made up of a proton and a (C) 4000 days (D) infinite time
hypothetical particle of double the mass of electron but
having the same charge as the electron. Apply the Bohr 107. A radioactive nuclide is produced at the constant rate of
Atom Model and consider all possible transitions of this n per second (say, by bombarding a target with
hypothetical particle to the first excited level. The neutrons). The expected number N of nuclei in
longest wavelength photon that will be emitted has existence t seconds after the number is N O is given by
wavelength 1,. (given in terms of Rydberg constant R
for the Hydrogen atom) equal to
(A) N =N,e-" (B) N =f: +N0e-"

(A) -2._ (B) ~ (C) N=f:+( N,-f}-" (D) N=f:+( N,+f:}-"


SR SR
(where 1,. is the decay constant of the sample)

================================== 4.65 =
www.puucho.com
Rahul Sardana with www.puucho.com

Advanced JEE Physics Optics & Modern Pl1ysics

108. The absorption coefficient of X-rays for. a given


wavelength is larger for
(A) lithium (B) lead
(C) aluminium (D) copper

109. For X-rays the wavelength ,. (in A) in terms of energy


E (in keV) is given by
(A) ,_ = 6.20 (B) ,_ = 12.40
E E
(D)

117. The energy associated with a thermal neutron is of the


110. As ·per Bohr Model, the minimum energy (in eV) order of
required to remove an electron from the ground state of (A) 10KeV (B) 1 Kev
doubly ionised Li atom ( Z = 3) is (C) 0.1 MeV (D) 0.DlMeV
(A) 1.51 (B) 13.6
(C) 40.8 (D) 122.4 118. The ionisation potential of hydrogen atom is 13.6 volts.
The energy of the atom in n = 2 state will be
111. The K« X-ray emission line of tungsten occurs at (A) -10.2 eV (B) -6.4 eV
A. = 0.021 nm . The energy difference between K and L (C) -4.4 eV (D) -3.4 eV
levels in this atoms is about ,
(A) 0.51 MeV (B) 1.2 MeV 119. A radioactive nuclide can decay simultaneously by two
(C) 59 keV (D) 13.6 eV different processes which have individual decay
constants "-, and A- 2 respectively. The effective decay
112. If the atom· "" Fm"'7 follows the Bohr's model and the
constant of the nuclide is ,. given by
radius of last orbit of 100 Fm 257 is n times the Bohr 1 1 1
radius, then find n (A) "-=~"-1"- 2 (B) -=-+-
"- "-, "-2
(A) 100 · (B) 200
(C) 4 (D) 1
4

113. A nucleus with mass number 220 initially at rest emits


120. Masses of two isobars ::cu
and ::zn are 63.9298 u
an a-particle. If the Q value of the reaction is 5.5 MeV, and 63.9292 u respectively. It can be concluded from
calculate the kinetic energy of the a-particle this data that
(A) 4.4 MeV (B) 5.4 MeV (A) both the isobars are stable.
(C) 5.6 MeV (D) 6.5 MeV (B) 64 Zn is radioactive, decaying to "Cu through
P- decay.
114. The radioactivity ofa sample is R, at time 11 and R2 at (C) 64 Cu is radioactive, decaying to "Zn through
time 12 • If the half life of the sample be T , the number y- decay.
of atoms that have disintegrated in time ( 12 - 11 ) is (D) "Cu is radioactive, decaying to "Zn through
·proportional to P- decay.
(A) R1t1 -R,12 (B) (R,-R;f1
121. A fraction f, of a radioactive sample decays in one half
(C) (R,-R,) (D) (R,-R,)T life, and a fraction A decays in one mean life.
T
(A) f, > /2
115. In the given reaction, the radioactive radiations are (B) f, </2
emitted in the sequence as (C) f, =!2
zXA----+z+t yA~z-1 TA--l->z-t TA-& (D) Data insufficient to arrive at a conclusion
(A) a, p, y (B) p, a,
y
(C) y, a, P (D) a, y, P 122. The half life of 131 I is 8 days. Given a sample of 131
I at
time t = 0 . We can assert that
116. The graph between the instantaneous concentration (A) no nucleus will decay before I= 4 days
( N) of a radioactive element and time (I) is (B) no nucleus will decay before t =8 days

= ==================================
4.66

www.puucho.com
Rahul Sardana with www.puucho.com

Atomic & Nuclear Physics

(C) all nuclei will decay before t = 16 days 130. Let the potential energy of a hydrogen atom in the
(D) a given nucleus may decay at any time after I= 0 ground state be zero. Then its energy in the first excited
state will be·
123. In the process of fission, the binding energy per nucleon (A) 10.2 eV (B) 13.6 eV
(A) increases (C) 23.8 eV (D) 27.2 eV
(B) decreases
(C) remains unchanged
131. In an excited state of hydrogen like atom an electron
(D) increases for mass number A< 56 nuclei but
has a total energy of -3.4 eV . If the kinetic energy of
decreases for mass number A> 56 nuclei
the electron is E and its de Broglie wavelength is '/..,
124. A proton, a deutron and an alpha particle are then
accelerated through potentials of V, 2V and 4V (A) E=6.8eV,'/..-6.6x10-10 m
respectively. Their velocities will bear a ratio (B) E=3.4eV, '/..-6.6x10-10 m
(A) 1:1:1 (B) 1:.Jz:1 (C) E=3.4 eV ,'/..-6.6x10-11 m
(C) .Jz:1:1 (D) 1:1:.Jz (D) E=6.8eV, '/..-6.6x10-11 m

125. Let v 1 be the frequency of the series limit of the Lyman


132. The radioactivity of a substance is measured in terms of
series, v 2 be the frequency of the first line of the Lyman disintegration per second. 3 x 10 10 disintegration per
series, and v 3 be the frequency of the series limit of the second is
Balmer series. (A) 1 eV (B) 1 MeV
(A) v 1 -v 2 =v3 _(B) v 2 -v 1 =v3 (C) 1 rutherford (D) 1 curie
1
(C) v3 = (D) v 1 +v 2 =v3 133. The shortest wavelength of the Brackett series of a
2(v 1 +v,) hydrogen like atom having atomic number z, is the
same as the shortest wavelength of the Balmer series of
126. The wavelength of the first spectral line in the Balmer
hydrogen atom. The value of z is
series of hydrogen atom is 6561 A. The wavelength of (A) 2 (B) 3
the second spectral line in the Balmer series of singly' (C) 4 (D) 6
ionized helium atom is
(A) 1215 A (BJ 1640 A 134. The shortest wavelength in the Lyman series is 912 A.
(C) 2430 A (D) 4687 A Then the longest wavelength in this series is
(A) 1216 A (B) 1824 A
127. 22
Ne nucleus, after absorbing energy, decays into two (C) 2434 A (D) 3648 A
a- particles and an unknown nucleus. The unknown
135. The maximum angular speed of the electron of a
nucleus is
hydrogen atom in a stationary orbit is
(A) Nitrogen (B) Carbon
(A) 6.2 x 10 5 rads-1 (B) 4.1 x 1016 rads-1
(C) Oxygen (D) Boron
(C) 2.4 x 1010 rads-' (D) 9.2 x 10 6 rads-1
128. According to Bohr's theory of hydrogen atom, the
product of the binding energy of the electron in the nth 136. A hydrogen atom is in an excited state of principle
orbit and its radius in the nth orbit quantum number n . It emits a photon of wavelength A
(A) is inversely proportional to n3 when returns to the ground state. The value of n is
(B) is proportional to n2
(C) has a constant value 7.2 eV A
(A) ,/')..R('/..R-1) (B) ~
(D) has a constant value of 10.2 eV A
(D) ,/')..(R-1)
129. If the half life and the mean life of the radioactive
element is denoted by I and T respectively then 137. A freshly prepared radioactive source of half-life 2 h
emits radiation of intensity which is 64 times the
(A) l=T (B) I=.!_ permissible safe level. The minimum time after which it
T
(C) I< T (D) l>T would be possible to work safely with this source is
(A) 6h (B) 12h
(C) 24 h (D) 128 h

c:=================================== = 4.67

www.puucho.com
Rahul Sardana with www.puucho.com

Advanced JEE Physics Optics & Modern Physics

138. Consider <>-particles , P-particles and y-rays each (A) 60 (B) 32


having an energy of 0.5 MeV. In increasing order of (C) 4 (D) 64
penetrating powers, the radiations are
(A) <>, p, y (B) <>, y, p 146. The binding energy of ap electron in the ground state of
He is equal to 24.6 eV. The energy required to remove
(C) p, y, (l (D) y, p, (l
both the electrons is
(A) 24.6 eV (B) 38.2 eV
139. The equation: (C) 49.2 eV (D) 79 eV
4;H+ ~iHe 2+ +2e- +26 MeV
represents 147. The binding energies of nuclei X and Y are E, and
(A) p-decay (B) y-decay E2 respectively. Two atoms of X ftlse to give one atom
(C) fusion (D) fission
of Y and an energy Q is released. Then
140. If N 1 atoms of a radioactive element emit N 2 beta (A) Q>E2 -2E1 (B) Q<2E,-E,
particles per second, then the decay constant of the (C) Q=E,-2E, (D) Q=2E,-E,
element (in s-1 ) is
148. The shortest wavelength of X-rays emitted from an
(A) N, (B) N, X-ray tube depends on·
N, N, (A) the current in the tube
(C) N,log,(2) (D) N,log,(2) (B) the voltage applied to the tube
(C) the nature of the gas in tube
141. The decay constant of a radioactive sample is A.. The (D) the atomic number of the target material
half-life and mean-life of the sample are respectively
149. Beta rays emitted by a radioactive material are
given by
(A) electromagnetic radiations
(A) ! and (log, )2 (B) (log,2} and! (B) the electrons orbiting around the nucleus
A. A. A. A. (C) charged particles emitted by the nucleus
(C) A.(log,2) and i (D) --""-and!
(Iog,2) "-
(D) neutral particles

150. A radioactive nucleus is being produced at a constant


142. Fast neutrons can easily be slowed down by rate a per second. Its decay constant is A, . If N O are
(A) . the use oflead shielding the number of nuclei at time t = 0 , then maximum
(B) passing them through heavy water number of nuclei possible are
(C) elastic collisions with heavy nuclei Cl
(A) "- (B) N +~
(D) applying a strong electric field 0 A.

143. The electric potential between a proton and an electron (C) N, (D)

is given by V =V, log{ f,), where r0 is a constant.


151. The X-ray beam coming from an X-ray tube will be
Assuming Bohr's model to be applicable, write (A) monochromatic
variation of r11 with n , n being the principal quantum (B) having all wavelengths smaller than a certain
number. maximum wavelength
1 (C) having all wavelengths larger than a certain
(A) r, oc n (B) r oc- minimum wavelength
" n (D) having all wavelengths lying between a minimum
1 and a maximum wavelength
(C) ,. oc n' (D) r oc-
" n' 152. A radioactive isotope A decays into another isotope B
which has a half-life equal to half the half-life of A .
144. After 280 days, the activity of a radioactive sample is Both isotopes emit <> -particles during their decay B
6000 dps. The activity reduces to 3000 dps after another decays into a stable nucleus. If a sample consists
140 days. The initial activity of the sample in dps is initially of atoms of A only, then the net activity of the
(A) 6000 (B) 9000 sample initially
(C) 3000 (D) 24000 (A) decreases with time
(B) increases with time
145. If elements with principal quantum number n > 4 were (C) remains constant
not allowed in nature, the number of possible elements (D) any of the above may be true
would be
= 4.68 ==================================
www.puucho.com
Rahul Sardana with www.puucho.com

Atomic & Nuclear Physics


153. The probability of survival of a radioactive nucleus for
(B) L,, =Lu and /Eu/</Eui
one mean life is
I (C) Lu =Lu and/Eu/>/Eu/
(A) 1-~ (B)
e (D) Lu <Lu and IEul<IEul
(C) log, (2) (D) _ log, (2)
1
e e 161. The potential energy associated with an electron in the
orbit
154. The binding energy per nucleon of 1 H 2 and 2
He 4 are (A) increases with the increase in radii of the orbit
I.I eV and 7 MeV respectively. The energy released (B) decreases with the increase in the radii of the orbit
(C) remains the same with the change in the radii of
in the process 1 H 2 + 1H 2 = 2 He4 is
the orbit
(A) 20.8 MeV (B) 16.6 MeV (D) None of the above
(C) 25.2 MeV (D) 23.6 MeV
162. Suppose a radioactive substance disintegrates
155. In a hydrogen atom, the electron is in nth excited state. completely in 10 days. Each day it disintegrates at a rate
It comes down to 'first excited state by emitting ten which is twice the rate of the previous day. The
different wavelengths. The value of n is percentage of the material left to be disintegrated after
(A) 6 (B) 7 passing of 9 days is
(C) 8 (D) 9 (A) 10 (B) 20
(C) 25 (D) 50
156. There are two radioactive substances A and B such
that decay constant of B is twice that of A . Initially 163. Mirror nuclei are those in which the number of protons
both have equal number of nuclei. After n half lives of· in the parent nuclei
A , the rate of disintegration of both are equal. The (A) is less than the daughter nuclei by one
value of n is (B) is more. than the daughter nuclei by two
(A) 16 (B) 4 (C) equal to the neutron number
(C) 2 (D) 1 (D) exceeds the number of neutrons in the daughter
nuclei by unity by one positron ~mission
157. The figure shows the
variation of photocurrent 164. Half life period and mean life time of a radioactive
with anode potential for a element are
photosensitive surface for (A) inversely proportional to each other
three different radiations. (B) directly proportional to each other
Let I,, I, and I, be the (C) equal to each other
(D) not related to each other
intensities and f,, , f, and
J,, be the frequencies for 165. If P, , P1 and P, be the penetrating powers of a , jl and
the c-qrves a, b and c respectively y radiations respectively then
(A) f, = f, and I, ,'1,
(B) f, = J,, and I,= I,
w ~=~=~ ~ ~>~>~
(C) f, = f, and I, = I, (D) f, = J,, and I, = I,
~ ~<~<~ ~ ~=~<~
158. The ratio of the energies of the hydrogen atom in the 166. A radioactive nuclide emits an a, j3 and a y ray in close
first to the second excited state succession, the atomic mass of the end product is
reduced by
(A) 4: 1 (B) 1 : 4
(C) 4:9 (D) 9:4 (A) 8 a.m.u. (B) about 4 a.m.u.
(C) 2 a.m.u. (D) 1 a.m.u.
159. The ratio of the wavelength of first line of Lyman series
to the first line of Balmer series is · 167. A star initially has 10 40 deutrons . It produces energy
2 2
1 H + 1 H -------+ 1 H + p
3
(A) 1 : 4 (B) 5 : 27 via the processes and ·
(C) 27: 20 (D) 20 : 27 1H
2
+ 1H 3 -------+ 2 He4 +n. If the average power radiated
by the star is 1016 W, the deutron supply of the star is
160. A hydrogen atom and a Li... ion are both in the second
excited state. If Ln and Lu are their respective exhausted in a time of the order of
(A) 10' s , .(B) 10' s
electronic angular momenta and EH and Eu their
respective energies then, (C) 10u s (D) 1016 s
(A) LH >Lu andiEHJ>IEul

4.69 =
www.puucho.com
Rahul Sardana with www.puucho.com

AdvaHced JEE Physics Optics & Modern Physics

(The masses of nuclei are: m(H')=2.014u, h


(A) 2h (B)
2
m(p)=l.007u, m(n)=l.008u, m(He')=4.001 u)
h h
(C) (D)
2it 4it
168. The potential difference applied to an X-ray tube is
5 kV and the current through it is 3.2 mA . The
177. An electron in the ground state of hydrogen has an
number of electrons striking the target per second is angular momentum½_, and an electron in the first
(A) 2xl0 16 (B) 5xl06 excited state of lithium has an angular momentum L2 •
(C) lx1017 (D) 4xl015
(A) L, = L, (B) L1 = 4L,
169. The wavelength K. of X-rays produced by the X-ray (C) L, = 2L, (D) L, = 2L,.

tube is 0.76 A . The atomic number of the anode


178. A certain radioactive element disintegrates for an
material of the tube is interval of time equal to its mean life. The fraction of the
(A) 30 (B) 40 original amount that remains undecay~d is
(C) 50 (D) 60 1
(A) e (B)
e
170. In Rutherford experiment the number of a-particles
1
scattered at angle 90° is 25. How many particles are (C) e' (D) """Ji
scattered at an angle 60°
(A) 100 (B) 85
179. If N denotes the concentration of a radioactive
(C) 70 (D) 55
element, then the rate of change of concentration with
171. The kinetic energy of an electron in the first orbit of H time ( t) can be written as
atom is 13.6 eV. The total energy of an electron in the (A) dN a:N dN
(B) --ocN
second orbit of He+ iS dt dt
(A) 13.6 eV (B) 3.4 eV (C) _ dN a: e" dN -N
(D) --oce
dt dt
(C) -13.6 eV (D) -3.4 eV
180. In the sun about 4 billion kg of matter is converted to
172. If the elements with principal quantum number n > 4 energy each second. The power output of the sun in
were not allowed in nature, the possible number of watt is
elements would be (A) 3.6xl026 (B) 0.36 X 1026
(A) 4 (B) 32
(C) 36x10 26 (D) 0.036x10 26
(C) 60 (D) 64
181. Order of magnitude of density of uranium nucleus is
173. The ratio of areas between the electron orbits for the
first excited state to the ground state for the hydrogen ( mp = 1.67 X 10-27 kg)
atom is (A) 1020 kgm-' (B) 10'.' kgm""
(A) 2: 1 (B) 4:1 14
(C) 8:1 (D) 16: 1 (C) 10 kgm"" (D) 1011 kgm-'

174. The wavelength of the first line of the Balmer series in 182. The electron emitted in beta radiation originates from
the hydrogen atom spectrum is (A) inner orbits of atoms
(A) 6563 A (B) 6365 A (B) free electrons existing in nuclei
(C) 6563 m (D) 6563 cm (C) decay of a neutron in a nucleus
(D) photon escaping from the nucleus
175. An electron with kinetic energy E eV collides with a
hydrogen atom in the ground state. The collision is 183. Electrons with energy 80 keV are incident on the
observed to be elastic for tungsten target of an X-ray tube. K shell electrons of
(A) 0<E<oo (B)' 0<E<10.2eV tungsten have -72.5 keV energy. X-rays emitted by the
(C) 0< E <13.6 eV (D) 0< E <3.4 eV tube contain only
(A) a continuous X-ray spectrum (Bremsstrahlung)
176. The angular momentum of the electron in hydrogen with a minimum wavelength of - 0.155 A .
atom in the ground state is (B) · a continuous X-ray spectrum (Bremsstrahlung)
with all wavelengths.

= 4.70 ================================::::i
www.puucho.com
Rahul Sardana with www.puucho.com

Atomic & Nuclear Physics


(CJ the characteristic X-ray spectrum of tungsten.
(D) a continuous X-ray spectrum (Bremsstrahlung) 188. If a star can convert all the He nuclei completely into
with a minimum wavelength of - 0.155 A and the oxygen nuclei. The energy released per oxygen nuclei is
'
characteristic X-ray spectrum of tungsten. [Mass of the helium nucleus is 4.0026 amu and mass of
oxygen nucleus is 15.9994 amu]
184. The intensity of X-rays from a Coolidge tube is plotted (A) 7.6 MeV (B) 56.12 MeV
against wavelength A as shown. The minimum (CJ 10.24 MeV (D) 23.4 MeV
wavelength found is 1., and the wavelength of the K.
189. Consider the following reaction
line is Ak • As the accelerating voltage is increased
1H2 +1H2------+ 2He4. +Q.
I
If m{iH') =2.0141 u; m( 2 He4 ) =4.0024 u.
The energy Q released (in MeV) in this fusion reaction
is
(A) 12 (B) 6
1.
(CJ 24 (D) 48
1., i.;
190. For uranium nucleus how does its mass vary with
(A) (1.,-1.J increases (B) (1.,-1.,) decreases volume?
(CJ Ak increases (D) A" decreases 1
(A) mocV (B) moc-
V
e' (CJ m « .Jv (D) mccV2
185. The quantity - - has a value
2he0 c
2 191. A radioactive element X converts into another stable
(A) _!_ ms-1 (B) -ms
-1

137 137 element Y . If half life of X is 2 hr, initially only X to


1 2 be present and after time t, the ratio of atoms of X and
(CJ (D) Y is found to be 1 : 4 , then time t in hr is
137 137
(A) 2 (B) 4
186. A radioactive sample consists of two distinct species (CJ between 4 and 6 (D) 6
having equal number of atoms initially. The mean
lifetime of one species is t and that of the other is St . 192. A radioactive material of half-life T was produced in a
The decay products in both cases are stable. A plot is nuclear reactor at different instants, the quantity
made of the total number of radioactive nuclei as a produced second time was twice of that produced first
Junction of time. Which of the following figures best ·time. If now tlwir present activities· are A1 and A 2
represents the form of this plot? respectively then their age difference equals
N N
(A) lo;, 2 llog.( ~' JI (B) Tl log.( )I 1:
(A) (B)
(CJ _
log,T2 J1og,(~)1
2A, (D) rJ1og,(~)1
2A,

193. de-Broglie wavelength of an electron in the nth Bohr


N N orbit is An and the angular momentum is In, then
1
(A) In cc"'" (B) 1. oc-
(CJ (D) " I.
1
(CJ "-n OC J~ (D) 1. oc-
n I!
194. When an electron in the hydrogen atom in ground state
187. The largest wavelength in the ultraviolet region of the
absorbs a photon of energy 12.1 eV, its angular
hydrogen spectrum is 122 nm . The smallest
momentum
wavelength in the infrared region of the hydrogen
spectrum (to the nearest integer) is (A) decreases by 2.llxl0-34 Js
(A) 802 nm (B) 823 nm (B) decreases by l.055xl0-34 Js
(CJ 1882 nm (D) 1648 nm (CJ' increases by 2.11 x 10-34 Js

================================== 4.71 -
www.puucho.com
Rahul Sardana with www.puucho.com

Advanced JEE Physics Optics & Modern Physics

(D) increases by 1.~ x 10-34 Js 200. When a hydrogen atom emits a photon during the
. transition n = 5 to n = 1, its recoil speed is
195. A radioactive nucleus A finally transforms into a stable · approximately
nucleus B . Then A and B can be (A) 0.1 mms-1 (B) 3 mms-1
(A) isotones (B) isotopes (C) 4 ms"' (D) 800 ms-'
(C) isob.;,s · (D) None of these . .
3
201. A potential difference of 10 V is applied 'across an
196. Number of nuclei of a radioactive substance at time X-ray ·tube. The ratio of the de-Broglie wavelength of
t = 0 are 1000 and 900 at time t = 2 s : Then number of the incident electrons to the shortest wavelength of
nuclei at time t = 4 s will be X-rays produced is
(A) 700 (B) 790
(C) 800 (D) 810
(; =1.8 x 1014 Ckg·1 for an electron)

() 1 (B) 1
'197. H 92 U238 changes to 85 At 210 by a series
, of a and P A 10 4 100
decays, the number of a and p decays undergone is
(A) 5and7 (B) 7and5 (C) 2~ (D) 1
(C) 7and7 (D)° 7and9 '
202 During a nuclear fusion reaction
198. In hydrogen and hydrogen like atoins the ratio of (A) a heavy· nucleus breaks into two fragments by
di#'erence of energies E2n - E,. and E4,. - £2,. varies with itself
atomic number z and principle quantum number 'n as (B) a light nucleus bombarded by the1111¥ neutrons
z' z' breaks up ·
(A)
--;;, (B)
~- (C) a heavy nucleus bombarded by thermal neutrons
breaks up
(C) -nz (D) (;)°' (D) two light nuclei combine to give a heavier nucleus
and possibly other products

199. Consider the electronic energy level diagram of 203. Four physical quantities are listed in Column I. Their
H -atom. The photons associated with shortest and values ar~'listed in Column II in a raildom order
longest wavelengths would be emitted from the ,atom Column I · Column II
by the.transitions labelled A Thermal energy of air (i) 0.02 eV
---------~~n=oo molecules at room temperature.
~--~------•~A~n=4 ·B Binciing energy of heavy nuclei (ii) 2 eV
-=•D__-+"c-~-----n=3 per nucleon.
' B
'
C X-ray photon energy. (iii) 10 keV
D Photon ener of visible Ii ht. iv 7 MeV
The correct matching of columns I and II is given by
A B . ·C D
---~~-~----n=2 (A) iv iii ii
(B) iii ii iv.
-----------n= 1
(C) ii iii iv
(A) D and C respectively · . I
(D) ii iv iii
(B) C and A respectively ,
(C) C and D respectively 204. Of the various series of the hydrogen spectrum, the one
(D) A and C respectively which lies wholly in the ultra violet region is
(A) Lyman series (B) Balmer series
(C) Passhen series (D) Brackett series

= 4.72 c:===================================
www.puucho.com
Rahul Sardana with www.puucho.com

This section contains Multiple Correct Choice Type Questions. Each question has four choices (A), (B), (C) and (D), out of which
ONE OR MORE is/ are correct.

1. The momentum of a single photon of red light of 5. An electron makes transition inside a hydrogen atom.
frequency 400xl012 Hz moving through free space is The orbital angular momentum of the electron may
(A) ZERO (B) 8.Bxl0-78 kgms-2 change by
h
(C) 1.65x10.. Mevc-1 (D) Data Insufficient (A) h (B)
h h
2. The radius of the orbit ofan electron in a Hydrogen like (C) 2it (D) 4it
atom is 4.5a0 , where a0 is the Bohr radius. Its orbital
31
apgular momentum is ' . It ;s given that h is Planck 6. An electron is excited from a lower energy state to a
· 2it higher energy state in a hydrogen atom. Which of the
constant and R is Rydberg constant. The possible following decrease in the excitation?
wavelength(s), when the atom de-excites, is (are) (A) potential energy (B) angular speed
(A) 9 (B) 9 (C) kinetic energy (D) angular mo~entum
32R 16R
"9 ' 4 7. In Bohr's model of the hydrogen atom, let R, V, T
(C) SR (D) 3R and E represent the radius of the· orbit, speed of the
electron, time period of revolution of electron and the
3. H+, He+ and o++ all having the same kinetic ·energy total energy of the electron respectively. The quantities
proportional to the quantum number n are
pass through a region in which there is a uniform
magnetic field perpendicular to their velocity, The (A) VR (B) RE
masses of H+, He+ and o++ are 1 u, 4 u and 16 u V
(C)
E
(DJ
R
I
respectively.
(A) H+ will be deflected the most.
8. Let mp be the mass of a proton, m,. the mass of ~
(B) o++ will be deflected the most.
neutron, M 1 the mass ~f a ~Ne nucleus and M2 the
(C) He+ and o++ will be deflected equally.
(D) All will be deflected equally, mass of a : Ca nucleus. Then,
(A) M, = 2M1 (B) M, > 2M1
4. Which of the following statement(s) is (are) correct? (C) M 2 <2M1 (D) M1 <10(m0 +mr)
(A) The rest mass of a stable nucleus is less than the
sum of the rest masses of its separated nucleons
9. The potential difference applied to an X-ray tube is
(B) The rest mass of a stable nucleus is greater than the
increased. As a result, in the emitted radiation
sum of the rest masses of its separated nucleons
(C) In.nuclear fission, energy is released by fusing two {A) the intensity increases
nuclei of medium mass (approximately 100 amu) (B) ,the minimum wavelength increases
(D) In nuclear fission, energy is released by {C) the intensity remains unchanged
fragmentation of a very heavy nucleus
(D) the minimum wavelength decreases

4.73 =
www.puucho.com
Rahul Sardana with www.puucho.com

Advanced JEE Physics Optics & Modern Physics

10. The electron in a hydrogen atom makes a 16. Consider an atom whose energy level diagram is shown
transition "1. ---+ n2 where, "1. and n2 are the principal in figure.
quantum numbers of the two states. Assume the Bohr ---------E=O
Model to be valid. The time period of the electron in - - - - - - - - - - E5 =-8eV
initial state is eight times that in the final state. The
possible values of n,_ and n2 are - - - - - - - - - - E , =-13 eV
- - - - - - - - - - E , =-15 eV
(A) n, =4, n, =2 (B) n,_ =8, n, =2
(q n1 = 8, n, = l (D) n, = 6, n, = 3 - - - - - - - - - E , = - 1 9 eV

11. The wavelength of K. X-rays for lead isotopes Pb208 ,


Pb 206 , Pb"" are 1,.1 , 1,.2 and 1,.3 respectively. Then ----------E,=-25eV
(A) ,.,=,.,=,., (B) ,_,>,.,>,., Suppose an atom starts at level 3 then
(A) Shortest wavelength photon the atom can emit is
(q ,., < ,., < ,., (D) ,., = ~,.,,.,
1.24x10-7 m
(B) Longest wavelength photon that it can absorb is
12. A radioactive sample has initial concentration NO of
6.2x10-7 m
nuclei. (q Lowest frequency photon that can ionize the atom
(A) The number of undecayed nuclei present in the is 3.62x10 15 Hz
sample decays exponentially with time. (D) The number of ways of de-excitation of atom to
(B) The activity (R) of the sample at any instant is ground state is 3
directly proportional to the number of undecayed
nuclei present in the sample at that time. 17. A positronium atom consists of a positron and electron
(q The number of decayed nuclei grows linearly with revolving aronnd their common centre of mass. Then
time. compared to hydrogen atom the positronium atom has
(D) The number of decayed nuclei grows (A) ground state energy half of hydrogen atom
exponentially with time. (B) Rydberg constant half of hydrogen atom
(q radius of first orbit of electron double that in case
13. Suppose the potential energy between electron and of hydrogen atom
. 2
proton at a distance r is given by - Ke, . If m be the (D) velocity of electron in first orbit same as in case of
3r hydrogen atom
mass of electron, then application of Bohr's theory to
hydrogen atom in this case shows that 18. The ground state and first excited state energies of
(A) energy in the nth orbit is proportional to n' hydrogen atom are -13.6 eV and -3.4 eV respectively.
(B) energy is proportional to m"' li potential energy in gronnd state is taken to be zero.
(q energy the nth orbit is proportional to n-2 Then
(A) potential energy in the first excited state would be
(D) energy is proportional to m'
20.4 eV
14. The mass number of a nucleus is (B) total energy in the first excited state would be
(A) always less than its atomic number 23.8.eV
(B) always more than its atomic number (q kinetic energy in the first excited state would be
(C) sometimes equal to its atomic number 3.4eV
(D) sometimes more than and sometimes equal to its (D) total energy in the ground state would be 13.6 eV
atomic number
19. When a hydrogen atom is excited from ground state to
15. A muonic hydrogen atom is a hydrogen atom with first excited state then
electron replaced by a muon whose mass is 212 times (A) its kinetic energy increases by 10.2 eV .
the mass of an electron, then (B) its kinetic energy decreases by 10.2 eV .
(A) Bohr radius of the muonic atom is 250 fm (q its potential energy increases by 20.4 eV.
(B) Ground state energy of muonic atom is 2883 eV (D) its angular momentum increases by 1.05 x 10""' Js
. . h,,
groun d state ts
(q angu1ar momentum m
2 20. An electron orbiting in a circular orbit around the
. d . 106h nucleus of an atom
(D) angular momentum m groun state 1s - - (A) has a magnetic dipole moment
"
=4.74

www.puucho.com
Rahul Sardana with www.puucho.com

Atomic & Nuclear Physics


(BJ exerts an electric force on the nucleus equal to that (D) Gap between energy levels will now be doubled
on it by the nucleus
(C) does produces a magnetic induction at the nucleus 26. From the following equations pick out the possible
(D) has a net energy inversely proportional to its nuclear fusion reactions
distance from the nucleus (A) ,c"+,H'---->,C"+4.3MeV
21. In the Bohr model of the hydrogen atom (B) 6 C12 + 1H 1 ----> 7 N 13 +2MeV
(A) the radius of the nth orbit is proportional to n2 (C) ,N"
+ 1 H 1 ----> 80 15 + 7.3 MeV
(B) the total energy of the electron in the nth orbit is (D) 92U235 +on1_____,. s-iXe140 +36Sr94 +ont +on1 +y +
inversely proportional to n 200MeV
(C) the angular momentum of the electron in an orbit
is an integral multiple of ~· 27. An electron in hydrogen atom first jumps from second
7t excited state to ground state and then from first excited
(D) the m_agnitude of the potential energy of the state to ground state. Let the ratio of wavelength,
electron in any orbit is gre::i.ter than its kinetic momentum and energy, of photons emitted in these two
energy casesbe•a, band c respectively,then
\ . 1 9
22. The electron in a hydrogen atom makes a transition (A) c=-;; .(B) a=
from 2nd excited state to the ground state. Then
4
5 5
(A) it's KE. increases and total energy decreases b=- (D) c=-
27 27
(B) both its KE. and total energy increases
(C) frequency of emitted photons may be 4.6 x 1014 Hz 28. A hydrogen like atom of atomic number Z is in an
(D) frequency of emitted photons must be
excited state of quantum number 2n . It can emit a
2.9x1015 Hz
maximum energy photon of 204 eV . It makes a
transition to quantum state n , a photon of energy
23. The energy, the magnitude of linear momentum and
40.8 eV is emitted, then
orbital radius of an· elect,ron in a hydrogen atom
(A) Z=2 (B) Z=4
corresponding to the quantum nwnber n are E, P and
(C) n=l (D) n=2
r respectively. Then according to Bohr's theory of
hydrogen atom 29. Which of the following physical quantities in hydrogen
(A) PEr is proportional to .!. atom are independent of the quantum number n . The
n symbols have their usual meanings.
(B) P.IS proportion
E . al to n
(A) vr (B) :,
(C) Er is constant for all orbits (C) Er (D) v2 r
(D) Pr is proportional to n
30. Select the correct statement(s).
24. If potential energy in hydrogen atom with electron in (A) X-rays travel faster than ultraviolet rays in
ground state is taken to be 13.6 eV, then vacuum.
(A) potential energy in the first excited state would be (B) Balmer series of H-spectrum is found in visible
34eV region.
(B.) total energy in the first excited state would be (C) The characteristic X-rays are produced due to
37.4eV jumping of electrons from higher to lower shell.
(C) kinetic energy in the first excited state would be (D) In photoelectric emission process, the maximum
44.2eV energy of the photoelectrons must increase with
(D) total energy in the ground state would be 27.2 eV increasing intensity of incident light.

25. If electron of the hydrogen atom is replaced by another 31. Total energy of electron in the first orbit of hydrogen
particle of same charge but of double the mass, then atom is equal to the
select the correct option(s). (A) total energy of electron in 2nd orbit of He'
(A) Bohr radius will increase to double value (B) total energy of electron in 3rd orbit of He'
(B) Ionisation energy of the atom will be doubled (C) total energy of electron in 2nd orbit of Li"
(C) Speed of the new particle in a given state will be (D) total energy of electron in 3rd orbit to Li"
one fourth of what electron will possess in the
same orbit

c:===================================== 4.75
' I www.puucho.com
;
Rahul Sardana with www.puucho.com

Advanced JEE Physics Optics & Modem Physics

32. Which of the following transitions in He• ion will give


(C) the kinetic energy is proportional to \
rise to a spectral ]ine having the same wavelength as n
some spectral line in the hydrogen atom? (D) the angular momentum is proportional to n
(A) n=4 to n=2 (B) n=6 to n=2
(C) n=6 to n=3 (D) n=8 to n=4 39. Whenever hydrogen atom emits a photon in the Balmer
series
33. When atomic number Z is doubled in atoms, which of (A) it may emit another photon in the Balmer series.
the following statements are consistent with Bohr's (B) it must emit another photon in the Lyman series.
theory? (C) the second photon, if emitted, will have a
(A) Energy of a state is doubled. wavelength of about 122 nm .
(B) Radius of an orbit is doubled. (D) it may emit a second photon but the wavelength of
(C) Velocity of electrons in an orbit is doubled. this photon cannot be predicted.
(D) Radius of an orbit is halved.

34. Let A,, be the area enclosed by the nth orbit in a


40. The graph between i and stopping potential (V) of
three metals having work function $1 , $, and $3 in an
hydrogen atom. The graph of log,(~:) against log,n
experiment of photoelectric effect is plotted as shown in
(A) will pass through the origin. the figure. Which of the following statement(s) is/ are
(B) will have a slope of four units at all points. correct? [Here 1'. is the wavelength.of the incident ray]
(C) will be a monotonically increasing nonlinear curve. V
(D) will be a straight line.

35. A particular hydrogen like atom has its ground state metal 1 metal 2 metal 3
total energy - 54.4 eV, then
(A) its atomic number is 2 8
(B) it can absorb a photon of 40.8 eV 0.001 0.002 0.004 1/1. nm-'
(C) in its ground state it cannot emit photon
(D) for its ground state its potential energy is (A) Ratio of work functions $1 : $2 : $3 = 1: 2: 4.
-108.8 eV and kinetic energy is +54.4 eV (B) Ratio of work functions $1 : $2 : $3 = 4: 2: 1.

· 36. ff the potential energy of the electron in the first (C) "dirtl
tan 8 IS ec y proportio , where 1·
·na1 to -h e 1 1s
e
allowed orbit in hydrogen atom is E then its
Planck's constant and c is the speed of light.
(A) ionisation potential is _.§_ (D) The violet colour light can eject photoelectrons
2 from metals 2 and 3.
. . E
(B) kinetic energy IS -
2 41. The wavelengths and frequencies of photons in
. E transitions 1, 2 and 3 for hydrogen like atom are }.1 , A. 2 ,
(C) totaI energy IS -
2 A. 3 , v1 , v2 and v 3 respectively. Then
(D) None of these

37. Hydrogen atoms absorb radiations of wavelength 1'.0


and consequently emit radiations of 6 different
wavelengths of which two wavelengths are shorter than 2 3
1'.0 • Choose the correct altemative(s).
(A)
(B)
The final excited state of the atoms is n = 4
The initial state of the atoms may be n = 2 V1V2
·- _.__~:---~

(C) The initial state of the atoms may be n = 3 (A) V3=-- (B) V3=V1+V2
V1 +V2
(D) There are three transitions belonging to Lyman
series (C) A. 3 = ~ (D) A3=A1+A2
A1 +A.2
38. For the electron in the nth allowed orbit is
. . proportion
. al to -1 42. In a Young's double slit experiment, the separation
(A) th!e mear momentum 1s
n between the two slits is d and the wavelength of the
(B) the radius is proportional to n light is 1'. . The intensity of light falling on slit 1 is four

,--, 4.76 ================================::i


www.puucho.com
Rahul Sardana with www.puucho.com

Atomic & Nuclear Physics


times the intensity of light falling on slit 2. Choose the B/A
correct choice(sJ. .
(AJ If d =1-., the screen will contain only one 8
maximum.
6
(BJ If 1-. < d < 21-. , at least one more maximum (besides
the central maximum) will be observed on the 4
screen.
(q If the intensity of light falling on slit 1 is reduced 2
so that it becomes equal to that of slit 2, the
intensities of the observed dark and bright fringes O 100 200 A
will increase
(DJ If the intensity of light falling on slit 2 is increased (AJ Fusion of two nuclei with mass numbers lying in
so that it becomes equal to that of slit 1, the the range of 1 <A< 50 will release energy.
intensities of the observed dark and bright fringes (BJ Fusion of two nuclei with mass numbers lying in
will increase the range of 51 <A< 100 wlll release energy.
(q Fission of a nucleus lying in the mass range of
43. Assume tJ\at the nuclear binding energy per nucleon 100 < A < 200 wlll release energy when broken

(!) versus mass number (AJ is as shown in the


into two equai fragments.
(DJ Fission of a nucleus lying in the mass range of
figure. Use this plot io choose the correct choice(sJ 200 < A < 260 will release energy when broken
given below. into two equal fragments.

================================:::i 4.77 =
www.puucho.com
Rahul Sardana with www.puucho.com

This section contains Reasoning type questions, each having four choices (A), (B), (q and (D) out of which ONLY ONE is
correct. Each question contains STAUlMENT 1 and STATEMENT 2. You have to mark your answer as
Bubble (A) If both statements are TRUE and STATEMENT 2 is the correct explanation of STATEMENT 1.
Bubble (B) If both statements are TRUE but STATEMENT 2 is not the correct explanation of STATEMENT 1.
Bubble.(C) IfSTATEMENT1isTRUEandSTATEMENT2isFALSE. ,
Bubble (D) If STATEMENT 1 is FALSE but STATEMENT 2 is TRUE.

1. Statement-I: 5. Statement-I:
If the accelerating potential in an X-ray tube is In .a hydrogen atom energy of emitted photon
increased, the wavelengths of the characteristic X-rays corresponding to transition from n = 2 to n = 1 is much
do not change. · greater as compared to transition from n -~ oo to n =2 .
Statement-2: Statement-2:
When an electron ·beam strikes the target in an X-ray Wavelength of photon is directly proportional to the
tube, part of the kinetic energy is converted into X-ray energy of emitted photon.
energy.
6. Statement-I:
2. Statement-I: Magnetic moment of an atom is due to both, the orbital
According to classical theory, the proposed path of an motion and spin motion of every electron.
electron in Rutherford· atom model will be parabolic.
Statement-2:
Statement-2: A charged particle produces a magnetic field.
According to electromagnetic theory, an accelerated
particle continuously emits radiation. 7. Statement-I:
Total energy in an orbit is negative in an atom.
3. Statement-I:
Statement-2:
An electron in hydrogen atom passes from n = 4 to
Electron is bounded by electrostatic attraction between
n =1 level. The maximum number of photons that can
electron and nucleus.
be emitted is 6.
Statement-2: B. Statement-I:
No. of photons emitted can never be more than 5. Between any two given energy levels, the number of
absorption transition is always less than number of .
4. Statement-I: emission transition.
The wavelength of first Balmer line of deuterium is Statement-2:
slightly more than that of hydrogen. Absorption transition starts from the lowest energy
Statement-2: level only and may end at any higher level. But
In.the centre of mass of..an atom reference frame both emission. transitions may starts from any higher energy
nucleus and electron are non-stationary. level and end at any energy level below it.

= ==================================
4.78

www.puucho.com
Rahul Sardana with www.puucho.com

Atomic & Nuclear Physics


9. Statement-1: 15. Statement-1:
Total energy of revolving electron in any stationary The ionising power of J3- particle is less compared
orbit is negative.
a- particle but their penetrating power is more.
Statement-2: Statement-2:
Energy is a scalar quantity and it can take positive and The mass of P- particles is less than .the mass of
negative value.
a- particle.
10. Statement-1:
16. Statement-1:
In outermost stationary orbit, energy of electron is least
negative. A certain radioactive substance has a half life period of
30 days. The disintegration constant is 0.0231 day-1 •
Statement-2:
In outermpst orbit, electron is at maximum distance Statement-2:
from nucleus. Decay constant _varies inversely as half life.

11. Statement-1: 17. Statement-1: ,


Speed of electron in the n"' orbit of hydrogen atom is Half life of a certain radio-active element is 100 days,
1 After 200 days fraction left undecayed will be 50%.
VH oc-,
n Statement-2:
Statement-2: --(-1)'1'~
-N , where symbols have usual meaning.
Speed of electron in n"' orbit of hydrogen like atom is N, 2
Z'
VHlike oc- 18. Statement-1:
n
fl- particles emitted from radioactive nuclei has
12. _Statement-1: continuous energy ranging from zero to a certain
Atomic hydrogen gas excites to third excited state. The maximum value.
number of spectral lines in emission spectrum obtained Statement-2:
is 6.
In fl- decay a neutron is converted into a proton, an
Statement-2: electron and an antineutrino. -The total energy emitted
Nu~ber of spectral lines in emission spectrum can in P- decay is shared by P- particle and antineutrino.
never be less than 6.
19. Statement-1:
13. Statement-1: r- photons are emitted during annihilation process of
In a hydrogen atom energy of emitted photon electron and positron.
corresponding to transition from n = 2 to n =1 is
much greater as compared to transition from n ---+ co to Statement-2:
n=2. High energy photons are.emitted due to conversion of
mass into energy.
Statement-2:
Wavelength of photon is directly proportional to the 20. Statement-1:
energy of emitted photon · 25% of the radioactive nucleus remains active after
200 days for an element of half life 100 days.
14. Statement-1:
Statement-2:
The difference between the wavelengths of series limit
of the Lyman series of spectral lines and that (the series
limit) of the Paschen series is equal to the wavelength of
N =N,(½)f where symbols have usual meaning.
a spectral line of the Lyman series (for the hydrogen
atom). 21. Statement-1:
If binding energy per nucleon increases after a nuclear
Statement-2: reaction then the reaction is exothermic.
The wave number of an atomic transition can. be
Statement-2:
calculate~ from the formula V = R( ~ 2 - : 2 ), where V H there is decrease in mass in nuclear reaction then the
reaction is exothermic.
is the wave number, R is the Rydberg's \constant, m
and n' are the quantum numbers of the initial and final
states.

4.79 =
www.puucho.com
Rahul Sardana with www.puucho.com

Advanced JEE Physics Optics & Modern Physics

22. Statement-1:
If the current in the filament of electron gun in a X-ray 26. Statement-1:
tube is increased, the penetration power of X-rays is Radioactivity is independent of the physical and
increased. chemical conditions of substance.
Statement-2: Statement-2:
Increasing current increases the number of electrons Radioactivity is a property of the nucleus.
emitted by the electron gun.
27. Statement-1:
23. Statement-1: The Wave number. corresponding to the transition
It is necessary to keep high vacuum in Coolidge tube to between the atomic levels n = 3 and n = 2 of a
produce X-rays. hydrogen atom i.e. V32 ,.is related to the wave-numbers
Statement-2: v21 and v
31 for a hydrogen atom by the relation
High vacuum is kept in Coolidge tube so that the V31 =V21 +V32·
electron emitting from the filament of the tube may not
lose their energy in colliding with the atom of the gas in Statement-2:
the tube. ,·
The' wave-number V""' =.!. for a transition is given by
24. Statement-1:
the expression
"'·· --¾-),
.!.A.lffll =R(·n12 where R is the
If maximum frequency of Balmer and Paschen series is m
f, and /, respectively, then frequency of first line of
Rydberg constant and m, n are integers representing
Balmer series is (!, - f,) . the initial and final principal quantum numbers.
Statement-2: 28. Statement-1:
_ Difference of energy level between two orbits is When light is passed through a sample of hydrogen
constant for an atom. atoms in ground state, then wavelengths of absorption
lines are same as wavelengths of lines of Lyman series
25. Statement-1: in emission spectrum.
The wavelength of the K. line from an element and
Statement-2:
that of the K, line satisfy the relation i,,. > ).!\,
In ground state hydrogen atom will absorb only those
Statement-2: radiation which will excite to higher energy level.
The energy separation between the K0 levels is smaller
than th~! of the K, levels .

= 4.80 ==================================
www.puucho.com
Rahul Sardana with www.puucho.com

This section contains Linked Comprehension Type Questions or Paragraph based Questions. Each set consists of a Paragraph
followed by questions. Each question has four choices (A), (B), (C) and (D), out of which only one is correct. (For the sake of
competitiveness there may be a few questions that may have more than one correct options)

Comprehension 1 as X-ray or may all be used to eject an M-level electron from


In a mixture of H -He· gas ( He• is singly ionized He the atom. Based on above information, answer the following
questions.
atom), H atoms and He+ ions are excited to their respective
first excited states. Subsequently, H atoms transfer their 4. The minimum potential difference through which
total excitation energy to He+ ions (by collisions). Assume electron may be accelerated from rest to cause the
that the Bohr model of atom is exactly valid. Based on above ejection of K-level electron from the atom is
information, answer the following questions. (A) 1.875xl0 4 V (B) 3x10-1' V
14
(C) 3.33 x 10 V (D) 10 V
1. The quantum number II of the state finally populated
in He+ ions is 5. Energy released when L-level electron moves to fill the
(A) 2 (B) 3 vacancy in the K-level is
(C) 4 (D) 5 (A) 3 X 10-lS J (B) 3x10-1' J
2.
(C) 2.7 X 10-lS j (D) 2.7 x10-1' J
The wavelength of light emitted in the visible region by
He+ ions after collisions with H atoms is
6. Kinetic energy of the electron emitted from the M-level
(A) 6.5x10-7 m (B) 5.6x10-' m is
(C) 4.8xl0-7 m (D) 4x10-7 rn (A) 3 x 10-1• j (B) 3 x 10-17 j
(C) 2.7 X 10-lS j (D) 2.67 X lQ-15 j
3. The ratio of the kinetic energy of the n = 2 electron for
the H atom to that of He+ ion is
Comprehension 3
(A) 1 1
(B) 2 When a particle is restricted to move along x-axis
4 between x = 0 and x =a, where a is of nanometer
(C) 1 (D) 2 dhnension, its energy can take only certain specific values.
The allowed energies of the particle moving in such a
Comprehension 2 restricted region, correspond to the formation of standing
Simplified picture of waves with nodes at its ends x = 0 and x =a. The
electron energy levels in a wavelength of this standing wave is related to tlie linear
certain atom bombarded with ----M
momentum p of the particle according to the de-Broglie
high energy electrons is shown
relation. The energy of the particle of mass m is related to its
in the figure. The impact of -3 x 1O16 J ----L
one of these electrons has '
linear momentum as E = L . Thus, the energy of the
caused the complete removal -3 x 10 ,.r. J - - - - - K 2m
of K-level and is filled by an particle can be denoted by a quantum number n taking
electron from the L-level with a certain amount of energy values 1, 2, 3,.. .. ( 11 =1, called the ground state)
being released during the transition. This energy may appear corresponding to the number of loops in the standing wave.

.::======================================== = 4.81

www.puucho.com
Rahul Sardana with www.puucho.com

Advanced JEE Physics Optics & Modem Physics

Use the model described above to answer the following three 11. Assume that two deuteron nuclei in the core of fusion
questions for a particle moving in the line x = 0 to x = a . reactor at temperature T are moving towards each
[Take h = 6.6 x 10-34 Js and e = 1.6 x 10-19 C ]. other, each with kinetic energy 1.5 kT, when the
separation between them is large enough to neglect
7. The allowed energy for the particle for a particular Coulomb potential energy. Also neglect any interaction
value of n is proportional to from other particles in the core. The minimum
temperature T required for them to reach a separation
(A) .-, (B) a 2
of 4 x 10-15 m is in the range
(C) .-1 (D) a 2
(A) lx109 K<T<2xl0 9 K
(B) 2xl0' K<T<3xl0' K
8. !fthemassoftheparticleis m=lxlO-.,"kg and a=6.6 (C) 3xl09 K<T<4xl09 K
the energy of the particle in its ground state is closest to (D) 4xl09 K<T<5xl0 9 K
(A) 0.8 meV (B) 8 meV
(C) 80meV (D) 800meV 12. Results of calculations for four different designs of a
fusion reactor using D-D reaction are given below.
9. The speed of the particle that can take discrete values is Which of these is most promising based on Lawson
proportional to criterion?
3

(A)· n2 (B) ,,-1 (A) Deuteron density =2xl012 cm", confinement


1 /" time =5x10-3 s
(C) n' (D) n (B) Deuteron density =8x10 14 cm-3, confinement
time =9x10-1 s
Comprehension 4
(C) Deuteron density = 4x1023. cm-3 , confinement
Scientists are working hard to develop nuclear fusion
time =lx10-11 s
i
reactor. Nuclei of heavy hydrogen, H known as deuteron
(D) Deuteron density =lx1024 cm-3 , confinement
and denoted by D can be thought of as a candidate for
time =4x10-12 s
fusion reactor. The D-D reaction is
iH + iH-------+~He+ n+energy. In the core of fusion reactor,
Comprehension 5
a gas of heavy hydrogen is fully ionized into deuteron nuclei
The mass of a nucleus ~X is less than the sum of the
and electrons. This collection of : H nuclei and electrons is
masses of (A-Z) number of neutrons and Z number of
known as plasma. The nucl_ei move randomly in the reactor
protons in the nucleus. The energy equivalent to the
core and occasionally come close enough for nuclear fusion
corresponding mass difference is known as the binding
to take place. Usually, the temperatures in the reactor core
energy of the nucleus. A heavy nucleus of mass M can
are too high and no material wall can be used to confine the
plasma. Special techniques are used which confine the break into two light nuclei of masses m1 and m2 only if
plasma for a time 10 before the particles fly away from the ( 1"1_ + m2 ) < M . Also two light nuclei of masses m3 and m4
core. If n is the density (number/volume) of deuterons, the can rmdergo complete fusion and form a heavy nucle~s of
product nt0 is called Lawson number. In one of the criteria, mass M' only if (1~ +m 4 ) > M'. The masses of some neutral
a reactor is termed successful if Lawson number is greater a toms are Enven m th e ta ble b eow
1
than 5 x 1014 scm" . It may be helpful to use the following : /H 1.007825 u . ;H 2.014102 u
Boltzmann constant k = 8.6 x 10-5 eVIC1 ;
2 ;u 6.015123 u ;u 7.016004 u
-'-=1.44x10-9 eVm. Based on above information, answer
41tE 0 151.919803 u 2
205.974455 u
~Gd :Pb
the following questions.
;H 3.016050 u !He 4.002603 u
10. In the core of nuclear fusion re'actor, the gas becomes
plasma because of :zn 69.925325 u :se 81.916709 u
(A) strong nuclear force acting between the deuterons 2:Bi 208.980388 u 2
~Po 209.982876 u
(B) Coulomb force acting between the deuterons
(C) Coulomb force acting between deuteron-electron (1 u = 932 Mevc-2 )
pairs Based on a~ove information, answer the following questions.
(D) the high temperature maintained inside the reactor
core 13. The correct statement is
(A) The nucleus !Li can emit an alpha particle

= 4.82

www.puucho.com
Rahul Sardana with www.puucho.com

Atomic & Nuclear Physics


(B) The nucleus ~Po can emit a proton a constant. But experimentally, it was observed that the
(C) Deuteron and alpha particle can undergo complete electron kinetic energy has a continuous spectrum.
fusion. Considering a three-body decay process, i.e., n ~ p + e- + V,,
(D) The nuclei ;::zn and ::s,
can undergo complete around 1930, Pauli explained the observed electron energy
fusion spectrum. Assuming the anti-neutrino (Ve) to be massless
and possessing negligible energy and the neutron to be at
14. The kinetic energy (in keV) of the alpha particle, when rest, momentum and energy conservation principles are
the nucleus 2:Po at rest undergoes alpha decay, is applied. From this calculation, the maximum kinetic energy
(A) 5319 (B) 5422 of the electron is 0.8 x 106 eV . The kinetic energy carried by
(C) 5('07 (D) 5818 the proton is only the r€coil energy. Based on above
information, answer the following questions.
Comprehension 6
A dense collection of equal number of electrons and 17. What is the maximum energy of the anti-neutrino?
positive ions is called neutral plasma. Certain solids (A) ZERO
containing fix;ed positive ions surrounded by free electrons (B) Much less than 0.8xl0 6 eV
can be treated as neutral plasma. Let N be the number (C) Nearly 0.8x10 6 eV
density of free electrons, each of mass m . When the (D) Much larger than 0.8x10 6 eV
electrons are subjected to an electric field, they are displa'ced
relatively away from the heavy positive ions. If the electric
18. If the anti-neutrino had a mass of 3 eve-' (where c is
field becomes zero, the electrons begin to oscillate about the
the speed of light) instead of zero mass, what should be
positive ions with a natural angular frequency cop, which is
the range of the kinetic energy, K, of the electron?
called the plasma frequency. To sustain the oscillations, a (A) 0,; K,; 0.8 xl06 eV
time varying electric field needs to be applied that has an (B) 3 eV,;K,;0,8xl06 eV
angular frequency w , where a part of the energy is absorbed
(C) 3eV,;K<0.8xl0 6 eV
and a part of it is reflected. As co approaches cop, all the free
(D) 0,;K<0.8xl0 6 eV
electrons are set to resonance together and all the energy is
reflected. This is the explanation of high reflectivity of
Comprehension 8
metals. Based on above information, answer the following
questions. A radionuclide with half life -T is produced in a reactor
at a constant rate P nuclei per second. During each decay,
15. Taking the electronic charge as e and the permittivity energy E0 is released. H production of radionuclide is
as e0 , use dimensional analy'sis to determine the correct started at t = 0 . Based on above information, answer the
expression for roP following questions.

~ fJJie
(A)
19. The'number of nuclei in the radionuclide at any instant
(B).
0
t is given by
PT ( t1og,2)

~ ~
(C) (D) (A) N = - - 1-e T
2 log,2
0

(B) N=PT(l_,-f)
16. Estimate the wavelength at which plasma reflection will
occur for a metal having the density of electrons (C) N =PTlog,2(1-e n:,,,_)
N ~ 4 x 1027 m-3 • Take t 0 ~ 10-11 and m R:' 10-30 , where
these quantities are in proper SI units (D) N=PT(l-e 2~)

(A) 800 nm (B) 600 nm


(C) 300 nm (D) 200 nm 20. The rate of release of energy as a function of time is

Comprehension 7 (A)
PE ( -""•·')
log.°z 1-e T (B)
(
PE, 1-e
~i T

The f3- decay process, discovered around 1900, is


basically the decay of a neutron (n) . In the laboratory, a (C) PE,Iog,2(1-e ''°;·') (D) PE,(1_,-,',)
proton (p) and an electron (,-) and observed as the decay
products of the neutron. Therefore, considering the decay of 21. Total energy released upto time t is
a neutron as a two body decay process, it was predicted PTE ( Hog,2)
(A) PtE0 - - - ' 1-,-T-
theoretically that the kinetic energy of the electron should be log,2

c:================================= = 4.83

www.puucho.com
Rahul Sardana with www.puucho.com

Advanced JEE Physics Optics & Modenz Physics

llog,2) Comprehension 11
(B) PtE,-PTE, ( 1-e r Nuclear reactions are performed for artificial
transmutation of elements for there are two types of nuclear
(C) PtE0 -PTE0 log,2(1-e "°;·') reactions, exoergic and endoergic. In exoergic reactions
(D) None of these energy is released. In endoergic reactions energy has to
·supplied for the reaction proceed. In exoergic reactions
Comprehension 9 nuclear energy is converted into.kinetic energy. In endoergic
Two radioactive nuclei A and B, both convert into a reactions, energy input is required in the form of kinetic
stable nucleus C . The nucleus A converts into C after energy to be converted into nuclear binding energy. The
emitting two a- particles and three P- particles, the nucleus minimwn energy required for the reaction to take place is
B converts into C after emitting one ex- particl~ and five called threshold energy. Consider the reaction given by
P- particles. A time t = 0 , nuclei of A are 4NO and that of p+ :H------+iH + iH
B are N 0 • The half-life of A for converting to C is Given that the atomic masses are

1 min and that of B is 2 min. Initially number of nuclei of C mUH-atom) =1.007825 amu (u)
are zero. Based on above information, answer the following m(;H-atom) = 3.016049 arnu (u)
questions.
m(;H-atom)=2.014102 amu (u)
22. If atomic numbers and mass numbers of A and B Based on above information, answer the following questions.
are Z1 , 2 2 , A1 and A2 respectively, then
28. Protons are incident on : H at rest. The threshold
(A) 2,-22 =6
energy for the reaction is
(B) A, -A, =4 (A) 5.4MeV (B) lOMeV
(C) Both (A) and (B) are correct (C) 2MeV (D) 8 MeV
(D) Both (A) and (B) are wrong
29. When ; H are incident o;, protons, then threshold
23. The number of nuclei of C, when number of nuclei of energy is
A and B are equal is
(A) l0MeV (B) 8 MeV
(A) 2N, (B) 3N, (C) 16 MeV (D) 20MeV
9N, SN,
(C) (D)
2 2 30. Which of the following statement is correct?
(A) Less energy is required for nuclear reaction if light
24. The rate of disintegrations of · A and B are equal at part is at re~t and heavy particle is incident
time t0 (in minute), then (B) More energy is required for nuclear reaction when
(A) 10 =4 (B) t, =6 heavy particle is incident and light particle is at
rest
(C) t, =8 (D) t0 =2
(C) Threshold energy does not depend on which
particle is at rest
Comprehension 10 (D) Thr.eshold energy does not depend on Q-value of
The electric current in an X-ray tube (from the target to reaction
the filament) operating at 40 kV is 10 mA. Assume that on an
average, 1 % of the total kinetic energy of the electrons Comprehension 12
hitting the target is converted into X-rays. Based on above Consider a different atomic model in which electron
information, answer the following questions. revolves around the nucleus (proton) at a separation r
under the action of force which is different from the
25. The total power emitted as X~rays is electrostatic force of attraction. The potential energy
(A) 40 W (B) 400 W between an electron and the proton due to this force is given
(C) 4 W (D) 0.4 W k
by U = -"""i'" where k is a constant. Based on above
r
26. The heat produced in the target every second is
information, answer the following questions.
(A) 29.6 J (B) 396 J
(C) 3600 J (D) 360 J
31. The radius of nth Bohr's orbit is
27. Minimum wavelength of X-ray produced is nearly it
(A) r=-vkm = (B) 2it
r=-vkm =
(A) 0.3 A (B) 3 A nh nh
(C) 30 A (DJ 300 A
(C)
4it ,,-
r =-vkm (D) r = Bit ./km
nh nh

= 4.84 c:::===============================:::i
www.puucho.com
Rahul Sardana with www.puucho.com

Atomic & Nuclear Physics

32. The velocity in the nth orbit is given by l

nh n'h (A) V .,'. A' (B) ,VocA3


(A) V 81t'm../km (B) V 81t'm../km (C) VocA 3 ' (D) ·VocA
nh'
(C) V (D) V
81t'in../km Comprehension 14
Consider the nuclear fission reaction W ~ X + Y .
33. The kinetic energy of the electron in nth orbit is given Based on the graph given showing binding energy per
by nucleon vs number of nucleons, answer the following.
n4h4 BE
(B) KE 4 2 A
128rt m k
(D) KE z
E,
.-~-..x
-------
E, ____ J___ J__ y
34. The potential energy of the electron in the nth orbit is _____ J' ___ J' __ ~-~
' w
E, I I I I
given by I I I I
I I l I
n4h4 n4h4 I I l I
(A) PE (B) PE I
I
I
I
I
I
I
I
64kn4 nz2 l28k1t4m2 I
I
I
I
I
I
I
I
n2h2 n4h' N, N3 N2 N1
(C) PE (D) PE
128krt4 m2 256k1t4 m2 Mass number

35. The total energy of the electron ii, the nth orbit is given 39. The Q value of the reaction is
by (A) E1N1 -(E,N, +E,N,) (B) , {E,N, +E3 N 3 -E,.N,)
n-1h 4 n4h4
(A) T E = - - 4 2
~ (B) TE (C) E,N, +E,_N1 -E,N, (D) E1N 1 +E,N,-E2 N2
4 2
128rt m k 128rt m k
n4h4 40. If Mw is the mass of W, Mx is mass of X and My js
(C) TE 256krt'm' (D) TE
256krt4 m2 mass of Y nucleus, choose the correct statement.

Comprehension 13
W ~>~>~ 00 ~<~<~
N1 N, N3 N1 N, N,
-All nuclei consist of two type of particles i.e., protons
Mw My Mx Mw My Mx
and neutrons. Nuclear force is the ~trongest force and the (C) - . - < - > - (D) - - > - < -
stability of nucleus is determined by the neutron-proton N, N2 N3 N, N, N,
ratio or mass defect or Binding energy per nucleons or
packing fraction. The shape of nucleus is calc;ulated by 41. The graph representing the relationship between atomic
quadrupole moment. The spin of nucleus depends on even
mass number A and the total binding energy (BE) of
or odd mass number. The volume of nucleus depends on the
the nucleus, for nuclei heavier than Z is
~ass number. The whole mass of atom (nearly ·99%) is
~
centred at the nucleus and the_'magnetic moment of nucleus e> ~
is measured in terms of the nuclear magnetons. Based on w

C
w

C

above information, answer the following questions.


~
C,
(A) '5.5 (B) "'
C
'8 '
C

36. The correct statement(s) about nuclear force is/ are "'s m
'jjj
(A) charge dependent ,g ,g
(B) short ranges forces Atomic mass number Atomic mass number
(C) non conservative force
(DJ spin dependent force
~ ~
37. Binding energy per nucle6n is maximum •
C
w

C
w
(A) for lighter order element Qow mass number)
(B) for heavier order element (high mass number)
(C)
C,
C
'6
C
~ (D)
C,
C
'6 /
(C) for middle order element
m ifi
s
,g
iii
(D) equal for all order elements ,g
Atomic mass number, Atomic mass number
38. Volume (V) of the nucleus is related with mass
number (A) as

::::::================================= = 4.85

www.puucho.com
Rahul Sardana with www.puucho.com

Advanced JEE Physics Optics & Modem Physics

42. When a slow neutron is captured by a U 235 nucleus, a (q 0.384% (D) 0.484%
fission results which releases 200 MeV of energy. If the
output of a nuclear reactor is 1.6 MW , the rate of Comprehension 16
nuclei (per sec) undergoing fission is Many unstable nuclei c~ decay spontaneously to a
(A) 8 x 10" (B) 1 x 1013 nucleus of lower mass but different combination of nucleons.
The process of spontaneous emission of radiation is called
(q lx10" (D) 5xW" Radioactivity in which the decay rate is actually
exponentially decrease with time. Also, radioactive decay is
43. The binding energy per nucleon versus mass number is
a statistical process which is independent of all external
best represented by the curve
conditions. Based on above information, answer the
MeV
following questions.
9
I
\
1---~-
,,--~-
47. If TH is the half life and TM is the mean life, which of
\ ,,,.,,, ...
,,,"\ the following statements is correct ?
o " " - - - ' - - ~ - ~ ~ • Mass No. (A) (A) TM >TH
' , 120 .," A
~-' (B) Both are equal
-~ C (q TM <TH
"ffi -9 MeV
(D) Nothing can be said
(A) A (B) B 48. If n is the number of a- particles being emitted per
(q C (D) None of these second by N atoms of a radioactive element, then half
life of element, in second, will be
Comprehension 15 n N
In a nuclear fusion reactor, the reaction occurs in two (A) - s (B) - s
N n
stages.
0.693 0.693n
Stage-I: Two deuterium ( 1 D2 ) nuclei fuse to form a (C) - - s (D) - - - s
n N
tritium ( 1 T 3 ) nucleus with a proton as a by-
product. The reaction 1113-y be represented as 49. The activity of radioactive substance of decay constant
D(D, p)T. A. is R1 at time t1 and R2 at time t2 ( > t1 ) , then
Stage-II : A tritium nucleus fuses with another deuterium (A) R111 = R,t, (B) R, - R, = constant
nucleus to form a helium ( 2 He 4 ) nucleus with a t2 -t1
neutron as a by product. The reaction is (C) R, = R,,-,t•,-•,I (D) R,t, = R,t,
represented as T(D, n)a..
Comprehension 17
Given that m(,D')=2.014102u (atom)
An .electron orbits a stationary nucleus of charge +ze
m(. T') = 3.016049u (atom) where z is a constant and e is the magnitude of electronic
m(,He')=4.002603u (atom) charge. It requires 47.2 eV to excite the electron form the
second Bohr orbit to third Bohr orbit. Based· on above
m(. H') = 1.007825u (atom) information, answer the following questions.
m(,n') =1.008665u
Based on above information, answer the following questions. 50. The value of z is
(A) 5 (B) 4
44. The energy released in the Stage-II of fusion reaction, is (C) 3 (D) 2
(A) 4.033 MeV (B) 17.587 MeV
(q 40.33 MeV (D) 1.7587 MeV 51. The radius of first Bohr orbit is
(A) 0.53 A (B) 0.106 A
45. The energy released in the combined reaction per (CJ o.318 A (D) 0.53nm
deuterium, is given by
(A) 4.207MeV (B) 5.207MeV Comprehension 18
(C) 6.207MeV (D) 7.207MeV
In quantum mechanics, some quantities are discrete and
cannot be continuous. One of these quantities is the energy.
46. The percentage of the mass energy of the initial
Energy can only take certain values, like E1 , E2 , E3 , E4 , .•.. ,
deuterium is released is
(A) 0.184% (B) 0.284% which are called energy levels. The energy cannot take any

= 4.86 i::::=================================
www.puucho.com
Rahul Sardana with www.puucho.com

Atomic & Nuclear Physics

values between E1 and E2 or E2 and E3 or E3 and E4 , etc.


(C) J neh (D) J2neh
Certain transitions from one energy level to another result in 2nBe 1tBe
the emission of a photon of radiation, whereas others can
only take place if a photon is absorbed. The energy levels in 56. The kinetic energy of the electron in nth orbit is
a newly discovered gas are expressed as
En=-E1z2 (A) nh(:) (B) 2nh(:)
n' .

where, -E1z is the ground state energy. Taking z =1 for


2
(C) n;(:) (D) n:(:)
simplicity, but not assuming that the gas is hydrogen, an
experiment is designed to measure the energy as a functions 57. Potential energy of interaction between the magnetic
of the level. The results obtained in the experiment are given moment of the orbital current due to the electron
in the table.
------~------,
I n E,,(eV)
moving in its orbit and the magnetic field B is
(A) nhBe (B) nhBe
2 -144 2m m
3 -64 2nhBe nhBe
(C) (D)
4 -36 m 4m
Based on the information given, answer the following
questions. 58. Total magnetic flux due to the magnetic field B
passing through the nth orbit is
52. The ionization energy of the gas is 1t nh nnh
(A) 244 eV (B) 576 eV (A) (B)
2 e e
(C) 144 eV (D) +13.6 eV 2nnh
(C) 4nnh (D)
e e
53. The ground state energy is
(A) -144 eV (B) +144 eV
Comprehension 20
(C) -244 eV . (D) None of these
A radioactive sample has mass m, decay constant A
and molecular weight M . H Avogadro number is NA then
54. The graph of En versus \ is best represented as answer the following questions.
n
59. The initial number of nuclei present is
m
'•', {A) (B) MN,
{A)
•, N, m
' ', (B)
',_ (C) .!!!..N
M A (D) MN,

60. The number of decayed nuclei after time t is

(D)
..
' ,
,
(A) -m N ,e-J..1
M
(B) -e
M
m -it

'•, (C) .!!!..N (1-,-")


M A
(D) .!!!..(1_,-'-')
' ... M

61. The activity of sample after time t is


Comprehension 19
An electron is orbiting in a circular orbit of radius r (A) ( m~;'" }-" (B) :NA1"(1-e-'-')
under the influence of a constant magnetic field of strength
(C) MN,1',-" (D) None of these
B. Assuming that Bohr's postulate regarding the m
quantisation of angular momentum holds good for this
electron of charge e and mass m , answer the following
Comprehension 21
questions.
In atom X , a single electron orbits around a stationary
nucleus of charge +Ze where Z is a constant and e is the
55. The radius of nth orbit of the electron will be
magnitude of the electronic charge. It requires 47.2 eV to
(A) J2nh (B) ,nh excite the electron from the second Bohr orbit to the third
nBe V27tlk
4.87=

www.puucho.com
Rahul Sardana with www.puucho.com

Advanced JEE Physics Optics & Modern Physics


Bohr orbit. Take ionization energy of the hydrogen atom particular upper (excited) energy level B and there are no
13.6 eV. . ' atoms.in any other energy level. The atoms of the gas make
transition to a higher energy level by absorbing
62. The value of Z is. monochromatic light of photon energy 2.7 eV. Subsequently
(A) 3 (B) 4 the . atoms emit radiation of only sb< different photons
(C) 5 (D) 6 energies. Some of emitted photons have energy 2.7 eV.
Some have·more and some have less than 2.7 eV. Based on
63. Kinetic energy of electron in ground state of atom X is the information given, answer the following questions.
(A) 122.4 eV (B) 170 eV '
(C) 340 eV (D) 680 eV 68. The principal quantum number of initially excited level
B is .
64. The wavelength of electromagnetic radiation required (A) 1' ' 1 (B) 2
to remove the electron from 3'• Bohr orbit to infinity is (C) 3 (D) None of these
(A) 329 A ,. (B) 428 A
(C) 546 A (D) 636 A 69. The magnitude of ionisation energy of.gas atoms is
(A) 13.6 eV (B) 12.7 eV
Comprehension 22 (C) 54.4 eV (D) None of these
The nuclei of' a radioactive element X are being '
produced at _a constant rate a and this element decays ti;). a '
70. The maximum and minimum energies of the emitted
stable nucleus Y with a decay constant 1,. and half-life T1/2 • photons ·
At time t = 0 , there are N O nuclei of the element X . ~ased (A) 12.75 eV and 0.66 eV
(B) 13.6 eV and 12.75 eV
on the information given, answer the following questions. (C) :i3.6 eV and 0.66 eV
(D) None of the'se
65. The number N x of nuclei of X at time t :a T1/2 is

(A) a-1,.N,
. 21,.
(B) (21,.N -a).!
' .o . 1,. . '
Comprehension 24
The figure shown an energy level diagram for the
" (1,.N,+f)1i
_(C) (D)
a+'>..N~,
21,.
hydrogen atom. Several transitions are marked as I, II, ill, IV,
V, & VI. The diagram is only indicative and not to scale.
Based on the information given, answer the. following
66. The number Ny of nuclei of Y at time I is questions.
"'-----------
(A) at-( a-tNo}-" +( a-tN,) 4
'
.

3 ' mT-
(B) at+(a-~,}-"-(a-tN')
2
II 1vl V JVI

(C) at+( a-tN, }-" I

(D) at-( a-tN, }-"

71. A Balmer series photon is absorbed in


67. The number Ny of nuclei of Y at l=T1/2 is
(A) II (B) III
(A) ~lo (2)+I(a-1,.N,) (C) IV (D) VI
,.g, 2 1,.' '.
72. The wavelength of. the radiation involved in transition
(B) flog,(2)+½( a-tN,) fl is
(A) 291nm (B) 364nm
i
(C) . log,(2l'-½( a-tN,) (C) 487nm (D) 652nm

(D) f Iog,(2)-2( a-tN,) 73. ,The transition that occur, when a hydrogen atom is
irradiated with radiation of wavelength 1025 A
(A) I (B) II
Comprehension 23 . (C) IV (D) V
, A gas of .identical hydrog€n like atoms has some ato~,
in lowest (ground) energy level A and some atoms in

, 4.88 ::.::=====================================
www.puucho.com
Rahul Sardana with www.puucho.com

Each question in this section contains statements given in two colwnns which have to be matched. The statements in
1

COLUMN-I are labelled A, B, C and D, while the statements in COLUMN-II are labelled p, q, r, s (and t). Any given statement
in COLUMN-I can have correct matching with ONE OR MORE statement(s) in COLUMN-II. The appropriate bubbles
corresponding to the answers to these questions have to be darkened as illustrated in the following examples: '
If the correct matches are A-> p, s.and t; B-> q and r; C-> p and q; and D-> sand t; then the correct darkening of bubbles will
look like the following :
P q r s I
A@@)0@(D
B@@)0@G)
c@@)0@G)
D@@©®CD

1. Some laws/processes are given in COLUMN-I. Match these with the physical phenomena given in
COLUMN-II.
[ COLUMN-I COLUMN-II
(A) Nuclear fusion (p) Converts some matter into energy
(B) Nuclear fission (q) Generally possible for nuclei with low atomic
(C) p-decay number
(D) Exothennic nuclear reaction (r) Generally possible for nuclei with higher atomic
number
(s) Essentially proceeds by weak nuclear forces.

2. Some laws/processes are given in COLUMN-I. Match these with the physical phenomena given in
COLUMN-II.
I COLUMN-I COLUMN-II
(A) Transition between two atomic energy levels (p) Characteristic X-rays
(B) Electron emission from a material (q) Photoelectric effect
(C) Mosley's Law (r) Hydrogen spectrum
- (D) Change of photon energy into kinetic energy of (s) P-decay
electrons

4.89 =
www.puucho.com
Rahul Sardana with www.puucho.com

,
Advanced JEE Physics Optics & Modern Physics

3. Match the quantities in COLUMN-I with their dependence on the principal quantum number n and the atomic number
Z in COLUMN-IT.
I. - COLUMN-I COLUMN-II l
(A) Angular speed n'
(p)
(B) Time period Z'
(C) Angular momentum (q) n
(D) Magnetic moment Z'
Magnetic Field (r)
(E) 7
Z'
(s)
7
4. The energy, the magnitude of linear momentum, magnitude of angular momentum and orbital radius of an electron in a
hydrogen atom corresponding to the quantum number n are E, p, L and r respectively. Then according to Bohr's
theory of hydrogen atom match the expressions in COLUMN-I with statement in COLUMN-II.
I COLUMN-I COLUMN-II l
(A) Epr (p) is independent of n
(q) is directly proportional to n
(B) E.
E (r) is inversely proportional to n
(C) Er (s) is directly proportional to L
(D) pr

5. If f, is the maximum frequency of emitted photon of Lyman series, f, is minimum frequency of the emitted photon of
Lyman series and f, is maximum frequency of the emitted photon of Balmer series, then match the relations in given in
COLUMN-I with the corresponding relations in COLUMN-IL
COLUMN-IT
(A) f, (p) greater than f,
(B) f,. (q) is negative
(C) (f, - f,) (r) less than / 2
(s) is equal to / 3
(D) (f, - f,)

6. Match the quantities in COLUMN-I with their respective v~ues in COLUMN-II.


COLUMN-I ,. '·. COLUMN-IT
(A) Pair Production (p) FewMeV
(B) Inverse photoelectric effect (q) 20KeV
(C) De-excitation of Be+4 atom from second excited state (r) 54eV
(D) K X-ray photons of molybdenum Z = 42
0
(s) O.leV

7. Match the processes given in COLUMN-I with their characteristics in COLUMN-II.


I COLUMN-I COLUMN-II
(A) . Alpha decay (p) Mono energetic particles are emitted
(B) Beta decay (q) Poly energetic particles are emitted
(C) Positron emission (r) Angular momentum is conserved
(D) Electron capture (s) Can take place inside and outside nucleus

= 4.90 ================================::::i
www.puucho.com
Rahul Sardana with www.puucho.com

Atomic & Nuclear Physics


8. Match the quantities in COLUMN- I with their respective values in COLUMN-II.
COLUMN-I COLUMN-II
(A) 1 rutherford {p) 1 disintegration/ sec
(B) 1 becquerel (q) 3.7xl010 dis/sec
(C) 1 curie
(r) 10 6 dis/sec
(D) Activity of 1 g Ra"'
(s) 1010 dis/ sec

9. Match the processes given in COLUMN-I with their characteristics in COLUMN-II.


[ COLUMN-I COLUMN-II
(A) a- decay {p) Atomic number of the product nucleus
(B) p• - decay decreases
(q) Atomic number of the product -nucleus
(C) p- -decay
increases
(D) Electron capture (r) Atomic number of the product nucleus not
necessarily changes
(s) some mass is converted into energy

10. Match the processes given in COLUMN-I with their characteristics in COLUMN-II.
[ COLUMN-I COLUMN-II
(A) a-decay {p) Mass number decreases
(B) p-- decay (q) Atomic number decreases
(C) p• - decay (r) Mass number does not change
(s) Chemical symbol of nucleus changes
(D) r- decay
(t) Energy is released

11. Match the reactions/processes given in COLUMN-I with their characteristics given in COLUMN-II.
COLUMN-I COLUMN-II
(A) Spontaneous radioactive decay of an uranium nucleus {p) Number of protons is increased
initially at rest as given by reaction (q) Momentum is conserved
~u------+ ~Th + i He+ .... (r) Mass and energy are inter convertible
(B) Fusion reaction of two hydrogen nuclei as given by (s) Charge is conserved
reaction (t) Angular momentum is conserved
:H + :H-'J>iH + ....
(C) Fission of U"' nucleus initiated by a thermal neutron
as given by reaction
6n+~U-'J> 1,:tBa+ :Kr+36n
(D) p-- decay (negative beta decay)

12. Match COLUMN-I of the nuclear processes with COLUMN-II containing parent nucleus and one of the end
products of each process and then select the correct answer using the codes given below the lists.
[ COLUMN-I COLUMN-II
{A) Alpha decay {p) i:0~1;N + ....
(B) P'-decay
(q) ~U~2;,Th+ ....
(C) Fission 1
(r) :Bi~ 1~Pb+ ....
(D) Proton emission
(s) ~Pu~ 1:La+ ....

4.91
=
www.puucho.com
Rahul Sardana with www.puucho.com

In this section the answer to each of the question is a four digit integer, ranging from 0 to 9999. The appropri;te bubbles below
the respective question numbers in the ORS have to be darkened. For example, if the correct answer to question number X (say)
is 6092, then the correct darkening of bubbles wil! look like the following :
X.@e@@
· CD (D'G) CD
®®®·
@@@@
,@@©@
®@@@
-~~~~
'@@®<ID
@®e@
1. How many head-on, elastic collisions must a, neutron 6. At time t = 0, activity of a radioactive substance is
have with deuterium nucleus to reduce its energy from 1600 Bq·, at J=8 s activity remains 100 Bq. Find the
1 MeV to 0.025 eV. activiiy, in ilq, at t = 2 s .
2. The binding energy of an electron in the ground state of
7. · A "'Cd radionuclide goes through the transformation
He atom is equal to E, = 24.6 eV. Find the energy
chain.
required, in eV, to remove both, electrons from the 118Cd 30 min 118ln 4,Smin 1185n (stable)
atom.
The half lives are written below the respective arrows.
3. Find the quantum number n corresponding to nth A time t = 0 only Cd was present. Find the percentage
excited state of He+ ion if on transition to the ground of nuclei transformed into stable ov~r 60 minutes.
state the ion emits two photons in succession with
wavelengths 108.5 nm and 30.4 nm . The ionization 8. The work functions of Silver and Sodium are 4.6 and
2.3 eV, respectively. Find the ratio of the slope of the
energy of the hydrogen atom is 13.6 eV.
stopping potential versus frequency plot for Silver to
that of Sodium.
4. From what material ( Z value) is the anode of an X-ray
tube made, if the K. . line wavelength of the
9. A freshly prepared sample of a radioisotope of half-life
characteristic spectrum is 0.76 A ? 1386 s has activity 103 disintegrations per second.
Given that log, 2 = 0.693, find the fraction of the initial
7
5. A Li target is bombarded with a proton beam aµrent number of nuclei (expressed in nearest, integer
of 10°" A for 1 hour to produce 7 Be of activity percentage) that will decay in the first 80 s after
1.8 x 10 8 disintegrations per second. Assuming that one preparation of the sample.
7
Be radioactive nucleus is produced by bombarding
1000 protons, determine its half-life, in days, to the 10. A silver sphere of radius 1' cm. and work 'function
nearest three digit integer. 4.7 eV is suspended from an insulting thread in free-

= c::::=================================
4.92

www.puucho.com
Rahul Sardana with www.puucho.com

Atomic & Nuclear Physics


space. It is under continuous illumination of 200 run
wavelength light. As photoelectrons are emitted, the 17. Emission spectrum of hydrogen atom has two lines of
sphere gets charged and acquires a potential. The Balmer series with wavelength 4102 A and 4861 A . To
maximum number of photoelectrons emitted from the what series does a spectral line belong if its wave
sphere is Ax 10 2 (where 1 <A< 10 ). Find the value of number is equal to the difference of wave numbers of
z. the above two lines? What is the wavelength of this line
in A ?Given, R=l.097xl07 m-1 •
11. The activity of a freshly prepared radioactive sample is
10 10 disintegrations per second, whose mean life is 18. A hydrogen like atom (atomic number Z) is in a higher
109 s. The mass of an atom of this radioisotope is excited state of quantum number n . The excited atom
10-" kg . Find the mass (in mg) of the radioactive can make a transition to the first excited state by
sample. successively emitting two phOtons of energy 10.2 eV
and 17 eV respectively. Alternately, the atom from .the
12. A proton is fired from very far away towards a nucleus same excited state can make a transition to the second
with charge Q = 120e, where e is the electronic charge. excited state by successively emitting two photons of
It makes a closest approach of 10 frn to the nucleus. energies 4.25 eV and 5.95 eV respectively.
Find the de-Broglie wavelength (in units of fm) of the Determine the values of n and Z .
proton at its start. (Ionization energy of H-atom = 13.6 eV)

Take the proton mass, m, =(¾)x10-27 kg, 19. The ionization energy of a hydrogen like Bohr atom is
4rydberg.
1
!'. = 4.2 x 10-15 JsC-1 , - - = 9 x 109 mF-1 and (a) What is the wavelength of the radiation emitted, in
e 41ts0
A, when the electron jumps from the first exdted
1 frn =10-15 m . state to the ground state?
(b) What is the atomic number Z of the atom?
13. A stream of a. -particles is incident on a sample of (c) Also the radius of the first orbit for this atom as a
hydrogen gas. What should be the minimum kinetic
energy of a. -particles, in e V , to ionize the hydrogen multiple of the Bohr's radius a0 is ao , where * is
atoms. •
not readable. Find • .
14. Hydrogen gas in the atomic state is excited to an energy 20. A doubly ionized lithium atom is hydrogen like with
level such that the electrostatic potential energy of H - atomic number 3. Find the wavelength of the radiation
atom becomes -1.7 eV. Now a photoelectric plate
to the nearest three digit integer , in A , required to
having work function W = 2.3 eV is exposed to the
excite the electron in Li++- from the first to the third
emission spectra of this gas. Assuming all the
Bohr orbit. The ionization energy of the hydrogen atom
transitions to be possible, find the minimum de-Broglie
is 13.6 eV.
wavelength of the ejected photoelectrons in A to the
nearest integer.
21. The electric current in an X-ray tube operating at
40 kV is 10 mA. Assume that on an average 1 % of the
15. A hydrogen like atom (described by the Bohr model) is
total kinetic energy of the electrons hitting the target are
observed to emit six wavelengths, originating from all
converted into X-rays.
possible transitions between a group of levels. These
(a) What is the total power emitted as X-rays, in watt?
levels have energies between -0.85 eV and -0.544 eV
(b) How much heat, in joule is produced in the target
(including both these values). every second?
(a) Find the atomic number of the atom.
(b) Calculate the smallest wavelength emitted in these
22. The mean lives of a radioactive substance are 1200 yr
transitions, in A. and 600 yr for a. -emission and P-emission
(Take ground state energy of hydrogen atom to be
-13.6 eV). respectively. Find out the time, in year, during which
three fourth of a sample will decay if it is decaying both
16. A neutron with an energy of 4.6 MeV collides with by a. -emission and P-emission simultaneously.
protons and is retarded. Assuming that upon each Given log, (4) = 1.4
collision neutron is deflected by 45° find the number of
collisions which will reduce its energy to 0.23 eV.

================================:::] 4.93 =
www.puucho.com
Rahul Sardana with www.puucho.com

IB30 . . . _' ·_ ..'.: . ,; -~: ': ;' . _. . . ..... ·-~: :.··, ,:~ BASEDlONrATOMIClSJRUCliUREl&lP.ROP.ERlilES
1. (a) 113.74 A 14. (a) 1.587 A
(b) 3 (b) 1.095 ~ 1o• ms·'
2. 122.4 eV neh µonm2e1
15.
3. 793.3 A 4nm ' 8s0 h6n5

4.

5.
2eV, 0.754V

7.52x1018 Hz
16. (~r
Samit'
.

17, (a) 42
6. 15865 volt
(b) o.163 A
7. o.31A
18. FlrstOrblt: 'KE=13.60eV, PE=0eV, TE=13.60eV
8. (a) He
Second Orbit: KE= 3.40 eV, PE= 20.40 eV, TE= 23.80 eV
(b) 54.4 eV
19. 0.55 eV
(c) 10:2ev
9. 12;1 eV 20. -./2
21. (a) 3.4 eV
10. ·-26.9 eV, -12 eV
11. 10.2 V, 2, 10.2 eV, 51 eV (bl 6.63 A

12. 3.1x106 ms-1

'13. (a) 15.6 eV

(b) 2335A
(c) 12.52 V

(rl) 1.01x107 m-1


(e) (I) 6 eV , (ii) 0.7 eV

= ==================================
4.94

www.puucho.com
Rahul Sardana with www.puucho.com

Atomic & Nucleqr Physics

1. 81

2. 1,55x106 J

3. 1.4x1QH' s

4. 3.81x.104 year-1 and 3.14x10-2 year-1 9. 0.39


10. (a) 5724 years
5. (a) 0.113 min-'
(b) 6.132mln (b) 12.58 dismin-19-1

6. 1.2sx10-11 % 11. 1.12x10.. g

7. (a) L42x10" per year 12. 1.88x104

(b) 1.41x10" per year 13. 0.205 mCi

(c) 5.4x10' years

BASED ON BINDING ENERG:V. l/1 NUCl!EAR REACTIONS


1. No; first reaction is not p6ssible. : 8. 120.35 g
2. 1.54 MeV
9. (a) 8.09x10" J
3. 8.78 day
(b) 2.?xio• kg
4. 4.7x10-14 kg
10. (a) -1.18 MeV
5. 0.1308 MeV (b) 43°
6. K.., =0, K= =4.4 MeV 11. 127.6 MeV
7. 0,961 MeV

1. B· 2. D 3. C 4. D 5. B
6. D 7. D 8. D 9. A 10. A
I , 11. C 12. D 13. B 14. A 15. D
16. C 17. A 18. D 19. A 20. C
I· 21. B · 22; A 23. D 24. D 25. B
26. C 27. D 28. D 29. A 30. C
'31. A" 32. D 33. B 34. C 35. C
36. D 37. D 38. A 39. D 40. A
, 41. D 42. A'- 43. B 44. B 45. D
46. B 47. C 48. B 49. C 50. B
51. C 52. C 53. D 54. C 55. B
56. C 57. A 58. D 59. D 60. B
61. C 62•. C 63. C 64. D 65. C
66. C 67. B 68. D 69. D 70. C
71. B 72. A 73. A 74. A 75. B
I
4.95 =
www.puucho.com
Rahul Sardana with www.puucho.com

Advanced JEE Physics Optics & Modern Physics

76. B n. C 78. C 79. C 80. C


---------
81. D 82. A 83. A 84. C 85., B 7
86. A 87. A 88. C 89. C 90. B
91. A 92. C 93. D 94. C 95.' C
- ~ :::J
96. D 97. D 98. B 99. B 100. C
[iii1. A 102. B 103. D 104.
-B - - 105. C
--·----
106. D 107. C 108. B 109. B 110. D
L11~.- C. 112. D 113. B 114. D 115. B
116. D 117. D 118. D 119. D 120. D
121. B 122. D 123. A 124. D 125. A 7
126. A 127. B 128. C 129. C 130. C
131. B 132. D 133. A 134•. A 135. B
136. B 137. B 138. A 139. C 140. A
141. B 142. B 143. A 144. D 145. A
146. D 147. C 148. B 149. C 150. A
151. C 152'. B 153. B 154. D 155. A
156. D 157. A 158. D 159. B 160. B
161. A 162. D 163. D 164. B 165. C
166. B 167. C 168. A 169. B 170. A
171. C 172. C 173. D 174. A 175. B
176. C 177. C 178. B 179. B 180. A
181. B 182. C "183. D 184. A 185. C
186. D 187. B 188. C 189. C 190. A
191. C 192. C , 193. A 194. c 195. B
196. D 197. ,c 198. D 199. C 200. C
201. D 202. D 203. A 204. A

1. BC 2. AC 3. A<;, 4. AD 5. BC
6. BC 7. ACD 8. CD 9. AD 10. AD
11. AD 12. ABD 13. AB 14. CD 15. ABC
16. ABC 17. ABCD 18. ABCD 19. BCD 20. ABCD
21. AD 22. AC 23. ABCD 24. ABD 25. BD
26. BC 27. ACD 28. BD 29. BCD 30. BC
31. AD 32. AD 33. CD 34. ABD 35. BCD
36. ABC 37. ABD 38. ACD 39. BC 40. AC
41. BC 42. AB 43. BD

= c:::==================================:::::i
4.96,

www.puucho.com
Rahul Sardana with www.puucho.com

Atomic & Nuclear Physics

I
~-
1. B 2. D 3. C 4. D
-----------·------------------- 5. C
----------'·I
6. C 7. A 8. A 9. B 10. B
L_21_. ~ _ _ _ _ _ _ _1_2._c
_ _ _ _ _ _ _1_3._c
_______14_._0_ _ _ _ _ _1_5_._s
____ 7___,
16. A 17, D 18. A 19. A 20. D
21. B 22. D 23. ' A 24. A 25, B J
a A ~ A a A

1. C 2. C 3. A 4. A 5. C
6. D 7. A 8. B
10. D 9. D
C,1.
16. B
A
17. C
------------··-
12.
- -C - - - - - - 1~
B,
18. D
- -A- ~ - - - - 1~
-------------- C
·------------ - - - - - - ----------
19. A
13.
-
20. B
__, ____
l- - - - - -
21. A 22. B 23. C _ _ _2_4. -s-----·-2'--5-_- c - - - ' - - 7
26. B 27. A 28. A 29. C 30. C
r---
~
-~----~---·-·-· ---------- a j
~ ' - - C ----- -
36. B
---- D
- - - - - -~
- -B- - K D C
---------
~ C ~ D ~ B • A
;-- 41. C 42, D 43. A ---4-4-.- B - - - ~ - - - 45. D j
~-------- -------- ----------~---- ---
~ C ~
-------------
- C
L. 51. B
R A
52. B
• C
------'-~~ __D_______ _!;~:.,._A_______ 55,
A
B
a C ~ A - B ~ C ~ C
-------------- -- ----,
' - ~-
---A
- - - - - - -~- C
.
~ C
-- - - - ------------- KA ~ D
-
L!.:: ..E _
B ~ C
---1-2-.- c - _-
_ :_-_--~--
~ B ~
_:·-,_-a-.-_-_o_______-__:::_:::_-=----_-__-_-_-_ --~=----_-________ _m
A A

• ANSWERS•-·

1. A-> (p, q) 2. A-> (p, r) 3. A-> (r) ·4, A-> (p, s) 5. A-> (p) ----~,
B-> (p, r) B-> (p, q, s) B ->,(p) B-> (q, s) B-> (p)
C ->-(P, s) C-> (p) C->(q) C-> (q, s) c ....!_Jq)1
D-> (p, q, r) D..; (q) D-> (q) D-> (s) D-> (s)
______________ E-> (s)
6. A-> (p) 7. A-> (p, r) 8. A-> (r)
--··-·~-- - - -
9.
-------·---
A-> (p, s)
-
10. A-> (p, q, ~. t)
B .... (q) B .... (q, r, s) B .... (p) B -> (p) B-> (r, s, t)
C-> (r) C -> (q, r) C -> (q) C-> (q, s) C-> (q, r, s, t)
D-> (q) D-> (p, r) D-> (q) D -> (p) D -> (r, t)
'-"----
11. A->(q,r,s,t) 12. A-;(q)
8-> (q, r,,s, t) 8-> (p)
C-> (q, r, s, t) C-> (s)
~---D_->_(~p,_q~,_r,~··~t)_ _ _ _ £.=.i9,__________ -----~

================================== = 4.97

www.puucho.com
Rahul Sardana with www.puucho.com

Advanced JEE Physics Optics & Modern Physics

I1. 8 2. 79 3. ·5 4. ·41
5. 100 6. 800 7. 31 8.
I 9. 4 10. 8 11. 1. 12. i . .
13. 68 14. 4 15. (a) 4, (b) 40441 16. 24
\11. 26206 18. 6 19. (a) 300, (b) 2, (c) 2 20. 114
21. (a} 4, (b) 396 22. 560

= =================================
4.98

www.puucho.com
Rahul Sardana with www.puucho.com

1. (a) E3 -E1 (-13.6)(3)' [(-13.6)(3)']=lOB. 8 eV t. (2"'-1)'


, (3)' (1)' \:u = Zcu -1

(b)
=>
Number of lines in emission spectrum is
=> t. =t.(~:=;J
N = n(n-1) => i. =1.88 x 10" ( 57 -l)'
29-1
2
=> N . (3)(:-1) 3 => t. = 7.52x10" Hz

12 75
6. Since, A= ~ , so we have
2. S' E (13.6)2' V
mce, n- n2 e

So, ibnisation energy, n =1, for u++ (Z =3) is


i., _ i., = 12375 _ 12375 = 12375
V, V,
(.!.- _1_)
V 1.5V
IE- (13.6)(3)' eV where i., - i., = 26 pm = 0.26 A
(1)'
=> 0.26 = (12375)(3~)
=> IE= 122.4 eV
=> V = 15865 volt
3. Energy .of electron in ground state of hydrogen atom is
-13.6 eV . Earlier it had a kinetic energy of 2 eV 7. Cutoff wavelength Am1n is given by,
Therefore, energy of ·photon released during formation of
,hydrogen atom, / i. . (in A)- 12375 12375 _ 0_ 31 A
&E = 2-(-13.6) =15.6 eV mm V (in volt) 40 X 103

=> 1. = 12375 = 12375 = 793.3 A 8. (a) As the atoms finally emit radiation of only 3 different
&E 15.6 photon energies final excited state corresPonds to n = 3 .
So, the initial excited state corresponds to n = 2
4. For shorter wavelength, we have
=> 2'(13.6)(.!-.!)= 12375
E E
,i = • - E' (-13.6)(3)'
(4)'
[(-1~:i!(3)']=5.95 eV
=> 2=2
4 9 1654

Using Einstein's Photo-Electric Equation, we get Therefore, it fs helium atom.


W =E-K= = (5.95-3.95) eV =2 eV (b) Ionization energy is
For longer wavelength, we have IE=2'(13.6 eV)=(2)'(13.6 eV)
=> IE = 54.4 eV
&E=E -E (-13.6)(3)' [(-13.6)(3)']=2.754 eV
' ' (5)' (4)' (c) N=n(n-1)= 6
Again using Einstein's Photo-Electric Equation, we get 2
Kmax =E-W=0.754 eV So, E = E, -E, =(13.6)(4)(.!-J...) = 10.2 eV
Hence stopping potential is 0.754 V , · 4 16

5. Using the equation, Ji= a(2-b) 9. Energy of photon corresponding to i. = 6563 A is


(b =1)
Since, b-= 1, so we get

4;99 =
www.puucho.com
Rahul Sardana with www.puucho.com

Advanced JEE Physics Optics & Modeni Physics

12375 Since, K=·-½mv 2 =4.352x10-


110
AE= eV=1.88eV
6563
~--~-~-n=3 · 2x4.352x1'0-18
1.88 eV n = 2 V=
9.1x10-31
=>· v=3.1x10 6 ms-1
12.1 eV
13. (a) From figure, we observe that
lonisati?n) =15 _6 eV
~------n=1 ( Potential
This is the difference in energy between n = 3 and n =2 .
Hence the single electron in the hydrogen atom should excite at (b) ' = 12375 = 2335 A
min 5.3
least upto n =3 and for this the minimum energy of the striking
(e) AE 31 = -3.08_-(-15.6) = 12.52 eV
electron should be 12.1 eV. ·
Therefore, excitation potential-for state n = 3 is 12.52 V
10. Given E3 =0 (d)
1 _. AE,, A-'_ 12.52 A-'
,,, - 12375 - 12375
,, =460 A
-~--~--n=3
=> E, -E, = 12375 =26.9 eV
460
1'-'''---n =2
-h---'-
=> ,,,
....:!...~1.01x10 7 m-1

(e) (i) E2 -E1 =10.3 eV > 6 eV


Since, E3 =O
Hence, the striking electron cannot excite the
=> 0-E1 =26.9eV hypothetical atoms. So the electron will keep its
=> E, = -26.9 eV energy with itself.
- ' U . - - - - - n =1 => Krnrn=6eV
Further, ,, = 1035 A
(ii) E, -E, =10.3 eV <11 eV
=> E3 -E 2 = 12375 A =12 eV So, the electron can excite the atom.
1035
=> K•• =(11-10.3) eV=0.7eV
=> E2 =-12eV

n'
11. When hydrogen atom is excited, then we have 14. (a) z = 3 for u+2 • Further we know that rn = -a 0
z
eV = Eo 1 n2 (1-..!.) ... (1)
Substituting, n = 3, z = 3 and a 0 =0.529 A , we get r3 for
When ion is excited, then u+2

eV=E
o z'(..!._..!.)
22 nf
... (2) (3)'
r, =w(0.529) A =1.587 A
Wavelength of emitted light is
(b) z = 2 for He+ . Also we know that
he= E
A1
(1-..!.)
1 n2
o
... (3) z
Vn = r,V1
he =E z'(1-..!.) ... (4) Substituting n=4, z=2 and v,=2.19x10 6 ms-1 ,weget
A.2 °1n~
Further it is given that for He+,

!:1.
,, 1
=~ ... (5) v 4 =(¾)<2.19x10') ms-1

Solving the above equations, we get => v4 =1.095x106 ms-1


2=2, n=2, n,=4 and V=10.2V
_Energy of emitted photon by the hydrogen atom is 2
15. Magnetic moment µ=NiA=(f)(rcr )
AE = E, -E, = 10.2 eV
and by the ion is
=(-e_-)(.r')= evr ... (1)
=> µ 2rcr/v 2
AE' = E, -E, = (13.6)(2)'(1- ~) = 51 eV
1
We know that mvr = nh ... (2)
2,
12. Energy of photon of the flrst line of Lyman series is
Solving equations (1) and (2)
E = E, -E, = (13.6)(2)'(1-¾) = 40.8 eV neh
µ = 4rcm
Energy required to ionize the hydrogen atom is 13.6 eV.
Therefore, kinetic energy of electron emitted from the hydrogen Magnetic induction, B = µoi = µoe
atom is 2r 2rT
K=(40.8-13.6) eV=27.2eV B= µ 0ev µ 0ev ... (3)
=>
=> K=4.352x10-18 J (2r)(2.r) 4.r'

=4.100 =================================:;::::==
www.puucho.com
Rahul Sardana with www.puucho.com

Atomic & Nuclear P/zysics


From Newton's Second Law, we have
Orbit K(eV) U(eV) E(eV)
2
0 mv2
4m;:/! =-r- First 13.60 0 13.60
Second 3.40 20.40 23.80
=> v2=--
e'
".(4)
4m:0mr 19. Wavelengths corresponding to minimum w.avele_ngth {Amin) or
Solving these equations, we get maximum energy will emit photoelectrons having maximum
µ nm2e 7 kinetic energy.
B=-'--
8E0h5n5 {Amln) belonging to Balmer·series and Lying in the given range
(450 nm to 750 nm) corresponds to transition from
16. The force at a distance r is, (n=4 to n=2). Here,
F=-dU =-2ar
dr E4 = -13.6
-- 2 = -0.85 e
V
(4)
Suppose r be the radius of nth orbit. Then the necessary
centripetal force is provided by the above force. Thus, 13 6
and E =- · =-3.4eV
mv' ' (2)'
-=2ar ... (1)
r => IIE =E, - E, =2.55 eV
Further, the quantization of angular momentum gives, So, Kmax =Energy of photon-work function
nh
mvr=- ... (2) => K==.2.55-2=0.55eV
2,
Solving equations (1) and (2) for r , we get
20. For Q::::;;x::::;;1, PE=E0
r =( n'h' )v• => (Kinetic energy K,) =(Total energy)-(PE)
Sarni'
=> K,=2Eo-Eo=Eo
17. (a) Given that
=> 1'--h~
2 {12375 X 10-,0} =____1:_-,---,'-
... (1)
' - ~2mE,
34,3 x l O' 1.09x10'(Z-1)'(1-¾) For X>1, PE=O
=> Z=42 :::::) Kinetic energy K 2 = Total energy = 2E0
(b) Energy of K, line =100-24=76 keV A h
=> ... (2)
1' = 12375 A ' ~2m(E,)
=> 76x10 3
Ka From equations (1) and (2), we get
=> 1',. =0.163 A ~=-,/2.
",
18. Since, E, =-13.60 eV
Also, we know that
21. (a) Kinetic energy of electron in the orbits of hydrogen and
hydrogen like atoms =[Total energy[
TE=-KE=PE So, Kinetic energy = 3.4 eV
2
(b) The de-Broglie wavelength is given by
=> K1 =-E, =13.60 eV
1'=_11_=_h_
=> U1 =2E 1 =-27.20 eV p ,/2Km
13.6Z' where K is the kinetic energy of an electron
Further, E0 =---, - Substituting the values, we get
n
1'- (6.6x10,.. Js)
=> E =_§_=-3.40 eV
' (2)' - ~2(3.4x1.6x10·" J)(9.1x10'" kg)
=> K,,, 3.40 eV and ::::) A=6.63x10-10 m
=> U2 =-6:80 eV => 1' =6.63 A
Now U1 = O , i.e., potential energy has been increased by
27.20 eV. So, We will increase U and E in all energy states by
27.20_eV while kinetic ·energy will remain unchanged. So, we
have

c:=================================4.101 =
www.puucho.com
Rahul Sardana with www.puucho.com

1. In 1 second 90% of the nuclei have remained undecayed, so in - => R0 =3.37x1017 per sec
another 1 second 90% of 90 i.e., 81 nuclei will remain
undecayed. After 70 hours actiVity
A=R 0e-11
1 0 693
2. Since, N = 0-3 x 6.02 x 1023 =2.87 x 1018 :::::,

A=(3.37x1017 )e
-( ·
14 3
·
x24)(ro)
=2.92x1017 per sec
210
During one mean life period 63.8% nuclei are decayed. In fruits, the activity was observed 1 µCi or 3.7x 10" per sec·.-..,.
Hence, energy released is Therefore, percentage of activity transmitted from root to the fruit
E =0.638x2:87x1018 xs:3x1 .6x10-13 J= 1.55x106 J is
'
~~rcentag8 ~f ). = 3.7~10" x100= 1.26 x 10-11 %
3. N = __g_x 6.02x1023 = 5.06 x 1021 ( ActIvIty Transmitted 2.92 x 1017 .
238
Since, R = i.N 1
7. (a) N=--x6.02x10"
A=~ 2.5x104 4.94x10-1e s-1 109
N· 5.06x1021 3 1
R=AN O.BQ x--x6.02x1023
=> . tV2-- 0 ·693
). -14
- • X 10" 5
2.7x10' 109
=> R=1.42x1014 peryear _ .
(b) After 2 year, R =Roe-11 = (1.42 X 1014 )eE--0.693/2.7•101](2)
R1 A1N 1.2
4.
R, = i.,N = 98.8 =>R=1.41x1014 peryear
=> "-2 = 82.3.3).., ... (1) (c) After 2 half lives activity reduces to 25% of the original
value, so we have
0 693 t = 21,,, = 5.4 x 1O' years
Further, i. = · year·'= 0.0318 year·• ... (2)
21.8
Also (-dN)=(- dN,)+(-dN;)
'dt dt dt 8. Since N = "- 1N° (e-li1 -e-l.z1 )
' 2 "-2 -A,
=> AN = A,N + A2 N
When _A1 » A2 we have, N2 ~ N0 e-1.z1
=> A=A1 +A2 ... (3)
Physically this means that parent nuclei practically instantly
Solving equations (1), (2) and.(3), we get transform into daughter nuclei, which then decay According-to
A, =3.81x10-4 year-1 and A. 2 =3.14x10-2 year-1 the Law of Radioactive Decay with a certain decay constant.
N,
5. (a) R=R0e-"
:::,. 2700 = 4750e-s1
~N, ------------ ----
=> i..=0.113min-1 i.,
0.693
(b) tv2 =-i.- = 6.132 min

6. R, =l.N

R0 °·
14.3x3600x 24
693
x 6.02 x 1023 per sec

=4.102c::=================================

www.puucho.com
Rahul Sardana with www.puucho.com

Atomic & Nuclear Physics

-N - !:I_ N -,,
2 - oe 12. '
Let x the desired ratio, then, mass of Co 58 in 1 g be x
i.,
=> N = ~ x 6.02 x10 23
When A, « A2 , then N2 r::: !:l.N0 e-'"'1 58
i.,
x ~ x 6.02 x 1023
0 693
Given, 2.2 x 1012 =AN= - ·
i.e., N2 versus t graph in this case is as shown in.the figure. 71.3x24x 3600 58
=> X=1.88x10-3
9. Probability of a nucleus to decay in time t is
_.!:!_ = 1- ~-U =1- 8-(flO)(S) = 0.39 13. Since, R = J.N - Nlog, ( 2 )
N, Iv,
==> R oc _!i_
10. (a) R =R,e-" 1,,
=> 7.3 =s.se-1..<4500- 25001
. N,
G,ven, N,
4 d (tv,), 14
=> A =1.21x10-4 year-1 =1 an (Iv,), = 25
0.693
=> Iv, = -,.- = 5724 years
=>
R, N, (t,,), 4 25 100
-=-x--=-x-=-
(b) Further applying, R =R,e-"
R, N, (t,,), 1 14 14 ·

==> Ro = Reu =(7.3)e(1.21-10.:.i)(45ool => R,=~~~x3=2.63mCi


=> R0 = 12.58 dismin- 9- 1 1
and R, =3-2.63=0.37 mCi
After 60 days, we have
10
N=!:!,_ 5x10:-3x3.7x10 _x , _0.693,.60
11. 3 2 10 5 R; =R,e-i..1 = (2.63)e 14 = 0.135 mCi
A ( 0.693 )
0.693 60
138x24x3600
R; = R2 e-i..21 =(0.37)e--g-~ =0.07 mCi
=> m=( 3.2x10" )(2 lO) So, total activity is
6.02x10" 9
R' =R; + R; = 0.205 mCi
=> m:=1.12x10--a ~

i::::::=================================== 4:103 =
www.puucho.com
Rahul Sardana with www.puucho.com

1. The reaction is possible (spontaneously) if th!3 binding energy of E 4.2x10' k


products is larger than that of 2 :u . 4. m=
c' (3x108 ) ' g
(a) Total binding energy of •::u=238x7.57=1801.66 MeV ::::- m =4.7x10-14 kg
2
and binding en~rgy of :Pb=206x7.83=1612:981 MeV
5. By Law of Conservation of Linear Momentum, we haVe
the binding energy of products is less than that of 2:U .
M.. v.. =mv
Note that the protons and neutrons are free and do not
have any binding energy. Hence, this reaction is not => v = M.. v..
possi~I~ spontaneously. It can take place only if M
1801.66-1612.98=188.68 MeV of energy is supplied Kinetic energy of Tl atom Is

(b)
from outside.
In the second case binding energy of products is larger (KE) =.!Mv' =.!M(M;v;)=(.!M
Tl2 2 M2 2aaM
v')(M;}
than the binding energy of the parent nucleus. Hence,
reaction is possible spontaneously. => (KE),, =(KE).(~)

2. Since, the masses are given in atomic mass units, so, we shall 4
first calculate the mass difference between reactants and
=> (KE),,= 6.082x MeV
208
products in the same units and then multiply the mass difference
=> (KE),, =0.1308 MeV
by 931.5 Mevu-1 • Thus, we have
0 =(14.00307u + 4.00260u -1.00783u -16.99913u) 6. Please observe that here, atomic masses are given (not the
nuclear masses), but still we can us~ them for calculating the
(931.5M~V) mass defect because mass of electrons get cancelled both
sides. Thus,
=> O=-1.20MeV
Mass defect <Im= (22.9945-22.9898) = 0.0047u
Q value is negative, so the reaction is endothermic.
Hence, the minimum kinetic.energy of a-particle to initiate this => o =(0.0047u)(931.5 Mevu-')
reaction is give_n by => 0=4.4 MeV
Hence, the energy of beta particles can range from O to
Km,, =loi(:: + 1) 4.4 MeV.

=> K =(1.20)(4.00260 +1)


"" 14.00307
7. o =Jm; ~(m; +m,)Jc'
=> Kmtn =1.54 MeV where m~ and m~ are the nuclear mass of 11
c and 11
8 , so
m8 = m~ +Sm 0
3. Total energy released is
me =m~+6m8
1
tlE = - -x6.02x1026 x185x 1.6x10-13 J = 7.58x 1013 J
235 => O=[(m,-6m,)-(m,-5m,+m,)]c'
Power P =100x106 =109 Js-1
Therefore, time => 0=[mc-m8 -2mp]c
2
{-: (~:::~)=(; 0:~::~)}
7 5 1013
I= · ~~9 sec=B.78 day => 0 =(11.011434-11.009305 -2 X 0.0005486)931
=> 0=0.961 MeV
The disintegration energy is equal to the maximum energy of the
emitted photon.

= ==================================
4.104

www.puucho.com
Rahul Sardana with www.puucho.com

Atomic & Nuclear Physics

P.;' =Pi +P~ +2P0PHCOS$


8. .6.m = 2(mass of 1H2 )-(mass of 2He 4 ) =0.0256u
2mctKa =2moKo + 2mHKH
=> <iE = 0.0256 x 931.5 MeV = 23.85 MeV · ~-~--~-
+2)(2m,K,)(2m,K,)cos$ ... (1)
Total energy required per day is
E = 200x10 x24x3600 J =1.728x 10
6 13
J
Let m be the mass of deuterium required. Then energy required
for reactor is
5 2
E' = ( ~ )( m~ )<6.02 X 10") (23.85 X 1.6 X 10-")
1 0
This should be equal to 1.728 x 1013 J
4 X 1QQ X 1. 728 X 1Q13 By Law of Conservation of Energy, we have
m 25x6.02x10 23 x23.85x1.6x10-13 g K,=O+K -K,=(-1.18+7.7-5.5) MeV
0

m=120.35g => K 0 =1.02 MeV

9. (a) Total number of atoms in 1 kg of U238 is


1
N =--x 6.02x1026 =2.53 x10 24 $~43°18'
238
=> Total energy released, <iE = (200 x 2.53 x 10") MeV
11. Since, .6.E=(8mP+Bmn-m 0 )x931.5 MeV
=> LlE=8.09x10 13 J
=> ;.E = 127.6 MeV
(b) m 8.09x1013 =2.7x109
30x10' g g
=> m=~.7x10 6 kg

10. (a) Q-value=(<im)931.5 MeV=-1.18 MeV


(b) By Law of Conservation of Linear Momentum, we have

c:=================================== 4.105 =
www.puucho.com
Rahul Sardana with www.puucho.com

1. [BJ
Let N be the number of nuclei at any time t . Then
dN =200-).N A= 0.693
dt 3.8
N dN I

f200-AN fdt Since _!i.=J_


N, 10
' ' 1 ~{
N= 200(1-e-•) - = e 3.s
). 10 --
Given-thatN=100 and ).=1s-•
log10= 0.6931
=> 100=200(1-e-') 3.8

=> .-, =(i) 2.3= 0;69\


3.8
=> t = log, (2) s t =12.62 days

3. [CJ 8. [DJ
Three half-lives of A is equivalent to six half-lives of 8 . Hence,
AA=..!_
we have
.. 2
Probabilities of-getting a and p-pa.rticles are same. So, rate of
disintegration are equal.
=> A.ANA = ABNB
=> NA =Ae =2
NB 'A.A 9. [AJ
nh
4. [DJ mvr=-
2•
= µoln
B
n •2rn => -"--=
mv
(2nr)
n
h -
where, - =de-Broglie wavelength
mv

10. [AJ
Fraction of nuclei which remain undecayed is
f=_.!!=Noe-11 =8-11
No . No

f=e -(
~)(!)
T
-1.,_,,,
2=02

f=-1-=_1
e'og../2 ../2.
5.

=4-106 ====================================
www.puucho.com
Rahul Sardana with www.puucho.com

Atomic & N11clear Physics


11. [CJ 17. [AJ
Since, EK. < EK, According to Ritz Combination Principle
{form< i < n}
=> AK.. > AK,
e.g., v 4 ....1 =v 4 .... 3 +v 3 ....1 or
12. [DJ Y4 ....1 =V4 .... 2 +v2 .... 1

Since, ,l,l.=l.,. -l... . .. (1)


19. [AJ
When V is made half, Amin becomes two times, however AK.
remains the same. ,:=R,[n;- (n~1)']
=> I::,.'),.,' = AK. - 2/1.mln
From (1 ), we have
.!_=R [ (2n+1) ]
A. H n2(n+1)2
Amin = AK.. - /::,,.'),,.
2 2
=> tit...'=AK. -2AK,, +2AA !-R
)..-
n {··• forlargen, 2n+1=2n n2(n+1) =n4 }
Hn4

=> tiA'=2M-AK. 1 2
=> M'<.2LU
i = Ra;;,
v=~=2RHc
13. [BJ l. n'
Assuming that ionization occurs as a result of a completely
inelastic collision, we can write 21. [BJ
mv,=(m+m")V M _v
where m is the mass of incident particle, mH the mass of ,:-=+c
{-) sign to be used for approaching
hydrogen atom, v 0 the initial velocity of incident particle and V
the final common velocity of the particle after collision. Prior to
collision, the KE of the incident particle was =>
,l,l.
l.
706-656
(+) sign to be used for receding

656 C V
+-
E _mvo
,- 2
' => V=2x10 7 ms-1

The total kinetic energy after collision


2 2 23. [DJ
E=(m+mH)V m v~ Let R0 be the initial activity. Then
2 2(m+m,)
The decrease in kinetic energy must be equal to ionization
R, =R0 e-At, and R2 = R0 e-i.12
energy, so we have R2 = e1,.(11-1,J
E; =E, -E =(~)E, m+mH
R,
R2 =R1e1.,(1i-1,J
=> §..=~
E0 m+mH 24. [DJ
i.e., the greater the mass m , the smaller the fraction of initial R,=AN1
kinetic energy that will be used for ionization.
N _R,
i-T

N - R2
' - l.
Time lapsed
4 So, number of atoms decayed is
T,o N=N1 -N2
Ti~e lapsed =400 µs
=> N=(R't')
16. [CJ
E=mc2 25. [BJ
After two half lives ..!.th fraction of nuclei will remain undecayed.
4

Or, ~th fraction will decay. Hence, the probability that~ nucleus
Since "!... « 1 4 .·
C

2 decays in two half lives is ¾·


=> E=m 0 c (1+;;2 )

================================:::.::4.107=
www.puucho.com
Rahul Sardana with www.puucho.com

Advanced JEE Physics Optics & Modern Physics

26. [CJ
The necessary centripetal force to the satellite is provided ·by the l = _.!_
9).
gravitational force of attraction.
mV2 GMm 33. [BJ
i.e. -r-=~r,-
).=~
.!mv' = GMm = KE eV,
2 2r
6,626 X 10-24 X 3 X 1Q 8
PE of the satellite is given by ).
( 20x10' )( 1.6x10-")
PE=-GMm
r ).=0.62 A
Now KE+PE=E 0
35. [CJ
=> GMm _ GMm =E
,2r r 0 0.693
Ax Av
GMm=E
2r ' ;::::) Av >Ax
or _ GMm =2E Since, (- dN) =).,N
r ' di '
Which is equal to PE.
And (-dN) =).,N
dt y
27. [DJ
II.= 21tr
where r = Radius of first orbit ~ 0.53 A
=> (-:),>(-:),
Decay rate of Y > Decay rate of X
=> ). = 2n(0.53) A= 3.33 A
39. [DJ
28. [DJ
0.000 1 x 1O_, = ( 0.5 x10-" )n'
Since, rn oc n2 2
Given, rn+i -rn =r11_1 Because -~dius of nth orbit is ~ual to}
=> (n+1)' -n' =(n-1)' { r0 =n r0 where r0 =0.529A
=> n=4 ;::::) n2 =1000
;::::) n = 31
29. [A]

S.mce, -=n.-
3h (h)
2rt 21t
42. [A]

n=3
.!oc(Z-1)'
).
13
K =JS_= ·6 =1.51eV
" (3)' 9 => ~=(z,-1)'
A. 2 Z1 -1
or 1
4= (z,;_1-1)'
31. [A] Solving this, we get, 2 2 =6
1240
E, = = 2.25 eV 43. [BJ
550
The continuous X-ray spectrum is shown in figure.
E, = 12400 = 2.75 ev E,_
450
1240
E, = =3.54 eV
350
E1 cannot emit photoelectrons from q and r plates. E2 cannot
emit photoelectrons from r .
Further, work function of p is least and it can emit
photoelectrons from all three wavelengths. Hence magnitude of
-+---'---~---).
)...
its stopping potential and saturation current both will be
maximum. All wavelengths > "-m1n are found, where

32. [DJ· ).m;, = ;2375 /


V in volt
N1 =Nae:1011
Here, V is the applied voltage.
N2 = N0e-;.1
44. [BJ
~ = ..:!. = 0 c-10J.+).Jt
N, e Using N = N0 e-u where A= 1090
2
= 1090 { 2 )
9).t = 1 ~" 3.8

= ==================================
4.108

www.puucho.com
Rahul Sardana with www.puucho.com

Atomic & Nuclear Physics


N rog.(2)t
9750 2 3
_Q_=Ne s.s A.=..! x 2.310910 = · = 0.461 min-1
20 ° 5 · · 975 5
Solving this equation with the help of given data we find
t = 16.5days. 53. [DJ

45. [DJ
The binding energy is numerically equal to the kinetic energy of
~ =(i)'
the electron, so N

1
-mv ' =E1 ... (1)
2
54. [CJ
Since, mvr = nh ... (2) Since nuclear density is independent of mass of nucleus, hence
2.
all possess equal density.
Dividing Equation (1) by Equation (2), we get
_y_ = 2E1 56. [CJ
2rcr nh If R0 be the initial activity of the sample, then R1= R0e;.li and
f = 2E, R2 = Roe-)J2
nh

46. [BJ
where A.= i {·.- Mean life = T =.!}
).
Magnetic moment e R -;.12
~=!__=e;.(1,-12)
Angular momentum 2m R1 e-;.t,
=> Magnetic moment cc Angular Momentum
=> Mccn /·.- L=n;.)
R, =R, exp e, ~t,)

58. [DJ
47. [CJ
Number of radionuclei become constant, when Rate of decay of sample 1, Ri = A.1N1e-;.,1 and for sample 2,
X =l.N R2 = "-2N2e-;.,1 .
N=.". When they are mixed, we have R = R1R2
).
=> R =: "-1N1e-;.,1A.2N2e-'-:!'
=> R = A.1N1A.2N2e-(i..,+l2J1

59. [DJ
• h
N=-X-=~ L=JnCn+1)-
log, (2) log,(2) 2,
y
=> L=v'S6(2:)
48. [BJ
Cut-off wavelength depends on the applied voltage not on the 60. [BJ
atomic number of the target. Characteristic wavelengths depend
on the atomic number of target. ~ = n2Ro

=> R, = 4( 0.528 A)
49. [CJ
Since decay is to be regarded as a statistical spontaneous => R,=2.112A
process, hence a. decay can be regarded as an Adiabatic
process. 61. [CJ
Speed of electron in the nth orbit of H like atom is
50. [BJ
Total initial binding energy is 236x 7.6 V0
1(cZ)
=137 n
=> U, = 1793.6 MeV
Total final binding energy is 2( 117)( 8.5) 62. [CJ
=> U, =1989 MeV Since Lip=!!
).
=> <l.U = 195.4 MeV
=> mv=hR(!--1...)
H H 1 25
51. [CJ
R = R0 e-;.1 V=hR,(24)
mH 25
=> l.=!lo (R')=2.31o (Ro)
tg"R tg"R v=4 ms-1
Decay constant

=================================== = 4.109

www.puucho.com
Rahul Sardana with www.puucho.com

· Advanced JEE Physics Optics & Modem Physics

63. [CJ
As velocity (or momentum) of; electron is fncreased, the vHa". =a(c(22 ))=ac

wavelength (A.=%) will decrease.


Hence, fringe width will decrease ( ro oc A).
72. [AJ
64. [DJ Momentum of striking electrons
In beta decay, atomic number is increased by 1 whereas the h
mass number remains the same. Therefore, following equation p=-i::
can be possible Kinetic energy of striking electrons
~Cu 4:zn+_1e0 p2 h2
K=-=--2
2m 2mA.
65. [CJ This is also, maximum energy of X-rays photons.
Activity R = ,N he h2
Number of nuclei (N) per mole are equal for both the ~= 2mA.2
substances, so we have
R oc, ~ ,._ _ 2mA.2c
0 - h
=>
73. [AJ

66. [CJ Activity of s, = .:!. (activity of s, )


2
R(J...-J...)=R (_!l_nf __!l_)
H 22 42 n!
. H => ,,N, =21 (,,N,)
Substituting n1 = 6 , n2 =12 makes both sides equal
A., N2
=>
"-2 = 2N1
67. [BJ
a=--
v' =>
T,
T2 = ~
2N 1
IT =half life = 10 ~ 2 )
r
(z)' Given N1 = 2N2
=> {torn= 1)
aoc (l) ~ T, =4
T,
=> a oc z3
74. [AJ
=> ~=(~)'
a,
=8 1
Rest mass of parent nucleus should be greater than the rest
mass of daughter nuclei.

68. [DJ 75. [BJ


Net rate of formation of Y at any time t is Total power drawn by Coolidge tube is VI .
~ P1o1a1 = VI =200 W
dN, N
dt= A. X -A.YNY Since only 0.5% is carried by X-rays, so
. . dN
NY 1s maximum when rif =0 P=u~~)P• .,
1
=> AXNX = A.YNY => P=-(200)W
200
=> P=1W
69. [DJ
Maximum wavelength of Lyman series will correspond' to the
76. [BJ
transition of electron from n = 2 to n = 1 and maximum
wavelength of Paschen series will correspond to n·= 4 to n =3. e2 cz

=> !:,. (¾-~) 7


., n-¾) 108

71.
e' 78. [CJ
where u = - -
2he0c Total energy radiated per minute from sun is
E,adlate<:t = er( 41tR~)
is the fine structure constant ( u = ~ ) Energy radiated annually is given by
1 7

=4.110 t:::===================================
www.puucho.com
Rahul Sardana with www.puucho.com

Atomic & Nuclear.Physics


Etotal = 24 X 60 X 365 X Erad1ated 87. [AJ
Charge number decreases by 4, but actually it must decrease by
E1
=> Annual loss of mass =.6.m=
C
f 1
=1.38x1017 kg 8 (due to emission of 4a-particles ). Hence 4p particles must
have also been emitted.
80. [CJ .
88. (CJ
If it was A
Y-;22 .1.=R
A
(.1.-J...)
" 4 16
Reactant: R=60x8.5=510 MeV
Since c= v"A.
Product: P = 2 x 30 x 5 = 300 MeV
C
6E =-210 MeV V=-
A
ENDOTHERMIC
If it was B V::: 3 X 10
8
X 10
7
X( / )
W->X+Z 6
R = 120x7.5 = 900 MeV v=J!._x10 15 Hz
P = 90x 8+30 x5 = 870 MeV 16
6E=-30MeV
89. [CJ
ENDOTHERMIC
If it was C
W-;2Y
E=R• hc(1-J...)
25
R = 120x7.5 = 900 MeV Momentum of photon emitted is
P = 2x 60 x8.5 = 1020 MeV
p =.§_=R
C •
h(24)
25
6E=120 MeV
Recoil momentum of H-atom will also be p.
EXOTHERMIC
mv-=p
If it was D
X-> Y +Z p (1.097x10')(6.626x10-")24
V=-
R = 90 x 8.0 =720 MeV m (25)(1.67x10·")
P = 60 x 8.5 + 30 x 5.0 = 660 MeV 1
=> V=4.178ms-
6E=-60 MeV
ENDOTHERMIC 90. [BJ
81. [DJ s·mce E =-n2e
13.6 V
f=6m=M-A
A A => E1 =-13.6 eV
=> E, =-3.4 eV
83. [AJ
=> E, =-1.50 eV
E=mc2
=> E, =-0.85 eV
=> E=(10-")(9x10")
From above we can see that
=> E=9x1010 'J E3 -E1 =12.1 eV

84. [CJ i.e. the electron must be making a transition from n = 3 to n = 1


level.
N ( -1 )"
Since-=
N, 2 => 6L=(3-1)_11_='1_
2, •
Time lapsed
where n 6L 6.626 x 10-"
Half life
3.14
Tv, => .6.L=2.11x10-34 Js
According to the problem n = 2 - = ..:!.
Tv2 2 91. [AJ
For the fifth electron to act as a dopant it must lie in the valence
shell of P.
So n = 3 . (TRICKY HINT!)
Z = 15 (for Phosphorus]
86. [AJ
. n2h2&0 n2
M=235-219=16 Smee rn = - - , - = - r0
1tme Z Z
Number of a-particles emitted = ~= 4 where r0 = 0.529 A= 52.9 pm
4
9
=> r, =
15 (52.9 pm) ... (1)
When phosphorus acts as dopant in silicon, the expression for
Bohr radius is

c:=================================== 4.111 =
www.puucho.com
Rahul Sardana with www.puucho.com

Advanced JEE Physics Optics & Modern Physics

, n2h2E 98. [BJ


[ =--=kr: h
n ·1rme2z n

r~ =12rn {·: &=k<,) ~= ../2mE or !:l.a:E112


A2 he i.,
=> r; =12r, =12[ 1~(52.9)pm] E

=> r; = 380.88 pm 99. [BJ


Since K.E. = -T.E.
92. [CJ
=> K.E.=+13.6eV
The first photon will excite the hydrogen atom (in ground state}
100. (CJ
in first excited state (as E2 -E 1 = 10.2eV ). Hence, during de-
excitation a photon of 10.2 eV will be released. The second En =-me'
~,m = mass of electron
Bn h E0
photon of energy 15 eV can ionise the atom.
mhypc!hetical e4
Hence the balance energy i.e., {15-13.6) eV=1.4 eV is => E =
n 8n2h2 s~
retained by the electron.
Therefore, by the second photon an electron of energy 1.4 eV E = _2(Rhc)
=>
will be released. " n'
The longest wavelength (or mmImum energy) photon
93. [DJ corresponds to transition between adjacent states.
Number of nuclei decreases exponentially i.e. n=3 to n=2
N =N0 e-.u and
=> ~=E 3-E
2 =-2Rhc(.!_.!)
Amruc 94
Rate of decay ( - ~~ ) = J.N
~=2Rhc(~)
Therefore, decay process lasts upto t 4 co Therefore, a given Amruc 36
nucleus may decay at any time after t = 0.
=> ). =~
= SR
94. [CJ
mvr = n~ =
2rt
2(_11_)
2.t
102. [BJ
According to Ritz Combination Principle
h Ec ... A =Ec...a + Ea...A
~ mvr=-
n he he he
So, de-Broglie wavelength is -=-+-
A3 A. 1 A. 2
i. = _t,_ = nr = (3.14)(2.116 Al= 6.64 A
mv

95. [CJ
Following nuclear reaction takes place 103. [DJ
1 0 For each principal quantum number n, number of electrons
0 n 4,H1 + _
1e +V
permitted equals the number of elements corresponding to the
V is antineutrino. quantum number.
=> (Total Number)= '-'2n' 'n(n+1)(2n+1)
96. [DJ of Elements L, 3
Shortest wavelength will correspond to maximum energy. As
value of atomic number (Z) increases, the magnitude of energy
104 [BJ
in diffetent energy states gets increased. Value of Z is
maximum for doubly ionised lithium atom (Z=3) among the
inR3 a:A
3
given elements. Hence, wavelength corresponding to this will be
least. => Ra:A 3 '
97. [DJ 105. [CJ
• n2 h2
E
The nuclear force of interaction between any pair of nucleons is
Smee, r = -'-,- identical i.e. force between two neutrons (F2 ) equals the force
e ,m
between neutron and proton ( F3 ) • However, between two
L=n(;J protons net force is equal to the resultant of nuclear force
=> nh=2•L between them (attractive in nature) and electrostatic force
between them (repulsive in nature). Hence F1 is a value lesser
e, (2,L)'
=> r- 2 than F2 and F3.SO F,<F2=F3.
e 1tm

=> '
Lr-2 = constant

=4.112 c:::::===================================
www.puucho.com
Rahul Sardana with www.puucho.com

Atomic & Nucleilr Physics


107. [C] 113. [BJ
dN=n-AN Given that K, + K, = 55 MeV ... (1)
di From Conservation of Linear Momentum,
Because the population N is simultaneously increasing at rate n P1 =P2
and decreasing due to decay at rate AN.
=> ~2K,(216m)=~2K,(4m)
=> f~=fdt
Non-AN 0 as P=-/2Km
=> K, = 54K, ... (2)
=> .!.en(n-ANo)=t
l. n-l.N Solving equations (1) and (2), we get K, = KE of ex- particle
=> N=~+(N
l.
-~)e-"
' l.
=5.4 MeV.

OBJECTIVE TRICK 114. [DJ


At t = 0, N = N0 which is satisfied by (C) only. ~ =AN 1 and R2 =AN2 ( A is same for a given sample)
As N2 <N1
108. [Bl
I= l 0e-µx Number of atoms disintegrated in time ( t2 -t,) is

Where 10 is intensity at x =O, N1 -N2 =R,_R,=(R,-R,)


A A A
I is intensity at a distance x and
=> µ is absorption coefficient; [µ] = L-1 N -N (R,-R,)T oc(R -R )T
1 2 0.693 1 2
µ is maximum for lead as lead has maximum ability to absorb
radiations in a minimum distance. 117. [DJ
1
109. [Bl E= k,T
2
l.= he at T=300K
E
( 6.626 x10_.., )( 3 x10') E = -½< 1.38 x 10-" )(300)
=> l.
1.6x10-" E{in keV) E= 1 (1.38x10-")(300)eV
=>
,___ 1.242x10...g m 2 1.6x10·19
=> E=0.01 eV
E(in keV) =>
,. 12.42 A 118. [DJ
=>
E(in keV)
E =-13.6 eV
n'
110. [D] => E2 = -3.4 eV
Z'
E, =,(13.6eV)
n 120. [DJ
=> E,=9(13.6eV) Expected mass of Cu must be less than that of zinc. So it is
unstable and radioactive, decaying to Zn through p decay.
=> En= 122.4 eV
121. [BJ
111. [C] ~ is the fraction decayed in one half life Tv 2 and t is the
,."- = 0.021 nm= 0.21 A
fraction decayed in one mean life ( T) .
Since, A.Ku corresponds to the transition of a'l electron from
L- shell to .K - shell, therefore,
T,, = 0·693
l.
= 0.693T

-(· V)- 12375 _ 12375 _ 58928 V


E,- E,- => T > Tv2
rne -l.(inA)- 021 - e
Further, since the fraction decayed is ( 1-e-u)
=> liE ~ 59 keV So,

112. [D]

(rm)=(:')( 0.53 A)= (nx 0.53)A t =1-e-i.r =1-e-1 =0.632


m' => t >I
-=n However, if we had· been asked about the fraction of sample
z
undecayed, then we must give our answer in the light of formula
m = 5 for ,00 Fm 257
(the outermost shell) and z = 100
~ =e-lt where N is number of undecayed nuclei in sample at
_(5)'_1 N,
n-----
100 4 time t.

====================================4.113=

www.puucho.com
Rahul Sardana with www.puucho.com

Advanced JEE Physics Optics & Modem Physics

124. [DJ 130. [CJ


Since we assume the potential energy to be zero in the ground
V=J2~V state. So,
Total Energy=U+K=Kinelic Energy (·.- U=0)
125. [AJ => Total Energy= 13.6 eV (in ground state)
Series limit (i.e. shortest wavelength) of Lyman implies transition If the potential energy is not assigned a zero value, then total
from oo---+ 1 energy is -13.6 eV.
First line (or longest wavelength) of Lyman series implies So, we conclude that making potential energy zero increases the
transition from 2---+ 1 value of total energy by 13.6-(-13.6) = 27.2 eV.
Series limit of Balmer implies transition from oo ---+ 2 Now actual energy in second orbit =-3.4 eV
-- --- - - - , - - - - - - - - - - - 00 Hence new value is (-3.4+27.2) eV = 23.8 eV

131. [BJ
---t--+--+-----n=3 13 6
Since, E = - ·
_,,-~-+------n=2 n'
. -34-_ 13.6
=> • - n2
~--~~----n=1
=> n=2
According to Ritz Combination Princip_le
E.,, ... 1 = E""...2 + E2_.1 Since, A=_!:_
mv
=> hv1 ·= hv3 + hv2 Velocity of electron in second orbit is
C C
v, = 2(137) = 274
126. [AJ , = 6.626 x 10"'
=>
(9.1 X 10-'"l(
3 10')
X
:, =R[¾-¾] 274
=> A - 6.6 x·10-10 m
_!__ = 4R[_! -
,, 4
2.J
16 Further Kinetic Energy =-(Total Energy)
=-> E = -(-3.4 eV) = 3.4 eV
A. 2 =~
"-1 27 133. [AJ
Shortest wavelength of Brackett series corresponds to the
=> ,,=121sA
transition of electron between n1 = 4 and n2 4 co and the

127. [BJ shortest wavelength of Balmer series corresponds to the


transition of electron between n1 = 2 and n2 4 co • So,
~Ne-----+2;He+~ X
=> A =14
(2'>(1;~6)=(1~6)
=> 2=6
i.e. ~4 C is the unknown nucleus. => z2 =4 or Z=2

134. [AJ
128. [CJ
The longest waveilength in a series is obtained when a transition
1 2 takes place between the lowest consecutive levels. Here
En oc 2 and r" cc n
n transition must take place from n = 2 to n =1
So, Eln is independent of n
135. [BJ
=> E,r, = (13.6 eV)(0.53 A) Maximum angular speed will be in its ground state,.so, we have
=> E,r,=7.2eVA, v, 2.2x106 ms- 1
ro =-=
max r1 0.529 X 1 o-lO m
=> El1 = constant
=> ffimax =4.1X10 ra"dS-
16 1

129. [CJ
t = 0.693 136. [BJ .

T=_!
' he =Rhc(1-_!_)
, n'

' => n- ~
=> t=0.693 T \/'i:R-1
=> T>t

4.114 c==================::::=::::======================
www.puucho.com
Rahul Sardana with www.puucho.com

Atomic & Nuclear Physics

137. [BJ 145. [AJ


The maximum number of electrons in an orbit are 2n 2 • lf n > 4,
From R=R,(iJ is not allowed, then the number of maximum electrons that can
be in first four orbits are:
Wehave. 1=64(iJ 2(1)' +2(2)' +2(3)' +2( 4 )' =2+8+18+32=60
or n = 6 = number of half-lives. Therefore, possible elements can be 60.
t=nxt,12 =6x2=12 h
146. [DJ
When one electron is removed, the remaining atom is hydrogen
138. [AJ like atom whose energy in first orbit is
Penetrating power is maximum for r - rays, then of p- particles
E, =-(2)'(13.6 eV) (2=2)
and then a - particles because basically it depends on the
velocity. However, ionization power is in reverse order. => E, = -54.4 eV
Therefore, to remove the second electron an additional energy
139. [CJ of 54.4 eV is required. Thus, to remove both the electrons
j3 and y - decay take place from a radioactive nucleus. (24.6 + 54.4) eV = 79 eV energy is required.
Hence, options· (A) and {B) are wrong. During fusion process two
or more lighter nuclei combine to form a heavy nucleus. 147. [CJ
During fusion, binding energy of daughter nucleus is always
140. [AJ greater than the total binding energy of the parent nuclei. The
Activity of a radioactive substance R = AN difference of binding energies is the energy released. Hence,
0=E2 -2E1
=> 1.=!1_
N
Since, R = N2 particles per second 148. [BJ
Shortest wavelength or cut-off wavelength depend only upon the
and N=N, voltage applied to the Coolidge tube.
A= N2
N, 149. [CJ
Beta particles are fast moving electrons which are emitted by the
nucleus.
142. [BJ
Heavy water is used as moderators in nuclear reactors to slow 150. [AJ
down the neutrons. Maximum number of nuclei will be present when
Rate of decay= Rate of formation
143. [AJ
=> AN= a.
U =eV =eV,log,( t) N=~.
1.
lfl=[- dU[= eV,
dr r 152. [BJ
This force will provide the necessary centripetal force. At time t = o, A gets converted to B which has a higher
mv eV0 2 activity. Therefore, activity increases.
Hence, --=-
r r
153. [BJ
=> v=l!F ... (1) P(survivaO = N(t) = Noe-)J = e·~t
No No
Moreover,
nh 1
For, t=-, we have
mvr=- ... (2)
2•
Dividing equation (2) by (1), we have '
P(survival) = .1.e
mr- ( -n h )
-~
- 21t eV0
154. [DJ
Energy released is given by
Total Binding ) (Total Binding)
144. [DJ 6E= ( -2
Energy of 2He4 Energy of ,H2
Activity reduces from 6000 dps to 3000 dps in 140 days. It
implies that half-life of the radioactive sample is 140 days. In
=> 6E=(4)(7)-2(1.1)(2)=23.6 MeV

280 days (or two half~lives) activity will remain ..:!.th of the 155. [AJ
. 4
n(n-1)
initial activity. Hence, the initial activity of the sample is Number of possible emission lines are - -- when electron
2
4 x 6000 dps = 24000 dps
jumps from nth state to ground state. In this question this value
is

====================================4.115 =
www.puucho.com
Rahul Sardana with www.puucho.com

....,_
Advanced JEE Physics Optics & Modim ., I

N= (n-1)(n-2) 162. [DJ


I
2 On the last day we have 100% decay i.e. on the ninth day 50%
decay must be there or 50% must be left.
(n-1Hn-2)
=> 10
2 167. [CJ
Solving this, we get ,H2 + ,H2-------+ iH3 + p
n=6
2 3 4
1H + 1H -------+ 2He + n
156. [DJ :::::,. 3 1H2 ~ 2He4 + p + n
Let AA = A. and A8 = 2A
<im = m( ,He')+ m(p) + m( n)-3m( ,H')
Initially rate of disintegration of A is AN0 and that of B is .
=> <im = [ 4.001 + 1.007 + 1.008 -3( 2.014)] u
(2l.)N0 •
=> t,m = -0.026 u
After one half-life of A , rate of disintegration of A will becomes
t"'-;.:r:AN -,
0 and that of B would also be -
AN 0 => l;,El=c'l;,mJ
• - , because
"· 2 2 => ,iE = ( 9 x10")( 0.026 x1 .67x1D"")
(Half-life of B) =½(Half-life of A) => t,E = (931.5)(0.026) MeV.
So, after one half-life of A or two half-lives of B , we have => .6.E = 3.87x10-12 J
As each reaction involves 3 deutrons, so total number of
(-~~). =(-~~). reactions involved in the process
1040
If each reaction
~ n=1 3
produces an energy .6.E , then
157. [AJ 40
10 28
Saturation current is proportional to intensity while stopping E1o1a1 =-.6.E=1.29x10 J
. 3
potential increases with-increase in frequency.
E1ota1 =Pt
Hence, f3 = t while Ia < lb
Time of exhaustion of the star
158. [DJ t 1.29x10"
First excited state is n = 2 and second excited state is n = 3 . 1016
Also, :::::,. .t=1.29x1012 s
Ea::_!_
" n' 168. [AJ
E, 9 l=.9.=ne
=> E3 =4 t t

159. [BJ
=> 3.2 X 10-" =(-'/-){ 1.6 1o-")
X

_!_=Rs(.!-.!)
l., 4 9 => ('t:)=2x10"
5
l., 36 5 169. [BJ
l., = 3 = 27
4
_!=Z'R
A
(_l __!_)
"'n~n~

160. [BJ For K"' line, n1 =1 and n2 =2


Second excited state implies n = 3
=> _!
l.
= Z'R• (~)
4
=> L,=n(;,)=3(;,)
=> 2=~3l.~.
=> Lu =n(;.)=3(;,)
=> 2=39.9=40
Z'
JE,J=--,.(13.6) eV 170. [AJ
n
1' . Since in Rutherford experiment
=> JE,J= (13.6) eV 1
9 Noc---
3' sin'(!)
=> JEuJ= ,(13.6)eV
3
N90 sin4 ( 30)
=> JEul=13.6 eV =>
N60 sin" ( 45)
=> IEu I= 9IE,J
=> IEul>JE,J

=1.116c====================================
www.puucho.com
Rahul Sardana with www.puucho.com

Atomic & Nuclear Physics

180. [A)
N., 16 .!!! = 4 X 109 kgs-l
=>
N50 =T t
4 E=mc2
=> N60 =100

171. (CJ
=> f )c'
=Uf

Total Energy=-{K.E.)=-13.6 eV => _§_=4x10 9 x9x1016


t

172. (CJ => _§_=3.6x10 26 Js-1


2
t
For quantum number n we have 2n electrons.
For n =1 we have 2 elements => _§_=3.6x10" W,
For n =2 we have 8 elements t
For n=3 we have 18 elements
181. [BJ
For n =4 we have 32 elements
So, total number of elements is A(1.67x10·")
p
2+8+18+32=60 inR3
·3
173. [DJ
R., a: n2
'
Since, R = R0 A 3 where A0 = 1.1 fm
A(1.67x10"")
:::::, An ocn-4 => p
Hence _±nA3A
3 0

A2 n~ 16
=> p = 2.9 X 10 kgm-317
Ai= n~ = -

183. [DJ
174. [AJ
A0 =~c =1.55 x 10-11 m
±=Rs(¾-¾)
=> l.,=0.155A
=> l.=~=6563A is the minimum wavelength of continuous X-rays which carry
5R,
energy equivalent to energy of Incident electrons.
As this energy of incident radiation is more than that of K-shell
175. [BJ electrons, the characteristic X-rays appear as peaks on the
For an elastic collision to take place there must be no loss in the continuous spectrum.
energy of electron. The hydrogen atom will absorb energy from
the colliding electron only if it can go from ground state to first 184. (AJ
excited state i.e. from n = 1 to n =2 state. For this Hydrogen Ac decreases with increase in accelerating voltage in
atom must absorb energy
E, -E, = -3.4-(-13.6) =10.2 eV. accordance with the expression given by

So, if the electron possesses energy less than 10.2 eV it would


l. _ he
c - eV
never loose it and hence collision would be elastic.
Wavelength for K 11 line is not affected as it is due to the
177. [CJ electronic transition between n = 2 and n = 1 in the target
h element. Hence ( AK - Ac ) increase with increase In the
L,=(1)- ... (1)
2n
accelerating voltage.
{Using Bohr's Quantisation Rule)
In the first excited state of Li 185. [CJ
h
L, =(2)- ... (2) ~ 1
2, - - ;,. Fine structure constant (a) = -
2he0C 137
=> L2 =2 and !,d/M0L'T'
L,
187. [BJ
178. (BJ
The series in U-V region is Lymen series. Longest wavelength
N=N0e-" corresponds to minimum energy which occurs in transition from
1 n=2 to n=1.
Fort= Tav =I, we hav:> 1
=> 122 = ( /R ) ... (1)
i\l= No
e
¾-d,
Th8 smallest wavelength In the Infrared region corresponds to
maximum energy of Paschen series.

==================================== = 4.117

www.puucho.com
Rahul Sardana with www.puucho.com

Advanced JEE Physics

193. [AJ
•= 1/R ... (2)
, =(2nr) '-,"-,
(_1__..1..)
3' a:, • n
Solving equations (1) and (2), we get ·
'
{·.- r o::·r1~.
1"=823.5 nm

188. [CJ · I·: J• = nh)


2•
'

4(2He4) =a01a
Mass defect, t,m = { 4( 4.0026 )-15.9994} amu = 0.011 amu 194. [CJ
Energy released per oxygen nuclei After absorbing a photon of energy 12.1 eV electron jumps from
=(0.011)(931.48) MeV=10.24 MeV ground state (n =1) to second excited state (n = 3). Therefore
change in angular momentum is
189. [CJ t,L=L,-L,
2 2 4
1H + ,H ------t 2He + Q
t,L = 3 (__11_) - __11_ = !1_
=> t,m=m(;He')-2m(,H') 21t 21t 1t

=> t,m=4.0024-2(2.0141) t,L= B.BxlO""' Js=2.11x10.,.,. Js


3.14
=> .6.m·= --:-0.0258 u
Since, Q = c 26.m 195. [BJ
=> a= ( 0.0258 )( 931.5) MeV A and B can be isotopes if number of p-decays is two times
the number of a-decays .
=> Q =24 MeV

196. [DJ
190. [AJ
ln 2s only 90% nuclei are left behind. So, in the next 2s 90% of
Nuclear density is constant hence, mass o:: volume or m oc V.
900 i.e., 810 nuclei will be left.
191. [CJ
197. [CJ
Let N0 be the number of atoms of X at time t = 0. Then at
Let number of a-decays be x and number of p-decays be y.
t = 4 h (two half lives), we have Then
N =No and N =3No 92-2x+y=85
x 4 V 4 => 2x-y=7 ... (1)
=> .Nx=_! and 238-4x = 210
N, 3 => X=7
and at t = 6h (three half-lives), we have Substituting this value in equation (1 ), we get
N 7N Y=7
N
x
=___Q_
8
and N
V
=-'
8 198. [DJ
NK _ 1
N, -7 4 = constant

The given ratio ¾lies between i and f


Therefore, t lies between 4 h and 6 h 199. [CJ .
Since, t,E=K(-1,---1,-)=hc
192. [CJ n1 n2 A
= AN0e- 1t ln the given diagram, emission from n =4-+ n = 2 would give
1
Since, A,
photon of maximum energy (shortest wavelength) and
=> . 1. 1 ('N,)
t, = A ago A ••. (1) n = 4-+ n = 3 transition would give photon of ITlinimum energy

Since, A, =(:\.)(2N0 )e·",


' (longest wavelength).

=>
_ 1
t2 -i 1 (2AN,)
age ,;:- ... (2)
200. [CJ
Energy of photon is
' E=13.6(1,- ~) eV
=> t, -t, =..!.log,(~) 2
' 2A, => E=13eV

=> t, -t, = log~( 2 ) log, ( ~J Since, momentum is conserved, so we have


E
-=mv
C
E
=> V=-
mc

=4.118 ====================================
www.puucho.com
Rahul Sardana with www.puucho.com

--
Atomic & Nuclear Physics

v- 13x1.6x10-19 _
4 m5-1 1-2 - ~
=> - eV
-1.67x10--27 x3x108
=> !:i.:.! /eV
201. [DJ 1.
2 cV2m
For incident electron, we have
Substituting the values, we get
h h h
'• - p - ,J2mK - ,J2meV
Far shortest wavelength of X-rays, we have
,,
!:1 =1

c=========================================i = 4.119

www.puucho.com
Rahul Sardana with www.puucho.com

1. [B, CJ ,
For a photon => Deflection "' ,!rn
I\
~
pc=hv
hv 4. [A,DJ
=> P=-
c In nuclear fusion two or more lighter nuclei are combined to form
=> p=8.Bx10-2a kgms-2 a relatively heavy nucleu~ and thus, releasing the energy.
=> p·= 1.65x10-6MeVc-1 5. [B, CJ
Let electron jump from n1 --+ n2 •
2. [A, CJ
Since L= nh = 3h So llL =(n, - n, )_I!_
21t 21t 2n
=> n=3
(According to Bohfs Quantisation Rule)
Since, n1 and n2 are integers (n 1,n2 >1),so n2 -n1 is also an
Since, rn =a0 zn' integral value and hence AL must be an integral multiple of

n' _I!_ . This is satisfied by OPTION (C) and OP.TION (8).


=> 4.5a0 = a0 Z 2n

=> 2=2 6. [B, CJ


Possible transitions are 3 --+ 2 , 3 -t 1 and 2 --+ 1 1 1
For 3--+2,wehave
Ucc-oc-
r n'
1 -R(2)'(1 ))=4R(9::4)- 5R Koc~cc_!_
1 4 9 · ~ 9 r n'
=> vcc~ and
n
For 3--+1, we have Locn
i =R(2)'(1-¾)=4R(¾)= 3~ 7. [A, C, DJ
In Bohr's model of Hydrogen atom
=> 1=~ Roe n2
32R
·For 2--+1,wehave V"' .!_
i =R(2)'(1-¾) = 4R(¾) = 3R n

To::n3 and E a: 2
1
n
=> 1=-2.. => VR oc n
3R
=> TEocn
3, [A, CJ T
=> -ocn
mv .J2mE R
r=-=--
qB qB V n
_.,
=>
. Jm E
=> r,oc-
q

=4.12oc::=================================

www.puucho.com
Rahul Sardana with www.puucho.com

Atomic & Nuclear Physics

8. [C, DJ 14. [C, DJ


For ~~Ne nucleus to exist In case of 1H1, mass number and atomic number are equal and
M1 <10mp+10m0 in case of 1H2 , mass number is greater than its atomic number.

M1 <10(mP +mn)
15. [A, B, CJ
Further since ~gca has 20 more nucl~ans and thus it requires Since Bohr's radius is given by
more energy to hold all of them together and hence M1 :;r: 2M2 n2h2£
r=-- '
nme2Z
instead M1 > 2M2 because some additional mass defect must
1
occur to provide an additional B.E. to ~gca nucleus. => ra:-
m
_!_=me
9. r0 mµ
12375
V (in volts)
With increase in V, Am will decrease. With decrease in Am
Since, Fa: m
energy of emitted photons will increase. And hence intensity will
increase even if number of photons emitted per second are
=> E=E,(::)
constant. Because intensity is basically energy per unit area per
unit time. => E = (13.6 eV)(212)
=> E=2883 eV
10. [A, DJ Angular momentum is given as
Toc n3
L=mvr= nh
=> ~ =8 2,
T, For ground state n = 1 , ~o we have
satisfied by both (A) and (D)
L=__tl__
2•
11. [A, DJ
Characteristic X-rays depend upon the atomic number Z , so
16. [A, B, CJ
A,=A 2 =A3
S.mce, -=A
he E
OR ,, =~,,,, 1.
1.=~
12. [A, B, DJ t;E
N = N0 e-~ 1
· he and ,
N = Number of undecayed nuclei in the sample at time t. Amin =:-E
Ll max
= =/1 E~ min
Total number of undecayed nuclei equals (N0 -N) Emission will take place when transition takes place from n = 3
=> (N,-N)=N,(1-e-") to n=1
which is growing exponentially with time.
12400
=> 1. = =1240A=1.24x10-' m
mi, 10(eV)
Activity R =-1.N = dN Absorption will take place when transition takes place from
dt
n=3ton=4.

13. [A, BJ 12400


=> ,
. dU Ke 2
= = 2(eV) =6200A=6.2x10-' m
Smee, IFI =- =-,- ... (1) Lowest frequency photon that can ionise the atom will have to
dr r
remove the electron from n = 3 state, so we have E =15 eV
Ke 2 mv2
=> 7=-,- ... (2) => 15 X 1.6 X 10-19 = 6.63 X 1Q-3 1V
15
According to Bohr's Quantisation Rule, we have => v=3.62x10 Hz
nh The total number of ways of de-excitation of atom to ground
mvr=- ... (3) state is 2.
2,
From (2) and (3), we get 17. [A, B, C, DJ
r= (4•'h2·n2
e'K)!:!_ =K1n2
(m) ... (4)
Since reduced mass is
mm m
µ = -1 -2- = -
Since total energy E is half the potential energy, so we have m1 +m 2 2
Ke2 Ke2 Ke2n6 Now, energy is directly proportional to mass, so
E=--=
6r' 6Kfm 3 E0 oc m

=> Ea:µa:~
So, total energy is Ea: n6 and Ea: m-3 2

====================================4.121 =
www.puucho.com
Rahul Sardana with www.puucho.com

Adva11ced JEE Physics Optics & Modern Physics

E e·(m) (KE) 1 = 14.4x10·" 10 eV =13.58 eV


=>
E, 2 ~-- .... =>
2xO.53x1O-
Since, E0 =R0 hc ,
,' I
'
',
' => (KE)=
2
14.4x10·" eV=3.39eV
So, Rydberg constant is ':' '
'tCM ''
l
2xO.53x10-10 x4
KE decreases by 10.2 eV
E =>
R0 = ----2... a: m
'' '' ,,' 10
he 1 e' 14.4x1O-
....:..===---ev
=> Recµ '
',. . . . 4' . .--.,."', Now PE=
4n&0 r

=> _I:!_=~=- e'(m) (PE)= 14.4x 1o·" eV=-27.1eV


R0 m 2 => 1
0.53 X 1o-lO
Radius of orbit is 10

=> (PE)2 =- 14.4x10· -6.79 eV


n2 h2 e 0.53 X 10-iO X 4
r = - -2 '
nme Z => PE increases by 20.4 eV
So.for n=1, 2=1,wehave
Now Angular momentum is L = mvr = nh
h 2e 1 2n
r =--'-«-
2
0 nme m
h
1 =>
::::) rix- 2n
µ
21. [A, DJ
::::) _!_=~=2
r, µ n'
r0 « Z and IPEl=2(KE)
Velocity of electron in first orbit is
e'
V=-- 22. [A, CJ
2hs,
which is independent of mass, hence for positronium atom and 2
Since, v = CRZ (-;.- ~ )
hydrogen atom velocity of electron is same in both cases. n2 n1

18. [A, B, C, DJ => v,~, =(3x10')(1.1x10')(1)(¾-¾)


Under normal conditions, the total energy (E) , kinetic energy
15 5
(K) and potential energy (U) are => V 3 _,. 2 =(3.3x10 { )
36
E1 =-13.6 eV] 16 5
=> v 3 ... 2 = 36· x1O15 =4 .6x1014 Hz
K1 = 13.6 eV for ground state
U, =-27.2eV Similarly for transition from 3 to 1, we have

E, = -3.4 eV] v,~, =(3x10')(1.1x1o'l(1-¾)


K2 = 3.4 eV for first excited state
U, =-6.8 eV => v 3 .,.1 =2.9x1O 15 Hz

If PE in ground state is taken to be zero, then KE remains Please note that (0) is incorrect as it says "must be" because
unchange~, however new PE and TE are increased by from second excited state i.e., n = 3 , photons are emitted for
27.2 eV. electron transitions from 3 ~ 2 , 2 --t 1 , 3 --t 1 .
So, for ground state, we have
E, =(-13.6+27.2) eV=13.6 eV 23. [A, B, C, D]

K, = 13.6 eV (same) Eo::_!_oc_!_ ... (1)


r2 n2
U,=(-27.2+27.2) eV=OeV
Pocvoc..!. ... (2)
For first excited·state, we have n
E,=(-3.4+27.2) eV=23.8.eV r o:: n2 ... (3)
K, = 3.4 eV (same)
=> PEr oc _!
u,~(-6.8+27.2) eV=20.4eV n
p
=> -« n
19. [B,C,DJ E
Ground state is n = 1 => Er oc n°
So, first excited state is n = 2 => Pr oc n
1 e'
KE=-- (for Z=1)
24. [A, B, DJ
41te0 2r
Actually, in the ground state, we have
=> KE= 14.4x10-10 eV KE=+13.6 eV
2r PE=-27.2eV and
Since, r = O.53n2 A TE=-13.6 eV
In the first excited state, we have

=4.122==================================

www.puucho.com
Rahul Sardana with www.puucho.com

Atomic & Nuclear Physics


KE= +3.4 eV, PE= -6.8 eV , TE= -3.'l eV 30. [B, CJ
Now PE and TE both will be increased by Intensity of incident light may increase.by decreasing distance of
13.6-(-27.2)=40.8 eV source without increasing frequency.
KE remains unchanged being independent of reference.
31. [A, DJ
25. [B, DJ
. 1
Since, E, =-(13.6>(~:) eV
S ,nee, r cc -
m E,=-13.6eV for 2=1 and n=1
and· E0 ?= m Similarly for Z = n , we have
E=E, =-13.6 eV
For an orbit, mvr = nh is a constant, because n = constant
2. i.e., for second orbit of He· and for·third orbit' of u++, we have
=> vccm energy equal to -13.6 eV
26. [B, CJ
32. [A, DJ
In fusion reaction, two or more lighter nuclei combine to form a
comparatively heavier nucleus. Let the transition in He· ion be from level a---+ b and that in H
atom be from p ---+ q . Then A1 = 1 2 gives
27. [A, C, DJ
From n=3 to r:,=2 4(b;- ~' )=(q;- p;)
.:!. oc
A
(1--1-)
nf n! ~
4 4
b2 - a2 = q2 - P2
he
S"1nce, E =~ This equation will be satisfied when
a=2p and b=2q
=> E oc (J__J_)
nf · n~
So, a and b are even integers greater than 1, satisfied by (A)
and (D).
For a photon of energy E , mOmentum p is
33. [C, D]
p = _§_ , where v is the speed of light.
V
22 n2
E=E 0 2 , r=a 0 - , V=V 0 -
z
n 2 n
3
A1 f- 22 4 where E0 is energy of electron in ground state, a0 is radius of
=> ,, =-1--1- =5 ground state orbit and v 0 is velocity of electron in ground state.
~-3 2 36
~=a=27 34. [A, B, DJ
=>
,, 5 2'
r=a 0 n2
he
a 2n4 log,(~)
Further 5.. =!:!. = '- =_§_ 2
=> A= nr
2
=·1ty
'E he 1
2
,, 27 1

=> An= A1n4


5 1
::;,· C=-=- => loge An = log Al+ logn4
27 a
=> log 8 An -loge A1 = 41090 n
So, b=E.t=5..=c=2.
P2 E2 27 => logen
log.(::)= 41og N 0

28. [B, DJ So, th~ graph will be a straight line passing through origin having
1
t.E=204=13.6 2'(!---)
a slope of four units at all the points.
2
1 4n
35. [B, C, DJ
=> 40.8 = 13.6 2'(-1,--\-) E _ -13.62' V
n 4n ,--n-,~ e
Sati~fied for, Z = 4 and n = 2
Since, E1 = -54.4 eV
29. [B, C, DJ
~
2
- 13.62 = -54.4
voc-
1
n {-.- V =( 2~:J¾} ~

Z 2 =4
(1)' .

Eoc __!_ => 2=2


n' Also, 40.8 eV is the difference between two energy levels
r cc n2 n=2 and n=1.
Also the electron cannot ·tall from the ground state and hence
=> ~ , Er ,
V
v 2r are independent of n .

cannot emit photon. So,

i=:===================================4.123=

www.puucho.com
Rahul Sardana with www.puucho.com

Advanced JEE Physics Optics & Modem Physics

-E=K and E=~ _1_ = 0.001 nm-' => 1.,,=10000A


2 1.,,
38. [A, C, DJ - 1- = 0.002 nm-' => 1.,, = sooo A
1.,,
Since, p = mv where v a: ..:!.
n - 1- = 0.004 nm-' => 1.,, =2sooA
1 1..,
:::::, poc-
n Violet colour has wavelength 40~0 A.
So violet colour can eject photoelectrons from Metal -1 and
Since, KE oc ! and r oc n 2

r Metal-2.
. 1
:::::, KEoc- 41. [8,CJ
n' According to Ritz Combination Principle, we have
Further L = nh E3 =E, +E 2
2,
:::> Locn => hv 3 =hv1 + hv2
=> v3 =V1 +v 2
39. [B, CJ C C C
Any transition in the Balmer series must end up at n = 2 . This
must be followed by the transition from n = 2 to n = 1 by
=> -=-+-
A.3 A, A2 I·- v=fl
emitting a photon of energy 10..2 eV . This 10.2 eV photon => ,-
A-~
corresponds to a wavelength of about 122 nm , which belongs A, + "-2
to the Lyman series. ·
42. · [A, BJ
40. [A, CJ For d =A, there will be only one, central maxima.
From the relation, For A < d < 2.l , there will be three maximas on the screen
· he·
eV=T-$ or corresponding to path differerice, AX= O and /!x = :U.

This is equation of straight line. 43. [B, DJ


In fusion two or more lighter nuclei combine to make a
Slope is tan 8 = he
e comparatively heavier nucleus.
In fission, a heavy nucleus breaks into two or more
=> $, :$,: $, =~:~ :~= _1_,-1- :-1-= 1:2:4 comparatively lighter nuclei.
101 A02 Aro Ao, A~ '-ro Further, energy will be released in a nuclear process if total
binding energy increases.

· 4.124t:::=====================================

www.puucho.com
Rahul Sardana with www.puucho.com

1. [BJ 10. [BJ


Cut-off wavelength depends on the accelerating voltage, not the At outermost orbit total energy is zero and electron is free from the .
characteristiC wavelengths. Further, approximately 2% kinetic influence of the nucleus of the atom.
energy of the electrons is utilised, in producing X-ray. Rest 98%
is lost in heat. 11. [CJ

2. [DJ Speed of electron in H like atom is vH u1<e = (~)~


According to classical electromagnetic theory, an accelerated 2he0 n
charge continuously emits radiation. As electrons revolving in
circular paths are constantly experiencing · centripetal 12. [CJ
acceleration, hence, they will be losing their energy continuoUsly 1
and the orbital radius will go on decreasing and form spirals and No. of lines in emission spectrum is N = n(n - )

4.
finally the electron will fall on the nucleus.

[DJ
N=
2
4(4-1)
=S
2

i oc R ex: µ , where µ is the reduced mass of the system.


and this ·depends on number of energy levels available for
transition.

=> µD >µH 13. [CJ


=> Ro>RH Lyman series - its energy is in the ultra violet region
Balmer series - its energy is in visible region
=> A. 0 <AH
In the centre of mass frame both the nucleus and the electron 17. [DJ
revolve about the common axis passing through the centre of After 200 days the number of un-decayed nuclei in the sample
mass.
will be ¾the initial number of un-decayed nuclei in the sample
5. [CJ
initially.
Since ').. = ~ , so the wavelength will be inversely proportional
aE - . 22. [DJ
t.o the energy difference between the levels. The energy Statement-1 is false, the· penetration power depends upon
difference is more when the transition takes place from n --+ oo accelerating potential.
to n =2 than when the transition takes place from n =2 to Statement-2 is true, increasing current increases the
n=1. temperature of filament causing it to emit more electrons.

7. [AJ 24. [AJ


Both Statement-1 & Statement-2 are correct & Statement-2 is h\ = 0-E, and
the correct explanation of Statement-1.
ht= 0-E,
8. [AJ => h(\-t)=E,-E,
Both Statement-1 and Statement-2 are true and Statement-2 is
the correct explanation of Statement-1. So, Statement-1 and Statement-2 both are Correct and
Statement-2 correctly explains Statement-1.
9. [BJ
Total energy is negative because electron is bound to the atom due
to coulomb attraction and in the bound system energy is
negative.

t:::=================================4.125=
www.puucho.com
Rahul Sardana with www.puucho.com

1. [CJ 3x10-15
Energy given tiy H-atoms ii, transition from n = 2 to n := 1 is V --"---'-"-
1.6x10-19
1.875x10' V
equal to ·energy taken by He... atom in transition from n = 2 to
n=4. 5. [CJ
li.E=3x10-15 -0.3x10-15 J
n =2 -----'9 · . -3.4 eV ~ .6.E=2.7x10-15 J
t IIE =10.2 eV
n = 1 - - - - - ' - - - - - - - - -13.6 eV
H-atom 6. [DJ

: ~:v
The difference of. the energy will be gained by the emitted
electron as kinetic energy, so
~:: _____I,_____ KE= 2.7x10-15 -3x10-17
KE=2.67x10-15 J
IIE = 10.2 eV =:,,
n=2-----'· · -13.6·eV
7. [A]
n = 1 - - - - - - - - - - -54.4 eV nl.
a=-
He' 2
2=2
c::::, c::::, 0 0 0
c::>C>X=a
2. [CJ
X=O
Visible light lies in the range, l., = 4000 A to l., = 7000 A. ------a------+<
Energy of photons corresponding to these waveleingths (in eV) l.=2a=_t,_=_h_ ... (1)
would be, .n p ./2Em
12375 12375
E =
' 4000
=3.09eV E =
' ' 7000
=1.77eV . ./E oc ]_
a
From energy level diagram of He+ atom we can see that in
Ea:__!_
transition from n =4 to n = 3 energy of photon released will lie a'
between E1 and E2 •
8. [BJ
IIE., = -3.4-(-6.04) =2.64 eV
From equation (1), we get
Wavelength of photon corresponding to this energy, n2h2
12375 E=--
l. = A= 4687.5 A Ba2m
2.64
In ground state n =1
). = 4.68 X 10·' ffi h'
~ E, = Bma2
3. [AJ
Substituting the values, we get
Kin~tic energy· K ~ 2 2 E,=8meV
=> K, -(1)'--
KHe• -2 -4
1 9. [DJ
From equation (1 ), we get
4. [AJ
Pa: n ~

eV=3x10-15 ~ mva:n

4.126 c:::=====================================
www.puucho.com
Rahul Sardana with www.puucho.com

Atomic & N11clear Physics

E =103E
" 2 ""
11. [AJ
Fror'n conservcition·of mechanical energy, we have
. U1 +K1 =U,+K,
Now, since E" =( m,,.
mPb +ma
)a
=> 0+2(1.5 kT)=-1-(e)(e)+O => E" =(2ii:4)a
4ru;0 d
Substituting the values, we get E 0 =103
105
(5.422) =53m MeV
T =1.4x10' K
15. [CJ
12. [BJ .:-~~-~
m-!l xC 2 xNm-2
As given in the paragraph, a reactcir is termed successful, if {N =kg ms"'}
nt0 >5x1014 scm-a KgxC 2

=R
13. (CJ
m(~H) +m( ;He)= 2.014102 + 4.002603 16. [BJ
=> m(~H)+m{;He)=6.016705 u A= 2nc
(0

Since, m{:u) = 6.015123 u


1"= ~~6oonm
=> m1 +m2 > M
1
So, (A) is incorrect. . vrne;
m{:H) + m{':;Bi) =1.007825 + 208.980388
m{:H)+m{':;Bi)=209.988213 u 17. [CJ
M~ximum energy of the antineutrino will be nearly 0.8 x 106 eV .
Since, m( 2!~Po)=209.982876 u
=> m1 +m2 >M 18. (DJ
So, B is incorrect Minimum kinetic energy of electron can be zero or greater than
zero. But maximum ki!'letic energy will be less than
m{ ~H) + m(;He) =2.014102 + 4.002603
0.8x10 6 eV.
=> m(~H)+m{;He)=6.016705 u
19. [A]
Since, :Li=6.015123u
=> (m,+m,)>M' Since, dN =P - ,N
dt
So, (C) is correct and hence deuteron and alpha particle can go dN log, 2
complete fusion. --=dt, where A.=--
P-,N T
+
m{;:zn) + m(:!Se) =69.925325 81 .916709
PT ( _.,,,,_,)
=> m{;:zn)+m(:!se)=151.842034 u N=--1-e T
loge2
Since, 1:Gd=151.919803 u
20. [BJ
=> m3 +m4 <M'
So, (D) is incorrect. Since, A=AN=P 1-e ( _.,,,,_,) T

14. [AJ Rate of energy release is AE 0 =PE0 1- e - T - ( '""-')


2
~Po.= 2:Pb +!He+ .dE
m(':\Pb)=205.974455 u 21. [AJ
Energy released upto time t is
m{;He)=4.002603 u Ereleased =(Pt-N)E0
=> m( 2~~Pb)+m(;He)=209.977058 u
Now, ilm =209.977058-209.982876 22. [BJ
=> ilm = 0.005818 u z, -(2)(2)+ (3)(1) = z, -(2)(1) + (5)(1) = z,
=> Q = IIE = 0.005818 x 931.5 2 1 -22 =4
~ O=5.419467MeV=5419.467keV
=> 0=5419.5 keV
A, -(4)(2) =A, -(1)(4)
.. =A,
By Law of Conservation of Momentum, we have
O=p" -P1ead
23. (CJ
=> Pa =·P1ead For A
=> ~2m 0 E0 =V2m,,.E,..
=> 4E.,_ = 206EPb

r:::::===================================4.127=
www.puucho.com
Rahul Sardana with www.puucho.com

Advanced JEE Physics Optics & Modern Physics


30. [Cl.
From above calculations we observe that less energy is required
For8 for a nuclear reaction, which a light particle is incident on a
heavy target than if a heavy particle is incident on a light target.
N ~ N o ~ No
0
2 4 31. [CJ
After 4 minute, we have
F=-dU
NA=Na=~o dr

=> F= 4k
9 0 r'
=> N0 =(4N0 +N0 )-(~+~')= ~
mv2 4k
=> -,-=7
24. [8]
Given AA =R8 => mv
2
=74k ... (1)
=> A.ANA = AaNa Now, according to Bohr's Quantisation rule, we have
=> (lot 2)(4N,e-•••) =N,Cot 2)•-"" . nh
V=~- ... (2)
2nmr
=> t0 =6 min ( n'h' ) 4k
=> m 41t2m2r2 =7
25. [C] . n2h2 4k
p =(1 o/o) X 4Q X 103 X 1 Q x1 Q-3 =>
4x"'m =7
=> P=4 W ., 16x2mk
=> r =~
26. [BJ
Total power is P= Vl=40x10 3 x10x10--3 => r = 4n ,.iii,iZ ... (3)
nh
=> P101a1 =400W
So, heat produced per second is 32. [DJ
ptotal - pemitted =400 -4 =396 W Substituting (3) i~ (2), we get
nh
V
27. [A]
12400 _ A 2nm(~~ ,.lii,iZ)
03
40x1000 n"'h 2
=> V-
Bn'm,./ii,iZ
28. [A]
If x+X----.+Y+y 33. [8]
For the above nuclear reaction, threshold energy is given as
KE1 1(n'h')
2

E,.=-0(1+::) .
=2mv = 2m 64n4m3k
n4h4
The Q~value of reaction is given by => KE-
128n4 m2k
Q = (1.007825 + 3.016049-2x 2.014102) X 931.5
=> Q = -4.033 MeV 34. [D]
When protons are incident on ~H , then PE=U=-~
r'
X = iH and X = ~H
kn 4h4
1 007825 =>
K =4.033 Mev(1+ · )
• 3.016049
=> K. = 5.381 MeV
=>
29. [CJ
When ~H is incident on protons
35. [CJ
x=~H and X=!H TE=KE+PE

=> K =(4.033 Mev)(1+ 3.016049) =>


• 1.007825
=> K• =16.10 MeV
=>

=4.128 c::===================================
www.puucho.com
Rahul Sardana with www.puucho.com

Atomic & Nuclear Physics


37. [CJ 50. [AJ
B.E. per nucleon for intermediate nucleus is more than lighter or
heavier nuclei. Since .!. = RZ' (J...
' ')..,
-J...)
22 32
38. [DJ => h·c =(Rhc)z'(.!.-.!.)
1c 4 9
where Rhc = 13.6 eV

Since, rNu =r0 A3' Also, given that he= 47.2 eV


1c
=> VNu = (i1trg )A =>
5
47.2=13.62'(3 6)
=> VNu oc A => 2 2 ~2s
=> 2=5
39. [BJ
Q = (N,E, +N,E,)-N,E, 51. [BJ
n'
42. [DJ Since, '" = (0.53)
2 A
Sfnce, 1 kW= 3.1 x1013 fission , = o.53 A
sec ' 5
=> 1.6x103 kW=a1.6x10 3 x3.1x1013 ,, =0.106 A
= 5 x 1016 fission per second
52. [BJ
44. [BJ
s·rnce, E2 = E,
The reaction for the second stage is given by 4
1Ta +p2-l-2He4 +on, +E2 => E,=4E,=4(-144)=-576 eV
=> Lim= (3.016049 + 2.014102) -(4.002603 + 1.008665) So, ionization energy = 576 eV
=> .ti.m=o.0188Bu
=> E, =0.01888x931=17.587 MeV 53. [DJ
E1 =-576 eV
45. [OJ
Total energy released E=E1 +E2 =4.033+17.587 54. [AJ
=> E = 21.62 MeV s·mce, E
n
E,22
= -7
2 62
So, energy released/ deuteron is \ = 7.207 MeV
So, graph of En vs ~ is a straight line passing through origin
n
46. [CJ with negative slope. The distance between successive points is
( % of Rest Mass Of) 7.207x 100 non-uniform.
0.384%
deuterium released 2.014102x931.4
55. [BJ

47. [AJ
mv'
-==evB •.. (1)
r
,: _ .!_ d ,: _ 0.693
M-t,.an H- 'A, V eB
... (2)
=> TM> TH m
According to Bohr Quantisation rule,
48.
nh
[CJ mvr=-
2rr
A= dN = 1.N nh
di => vr=~- ... (3)
2rrm
From (2) & (3), we get

T = 0.693 r=~ ... (4)


n
56. [CJ
49. [CJ
R1 = R0 e-J.1 and R2 = R0 e-1:?1 Kinetic energy, K == imv2

~=e"(1,-t1)
R, K=.!.m(n'h')
2 2
2 mr
R = R,e-i.(i,-1,)
2
K=inh(:) .•. (5)

:::::=================================== = 4.129

www.puucho.com
Rahul Sardana with www.puucho.com

Advanced JEE Physics Optics & Modern Physics

57. [AJ => N, =.2.[u-(u-1-N,)e·"] ... (1)


S .1nce, T =2.r
-
v
2
At t=Tv 2
'loge2 we have
=-,-.
::::,
. e
1·=-=--
eB
T 2nm
N, =±(u-(u-,N,)e'(!¥))
Since, Area= nr
2
= n:(;:)
=> N, =.2.[ u-(u-1-N,)e·"'-']
/-
=> M = iA = ( e'B )(nnh) = neh
2nm Be 2m Sine~, e109•x = X
Since, PE=U=MBsin(90°)
e-lo9o2 = elog.m ::: .!.
U = nhBe 2
2m
N,=±[u-(u-;N,)]
58. [BJ
~ = 1tr 2B N =a.+ANo
' 21-
~ = n(~)B = nnh
Be e 66. [BJ
Further, due to the decay of X to the stable nucleus Y, we
59. [CJ have
No= mNA dN, =1-N
M dt X

Substituting value of N. from equation (1) and solving, we get ·


60. [CJ
The number of undecayed nuclei after time t is N = N0 e-1,,
1 u-1-N,) _,, (u-1-N,)
Nv=a t + ( - ... (2)
==> -,- e - --,-
So, the number of decayed nuclei is
N' = N, -N = N, (1-e·") 67. [CJ
2
Substituting the value of Tv, = log~( ) in equation (2), we get

N =~log (2)+(u-1-N,)-(u-1-N,)
YA,e 2 )., /,,.

u
Nv =iloge 2 -z
( ) 1 (u-1-N,)
--A-_
62. [CJ
AE=13.6 z'(_!__ _ _!__) 68. [BJ
n~ n! E2 -E 4 = -2.5 eV < 2.7 eV
So, the electron will be making a transition to n = 4.
=> 47.2=13.6 z'(¾-¾)
==> n=2
=> Z=5 4 (-0.85 eV)
t
63. [CJ
''
' '' 3 (-1.5 eV)
Total energy TE= 13.6(5)' = 340 eV
Since, TE = KE '''
=> KE=340V '' 2 (-3.4 eV)

64. [AJ
he 13.6(Z)' n = 1 (-13.6 eV)
1" n'
he n2 69. [AJ
1- - (13.6 eV)Z' E =-13.6z,
' n'
8
A 6.63x10-34 x3x10 x9
13.6 X 25 X 1,6 X 10-19
:::> E0 =E2 =-3.4 2 2
1-=329A and E0 =E4 = -0.85 2 2
Now, E0 -E 8 =2.7eV
65. [DJ => Z=1
dNx = a.-AN The ionisation energy is IE= 13.6 eV
dt X

=4.130====================================
www.puucho.com
Rahul Sardana with www.puucho.com

Atomic & Nuclear Physics


70. [AJ he
Emax =E 4 -E1 =12.75 eV => -0.85+3.4=2.55 eV=-
-l.
and Emio = E4 -E3 = 0.66 eV => l.=
12400
=4862A=486.2nm
2.55
72. [CJ
73. [DJ
E -E = he
' ' l. t.E =he= 12400 = 12 _09
13.6x(1)' (-13.6)x(1)' he l. 1025
16 4

========================================= = 4.131

www.puucho.com
Rahul Sardana with www.puucho.com

1. A-->(p,q)
B--> (p, r)
C--> (p, s) For t,wehave "1=2 and n2 ---+co
D --> (p, q, r)
k
~=-
2. A--> (p, r) 4
B--> (p, q, s) \-t =~
C--> (p)
D--> (q)
6. A--> (p)
B--> (q)
3. A--> (r)
C--> (r)
B--> (p)
D--> (q)
C--> (q)
y---+ e- +e+ ~
D--> (q)
E--> (s) For pair production, we have
E = 2m8 c 2 = 2 x 0.51 MeV = 1.02 MeV
4. A--> (p, s) Inverse photoelectric effect is X-ray production and energy
B--> (q, s) involved in it is of order of tens of KeV
C--> (q, s)
For de-excitation of Be+4 from first excited state, we have
D--> (s)
z2 42
EocJ_ E =-x 13.6 =,x13.6
n' n' Z
1 => E • 54.4 eV = 55 eV
:::) p oc - For K X-ray photon of molybdenum, we have
n 0

:::> ro::n2 E(K0 )=hv= hx3 cR (Z-1)'


E 1 1 4
::::::, -cc- and Epra:-
p n n E(K ) =_:l_(hcR)(Z-1)' =_:l_E, (Z-1)'
0
4 4
.!.. cc n3 and Er = constant
p => E(K0 )=¾x13.6x(42-1)' =17.146x10' eV

5. A--> (p) => E(K 0


) •17 KeV
s:...(p)
C--> (q) . 7. A--> (p, r)
D--> (s) B--> (q, r, s)
f oc (J.._J..)
n~ n!
C--> (q, r)
D--> (p, r) /
For a- decay, we have
::::::, f = k(~---i), where k is a constant ~x~A~~Y + !He
0
1 °2 ,
For ~.we have n1 =1 and n2 ...+oo Q =(KE)v + (KE)"
Since, (KElv « (KE)0 , we have
=> \ =k
O=(KElo
For ~.we have n,=1 and n2 =2

= c::========================================::!
4.132

www.puucho.com
Rahul Sardana with www.puucho.com

Atomic & Nuclear Physics


So, kinetic energy of all the emitted a- particles is equal to (a) angular momentum is conserved.
Q i.e., mono-energetic a- particles are emitted. (b) it cannot take place outside nucleus i.e., in free
Angular momentum is conserved in a- decay space.
For ~- Decay, we have
8. A-; (r)
a neutron decaying to a proton, so
:x------t z:
1Y + V + e·
8--> (p)
c-. (q)
Q ~(KE),+ (KE), + E, D--> (q)
Since, (KE), « (KE),
9. A--> (p, s)
=> a~(KE),+E, 8-; (p)
Also note that, C--> (q, s)
(a) E-.. is the energy of anti-neutrino and E,, takes on D-; (p)
values from zero to maximum. Hence poly-energetic 10. A-; (p, q, s, t)
particles are emitted. i.e., poly-energetic antineutrinos
8--> (r, s, t)
are emitted.
C--> (q, r, s, t)
(b) Due to emission of antineutrinos spin angular
momentum is conserved. D--> (r, t)
(c) Neutron can decay in free space i.e., outside nucleus
For Positron emission, we have 11. A--> (q, r, s, t)
a proton decaying to a neutron, so 8--> (q, r, s, t)
C--> (q, r, s, t)
~X------t l 1Y + v + e+
D--> (p, q, r, s, t)
Same explanation as above in case of Beta decay can be
applied here too. 12. A--> (q)
Proton cannot decay in free space i.e., outside nucleus 8--> (p)
because rest mass of proton is less than that of neutron.
C--> (s)
For Electron capture, we have
D--> (r)
~X+e- --),l1Y +v In alpha decay, charge number decreases by 2 and mass
Q =(KE), +E, number decreases by 4.
In ~+ - decay, charge number decreases by 1 and mass number
Since, (KE), « E,
remains same.
=> a~E. In proton emission, charge no, decreases by 1 and mass no,
All the neutrinos emitted are of equal energies and their decreases by 1.
energies are approximately equal to 0. That is mono-
energetic neutrinos are emitted. So, for electron capture
we have that

c:===================================4.133=
www.puucho.com
Rahul Sardana with www.puucho.com

1. From our knowledg-e of Collision Theory, the fraction of kinetic E -E = 12375 + 12375
energy lost by neutron is 3.
" ' 1085 304
AK 4m,m, 4(1)(2) 8
K,-(m,+m,)' (1+2)' 9 => (13.6)(2'>(1- n~)=52.1eV
where K1 is the initial kinetic energy of neutron and .6.K is the 1 .
=> 1-2=0.96 {·.- 2=2}
energy loss. n
=> nz5
After first collision 6K1 =¾K 0

After second collision AK 2 =¾K, and so on 4. Since .! = R(2-1l'(..!._1-)


' A - i2 2
· 2

So, total energy loss is


. • 8 => _ ~ _,, (1.09x10')(2-1)'(¾)
0 76 10
AK= .6.K1 +AK2 + ... +AKn =9(K0 +K1 + .... +Kn-1) => 2-1~40
=> Zz41
where, K1 =K0 - AK1 = ~ 0

5. At time t, let say there are N atoms of 7 Be (radioactive). Then


K, =~, =(¾)' K, net rate of formation of 7 Be nuclei at this instant is,
dN 10"'
l.N
di 1.6x10"" x1000
dN =6.25x10" -l.N
di
"I dN
11
""'I dt
1 !e.25x10 -AN ·O
AK=-K 9 =1--
8 ( - -~) 1
9 o 1-l gn where N0 are the number of nuclei at t = 1 hr or 3600 second.
9
Since, K, =10' eV and AK=(10'-0.025) eV
=> _ _!_ 10 (6.25x10 -AN0
A _ge 6.25x1011
11
)= 3600
7
1 K, - AK 0.025 Since, AN 0 =Activity of Be at t=1 hr=1.8x108
=> 9° =-K--=105
disintegrations/second
=> 9"=4x107
' => _ _! 10 (6.25x10"-1.8x10')= 3600
Taking log both sides, we get A ge 6.25x1011
n=8 => A =8 x 10~ sec-1
Therefore, half life
2. After removing the first electron it will become He· ion. The 0.693
tV2 = --_,- = 8.66 x 106 sec = 100.26 days
ionization energy of single electron in He .. ion (Z = 2) is 8x10
IE=13.6(2')=54.4 eV => t,,, ~ 100 days
Therefore, total energy required to remove both the electrons is
given by
E=(24.6+54.4) eV=79 eV

=4.134 ==================================
www.puucho.com
Rahul Sardana with www.puucho.com

Atomic & Nuclear Physics


he
6. Since. A=R,(¾J 10. ~-$=BV0

where, n is the number of half lives. ne


V ---
o - 4nl:or
Given, R=~~
=>. 1240 eV-4.7 ev=( xne) eV
A,= A .(.!.)' 200 4m:or
16 ' 2 9
__ _ 9x19 Xnx~.6x.10-19
=:> n=4 62 47
10
Four half lives are equivalent to 8 s . Hence, 2 s is equal to
one half life. So in one half life activity will remain half of 1.5x10-2 ~ 100
n
1600 Bq i.e., 800 Bq. 9x1.6x10-10

7. At time t=t 11. dN =-l.N =-10"


dt
N, = N,e-"
and N2 = NoA1 (e-Ai' -e-¼') => N=10'°xG:)
-A1 A: =:> m1o1a1=Nxm 1
Ns=No-N1.-N2
m101a1 =1010 x10 9 x10--25 x10 6 mg
N =N [1-e-" __l._,
3 o A2-A1
-<e-'' -e-"'l] m10ta1 =1 mg

=> Na = 1- e-1,1 - _l._,-(e->.,' - e-i..2') 120exe


No "-2-~1 12.
4n:£0 (10x 10-15 )
Since, A.1 =-
0
·:i = 3
0.0231 min-1
A2 = .!!..x 4nso X 10-14
2m 120e2
l. 0.693 0 0154 mrn
=4S=. . _,
2
"-=!:x 41t,i:;0 x10-""
and t = 60 minutes e 2x120xm
N3 =1 _ 8 --0.02a1>60 0.0231 (e-o.ll231>ro _ 8 -0.0154•60) A= 4.2x 10-1s x /r---1-0~-,~,-x-3___
N, 0.0154-0.0231 Ysx109 x2x12oxsx10- 21
A=7x10-15 m
N, =1-0.25+3(0.25-0.4)=0.31
N, . l.=7fm
N
%age =..2.x100% =31% 13. Let m 1 and m2 be the mass of a-particle and hydrogen atom.
N,
By Law of Conservation of Momentum, we get
m1u, = (m1 + m2 )v
8. V=~-.P,
e e where u1 is the initial vetocity of the incident a-particle and v
h is the final common velocity (or velocity of centre of mass),of the
Slope=-
e particles. ·
It is same for both, so By Law of Conservation of Energy, we get
Ratio =1 1 , 1( ) , E
2m1v, =2 m1 +m2 v +.6. o

9. where .6.E0 is the Ionization Energy


The loss in KE of the a -particles must be gained by the atom as
=> ~=e-i..1
N, ionisatio_n energy, so we h'ave·

=>
N -~•so
-=e 1386
.!.(
2 m +m1
1
m m2 )v~ =.6.Eo
2
N,
N 0.693~80 => 1
K1 =-m1V1, = (m
--
1 +m2) E
- t,. o
=> -=e~ 2 m,
N,
!!.=e--0.04 => K 1 =(1+ ::).6.E0
=>
N,
=> K, =(t+f)<13.6) ev
=> .!'!.
N,
= ( .!.)""
e => K 1 =68eV

Fraction of nuclei decayed


·.. -= N= (1)'·" =
1- - 1- - 0.04 =4%
N, e

==================================4.135=

www.puucho.com
Rahul Sardana with www.puucho.com

Advanced JEE Physics Optics & Modern Physics

14. U=-1.7eV 4.6x106


n
2
13 5 0.23
E=~=-0.85 eV = - -
2 n' Taking log both sides, we get
::::> n=4 n~24
Ejected photcielectron will have minimum de-Broglie wavelength
corresponding to transition from n =4 to n = 1 , so we have 17. Given, l., =4102 A, l., =4861 A
IIE = E, -E, =-0.85-(-13.6) = 12.75 eV For Balmer Series, we have
Using Einstein's Photo-Electric Equation, we get => _!_ = R(_!_ - _!_)
Kmlll( = .6.E-W =10.45 eV A,1 22 n:
=> ,. = J10.45
150 A (for an electron}
Substituting values of A.1 and R , we get
n1 =6
=> l.=3.8A~4A Similarly, for Balmer Series, we have
..!._ = R(_!_ - _!_)
15. (a) Since N=n(n- 1)=6 A.2 22 n:
' 2 Substituting values of A. 2 and R, we get
=> n=4
n2 =4
i.e., if n1 =n, then n2 =n-4
Now difference of wave numbers of above two lines is
(13.6)2'
=> 0.544 ... (1) _!_ __!_=_!=A
n' A.1 A.2 A,

=>
(13.6)2'
(n-4)'
0.85 ... (2) => 1 _ 1 _ 1 -R( 1
"-1"-2"- n:n:
J)
Solving equations (1} and (2). we get Transition n2 ~ n1 or 6 ~ 4 corresponds to second line of
2=4 and n=20
Brackett series, whose wavelength is given by
(b) l. _ 12375 40441 A
m•• - -0.544-(0.85) _!=R(..!.._..!..)
2 2
A 4 6
Substituting the values, we get
16. Mass of neutron ~ mass of proton = m
K, l.=26206 A
y

Lx
18. From the given conditions
m m E, -E, = (10.2+17) eV = 27.2 eV ... (1)
Q1---, K, 0 and E, -E, = (4.25 + 5.95) eV = 10.2 eV ... (2)
Neutron Proton
Equation (1) - (2) gives
E3 -E2 =17eV
K,
=> 2 2 (13.6)(¾-¾)=17
Applying Law of Conservation of Linear Momentum
along y-direclion, we get
=> 2'(13.6i(; )=17
J2mK 1 sin45° =~2mK2 sine ... (1) 6
along x-direction, we get ~ 2 2 =9
~2mK0 -~2mK, cos45° = ~2mK, case ..,(2) ~ 2=3
Squaring and adding equations (1) and (2), we get From equation (1), we get
K 2 = K1 + K 0 - ~2K0K1 ... (3) 2'(13.6i(¾- ~' )=21.2
By Law of Conservation of Energy, we get
K2=Ko-K1 ... (4) => (3)'(13.6{¾- n1, )=27.2
Solving equations (3) and (4), we get
K - K, => _! - _!_ = 0.222
,- 2 4 n'
1
=> ,=0.0278
n
i.e., after each collision energy remains half. Therefore, after n => n2 =36
collisions, we have => n=6

K, =K,GJ 19. (a) 1 rydberg =2.2x10-" J=Rhc


Ionisation energy is given as 4 rydberg, so
=> 0.23=(4.6x10'i(¾J IE=8.8x10-18 J S.Sx 1o-,s 55 eV
. 1.sx10-19

=4.136 ==================================
www.puucho.com
Rahul Sardana with www.puucho.com

Atomic & Nuclear Physics


::::;i, Energy in first orbit is 21. Total number of electrons striking the target per second
E, =-55 eV n 10x10--3 6.25x101s
Energy of radiation emitted when electron jumps from first 1.6x10-19
excited state (n = 2) to ground state (n = 1) is Kinetic energy of an electron is
K =40x10 eV =40x103 x1 .6x1Q-19 =6.4x1Q-15 J
3
E 3
E,, = ( ), -E, =- E, =41.25 eV
2 4 Total energy of electrons striking the target (per second) is
~o, wavelength of photon emitted in this transition is E = 6.25x1016 x6.4x10-15 =400 J
l.= 12375 = 3 ooA (a) Total power emitted as X-rays is 1% of 400 W. So
41.25 P=4W
(b) Let Z be tt~e atomic number of given element. Then (b) Heat produced per second is
E, =(-13.6)(2') H=(400-4) Js·'=396Js·'
=> -55=(-13.6)(2') 22. When an substance decays by a and J3 emission
=> 2=2
simultaneously, the average disintegration constant A.av is given
(c) Since, r = ~ , so the radius of first orbit of this atom is by
A.av =A.a +All
where A" = disintegration constant for a -emission only
A.11 = disintegration constant for p -emission only

(13.6)z' V Mean life is given by T = _l_


20. E, m ).~
n' e
Substituting z =3 , we get
1 1 1 1 1
E =- 122.4 eV -=-+-=--+-
' n' Tm T. T, 1200 600
122 4 => Tm =400 yr
=> E, =- ; =-122.4 eV
(1)
122.4
and E3 = - -2- =-13.6 eV
Since, t = .:!.109,
A
(N,)
NI
= Tm log, (N,)
N,
(3)
=> llE = E, -E, = 108.8 eV => t=4001og,(~i)=400log,(4)
The-corresponding wavelength is => t=400x1.4= 560year
l.= 12375 A=12375A= 11374 A
tiE(in eV) 108.8 ·
=> :\.=114A

::=:===================================4.137=

www.puucho.com
/

You might also like